You are on page 1of 1143

Virginia Commonwealth

University
School of Medicine
2008-2009 Academic Year

M-1 Physiology
Spring 2009
Web Syllabus: This paper syllabus is printed directly from the eCurriculum, which contains links to other
electronic resources. Please refer to the course website for the most up to date information, including the
course schedule, which is subject to change.

“Section 504 of the Rehabilitation Act of 1973 and the Americans with Disabilities Act of 1990 require
Virginia Commonwealth University to provide academic adjustments or accommodations for students with
documented disabilities. Students seeking academic adjustments or accommodations must self-identify
with the Coordinator of Services for Students with Disabilities on the appropriate campus. After meeting
with the Coordinator, students are encouraged to meet with their instructors to discuss their needs, and if
applicable, any lab safety concerns related to their disabilities.”
Physiology
Table of Contents:
Block Schedule................................................................ 1
Physiology Overview........................................................... 17
Compartments Cell Membranes - Dr. Costanzo.................................... 20
Chemical Signaling, I-V - Dr. Kalimi.......................................... 33
Membrane Transport I - Dr. Costanzo........................................... 61
Membrane Transport 2 - Dr. Costanzo........................................... 74
Membrane Potentials - Dr. Logothetis.......................................... 82
Excitable Cells 1 - Dr. Logothetis............................................ 98
Excitable Cells 2 - Dr. Logothetis............................................ 114
Neuromuscular Transmission - Dr. Logothetis................................... 131
Skeletal Muscle Mechanics - Dr. Feher......................................... 150
Contractile Mechanisms in Skeletal Muscle - Dr. Feher......................... 172
Autonomic Nervous System 1 - Dr. Biber........................................ 191
Excitation-Contraction Coupling - Dr. Feher................................... 206
Autonomic Nervous System 2 & 3 - Dr. Biber.................................... 222
Muscle Energetics and Fatigue - Dr. Feher..................................... 244
Smooth Muscle - Dr. Karnam.................................................... 275
Autonomic Nervous System 4 - Dr. Biber........................................ 295
Cell Physiology Problem Solving - Dr. Costanzo................................ 314
Review Session - Dr. Feher.................................................... 318
Introduction to Cardiovascular System - Dr. Baumgarten........................ 325
Contraction of Cardiac Muscle - Dr. Baumgarten................................ 332
Cardiac Function Curve - Dr. Baumgarten....................................... 343
Hemodynamics 1 & 2 - Dr. Pittman.............................................. 352
Cardiac Oxygen Consumption - Dr. Baumgarten................................... 372
Control of Cardiac Output 1 - Dr. Ford........................................ 378
Control of Cardiac Output 2 - Dr. Ford........................................ 387
The Arterial and Venous Systems - Dr. Pittman................................. 396
Electrical Activity of the Heart - Dr. Baumgarten............................. 407
Microcirculation - Dr. Pittman................................................ 423
Peripheral Circulation and its Control - Dr. Pittman.......................... 433
Autonomic Reg of Card Function - Dr. Baumgarten............................... 447
Special Circulations 1 - Dr. Pittman.......................................... 455
Cardiovascular Physiology Problem Solving 1 - Dr. Baumgarten.................. 465
Special Circulations 2 - Dr. Pittman.......................................... 467
Regulation of Arterial Blood Pressure 1 - Dr. Ford............................ 476
Electrocardiogram 1 and 2 - Dr. Baumgarten.................................... 490
Regulation of Arterial Blood Pressure 2 - Dr. Ford............................ 504
Exercise and Hemorrhage - Dr. Ford........................................... 514
Cardiovascular Physiology Problem Solving 2 - Dr. Pittman..................... 528
Cardiac Cycle - Dr. Baumgarten................................................ 529
Body Temperature Regulation - Dr. Ford........................................ 539
Lab Group Assignments......................................................... 555
Cardiovascular Physiology Lab - Dr. Baumgarten................................ 561
Cardiovascular Practice Exam & Review Questions - Dr. Baumgarten............. 576
Body Fluids 1 and 2 - Dr. Costanzo............................................ 586
Introduction to Gastrointestinal Physiology & GI Hormones - Dr. Grider........ 601
Regulation of Body Fluids: Na+ & Water - Dr. Costanzo......................... 621
Gastrointestinal Motility 1 - Dr. Grider...................................... 632
Clearance, RBF and GFR - Dr. Costanzo......................................... 650
Reabsorption and Secretion 1 and 2 - Dr. Costanzo............................. 667
Gastrointestinal Motility 2 - Dr. Grider...................................... 675
GI Secretion 1 - Dr. Grider................................................... 685
GI Secretion 2 - Dr. Grider................................................... 701
Na+ Transport 1 and 2 - Dr. Costanzo.......................................... 717
Physiology
Table of Contents:
Digestion and Absorption 1 - Dr. Grider....................................... 731
Digestion and Absorption 2 - Dr. Grider....................................... 743
K+ Transport - Dr. Costanzo................................................... 755
Concentration & Dilution of Urine - Dr. Costanzo.............................. 761
GI Clinical Correlation: PUD - Dr. Grider..................................... 780
Renal Problem Solving - Dr. Costanzo.......................................... 788
GI Tract Review - Dr. Grider.................................................. 793
Lung Volumes and Capacities - Dr. Costanzo.................................... 797
Mechanics of Breathing 1 and 2 - Dr. Costanzo................................. 805
Physical Chemistry of Gases: Gas Exchange - Dr. Costanzo...................... 824
O2 Transport - Dr. Costanzo................................................... 834
CO2 Transport - Dr. Costanzo.................................................. 844
Lab Group Assignments......................................................... 847
Spriometry Lab - Dr. Costanzo................................................. 853
Pulmonary Circulation - Dr. Costanzo......................................... 866
Acid-Base 1, 2, and 3 - Dr. Costanzo.......................................... 875
Ventilation/Perfusion (V/Q) 1 & 2 - Dr. Costanzo.............................. 896
Renal Acid-Base 1 and 2 - Dr. Costanzo........................................ 918
Respiratory Rhythm - Dr. Costanzo............................................. 930
Ventilation in Exercise - Dr. Costanzo........................................ 935
Acidosis Alkalosis - Dr. Costanzo............................................. 939
Respiratory Pathophysiology Cases - Dr. Costanzo.............................. 964
Acid Base Problem Solving - Dr. Costanzo...................................... 970
Endocrinology - Dr. Kalimi.................................................... 973
Thyroid Hormones 1, 2, & 3 - Dr. Kalimi....................................... 987
The Posterior Pituitary Gland - Dr. Witorsch................................. 000
Adrenal Cortex 1, 2, & 3 - Dr. Witorsch....................................... 011
Growth Hormone 1 & 2 - Dr. Kalimi............................................. 051
Physiology of PTH and Vitamin D - Dr. Costanzo................................ 058
Male Reproduction 1, 2, & 3 - Dr. Witorsch.................................... 077
Insulin - Dr. Kalimi.......................................................... 095
Female Reproduction 1 and 2 - Dr. Witorsch.................................... 107
Growth Factors - Dr. Kalimi................................................... 122
Female Reproduction 3 - Dr. Witorsch.......................................... 129

Please check the eCurriculum for other session resources (PowerPoint & Audio
files).

The lecture recordings, syllabi, powerpoints, and other educational materials


posted in the eCurriculum system are for the purposes of personal study. A
student shall acquire no intellectual property rights in the recordings,
syllabi, and powerpoints or their contents whatsoever, and is absolutely
prohibited from posting these materials to an external website, broadcasting
or supplying these materials to any other person for commercial gain or
otherwise. The content of lectures, lecture syllabi, powerpoints, and other
related material may be the intellectual property of the University or
individual lecturers/tutors, depending in each case on circumstances
and interpretation of law. It could be considered a violation of the VCU Honor
Code and the Intellectual Property Policy for a student to use these materials
except for the express purpose stated above. Any such violation would be
punishable as stipulated by the VCU Honor Code, and could include dismissal
from VCU.
Print Blockschedule https://ecurriculum.som.vcu.edu/portal/student/blockschedule_p.aspx?cla...

Block Schedule

M1 Blockschedule
1/5/2009 - 1/9/2009
Mon 1/5/2009 Tue 1/6/2009 Wed 1/7/2009 Thu 1/8/2009 Fri 1/9/2009
8 AM PHYS: Compartments
and cell membranes
PHYS: Membrane
Transport 1
PHYS: Membrane
transport 2
PHYS: Membrane
potentials: Ion Movement
PHYS: Membrane
potentials in excitable
Cells 1
Costanzo, L. Costanzo, L. Costanzo, L. Logothetis
Logothetis

9 AM PHYS: Chemical
signaling 1
PHYS: Chemical
signaling 2
PHYS: Chemical
signaling 3
PHYS: Chemical
signaling 4
PHYS: Chemical
Signaling 5

Kalimi Kalimi Kalimi Kalimi Kalimi

10 AM HIST: Microscopy
Concepts and Cell
HIST: Cell HIST: Cell HIST: Cell HIST: Epithelium:
Membranes
Bigbee Bigbee Bigbee
Bigbee Merchant

11 AM ANAT: Introduction to
Embryology
ANAT: Fertilization &
Implantation
ANAT: Gastrulation &
Nuerulation
ANAT: Development of
body cavities
ANAT: GI development

Sholley
Colello Colello Colello Sholley

12 PM

1 PM HIST: Intro to Digital


Histology CD
FCM : SG Session 9 FCM : SG Session 9 FCM : Fall Skills Exam
Remediation

PHYS: Intro to
Physiology

2 PM

3 PM

4 PM

5 PM

Blockschedule printed from the web on 11/20/2008. Dates are subject to change. Please refer to the eCurriculum website at https://ecurriculum.som.vcu.edu/portal
for the most recent schedule.

1
1 of 1 11/20/2008 9:32 AM
Print Blockschedule https://ecurriculum.som.vcu.edu/portal/student/blockschedule_p.aspx?cla...

Block Schedule

M1 Blockschedule
1/12/2009 - 1/16/2009
Mon 1/12/2009 Tue 1/13/2009 Wed 1/14/2009 Thu 1/15/2009 Fri 1/16/2009
8 AM PHYS: Membrane
potentials in excitable
PHYS: Neuromuscular
transmission
PHYS: Contractile
Mechanisms
PHYS: Excitation-
Contraction coupling
PHYS: Skeletal Muscle
Energetics
cells 2
Logothetis Feher Feher Feher
Logothetis

9 AM HIST: Connective
Tissues and Blood
PHYS: Skeletal Muscle
Mechanics
PHYS: Autonomic
nervous system 1
PHYS: Autonomic
nervous system 2
PHYS: Autonomic
nervous system 3

Haar Feher Biber Biber Biber

10 AM HIST: Muscle Tissue HIST:


Nervous
HIST:
Muscle
HIST: Nervous Tissue PHYS: Smooth muscle 1

Bigbee Tissue Tissue Bigbee Karnam

Bigbee Bigbee

11 AM HIST: Epithelium: Glands ANAT: Development of


Heart
ANAT: Development of ANAT: Urogenital System
the Vascular Supply Development
ANAT: Limb
development
Merchant
Colello Colello

12 PM Project HEART

1 PM Electives FCM : SG Session 9 FCM : SG Session 9

2 PM

3 PM

4 PM

5 PM

Blockschedule printed from the web on 11/20/2008. Dates are subject to change. Please refer to the eCurriculum website at https://ecurriculum.som.vcu.edu/portal
for the most recent schedule.

2
1 of 1 11/20/2008 9:47 AM
Print Blockschedule https://ecurriculum.som.vcu.edu/portal/student/blockschedule_p.aspx?cla...

Block Schedule

M1 Blockschedule
1/19/2009 - 1/23/2009
Mon 1/19/2009 Tue 1/20/2009 Wed 1/21/2009 Thu 1/22/2009 Fri 1/23/2009
8 AM Martin Luther King Day PHYS: Smooth muscle 2 PHYS: Problem Solving ANAT: Embryology Exam HIST: Histology Practical
1 Exam
Karnam Costanzo, L. Location: CBIL

9 AM PHYS: Autonomic
nervous system 4

Biber ANAT: Period of Fetal


Development

10 AM ANAT: Development of Colello PHYS: Physiology


the Pharynx Review

ANAT: Embryology Biber


Review Costanzo, L.
Feher

11 AM ANAT: Face and Skull Colello Kalimi


Development Karnam
Logothetis
Location: MSB Auditorium

12 PM

1 PM Electives FCM : SG Session 10 FCM : SG Session 10 PHYS: PHYSIOLOGY 1,


HISTOLOGY 1 EXAMS

2 PM

3 PM

4 PM

5 PM

Blockschedule printed from the web on 11/20/2008. Dates are subject to change. Please refer to the eCurriculum website at https://ecurriculum.som.vcu.edu/portal
for the most recent schedule.

3
1 of 1 11/20/2008 9:48 AM
Print Blockschedule https://ecurriculum.som.vcu.edu/portal/student/blockschedule_p.aspx?cla...

Block Schedule

M1 Blockschedule
1/26/2009 - 1/30/2009
Mon 1/26/2009 Tue 1/27/2009 Wed 1/28/2009 Thu 1/29/2009 Fri 1/30/2009
8 AM PHYS: CV system
overview
PHYS: Cardiac ejection PHYS: Hemodynamics 1 PHYS: Hemodynamics 2 PHYS: Arterial system

Baumgarten Pittman Pittman Pittman


Baumgarten

9 AM PHYS: Cardiac muscle


contraction
PHYS: Cardiac function
curve
PHYS: Cardiac oxygen PHYS: Control of cardiac
consumption output 2
PHYS: Cardiac
electrophysiology 1

Baumgarten Baumgarten Baumgarten Ford Baumgarten

10 AM HIST: Cardiovascular
System
BEHS: Introduction to
Human Development
PHYS: Control of cardiac BEHS: Preschool Years
output 1
BEHS: Brain
Development
O'Keefe
Merchant Sonenklar Ford Sonenklar

11 AM BEHS: The BEHS:


First Couple Developmen
HIST: Skin BEHS: The School-Age
Child
BEHS: Adolescence

Years of Life During the Merchant Meyer


Second O'Keefe
Sonenklar Year of Life

12 PM Histo 1 Practical
Self-grade
Location: CBIL

1 PM Electives FCM : SG Session 10 FCM : SG Session 10

2 PM

3 PM

4 PM

5 PM

Blockschedule printed from the web on 11/20/2008. Dates are subject to change. Please refer to the eCurriculum website at https://ecurriculum.som.vcu.edu/portal
for the most recent schedule.

4
1 of 1 11/20/2008 9:48 AM
Print Blockschedule https://ecurriculum.som.vcu.edu/portal/student/blockschedule_p.aspx?cla...

Block Schedule

M1 Blockschedule
2/2/2009 - 2/6/2009
Mon 2/2/2009 Tue 2/3/2009 Wed 2/4/2009 Thu 2/5/2009 Fri 2/6/2009
8 AM PHYS: Microcirculation PHYS: Peripheral
circulation and its control
PHYS: Cardiovascular
problem solving 1
Pittman
Pittman Baumgarten

9 AM PHYS: Cardiac
electrophysiology 2
PHYS: Cardiac
autonomics
PHYS: Special
circulations 1
PHYS: Special
circulations 2
PHYS: Regulation of
arterial pressure 1

Baumgarten Baumgarten Pittman Pittman Ford

10 AM BEHS: Young Adulthood BEHS: Child


Maltreatment
HIST: Cartilage HIST: Bone BEHS: Dying and Death

Sonenklar Haar Haar Silverman


Meyer

11 AM BEHS: Middle Years BEHS: Trauma and the


Developing Brain
BEHS: Older Adulthood HIST: Bone Development

Sonenklar Sonenklar Haar


Sood Sood

12 PM

1 PM HIST: Histology of Aging


Organs
Electives HIST:
Histology
FCM : SG Session 11 FCM : SG Session 11

Small Group
Boling Laboratories

2 PM HIST: Histology Small


Group Laboratory

3 PM

4 PM

5 PM

Blockschedule printed from the web on 11/20/2008. Dates are subject to change. Please refer to the eCurriculum website at https://ecurriculum.som.vcu.edu/portal
for the most recent schedule.

5
1 of 1 11/20/2008 9:48 AM
Print Blockschedule https://ecurriculum.som.vcu.edu/portal/student/blockschedule_p.aspx?cla...

Block Schedule

M1 Blockschedule
2/9/2009 - 2/13/2009
Mon 2/9/2009 Tue 2/10/2009 Wed 2/11/2009 Thu 2/12/2009 Fri 2/13/2009
8 AM PHYS: ECG
measurement 1
PHYS: ECG
measurement 2
PHYS: Cardiovascular
problem solving 2

Baumgarten Baumgarten Pittman

9 AM PHYS: Regulation of
arterial pressure 2
PHYS: Exercise and
hemorrhage
PHYS: Cardiac cycle PHYS: Physiology CV
Lab, Group 2
PHYS: Physiology CV
Lab, Group 3
Baumgarten
Ford Ford Baumgarten Baumgarten

10 AM BEHS: Emotional
Problems of Medial
BEHS: Divorce PHYS: Temperature
regulation
Students, Doctors and Sonenklar
Their Families Ford

Silverman

11 AM BEHS: Your Patients and


their Reactions to Illness
BEHS: Elder Abuse BEHS: Behavioral
Sciences Review
and Injury Sonenklar
Sonenklar
Silverman

12 PM

1 PM HIST: Histology of Aging


Organs
Electives HIST:
Histology
FCM : SG Session 11 FCM : SG Session 11 BEHS: Behavioral
Sciences Exam
Small Group
Boling Laboratories

2 PM HIST: Histology Small


Group Laboratory

3 PM

4 PM

5 PM

Blockschedule printed from the web on 11/20/2008. Dates are subject to change. Please refer to the eCurriculum website at https://ecurriculum.som.vcu.edu/portal
for the most recent schedule.

6
1 of 1 11/20/2008 9:49 AM
Print Blockschedule https://ecurriculum.som.vcu.edu/portal/student/blockschedule_p.aspx?cla...

Block Schedule

M1 Blockschedule
2/16/2009 - 2/20/2009
Mon 2/16/2009 Tue 2/17/2009 Wed 2/18/2009 Thu 2/19/2009 Fri 2/20/2009
8 AM HIST: Histology Practical PHYS: PHYSIOLOGY 2,
2 Exam HISTOLOGY 2 EXAMS
Location: CBIL

9 AM PHYS: Physiology CV
Lab, Group 1
PHYS: Body Fluids 1 PHYS: Body Fluids 2

Costanzo, L. Costanzo, L.
Baumgarten

10 AM HIST: Urinary System PHYS: Introduction to GI


physiology and GI
Merchant hormones

Grider

11 AM HIST: Oral Cavity and


Glands

Merchant

12 PM

1 PM PHYS: Physiology
Review
Electives FCM : SG Session 12 FCM : SG Session 12

Baumgarten
Ford
Pittman

2 PM

3 PM

4 PM

5 PM

Blockschedule printed from the web on 11/20/2008. Dates are subject to change. Please refer to the eCurriculum website at https://ecurriculum.som.vcu.edu/portal
for the most recent schedule.

7
1 of 1 11/20/2008 9:49 AM
Print Blockschedule https://ecurriculum.som.vcu.edu/portal/student/blockschedule_p.aspx?cla...

Block Schedule

M1 Blockschedule
2/23/2009 - 2/27/2009
Mon 2/23/2009 Tue 2/24/2009 Wed 2/25/2009 Thu 2/26/2009 Fri 2/27/2009
8 AM

9 AM HIST: Digestive System


and Glands I
PHYS: Clearance, RBF, PHYS: Reabsorption and
GFR Secretion 1
PHYS: Secretion 2:
Pancreatic, intestinal,
PHYS: Na transport 2

and biliary Costanzo, L.


Merchant Costanzo, L. Costanzo, L.
Grider

10 AM PHYS: Regulation of
Body Fluids
HIST: Digestive System
and Glands 2
PHYS: Motility 2:
Intestinal and colonic
PHYS: Reabsorption and
Secretion 2
PHYS: Absorption 1

Grider
Costanzo, L. Merchant Grider Costanzo, L.

11 AM PHYS: Motility 1: Oral,


esophageal, and gastric
PHYS: Secretion 1:
Salivary and gastric
PHYS: Na transport 1 PHYS: Absorption 2

Costanzo, L. Grider
Grider Grider

12 PM Histo 2 Practical
Self-grade
Project HEART

Location: CBIL

1 PM FCM : SG Session 12 FCM : SG Session 12 FCM : Clinical Skills


Practice Sessions - By
Appointment

2 PM

3 PM

4 PM

5 PM

Blockschedule printed from the web on 11/20/2008. Dates are subject to change. Please refer to the eCurriculum website at https://ecurriculum.som.vcu.edu/portal
for the most recent schedule.

8
1 of 1 11/20/2008 9:49 AM
Print Blockschedule https://ecurriculum.som.vcu.edu/portal/student/blockschedule_p.aspx?cla...

Block Schedule

M1 Blockschedule
3/2/2009 - 3/6/2009
Mon 3/2/2009 Tue 3/3/2009 Wed 3/4/2009 Thu 3/5/2009 Fri 3/6/2009
8 AM HIST: Histology 3
Practical and Written
PHYS: Physiology review PHYS: PHYSIOLOGY 3
EXAM
Exams Costanzo, L.
Location: CBIL Grider

9 AM PHYS: K transport PHYS: GI Cases

Costanzo, L. Grider

10 AM PHYS: Concentration
and Dilution of Urine 1
PHYS: Renal Problem
Solving

Costanzo, L. Costanzo, L.

11 AM PHYS: Concentration
and Dilution of Urine 2

Costanzo, L.

12 PM

1 PM FCM : SG Session 13 FCM : SG Session 13 FCM : Clinical Skills


Practice Sessions - By
Appointment

2 PM

3 PM

4 PM

5 PM

Blockschedule printed from the web on 11/20/2008. Dates are subject to change. Please refer to the eCurriculum website at https://ecurriculum.som.vcu.edu/portal
for the most recent schedule.

9
1 of 1 11/20/2008 9:50 AM
Print Blockschedule https://ecurriculum.som.vcu.edu/portal/student/blockschedule_p.aspx?cla...

Block Schedule

M1 Blockschedule
3/9/2009 - 3/13/2009
Mon 3/9/2009 Tue 3/10/2009 Wed 3/11/2009 Thu 3/12/2009 Fri 3/13/2009
8 AM Spring Break Spring Break Spring Break Spring Break Spring Break

9 AM

10 AM

11 AM

12 PM

1 PM

2 PM

3 PM

4 PM

5 PM

Blockschedule printed from the web on 11/20/2008. Dates are subject to change. Please refer to the eCurriculum website at https://ecurriculum.som.vcu.edu/portal
for the most recent schedule.

10
1 of 1 11/20/2008 9:51 AM
Print Blockschedule https://ecurriculum.som.vcu.edu/portal/student/blockschedule_p.aspx?cla...

Block Schedule

M1 Blockschedule
3/16/2009 - 3/20/2009
Mon 3/16/2009 Tue 3/17/2009 Wed 3/18/2009 Thu 3/19/2009 Fri 3/20/2009
8 AM PHYS: Lung volumes
and capacities
PHYS: Mechanics of
breathing 2
PHYS: Oxygen transport PHYS: Physiology
Spirometry lab, Group 1
PHYS: Physiology
Spirometry lab, Group 2
Costanzo, L.
Costanzo, L. Costanzo, L. Costanzo, L. Costanzo, L.

9 AM PHYS: Mechanics of
breathing 1
PHYS: Physical
chemistry of gases and
PHYS: Carbon dioxide
transport
gas exchange
Costanzo, L. Costanzo, L.
Costanzo, L.

10 AM HIST: Respiratory
System
HIST: Lymphoid Tissues
and Organs
IMMU: Overview of the
Immune System
IMMU: Recognition of
Antigen by B Cells:
IMMU: Recognition of
Antigen by B Cells: Ig
Immunoglobulin Expression and the Role
Haar Haar Lebman Structure and Function of Antigen in
Immunoglobulin
Lebman Diversity

11 AM IMMU: Cells and Tissues IMMU: Recognition of


Antigen by B Cells:
IMMU: The Development
of Lymphocytes: B Cell
Lebman Antigen Independent Development in the Bone
Development of the Marrow
Immunoglobulin
Diversity Lebman

12 PM Histo 3 Practical
Self-grade
Project HEART

Location: CBIL

1 PM FCM : SG Session 13 FCM : SG Session 13 FCM :


Clinical
PHYS:
Physiology
Skills Spirometry
Practice Lab, Group
Sessions - 3
By

2 PM
Appointment Costanzo, L.

3 PM

4 PM

5 PM

Blockschedule printed from the web on 11/20/2008. Dates are subject to change. Please refer to the eCurriculum website at https://ecurriculum.som.vcu.edu/portal
for the most recent schedule.

11
1 of 1 11/20/2008 9:51 AM
Print Blockschedule https://ecurriculum.som.vcu.edu/portal/student/blockschedule_p.aspx?cla...

Block Schedule

M1 Blockschedule
3/23/2009 - 3/27/2009
Mon 3/23/2009 Tue 3/24/2009 Wed 3/25/2009 Thu 3/26/2009 Fri 3/27/2009
8 AM PHYS: Pulmonary
Circulation
PHYS: Acid-base 2 PHYS: Acid-base 3 PHYS: Renal acid-base 1 PHYS: Renal acid-base 2

Costanzo, L. Costanzo, L. Costanzo, L. Costanzo, L.


Costanzo, L.

9 AM PHYS: Acid-base 1 PHYS: Ventilation -


perfusion 1
PHYS: Ventilation -
perfusion 2
PHYS: Respiratory
rhythm and regulation of
PHYS: Ventilation in
exercise and at high
Costanzo, L. breathing altitude
Costanzo, L. Costanzo, L.
Costanzo, L. Costanzo, L.

10 AM IMMU: The Development IMMU: Antigen


of Lymphocytes: B Cell Presentation to the TCR
IMMU: The Development
of T Lymphocytes
IMMU: Adaptive
Immunity: activation of
IMMU: Adaptive
Immunity: Effector T Cell
Development in Part 1: Generation of naive T cells Function - I
Peripheral Lymphoid Ligands for the TCR Straus
Tissue Straus Smeltz
Lebman

11 AM IMMU: Recognition of IMMU: Antigen


Antigen by T Cells: TCR Presentation to the TCR
IMMU: The Development IMMU: Elements of the
of Lymphocytes: Positive Immune System:
IMMU: Adaptive
Immunity: Effector T Cell
Part 2: MHC and Negative Selection of Cytokines Function - II
Lebman T Cells
Lebman Smeltz Smeltz
Straus

12 PM Histo 4 Exam Self-grade


Location: CBIL

1 PM HIST: Histology 4
Practical and Written
FCM : SG Session 14 FCM : SG Session 14 PHYS: Physiology
review--Part 1
Exams
Location: CBIL Costanzo, L.

2 PM FCM : Clinical Skills


Practice Sessions - By
Appointment

3 PM

4 PM

5 PM

Blockschedule printed from the web on 11/20/2008. Dates are subject to change. Please refer to the eCurriculum website at https://ecurriculum.som.vcu.edu/portal
for the most recent schedule.

12
1 of 1 11/20/2008 9:51 AM
Print Blockschedule https://ecurriculum.som.vcu.edu/portal/student/blockschedule_p.aspx?cla...

Block Schedule

M1 Blockschedule
3/30/2009 - 4/3/2009
Mon 3/30/2009 Tue 3/31/2009 Wed 4/1/2009 Thu 4/2/2009 Fri 4/3/2009
8 AM PHYS: Metabolic
acidosis and alkalosis
PHYS: Respiratory
acidosis and alkalosis
PHYS: Physiology review PHYS: FREE STUDY PHYS: PHYSIOLOGY 4,
IMMUNOLOGY 1 EXAMS
Costanzo, L.
Costanzo, L. Costanzo, L.

9 AM PHYS: Respiratory
pathophysiology cases
PHYS: Acid-base
problem solving

Costanzo, L. Costanzo, L.

10 AM IMMU: Mechanisms of T
Cell Tolerance
IMMU: Adaptive
Immunity: B Cells and
IMMU: Immunology
Review
Antibodies
McCoy Barbour
Lebman Lebman
McCoy

11 AM IMMU: Adaptive IMMU: Elements of the Straus


Immunity: B Cells and Immune System: Smeltz
Antibodies Complement

Lebman Barbour

12 PM

1 PM FCM : SG Session 14 FCM : SG Session 14

2 PM

3 PM

4 PM

5 PM

Blockschedule printed from the web on 11/20/2008. Dates are subject to change. Please refer to the eCurriculum website at https://ecurriculum.som.vcu.edu/portal
for the most recent schedule.

13
1 of 1 11/20/2008 9:52 AM
Print Blockschedule https://ecurriculum.som.vcu.edu/portal/student/blockschedule_p.aspx?cla...

Block Schedule

M1 Blockschedule
4/6/2009 - 4/10/2009
Mon 4/6/2009 Tue 4/7/2009 Wed 4/8/2009 Thu 4/9/2009 Fri 4/10/2009
8 AM PHYS: Mechanism of
hormone action
PHYS: Thyroid 1 PHYS: Thyroid 2 PHYS: Thyroid 3 PHYS: Growth hormones
1
Kalimi Kalimi Kalimi
Kalimi Kalimi

9 AM HIST: Endocrine System HIST: Endocrine System PHYS: ADH and oxytocin PHYS: Adrenal cortex 1 PHYS: Growth hormones
2
Haar Haar Witorsch Witorsch
Kalimi

10 AM IMMU: Innate Immunity


Part I - The Immediate
IMMU: The Body's
Defense Against
IMMU: Failures of the
Body's Defenses:
PHYS: Adrenal cortex 2 IMMU: Principles of
Vaccination
Response Infection: Memory Pathogen Evasion or Witorsch
Subversion of the Conrad
Lebman Conrad Immune System

11 AM IMMU: Innate Immunity


Part 2 - The Induced
IMMU: The Adaptive
Response to Infection
IMMU: Failures of the
Body's Defenses:
IMMU: Immune
Response at Mucosal
IMMU: Over-Reactions of
the Immune System:
Response Inherited Surfaces Type I Hypersensitivity
Conrad Immunodeficiency
Lebman Diseases Lebman Schwartz

12 PM
FCM : SG Session 15 - FCM : SG Session 15 -
CS Exam 2 CS Exam 2

1 PM

2 PM

3 PM

4 PM

5 PM

Blockschedule printed from the web on 11/20/2008. Dates are subject to change. Please refer to the eCurriculum website at https://ecurriculum.som.vcu.edu/portal
for the most recent schedule.

14
1 of 1 11/20/2008 9:52 AM
Print Blockschedule https://ecurriculum.som.vcu.edu/portal/student/blockschedule_p.aspx?cla...

Block Schedule

M1 Blockschedule
4/13/2009 - 4/17/2009
Mon 4/13/2009 Tue 4/14/2009 Wed 4/15/2009 Thu 4/16/2009 Fri 4/17/2009
8 AM HIST: Male Reproductive PHYS: Adrenal cortex 3
System
IMMU: Disruption of
Healthy Tissue by the
PHYS: Male reproduction
1
PHYS: Insulin 1

Witorsch Immune Response: Kalimi


Bigbee Autoimmunity Witorsch

Tew

9 AM HIST: Female
Reproductive System
PHYS: Male reproduction
2
PHYS: Insulin 2

Kalimi
Bigbee Witorsch

10 AM IMMU: Over-Reactions of
the Immune System:
PHYS: PTH/Vitamin D 1
and 2
IMMU: Disruption of
Healthy Tissue by the
IMMU: Tumor
Immunology
Types II, III, and IV Immune Response:
Hypersensitivity Costanzo, L. Genetic and Manjili
Environmental Factors
Schwartz

11 AM IMMU: HIV/AIDS IMMU: Aging and the


Immune Response
Nixon
Boling

12 PM Project HEART

1 PM HIST: Placenta & Breast FCM : SG Session 15 -


CS Exam 2
FCM : SG Session 15 -
CS Exam 2
Bigbee

2 PM

3 PM

4 PM

5 PM

Blockschedule printed from the web on 11/20/2008. Dates are subject to change. Please refer to the eCurriculum website at https://ecurriculum.som.vcu.edu/portal
for the most recent schedule.

15
1 of 1 11/20/2008 9:52 AM
Print Blockschedule https://ecurriculum.som.vcu.edu/portal/student/blockschedule_p.aspx?cla...

Block Schedule

M1 Blockschedule
4/20/2009 - 4/24/2009
Mon 4/20/2009 Tue 4/21/2009 Wed 4/22/2009 Thu 4/23/2009 Fri 4/24/2009
8 AM PHYS: Insulin 3 PHYS: Male reproduction
3
PHYS: Growth factors PHYS: Physiology review

Kalimi Kalimi Costanzo, L.


Witorsch Kalimi
Witorsch

9 AM IMMU: Transplantation PHYS: Female


reproduction 1
PHYS: Female
reproduction 3
Straus
Witorsch Witorsch

10 AM IMMU: Immunology Small IMMU: Immunology Small


Groups: Case Study Groups: Case Study
IMMU: Immunology
Review
Reviews Reviews
Conrad
Lebman
Ryan

11 AM HIST: Histology 5 Tew


Practical and Written
Exam
Location: CBIL

12 PM

1 PM PHYS: Female
reproduction 2
FCM : SG Session 16 FCM : SG Session 16 PHYS: PHYSIOLOGY 5,
IMMUNOLOGY 2 EXAMS

Witorsch

2 PM

3 PM

4 PM

5 PM

Blockschedule printed from the web on 11/20/2008. Dates are subject to change. Please refer to the eCurriculum website at https://ecurriculum.som.vcu.edu/portal
for the most recent schedule.

16
1 of 1 11/20/2008 9:53 AM
Physiology Overview

Faculty. The MI Physiology Course is team-taught by faculty from the Department of


Physiology at Virginia Commonwealth University. Their names, Sanger Hall room
numbers, phone numbers, and email addresses are listed below. Please contact the
appropriate faculty member outside of class if you are having difficulty with his/her
material. We expect to see you often!

Faculty member Room Phone Email


Dr. Clive Baumgarten 4-034 828-4773 baumgart@vcu.edu
Dr. Margaret Biber 2-001 828-9756 mbiber@vcu.edu
Dr. Linda Costanzo 1-003 828-7559 lcostanz@vcu.edu
Dr. Joseph Feher 4-040 828-9761 feher@vcu.edu
Dr. George Ford 3-009 828-9501 ford@vcu.edu
Dr. Jack Grider 12-004 828-1853 jgrider@vcu.edu
Dr. Mohammed Kalimi B1-008 828-9500 mkalimi@vcu.edu
Dr. Srinivasa Karnam 12-002 828-8504 skarnam@vcu.edu
Dr. Diomedes Logothetis 3-038a 267-0044 delogothetis@vcu.edu
Dr. Roland Pittman 6-018 828-9545 pittman@vcu.edu
Dr. Raphael Witorsch 4-061 828-4486 witorsch@vcu.edu

Method of Presentation. The material in this course is presented primarily in lecture


format by organ system (i.e., cell, muscle, and autonomics; cardiovascular;
gastrointestinal; renal; respiratory; acid-base; and endocrine and reproduction). In
addition to lectures, there are case-based problem solving sessions and laboratories, all of
which will be tested on examinations. Important: If discrepancies arise between the
material taught in lecture/syllabus and the recommended textbooks, the lecture/syllabus
will take precedence.

Syllabus. Each lecturer has prepared a syllabus to accompany his/her lectures that
reflects the topics that he/she considers most important. The syllabus will be your
constant companion and best friend for this course! The best strategy for doing well is to
pre-read the syllabus before lecture — even a cursory preview of the lecture will give you
a sense of where the lecture is “going” and alert you to difficult concepts.

Laboratories. Laboratories will be used to illustrate and apply important principles that
may be difficult to grasp in a lecture and to provide an active, hands-on experience. There
are two laboratories during the physiology course, which will be carried out in small
groups. Each group is assigned a faculty member who will guide the students through the
laboratory exercises. The laboratory handouts and bench assignments are included in the
syllabus. (The laboratory dates and times are shown on the master schedule; you can
determine from the bench assignments whether you are in Group 1, 2, or 3.) Laboratories
will be tested on the regular physiology examinations (see examinations).
Physiology Overview

Attendance. It is expected that you will attend lectures, problem-solving sessions, and
laboratories. Physiology is a highly conceptual subject. While certain “facts” can be
memorized, in general the subject matter must be understood on the basis of its
principles. Lectures are an important time for faculty members to help you navigate and
solidify these principles. Get into the rhythm of pre-reading, attending lecture,
reviewing....your life will be immeasurably better!

Textbooks. There is no required textbook for MI Physiology. You may wish to purchase
one of these comprehensive textbooks to complement the lectures and syllabus.

Physiology 3rd Edition, L.S. Costanzo, W.B. Saunders, 2006. Didactic, concise coverage
of all topics.

Physiology, R. M. Berne and M.N. Levy, 6th Edition, Mosby 2008. In-depth coverage of
all topics.

In addition, the following book helps students gain mastery of the most important
physiologic principles through case-based questions and problems. This is a book to be
used in MI Physiology and, again, throughout the MII year.

Physiology Cases and Problems, 3rd Edition, L. S. Costanzo, Lippincott Williams and
Wilkins, 2009.

In addition to these two comprehensive texts, there are excellent physiology monographs:

Cardiovascular Physiology, R.M. Berne and M.N. Levy, 8th Edition, Mosby, 2001.
Gastrointestinal Physiology, L.R. Johnson, 6th Edition, Mosby, 2001.
Renal Function, H. Valtin and J. Schafer, 3rd Edition, Little-Brown, 1995.
Respiratory Physiology, J.B. West, 7th Edition, Lippincott Williams and Wilkins, 2005.

Geriatrics. Throughout the course, we will emphasize geriatric correlates of physiologic


material, as appropriate. In addition, Dr. Catherine Kelso and I have developed several
brief geriatric cases that we will present on the electronic bulletin board; open-ended
questions will be posed along with each case. There will be one examination
question/geriatric case, for which you can prepare by reviewing the case and its
questions/answers on the e-board.

Examinations. There are five examinations in MI Physiology. For the examinations,


there will be 3 questions per lecture hour and 3 questions per laboratory or problem-
solving session and 1 question for each geriatric case presented on the electronic
bulletin board. All questions will be multiple-choice format. The physiology
examinations will be secured, thus students must hand in their exams with their bubble
sheets. The item analysis of the examination will be used to detect inappropriate
questions.

18
Physiology Overview

Grading. Every examination question in the physiology course counts equally (per
above). Each examination will have a slightly different number of questions, depending
on the number of lectures, labs, and problem sessions tested in that block’s exam. Your
final percentage grade in the course will be the total number of questions you answered
correctly, divided by the total number of questions asked. (You can track your course
grade in the same way.)

Make-up Examinations. It is recognized that valid circumstances may preclude


attendance at a particular examination. Under these circumstances, the student must
request permission in advance from the School of Medicine Curriculum Office to take a
test early or to take a re-test. Such requests may or may not be granted depending on the
reason for missing the scheduled examination. The re-test may not be the same
examination administered to the rest of the class and it may consist of essay or oral
questions by one or more examiners.

Finally, our goals for you in the physiology course are that you will:

• Gain a solid understanding of the principles of physiology that underlie


pathophysiology and clinical medicine.
• Appreciate the physiologic connections between and among systems.
• Know how to apply physiologic concepts in problem-solving, especially for
pathophysiology.
• Care about physiology and be motivated to learn more.
• Always “think physiologically.”

Please let me know if there is any way I can be of help.

Linda S. Costanzo
Course Director

19
Compartments Cell Membranes - Dr. Costanzo

Body Fluid Compartments and Cell Membranes


Linda S. Costanzo, Ph.D.

OBJECTIVES:

1. Describe the major and minor body fluid compartments.


2. Compare the total cation and anion concentrations in mEq/l in each compartment
and comment on their relation to the law of macroscopic electroneutrality.
3. Compare the ion concentrations between the plasma water and the interstitial fluid
in terms of the Gibbs-Donnan equilibrium.
4. Describe the cell membrane lipid bilayer structure and the role of the
phosophlipids, cholesterol, and glycolipids in cell membranes.
5. Describe the concept of the fluid mosaic membrane model comparing integral and
peripheral proteins.
6. Describe the role of the cytoskeleton in cell membrane protein localization and
the unique role of glycoproteins.
7. Categorize types of membrane transport with respect to whether they are
“downhill” or “uphill” and whether they are carrier-mediated.

Suggested Reading:

Physiology, edited by: R.M. Berne and M. N. Levy, Mosby, 6th Ed. pp 5-7; 20-24.
Physiology, L.S. Costanzo, Saunders, pp 1-6, 8-12

I. VOLUME AND DISTRIBUTION OF BODY FLUIDS

Water content (total body water, or TBW) comprises about 60% of body weight. The
percentage varies between 50-70%, depending on gender and amount of adipose
tissue. Males tend to have a higher percentage of water than females. Water content is
inversely correlated with adipose tissue. Infants have up to 75% body weight as
water, which is why severe diarrhea can be life-threatening.

Water is distributed between two major compartments: intracellular fluid (ICF) and
extracellular fluid (ECF), which are separated from each other by cell membranes.
ICF is 2/3 of TBW and ECF is 1/3 of TBW. ECF is further sub-divided into two
compartments, the interstitial fluid and plasma compartments, which are separated
from each other by capillary walls. Interstitial fluid is 3/4 of ECF, and plasma water is
1/4 of ECF. Lymph, which is part of the ECF, is interstitial fluid that is collected in
the lymphatic vessels and then returned to the plasma compartment.

An additional minor compartment is the transcellular fluid, which is not part of ICF
or ECF. Transcellular fluids are separated from the rest of the body fluids by a layer
of cells, and they include gastrointestinal, peritoneal, pleural, and cerebrospinal
fluids. Collectively, the volume of transcellular fluids is small, so they are ignored in
the above summary numbers.

20
Compartments Cell Membranes - Dr. Costanzo

A simple tool is the 60-40-20 rule. Approximately 60% of body weight is water
(TBW), 40% of body weight is ICF, and 20% is ECF. (ICF is 2/3 of TBW, i.e., 40%
of body weight; ECF is 1/3 of TBW, i.e., 20% of body weight.)

II. COMPOSITION OF BODY FLUIDS

A. Units

A few tidbits on units. Please save for reference throughout the course!

1. Concentrations in body fluids are often expressed in molarity, such as


mmol/L.

2. For electrolytes, we sometimes use equivalents, such as mEq/L, which


is the concentration in mmol/L x charge on the ion. Thus, for univalent
ions, mEq/L = mmol/L; for divalent ions, mEq/L = 2 x mmol/L. For
example, a Na+ concentration of 1 mmol/L = 1 mEq/L; a Ca2+
concentration of 1 mmol/L = 2 mEq/L.

21
Compartments Cell Membranes - Dr. Costanzo

3. Osmolarity is total solute concentration, expressed in units of


mOsmoles/liter. Osmolarity is concentration of solute particles, or
concentration in mmol/L x number of particles that dissociate in
solution (called “g,” the osmotic coefficient). For example, the
osmolarity of 150 mmol/L NaCl = 150 mmol/L x 2 = 300 mOsm/L
(since NaCl dissociates into two particles in solution, i.e., g = 2).
Osmolality is virtually the same thing as osmolarity, but expressed as
mOsmoles/kg H20. Plasma osmolarity can be approximated as 2 x
[Na+].

4. Substances like proteins are conventionally expressed in g/dL, where


a dL (deciliter) is 100 ml and is also called “%.”

5. % can mean “g per 100 ml.” For example, 0.9% NaCl is 0.9 g
NaCl/100 ml. It’s weird, but that’s what it means. Likewise, mg %
can mean “mg per 100 ml.” For example, 5 mg% KCl means 5 mg
KCl/100 ml.

B. Composition

The approximate ionic compositions of the plasma water, interstitial fluid, and
intracellular compartments are shown in Table 1. Plasma water and interstitial
fluid are ECF, while the muscle cell values represent ICF.

Interstitial fluid
Ion Plasma Water (mEq/l) Muscle cell (mEq/L)
(mEq/l)
Cations
Na+ 147.4 140.0 17.5
K+ 4.2 4.0 120.0
free Ca2+ 2.7 2.4 ~ 10-4 (rest bound)
free Mg2+ 1.2 1.1 10.0
Total Cations 155.5 147.5 147.5
Anions
Cl- 104.8 110.0 10.0
HCO3- 25.8 27.1 12.0
phosphates 2.2 2.3 40.0
proteins 15.0 0.0 54.0
other anions 7.7 8.1 31.5 (ATP etc)
Total Anions 155.5 147.5 147.5
Table 1.

1. Note that the total ion concentration, in mEq/l, for any compartment
(e.g., plasma water) obeys the law of macroscopic electroneutrality,

22
Compartments Cell Membranes - Dr. Costanzo

i.e. the concentration of positive charges must always equal the


concentration of negative charges. This law applies, and is always
true, for any body fluid compartment. (Remember, macroscopic
electroneutrality is not just a good idea…it’s the law!)

2. The individual ionic compositions of the ICF are very different from
those of the ECF. For example, the Na+ concentration is much lower in
the ICF than in the ECF, while the K+ concentration is much higher in
the ICF than in the ECF. These differences in concentration across cell
membranes are created and maintained by a cell membrane Na+/K+
pump, that will be discussed in a subsequent lecture. The large Na+
gradient across cell membranes that is created by the Na+/K+ pump is,
in turn, utilized by cells in many critical functions; for example, the
Na+ gradient is the basis of the upstroke of the action potential in
nerve and muscle and is the energy source for the uphill transport of
various other solutes (see later lectures).

Also, the free Ca2+ concentration is much lower in the ICF than in the
ECF. Cell membrane Ca2+ ATPase and Ca2+-Na+ exchange help to
keep intracellular free Ca2+ in the submicromolar range. Also, a large
fraction of the intracellular Ca2+ is sequestered in cell organelles and is
released only transiently in connection with important cell functions,
such as muscle contraction, signal transduction, and release of
hormones or neurotransmitter.

3. The compositions of the plasma and interstitial fluid (both part of


the ECF) are very similar, but not identical. The major difference in
the composition of plasma and interstitial fluid compartments is that
the plasma contains large, negatively charged proteins, while the
interstitial fluid is essentially protein-free. This difference, in turn,
accounts for the small differences in concentration of small,
diffuseable ions (e.g., Na+, K+, Cl-) in these two compartments. The
reason for these differences is that the small ions are free to diffuse
between the plasma and the interstitial fluid across the capillary
membranes, while the large protein anions are restricted to the plasma.
Because the law of macroscopic electroneutrality must be obeyed in
both plasma and interstitial fluid compartments, the plasma (with its
negatively charged protein) will have a slightly higher concentration of
diffuseable cations and a slightly lower concentration of diffuseable
anions relative to the interstitial fluid. This special equilibrium, due to
the presence of protein on one side of the membrane, is called the
Gibbs- Donnan equilibrium, which is discussed below.

III. GIBBS-DONNAN EQUILIBRIUM

23
Compartments Cell Membranes - Dr. Costanzo

A. Hypothetical example

Figure 2.

This simplified drawing explains what happens when a Gibbs-Donnan


equilibrium is established. Solution 2 has protein (Pr-), and Solution 1 has no
protein. In the initial system (before Gibbs-Donnan equilibrium is established),
there is 9 mEq/l of NaCl in Solution 1 and 9 mEq/l of NaPr in Solution 2. The
anion Pr- is assumed to be impermeable across the membrane that separates the
two solutions, but Na+ and Cl- are freely permeable across the membrane. Since
Solution 2 initially contains no Cl-, Cl- will begin moving from Solution 1 to 2
down its concentration gradient. To preserve macroscopic electroneutrality (the
law!), an equal number of Na+ must move with the Cl- . As the Na+ concentration
in phase 2 increases, Na+ backflux from Solution 2 to Solution 1 will eventually
prevent further net Na+ diffusion, and eventually also stop the further movement
of Cl-. Because of the presence of the impermeant protein and because of the
requirement for electroneutrality of both solutions, neither Na+ nor Cl- can
achieve equal concentrations on both sides; however, at equilibrium (the Gibbs-
Donnan equilibrium), they achieve a compromise position. This equilibrium
condition is expressed as an equality of ion products:

where the subscripts 1 and 2 refer to the two solutions, and x is the number of
meq/l of Cl- and Na+ that have moved from Solution 1 to Solution 2. Solving the
equation, for this example, we find that x = 3. At equilibrium, [Na+]1 = 6 mEq/l,
[Cl-]1 = 6 mEq/l, [Na+]2 = 12 mEq/l, and [Cl-]2 = 3 mEq/l. [Pr-]2 remains at 9
mEq/l. Note that at equilibrium, macroscopic electroneutrality still holds! In this
case [Na+]1 = [Cl-]1, = 6 mEq/l and [Na+]2 = [Cl-]2 + [Pr-]2 = 12 mEq/l. The
equilibrium condition also defines a constant ratio, r, called the Gibbs-Donnan
ratio. This is:

24
Compartments Cell Membranes - Dr. Costanzo

r = [Na+]1 /[Na+]2 = [Cl-]2/[Cl-]1

In this hypothetical example, r = 0.5.

B. Real life example

Extending to real life, we are interested in the Gibbs-Donnan ratio for plasma (p)
and interstitial fluid (i), where r for the common ions is:

Substituting actual values in plasma and interstitial fluid from Table 1, we find
that r = 0.95. Again, this redistribution of small ions across the capillary
membrane is due to the presence of negatively charged protein in the plasma but
not in the interstitial fluid.

IV. CELL MEMBRANE STRUCTURE

The membranes that separate the ICF and ECF serve as physical barriers and
also contain a variety of proteins involved in transport of substances between
the ICF and ECF. In addition, membrane proteins act as enzymes, receptors
for ligands such as hormones and neurotransmitters, and as antigens.

The basic structure of the cell membrane is a lipid bilayer, which consists of
phospholipids, cholesterol, and sphingolipids. The phospholipids present in
cell membranes are characterized as amphipathic, i.e. part of their structure is
non-polar and hydrophobic, and part of their structure is polar and
hydrophilic. Such amphipathic molecules are most stable when they are
“sitting” at the interface between an aqueous phase (polar) and a lipid or oil
phase (non-polar). For example at the interface between oil and water,
phospholipids form a monolayer with the polar “head” of the molecule in the
aqueous phase and the non-polar long-chain fatty acid “tails” in the non-polar
oil phase. Phospholipids can also form a stable structure separating two
aqueous solutions (e.g., ICF and ECF); in this arrangement, the non-polar tails
point toward each other to form a bilayer, and the polar heads make contact
with the aqueous solutions on either side. This lipid bilayer is the backbone of
the cell membrane and accounts for the typical membrane thickness of about
10 nm.

lipid monolayer

25
Compartments Cell Membranes - Dr. Costanzo

lipid bilayer

Figure 3.

A. Typical Lipid Profile of a Cell Membrane

Lipid Percent by Weight


Phosphatidylcholine 17.5
Sphingomyelin 16.0
Phosphatidylethanolamine 16.6
Phosphatidylserine 7.9
Phosphatidylinositol 1.2
Phosphatidic acid 0.6
Lysophosphatidylcholine 0.9
Cholesterol 26.0
Glycolipids 11.0
Table 2.

1. Phospholipids - The first seven lipids in Table 2 are phospholipids.


The most abundant of these provide the structural barrier. We have
already discussed their critical amphipathic properties.
Phosphatidylinositol is present mainly on the cytoplasmic (ICF) side
of the bilayer and is of special functional importance because it serves
as the substrate for phosphatidylinositol 4,5-diphosphate (PIP2). PIP2
in turn can be broken down to inositol 1, 4, 5 trisphosphate (IP3) and
diacylglycerol (DAG) by the activation of the enzyme, phospholipase
C (PLC) The activation of PLC normally occurs as part of a signal
transduction cascade in response to the binding of a ligand (e.g. a
hormone) to a membrane receptor protein. Both IP3 and DAG serve as
cell “second messengers”.

2. Cholesterol – helps to buffer the fluidity of the non-polar fatty acids


of the phospholipids. If present in the proper amount, cholesterol
prevents the membrane from becoming either too rigid (wax-like) or

26
Compartments Cell Membranes - Dr. Costanzo

too fluid (olive oil-like).

3. Glycolipids – These lipids are linked to carbohydrate moieties that


extend out from the bilayer into the ECF. They often act as membrane
antigens (such as blood group antigens A and B) or, in some cases, as
membrane receptors.

B. Membrane Proteins – Fluid Mosaic Model

In this concept of membrane structure, proteins are thought to be either


imbedded in or adsorbed onto the surface of the lipid bilayer (see figure
below). They are not rigidly held in place (although there are exceptions, see
below), but may diffuse in the plane of the bilayer. There are two main types
of membrane protein:

1. Integral or intrinsic proteins – these proteins usually span the bilayer


and have hydrophobic amino acids that make contact with the fatty
acid chains of the bilayer and hydrophilic amino acids that contact the
ECF and the ICF. It requires detergent action to remove integral
proteins from the lipid. Functionally, integral proteins may be
receptors, enzymes, ion channels, carrier proteins etc.

2. Peripheral or extrinsic proteins – these proteins are bound to either


the interior or exterior membrane surface, and are associated with the
membrane through hydrophilic interactions.

Figure 4.

3. Localization of membrane proteins – Some cell types require


membrane proteins to be confined to specific regions of the membrane
(for example, in epithelial cells, the Na+/K+- ATPase (Na+/K+-pump).
Epithelial cells, such as those lining the intestinal mucosa, are
polarized cells that sit between two different extracellular fluids; their
apical membranes contact the aqueous fluid in the lumen of the
intestine, while their basolateral membranes contact the interstitial
fluid. For functional reasons that will be described later, the Na+/K+-
ATPase is confined exclusively to the basolateral membrane. This can

27
Compartments Cell Membranes - Dr. Costanzo

occur because the Na+/K+- ATPase is linked to the membrane


cytoskeleton via linking proteins, ankyrin and fodrin (see figure
below). The linkage prevents the ATPase from diffusing from the
basolateral to the apical cell membrane.

Figure 5.

4. Membrane Glycoproteins – Carbohydrate moieties can link to


membrane proteins especially at the ECF side of the membrane.
Growth, differentiation, and maintenance of many cell types depend
on interactions between anchoring glycoproteins in the extracellular
matrix, such as fibronectin, and specific glycoproteins on the cell
surface, called integrins. The interaction sets off intracellular
transduction events that, during early development, guide
differentiation, and, at cell maturity, regulate cell division and
programmed cell death (apoptosis).

V. OVERVIEW OF MEMBRANE TRANSPORT

The types of membrane transport are:

• Simple diffusion
• Facilitated diffusion
• Primary active transport
• Secondary active transport (cotransport and countertransport)
• Osmosis (of water)

A. Energetics

1. Downhill transport means a substance is transported down an


electrochemical gradient and requires no consumption of metabolic
energy. Simple and facilitated diffusion illustrate downhill transport.

2. Uphill transport means a substance is transported against an


electrochemical gradient and requires consumption of metabolic
energy (either directly in the form of ATP or indirectly in the form of

28
Compartments Cell Membranes - Dr. Costanzo

an ion gradient). Primary and secondary active transport illustrate


uphill transport.

B. Carrier- or non-carrier mediated

Transport is further characterized by whether it is carrier-mediated or not.


Carrier-mediated transport includes all types except simple diffusion. The
features of carrier-mediated transport include: saturation,
stereospecificity, and competition.

• Saturation. Because carrier proteins have a limited number of


binding sites for the transported solute, saturation of transport
occurs. The point at which all binding sites are occupied by solute
is called the transport maximum or Tm. (Tm is analogous to
Vmax in enzyme kinetics.)

• Stereospecificity. Binding of solute to transporters is highly


stereospecific. For example, in the small intestine, the transporter
for glucose absorption recognizes the natural isomer, D-glucose,

29
Compartments Cell Membranes - Dr. Costanzo

but does not recognize or transport the L-isomer. (Simple


diffusion, in contrast, does not distinguish between the natural and
unnatural isomers.)

• Competition. Although transporters exhibit a high degree of


stereospecificity, still they can be “tricked” into transporting
chemically similar solutes. Thus, D-galactose can compete for the
D-glucose transporter and, by occupying some of the glucose
binding sites, inhibit D-glucose transport; this is an example of
competitive inhibition. There is also a phenomenon of non-
competitive inhibition, in which compounds that are structurally
unrelated to the transported compound bind to the transporter and
prevent it from functioning. In the case of D-glucose transport,
phloretin is a noncompetitive inhibitor.

VI. PRACTICE PROBLEMS

1. A body fluid compartment has a Na+ concentration of 15 mM; this compartment


is most likely the:

A. interstitial fluid compartment


B. ICF
C. plasma
D. small intestinal lumen
E. cerebrospinal fluid

2. A solution contains 0.5 mM MgCl2. The concentrations of Mg2+ and Cl- in meq/l are
respectively:

A. 0.5 , 0.5
B. 1,2
C. 2, 1
D. 1, 1
E. 0.5, 1

3. The Donnan ratio for diffusable ions between two solutions (Solution 1 relative to
Solution 2) is found to be 0.95. Accordingly, which of the following describes the Gibbs-

30
Compartments Cell Membranes - Dr. Costanzo

Donnan equilibrium condition for these two solutions?

A. [Na+]1 = 140 mM, [Na+]2 = 155 mM

B. [Cl-]1 = 106 mM, [Cl-]2 = 100.7 mM

C. [K+]1 = 4 mM, [K+]2 = 4.7 mM

D. [Ca2+]1 = 2 mM, [Ca2+]2 = 2.5 mM

E. [Mg2+]1 = 1 mM, [Mg2+]2 = 1.25 mM

4. The plasma contains anionic proteins that cannot cross the capillary membranes. When
the plasma is in equilibrium with the interstitial fluid:

A. the concentration of permeable anions is higher in the plasma


than in the interstitial fluid.
B. the concentration of permeable cations is higher in the
interstitial fluid than in the plasma.
C. the concentration of permeable anions will be the same in each
fluid phase.
D. the concentration of permeable cations will be the same in each
fluid phase.
E. the concentration of permeable anions is higher in the
interstitial fluid than in the plasma.

5. An impermeable anion is in phase 1 which is in Gibbs-Donnan equilibrium with


phase 2 containing only permeable ions. The following data are obtained: [K+]1 = 6 mM,
[K+]2 = 3.6 mM, and [Cl-]1 = 100 mM. [Cl-]2 is then:

A.100 mM

B.110 mM

C. 90 mM

D. 167 mM

E. 3.6 mM

31
Compartments Cell Membranes - Dr. Costanzo

6. In problem 5, at least one other permeable cation must also be present because:

A. there must always be 2 different cations present.


B. [K+]2 < [Cl-]2 which appears to violate macroscopic
electroneutrality.
C. [Cl-]1 must equal [Cl-]2.
D. the permeable anion concentrations in phase 1 must always equal
the permeable cation concentrations in phase 1.
E. [Cl-]1 < [Cl-]2.

7. The main permeability barrier in the cell plasma membrane resides in the:

A. peripheral proteins

B. integral proteins
C. lipid bilayer
D. lipid monolayer
E. phosphatidalinositol layer

ANSWERS

1. B (a Na+ concentration in this range can only be in the ICF given the choices)
2. D ( [Mg2+] = (2 meq/mmole)(0.5 mmole/l) = 1 meq/l, [Cl-] = (1 meq/mmole)(1
mmole/l) = 1 meq/l)
3. B (this is the only choice that expresses the correct Donnan ratio, viz. [Cl-]2 /[Cl-]1
= 100.7/106 = 0.95)
4. E (Since the interstitial fluid has no protein anions, electroneutrality must be
satisfied by the permeable anions, which means that the latter will be higher in the
interstitial fluid compartment).
5. D (Since: [K+]1 /[K+]2 = [Cl-]2/[Cl-]1, then [Cl-]2 = (6)(100)/3.6 = 167 mM)
6. B (There must be a cation, M+ (assuming a monovalent cation for simplicity)
whose concentration in phase 2 is 163. 4 mM, so that [K+]2 + [M+]2 = [Cl-]2. This
means that [M+]1 = 272.3 mM. The impermeable anion concentration must then
have been 178.3 mM).
7. C

32
Chemical Signaling, I-V - Dr. Kalimi

Chemical Signaling, I-V


Mohammed Y. Kalimi, Ph.D.

Objectives:
After studying this lecture material, the student will:

1. Understand the concept of receptor recognition by various local chemical


mediators and relationship between receptor structure and functions.
2. Identify the various receptor-stimulus-transduction mechanisms and understand
the capacity of cells to modulate a response to a stimulus.
3. Recognize the second messengers and discuss the transduction processes involved
in their synthesis.
4. Discuss the amplification events resulting from second messenger production.

Key Words:

A. Receptors:

1. Intracellular (cytoplasmic or nuclear)


2. Extracellular (cell surface or cell membrane associated)

B. Second Messengers:

Cyclic AMP, Cyclic GMP, 1,4,5 Inositoltriphosphate(IP3), Diacylglycerol(DAG),


Calcium, Eicosanoids (prostaglandins, prostacyclins, thromboxanes, and
leukotriens).

C. Signal Amplifications:

Protein kinases, Protein phosphatases, and GTP-binding proteins

I. Chemical Signals:

Cell to cell communication is important for normal function, growth and


differentiation of cells in a multi-cellular organism.

Cells communicate with each other through various chemical signaling molecules.

A. Three ways of chemical cell communication include:


Direct, Indirect and Gap junctions.
B. Example of chemical signaling molecules:
Hormones, Neurotransmitters, Growth factors and Local mediators.

33
Chemical Signaling, I-V - Dr. Kalimi

C. Chemical nature of signaling molecules:


Proteins, Peptides, Amino acids or Amino acid derivatives, Fatty acid
derivatives (eicosanoids), Steroids, and Dissolved gases such as Nitric Oxide
D. Various chemical signaling modes:

1. Endocrine:
Endocrine gland cells synthesize and secrete steroid and peptide
hormones which travel through the blood stream to stimulate target
tissues and elicit a specialized response through the binding of a specific
protein called a receptor.
2. Paracrine:
Hormones released from the cell regulate the function of neighboring
cells without leaving the vicinity of the gland. For example, the peptide
hormone somatostatin, released from the delta cells of the pancreas,
influence secretion of insulin from beta cells.
3. Autocrine:
Signals such as prostaglandins are continuously synthesized, released, and
reuptaken. They can stimulate cells in which they themselves are
synthesized or they can stimulate neighboring cells of the same type.
4. Synaptic:
Neurotransmitters such as norepinephrine, acetylcholine etc. are released
from the nerve terminal into the synaptic cleft between nerve-nerve and
neuro-muscular junctions.

E. Chemical signaling molecules can be broadly divided into two main


groups:

1. Hydrophobic or lipid soluble


2. Hydrophilic or water soluble

Table 1: Characteristic of hydrophobic (steroid) and hydrophilic (peptide)


hormones:
Characteristic Hydrophobic Hydrophilic

Steroids, thyroid hormones, Peptides, biogenic


vitamin A amines

Binding Protein Bound to blood globulin Uncommon except for


IGF-1

Half life Long (hours to days) Short (minutes)

Receptors Intracellular (cytoplasm, nucleus) Extracellular

Mechanism of Direct Indirect through


action second messengers

34
Chemical Signaling, I-V - Dr. Kalimi

F. Local chemical mediators:

Rapidly synthesized, taken up, destroyed or immobilized by cells. Example:

• Histamine
• Growth factors (polypeptide in nature, stimulate DNA synthesis, cell
division and growth).
• Eicosanoids (prostaglandins)

II. Receptors

Receptors are cellular proteins, which bind to specific sites on molecules or


ligands and elicit specific responses in cells.

• Binding of ligand to receptors is of high affinity, reversible, saturable, and


specific.
• Receptor structure is specific and unique as discussed below and therefore
studies are being carried out to understand the relationship between the
structure and function of receptors.

A. Intracellular receptors:

Includes steroid hormone (estrogen, androgen, progesterone, aldosterone,


glucocorticoids, vitamin D) receptors, vitamin A, and thyroid hormone
receptors.

1. The binding of steroids to the cytoplasmic and/or nuclear receptors


results in a conformational change, called an activation or
transformation, subsequent dimerization allows the transformed steroid-
receptor complex to bind with the SRE (specific transcriptional enhancer
DNA sequence).
2. Binding of the activated steroid-receptor complex dimer to the SRE in the
regulatory region of the target gene promoter results in initiation of
hormone and cell specific activation or suppression of specific gene
transcription, and accumulation of mRNAs in the cytoplasm.
3. The translation of mRNAs on ribosomes results in the synthesis of a
variety of specific proteins.
4. These proteins are known to bring about various metabolic and
physiological functions.

a. In general, agonist ligands of receptors promote binding of


coactivator proteins that promote transcription initiation while
binding of antagonists promote interaction with repressor proteins
that facilitate transcription repression.
b. Steroid hormone receptors are members of a super family of
functionally and structurally related transcription factors.

35
Chemical Signaling, I-V - Dr. Kalimi

c. They are single polypeptides displaying remarkable structural


similarities, but at the same time having wide variation in ligand
sensitivity including tissue and cell specific responsiveness.
d. By DNA cloning, and the comparision of the deduced amino acid
sequence of different steroid hormone receptors, it has been observed
that these receptors have a unique modular structure possessing
three primary domains and also several sub domains or regions (Fig.
1).

Figure 1: Schematic representation of the nuclear


receptors, showing the DNA-binding domain (solid), the
steroid binding domain (hatched). The receptors contain
777 (A), 930 (B), and 595 (C), amino acids.

i. A relatively well conserved C-terminal hormone binding


domain (a ligand-dependent activation function).The amino acid
residues responsible for the interaction with heat shock protein
(HSP 90) also reside in this region. It appears that the C- terminal
domain also contains amino acid sequences responsible for the
dimerization.
ii. A central DNA binding domain is highly conserved, cysteine
rich and is folded into two zinc finger motifs. Each finger is
composed of four cysteines, which tetrahedrally encompass a zinc
ion. The first finger determines the response element specificity.
The second finger is likely involved in receptor dimerization and
protein-protein interactions.

36
Chemical Signaling, I-V - Dr. Kalimi

iii. The -NH2 terminal domain is the least conserved both in size and
sequence. For example, the progesterone receptor has 567 amino
acids, while estrogen has 185 in this region. This domain may
required for the maximum gene transcription activity (a
ligand-independent transcriptional function).

B. Extracellular or cell surface receptors:

Peptides (also biogenic amines, neurotransmitters and eicosanoids) bring


about biological responses by binding to their respective cell surface receptors
with high affinity and stereo-chemical specificity.

These receptors are mobile and leave the cell surface (endocytosis and down
regulation).

These receptors are integral membrane proteins having three main domains
(fig 2):

• extracellular
• transmembrane
• intracellular or cytoplasmic.

Figure 2

It is proposed that the binding of peptide hormones at the extracellular domain


induces conformational changes thus internalizing the signal.

Agonist hormones generate the signal transducing conformation, while


antagonists can bind to the receptor but do not activate the system.

The extracellular and transmembrane domains of the receptor help to bring


intracellular or cytoplasmic domains closer by a hormone-dependent
dimerization or oligomerization. Thus, binding of peptide hormone to the
cell membrane receptor can transmit the hormonal signal into the cell.

37
Chemical Signaling, I-V - Dr. Kalimi

These receptors can be broadly classified according to their output into three
main groups:

1. Enzyme-linked (intrinsic or associated tyrosine kinase activity).

2. Those linked to heterotrimeric G proteins.

There are two forms of G-proteins

o Monomeric G-Ras.
o Heterotrimeric G-proteins.

ƒ Heterotrimeric G-proteins are composed of alpha, beta and gamma


subunits in decreasing order of mass.
ƒ G-Ras proteins are generally composed of a single polypeptide
(monomeric) and exhibit sequence homology with the alpha subunits
of heterotrimeric G-proteins.

ƒ G-proteins are members of a super family of proteins that can exist in


two states, an active one that contains a bound GTP molecule and an
inactive one that contains GDP.

3. Those that form ion channels.

1. Receptors with intrinsic tyrosine kinase activity:

Insulin, IGF-I, EGF, and PDGF receptors are protein tyrosine kinases
(PTK) that are directly activated by receptor occupation.

These receptors have a single transmembrane domain separating an


amino terminal extracellular domain and a carboxyl-terminal cytoplasmic
domain.

The extracellular domain (amino-terminus), contains the hormone


binding sites.

The cytoplasmic, intracellular, domain contains the tyrosine kinase


activity and several phosphorylation sites. The best substrate for this
kinase is the receptor itself; that is, the receptor undergoes auto
phosphorylation.

The receptors with intrinsic tyrosine kinase activity can be subdivided into
three groups.

a. Group I:

38
Chemical Signaling, I-V - Dr. Kalimi

The receptor is a single protein (no subunits). Example,


Epidermal Growth Factor (EGF) receptor (fig. 3)

b. Group II:

The receptors in this group, Insulin (fig. 3)and Insulin like Growth
Factor (IGF-I) receptors are cleaved into two subunits.

The amino-terminal alpha subunit remains entirely extracellular


and binds to hormone, while the beta subunit, possessing
tyrosine kinase activity, crosses the membrane. Furthermore each
alpha-beta pair covalently binds with another pair to form tetramer.

Group 1. Group 2.

Figure 3

c. Group III:

The receptor is a single protein (no subunits). Example, Platelet


Derived Growth Factor (PDGF) receptor (fig. 4).

The extracellular region of the PDGF receptor consists of five


immunoglobulin-like domains that constitute the PDGF binding site.

39
Chemical Signaling, I-V - Dr. Kalimi

Group 3.

Figure 4

2. Receptors having associated tyrosine kinase activity:

Receptors for prolactin, growth hormone, erythropoietin, interferons


(IFNs) and interleukins 2, 7, 10, 13 and 15 have a homology in the
extracellular domains having four cysteine moieties, C1-C4 for disulfide
bonding and a potential site for glycosylation.

Like the members of the epidermal growth factor receptor family, these
receptors possess a single transmembrane domain (fig. 5).

Many of these receptors work through associated tyrosine kinases also


called Janus kinases (function similar to receptor tyrosine kinases
described earlier, except that their kinase domain is encoded by a separate
gene and is noncovalently associated with receptor polypeptide chain),
which phosphorylate various target proteins when the ligands binds its
receptor.

40
Chemical Signaling, I-V - Dr. Kalimi

Figure 5: Receptors linked to associated tyrosine kinase activity

3. Receptors linked to heterotrimeric G proteins:

a. Beta 1 and beta 2-adrenergic receptors activate adenylyl cyclase


b. Alpha 2-adrenergic receptor inhibits adenylyl cyclase
c. Alpha 1-adrenergic receptor activates phospholipase C

41
Chemical Signaling, I-V - Dr. Kalimi

Figure 6: Schematic presentation of receptors linked to heterotrimeric G-


proteins

This receptor is highly hydrophobic. Seven putative transmembrane


domains can be arranged based on seven sequences of hydrophobic amino
acids (fig. 6).

Rhodopsin, muscarinic and alpha 2-adrenergic receptors reveal similar


structural organizations. The NH2-terminus is extracellular while the
COOH-terminus is intracellular.

4. Receptors forming an ion channel:

There are two major receptor families in this group:

a. those that form a sodium and potassium channel (Nicotine


acetyl choline receptor)
b. those that form a chloride channel (gamma-amino butyric
acid[GABA] and glycine receptors).

The structures of these receptors are quite complex. For example, the
muscle nicotine acetyl choline receptor is composed of five subunits: 2α,
β, γ, and δ (each subunit has an extracellular domain, four or five
transmembrane domains, and a large cytoplasmic domain) (fig. 7).

Figure 7

42
Chemical Signaling, I-V - Dr. Kalimi

When the five subunits are clustered together (fig. 8), they form a tube-
like structure (pore) that penetrates the membrane. Binding of
acetylcholine causes the pore or ion channel to open within microseconds.

Figure 8: Transverse section of Nicotinic Ach


Receptor showing the arrangements of the five
subunits

III. Transduction of the signal from the cell surface into the nucleus:

The binding of peptides with high affinity to their respective cell surface receptors
results in transduction of the signal from the cell surface to the nucleus. In this
section six major signal transduction pathways will be described by which
signals can be transmitted inside the cell after the binding of ligand to receptor.

1. cAMP pathway
2. Calcium-Phospholipid pathway
3. cGMP pathway
Pathways 1,2, and 3 are linked to heterotrimeric G-proteins
4. G-Ras protein- linked pathway (linked to intrinsic tyrosine kinase)
5. JAK-STAT pathway (linked to associated tyrosine kinase or Janus kinase)
6. Cytoplasmic serine phosphorylation and or proteolysis pathways

A. cAMP as a Second Messenger (cAMP pathway):

Heterotrimeric G-protein, is involved as a coupling protein between receptor


binding to the ligand and the activation of the enzyme adenylyl cyclase.

In bacteria, receptor and adenylyl cyclase interact directly but in animal cells an
additional component (G protein) is required.

43
Chemical Signaling, I-V - Dr. Kalimi

There are two types of G-proteins involved in cAMP pathway:

• Stimulatory G protein (Gs) which activates adenylyl cyclase


leading to the production of cAMP.
• Inhibitory G protein (Gi) which inhibits adenylyl cyclase leading to
the
inhibition of cAMP production.

1. Stimulatory G protein (Gs):

In its inactive form, Gs exists as a trimer of three polypeptides chains α, β


and γ in order of decreasing mass, with GDP bound to Gsα.

Binding of ligand to the receptor results in alteration in the G-nucleotide-


binding site on Gsα, allowing GTP to bind in place of GDP (GDP-GTP
exchange).

The binding of GTP results in dissociation of Gsα from Gβγ, allowing Gsα
to bind tightly to an adenylyl cyclase molecule.

Binding of Gsα-GTP to adenylyl cyclase activates the enzyme adenylyl


cyclase which produces cyclic AMP from ATP.

The precise role of Gβγ is not known. However it has been shown that
Gβγ subunits:

a. contribute to the receptor recognition site,


b. may directly regulate effectors such as the enzyme phospholipase
A2,
c. also help anchor the α subunit to the plasma membrane.

The interaction between receptor, G-protein, and adenylyl cyclase are


consecutive and reversible.

Within a minute, Gsα hydrolyzes its bound GTP (due to intrinsic GTPase
activity), to GDP, causing Gsα to dissociate from adenylyl cyclase and
reassociate with Gβγ to restart the cycle again by achieving the basal
cellular
activity level.

It has been shown that guanosine nucleotide releasing factors (GRFs)


facilitate the enhanced dissociation of GDP, so GTP can rapidly bind to
Gα following hormonal binding to the receptor.

The GTPase activating proteins (GAPs) on the other hand, facilitate the

44
Chemical Signaling, I-V - Dr. Kalimi

hydrolysis of GTP to GDP by GTPase. The relative activities of GRFs and


GAPs on G-proteins at a given time determine the rate of G-protein
activation (this is also true for the G-Ras protein).

Gs allows (i) amplification of the signal and (ii) regulation of the


activation process between the receptor and adenylyl cyclase.

2. Synthesis of cyclic AMP from ATP by the activated plasma bound


enzyme adenylyl cyclase:

ATP → Adenylyl Cyclase → cAMP

3. Activation of cyclic AMP-dependent protein kinase (PKA or phospho


kinase A) by cyclic AMP (fig.9):

PKA (inactive) is made up of two regulatory (R2) and two catalytic (C2)
subunits.

Binding of cyclic AMP to R2 results in dissociation of R2 from C2 (Fig.9).

The released C2 (PKA active) phosphorylate various proteins and


transfer the terminal phosphate group from ATP to specific serine or
threonine residues, causing a cascade of cellular events (phosphorylation
of target enzymes, ion channels or transcription factors). For example,
serine phosphorylation of the cAMP response element-binding protein
(CREB).

These phosphorylated proteins can enter the nucleus and phosphorylate


many different resident nuclear proteins mediating final cellular responses.

Figure 9

45
Chemical Signaling, I-V - Dr. Kalimi

Once completed, the cell quickly returns to basal cellular activity level by:

a. Dephosphorylation of phosphorylated serines and threonines by


four groups of serine/threonine phosphoprotein phosphatases

Phosphoprotein + Phosphates Phospho protein + Pi

b. Metabolizing cAMP to inactive 5'AMP by one or more cyclic


AMP phosphodiesterases (PDE).

cAMP → Phosphodiesterase → 5’AMP (inactive)

Control of cAMP levels is maintained by activating or inhibiting


adenylyl cyclase rather than changing phosphodiesterase activity.

Tissue specificity resides in the substrates, which are


phosphorylated by active phospho kinase A (PKA).

4. Olfaction pathway (cAMP-linked) of signal transduction:

Olfactory receptor neurons in the lining of the nose recognize odors by


means of specific heterotrimeric G-protein-linked olfactory receptors
(G-olf). Our current understanding of the olfactory pathway is
summarized as follows:

a. ↑ odorant-receptor complex,
b. ↑ activation of olfactory specific G-olf protein,
c. ↑ adenylyl cyclase,
d. ↑ cAMP,
e. ↑ activation of a cyclic nucleotide-gated (CNG) channel in the
plasma membrane (sodium ions entering the cell through the open
channels) and depolarization of olfactory receptor neurons,
f. ↑ physiological responses.

46
Chemical Signaling, I-V - Dr. Kalimi

Figure 10

5. Clinical Aspect of Gsα:

Cholera toxin (produced by the microorganism that causes cholera, Vibrio


Cholerae) is an enzyme that catalyzes the transfer of ADP-ribose from
intracellular NAD+ to the alpha subunit of the Gs protein.

This covalent modification (binding of ADP-ribose to Gsα) locks GTP


into an unhydrolyzable state (inactivation of GTPase activity of Gsa),
keeping the adenylyl cyclase molecule activated which prolongs elevation
of cyclic AMP causing Na+, Cl- and water influx into the gut and therefore
diarrhea, dehydration and may result in death.

47
Chemical Signaling, I-V - Dr. Kalimi

a. Inhibitory G protein (Gi):

Gi contains the same Gβγ complex as Gs only the α subunit (Giα)


is different.

Binding of hormone to the receptor causes release of GDP from


Giα and association of GTP. GTP bound Giα dissociates from Gβγ
and binds to adenylyl cyclase directly inhibiting the production
of cAMP.

6. Clinical Aspect of Giα:

Pertussis toxin made by the bacterium Bordatella Pertussis that causes


whooping cough, increases cAMP levels by ADP-ribosylation of
Giα.

The ADP-ribosylated Giα complex is prevented from interacting with


receptors (uncoupling of the receptor from its G protein) and
therefore fails to inhibit adenylyl cyclase in response to receptor
activation.

B. Calcium as a Second Messenger (Calcium-Phospholipid Pathway):

1. The regulation of calcium levels in cells:

a. Free cytosolic calcium (10-7 M), extracellular calcium (10-3 M) and a


specialized intracellular calcium- sequestering compartment such as
mitochondria etc. (10-5 M).
b. Cytosolic Ca2+ can be kept low by:

i. Ca2+ ATPase pump which removes calcium out of cytosol by


using ATP as a source of energy.
ii. Na+ - Ca2+ exchanger which couples the efflux of Ca2+ to the
influx of Na+
iii. Ca2+ pump in the specialized intracellular compartment using
Ca2+ ATPase which takes up calcium from the cytosol into the
calcium-sequestering compartment.

c. Cytosolic calcium level can be raised from 10-7 M to 5x10-6 M by:

i. Voltage-gated calcium channels in electrically excitable


tissues (when the nerve terminal membrane is depolarized by
an action potential).
ii. Receptor - activated Ca2+ channels, by binding of an
extracellular signaling molecule to a cell- surface receptor (the
calcium-phospholipid pathway).

48
Chemical Signaling, I-V - Dr. Kalimi

2. The Calcium- Phospholipid Pathway (fig. 11):

a. Binding of the signaling molecule to the receptor and activation of the


membrane bound enzyme phospholipase "C" (PLCγ) through G
binding protein (another class of heterotrimeric G-proteins; designated
Gq).

The GDP-GTP exchange reaction, dissociation of Gα-GTP, binding of


Gα-GTP to phospholipase C, and hydrolysis of GαGTP by GTPase is
similar to the mechanism described for adenylyl cyclase.

b. Conversion of plasma membrane phospholipid, phosphotidyl inositol


4,5,biphosphate (PIP2) by phospholipase “C" to yield inositol 1,4,5,
triphosphate (IP3) and diacylglycerol.

PIP2 + Phospholipase C → IP3+ diacylglycerol

i. Inositol trisphosphate (IP3) releases Ca2+ from endoplasmic


reticulum storage sites.

IP3 is rapidly dephosphorylated by phosphatase or converted to 1,3,4,5


tetraphosphate (IP4) by phosphorylation.

Inositol tetraphosphate (IP4) increases membrane permeability to


extracellular Ca2+.

ii. Diacylglycerol activates the enzyme "protein kinase C” (it is


also called C-kinase, or PKC because of its calcium
dependency). In addition diacylglycerol liberates arachidonic
acid (diacylglycerol → diacylglyceride lipase → arachidonic
acid) and provides phosphatidic acid for the resynthesis of
phosphatidylinositols.

iii. Protein kinase C (PKC):

• Phosphorylates serine and threonine molecules of the


protein by transferring phosphate from ATP

Protein + ATP+ PKC → Phosphoprotein + ADP

• Increases ion channels in the brain


• Increases gene transcription by phosphorylation of gene
regulatory proteins directly or indirectly by cascade.

49
Chemical Signaling, I-V - Dr. Kalimi

ƒ The effects of IP3 can be mimicked by calcium


ionophore, A23187 or ionomycin and the
effects of diacylglycerol can be mimicked by
phorbol esters, which are plant products that
bind to PKC and activate it directly.

Figure 11

3. Calmodulin (Calcium binding protein):

Calmodulin:- A multifunctional Ca2+-binding regulatory protein


(ubiquitous in plant and animal) involved in enzyme regulation.

Calmodulin is a highly conserved, single polypeptide chain protein having


four high-affinity Ca2+ binding sites.

Calmodulin undergoes conformational changes upon binding to calcium


(Fig.12). This results in the binding of calmodulin to various target
proteins in the cell, therby altering their activity. For example, activation
of Ca-M-kinases or calcium/calmodulin-dependent protein kinases
(Ca2+/CaM-PK). These kinases phosphorylate serine and threonine
residues on proteins.

Tissue specificity of calmodulin depends on which calcium-regulated


pathways are present in the given cells.

Calmodulin has no direct enzymatic activity but activates other proteins


by binding to them.

50
Chemical Signaling, I-V - Dr. Kalimi

Figure 12

4. Interaction between calcium and cAMP pathways:

cAMP and calcium as allosteric effectors:

a. Calcium and cyclic AMP levels can influence each other. For example,
calcium-calmodulin binds to and regulates enzymes that breakdown
and synthesize cyclic AMP, cyclic AMP phosphodiesterase and
adenylyl cyclase respectively. A-kinase can phosphorylate some Ca2+
channels and pumps
and change their activity.
b. Some CaM-kinases are phosphorylated by A-kinase
c. A-kinase and CaM-kinases frequently phosphorylate different sites on
the same protein, and these proteins can be regulated by both cyclic
AMP and Ca2+.

C. Cyclic GMP as an intracellular messenger (cGMP pathway):

1. Atrial Natriuretic Peptides (ANP) pathway (direct linkage between


ligand binding and activation of guanylyl cyclases, no G- protein
involvement):

Atrial natriuretic peptides (ANPs) are secreted by muscle cells in the atrium of
heart when blood pressure rises. ANPs increase Na+ and water excretion and
enhance relaxation of smooth muscle cells of blood vessels, thus lowering
blood pressure. The following steps are involved in the signal transduction by
ANPs.

Binding of ANP to the extracellular domain of the receptor activates guanylyl


cyclase on the intracellular catalytical domain.

51
Chemical Signaling, I-V - Dr. Kalimi

a. ↑ guanylyl cyclase,
b. ↑ cGMP ,
c. ↑ G-kinase,
d. ↑ phosphorylation of serine or threonine residues,
e. ↑ biological responses.

2. Direct regulation of Na+ channels in retinal rod cells by cGMP


(signal transduction without phosphokinases):

a. In absence of light Na+-channels are open.


b. On exposure to light, rhodopsin on the membrane of rod cells is
activated which in turn activates the G protein (Gt) transducin.
c. On activation of transducin there is a dissociation of the alpha
subunit from Gt, which activates cyclic GMP phosphodiesterase by
binding to it.
d. Activated cyclic GMP phosphodiesterase hydrolyses channel bound
cGMP and closes Na+ channels. In this way a light signal can be
converted into an electrical signal.

3. Nitric oxide (NO) pathway of signal transduction: The small molecules


of gaseous nitric oxide can pass readily across the target cell plasma
membrane and bring about diverse physiological responses such as
relaxation of smooth muscle cells, nerve transmission, dilation of blood
vessels (penile erection) and activation of immune cells.

It has been shown that nitric oxide is made by the enzyme nitric oxide
synthase by the deamination of the amino acid arginine. For example,

a. Smooth muscle cells + nicotine or acetylcholine → L-Argentine +


Nitric Oxide Synthase (NOS) → L-Citrulline + ↑ NO.
b. NO by diffusion along the cell membrane activates enzyme
guanylyl cyclase and the intracellular production of cyclic
GMP.
c. ↑ cGMP → ↑ cGMP kinase → ↑ phosphorylation of proteins →
↑ cellular responses.

NO is inactivated by the formation of nitrates and nitrites, and cyclic GMP


is converted to inactive 5'GMP by the enzyme phosphodiesterase (cGMP
+ phosphodiesterase = 5’GMP ) and the cycle is repeated.

D. G-Ras Protein-Linked Pathway (fig. 13):

Growth factors such as EGF and PDGF, insulin and IGF-I send messages
from the cell membrane to the cell interior via the "Ras pathway".

52
Chemical Signaling, I-V - Dr. Kalimi

1. The binding of ligand to their receptors results in the dimerization and


autophosphorylation of the receptor on multiple tyrosine molecules.
2. These phosphotyrosine residues, in turn, allow the activated receptors
to bind to the cytoplasmic protein Grb-2 (growth receptor binding
protein) through its SH2 domain.
3. The SH2 domain has no catalytic activity but specifically recognizes
phosphotyrosine residues (adaptor protein). Grb-2 interacts through its
SH3 domain with SOS (son of sevenless protein), which activates G-
Ras by GDP for GTP (GDP-GTP exchange).
4. Activated G-Ras protein (GTP bound) acts on a series of protein
kinases including MAPKs (mitogen-activated protein kinases) and the
transcription factors c myc, Elk-1 etc. In this way, by a series of
phosphorylation-dephosphorylation (both on tyrosine and threonine
residues) reactions, growth factor signals transmit from the cell
membrane into the cytoplasm and then into the nucleus.

Figure 13: Summary of G-Ras Pathway

E. JAK-STAT (Signal Transduction and Activator of Transcription)


Pathway (fig. 14):

Receptors for prolactin, growth hormone, erythropoietin, etc. send


messages from the cell membrane to their cytosolic and nuclear targets via the
JAK-STAT pathway.

53
Chemical Signaling, I-V - Dr. Kalimi

The Janus kinases are activated upon ligand-induced receptor homo-or-


heterodimerization. The janus kinases activated by tyrosine kinase
phosphorylation, in turn phosphorylates one or more STATs which then
translocate to the cell nucleus to direct transcriptional responses.

Figure 14: Summary of JAK-STAT Pathway

F. Cytoplasmic serine phosphorylation and/or proteolysis pathways:

The SMAD, NFkB, Wnt, CI/Gli, Tubby and Notch signaling pathways
also deliver an active transcription factor to the nucleus following cell surface
receptor binding by an extracellular protein ligand. This is followed by the
phosphorylation and proteolysis steps.

54
Chemical Signaling, I-V - Dr. Kalimi

The concept of cross talk at a number of steps in the signaling cascade is


emerging as the accepted biological response of various chemical signaling.
For example, it is quite likely that other signal pathways such as G-Ras
protein, phospholipase C, phospholipase A2, and calcium channels may also
be activated or inhibited by a chemical signal working through the cAMP
pathway.

Concluding Remarks:

By using powerful molecular biological techniques including the cloning of


peptide receptor genes, site-directed mutagenesis, development of chimeric
receptors through insertion and deletion of relevant DNA sequences and site-
directed antibodies, we have begun to learn the structure-function relationship
of cell surface receptors involved in the signal transduction. Future X-ray
crystallographic studies using crystallized receptors will further enhance our
understanding of receptor function.

Clinical Aspects:

An Increasing number of human diseases are known to involve mutations,


over expression, or malfunctioning of protein kinases and phosphatases, and
their regulators and effectors. At least 18 phospho tyrosine kinase (PTK)
genes have been identified as oncogenes. Many genetic diseases including
Coffin-Lowry syndrome and myotonic dystrophy result from mutations in
protein-serine kinases and phosphatases.

The widespread involvement of protein kinases and phosphatases in disease


has led to a massive effort to develop drugs that either activate or more
usually inhibit individual protein kinases or phosphatases. Drugs designed to
target these enzymes fall into two categories-monoclonal antibodies or
modified protein ligands, and small molecules.

IV. Amplification Responses

Hormones generally circulate at concentrations from 10-9 to 10-12 M. They


produce much larger changes in a variety of biological parameters as a result of
signal amplification, in which the rather weak hormonal signal is amplified in to a
larger biological response.

The signal is amplified when transmits from cell surface to the inside of the cell.
For example, a single receptor-hormone complex can activate multiple G-
proteins, which can in turn activate many fold adenylyl cyclase and thus generate
copious amounts of cAMP. Thus
10-9 moles of hormone can generate as much as 10-6 moles of cAMP.

55
Chemical Signaling, I-V - Dr. Kalimi

Signal amplification requires rapid synthesis and degradation i.e. fast turnover of
cAMP, calcium and other intermediates involved in second messenger mediated
pathways.

To maintain proper feedback control during transmission of the signal, a balance


between enzymes kinases (serine/threonines kinases or tyrosine kinases), which
phosphorylate proteins and phosphatases, which deactivate phosphoproteins by
dephosphorylation, must be simultaneously maintained.

Signal amplification is achieved by:

1. Linear graded responses: Example, steroid hormone action. Response is


linear to increased ligand concentration, formation of ligand-receptor
complexes and binding of ligand-receptor complex to specific DNA sites.
2. All-or-none responses: Undetectable below certain threshold
concentration and then reaching maximum as soon as this concentration is
exceeded. Example, activation of phosphokinase A by cAMP and
calmodulin by calcium. Two or more calcium ions must bind before
calmodulin adopts its activating confirmation. Thus fifty-fold calmodulin
activation occurs when the free intracellular calcium concentration
increases only tenfold.
3. Simultaneous activation and inhibition of enzymes: Example, activation
of intracellular cAMP levels promotes breakdown of glycogen in skeletal
muscle cells by simultaneous activation of phosphorylase kinase and
inhibition of phosphoprotein phosphatase by cAMP.
4. Positive feedback: Example, activation of acetylcholine receptors at the
neuromuscular junction results in net influx of Na+ by increased
depolarization of nerve or muscle membrane. This in turn increases further
Na+ influx.

Adaptation to Stimuli or Desensitization (loss of signal responsiveness on


prolonged exposure of cells to ligands)(Fig. 15):

Peptide receptors are mobile in nature and can leave the cell surface and
internalize (endocytosis).

The ligand-bound receptor initiates at least two processes:

One leads to the generation of second messengers that begin with either the
activation of a G-protein or a tyrosine kinase as described above.

The second process is desensitization or loss of signal responsiveness upon


prolonged exposure of the cell to hormone through receptor modification.

56
Chemical Signaling, I-V - Dr. Kalimi

The internalization process plays an important role in hormone degradation and in


maintaining the concentration of the receptor on the cell surface.

Since internalization may help explain the loss of the receptor resulting in
hormonal resistant state under prolonged exposure to hormones, the
desensitization and down regulation of peptide receptors through internalization is
of great clinical interest.

1. Loss of specific cell surface receptors or down regulation: The binding


of ligand to the receptor decreases the half-life of the receptor. For
example, EGF receptor has a half-life of 10 hours in the absence of EGF,
however, when exposed to EGF, it has a half-life of one hour.
2. Receptor sequestering: no degradation by lysosomes. Only endocytosis
and recycling back to cell surface. (Fig. 15)
3. Inactivation of receptors by reversible phosphorylation: homologous
desensitization, affecting a single receptor signal transduction pathway.
For example, phosphorylation of ß2 -adrenergic receptor by activated
cyclic AMP interferes with the receptor's ability to activate Gs. (Fig. 15)
4. Alteration in G proteins involved in signal transduction: heterologous
desensitization, affecting many signal pathways which are coupled to
those particular G-proteins. (Fig. 15)

57
Chemical Signaling, I-V - Dr. Kalimi

Figure 15.

V. Chemical Signaling: Review and Summary

A. Steroid hormone action: Binding of steroid to the nuclear receptor and


dimerization of the hormone-receptor complex, transcription (mRNA synthesis),
translation (enzyme or protein synthesis), and physiological responses.

58
Chemical Signaling, I-V - Dr. Kalimi

B. Steroid receptor structure: Three distinct domains, (i) C-terminal domain


steroid binding site, hsp 90 binding site and receptor dimerization site, (ii) central
domain- cysteine rich DNA binding site having two zinc fingers. Finger I-
response element specificity and finger II- receptor dimerization and (iii) N-
terminal domain- transactivation site.

C. Structure of membrane receptors: Common features- N-terminal extracellular


domain, transmembrane domain and C-terminal cytoplasmic domain.

• Single transmembrane domain (intrinsic or associated tyrosine kinase -linked


receptors)

1. Intrinsic tyrosine kinase-linked receptors:

• Group I: EGF receptor- Single peptide.


• Group II: Insulin and IGF-I receptors: α (hormone binding site) and β
subunits (intrinsic tyrosine phosphokinase activity).
• Group III: PDGF receptor: Extracellular domain consists of five
immunoglobulin like domains.
• Heterotrimeric G-Protein-linked receptors: Seven transmembrane
domains α1,α2, β1,β2 , Rhodopsin, Muscarinic receptors.
• Ion channel-linked (glycine, GABA, nicotine ACh )receptors: Four or
five transmembrane domains

E. Summary of cAMP mechanism: ↑ Receptor occupation, ↑ GDP-GTP exchange


on the α subunit of the G protein (G-protein is made up of α, β and γ subunits),
dissociation of GTP-bound α subunit from βγ subunits and association with
adenylyl cyclase, ↑ activation of adenylyl cyclase (conversion of Gα -GTP to
GDP by intrinsic GTPase and reassociation with G βγ to restart the cycle), ↑
conversion of ATP to cAMP, ↑ cAMP, ↑ activation of PKA (cAMP+R2+C2 →
cAMPR2 + active kinase C2).

F. Summary of calcium-phospholipid mechanism: ↑ Activation of Phospholipase


C, ↑ conversion of PIP2 to DAG and IP3. ↑ IP3, ↑ intracellular calcium , ↑DAG, ↑
PKC.

VI. Practice Questions

1. G-Ras protein is associated with all of the following EXCEPT:

A. Intrinsic GTPase activity


B. GDP-GTP exchange activity
C. Alpha, beta, gamma subunits
D. Insulin action
E. Tyrosine kinase-linked receptors

59
Chemical Signaling, I-V - Dr. Kalimi

2. Select one correct statement:

A. The binding of cAMP to the regulatory subunit activates cAMP


dependent protein kinase
B. Peptide hormones have very long half-life in the blood
C. Receptor down regulation is involved in the signal amplification
D. Positive feedback is involved in the signal desensitization
E. Calmodulin possesses intrinsic enzymatic activity

3. The JAK-STAT pathway of signal transduction is associated with:

A. Intrinsic tyrosine kinase activity


B. Receptors having a seven transmembrane domains
C. Activation of phospholipase ‘C’
D. Janus kinases
E. Nitric oxide pathway

4. Nicotine acetyl choline receptor:

A. Is made up of five subunits


B. Possesses ligand binding sites on beta subunits
C. Has single transmembrane domain
D. Is responsible for opening the chloride channels
E. Has associated tyrosine kinase activity

5. Select one letter heading for each numbered word or phrase.

A. The receptor linked to heterotrimeric G proteins


B. Guanosine nucleotide releasing factors(GRFs)
C. Cholera toxin
D. G alpha subunit
E. Adenylyl cyclase

6. Generally possesses a seven transmembrane domains


7. Converts ATP to cAMP
8. Possesses intrinsic GTPase activity

Answer Key:

1. C 2. A 3.D 4. A 5. 1 A, 2 E, 3. D

60
Membrane Transport I - Dr. Costanzo

Membrane Transport I
Linda S. Costanzo, PhD.

OBJECTIVES:

1. Understand how a concentration gradient gives a direction to diffusion.


2. Give an equation that describes how the diffusion coefficient depends on
temperature, molecular radius, and viscosity.
3. Understand the concept of partition coefficient and how it relates the
concentration changes at lipid bilayer/aqueous interfaces.
4. Write and understand the equation for simple diffusion of non-electrolytes across
biological membranes.
5. Relate the permeability coefficient to the diffusion coefficient, partition
coefficient, and membrane thickness.
6. Describe the process of simple diffusion of electrolytes and understand how the
relative permeability of each ion dictates the overall permeability of the salt.
7. Describe the features of facilitated diffusion and compare it to simple diffusion.
8. Describe the features of primary active transport, know the three major examples,
and understand the function of the Na+-K+ ATPase at the molecular level,
including the action of cardiac glycosides.
9. Describe the features of secondary active transport, including cotransport and
countertransport. Know major examples of co- and countertransport.

Suggested Reading:

Physiology, edited by: R.M. Berne and M. N. Levy, Mosby, 6th Ed. pp 7-15.
Physiology, L.S. Costanzo, Saunders, pp 6-12

I. SIMPLE DIFFUSION OF NONELECTROLYTES

A. Diffusion arises from the random thermal motion of molecules (also


called Brownian motion). Because it is random, the movement of any one
molecule has no preferred direction in space. However, if a concentration
gradient exists, then over time there will be a net flux of molecules from
the region with the higher concentration toward the region with the lower
concentration until the gradient is gone (dissipated). This can be seen from
the figure below.

61
Membrane Transport I - Dr. Costanzo

Figure 1.

Chamber A on the left has a higher concentration of molecules. Thus, on


the average, more molecules will move from A to B than will move from
B to A; i.e., there will be net movement of molecules from A to B. The
rate at which this net flux occurs will depend on the permeability of the
barrier to the molecules, the surface area available, and the size of the
concentration gradient (Fick’s first law of diffusion).

B. Importance of diffusion – Over short distances (< 100 :m), diffusion in


an aqueous medium can be an efficient means of transport. For example,
small molecules can diffuse 100 :m in about 5 sec. That is just about the
limit that cells can tolerate to be replenished with nutrients and oxygen.
Normally (and conveniently), no cell in the body is more than 100 :m
from the nearest open capillary. For another example, diffusion across a
single small cell might require only 50 msec – that’s do-able. On the other
hand diffusion of a neurotransmitter from the cell body of a neuron, where
it is produced, to its axon terminal 1 meter away, where it is released,
would take aoubt 15 years. (Therefore large cells must contain specialized
molecular motors associated with their cytoskeletons to rapidly move
substances long distances through the cell.)

C. Diffusion Coefficient (D) is a characteristic that determines how fast a


solute will diffuse. The diffusion coefficient of a small molecule is
inversely proportional to its molecular weight. Also D is proportional to
temperature, T, in degrees K (not surprising since diffusion is due to
thermal motion), inversely proportional to the mean radius of the molecule
(r), and inversely proportional to the viscosity of the medium (η). This is
summarized by the Stokes-Einstein equation as:

62
Membrane Transport I - Dr. Costanzo

where k is Boltzmann’s constant (1.38 x 10-16 erg/molecule oK). At 37o C


(310o K) a 1 nm radius molecule (10-7 cm) in water (η = 0.00895 poise)
has a D = 2.5 x 10-6 cm2/sec.

D. Partition coefficient (K), or oil/water partition coefficient is the solubility


of a solute in oil (or lipid) relative to its solubility in water. Thus:

K = concentration in oil phase/concentration in water phase

If K = 1, the solute has no preference for oil or water; if K > 1, the solute
is hydrophobic, non-polar, and prefers oil; if K < 1, the solute is
hydrophilic, polar, and prefers water.

The higher the partition coefficient, the greater the oil solubility of the
solute, and the more readily it will dissolve in the lipid bilayer of cell
membranes and be transported by simple diffusion. Thus, small molecules
with relatively high lipid solubility tend to be more permeable across lipid
membranes; water-soluble, electrolyte solutes tend to be less permeable
across lipid membranes. One way to think about this is that the more
permeable molecules enter the lipid phase readily at each aqueous/lipid
interface. An equilibrium is set up at each interface so that:

where cbilayer is the concentration of the substance in the lipid bilayer


phase, caqueous is the concentration of the substance in the aqueous phase,
and K is the partition coefficient.

63
Membrane Transport I - Dr. Costanzo

Figure 2.

Figure 2 illustrates the effect of the partition coefficient on the flux of a


substance across a bilayer membrane. The membrane separates aqueous
solutions A and B. On the x axis, the membrane occupies the space
between units 4 and 6; compartments A and B lie between 2-4 and 8-10 on
the x-axis, respectively. In compartment A, the solute concentration is 2
(units don’t matter for this example) and in compartment B, the solute
concentration is 1. If the partition coefficient (K) = 1, then the
concentration of the solute just inside the lipid bilayer on side A will be 2,
on side B it will be 1, and the concentration difference across the
membrane will be 2-1 = 1. If K = 5, the concentration just inside the
bilayer on side A will be 10, on side B it will be 5, and the concentration
difference is 10-5 = 5. Take home message: a higher partition coefficient
amplifies the concentration difference inside the lipid bilayer and also will
amplify (increase) the flux by simple diffusion across the membrane.

E. Membrane thickness and membrane area. Intuitively, we know


that transport by simple diffusion will be proportional to available
surface area (A) and inversely proportional to membrane thickness (h,
or ∆x).

F. Fick’s first law of diffusion. Putting all of this information together,


we arrive at the diffusion equation (or Fick’s first law of diffusion):

64
Membrane Transport I - Dr. Costanzo

Here, J is the flux or flow (mmoles/sec), P is the permeability coefficient


(cm/sec), A is the surface area (cm2), and cA and cB are the concentrations
of the solute in compartments A and B, respectively (mmoles/cm3, where
cm3 = 1 ml). When cA > cB , the net flux is from A to B, and J is a positive
B

value.

The permeability coefficient, P, is directly proportional to K and D and


inversely proportional to the membrane thickness, h (or ∆x in some
books). We can measure P by rearranging Fick’s first law: by measuring
the flux across membrane, the concentration difference, and the surface
area, we can solve for P.

II. SIMPLE DIFFUSION OF ELECTROLYTES – Consider 2 compartments A and


B, which are separated by separated by a membrane.

Figure 3

65
Membrane Transport I - Dr. Costanzo

Compartment A contains a uni-univalent salt (i.e., the cation has valance +1 and the
anion has valance -1) at concentration, cA, which initially is 100 mM. Compartment B
contains the same salt at a lower concentration, cB, which initially is 10 mM. The salt is
completely dissociated into the cation and the anion. Both the cation and anion will try to
diffuse from A to B, each down their concentration gradient. However, electrical forces
will prevent them from diffusing completely independently. If the cation is more
permeable than the anion, the cations will diffuse ahead of the anions. But this lead is
self-limiting. The slightly-leading cations will create a positive potential in compartment
B. That positive potential on side B will slow further diffusion of the cations and speed
up further diffusion of the anions. Finally, a steady state will occur in which the fluxes of
the cations and anions are exactly equal. Since each compartment was electroneutral to
start with, and the fluxes of cations and anions are equal, the compartments remain
macroscopically electroneutral as ions are transferred from A to B. So:

where J+ and J- are the fluxess of cations and anions, respectively, and J is the flux of the
salt. The diffusion equation for the salt has the form of that for nonelectrolytes, i.e.,:

But the permeability coefficient, P, is an average of the individual ion permeability


coefficients, P+ and P-, i.e.,:

Note that the permeability of the salt is limited by its lesser permeable ion. Below is a
plot of cA and cB as they change in time from their starting points of 100 mM and 10 mM,
B

respectively. Eventually, each compartment will equilibrate to a concentration of 55 mM


(half-way between 100 and 10 mM). However, the time this equilibration takes will be
different, depending on the relative permeabilities of the ions. Two cases are shown, one
where P+/P- = 2 (P+ is twice as large as P-) and another where P+/P- = 1000 (P+ is 1000
times P-). In the first case, the concentrations equilibrate faster because the salt can
diffuse faster.

66
Membrane Transport I - Dr. Costanzo

Figure 4.

An important limiting case is when the membrane is permselective (it allows one ion of
the salt to cross, but is impermeable to the other ion). A cation permselective membrane
has P- = 0. That means that J- = 0. According to macroscopic electroneutrality, then, J+
must also be zero. As positive ions try to move from A to B, they create a positive
potential at the membrane in B relative to A. Since anions are impermeable, they cannot
diffuse along with the cations to reduce this potential, so the system rapidly reaches an
equilibrium where the electrical forces on the cations exactly balance the chemical
diffusion forces on the cations. (More on diffusion potentials and equilibrium potentials
in the Dr. Logothetis’ lectures.)

III. Facilitated diffusion

Facilitated diffusion (like simple diffusion) always proceeds down an


electrochemical gradient and, therefore, requires no input of metabolic energy.
Unlike simple diffusion, however, it is carrier-mediated and thus exhibits the
properties of saturation, stereospecificity, and competition. The carrier has
affinity for the solute on both sides of the membrane and can shuttle the solute in
either direction, depending on the prevailing gradient. At low solute
concentrations, facilitated diffusion typically proceeds faster than simple diffusion
because of the function of the carrier (it is facilitating!). At higher concentrations,
the rate of facilitated diffusion levels off because binding sites are saturated.
(Simple diffusion, in contrast, continues as long as there is an electrochemical
gradient.)

IV. Primary active transport

Active transport involves the movement of one or more solutes against an


electrochemical gradient, or uphill. If the solute is a non-electrolyte, we say it is
transported from low concentration to high concentration. If the solute is an
electrolyte, we say it is transported from low electrochemical potential to high
electrochemical potential.

67
Membrane Transport I - Dr. Costanzo

For primary active transport, energy is supplied to the transporter directly in the
form of ATP. In the process, ATP is hydrolyzed to ADP and Pi, releasing energy
from the terminal phosphate of ATP. This high energy phosphate is transferred to
the transport protein (e.g., Na+-K+ ATPase), initiating a cycle of phosphorylation
and dephosphorylation of the transporter.

There are three major examples of primary active transport: Na+-K+ ATPase,
Ca2+ ATPase, and H+-K+ ATPase.

A. Na-K+ ATPase is present in all cell membranes. It is the transport process


responsible for maintaining a low intracellular Na+ concentration and a
high intracellular K+ concentration. Both Na+ and K+ ions move against
an electrochemical gradient.

The figure below illustrates the functioning of the Na+-K+ ATPase or


pump, and the role of ATP. Note that the stoichiometry is 3Na+ for 2K+,
so the pump is electrogenic (in this case, more + charge is pumped out of
the cell than into the cell, creating a small negative potential difference
inside the cell membrane.

Cardiac glycosides such as ouabain and digitalis inhibit the Na+-K+


ATPase by binding near the K+ binding site on the extracellular side of the
transporter, thus preventing conversion of E-P (phosphorylated enzyme or
transporter) back to E (unphosphorylated enzyme). Thus, the Na+-K+
ATPase becomes "stuck" in it high energy form, and cycling and further
transport is prevented. What would you expect to happen to intracellular
concentrations of Na+ and K+ after treatment with digitalis?

68
Membrane Transport I - Dr. Costanzo

B. Ca2+ ATPase is found in membranes of the sarcoplasmic reticulum


(SERCA) and mitochondria, and in some cell membranes. For example,
in sarcoplasmic reticulum (SR), the Ca2+ ATPase is responsible for
pumping Ca2+ into the interior of the SR, helping to maintain a high stored
Ca2+ content. Between one and two Ca2+ are transported for each ATP
hydrolyzed, although the exact stoichiometry can vary.

C. H+-K+ ATPase is found in gastric parietal cells and in intercalated cells of


the renal distal tubule. In the stomach, the H+-K+ ATPase is responsible
for pumping H+ into the lumen. Thus, omeprazole, an inhibitor of the H+-
K+ ATPase is used to reduce gastric H+ secretion and treat ulcer disease.

IV. Secondary active transport

Secondary active transport is transport of one or more solutes against an


electrochemical gradient, coupled to the transport of another solute down an
electrochemical gradient. In almost all examples known so far, the "downhill"
solute is Na+. Energy for the uphill movement of solutes does not come directly
from ATP; rather, it comes from the downhill movement of Na+. The designation
"secondary active" comes from the fact that the Na+ gradient (which is providing
the energy for the uphill transport of other solutes) is maintained by the Na+-K+
ATPase (which uses ATP for its direct energy source). Secondary active
transport is inhibited by inhibitors of the Na+-K+ ATPase (e.g., digitalis or
ouabain) since they cause the intracellular Na+ concentration to increase, thus
abolishing the Na+ gradient that normally exists across cell membranes.

There are two types of secondary active transport, cotransport and


countertransport, which are distinguished by the direction of solute movement
across the cell membrane.

A. Cotransport (symport)

In cotransport, all solutes move in the same direction across the cell
membrane. In other words, the "uphill" solutes move in the same
direction as Na+, or into the cell. Cotransport is exemplified by Na+-
glucose cotransport (see figure) and Na+-amino acid cotransport (both
are present in renal proximal tubule and small intestine), and Na+-K+-2Cl-
cotransport in the renal thick ascending limb.

69
Membrane Transport I - Dr. Costanzo

B. Countertransport (antiport or exchange)

In countertransport, the "uphill" solute moves in the opposite direction to


Na+ across the cell membrane. Countertransport is exemplified by Ca2+-
Na+ exchange (present in many cell membranes) and Na+-H+ exchange
(present in the renal proximal tubule.

70
Membrane Transport I - Dr. Costanzo

In addition to the Na+-dependent exchangers (cation exchangers)


described above, anions can also be exchanged across cell membranes.
For example, if intracellular HCO3- level becomes high and the cell
becomes alkaline, a Cl--HCO3- exchanger will be activated, which causes
the extrusion of HCO3- from the cells in exchange for Cl-; in this way,
intracellular pH is restored.

IV. PRACTICE PROBLEMS

1. The diffusion coefficients of type 1 and type 2 molecules are 1 x 10-5


cm2/sec and 2 x 10-6 cm2/sec respectively. All other things being equal,
this implies that the ratio of the radius of type 2 molecules compared with
type 1 molecules is:

A. 0.2
B. 10
C. 1
D. 5

71
Membrane Transport I - Dr. Costanzo

E. 0.1

2. The oil/water partition coefficients for urea and ethanol are 10-4 and 10-3
respectively. Assuming that in a given membrane their diffusion
coefficients are equal, the ratio of the permeability coefficient for urea
compared to that of ethanol is:

A. 0.001
B. 0.0001
C. 0.1
D. 10
E. 100

3. Choose the maneuver that results in an increase in the permeability


coefficient of a substance with respect to a given membrane:

A. decrease in the oil/water partition coefficient


B. decrease in the diffusion coefficient
C. decrease in the membrane thickness
D. increase in the concentration difference
E. decrease in membrane area

4. Facilitated transport of a substance is characterized by:

A. a linear relation between the flow and the concentration of the


substance
B. the flow of the substance against its gradient
C. a mechanism with a high degree of specificity toward the
substance
D. a requirement for ATP hydrolysis
E. the absence of a concentration gradient for the substance

5. During the cycle that characterizes the Na+-K+-Pump

A. Na+ binds to the membrane pump protein on the outside


B. K+ binds to the membrane pump protein on the inside
C. Mg2+ is a cofactor in the activation of the ATPase
D. voltage-gated Na+ channels are activated
E. ouabain enhances the transport of Na+

6. An example of an electrogenic transport process is:

A. the Na+-H+ exchanger


B. the Na+, K+, 2Cl- cotransporter
C. the Cl-- HCO3- exchanger

72
Membrane Transport I - Dr. Costanzo

D. Na+-glucose cotransporter
E. the H+-K+ ATPase

7. A salt solution is placed in compartment A and pure water is placed in


compartment B. Over time, no salt is detected in compartment B. You would
conclude that:
A. both ions are impermeable
B. the water compartment is electrically positive
C. at least one of the ions is impermeable
D. the final salt concentration in compartment B will be a weighted
average of the original salt concentration
E. macroscopic electroneutrality is violated

ANSWERS

1. D. (r2/r1 = D1/D2, so r2 /r1 = (1 x 10-5)/(2 x 10-6) = 5)


2. C. (Purea/Pethanol = Kurea/Kethanol = 10-4/10-3 = 0.1)
3. C. (P is inversely proportional to membrane thickness)
4. C. (the specificity is in the binding reaction between the ligand and the integral
protein that acts as the transporter)
5. C.
6. D. (both Na+ and glucose enter the cell, but glucose is electroneutral, so the net
effect is the transfer of positive charge into the cell, so the cell membrane
potential is depolarized by the Na+-glucose cotransporter)
7. C. For this scenario to occur, only one of the ions needs to be impermeable.
Macroscopic electroneutrality will prevent the permeant ion from moving down
its concentration gradient.

73
Membrane Transport 2 - Dr. Costanzo

Membrane Transport II (Osmosis)


Linda S. Costanzo, Ph.D.

OBJECTIVES:

1. Be able to define and calculate osmolarity


2. Describe osmosis across a semipermeable membrane and the volume changes
that will occur as a result.
3. Be able to calculate osmotic pressure and understand the factors that
contribute to it.
4. Understand the role of reflection coefficient in determining effective osmotic
pressure.
5. Understand how a difference in effective osmotic pressure drives osmotic
water flow.
6. Compare osmosis to diffusion of water.

Suggested Reading:

Physiology, edited by: R.M. Berne and M. N. Levy, Mosby, 6th Ed. pp 15-
18.
Physiology, L.S. Costanzo, Saunders, pp 12-15

Most water flow across cell membranes occurs by osmosis, not by diffusion. Osmosis is
the topic of today's lecture, but first we will clarify the concepts of osmolarity and
osmotic pressure.

I. OSMOLARITY

Osmolarity is the concentration of osmotically active particles in a single solution.


To calculate osmolarity, one must know the concentration of the solute and
whether that solute dissociates in solution. For example, urea does not dissociate
in solution; one mole of urea is one mole in solution. However, in solution NaCl
dissociates into two particles (or nearly two particles) and CaCl2 dissociates into
three particles (or nearly three). The symbol "g" gives the number of particles in
solution for a given solute. "g" also takes into account whether there is complete
dissociation or not; for example, "g" for NaCl is not 2.0, but is slightly less than
2.0 (e.g., 1.85) because in solution some of the Na+ and Cl- interact with each
other. Thus, g for urea is 1.0; g for NaCl is slightly less than 2.0 as explained; g
for CaCl2 is slightly less than 3.0. The units of "g" are osmoles/mole. The units
of osmolarity are osmoles/L or mosmoles/L.

Osmolarity = g x C

74
Membrane Transport 2 - Dr. Costanzo

If two solutions have the same calculated osmolarity, they are called isosmotic. If
two solutions have different calculated osmolarities, the solution with the higher
osmolarity is hyperosmotic and the solution with the lower osmolarity is
hyposmotic.

Osmolarity is calculated for one solution. There is no membrane separating one


solution from another and no water flow (that's osmotic pressure and osmosis...
coming up!).

II. OSMOTIC PRESSURE AND OSMOSIS

A. Example of osmosis

Osmosis is the flow of water across a semipermeable membrane due to a


difference in solute concentration. This difference in solute concentration
creates an osmotic pressure difference and the osmotic pressure difference is
the driving force for water flow. The figure below illustrates the concept of
osmosis. In both A and B, there is a semipermeable membrane separating two
solutions, one containing solute (the circles), the other containing only water.
The membrane is impermeable to the solute, but permeable to water.

• In A, the two solutions are open to the atmosphere. The solute in


Solution 1 creates an osmotic pressure, which produces a reduction in
hydrostatic pressure in Solution 1 (the higher the osmotic pressure, the
lower the hydrostatic pressure). As a result, there is a hydrostatic pressure
gradient established between Solution 2 and Solution 1, with the greater
hydrostatic pressure in Solution 2. Water flows from 2 to 1, driven by this
hydrostatic pressure gradient. With time, the volume in 1 increases and
the volume in 2 decreases.

• In B, the two solutions are closed and pressure is applied with a piston
to stop the flow of water from 2 to 1. The pressure required to stop water
flow is the osmotic pressure of Solution 1. This pressure can be measured
experimentally.

75
Membrane Transport 2 - Dr. Costanzo

B. Calculating osmotic pressure


The osmotic pressure (B) of Solution 1 in the above example is determined by
two factors: the concentration of osmotically active particles in the solution and
whether the membrane is impermeable to the solute (i.e., whether the solute
remains in Solution 1). Effective osmotic pressure is calculated by the van't Hoff
equation, which converts solute concentration into pressure, taking into account
whether the solute remains in the original compartment:

B = g C F RT

where B is osmotic pressure (or effective osmotic pressure), g is number of solute


particles per mole in solution, C is concentration of solute, RT is gas constant x
absolute temperature, and F is a new term called reflection coefficient.

C. Reflection coefficient

The reflection coefficient (F) is a dimensionless (no units) number that describes
the ease with which the solute crosses the membrane. (Yes, reflection coefficient
is related to permeability, in case you were wondering.) Values for reflection
coefficient vary between 1.0 and zero as follows:

76
Membrane Transport 2 - Dr. Costanzo

• F = 1.0 means the membrane is impermeable to the solute and the solute
stays in the original compartment. Referring to the figure above, solute
would remain in Solution 1. In this case, the solute exerts its full osmotic
effect, the osmotic pressure is maximal, and water flow is maximal.
Examples of solutes with F = 1.0 are proteins, which are impermeable
across cell membranes and capillaries.

• F = 0 means that the membrane is freely permeable to the solute and the
solute can diffuse down its concentration gradient from the original
compartment. Recalling diffusion for a moment, this means that
eventually, the solute concentration will be the same in both
compartments. In this case, there will be no effective osmotic pressure
and no water flow. Notice that, in the van't Hoff equation, when F = 0 the
effective osmotic pressure also equals zero. An example of a solute with F
= 0 (or close to 0) is urea.

• F = between 0 and 1.0. Most solutes fall in between — they are


neither completely impermeable nor freely permeable. Thus, effective
osmotic pressure and water flow fall somewhere between their maximal
possible values and zero. Notice in the van't Hoff equation, when F is
between 0 and 1.0, that effective osmotic pressure is less than maximal,
but greater than zero. There are many examples of solutes with F between
0 and 1: mannitol, glucose, NaCl, KCl, CaCl2. In general, the larger the
solute, the higher the value of F.

D. Osmotic pressure difference drives water flow

For osmosis to occur (water flow due to an osmotic pressure difference), what we
really care about is the effective osmotic pressure difference between two

77
Membrane Transport 2 - Dr. Costanzo

solutions separated by a membrane. In practice, if you're given two solutions,


calculate the effective osmotic pressure of each one (units of pressure in mm Hg
or atm), then take the difference to determine the driving force for water flow.
Follow these rules to determine the direction of water flow:

• If the two solutions have the same effective osmotic pressure, they are
isotonic and no water flows between them.

• If the two solutions have different effective osmotic pressures, the one
with the higher pressure is called hypertonic and the one with the lower
pressure is called hypotonic. Very important: water flows from the
hypotonic solution into the hypertonic solution, from lower effective
osmotic pressure to higher effective osmotic pressure.

Consider the following example, based on the figure below that illustrates the
difference between osmolarity and osmotic pressure and the importance of the
reflection
coeffi

g = 1 for all solutes


F albumin = 1.0
F globulin = 1.0
F urea = 0

In A, the two solutions are isosmotic, since their calculated osmolarities are equal.
They also are isotonic because their calculated effective osmotic pressures are
equal (same F for albumin and globulin). In B, the two solutions are also
isosmotic, but they are not isotonic: the urea solution is hypotonic and the

78
Membrane Transport 2 - Dr. Costanzo

albumin solution is hypertonic; thus water would flow from right to left, from
hypotonic to hypertonic.

III. OSMOSIS VS DIFFUSION OF WATER

Osmosis of water should be distinguished from diffusion of water (with osmosis


being much more important across biological membranes). Diffusion of water
obeys Fick's Law of diffusion and depends on the concentration difference of
water and the surface area for diffusion. Surface area for diffusion is proportional
to the area of water-filled pores in the membrane (r2). Osmosis of water depends
on an osmotic pressure difference (see above). Water flow due to a pressure
difference (rather than a concentration difference) is based on Poiseuille's law
which states that flow is proportional to r4 of the water-filled pores. Thus,
osmosis of water is much faster than diffusion of water because of its fourth
power relationship with pore radius!

Flow = B r4 )P

8 0 )x

III. PRACTICE PROBLEMS

1. Red blood cells are placed in 150 mM NaCl and no change in their cell
volume is observed. It can be concluded that:

A. 150 mM NaCl is hypertonic


B. 150 mM NaCl is hypotonic
C. ICF has an effective osmotic pressure of 300 mOsM.
D. NaCl acts as if it were an undissociated molecule
E. NaCl has a higher permeability than water in red blood cells.

2. Red blood cells are placed in 300 mM solution of a nonelectrolyte with a


reflection coefficient of 0.4 and the cells swell. It may be concluded that:

A. a 300 mM solution of this substance is isotonic.


B. the solution has approximately the effective osmotic pressure of 60
mM NaCl.

79
Membrane Transport 2 - Dr. Costanzo

C. this substance will remain in the extracellular fluid compartment.


D. the solution is isosmotic with 60 mM NaCl.
E. the effective osmotic pressure of this solution is 150 mOsM.

3. Solutions 1 and 2 contain different concentrations of the same solute. After a


brief time, the volume of Solution 1 decreases, and the volume of Solution 2
increases. Which of the following statements explains or describes these
initial volume changes?

A. Initially, the effective osmotic pressure of Solution 1 is greater


than the effective osmotic pressure of Solution 2.
B. The observed volume changes would have been greater if the
reflection coefficient of the solute is 1.0 rather than 0.5.
C. The observed volume changes would have been greater if the
solute concentration of Solution 1 was doubled.
D. The reflection coefficient of the solute is zero.

4. Which of the following pairs of solutions is both isosmotic and isotonic.


Assume that NaCl is completely dissociated (i.e., g = 2) and the following
values for σ.

Protein 1.0

Sucrose 0.6

NaCl 0.5

Urea 0

Water 0

A. 100 mM protein; 100 mM sucrose


B. 100 mM sucrose; 100 mM NaCl
C. 100 mM urea; 50 mM NaCl
D. 100 mM urea; water
E. None of the above

ANSWERS

80
Membrane Transport 2 - Dr. Costanzo

1. C. (Since no volume change occurred, 150 mM NaCl must be isotonic with


respect to the ICF. That is it must be in osmotic equilibrium with it. Since the
osmotic pressure of the NaCl is approximately 2 x 150 = 300 mOsM, this must
also be the osmotic pressure of the ICF.)

2. B. (Since the substance is a nonelectrolyte its osmotic pressure is 1


mOsmole/mmole x 300 mM = 300 mOsM. However, its effective osmotic
pressure with respect to red blood cell membranes is: σ x 300 = 0.4 x 300 = 120
mOsM. That is it has the same effect as a substance with σ = 1, but at an osmotic
pressure of 120 mOsM. Such as substance would be NaCl at 60 mM, since it
would also have an osmotic pressure of 2 x 60 = 120 mOsM. This is a hypotonic
solution. That is why a 300 mOsm solution of this substance causes the cells to
swell. The solution is clearly not isosmotic with respect to 60 mM NaCl. The
latter has far fewer dissolved particles than the solution in question.)
3. B. Since water flowed into Solution 2 (its volume increased), the initial effective
osmotic pressure of Solution 2 must have been greater than that of Solution 1.
The higher the reflection coefficient, the greater the effective osmotic pressure
difference and the greater the water flow. If the solute concentration of Solution
was doubled, the initial effective osmotic pressure difference would have been
decreased. If the reflection coefficient was zero, there would have been no water
flow.
4. E.

81
Membrane Potentials - Dr. Logothetis

Title: Membrane Potentials


Subtitle: Ion Movement: Forces and Measurement
Diomedes E. Logothetis, Ph.D.

Lecture goals:

This lecture will discuss the chemical and electrical forces determining the direction and
magnitude of ion movement through permeable pathways across the plasma membrane,
the resting membrane potential and the use of voltage clamp and patch-clamp techniques
to study ion channel function. The “Learning Objectives” below define the topics and
what students should know regarding each topic.

Learning Objectives

1. Understand how distribution of unbalanced charges at the membrane boundary


accounts for membrane potentials.
2. Know the distribution (low vs. high) of the four major ions in most mammalian
cells.
3. Know the thermodynamic derivation of the Nernst Equation and be comfortable
explaining it qualitatively.
4. Given a membrane potential be able to determine the flow of ions (for a particular
distribution in and out of the cell) considering the relative magnitude and
direction of the chemical and electrical forces.
5. Be able to use Ohm’s law to determine the currents flowing through the cell
membrane.
6. Given a membrane potential be able to determine the flow of ions (for a particular
distribution in and out of the cell) considering Ohm’s Law.
7. Know the different types of Ion Channels and the major features they possess.
8. Understand how the balance of currents determines the membrane potential.
9. Know the voltage clamp and all modes of the patch clamp techniques.
10. Predict how ion flux through a particular ion channel will influence the membrane
potential.

Readings

• See Lecture Notes and

• Physiology, 3rd Edition, L.S. Costanzo, Saunders, pp 15-17

82
Membrane Potentials - Dr. Logothetis

Electrical signaling

Ion movement across the plasma membrane through ion channel proteins serves
distinct signaling roles such as changes in internal Ca2+ concentration or changes in the
membrane potential, thus allowing rapid communication of the cell with its external
environment. Although electrical signals are mainly thought to be the language of the
nervous system, they are also generated in almost all cells in response to a variety of
stimuli. Thus, the stereotypic electrical signals called spikes or action potentials serve as
important signals in many non-neuronal types of cells: for example, eggs generate an
action potential when they meet the first sperm, macrophages respond with a kind of
action potential to certain factors in complement as part of their chemotactic reaction and
secretory cells in many glands --- pancreas, pituitary, adrenal medulla for example --
undergo action potentials when the contents of their secretory granules are to be released.

Membrane Potential and Cell Capacitance


Sodium (Na+), potassium (K+), calcium (Ca2+) and chloride (Cl-) ions are
unequally distributed on either side of the plasma membrane. Yet, electroneutrality is
achieved in the bulk intracellular and extracellular solutions as positively and negatively
charged molecules screen the charge of each other. Suppose that we have some way of
taking individual positive ions (cations) out of the cytoplasm of a cell and placing them
outside (we will see later ways in which this can be done). As we move more and more
cations out of the cell it becomes more difficult (takes more work) to move each
additional ion because the partners of the cations, the negatively charged ions (anions)
that are left behind, attract additional cations from leaving. Moreover, we are building an
excess of cations outside that repel more cations from coming out. The amount of work
to transport one ion with valence z out of the cell is:
Work = -ze0V eqn. 1
where e0 is the elementary charge. This equation is the fundamental definition of the
membrane potential V. It is equal to the work that it takes to move a charge of valence z
across the membrane. Meanwhile, we just argued that the work increases the more ions
we transport across the membrane. The relationship between the work and the total
charge that has been transported turns out to be a simple proportionality, such that:
V=Q/C eqn. 2
where we say that Q is the net unbalanced charge left inside the cell, and C is a quantity
called the capacitance. From your physics courses you have learned that capacitance is
simply the capability of an insulator to separate electric charge. The capacitance of the
cell membrane resides specifically in the lipid bilayer, which insulates the extra- from the
intracellular aqueous phase with its hydrophobic core. Clearly, if a cell has a large
capacitance, you can move a lot of charge before V changes very much. The physical
properties of a membrane, which determine its capacitance, are the dielectric constant ε,
the membrane area A and the membrane thickness d.

C = εA/4πd eqn. 3

Let's remember from eqn. 3, that the membrane capacitance is proportional to the
membrane area (see below for the reason) and inversely proportional to the membrane
thickness (the larger the distance between the plates of the capacitor the weaker the
attractive forces that keep the charges on each side). As we shall see in later lectures, this
has great significance for the understanding of the pathophysiology of demyelinating
diseases such as multiple sclerosis.

83
Membrane Potentials - Dr. Logothetis

Below we see a diagram (Fig. 1) of what our negatively-charged cell is like: in both the
interior and in the exterior solution almost all the ions have balancing counter ions.

Figure 1. The net excess of


positive charges outside and
negative charges inside the
membrane of a cell at rest
represents a small fraction
of the total number of ions
inside and outside the cell
(the ratio of the width of the
region of charge separation
to cell diameter is
exaggerated here for
purposes of illustration).

There are two things to notice about this picture. First, the unbalanced charges are at the
membrane boundary. That is because unbalanced charges repel each other; if two of
them were somewhere in the middle of the cell or out in the extracellular solution, they
would repel each other and start to move apart. Thus charges always congregate at
boundaries, such as at the membrane, where they cannot move any further. This is why
the capacitance depends on the membrane area: if the area is larger, the unbalanced
charges are farther apart and there is less repulsion; that is, V is smaller.
The second thing to notice is that the ions that are next to the membrane are not
necessarily the same ones that we moved in the first place. Ions are always in motion, so
they are often exchanging places. Also, if you were to introduce an extra ion into the
center of the cell, it would start repelling its neighbors, causing them to move; they would
repel their neighbors and so on, until the net effect occurs that one unbalanced charge
winds up at the membrane surface. This process (which is just the conduction of
electricity in an ionic solution) is much faster than diffusion. That is, it would take much
longer for the original ion to diffuse over to the membrane than it takes for its electrical
influence to be felt. This is the fundamental reason why electrical signaling is the fastest
signaling process in cells.
To calculate the fraction of uncompensated ions on each side of the membrane
required to produce a specific membrane potential difference in a cell of a given
geometry, consider the space-charge neutrality principle. According to this principle, in a
given volume, the total charge of cations is approximately equal to the total charge of
anions. The membrane capacitance of a typical cell is 1 μF/cm2, which means that 10-6
uncompensated coulombs of charge on each side of the 1 cm2 membrane are needed to
produce 1 V across the membrane.

The Nernst equation and the resting potential

In the previous section we created a negative membrane potential (Fig. 1) by


removing some cations out of the cytoplasm and placing them outside the cell, thus
creating a charge imbalance. Indeed, when an experimenter records the potential
difference across the membrane (potential difference between two microelectrodes, one
inserted into a resting cell while the other outside the cell in the bath), (s)he records a

84
Membrane Potentials - Dr. Logothetis

constant ("resting") membrane potential difference of -80 to -90 mV (defined as Vin-


Vout). What is the basis for this resting potential?
Imagine that we could have a pathway that selectively allows for positive ions, that is
lets cations go through but is impermeable to anions. As it turns out the negative resting
potential of most cells, such as a muscle cell is caused by the fact that the resting
membrane is almost exclusively permeable to potassium ions (i.e. specific proteins that
allow potassium ions to leave the cell at rest). The membrane potential develops because
K ions, which are 30 times more concentrated inside the cell, have a tendency to leave the
cell (through specialized potassium permeable proteins). The resulting charge separation
(less positive charge inside) sets up the negative membrane potential.

The exact relation between concentration difference and membrane potential is


given by the Nernst equation, which we will now derive.

Let us start by remembering the extracellular and intracellular distribution of the


main ions, in a muscle cell, for example:
Extracellular Intracellular

Na+ 145 mM 15 mM
K+ 5 mM 145 mM
Cl- 125 mM 10 mM
Ca2+ 2 mM 0.0001 mM (!!)

We have already mentioned that ions cross membranes through specialized


proteins that provide a hydrophilic pathway. We will now consider WHY the ions move.
Ion movement through specialized proteins is energetically passive, that is, ions move
towards a lower level of free energy. The free energy of an ion is the sum of two
components: Chemical and electrical.

The chemical energy varies with ion concentration and temperature and has the
form:
Chemical energy =μo + RT ln [X]
Here μo is the standard free energy of a 1 molar solution, R is the universal gas constant,
T the absolute temperature and [X] the concentration of ion X. The chemical energy
represents the fact that random thermal motion tends to drive particles from regions
where they are concentrated to regions where they are dilute. For our muscle cell, this
means that for instance for K- ions there exists a chemical driving force for K-efflux from
the cell. The magnitude of this chemical energy gradient is simply the difference between
the two free energies inside and outside of the membrane:
Chemical energy gradient = μo + RT ln [X]i - (μo + RT ln [X]o)
= RT ln ([X]i/[X]o)

The electrical energy is proportional to the potential and has the form:
Electrical energy = zFV (per mole of ion)
V is the potential, F the Faraday constant (96,500 coulombs/mole) and z the valence (+1
for potassium). An ion will want to move towards a potential of opposite sign to its
charge, thus a cation will be attracted by a region of negative potential. The electrical
driving force is the difference between the electrical energies inside and outside:
Electrical energy gradient = zFVin - zFVout
= zF (Vin-Vout)

85
Membrane Potentials - Dr. Logothetis

The sum of the chemical and electrical energy gradients is called the
electrochemical gradient. The transmembrane electrochemical gradient is the real driving
force for ion movement through specialized proteins. This driving force vanishes when
the sum of the chemical and electrical gradient equals zero. This condition is called the
equilibrium condition, because there is no net transmembrane flux anymore. At
equilibrium, the concentration gradient is exactly counterbalanced by an electrical
gradient of opposite sign.

RT ln ([K]i/[K]o) + zF (Vin-Vout) = 0

Rearranging yields the familiar Nernst equation:

VX = Vin-Vout = (RT/zF) ln ([X]o/[X]i) (eqn. 4)


= (RT/zF) ln (10) log ([X]o/[X]i)

VX is called the Nernst potential for ion X. It is the potential that a membrane
selective for ion X would stabilize at. Other equivalent names for the Nernst potential
are: equilibrium or reversal potential. To distinguish the Nernst potential of a particular
ion from the membrane potential or the resting potential we will designate it with the
letter “E”. With the physiological extra- and intracellular concentrations listed above, we
can now calculate the Nernst potentials for the major ions. This will tell us, at what value
of the transmembrane potential the net driving force for a particular ion would vanish. At
37oC, the expression RT/F ln (10)= 61 mV. Since z=2 for Ca ions RT/F ln (10)= 30.5
mV
ENa = 61 log (145/15) = 60 mV
EK = 61 log (5/145) =-89 mV
ECl =-61 log (125/10) =-67 mV
ECa =30.5 log (2/.0001) =131 mV

Thus, the measured value of the membrane resting potential in our muscle cell (-80 to -90
mV) is very close to the Nernst potential for K- ions. It is as if the cell membrane is K-
selective. This is confirmed by the fact that changes in extracellular K lead to predictable
changes in the membrane potential, while changes in the other ions have little effect on
the resting potential. The resting muscle membrane behaves like a K-selective membrane
because specific K-permeable proteins are the only pathway for ions to move under
resting conditions. It is worth noting that at the resting potential there exists a large
inwardly directed electrochemical driving force for both Na and Ca ions. It can therefore
be anticipated, that if Na and/or Ca pathways were to be permeable abruptly, this would
lead to a large inward current which would make the inside of the cell more positive than
before this change in permeability. We will see in a later lecture that this is precisely the
mechanism that leads to the generation of an action potential.

Ohm's law is central

Electrical phenomena arise whenever charges (denoted as Q, measured in Coulombs) of


opposite sign are separated or can move independently. Any net flow of charges (or a
change of charge with time, dQ/dt) is called a current (I), measured in Amperes. For our
discussion of cellular excitability we will mainly study the mechanisms determining
current flow across the plasma membrane of a cell. The magnitude of a current flowing
between two points (for instance from extracellular to intracellular) is determined by the
potential difference (or "voltage", or "voltage difference") between the two points
(denoted as V, in Volts) and the resistance to current flow, R, measured in Ohms.

86
Membrane Potentials - Dr. Logothetis

I=V/R OHM'S LAW (eqn. 5a)

(For those who are uncomfortable with electricity: try the hydraulic equivalent Q=P/R.
Here the potential difference corresponds to the pressure difference P and the current
corresponds to the flow Q, while R refers to resistance).

When Ohm's law is applied to biological cell membranes, it is often advantageous to


replace the electrical resistance by its reciprocal, the conductance g, measured in
reciprocal Ohms, or Siemens.

I=gV OHM's LAW (eqn. 5b)

For simplicity, we will assume that all resistive elements in the cell membrane behave
in an "ohmic way", i.e. that their current voltage relationship (abbreviated as I-V) is
described by eqn. 7b: the I-V relation is linear with a slope given by the conductance g.
This is shown graphically by the solid line in Fig. 2a, which represents the
transmembrane current (I) measured at different transmembrane potentials (V) in a
hypothetical cell. Fig. 2b shows the experimental arrangement, the so called “voltage-
clamp technique”, which enables us to construct I-V relationships and to study the
conductance characteristics of the cell membrane. Using this technique, we have inserted
two microelectrodes into our cell (glass microelectrodes have tip diameters of 0.1-0.5
microns and can be inserted into many cells without apparent damage to the membrane).
One is connected to a voltmeter to measure the transmembrane potential. The second
microelectrode is hooked up to a tunable current source (battery of variable output),
which allows us to inject current into the cell. These electrodes are then connected to a
feedback circuit that compares the measured voltage across the membrane with the
voltage desired by the experimenter. If these two values differ, then current is injected
into the cell to compensate for this difference. This continuous feedback cycle, in which
the voltage is measured and current is injected, effectively “clamps” the membrane at a
particular voltage. If specialized proteins that can offer a hydrophilic path to ions (called
ion channels, see below) were to open (allowing ions to flow through them), then the
resultant flow of ions into or out of the cell would be

Figure 4a Figure 4b

87
Membrane Potentials - Dr. Logothetis

Figure 2. Measuring the resistive properties of the plasma membrane to the flow of specific
ions. (a) A current-voltage relationship showing ohmic (linear) characteristics. Conventions
are also shown, i.e. negative current means negative inside with respect to the outside, outward
current means positive ions (e.g. K) moving from the inside to the outside. (b) A voltage clamp
experiment using two electrodes. One measures voltage (inside with respect to the outside)
and compares it to the voltage that the experimenter desires to hold (or clamp) the cell
membrane at. To adjust the voltage to the desired level the experimenter injects through the
second electrode (the current electrode) either positive or negative current. The current
injected in order to keep the membrane voltage from changing is precisely matching the
amount of current entering or leaving the cell through conducting pathways (at the desired
membrane potential).
compensated for by the injection of positive or negative current into the cell through the
current-injection electrode. The current injected through this electrode is necessarily
equal to the current flowing through. It is the injected current that is measured by the
experimenter. mThe convention used in Fig.2a is that the voltage is expressed as the
difference between the intracellular and the extracellular potential (V=Vin-Vout). At
negative values of V the cell is said to be hyperpolarized, whereas at positive membrane
potentials it is said to be depolarized. Positive charge moving from inside to outside is
called outward current and is represented as an upward (positive) current, while inward
current is shown as a negative current deflection.

Ion channels carry transmembrane current

How does current actually flow through the cell membrane via the channel protein? The
answer to this question is not obvious, since we know that cell membranes are composed
of a lipid bilayer with a very highly hydrophobic center, which is practically totally
impermeable to charged particles like ions. Thus, in electrical terms, a lipid bilayer
presents an almost infinite resistance to ionic current flow. For this reason, over the last
~50 years the presence of specialized membrane structures called ion channels was
postulated. Today we know that ion channels are large transmembrane protein molecules
embedded in the lipid bilayer. Each channel forms a relatively hydrophilic central pore,
which allows ions to cross from one side of the membrane to the other. Many different
channel proteins exist and we will discuss some of them in detail. The two most
important functional properties of ion channels are (1) the fact that the channels fluctuate
between open (conducting) and closed (non-conducting) states. This property is called
channel gating and its importance will become obvious when we will consider the factors
that control it, and (2) their ability to distinguish between different ions (channel

Figure 3.
Schematic representation of the
various types of ion channels.
Depiction of a voltage gated
channel showing three key
features. A voltage sensor that
somehow is coupled to the
movement of a gate that opens
the channel so that ions can flow
down the electrochemical
gradient. Ions are selected (e.g.
K+ versus Na+) through the
selectivity filter.

88
Membrane Potentials - Dr. Logothetis

selectivity). The nomenclature of ion channels is based upon these two most important
functional properties. Thus, they are classified into distinct categories according to the

Figure 4
Four classes of ion channels:
voltage gated, intra- or
extracellular ligand gated and
mechanically gated channels.
Extracellular ligands include
neurotransmitters, peptides,
hormones.
Intracellular ligands include
signaling molecules such as
cAMP, cGMP, Ca2+, Na+, G
proteins, PIP2, etc.
Mechanical stimuli include
physical cell deformation,
shear stress, osmotic
pressure.

stimuli that cause them to open. Ion channels that open in response to changes in voltage
across the membrane are called voltage-gated (see Figs 3 and 4), those that open in
response to binding of a ligand are called ligand-gated, those that open in response to
mechanical stress are called mechanically gated or mechanosensitive channels (see Fig
4). A class of potassium channels that are thought to be active at resting membrane
potentials and as such to be major contributors to the negative resting potentials of cells
are starting to be recognized as PIP2--gated (as they seem to be opened by interactions
with the membrane phospholipid phosphatidylinositol-bis-phosphate). Within these
categories ion channels that are selective for K+ ions are called potassium channels, for
Ca2+ ions, calcium channels and so on. Thus, we have distinct K+ channels that are
voltage-gated, ligand-gated, or mechanosensitive. Always keep in mind, that for each
channel type there are several to many forms (e.g. the count is over 100 for K+ channels)
The benefit of the voltage-clamp technique can be appreciated for voltage-gated
currents in particular. These ionic currents are both voltage and time dependent; they
become active at certain membrane potentials and do so at a particular rate. Keeping the
voltage constant in the voltage clamp allows these two variables to be separated; the
voltage dependence and the kinetics of the ionic currents flowing through the plasma
membrane can be directly measured.

The balance of currents determines the potential

The current-voltage relation we drew in Fig. 5 does not apply to a real cell since as we
said there is a negative resting potential (i.e. at equilibrium where the net current is zero,
the membrane potential is negative). Figure 5 draws the correct relation for a K-selective
channel. It will now intersect the current axis at VK. Ohm's law still describes the I-V
relation, but we have to introduce the voltage offset, and eqn. 4b becomes:
IK = gK (V-EK) (eqn. 6)

V-EK is of course just the electrochemical driving force at any potential V. If K-channels
are the only channels open, then as stated above, the membrane potential will be EK.
However, if the cell membrane also has some other measurable conductance, then V will
deviate from EK. Let us assume for instance, that the cell also has a measurable Na-

89
Membrane Potentials - Dr. Logothetis

conductance, with gNa:gK = 1:5.

INa= gNa (V-ENa)

INa will be zero at ENa and have a slope of 1/5 that of IK (see Fig. 5). The resting
potential VR in this case will settle at a value between ENa and EK where net K efflux is
exactly balanced by net Na influx. This point can easily be determined graphically from

Figure 5. Two linear (for the


sake of simplicity) resting
conductances are shown. One
conducts K ions while the
other Na (gNa : gK = 1:5). The
dashed line shows the sum of
the two conductances or the
“net” conductance. Resting
membrane potential can be
appreciated graphically as the
potential where inward and
outward currents are equal or
they add up to zero. VK and
VNa are the same as EK and
ENa.
Since INa=-IK

gNa (VR-ENa) = - gK (VR-EK)

rearranging VR = (ENa gNa)/(gK+gNa)+(EK gK)/(gK+gNa) (eqn. 7)

So VR can have any value between ENa and EK. The actual value of VR
becomes a weighted average of the two Nernst potentials for Na and K, where the weight
is given by the relative conductances. In our example with the above values for ENa, EK
and gNa:gK, VR = -64 mV.

As expected, if gNa>>gK then VR=ENa


and, if gK >>gNa then VR=EK

V will however only stay constant as long as the ionic gradients do not change. If there
was no independently operating, active transport system which maintains the ionic
gradients (we will discuss this in the last lecture) these gradients would indeed run down,
since at VR there is a constant K-efflux and Na-influx.

The patch clamp technique

The patch clamp technique (see Fig. 6), a variant of the voltage clamp technique
described earlier, has revolutionized the study of ion channels. Erwin Neher and Bert
Sakmann who are primarily responsible for this technique, received the Nobel Prize in
1991. This technique allows one to record current flow not only from hundreds to
thousands of channels present in the plasma membrane but also ionic currents from a
single channel protein. With this technique, a fire-polished glass micropipette with a tip
diameter of around 1 μm is pressed against the plasma membrane of a cell. Application
of a small amount of suction to the pipette greatly tightens the seal between the pipette

90
Membrane Potentials - Dr. Logothetis

and the membrane. The result is a seal with extremely high resistance between the inside
and outside of the pipette. Thus ion flow through open ion channels offers much less
resistance than through the pipette-membrane seal.

and outside of the pipette. Thus ion flow through open ion channels offers much less

Figure 6. All modes of the patch-clamp technique start with a clean pipette pressed
against an intact cell to form a gigaohm seal (the resistance to ion flow between and the
membrane is in the order of gigaohms). Currents can be recorded in this “on-cell” or
“cell-attached” mode as minute currents passing between the pipette solution and the
cytoplasm. Additional suction can break the isolated patch without affecting the
gigaohm seal, giving access to the cell cytoplasm and measuring currents from the
“whole-cell” membrane (minus the small ripped patch). Pulling the patch pipette away
from the cell can rip a small patch of membrane giving rise to the “outside-out” patch
where the experimenter has easy access to the solution on the external side of the patch.
Finally, an alternative configuration is the “inside-out” mode of recording, where the
pipette is pulled away from the “on-cell” mode, exposing the inner surface of the
membrane to the bath, where the experimenter can easily manipulate the internal
solutions.

This dramatically improves the signal to noise ratio and extends the utility of the
technique to the whole range of channels involved in electrical excitability, including
those with small conductance. The patch clamp technique is highly versatile, as one can
record channel activity in different arrangements. The "cell-attached" recording records
microscopic (from just one or a few ion channel) currents, while the intracellular
environment of the cell is intact. Pulling away from the cell, a small patch of the
membrane can be ripped away (inside-out or outside-out patch) allowing recording of
single-channel currents under absolute control of the intracellular solution that can now
be easily changed through the bath solution (e.g. for the inside-out patch). If one would
like to record from many ion channels further suction can be applied to the cell attached
mode to break the patch under the electrode and allow access to the remaining cell
membrane containing many ion channels (whole cell). Since there are many different
types of ion channels in the membrane, one has to take cautious care of adjusting the
composition of the solutions and including pharmacological agents that will allow
isolation of the current type that needs to be studied.

91
Membrane Potentials - Dr. Logothetis

Ionic currents recorded by the patch clamp technique

Figure 7 shows single channel records of a K channels found in cardiac cells. 150 mM K
bathed the patch from each side. At 0 mV no single channel openings were observed (EK
= 0 mV). As the membrane potential was held to increasingly more and more negative
levels, the channel openings became larger and larger (difference from the dotted zero
line). When the single channel currents obtained at each membrane voltage was plotted
(for both the negative potentials shown, as well as the positive potentials not shown) a
linear ohmic relationship was obtained (labeled control). If the external K concentration
was adjusted to 30 mM or the internal K concentration was changed to 45 mM, while
leaving the K concentration on the other side to 150 mM, the I-V curves shown were
obtained. Do the EK values obtained experimentally match the values predicted by the
Nernst equation?
Figure 8 shows whole cell records from a cation selective (Na, Ca and Mg), cyclic
nucleotide-gated channel from the rod outer segment membrane (see next lecture). 1 mM
cGMP activated this current linearly at both negative and positive membrane potential
changes.

Figure 7. left: Single-


channel currents
recorded from an
inwardly rectifying K
channel in the open cell-
attached configuration.
Numbers to the left of
each current trace refer
to holding potential. The
electrode was filled with
Ca-free, high-K solution.
The dotted line indicates
zero-current level. right:
Single-channel I-V
relationships obtained
from similar patches as
those shown on the left.
From: H. Matsuda, A.
Saigusa & H. Irisawa,
1987 Nature 325: 156-
159 and H. Matsuda,
1991, Journal of
Physiology 435: 89-99.

92
Membrane Potentials - Dr. Logothetis

Figure 8. Macroscopic current-voltage relation at a saturating cyclic GMP concentration.


A, recordings from one patch. The membrane potential was held at 0, and
positive/negative voltage steps of +/- 10 mV increments and lasting 1 s were delivered, in
the absence and the presence of 1000 μM-cyclic GMP. The steady-state current amplitude
at the end of each voltage step was measured. The current measurements in the two runs
without cyclic GMP were averaged and then subtracted from the corresponding
measurements with cyclic GMP. B, averaged current-voltage relation from seven patches.
The current in each experiment was normalized to unity at –60 mV before averaging was
done. Black circles show averaged current values; the horizontal bars show standard
deviations of the currents, which are very small. The straight line is drawn through points
between 0 and –60 mV, and is extrapolated linearly to positive voltages. From L. W.
Haynes and K. –W. Yau, 1990, Journal of Physiology (London) 429: 451-481.

93
Membrane Potentials - Dr. Logothetis

Practice Questions
Choose the correct answer.
1. If a few positive charges were moved from the inside to the outside of a cell, the
unbalanced negative charges would
a. distribute uniformly in the cytoplasm
b. end up at the membrane boundary
c. diffuse from the place they were left unbalanced to their final destination
d. violate the principle of electroneutrality
2. The Nernst potential describes
a. The electrical force that together with the chemical force drive ions to move in
a particular direction
b. The resting potential of a cell
c. The electrical force that balances the chemical force so that there is no net
ion movement
d The potential at which ion current reverses direction from an inward (Na+) to
an outward (K+) current
3. The patch-clamp technique is a versatile technique enabling recording of ion
channel activity in several modes
a. The on-cell or cell-attached recording records the activity of all the channels
present in the entire cell
b. The whole-cell configuration records the activity of one or a few channel
molecules out of the whole cell
c. The excised patch recordings (inside-out or outside-out) enable the
experimenter to easily change the intracellular solution (i.e. bath solution)
d. The gigaohm seal between the membrane and the glass pipette increases
the signal to noise ratio so greatly that one can resolve the activity of a single
protein in real time.
4. A particular mammalian cell displays a chloride equilibrium potential of -60 mV.
Due to a high resting potassium conductance (K+ equilibrium is at -90 mV) the
cell’s resting potential is at -80 mV. Please check the correct answer.
a) What would the direction of the chloride and potassium currents be at rest?
Chloride: ___inward ___outward ___no net current
Potassium: ___inward ___outward ___no net current
b) What would the direction of the chloride and potassium ion movements be at rest?
Chloride: ___inward ___outward ___no net movement
Potassium: ___inward ___outward ___no net movement

Answer the following:


5. Assume that a particular cell is equally permeable to chloride and potassium
ions and that gK = gCl. If the equilibrium potential for chloride is -60 mV, while for
potassium is -90 mV, what would you predict the resting membrane potential to be?
Show all work.

6. Describe how the voltage clamp technique works to keep the membrane voltage
from changing.

94
Membrane Potentials - Dr. Logothetis

Answers to Practice Questions:

1b

2c

3d

4a: Chloride current is inward; Potassium current is outward


4b: Chloride ion movement is outward; Potassium ion movement is outward

5. At rest IK = -ICl (inward and outward currents are equal and opposite in direction so
there is no net current).
Thus gK (Vr - EK) = -gCl (Vr - ECl)

gK (Vr - (-90 mV)) = -gCl (Vr - (-60 mV))

gK (Vr +90 mV) = -gCl (Vr +60 mV)

(Vr +90 mV) = -(Vr +60 mV)

2Vr = -150 mV

Vr = -75 mV

6. Two microelectrodes are inserted into a cell: one is connected to a voltmeter to


measure the transmembrane potential; the second is hooked up to a tunable current source
(battery of variable output), which allows us to inject current into the cell. These
electrodes are then connected to a feedback circuit that compares the measured voltage
across the membrane with the voltage desired by the experimenter. If these two values
differ, then current is injected into the cell to compensate for this difference. Thus the
amount of current we provide equals the amount of current passing through the cell
membrane. Hence with a voltage clamp experiment we get both the amount of current
flowing at the particular membrane voltage that we are clamping the membrane.
Measuring the amount of current flowing at different membrane voltages allows us to
plot the current-voltage relationship for a particular ion channel thus revealing its
conductance (inverse of resistance) characteristics.

95
Membrane Potentials - Dr. Logothetis

Extra Problems
(Answers to be provided on eboard)

1. One use of concentration gradients of ions across cell membranes is to drive the flow
of ions during action potentials of excitable cells. A concentration gradient of ions across
a membrane may be expressed in terms of an electrical potential at equilibrium by use of
the Nernst Equation.

a) The concentrations of some of the ions inside (i) and outside (o) of a particular muscle
cell are as follows:

Na+o = 140 mM Na+i = 10 mM


+
K o = 4 mM K+i = 140 mM
Ca2+o = 1 mM Ca2+i = 10-4 mM
Calculate the equilibrium potential for each of the ions in the muscle cell.
b) The actual measured membrane potential for the muscle cell was -90 millivolts. From
this information, what conclusion can you draw concerning the relative conductances of
sodium and potassium in these cells at rest (i.e. in the absence of action potentials)
assuming that sodium and potassium are the only ions that contribute to membrane
potentials. Is this last assumption valid?
c) The value of the membrane potential at the peak of the action potential is +25 mV.
Which ion species is most conductive at the peak of the action potential?

2. One way to measure membrane potentials in cells or organelles relies on the use of
lipid-soluble ions like TPP+, which distribute themselves passively across membranes
and achieve transmembrane concentration gradients which depend on the membrane
potential. A suspension of mitochondria is exposed to 10 μM TPP+. At equilibrium, the
intramitochondrial TPP+ concentration is measured as 3 mM. What is the membrane
potential across the mitochondrial membrane?

3. Suppose there were a neurotransmitter which selectively opened channels for protons.
If the external pH is 7.4 and the intracellular pH is 7.0, and the resting potential is –90
mV, would the transmitter be excitatory (depolarizing) or inhibitory (hyperpolarizing)?

4. Cl- ions permeate skeletal muscle membranes and are (almost) passively distributed.
In an unstimulated skeletal muscle fiber, where does the Nernst equilibrium potential for
Cl- lie relative to VNa and VK? a) Does the presence of a chloride permeability have any
effect on the shape of the action potential (assume that the chloride permeability is not
voltage dependent but that it is time-dependent)? b) Is there a net influx or efflux of Cl-
ions during the action potential?
Gamma-amino-butyric acid (GABA) is a neurotransmitter which opens chloride selective
channels (e.g. in spinal cord neurons). c) In a neuron with a resting potential of –80 mV,
what happens to the membrane potential when GABA is added? d) Will GABA change
the threshold for action potential generation (assume no direct action of GABA on any
channels other than Cl- channels)? e) If so, in which direction will the threshold move?

96
Membrane Potentials - Dr. Logothetis

5. Assume that at rest a particular neuron is permeable to potassium and sodium ions. If
you are given values for EK = -89 mV and for ENa = +60 mV then calculate the resting
potential given that in this cell gk = 5 gNa.

97
Excitable Cells 1 - Dr. Logothetis

Title: Membrane Potential in Excitable Cells 1


Subtitle: Voltage-Gated Ion Channels and the basis of the Action Potential
Diomedes E. Logothetis, Ph.D.

Lecture goals:

This first of two lectures will use our understanding of voltage-gated Na+ and K+ channel
function to explain how an action potential is generated.

Learning Objectives

1. Describe the ion substitution experiments performed by Hodgkin and Huxley in


elucidating the ion responsible for the biphasic currents obtained in their voltage
clamp experiments with the squid giant axon. What pharmacological agents can
also distinguish between these currents?
2. Know how to plot and explain the current-voltage relationship of voltage-gated
K+ and Na+ currents.
3. Contrast the topology of various voltage-gated channels and discuss their
distinguishing structural feature involved in voltage sensitivity.
4. Know how voltage dependence is recognized in single-channel records obtained
from voltage steps to different potentials. Predict the result of an ensemble
average of many single-channel current records in response to steps to a particular
voltage.
5. Predict the result of an ensemble average of many single-channel current records
in response to steps to a particular voltage.
6. Plot the probability of Na+ channel opening and the Na+ current obtained as a
function of voltage.
7. Describe an action potential in terms of its dependence on the time course of gNa
and gK.
8. Discuss the mechanism of Na+ channel inactivation and its control of the
refractory period for generation of multiple action potentials.

Reading

• See Lecture Notes and

• Physiology 3rd Edition, L.S. Costanzo, Saunders, pp 17-22

98
Excitable Cells 1 - Dr. Logothetis

Voltage-Gated Channels

We will now consider voltage-gated ion channels, the class of ion channels that
generate action potentials, the brief electrical signals by which excitable cells such as
neurons, muscle and endocrine cells communicate with one another. I will present
voltage-gated channels with a historical perspective, to underscore the monumental work
of two British scientists, Hodgkin and Huxley (Nobel laureates 1963) whose work has
influenced physiologists in as major a way as Watson and Crick influenced molecular
biologists. Hodgkin and Huxley, used the voltage-clamp technique to study a relatively
easy preparation, the squid giant axon. The experiment shown in Fig. 1 illustrates the
currents they obtained using different voltage-clamp protocols.

Figure 1. An axon is bathed in sea water


and voltage clamped by the axial wire
method. The membrane potential is held at
–65 mV and then hyperpolarized in a step
to –130 mV or depolarized in a step to 0
mV. Outward ionic current is shown as an
upward deflection. The membrane
permeability mechanisms are clearly
asymmetrical. Hyperpolarization produces
only a small inward current, while
depolarization elicits a larger and biphasic
current.

A hyperpolarizing step from -65 mV (resting potential) to -130 mV failed to


produce a sizable ionic current (Fig. 1A). In contrast when the axon was depolarized to 0
mV, a large transient inward current and a later outward sustained current was elicited.
The ionic permeability of the membrane was changed in a dramatic manner by the step
depolarization. Hodgkin and Huxley set out to determine which ions carry the current
and how the underlying membrane permeability mechanisms work.
First they reasoned that each ion seemed to move passively down its
electrochemical gradient, so basic thermodynamic arguments could be used to predict
whether the net movement of an ion would be inward or outward at a given membrane
potential. For example current carried by Na+ ions should be inward at potentials
negative to the equilibrium potential ENa, and outward at potentials positive to ENa. If the
membrane was voltage clamped to ENa, Na+ ions should make no contribution to the
membrane current, and if the current reverses sign around ENa, it is possibly carried by
Na+ ions. The same argument could be applied to K+, Ca2+, and Cl-.
Second, ions could be easily added to or removed from the external solutions. In
the extreme, if a permeant ion were totally replaced by an impermeant ion, one
component of current would be abolished.
Using these approaches Hodkin and Huxley in 1952 identified two major
components, INa and IK, in the ionic current. They first noticed that the early transient
currents reverse their sign from inward to outward at around +60 mV (also referred to as
the "reversal potential") as would be expected if they are carried by Na+ ions. The late
currents, however, are outward at all test potentials (more positive to -65 mV), as would

99
Excitable Cells 1 - Dr. Logothetis

be expected for a K+ current with a reversal potential more negative than -65 mV. The
identification of INa was then confirmed by replacing most of the NaCl of the external
medium by choline chloride (Fig. 2).
Figure 2. An illustration of the classical
a ionic substitution method for analyzing the
ionic basis of voltage-clamp currents.
Ionic currents are measured in a squid axon
membrane stepped from a holding potential
b of –65 mV to –9 mV. The component
carried by Na+ ions is dissected out by
substituting impermanant choline ions for
most of the external sodium. (a) The
voltage protocol applied to an axon in
c seawater, showing inward and outward
ionic currents. (b) Axon in low-sodium
solution with 90% of the NaCl substitued
by choline chloride, showing only outward
ionic current. (c) Algebraic difference
between experimental records A and B,
showing the transient inward component of
current due to the inward movement of
external Na+ ions.

The early inward transient current seen in the control ("100% Na+") disappears in
low Na ("10% Na+"), while the late outward current remains. Subtracting the low-Na+
+

record from the control record reconstructs the transient time course of the Na+ current,
INa, shown below. The properties of INa and IK are frequently summarized in terms of
current-voltage relations. Figure 3 shows the peak INa and the late IK plotted as a
function of the voltage-clamp potential.

Figure 3. The axon membrane potential is


stepped under voltage clamp from the negative
holding ptential (VH or EH) to various test
potentials. Peak transient sodium current and
steady-state potassium current from each trace
are polotted against the test potential. The
curvature of the two I-E relations between –50
to –20 mV reflects the voltage-dependent
opening of Na+ and K+ channels.

With modern terminology we would describe the Hodgkin and Huxley results to
indicate that the axon membrane has two major types of ionic channels: Na+ channels
with a positive reversal potential, ENa, and K+ channels with a negative reversal potential,
EK. Both channels are largely closed at rest and they open with depolarization at

100
Excitable Cells 1 - Dr. Logothetis

different rates. Nowdays, pharmacological tools exist to isolate Na+ from K+ current.
Tetrodotoxin (TTX), a paralytic poison of some puffer fish, block selectively Na+
channels. Tetraethylammonium ion (TEA) selectively blocks IK. Figure 4 shows a
family of voltage steps producing a family of curents before and after application of the
selective blockers. It is from traces like these that I-V curves are constructed.

Figure 4. Pharmacological dissection of INa


and IK. A node of Ranvier under voltage
clamp is held at –95 mV, hyperpolarized for
40 ms to –120 mV, and then depolarized to
various potentials ranging from –60 to +60
mV in 15-mV steps. (A) Normal INa and IK
in Ringer’s solution. (B) Same node after
external addition of 300 nM TTX. Only IK
remains. (C) Control measurements in
another node. (D) Same node after external
addition of 6 mM TEA. Only INa remains.

Voltage-gated channels are comprised of four (six membrane-spanning-segment)


subunits as compared to the inward rectifier K+ channels (having only two membrane-
spanning-segments) that are not voltage-gated and are critical determinants of the resting
potential.
Figure 5. Topology of the
intracellular ligand-gated
channels we considered in
the last lecture, compared to
voltage-gated channels
discussed in this lecture.
Each of the four subunits in
these channel proteins is
thought to contain either two
(M1-M2 for inward rectifier
K+ channels) or six
membrane-spanning α
helixes, S1-S6. In voltage-
gated channels the S4
segment contains several
positively charged amino
acids and is the voltage-
sensing α helix. The pore-
lining P segment lies
between S5 and S6. The N-
terminus of the polypeptide,
located in the cytosol,
contains a globular domain
essential for inactivation of
the open channel.

101
Excitable Cells 1 - Dr. Logothetis

What characterizes voltage-gated channels from other channel proteins is a unique motif
found in the fourth transmembrane domain (S4), where in this predicted α helix K+
channels are coded in single subunits just like the CNG channels. Na+ and Ca2+ voltage-
gated channels exist as four-subunit transcripts all coded in a single gene.every third
amino acid residue is a positively charged arginine or lysine. Strong evidence has
implicated this segment as an integral part of the molecular voltage sensor that upon
depolarization moves away from the internal membrane surface and pulls the gate open
allowing ions to flow down their electrochemical gradient (Fig. 5).

Channel gating underlies variable conductance

As mentioned above, ion channels fluctuate between open and closed states, the
process we have called channel gating. With patch-clamp recordings, it has been
possible, to measure the openings and closings of a single channel molecule directly.
Figure 6 shows such recordings from a tiny "patch" of membrane that contains just one
voltage-gated K+-channel. Two voltage steps are shown, one at -20 and the other at +20
mV. The current level fluctuates between the zero current level (closed channel) and a
distinct open level. As expected for a K+ channel, the current through the open channel is
outward at both potentials, but the outward current is larger at +20 mV. The other
important difference between the two current traces is the fact that the channel is open
more often at the positive potential. The fraction of time a channel spends in the open
state is called the open probability Po (Po ranges between 0 and 1).

Figure 6. Single-channel recordings from a voltage-gated K+ channel. The step to


+20 mV elicits single channel currents of larger amplitude that the step to -20 mV,
simply because of the greater driving force. Similarly, the step to +20 mV shows that
the channel spends more time in the open state that it does during the step to -20 mV.
It is precisely this increase in open probability seen at more depolarized voltages that
characterizes these channels as voltage sensitive.

Below in Figure 7 we see records from a patch-clamp experiment, where a single


voltage-gated K+ channel has been isolated (e.g. in the on-cell, inside-out or outside-out
modes). The membrane voltage is clamped between two levels, first at –100 mV for a
relatively long period of time and next to +50 mV for 40 ms. Below we see the gating of
a single K+ channel in response to this voltage step repeated nine times (top trace). Each
voltage step produces a current record as shown in each of the records shown below. The
dashed line indicates the zero current level (the channel is closed), while the transition to
the positive current indicates the potassium flow for as long as the channel stays in the
open position. Each time the channel opens and closes stochastically. This probabilistic
nature of ion channel activity is very representative of single molecular function at large.
If we were to signal average current records obtained from a voltage step of the
membrane to the same value repeated many times (e.g. forty times in this example), we

102
Excitable Cells 1 - Dr. Logothetis

would obtain the ensemble average record shown at the bottom of the figure. This
macroscopic record is identical to what one would obtain if forty K+ channels were to
open simultaneously during a voltage step to the same potential.

Figure 7. Gating in single K+


channels. (a) Nine consecutive
depolarizations yield non-
inactivating K+ channels. Notice
the channel re-openings and the
relatively long times for the first
channel opening following the
voltage stimulus (compare to
figure 8 showing single Na+
channel activity). (b) Ensemble
average of 40 repeats of the same
protocol. (From Hille: Ionic
Channels of Excitable
Membranes, 1992, Sinauer Press,
p. 69)

Figure 8 shows recordings of single Na+ channels in response to a voltage pulse. Notice
that currents are elicited with a smaller delay than K+ currents, they activate more rapidly
than K+ currents and inactivate despite the maintenance of the depolarizing stimulus, a
phenomenon termed inactivation (more about inactivation below).

Figure 8. Cell-attached patch-clamp


recording of unitary Na currents in a
toe muscle of adult mouse during a
voltage step from -80 to -40 mV. (A)
Ten consecutive depolarization steps.
One can see two superimposed
channel openings in the first record but
not in any of the others. Dashed line
indicates the current level when Na
channels are closed. (B) The
ensemble mean of 352 repeats of the
same protocol. (From Patlak and
Ortiz, from Hille: Ionic Channels of
Excitable Membranes, 1992, Sinauer
Press, p. 68)

103
Excitable Cells 1 - Dr. Logothetis

Figure 9 shows the behavior of macroscopic Na+ currents (top two panels) and compares
it to that of the total current in a neuron (containing K+ channels as well – bottom panel).
If we were to plot the normalized current as a function of voltage (maximum current at 0
mV would be 100%) we can obtain a plot of the probability that Na+ channels would be
open at a given membrane potential (top panel).
If we were to plot the macroscopic current flowing through many Na+ channels, as a
function of the membrane voltage steps that we clamp the plasma membrane, we would
obtain the I-V plot shown in the middle panel.

Figure 9

The open probability (Po) of Na channels increases


steeply with depolarization. Thus, the effective Na+
conductance is voltage dependent.

The sodium current INa= gNa (V-VNa) with its


voltage dependent conductance gNa= gmax Po. gmax
is the (maximal) conductance of a Na channel when
Po = 1.

Net ionic current, obtained as the sum of the Na+


current and the current through the resting membrane,
here represented as IR=gR (V-VR). Note N-shaped
form with three zero-current intersects. Middle
intersect is threshold; more about that in the next
lecture.

Many mechanisms exist which can alter Po in an ion channel. For the ion channels,
which generate the action potential, the most important regulator of Po is the
transmembrane voltage (membrane potential). These voltage-gated ion channels, contain
as we mentioned earlier a voltage sensor connected to a "gate", which keeps the channel
shut at negative potentials and opens it at more positive potentials. It is this voltage
dependent change in Po of Na+ and K+ channels that underlies the conductance changes
which lead to the nerve action potential. Note that a change in membrane potential has
two independent effects on voltage-gated channels: (1) It changes the driving force for
ion movement and thus the current through an open channel. (2) It changes the open
probability Po. The voltage dependence of Po for voltage-gated Na+ channels is shown in
the top panel of Fig. 9. The second panel shows the voltage dependence of the Na+
current where we have now scaled the maximum conductance gmax (conductance of an
open Na+ channel) with Po to obtain INa=gmax Po (V-ENa). The third panel of Fig. 9
finally shows the net membrane current as the sum of the Na+ channel properties and the
conductance of the resting membrane (mainly K+ channels). For simplicity's sake we
have not taken into account that Po for K+-channels is also voltage dependent. The net
ionic membrane current has an N-shaped form with three intersects of the zero-current
axis.

104
Excitable Cells 1 - Dr. Logothetis

We will examine this panel in greater detail in the next lecture in an attempt to determine
the threshold for action potential generation.

For now, let us just consider the strikingly different effects of an increase in the Na+ or
K+ conductance by membrane depolarization:

Figure 10. Effect of membrane


potential of voltage-gated K+ and Na+
channels.
Thus, a depolarization which activates only K+ channels terminates itself (negative
feedback) whereas a depolarization which activates Na+ channels becomes regenerative
through positive feedback between depolarization and further opening of Na+ channels.
This positive feedback leads to the explosive upstroke of the action potential, once the
threshold is reached (see below).

The last property of Na+ channels necessary for the understanding of the action
potential is Na+ channel inactivation. The voltage dependent increase of the opening
probability of Na+ channels is not maintained in time, but is rapidly transient. When Na+
channels are depolarized rapidly, but then held at a positive potential, they open first but
then enter soon a non conducting "inactivated" state (see Fig.8). The word "inactivated"
means that as long as a Na+ channel is in that state, it cannot be opened again by a
subsequent depolarization. This property of Na+ channels is an important factor in
terminating the action potential (along with the K+ channel mediated hyperpolarization).
A cartoon of the three important functional states of a voltage gated Na+-channel is
shown in Fig. 11.

Figure 11. Ball-and-chain model of


inactivation gating. Three gating states of
the channel: Resting (R), Open (O) and
Inactivated (I). The part of the channel that
interacts with the inactivation ball becomes
exposed only upon opening of the channel.

Return of Na+ channels from the inactivated to the resting (closed) state requires
repolarization of the cell membrane. Another action potential can only be elicited after a
large fraction of Na+ channels have returned from the inactivated to the resting state, i.e.
the Na+ channels have become available to open once again. The time necessary for this

105
Excitable Cells 1 - Dr. Logothetis

recovery from inactivation determines the so called "refractory period", the minimal time
required before the cell can be excited again to fire the next action potential. In nerve and
muscle cells, this refractory period is very short (a few milliseconds) but it is greatly
prolonged in heart cells. Because inactivation of Na+ channels during the action potential
shuts down the inward current carried by those channels, Na+ channel inactivation helps
terminate the action potential. Maintained depolarizations will tend (after an initial Na+
channel opening) to drive Na+ channels into the inactivated state and therefore render a
cell unexcitable. The relative number of resting versus inactivated Na+ channels is
steeply voltage dependent between -80 and -40 mV, so steady depolarizations in this
potential range will greatly reduce the number of available (resting) Na+ channels, and
therefore the excitability of a cell. An example of a clinically important case of
maintained depolarization is that of elevated serum potassium levels.

Variable conductances: the generation of the action potential

When excitable cells are depolarized from their resting potential beyond a certain level
(threshold – more about this in the next lecture), they respond with a relatively large,
stereotyped potential change, the action potential. It is the action potential propagating
away from the site of origin, which underlies impulse conduction in nerve, muscle and
heart. We will deal with impulse conduction below but first let us consider the ionic basis
for the generation of the action potential.

Figure 12 shows the typical configuration of a nerve action potential. An initial


depolarization from the resting potential leads into a very rapid depolarization, called the
upstroke of the action potential. After the upstroke, the action potential peaks at a value
positive to +30 mV and then repolarizes. In many cells repolarization is followed by an
"undershoot" (afterhyperpolarization) of the membrane potential, which returns to its
resting value a few milliseconds after the end of the action potential.

Figure 12 We have seen in the last paragraph that the


membrane potential is determined by the relative
conductances of the ion channels in the cell membrane.
~45 years ago Hodgkin and Huxley concluded that the
nerve action potential is generated by rapid conductance
changes of Na+ and K+ channels. For this work which
was the first successful utilization of the voltage clamp
technique Hodgkin and Huxley received the Nobel Prize
in 1963. In the presence of Na+ and K+ channels, the
membrane potential can swing between EK=-90 mV and
ENa=+60 mV as these channels open. At rest, gK>>gNa,
and V is just slightly more positive than EK.
Upstroke and peak of the action potential are the result of a massive increase of gNa, such
that at the peak of the action potential the membrane potential approaches ENa because now
gNa>>gK. Repolarization occurs because gNa falls back to its resting low level and gK
increases. The resulting afterhyperpolarization marks the closest agreement between Vm and
EK, at this time gK>>>gNa. As gK returns to its normal level, Vm depolarizes to the resting
level. The timecourse of the changes in gNa and gK underlying the generation of the nerve
action potential is also shown in the bottom panel.

106
Excitable Cells 1 - Dr. Logothetis

How can an action potential be triggered?


Where does the initial depolarizing stimulus come from in a real cell that triggers
an action potential? The depolarizing stimulus can come from two sources.
1. Through electrical connections: From a cell that generates its own action potentials
(i.e. a pacemaker cell – such as in the heart or some neurons) and through specialized
channels called gap junctions (that connect pacemaker cells to other non-pacemaking
cells - see last lecture) current travels from one cell to another depolarizing the membrane
and triggering an action potential. This is equivalent to injecting current through a
microelectrode into a cell.
2. Through other channels: The initial depolarization can come from other channels,
for example channels opened by neurotransmitters. Those could be the extracellular
ligand-gated (neurotransmitter-activated) ion channels we encountered in the last lecture,
such as the nicotinic acetylcholine receptor (AChR) [we will discuss in greater detail in a
later lecture]. This channel is opened by the binding of two acetylcholine molecules and
acts to depolarize the membrane of a skeletal muscle fiber (see last lecture). It is about
equally permeable to both K+ and Na+, so its equilibrium (or reversal) potential is near 0
mV. This means that it would carry inward current when Vm<0 mV, for example at the
resting potential. Another way to think about it is that near the resting potential there
would be less driving force for K+ ions to leave the cell (closer to EK) than for Na+ ions to
enter the cell (farther away from ENa). So when AChR channels open and the cell is at
the resting potential, there will be a net inward current that can cause the initial
depolarization that brings the cell membrane to the threshold potential.

Cell capacitance, the metabolic cost of an action potential and


The capacitance of a membrane determines the metabolic "cost" of an action
potential, because as you may remember from my first lecture, the higher the
capacitance, the more charge must be moved to generate the voltage of an action
potential. The charge is supplied by the+ ionic current, and the greater the current, the
more energy must be expended by the Na /K+ ATPase (see next lecture) to restore the ion
gradients. Since the specific capacitance of a lipid bilayer is fairly constant at 1 μF/cm2,
we can calculate the number of Na+ ions, which have to enter a cell to produce the typical
depolarization of ~ 100 mV associated with the action potential upstroke. Let's assume a
spherical cell (e.g. a nerve cell body) with a diameter of 20 μm.

Membrane area = 4πr2 = 1267 μm2

Capacitance = 1267 μm2 x 1 μF/cm2 = 12.67 pF (1pF = 10-12F)

Charge = 12.67 pF x 100 mV = 1.267 x 10-12 C

Each Na+ ion carries the elementary charge of 1.6 x 10-19 C, so

1.267 x 10-12 C/1.6 x 10-19


6
C/Na
+
+
ion
= 7.91 x 10 Na ions/action potential.

Will this lead to a measurable increase of the intracellular Na+ concentration? Our cell
has a volume of 4/3πr = 4187 μm .
3 3

At [Na+]i= 15mM the cell contains

107
Excitable Cells 1 - Dr. Logothetis

0.015 moles/liter x 6x101023 ions/mole


+
x 4187x10-15 liter
= 3.77 x 10 Na ions/cell

So, each action potential will increase [Na+]i by about 0.05 %, an increase that is not
measurable chemically. After prolonged activity however, i.e. hundreds of action
potentials, [Na+]i will start rising and active pumping is required to maintain the
transmembrane gradient.
The relative change in ion concentration produced by each action potential also varies
with the surface to volume ratio of a cell. Since in our example we have chosen a
spherical cell (minimal surface to volume ratio), the relative Na+ accumulation can be
higher in cells of different geometries.

108
Excitable Cells 1 - Dr. Logothetis

Practice Questions

Choose the correct answer.


1. Voltage-dependent Na+ channels show the following characteristics:

a. They activate and inactivate rapidly at depolarized potentials


b. The display a linear current-voltage relationship
c. They display negative feedback regulation
d. They underlie the repolarization phase of the action potential

2. Voltage-dependent K+ channels underlying the squid giant axon action potential


described by Hodgkin and Huxley show the following characteristics:

a. They activate and inactivate rapidly at depolarized potentials


b. They display a biphasic current-voltage relationship
c. They display negative feedback regulation
d. They underlie the upstroke phase of the action potential

3. The action potential of the squid giant axon is characterized by:

a. At rest gK < gNa, and VR is just slightly positive than EK


b. The upstroke and peak of the action potential are the result of a massive increase
in gNa
c. Repolarization occurs because gNa inactivates
d. During the afterhyperpolarization gK < gNa

Answer the following:


4. You are using the patch clamp technique to study the characteristics of a voltage-
dependent Na+ channel from human cells.

5. How would you set up the patch clamp experiment to ensure that you measure
sodium current alone?

6. If you performed the same experiment in the presence of tetrodotoxin (TTX),


what would occur?

109
Excitable Cells 1 - Dr. Logothetis

Answers to Practice Questions:

1a

2c

3b

4. Establish a gigaseal with the patch presumably containing the sodium channel. Only
sodium should be present in the pipette.

5. In order to insure that no K+ channels are functioning, TEA can be added.

6. Na+ channel would be inhibited, thus no current would be observed at any test
potentials.

110
Excitable Cells 1 - Dr. Logothetis

Extra Problems
(Answers to be provided on eboard)

(1) The unicellular organism Paramecium caudatum shows a resting potential (RP)
and an action potential (AP) that are similar in many respects to corresponding neural
potentials. With the cell in “typical pond water”, the following measurements were made
with an intracellular electrode:

If one varies [K+]out only, or [Ca2+]out only, one observes the following:

In the following questions, assume that the membrane of P. caudatum is normally


permeable only to K+, Ca2+, and water.

(a) In the resting state, which of these is true? Explain concisely.


i. GK > GCa

111
Excitable Cells 1 - Dr. Logothetis

ii. GK = GCa
iii. GK < GCa

(b) Which is true during the peak of the AP? Explain concisely.
(c) Compared to the ionic concentrations of “typical pond water”, is [K+]in
greater than, equal to, or less than [K+]out? Explain.
(d) Compare also [Ca2+]in with [Ca2+]out.
(e) When the posterior end of the organism is mechanically tapped, the
membrane transiently hyperpolarizes. What conductance change(s) might be
responsible? Explain.

(2) You are using the technique of patch clamping to study the characteristics of a
voltage-dependent Na+ channel from human cells.

(a) How would you set up the patch clamp experiment to ensure that you
measure sodium current alone?
(b) In experiment I, you maintain the membrane potential at –55 mV and
determine the ability of Na+ to move through the channel. You do the same in
experiments II, III, and IV, but you maintain the membrane potentials at –40
mV, -20 mV, and +20 mV, respectively. The results in terms of Na+
permeability are presented below:

Experiment Clamped Membrane Potential Na+ Permeability


(mV)

I -55 Absent
II -40 Absent
III -20 Present
IV +20 Present

Explain these results. What can you say about the approximate magnitude of the
threshold potential for these cells?

(c) If you performed the same experiment in the presence of tetrodotoxin (TTX),
what would occur?

(3) Ionic currents involved in the action potential of a cardiac muscle fiber have been
studied by the voltage-clamp technique. When membrane potential is stepped from its
resting value of –77 mV to –50 mV, an initial inward current is seen, which is carried by
Na+.

(a) Assume internal Na+ concentration is normally 30 mM, and external Na+
concentration is normally 150 mM. Draw to approximate scale the initial
current traces for a step to 0 mV (zero mV) when external Na+ concentration
is normal; and when external Na+ concentration is reduced to 30 mM, to 10
mM, and to 1 mM by replacement of Na+ with an impermeable cation.

112
Excitable Cells 1 - Dr. Logothetis

(b) If the peak inward Na+ current with normal Na+ concentration is 1 mA/cm2,
calculate the peak Na+ current in each of the cases.
(c) External Na+ concentration is adjusted so that the initial inward current during
a voltage clamp step to –50 mV is abolished. However, when the membrane
potential is stepped from –77 mV to –20 mV, a longer-lasting inward current
is recorded. Can this be due to the opening of further Na+ channels at this
membrane potential? Explain in one sentence. Assuming that internal and
external K+ and Cl- concentrations are comparable to those of frog muscle,
could it be due to the opening of K+ channels or of Cl- channels? Explain
each answer.
(d) The suggestion has been made that Ca2+ carries the current. If external
Ca2+ concentration is 2.5 mM and internal Ca2+ concentration is less than 10-2
mM, in what range is ECa? Would the Ca2+ current at a membrane potential
of –20 mV be in the right direction to account for the observed current?
Justify your answer.

(4) When a normal, healthy squid axon is voltage-clamped in artificial sea water, one
obtains the following membrane current record in response to a step change in membrane
potential from Vm = -70 mV to Vm = 0 mV.

Draw similar plots of Im vs. t (when Vm is stepped from –70 mV to 0 mV) when the
recordings are made under each of the following experimental conditions. For each of
your plots, explain in one or two sentences how and why your graph differs from that
drawn above.

(a) TTX is added to the bath surrounding the axon.


(b) TEA (tetraethylammonium) is added to the interior of the axon.
(c) [Na+]out is adjusted so that [Na+]out = [Na+]in.
(d) [K+]out is adjusted so that [K+]out = [K+]in.
Ouabain, the specific inhibitor of the Na+-K+ pump, is added to the bath five minutes
before the experiment.

113
Excitable Cells 2 - Dr. Logothetis

Title: Membrane Potential in Excitable Cells 2


Subtitle: Action Potential Propagation
Diomedes E. Logothetis, Ph.D.
(Dr. DeSimone’s lecture notes revised)

Learning Objectives:

1. Explain the concept of a threshold for an action potential and compare it to other
phases of the action potential where there is zero current (resting potential or peak
of the action potential).
2. Give a qualitative description of the conduction of an action potential along an
axon in terms of a local electrical circuit involving ion flow across the axolemma,
along the cytoplasmic core of the axon and within the extracellular fluid just
outside the axolemma.
3. State in general how nerve fiber conduction velocity varies with fiber diameter.
4. Define the time constant of an axon in terms of membrane resistance and
capacitance and also define it in terms of the time required to change an imposed
subthreshold membrane potential difference by a certain amount.
5. Define the length constant of an axon in terms of the distance over which an
imposed electrotonic depolarization decays back to its resting value. Also define it
in terms of fiber diameter, membrane resistance, and core resistance.
6. State a relation for the propagation velocity of an action potential in terms of the
axon length constant and time constant. Then relate this to the capacitance,
resistance and diameter.
7. Describe which electrical property is changed most by myelinization.
8. Give a reason for the regular gaps in the myelin shealth called nodes of Ranvier.
9. Define and discuss saltatory conduction.
10. Present the mechanism responsible for the diphasic shape of action potentials
recorded extracellularly.
11. Explain why the amplitude of extracellularly recorded diphasic action potentials
from a large multifiber nerve bundle (summated or compound action potential)
may vary with the stimulus strength.

Readings

• See Lecture notes and


• Physiology, 3rd Edition, L.S. Costanzo, Saunders, pp 22-25

114
Excitable Cells 2 - Dr. Logothetis

The ionic basis of key action potential characteristics

Threshold – A threshold exists because the nerve membrane potential is determined


by 2 opposing tendencies. At rest the membrane potential is dominated by the
hyperpolarizing efflux of K+ ions. As the membrane is depolarized the Na+
permeability increases, but until a threshold level is reached the hyperpolarizing
effect of K+ ion efflux is greater than the depolarizing effect of Na+ permeation. At
threshold the roles reverse and the depolarizing effect of Na+ becomes the dominating
process, i.e. at threshold a sufficient number of activation gates are simultaneously
open to allow Na+ to further depolarize the cell and activate even more channel
gates.

Absolute Refractory Period – This is the time interval after the peak of an action
potential during which a second depolarizing stimulus will fail to produce a second
action potential, irrespective of the stimulus intensity. The reason such a period exists
is that for a period of 0.5 - 1.0 msec following the peak of an action potential a large
proportion of the Na+ channels are in the nonconducting inactivation state. Most must
be in the resting state.

Relative Refractory Period – This is the time interval after the peak of an action
potential during which a second depolarizing stimulus will produce a second action
potential, but only if the stimulus intensity is increased. The relative refractory period
arises because toward the end of an action potential the K+ permeability remains
higher than the resting K+ permeability for a brief period. The membrane potential is,
therefore, more hyperpolarized than the normal resting potential (the hyperpolarizing
afterpotential or undershoot). It, therefore, requires a stronger depolarizing stimulus
to take the potential to threshold. Also a small proportion of the Na+ channels
remaining in the inactive state may also contribute to the relative refractory period.

Accommodation – Whether or not a depolarizing stimulus will produce an action


potential depends on the rate of depolarization as well as the magnitude. If the rate of
depolarization is too slow the threshold for firing will be raised or firing will not
occur at all. The reason is that a slow rate of depolarization allows a large proportion
of the Na+ channel inactivation gates to close before a large proportion of the
activation gates have opened.

115
Excitable Cells 2 - Dr. Logothetis

The threshold of the action potential

In Fig. 1a below we have plotted the net ionic current (solid line) obtained from the
sum of the Na current (dotted line) and a resting K current, shown by the dashed line.
This resting K current is not dependent on voltage (it is ohmic).

Figure 1a Figure 1b

What would happen if the membrane potential (Vm) was displaced from rest? If the
total net current is positive (meaning positive ions flow outward) Vm decreases (going
negative), while when the net current is negative (meaning positive ions flow inward)
Vm increases (going positive). The arrows shown in Fig 1a indicate "trajectories"
followed by Vm. Notice that there are three potentials at which the net current is zero.
Those are labeled: VR for VRest, Vt for Vthreshold, and Vp for Vpeak. We can see that VR
and Vp are stable points since when Vm becomes slightly depolarized or
hyperpolarized from these points, the direction of change of Vm will be such as to
bring the membrane potential back to that point. In Fig. 1b we have enlarged the area
of the graph around Vt. We can see that Vt is an unstable point. If Vm is just negative
to Vt then it will gradually return back to VR; if Vm is just positive to Vt then it will
instead depolarize more, stopping only when Vp is reached. Vt is clearly a threshold
that separates responses that go on to large depolarizations (suprathreshold responses
yielding action potentials) and those that do not (subthreshold responses). Another
way to think of the threshold is the membrane potential where the outward current
through K+ channels is just balanced by the inward current through Na+ channels.
The balance is such that a further depolarization results in a net inward current which
(as we saw in the previous lecture considering Na+ channel activation) yields the
positive feedback that starts the action potential. A common misconception is that
threshold refers to the potential where Na+ channels start to open. Notice in Fig 1a
that this is not the case. Na+ channels are opening at potentials negative to the
threshold. The threshold depends critically on both K+ and Na+ conductances. An
increase in the K+ conductance raises the threshold (i.e. a larger depolarization is

116
Excitable Cells 2 - Dr. Logothetis

required to elicit an action potential). This is an important consideration in neurons


and in heart muscle, where K+ conductance is under control of neuromodulator
substances and hormones.

Propagation of the potentials along an axon

General Considerations – So far we have considered the time course of the action
potential as if it were occurring in one place in a cell. In reality action potentials,
initiated in one part of a neuron, propagate along the axon in an unattenuated manner,
i.e. they behave like traveling waves of electrical excitation that do not decay and die
away with distance from their source. That is how information travels from the
peripheral nervous system to the central nervous system and then out to effector
organs, such as muscles and glands, without being degraded. The mechanism behind
the propagation of action potentials depends on the conductance changes that underlie
the action potential, already discussed, and certain physical and electrical
characteristics of axons, e.g. their diameters, membrane resistances, axoplasm
resistances, and membrane capacitances.

Propagation as a moving local circuit

Consider the figure below representing an axon in various states of stimulation.

Figure 2.

A. Resting Axon – At rest the K+ conductances prevail and the potential inside is
negative with respect to the outside.

117
Excitable Cells 2 - Dr. Logothetis

B. Suppose a depolarizing current is injected into the axon at some arbitrary


point and an action potential is initiated. At the peak of the action potential the
inside of the axon becomes positive with respect to the outside. But in
addition to that, the point where the action potential has just occurred is
positive with respect to adjacent points along the inside core of the axon,
which are still at rest and hence negative. Since the resistance to the flow of
ions is much lower along the axis of the axon (i.e. in the cytoplasm of the
axon) than across the axolemma, positive ions flow to into the immediate
adjacent regions.
C. If the ion flow is sufficient to depolarize the adjacent regions beyond
threshold, a “second” action potential will occur. In fact the flow of current is
continuous, so this isn’t really a second action potential, but a translocation of
the first. The action potential will “travel” in this manner along the axon (over
large distances if necessary) to reach its target (usually a synapse with another
neuron or a neuromuscular junction - to be discussed in the next lecture). Note
that the “all or none”property of the action potential at each successive point
along the axon insures that the action potential remains intact, i.e. does not
degrade with distance from its source. In this example, the initial stimulus is
assumed to be a depolarizing current applied by an investigator using a current
source. In that case action potential waves will travel in opposite directions
along the axon, something that does not usually occur in vivo.

What determines the velocity of propagation of action potentials?

This an important question because it is well known that action potentials travel at
different speeds in different nerves. Data on various fiber types indicate that fiber
diameter is one determinant.

Figure 3.

To understand how fiber diameter helps to determine action potential velocity, we


need to first understand the axon’s passive electric properties, i.e. the properties
that govern the spread of a passive or electrotonic potential (one that is
subthreshold so regenerative action potentials do not occur, i.e. the potentials

118
Excitable Cells 2 - Dr. Logothetis

decay with distance from their source). The two key quantities are the membrane
time constant and space constant.

A. Time Constant – Since all membranes have ion channels (i.e. conductances)
that move charge (i.e. ions) the aggregate of those conductances per unit area
make up the membrane’s electrical conductance per unit area (or resistance
per unit area since the reciprocal of conductance is resistance). In addition to a
resistive element all membranes act as if they also have a capacitive element
in parallel with the resistive element (see figure below).

Figure 4.

This aspect is seen when one tries to pass current across the membrane. The
current rapidly polarizes the membrane as if it were charging up an electrical
capacitor. This process requires some finite time to complete. Once the
capacitor is charged up, the current simply shunts through the resistive
elements (which are the ion channels of course). The figure at the right shows
the capacitive and resistive elements. The membrane potential when no
current is being passed (Resting potential) is shown as a battery in series with
the resistance. Suppose the axon resting potential is -60 mV and a
depolarizing current is switched on instantly that is sufficient to bring the
potential to -50 mV. How long will it take the potential to go from -60 to -50
mV? In other words how long does it take to polarize the membrane (i.e.
charge up the capacitor) and pass a sufficient number of ions through the
channels (i.e. the resistor). That depends on the time constant of the circuit, τ.
τ is the product of the resistance Rm and the capacitance, Cm,

And the membrane potential, Vm, changes according to

119
Excitable Cells 2 - Dr. Logothetis

Vm is plotted below for 3 values of the time constant, 1 msec, 10 msec, and 20
msec. As the time constant increases it takes longer for the potential to go
from -60 to -50 mV. So as the membrane resistance increases, it is harder to
change the potential (this makes sense because this also means that there a
fewer ion channels). So the larger the time constant the longer it takes to bring
the potential from resting to threshold. In the equation for Vm above, the time
constant has the interpretation as the time it takes for the potential to change
to 63% of the whole potential difference. The whole potential difference is 10
mV. When you let t = τ in the equation, the potential is -53.7 mV which is
63% of 10 mV ((-53.7 + 60)/10).

Figure 5.

B. Space (or Length) Constant – Another passive axon property important in


determining the velocity of propagation of an action potential is the length
constant. This notion comes into play when we consider the spread of an
electrotonic (subtheshold) depolarizing voltage from its source at one place in
the axon to more remote points farther down the length of the axon. Consider
the following situation:
1. A nerve cell body is depolarized from -60 mV to -50 mV and enough
current is continued to pass to hold the potential at -50 mV.
2. Positive ions will now flow down the axon from where it is -50 mV to
where it is -60 mV. This causes these neighboring regions to become
more depolarized so they reach a potential between -50 mV and -60
mV.

120
Excitable Cells 2 - Dr. Logothetis

3. Eventually a steady-state is reached where the potential declines


exponentially with distance down the axon. The potential is:

So at the source where x = 0, the potential is -50 mV and at large


distances from the source, the electrotonic potential decays down to
the original resting potential (i.e. when x = ∞, Vm = -60 mV). The
length constant in the equation is denoted as λ. When x = λ, the
potential has dropped to -56.3 mV, which is a 63% drop from its initial
value toward the final value (i.e. -60 mV). So λ is the distance from
the source for the electrotonic potential to decay by 63%. What is that
distance?

4. The value of the length constant for any given fiber is related to the
fiber diameter, d, the membrane resistance per unit area, Rm and the
core resistance per unit volume, Ri. The relation is:

Typical values for Rm and Ri are respectively: 2000 ohms cm2 and 60
ohms cm. For a Aα fiber of diameter 18.4 :m, λ = 1.24 mm. For a C
fiber of diameter 2.5 :m, λ = 0.46 mm. The figure below shows the
decay in potential for these 2 values of λ. So the bigger the fiber
diameter, the bigger the length constant and the farther the electrotonic
impulse travels before it decays away.

121
Excitable Cells 2 - Dr. Logothetis

Figure 6.

D. Velocity of the Action Potential – Suppose that the nerve cell body is
depolarized from -60 mV to some voltage above threshold and an action
potential occurs. Just as in the electrotonic case, positive ions will flow into
the adjoining axonal region. If the length constant is high, the current will be
adequate to depolarize the adjoining region above threshold potential, and the
action potential will be propagated. If the length constant is too small,
however, the spreading potential will fall below threshold in a very short
distance and the action potential will die out and not propagate. So we can
expect that the velocity of propagation (v) of the action potential will be
directly proportional to the axon’s length constant (λ). The time constant
plays a role here too. As positive ions flow from the region of the action
potential, it takes a certain amount of time to bring the potential in the
adjoining region up to threshold. The shorter this time the more certain it will
be that the action potential will propagate (if it takes too long Na-channel
inactivation will occur before a significant number of Na-channels are
activated). So we can expect that velocity of propagation (v) of the action
potential will be inversely proportional to the axon’s time constant (τ).
E. The Desirability of High Velocity – Animals with fast reflexes have a
significant survival advantage. What can be done to maximize the action
potential velocity? The membrane capacitance per unit area, resistance per
unit area, and resistance per unit core volume do not vary significantly
between simple axons (but see IV below). Thus it would seem that the only
way to get a faster action potential is to increase the diameter of the axon. This
is the strategy employed by many invertebrates such as leeches and squid. The
squid giant axon has a diameter between 0.5 - 1 mm (the reason it was so well
researched). There is one huge problem with this strategy. From the above
equation it is clear that to double the velocity of an action potential, we need
to increase its diameter by a factor of 4 (because velocity goes as the square

122
Excitable Cells 2 - Dr. Logothetis

root of diameter). Big axons, while they may solve the problem of speed in
simple invertebrates, are not practical in the complex vertebrate nervous
system where nerve tracts may contain thousands of fibers. The question
becomes then, how can you have speed and small diameter fibers at the same
time?

Figure 7.

Action potentials in myelinated axons

A. Why not insulate the axon? If there is a practical limit to the diameter of
an axon, is there then another way to increase velocity? The answer is yes,
by decreasing the membrane capacitance, Cm. If you wrap the axons in
layers of membrane rich in lipid, you significantly reduce the axon
dielectric constant, and, therefore, Cm is much smaller than for a bare
axon. This would cause Cm in the velocity equation to be smaller, so the
velocity would increase. This is what happens. Axons are wrapped in an
outer sheath of lipid-rich myelin (see figure below). Another effect of the
myelin sheath is to increase the membrane resistance Rm. This increases
the length constant and increases the spread of current down the axon.

The one problem with this is that if you wrap the whole axon, there is no
way for Na+ to flow into the axon to propagate an action potential.

123
Excitable Cells 2 - Dr. Logothetis

B. Nodes of Ranvier – The solution is to leave gaps in the myelin sheath.


These are the nodes of Ranvier (fig. below) which expose the axolemma
to the extracellular fluid every 1 to 2 mm. Each node is about 1 :m wide.

Figure 8.

C. Saltatory Conduction – The conduction of an action potential in a


myelinated nerve is a combination of both regenerative ion conduction and
passive electrotonic conduction. Consider figure 8. It represents an axon
wrapped in a myelin sheath that is conducting an action potential. Let’s
start at the node on the left in the figure. Here the bare axon is in contact
with the extracellular solution so the voltage-gated Na+ and K+ channels
can produce an action potential in the usual way.

The action potential then moves to the right, but in so doing, it also moves
beyond the node. Na+ can no longer flow into the axon so the amplitude of the
potential begins to decline. But because Rm is now much higher, the length
constant is much increased and because Cm is lower, the time constant is much
decreased. Hence there is a great boost in the velocity of the potential. But even
though it is now moving fast, there is no getting around the fact that the
potential is decaying. What saves the day is the timely appearance of a
successive node of Ranvier. Because the length constant is now large, the
amplitude of the potential never declines below threshold. So when the potential
reaches the next node, the voltage-gated Na+ channels become conducting and
we again have the complete action potential intact. The process continues in this
manner, so that the action potential seems to “jump” between the nodes on its
way to its final destination, hence the term “saltatory” (from the Latin or Italian
verb “saltare” which translates as “to jump”).

D. Energy Efficient – As well as increasing the velocity of the action


potential, conduction by myelinated nerves is more energy efficient
because Na+ flows into the axon only at the nodes and not continuously
along the whole axon length. This means that there is less Na+ for the cell

124
Excitable Cells 2 - Dr. Logothetis

Na+-K+ pump to have to remove, hence a saving in ATP.

E. Demyelinating Diseases – Disorders that destroy the myelin shealth


interfere with nervous conduction. The loss of myelin reduces the length
constant to the extent that an action potential cannot remain above
threshold from one node to the next. Conduction is, therefore, blocked.
Examples are of such diseases are multiple sclerosis and the peripheral
neuropathy associated with diabetes mellitus.

Extracellular recordings

Intracellular recordings are easiest to interpret, but technically difficult to implement


in humans because of their invasive nature. An alternative that proves satisfactory in
many clinical settings is extracellular recording. This is the method used to record
mass activity in a variety of tissues. Examples include: the electrocardiogram (ECG),
electromyogram (EMG), and electroencephalogram (EEG).

A. Extracellular Recording From a Single Axon – To understand an


extracellular recording, it is first useful to study one occurring in a single
axon. In the figure below in part 1 an axon has 2 metal electrodes A and B
in contact with its axolemma a ceratin distance apart. The electrodes are
connected to an amplifier that displays the voltage difference between
them as VB - VA. The axon is at rest so the inside has a resting potential of -
B

60 mV with respect to the outside. Another way of saying the same thing
is that the outside has a potential of +60 mV with respect to the inside.

Figure 9.

1. No action potential has occurred so the potential difference VB - VA =


B

(+60) - (+60) = 0

125
Excitable Cells 2 - Dr. Logothetis

2. An action potential is initiated at the left, so the potential at electrode


A now becomes inside positive (say +40 mV), or alternatively outside
negative with respect to the inside, i.e. VA = -40 mV. But electrode B is
still at rest, so VB = +60 mV. So now VB - VA = (+60) - (-40) = +100
B B

mV. So the tracing goes electropositive at point 2.


3. Now the action potential has reached both electrodes, so VB = -40 mV
B

and VA = -40 mV. So VB - VA = (-40) - (-40) = 0, i.e the potential


difference is again 0 mV.
4. Now electrode A has returned to rest conditions but electrode B still
sees the action potential, so VA = +60 mV and VB = -40 mV, So VB - VA
B B

= (-40) - (+60) = -100 mV, i.e. the recorded extracellular potential


difference turns negative.

Therefore, a single action potential recorded extracellularly is diphasic, i.e


it has both a positive and a negative component.

B. Extracellular Recording from a Whole Nerve – A vertebrate nerve is made up


of a large number of neurons and so it contains many axon fibers. Most fibers
will be myelinated, but some may also be unmyelinated. Also fiber diameter
will vary within the nerve bundle.

Figure 10.

In the figure above a whole nerve bundle has 2 sets of electrodes on the outer
nerve shealth (epineurium). These are a current passing set (stimulating
electrodes) and a voltage recording set.

126
Excitable Cells 2 - Dr. Logothetis

1. In part 1 a small current is passed into the nerve bundle. This current
must cross the epineurium and then activate single fibers in the bundle.
If the current intensity is too low, only a small stimulus artifact is
recorded, i.e. no action potentials are evoked.
2. In part 2 the current is more intense so the stimulus artifact is larger,
but the current is still inadequate to bring any fibers to threshold.
3. A more intense stimulus causes the fibers with the lowest thresholds to
propagate action potentials. These travel to the recording electrodes
are seen as diphasic potentials.
4. 4-6 Still more intense stimuli bring increasingly more fibers to
threshold causing the amplitude of the recorded action potential to
increase. This pattern is followed until all possible fibers have been
recruited.

Note: The reason the amplitude of this extracellular recording increases


with stimulus strength is that it represents the sum of action potentials
from many individual fibers that have different threshold potentials. As
the stimulus intensity increases, more fibers contribute to the recorded
potential. Of course the amplitude of any one action potential in any one
fiber is independent of the stimulus strength once the threshold potential
for that fiber has been reached. But the extracellular electrodes are
measuring a summated potential, and its amplitude will vary with the
number of fibers recruited at any one time. The summated action potential
in an extracellular recording is also called a compound action potential.

127
Excitable Cells 2 - Dr. Logothetis

Practice questions

1. Increasing the membrane capacitance causes:

A. the membrane space constant to decrease


B. the membrane time constant to increase
C. the neuron to depolarize faster
D. no change in membrane potential
E. the membrane resistance to increase

2. An unmyelinated neuron with a diameter of 1 :m has a length constant of


1 mm. A similar neuron with a space constant of 2 mm will have a
diameter of:

A. 2 :m
B. 1.414 :m
C. 4 :m
D. 1 :m
E. 10:m

3. Increasing the distance between nodes of Ranvier:

A. will increase the velocity of the action potential


B. will depolarize the axon
C. may cause the propagating potential wave to fall below threshold
D. will decrease action potential velocity in unmyelinated neurons
E. can never affect action potential propagation since it is “all or
none”

4. Assume extracellular electrodes are placed on a single axon at points A


and B respectively. Assume an action potential is traveling along the axon
in the direction A to B. When the action potential reaches A assume the
peak membrane potential is +40 mV (inside with respect to outside). At B
assume the membrane is still at a resting potential of –60 mV (inside with
respect to outside). The measured extracellular potential difference, VB –
VA, under these conditions is:
A. –100 mV
B. –40 mV
C. +60 mV
D. +100 mV
E. +40 mV

128
Excitable Cells 2 - Dr. Logothetis

5. The following statement is true about the threshold of the action potential

A. VR is an unstable point (meaning that any perturbation away from VR)


drives the membrane potential farther away from VR
B. Threshold refers to the potential where Na+ channels start to open
C. An increase in the K conductance raises the threshold
D. At the threshold potential the Na+ current is larger than the K+ current

129
Excitable Cells 2 - Dr. Logothetis

Answers to practice questions:

1. B. (The time constant = capacitance x resistance (i.e. τ = CmRm)).

2. C. (The length constant is proportional to the square root of the fiber


diameter, so

λ2/λ1=(d2/d1)0.5 . So d2 = d1(λ2/λ1)2 = (1)(2/1)2 = 4 :m.

3. C. (In between the nodes of Ranvier the amplitude of the propagating


action potential falls because no depolarizing current can cross the
membrane in the myelinated regions. If the nodes are too far apart, the
potential may fall below threshold by the time it reaches a successive node
and propagation will stop.)

4. D. (The extracellular potential at A is –40 mV (outside with respect to


inside), while at B it is +60 mV (again, outside with respect to inside). The
measured potential difference, VB – VA, is therefore, +60 - (-40) = +100
mV.

5. C. An increase in K+ conductance will drive the membrane potential


towards EK and will require greater inward current to excite a cell, which
means that the threshold for action potential generation will have been
raised.

130
Neuromuscular Transmission - Dr. Logothetis

Neuromuscular Transmission
Diomedes E. Logothetis, Ph.D.
(Dr. DeSimone’s lecture notes revised)

Learning Objectives:

1. Know the subunit composition of nicotinic ACh channels, general topology of the
α subunits and basic i/v characteristics.
2. Sketch the important structures in a motor end plate giving the relative structures
of the presynaptic terminal axon and the postsynaptic sarcolemma including: the
presynaptic active zones, synaptic cleft, and postsynaptic junctional folds.
3. State the reason for the 1 msec delay in the time between the arrival of an action
potential at the terminal presynaptic region and the first postsynaptic electrical
activity.
4. State the function of ACh in the causal chain leading to the end plate potential
(EPP).
5. State the mechanism by which the use of curare permits the separation of the EPP
from the postsynaptic action potential and name the active agent in curare.
6. State the effect of ACh binding to the ACh receptor on the Na+ and K+
permeability of the ACh receptor.
7. State the most important mode of ACh removal from the synaptic cleft.
8. State the effect acetylcholine esterase inhibitors on the amplitude and duration of
the EPP.
9. State the experimental results that indicate that the ACh receptor is a ligand-gated
nonselective cation channel.
10. State the effect of depolarization on the Ca2+ permeability of the terminal
presynaptic region.
11. State the effect of Ca2+ removal from the end plate region on the EPP.
12. Describe the experiment that proved the effect of Ca2+ during neuromuscular
transmission is on the presynaptic side of the end plate.
13. Discuss the significance of MEPPs for ACh release.
14. Know the general activation mechanism for NMDA and non-NMDA channels
and role in the physiology of central neurons (e.g. LTP).

Suggested Reading:

• See Lecture Notes and

• Physiology, 3rd Edition, L.S. Costanzo, Saunders, pp 24-28

131
Neuromuscular Transmission - Dr. Logothetis

Neurotransmitter Receptors

The diversity of neurotransmitters is extensive, but their receptors can be grouped into
two broad classes: ligand-gated ion channels and G protein-coupled receptors. By far the
most-studied receptor is the muscle nicotinic acetylcholine receptor, the first ligand-gated
ion channel to be purified, cloned, and characterized at the molecular level. The structure
and mechanism of this receptor are understood in considerable detail, and it provides a
paradigm for other neurotransmitter-gated ion channels. When activated, these receptors
induce rapid changes, within a few milliseconds, in the permeability and potential of the
postsynaptic membrane. In contrast, the postsynaptic responses triggered by activation of
G protein-coupled receptors occur much more slowly, over seconds or minutes, because
these receptors regulate opening and closing of ion channels indirectly.

Opening of Acetylcholine-Gated Cation Channels Leads to Muscle Contraction

The nicotinic acetylcholine receptor, a ligand-gated cation channel, admits both K+ and
Na+. Although found in some neurons, this receptor is best known for its role in synapses
between motor neurons and skeletal muscle cells. Patch-clamping studies on isolated
outside-out patches of muscle plasma membranes have shown that acetylcholine causes
opening of a cation channel in the receptor capable of transmitting 15,000-30,000 Na+ or
K+ ions a millisecond.

Since the resting potential of the muscle plasma membrane is near Ek, the potassium
equilibrium potential, opening of acetylcholine receptor channels causes little increase in
the efflux of K+ ions; Na+ ions, on the other hand, flow into the muscle cell. The
simultaneous increase in permeability to Na+ and K+ ions produces a net depolarization to
about –15mV from the muscle resting potential of –85 to –90 mV. This depolarization of
the muscle membrane generates an action potential, which – like an action potential in a
neuron – is conducted along the membrane surface via voltage-gated Na+ channels.
When the membrane depolarization reaches a specialized region, it triggers Ca2+
movement from its intracellular store, the sarcoplasmic reticulum, into the cytosol; the
resultant rise in cytosolic Ca2+ induces muscle contraction.

Two factors greatly assisted in the characterization of the nicotinic acetylcholine receptor.
First, this receptor can be rather easily purified from the electric organs of electric eels
and electric rays; these organs are derived from stacks of muscle cells (minus the
contractile proteins) and thus are richly endowed with this receptor. (In contrast, this
receptor constitutes a minute fraction of the total membrane protein in most nerve and
muscle tissues). Second, α-bungarotoxin, a neurotoxin present in snake venom, binds
specifically and irreversibly to nicotinic acetylcholine receptors. This toxin can be used
in purifying the receptor by affinity chromatography and in localizing it. For instance, in
autoradiographs of muscle-cell sections exposed to radioactive α-bungarotoxin, the toxin
is localized in the plasma membrane of postsynaptic striated muscle cells immediately
adjacent to the terminals of presynaptic neurons.

132
Neuromuscular Transmission - Dr. Logothetis

Careful monitoring of the membrane potential of the muscle membrane at a synapse with
a cholinergic motor neuron has demonstrated spontaneous, intermittent, and random ~2-
ms depolarizations for about 0.5-1.0 mV in the absence of stimulation of the motor
neuron. Each of these depolarizations is caused by the spontaneous release of
acetylcholine from a single synaptic vesicle. Indeed, demonstration of such spontaneous
small depolarizations led to the notion of the quantal release of acetylcholine (see below)
and thereby led to the hypothesis of vesicle exocytosis at synapses. The release of one
acetylcholine-containing synaptic vesicle results in the opening of about 3000 ion
channels in the postsynaptic membrane, far short of the number need to reach the
threshold depolarization that induces an action potential. Clearly, stimulation of muscle
contraction by a motor neuron requires the nearly simultaneous release of acetylcholine
from numerous synaptic vesicles.

All Five Subunit in the Nicotinic Acetylcholine Receptor Contribute to the Ion
Channel
The non-selective nAChR channel gives an equilibrium (or reversal) potential at 0 mV
and its current-voltage relationship is ohmic.

Figure 1. I-V
relationship of
nACR.

The acetylcholine receptor from skeletal muscle is a pentameric protein with a subunit
composition of α2βγδ.

Each molecule has a diameter of about 9nm and protrudes about 6nm into the
extracellular space and about 2nm into the cytosol (Figure 11-36). The α, β, γ, and δ
subunits have considerable sequence homology; on average, about 35-40 percent of the
residues in any two subunits are similar. The complete receptor has a five-fold
symmetry, and the actual cation channel is a tapered central pore, with a maximum
diameter of 2.5 nm, formed by segments from each of the five subunits (Figure 11-36).

Figure 2. Subunit composition and location of pore and gate within the α subunit.

133
Neuromuscular Transmission - Dr. Logothetis

The channel opens when the receptor cooperatively binds two acetylcholine molecules to
sites located at the interfaces of the αδ and αγ subunits. Once acetylcholine is bound to a
receptor, the channel is opened virtually instantaneously, probably within a few
microseconds. Studies measuring the permeability of different small cations suggest that
the open ion channels is, at its narrowest, about 0.65-0.80 nm in diameter, in agreement
with estimates from electron micrographs. This would be sufficient to allow passage of
both Na+ and K+ ions with their bound shell of water molecules.

Although the structure of the central ion channel is not known in molecular detail, much
evidence indicates that it is lined by five transmembrane M2 α helices, one from each of
the five subunits. The M2 helices are composed largely of hydrophobic or uncharged
polar amino acids, but negatively charged aspartate or glutamate residues are located at
each end, near the membrane faces and several serine or threonine residues are near the
middle. If a single negatively charged glutamate or aspartate in one subunit is mutated to
a positively charged lysine, and the mutant mRNA is injected together with mRNAs for
the other three wild-type subunits into frog oocytes, a functional channel is expressed, but
its ion conductivity – the number of ions that can cross it during the open state – is
reduced. The greater the number of glutamate or aspartate residues mutated (in one or
multiple subunits), the greater that reduction in conductivity. These findings suggest that
aspartate and glutamate residues – one residue from each of the five chains – form a ring
of negative charges on the external surface of the pore that help to screen out anions and
attract Na+ or K+ ions as they enter the channel (see Figure 11-36). A similar ring of
negative charges lining the cytosolic pore surface also helps select cations for passage.

The two acetylcholine-binding sites in the extracellular domain of the receptor lie ~4 to 5
nm from the center of the pore. Binding of acetylcholine thus must trigger
conformational changes in the receptor subunits that can cause channel opening at some
distance from the binding sites. Receptors in isolated postsynaptic membranes can be
trapped in the open or closed state by rapid freezing in liquid nitrogen. Images of such
preparations suggests that the five M2 helices rotate relative to the vertical axis of the
channel during opening and closing.

Synaptic transmission

Excitable cells communicate with each other by a process called synaptic transmission.
This occurs at specialized junctions called synapses connecting the communicating cells.
There are two types of synapses: electrical and chemical. Electrical synapses are formed
when cells are connected by structures called gap junctions. Gap junctions contain ion
channels that permit ion currents to flow between cells, so a depolarizing current in one
cell is easily transmitted to its neighbors. Such structures are prominent in cardiac muscle
and visceral smooth muscle where they coordinate excitation and contraction over an
entire organ. However, most communication between neurons, and between neurons and
skeletal muscles occurs through chemical synapses. The neuromuscular junction,
important in its own right, is one of the more thoroughly studied chemical synapses and is
prototypical of many other synapses.

134
Neuromuscular Transmission - Dr. Logothetis

Structure of neuromuscular junction

A. From Spinal Cord to Muscle Fibers – Spinal motor neurons have their cell bodies
in the ventral horn of the spinal cord. From there they send out long myelinated
axon fibers that contact individual muscle fibers (See figure below).

B. Motor End Plate – The fibers then lose their continuous myelin sheath and further
branch making synaptic contacts with each muscle cell. The synaptic region is
called the motor end plate. There is a distinct space between the presynaptic
terminal axon and the postsynaptic sarcolemma, this is called the synaptic cleft
(see figure below). In a region of the presynaptic axon called the active zone, the
cytoplasm contains clusters of dense synaptic vesicles. Opposite each of the active
zones the postsynaptic muscle membrane invaginates forming junctional folds.
The junctional folds contain a high density of the postsynaptic acetylcholine
(ACh) receptors. The synaptic cleft also contains a matrix of fibrous material to
which molecules of the enzyme acetylcholinesterase (AChE) are bound (see
figure 3).

Figure 3.

135
Neuromuscular Transmission - Dr. Logothetis

End plate potential (epp)

A. Sequence of events leading to ACh release – ACh is the excitatory


neurotransmitter at the neuromuscular junction. The events leading to its release
have been well-researched using a preparation consisting of a few skeletal muscle
fibers with the terminal axon of the motor neuron still attached to the muscle (i.e.
the motor end plate region is left intact). The isolated nerve-muscle preparation
(see Fig. 2) is immersed in a bath of oxygenated Ringer’s solution (an isotonic
solution containing the major ions found in the extracellular compartment). A
stimulator (a current source that can deliver depolarizing current to the nerve at
given intensities and frequencies) makes contact with the motor axon, and a
microelectrode impales the muscle cell allowing for the measurement of muscle
cell intracellular potentials. The following events (see Fig. 5, part A) are recorded
when the axon is stimulated:

Figure 4.

136
Neuromuscular Transmission - Dr. Logothetis

Figure 5.

1. An action potential travels down the axon to the presynaptic terminus.


2. This causes the release of ACh from the presynaptic neuron.
3. ACh then diffuses across the synaptic cleft and binds to ACh receptors on the
postsynaptic muscle cell membrane.
4. This leads to an action potential recorded on the postsynaptic side (i.e. the muscle
cell). Note in part A in the Figure there is a time delay (about 1 msec) between the
application of the stimulus to the axon and the first recorded change in potential in
the muscle. This delay is the time needed to propagate the action potential to the
terminal axon plus the time needed for ACh to diffuse across the synaptic cleft (a
distance of about 50 nm). Since the action potential propagates along the axon at a
rate of about 20 m/sec, virtually all of the synaptic delay is in the time required
for exocytosis and diffusion of ACh. The synaptic clefts can’t be much wider than
50 nm. If they are, the synaptic delay becomes intolerably long, and smooth
neuromuscular transmission is impaired.

B. Separating the EEP from the Action Potential – Careful examination of the
postsynaptic potential changes shows 2 components:

1. A slowly rising depolarizing potential between -90 mV (muscle resting potential)


and about -50 mV that begins to plateau at -50 mV. This is essentially the EPP.
2. Above -50 mV the slope of the potential increases and an action potential occurs.
So -50 mV is approximately the threshold potential for the muscle action
potential.

The question then becomes: How can the EPP be separated from the action
potential, so it can be better-studied? A specific blocker of the EPP is needed. A plant
alkaloid mixture called curare contains such a blocker. If curare is ingested or gets
into a wound, it can cause skeletal muscles to become paralyzed. The active

137
Neuromuscular Transmission - Dr. Logothetis

substance in the mixture is D-tubocurarine. It acts by competing with ACh for ACh
receptor binding sites. In the above figure (part B) a low dose of curare was added to
the bath before stimulating. We note that the EPP rises more slowly, but still manages
to reach threshold, so an action potential follows. However, at a higher dose, the EPP
is further reduced, so it fails to reach threshold (part C). Note curare blocks the EPP
but not action potentials per se.

C. Propagation of the EPP– The EPP arises as a consequence of ACh binding to its
postsynaptic receptor, so it originates only at the neuromuscular junction. Its
“job” is to bring the sarcolemma to threshold potential so action potentials can
propagate over the muscle, the precursor event in the process of muscle
contraction. In curare-treated muscles the subthreshold EPP behaves like any
electrotonic potential. It has its maximum amplitude at the postsynaptic end plate
and declines exponentially with distance from the synaptic region.

Mechanism of action of ACh

A. Sequence of Events
1. Nerve action potential reaches the motor neuron presynaptic terminus.
2. ACh is released into the synaptic cleft.
3. ACh diffuses to the postsynaptic membrane where it binds to ACh receptors
4. The permeability of the postsynaptic end plate membrane to both Na+ and K+
increases causing the postsynaptic membrane to depolarize. This is the EPP and
has a peak depolarization of 40-50 mV.
5. The threshold for the initiation of an action potential is reached in the
sarcolemma.
6. The action potential propagates along and into the muscle, initiating excitation-
contraction coupling.
7. The muscle developments tension.

B. Time Course of the EPP – The EPP rises to a peak within 2-3 msec and then
spontaneously decays away. The key factor determining the time course of the
EPP is the time course of the changes in ACh concentration in the synaptic cleft.
The factors controlling the ACh concentration are: 1) the amount released
presynaptically and 2) the rate at which ACh is removed from the synaptic cleft.
The factors that determine the rate of removal of ACh are: 1) diffusion away from
the synaptic cleft and, 2) more importantly chemical breakdown of ACh mediated
by AChE.

C. Effect of AChE on EPP – The importance of AChE in removing ACh from the
synaptic cleft is best appreciated when its effects are inhibited. Blockers of AChE
activity, such as physostigmine or neostigmine, cause an EPP of greater magnitude
and duration and thus potentiate the effects of ACh. That is the reason these
anticholinesterases are used to potentiate neuromuscular transmission in diseases such
as myasthenia gravis.

138
Neuromuscular Transmission - Dr. Logothetis

Figure 6.

Ionic Basis of the EPP – The ACh receptors (AChR) are ligand gated ion
channels. The ligand is ACh. When ACh binds to ACh receptors the
channel permeability increases to both Na+ and K+, i.e. the AChR is
clearly not the same as the voltage gated Na+ channel. The permeability of
the AChR is, therefore, unaffected by TTX. The key experiments in
proving that the AChR functions as a nonselective monovalent cation
channel were done by Sir Bernard Katz and his colleagues. They formed
the following hypothesis: If the AChR channel is nonselective between
Na+ and K+, then in the motor end plate the membrane potential should go
toward approximately 0 mV when ACh binds to the AChRs. They used
the isolated nerve-muscle preparation (see above), but with an additional
current passing electrode impaling the muscle (see Fig. below).

Figure 7.

When no current is passed through the current electrode, the membrane


potential is the muscle resting potential of -80 mV. When the stimulator
excites the motor axon, the EPP is seen to be a depolarizing potential.
Curare is added to prevent action potentials. The peak EPP does not
actually reach zero mV because other channels (that are ACh insensitive)
prevent the EPP from becoming depolarized to that extent. The key

139
Neuromuscular Transmission - Dr. Logothetis

observation is made when a depolarizing current is passed bringing the


membrane potential artificially to 0 mV. At 0 mV when the nerve is
stimulated no EPP is recorded. This must mean that 0 mV is the reversal
potential for the EPP. That can only occur if the AChR channel is passing
nearly equal amounts of Na+ and K+ in opposite directions. This fact
confirms the hypothesis that the AChR channel is nonselective between
Na+ and K+. This is further confirmed by passing enough current to fix the
membrane potential at +30 mV. Stimulation of the nerve causes the
potential to again move toward zero, i.e. toward the reversal potential, but
this time from the opposite direction.

Mechanism of ACh release from the presynaptic nerve terminal

A. Na+ and K+ are not directly involved in the presynaptic release of ACh –
The presynaptic nerve terminal must be depolarized in order for ACh to be
released from the nerve terminal. This led to the hypothesis that perhaps
slight changes in the Na+ or K+ concentration in the presynaptic terminal
(as occur naturally during an action potential) were the proximal cause of
ACh release. However, it is possible to depolarize the presynaptic nerve
terminal by passing current into it, and record postsynaptic EEPs even in
the presence of TTX and tetraethylammonium (TEA) which respectively
block Na+ and K+ channels. This rules out direct roles for Na+ and K+ in
ACh release.

140
Neuromuscular Transmission - Dr. Logothetis

Figure 8.

B. The effect of Ca2+ on the presynaptic nerve terminal is critical – It had


been well known that reducing Ca2+ in the extracellular medium interfered
with synaptic transmission. However, the role of extracellular Ca2+ was
greatly clarified by Katz and Miledi. They used the nerve-muscle prep, but
this time the external bathing medium contained no
2+
Ca . They positioned a micropipette containing CaCl2 near the
presynaptic terminal. The micropipette also contained a Ag/AgCl
electrode into which positive current could be injected at the throw of a
switch (see Part A of Fig. above). This enabled them to deliver a small
quantity of Ca2+ ions into the synaptic region. (When ions are applied this
way in a physiological experiment, the method is called iontophoresis).
The results are shown in Part B of the figure. In the column labeled “1"
the presynaptic nerve was stimulated, but no Ca2+ was allowed to flow
into the synapse. The result is no postsynaptic potential (EPP) was
recorded. In column 2 a pulse of Ca2+ was allowed to flow onto the
presynaptic terminal region just prior to nerve stimulation. The result is
that a postsynaptic potential (EPP) was recorded. In column 3 the nerve
was first stimulated briefly and then a small quantity of Ca2+ was released
into the the presynaptic terminal region. The result again was no
postsynaptic potential (EPP). These experiments prove that Ca2+ must be

141
Neuromuscular Transmission - Dr. Logothetis

present to permit ACh to be released from the presynaptic nerve terminal.


Note that the negative results in column 3 prove that the effect of Ca2+ is
on the presynaptic side and not the postsynaptic side.

C. The Ca2+ influx hypothesis for transmitter release – On the basis of the
above results Katz and his coworkers proposed that Ca2+ must flow into
the presynaptic terminal in order for transmitter (ACh in this case) to be
released. Normally the extracellular Ca2+ concentration greatly exceeds
the intracellular concentration, so an electrochemical ion gradient for Ca2+
entry exists. However, the Ca2+ permeability of the membrane is normally
very low. Katz et al. suggested that when an action potential depolarizes
the presynaptic terminal region, that voltage gated Ca2+ channels become
conducting. This increases the Ca2+ permeability allowing Ca2+ to flow
into the terminal. This somehow permits the release of ACh. This
hypothesis (which proves to be correct) is summarized in the Fig. below.

Figure 9.

D. Ca2+ dependent ACh release occurs in quantal units – After many years of
research we now know that in the presynaptic nerve terminal ACh is
“packaged” in vesicles (each containing roughly 10,000 ACh molecules).
When the terminal is depolarized Ca2+ enters and binds to the outside of
the vesicles. This promotes their fusion with the axolemma and the entire
contents of the vesicles are emptied into the synaptic cleft. The process is
called exocytosis. So ACh gets placed into the synaptic cleft in units of
10,000 molecules. These units were originally referred to as “quanta”
before the details of exocytosis were fully known. Sir Bernard Katz and
his colleagues performed the seminal physiological experiments that led to
conclusion that neurotransmitters must be released in quantal amounts.
The morphological evidence for exocytosis followed. In effect Katz et al.
observed that even when the motor nerve is not being stimulated, low
amplitude potentials can be recorded in the postsynaptic region of the
muscle. These potentials had a maximum amplitude of about 0.4 mV and
occurred roughly every 50 msec. They were unique to the motor end plate
region, were diminished in magnitude by curare and enhanced by
neostigmine, i.e. they behaved like EPPs except they were much smaller

142
Neuromuscular Transmission - Dr. Logothetis

and they did not require nerve stimulation to be observed. Katz called
them miniature EPPs or MEPPs. The fig below shows traces of MEPPs.
in part A. In part B the histogram shows the amplitudes of the MEPPs are
normally distributed about a mean value of 0.4 mV. It seems clear that the
MEPPs are due to random ACh release. The question now is: how many
ACh molecules in the synaptic region are required to produce a MEPP of
0.4 mV? By using iontophoresis Katz et al. were able to add known
amounts of ACh to the synaptic cleft. They found that they could get
MEPPs much smaller than 0.4 mV. So it was clear that MEPPs of 0.4 mV
were not the result of just a few ACh molecules. They were able to deduce
that 0.4 mV corresponded to about 10,000 molecules in the volume of the
synaptic cleft. So MEPPs were the result of the release of ACh in unit
amounts of 10,000, which is about the number of ACh molecules in the
average presynaptic vesicle.

Figure 10.

E. The relation between MEPPs and EPPs – MEPPs occur spontaneously,


i.e. with unstimulated nerve. What is their relation to EPPs, which occur
following nerve stimulation? Katz’s underlying hypothesis here was that
when the motor nerve is stimulated large numbers of ACh quanta were
released in a very short time because they were mobilized by Ca2+ influx.
So in effect the EPP should be the sum of many MEPPs. To try to control

143
Neuromuscular Transmission - Dr. Logothetis

the number of quanta released so they might be counted discretely, Katz et


al. used the usual isolated nerve-muscle preparation, but they kept the Ca2+
low. The results of the study are shown in the fig below. They stimulated
the motor axon and recorded the amplitude of the evoked EPP that
followed. They found that on the average the amplitudes of the evoked
EPPs were multiples of the 0.4 mV. This meant that indeed the EPP is
made up of a sum of 0.4 mV unit-MEPPs. Since under conditions of
normal Ca2+ concentrations, EPPs are about 50 mV in magnitude, this
means that normal depolarization of the nerve causes the rapid release of
at least 100 quanta. Since a quantum is now known to be the same as a
fused vesicle, we conclude that an EPP is the result of the fusion of 100 or
more vesicles with the presynaptic membrane and the release of about 1
million ACh molecules into the synaptic cleft.

144
Neuromuscular Transmission - Dr. Logothetis

Figure 11.

145
Neuromuscular Transmission - Dr. Logothetis

Summary Table

Properties of Action Potentials and Synaptic Potentials at the Skeletal Neuromuscular


Junction
Synaptic Potential
Action Potential
(End-Plate Potential)
Changes in membrane Initiated by depolarization (the end-
Initiated by ACh
conductance plate potential)
During rising phase Simultaneous chemically gated Specific voltage-dependant increase
(initiation) increase in gNa and gK in gNa
Passive decay; diffusion and Specific increase in gK and decrease
During falling phase
AChE 10-20 msec in gNa 1-3 sec
Duration 10-20 msec 1-3 msec
Equilibrium potential Reversal potential close to 0 mV ENa (+55 mV)
Blocked by curare; enhanced by Blocked by TTX, but not by curare;
Pharmacology
neostigmine; not blocked by TTX not affected by neostigmine
Propagation Propagates with decrement All-or-nothing
Regenerative rise, followed by
No evidence for regenerative
Other features absolute and relative refractory
action or refractory period
periods

146
Neuromuscular Transmission - Dr. Logothetis

Practice questions

1. The end plate potential:

A. is specifically blocked by tetrodotoxin

B. is specifically blocked by tetraethylammonium


C. has a reversal potential close to 0 mV
D. has a reversal potential of +60 mV
E. is due to Ca2+ influx into the postsynaptic membrane

2. Removing Ca2+ from the end plate region:

A. has no effect on the end plate potential


B. enhances the acetylcholinesterase activity
C. inhibits the release of acetylcholine
D. starts an action potential in the motor end plate
E. increases the amplitude of miniature end plate potentials.

Answers

1. C (Because the postsynaptic membrane becomes equally permeable to Na+


and K+)
2. C (Exocytosis of ACh vesicles requires a Ca2+ influx to cause their fusion
with the presynaptic membrane and release of ACh into the synaptic cleft)

APPENDIX

Ca2+-permeable channels

There are a number of channels that when gated open by a number of different
mechanisms allow Ca2+ ions to move down their steep electrochemical gradient into
cells. Given the critical importance of intracellular Ca2+ as a second messenger, Ca2+
entry exerts important effects on cellular function. Three major categories of Ca2+
channels are: a) the extracellular ligand-gated Glutamate receptors (in particular the N-
methyl D-Aspartate or NMDA type); b) the voltage-gated Ca2+ channels and c) Transient
Receptor Potential (TRP) channels found in sensory cells. Here we will only briefly
mention the NMDA receptor channels.

147
Neuromuscular Transmission - Dr. Logothetis

Two Types of Glutamate-Gated Cation Channels May Function in a Type of


“Cellular Memory”

Figure 12. Glutamate receptor activation results in Ca2+ entry into neuronal
cells.

148
Neuromuscular Transmission - Dr. Logothetis

The hippocampus is the region of the mammalian brain associated with many types of
short-term memory. Certain types of hippocampal neurons, here simply called
postsynaptic cells, receive inputs from hundreds of presynaptic cells. In long-term
potentiation a burst of stimulation of a post-synaptic neuron makes it more responsive to
subsequent stimulation by presynaptic neurons. For example, stimulation of a
hippocampal presynaptic nerve with 100 depolarizations acting over only 200
milliseconds causes an increased sensitivity of the postsynaptic neuron that lasts hours to
days. Changes in the responses of postsynaptic cells may underlie certain types of
memory.
Two types of glutamate-gated cation channels in the postsynaptic neuron participate in
long-term potentiation. Unlike other neurotranmitter-gated ion channels, both glutamate
receptors have four subunits, each containing a pore-lining M2 helix; both are excitatory
receptors, causing depolarization of the plasma membrane when activated. Because the
two receptors were initially distinguished by their ability to be activated by the non-
natural amino acid N-methyl-D-aspartate (NMDA), they are called NMDA glutamate
receptors and non-NMDA glutamate receptors.

As illustrated in Figure 12, non-NMDA receptors are “conventional” in that binding of


glutamate, released from the presynaptic cell, triggers their opening. NMDA glutamate
receptors are different in two key respects. First, they allow influx of Ca2+ as well as
Na+. Second, and more important, two conditions must be fulfilled for the ion channel to
open: glutamate must be bound and the membrane must be partly depolarized. In this
way, the NMDA receptor functions as a coincidence detector; that is, it integrates
activity of the postsynaptic cell – reflected in its depolarized plasma membrane – with
release of neurotransmitter from the presynaptic cell, generating a cellular response
greater than that caused by glutamate release alone. Once a post-synaptic cell becomes
“sensitized”, it takes fewer action potentials in the presynaptic neurons to induce a given
depolarization in the postsynaptic neuron; in other words, the synapse “learns” to have an
enhanced response to signals from the presynaptic cells.

Opening of NMDA receptors depends on membrane depolarization because of the


voltage-sensitive blocking of the ion channel by a Mg2+ ion from the extracellular
solution. A small depolarization of the membrane causes the Mg2+ ion to dissociate from
the receptor, thereby making it possible for glutamate binding to open the channel.
Mutagenesis of a single asparagine residue in the pore-lining M2 helix of the NMDA
receptor abolishes the effect of Mg2+, indicating that Mg2+ binds in the channel.

Since activation of a single synapse, even at high frequency, generally causes only a
small depolarization of the membrane of the postsynaptic cell, long-term potentiation is
induced only when many synapses simultaneously stimulate a single postsynaptic neuron.
Thus, the requirements for membrane depolarization explains why long-term potentiation
depends on the simultaneous activation of a large number of synapses on the postsynaptic
cell.

149
Skeletal Muscle Mechanics - Dr. Feher

Skeletal Muscle Mechanics


Joseph Feher, Ph.D.
LECTURE OUTLINE:
I. Introduction
A. Muscles either shorten or produce force.
B. Muscles perform diverse functions.
C. Muscles can be classified according to fine structure, neural control and
anatomy.

II. Isometric Force is measured while keeping muscle length constant.


A. Experimental Set-up requires a rigid device for measuring force.
B. The muscle twitch depends on the kind of muscle and is longer than the
action potential.
1. Slow twitch fibers
2. Fast twitch fibers
3. The action potential is a trigger.

C. Muscle force is determined by the number of motor units that are


recruited.
D. Muscle force is determined by the frequency of activation.
E. Muscle force is determined by the length of the muscle.
1. Passive force
2. Active force

III. Isotonic Force is measured while keeping muscle force constant.


A. Experimental Set-up requires an afterload.
B. The twitch here consists of an isometric and isotonic part.
C. The velocity of muscle contraction varies inversely with the force.
D. Muscle power varies with the speed of contraction, load, and muscle type.
1. Power is force x velocity

150
Skeletal Muscle Mechanics - Dr. Feher

2. Muscle power peaks at about one-third maximum force


3. Muscle power is 2-3 times greater in fast twitch fibers
4. Muscle power peaks at about one-third maximum velocity

E. Muscles can lengthen during contraction when loaded.


1. Lengthening is a negative velocity: eccentric and concentric
contractions
2. Eccentric contractions exert more force
3. Concentric and eccentric contractions have different functions

IV. Muscle architecture influences force and velocity of the whole muscle

A. Muscle force is usually proportional to muscle cross-sectional area


B. Muscle fibers can be oriented at an angle to the direction of force
C. Pinnation increases muscle force but decreases muscle velocity

V. Fatigue is the decrease in force upon repeated stimulation of muscle.


A. Fatigue is produced within a few seconds of maximal stimulation
B. Fatigue of slowly twitch muscles takes longer
C. Muscles can be classified according to their contractile properties
D. Most muscles are heterogeneous mixtures of different muscle fiber types
OBJECTIVES:
1. Distinguish between isometric and isotonic force
2. Recall the approximate twitch times of skeletal muscle and distinguish between
fast and slow twitch muscles
3. Define the term “motor unit”
4. Describe what is meant by “recruitment”
5. Describe the size principle
6. Calculate the lowest frequency at which skeletal muscle begins to show
summation

151
Skeletal Muscle Mechanics - Dr. Feher

7. Distinguish between active force and passive force when muscles are stretched
8. List three ways of grading force and describe their physiological role
9. Describe the relation between force and velocity
10. Describe the force and velocity at which maximum power output occurs
11. Distinguish between concentric, isometric and eccentric contractions
12. Describe the relation between muscle size and force
13. Describe the effect of pinnation on muscle force and velocity
14. Define fatigue
15. Distinguish muscle types on the basis of contractile properties
Suggested Reading: Berne and Levy, pp.242-245; 236-237

I. INTRODUCTION

A. Muscles either shorten or produce force

The primary action of a muscle is to contract. The usual use of this word
means to shorten, but physiologists often use it to mean to activate the
muscle. Activation of the muscles can produce force without actually
shortening, as you might do when holding something really heavy. Also,
activation of muscles is used to decelerate motion, and under these
circumstances muscles can actually lengthen.

B. Muscles perform diverse functions

1. Muscles that are connected to the skeleton (skeletal muscles)


move one bone relative to another. These muscles allow us to lift
weights and to move ourselves from place to place. These muscles
also include the tongue, the muscles that move the eyeballs, and
the upper third of the esophagus.
2. Muscles surrounding hollow organs. (heart, urinary bladder, gall
bladder, uterus) develop tension in the walls that produces pressure
within the organ according to LaPlace’s Law. Pressure propels
material out of the organ.

152
Skeletal Muscle Mechanics - Dr. Feher

3. Muscles in long hollow tubes ( the GI tract, the arterioles, the


ureters and the airways) set the diameter or the length and may
propel material by peristalsis. These are generally smooth muscle.
4. Muscles produce heat when activated. This is the reason for
shivering thermogenesis.

C. Muscles can be classified according to fine structure, neural control, and


anatomical arrangement.

1. Muscles are striated or smooth


2. Anatomic classification includes skeletal, cardiac and visceral
muscles
3. Neural control is voluntary or involuntary.
CLASSIFICATION OF MUSCLE

Figure 1. Classification of muscle can be based on control properties, anatomical


properties, or the fine structure observed histologically. The preferred classification
is that highlighted in the figure, which is a mixture of several classification schemes.

II. ISOMETRIC FORCE is measured while keeping muscle length constant

A. The experimental set-up requires a rigid device for measuring force

153
Skeletal Muscle Mechanics - Dr. Feher

Figure 2. In this set-up, action potentials on the nerve are initiated by an external stimulator. The muscle is tied
at one end to a stiff force transducer. Because the shortening of the muscle is very small, this contraction is
called an isometric contraction. A single stimulation causes a single action potential in the nerve, then in the
muscle, and then a single contraction of the muscle, called a muscle twitch.
B. The time of the muscle twitches depends on the kind of muscle and is
longer than the action potential

1. A very brief (5 ms) stimulus causes a single muscle contraction called


a muscle twitch. Force rises and then falls without the muscle
shortening. The time for the force to rise is shorter than the time it
takes to fall. When a gastrocnemius muscle of a rat is used, the twitch
time is about 50 milliseconds. When the soleus muscle is used, the
twitch time is over 150 ms. Thus, muscles can be distinguished
because of their contractile properties, including their twitch times.
The gastrocnemius is an example of a fast-twitch muscle; the soleus
is a slow-twitch muscle.
2. The recording in Fig. 2 also shows that the action potential on the
muscle lasts only a few ms, whereas the twitch lasts much longer. The
action potential is a trigger for the muscle contraction.

C. Muscle force depends on the number of motor units that are activated

154
Skeletal Muscle Mechanics - Dr. Feher

1. When the strength of the stimulus is gradually increased, the force of


the twitch also increases until the force reaches a plateau, as shown in
Fig. 3. Two questions: why does it take volts to stimulate the muscle
and why does force increase with increasing strength of stimulation?

Figure 3. Increase in the muscle twitch with increased


recruitment of motor fibers by increasing the strength of the
external stimulus.
2. The motor nerve supplying the muscle is a bundle of motor neurons.
The extracellular stimulus depolarizes the axons by passing an inward
current across their membranes, but most of the current from the
extracellular electrode does not go across the axon membrane. Thus,
higher voltages are necessary to depolarize all of the axons in the
bundle. This is why volts are necessary for the external stimulus, when
only a few millivolts of membrane depolarization is needed to initiate
an action potential.
3. Each motor nerve branches and connects to a set of muscle fibers,
making a neuromuscular junction with each of them. The motor
neuron and all of the muscle fibers it innervates make up a motor
unit. The entire muscle consists of a large number (many thousands)
of muscle fibers that are each enclosed in a plasma membrane called
the sarcolemma. These are generally large, multinucleated cells. The

155
Skeletal Muscle Mechanics - Dr. Feher

cells are typically 20-100 μm in diameter and can be many cm long.


Each muscle fiber is typically innervated by a single motor neuron,
typically in the middle of the fiber. However, motor neurons typically
innervate more than one muscle fiber.

Figure 4. The motor unit consists of a motor neuron and all of the muscle fibers innervated by it.
Here motor neuron 1 innervates fibers A, B, and C; motor neuron 2 innervates fibers D and E. When
only motor neuron 2 fires an action potential, only fibers D and E contribute to the force developed
by the muscle. When only motor neuron 1 fires an action potential, only fibers A, B and C contribute
to the force. When both motor neurons fire an action potential, all of the muscle fibers contribute to
the force. Thus, the force is greater when more motor units are recruited.

Figure 5. Intact nerve and extracellular electrode that stimulates it. As the strength of the
stimulus increases, more axons are activated until, eventually, all axons fire action potentials.
Since each axon innervates a set of muscle fibers, called its motor unit, progressive activation of
axons causes progressive increases in the number of activated muscle fibers, and progressive
increases in the total force produced by the muscle.

156
Skeletal Muscle Mechanics - Dr. Feher

4. The increase in force with increasing number of motor units is called


recruitment.

a. People can grade muscle force by recruiting motor units.

We can vary the force a muscle produces more or less


continuously from small to large forces. This means that
muscle force is not all-or-none; it is graded. This variation is
brought about by recruiting progressively more and more
motor units until all of the motor units are activated. Although
the gradation is fine, it is not really continuous because muscle
fibers are activated in a discrete (as opposed to continuous)
way. Because there are so many muscle fibers, the force
appears to vary nearly continuously.

b. The motor units are recruited in order of their size. This is


called the size principle.

Large motor units are innervated by large motor neurons, and


smaller motor units are innervated by smaller motor neurons.
The small motor neurons are more excitable, so these are
recruited first. This makes subjective sense. Delicate
movements that require dexterity but little force are
accomplished by recruiting small numbers of muscle fibers.
Gross motor movements that involve a lot of force use large
increments of force by recruiting successively larger motor
units.
D. Muscle force can be graded by the frequency of motor neuron firing.

1. The action potential is short compared to the muscle twitch.

The action potential on the motor neuron is very short, 1-2 ms.
Similarly, the action potential on the muscle cell membrane is also
short, on the order of 3-5 ms. The muscle twitches are long by
comparison, some 30 -200 ms, depending on the muscle type.
Therefore, we can stimulate the muscle with another action potential
before the muscle has relaxed. Indeed, we can stimulate a muscle
again before it reaches its peak tension. What happens when we do
this? Fig.6 shows the results of varying the frequency of stimulation at
maximum recruitment.

157
Skeletal Muscle Mechanics - Dr. Feher

Figure 6. Gradation of muscle force by the frequency of stimulation.


2. Repetitive stimulation of muscle results in summation of force.

Fig. 6 shows that when a muscle is stimulated before it has completely


relaxed, there is a new twitch that begins force where the first twitch
left off, resulting in a greater force. Thus, muscle force summates with
repetitive stimulation. When the frequency of stimulation is great
enough, the muscle produces a single forceful contraction during the
period of stimulation, with no waviness in the force. This condition is
called tetanus.

3. The tetanic frequency varies with muscle type.

The frequency required to reach tetanus depends on how fast the


muscle contracts and relaxes. If the muscle twitch is 100 ms long, then
summation will just begin when the next stimulation arrives at the end

158
Skeletal Muscle Mechanics - Dr. Feher

of relaxation. Since there are 10 100 ms intervals in a second,


summation for such a fiber should begin at a stimulation frequency of
about 10 Hz. (Hz is a Hertz, meaning a cycle per second; 10 Hz is
therefore 10 cycles or events per second). Typically most muscles in
the human tetanize between 20 and 100 Hz. The lowest frequency that
shows summation is the inverse of the twitch time: f = 1/p, where f is
the frequency and p is the period, or duration, or the twitch.

4. People can grade muscle force by the frequency of motor neuron


firing.

In addition to recruitment, summation is one of the main ways we have


of grading muscle force. We grade the force of a muscular contraction
by altering the frequency of motor neuron firing.

5. Tetanic force is much greater than the twitch force.

As you can see from Fig. 6, the tetanic force is much greater than the
twitch force. In general, tetanic force is about 5 times larger than the
twitch force, but the tetanus/twitch force ratio varies from about 2 to
10 in different muscles.
E. Muscle force also depends on the length of the muscle

1. Stretching a muscle produces a passive force.

The device shown in Figs. 2, 3 and 6 can be adjusted to vary the length
of the muscle. When muscles are relaxed, they exert no force.
However, when they are stretched without activation they produce a
passive force. This is due to elastic properties of the muscle material
itself and not to the development of force that depends on activation of
the muscle. This passive force increases steeply with increases in
length. The dependence of the passive force on the length of the
muscle is curvilinear.

2. The active tension rises and then falls with stretch of the muscle.

When the muscle is stimulated tetanically, it produces a force in


addition to the passive force. This force is called the active tension
because it depends on activation of the muscle by the stimulus. The
additional active force produced by stimulation depends on the length
of the muscle. The active force added by stimulation increases to a
maximum and then declines with further increases in muscle length.

159
Skeletal Muscle Mechanics - Dr. Feher

The relation between active force and muscle length is the length-
tension curve.

Figure 7. The length-tension relationship in muscle.


3. Physiologically, muscles operate near the maximum of their length-
tension curve.

Muscles have a resting length, L0, that is typically at the top of the
active length-tension curve, and close to the zero point of the passive
length-tension curve. This means that antagonist muscles pairs do not
fight against each other. Because muscles are attached to the skeleton,
their shortening is defined by the movement of the bones and the
origins and insertions of the muscles. Most muscles do not shorten or
lengthen by more than about 30% of their rest length. For this reason,
length is not an important variable in determining muscle strength
compared to recruitment and frequency of stimulation.
4. There are three ways to vary muscle force: recruit muscle fibers,
vary the frequency of activation, and vary the length. Of these,
recruitment and varying the frequency of activation are
physiologically most important.

160
Skeletal Muscle Mechanics - Dr. Feher

III. ISOTONIC FORCE is measured when the muscle exerts a constant force.

A. The experimental set-up requires an afterload.

The afterload is a weight supported by a shelf prior to activation of the


muscle. Thus, the muscle does not feel the afterload until after the muscle
has begun to contract.

Figure 8. Experimental set-up for measuring the force and velocity of isotonic contractions.
B. The twitch in an afterloaded muscle is divided into two parts: an isometric
part and an isotonic part.

When the muscle is stimulated, it cannot shorten until it produces a force


equal to the afterload. It takes some time for the muscle to develop this
force, and during this time it doesn’t shorten - the contraction is isometric.
After the muscle develops a force equal to the afterload, it lifts the
afterload and continues to shorten, keeping an approximately constant
force while it shortens. Because the force is constant during this interval,
the contraction is called an isotonic contraction.

C. The velocity of muscle contraction varies inversely with the afterload.

Everyday experience shows that the speed of muscle contraction depends


on the load that must be moved. We know that we can move a light load
quickly, whereas we move a heavy load slowly. The force-velocity curve
is produced by measuring the initial velocity and plotting it against the

161
Skeletal Muscle Mechanics - Dr. Feher

afterload. The initial velocity can be measured using a device such as that
shown in Fig. 8, where the afterload is varied. The force-velocity curve is
shown in Fig. 9. A.V. Hill first described this as a hyperbolic relationship.
More recent studies suggests that at the high force side it deviates from a
single hyperbolic relation. These force-velocity curves vary with muscle
length.

Figure 9. The force-velocity curve for an intact fast twitch or


slow twitch muscle.
D. Muscle power varies with the speed of contraction, the load and the
muscle type.

1. Power is the force times the velocity.

Power is defined in physics as the rate of energy production or


consumption. Thus, power has the units of energy per unit time. In
mechanical terms, energy is work. Work is further defined in
mechanics as force x distance. Thus, we have the following:
Power = Energy / time (1)
Energy = Work (2)
Work = Force x distance (3)
Inserting eq. (3) into eq. (2) and then into eq. (1), we get:
Power = Force x distance / time (4)
From the definition of velocity,
Velocity = distance / time (5)
We get:
Power = Force x velocity (6)

162
Skeletal Muscle Mechanics - Dr. Feher

Figure 10. Power-force curve for slow-twitch and fast-twitch muscles.

2. Muscle power peaks at about one-third maximal force.

The force-velocity curve is displayed in Fig. 10. We can obtain the power-
force curve by multiplying force and velocity at every point on the force-
velocity curve. This power is the instantaneous power produced during the
initial shortening of the muscle. Power is usually expressed in units of
watts, or N-m s-1, per unit weight of muscle.
3. Muscle power is about 2-3 times greater in fast twitch fibers than in
slow twitch fibers.
The results of Fig.10 show that the maximum force is not different for
slow and fast twitch muscles, but the power is about 2-3 times greater in
fast twitch fibers because of their greater speed of contraction.
4. Muscle power varies with speed of contraction and muscle type.
The muscle power can be plotted against the load, as in Fig. 10, or against
the speed of contraction, as in Fig. 11 below. The two curves come to very
different conclusions. The conclusion of Fig. 11 is that the contribution of
slow twitch fibers to the power of a contracting muscle depends on its
speed. During rapid contractions, the slow twitch fibers make almost no
contribution to the power, whereas at slow contractions it makes a large
contribution. Power output peaks at about one-third maximum velocity.

163
Skeletal Muscle Mechanics - Dr. Feher

Figure 11. Power vs. velocity curve. The dashed lines represent the
force-velocity curve and the solid lines are the power plotted against the
velocity. Power peaks at about one-third maximum velocity and is about
2-3 times greater in fast twitch fibers than in slow-twitch fibers. In a
mixed muscle consisting of both fast and slow twitch fibers, the
contribution each fiber type to the power of the muscle depends on the
velocity. At rapid contractions, all of the power derives from the fast-
twitch fibers. At slow contractions, the slow-twitch contributes much of
the power.
E. Muscles contraction can involve a lengthening of a muscle.
1. The force-velocity curve can be extended to negative velocities.

According to the way in which we measured velocity, a positive


velocity corresponds to a shortening of the muscle. If the support for a
very large afterload in Fig. 8 is removed, the large afterload will cause
the muscle to lengthen during contraction. This lengthening is a
negative velocity of the muscle. Contraction of the muscle during a
lengthening is called an eccentric contraction. Contraction of a
muscle that causes a shortening is called a concentric contraction.

2. Muscles can exert more force in an eccentric contraction

As seen in Fig. 12, muscles can exert about 40% more force in an
eccentric contraction compared to the maximal isometric force
measured at zero velocity.

164
Skeletal Muscle Mechanics - Dr. Feher

Figure 12. Concentric and eccentric contractions. Concentric contractions involve a shortening
of the muscle. Eccentric contractions involve a lengthening of the muscle. Isometric contractions
occur when the muscle length does not change, and occurs at zero velocity.
3. Concentric and Eccentric contractions have different functions.

Table 1 below lists the three types of contractions, their functions for
movement and the work performed.
Table 1. Types of Contractions
Type of Contraction Distance Change Function Work
Acceleration positive W = F x
Concentric shortening (+D)
(upstairs) (+D)
Isometric no change (0 D) Fixation zero
Deceleration negative W = F x (-
Eccentric lengthening (-D)
(downstairs) D)
IV. MUSCLE ARCHITECTURE influences force and velocity of the whole
muscle

A. Muscle force is usually proportional to muscle cross-sectional area

Muscle force is proportional to the size of the muscle, and size is generally
taken to be the cross-sectional area at the widest part of the muscle.
However, muscles are oddly shaped – they are not regular geometric
shapes. Also, muscles generate greatly different forces when the cross-
sectional area is the gross muscle area. The angle the muscle fibers make
with the tendons contributes to this variability in the gross muscle force.

165
Skeletal Muscle Mechanics - Dr. Feher

B. Muscle fibers can be oriented at an angle to the direction of force

There are three major orientations of the muscle fibers within muscles:
parallel fibers, fusiform and pinnate. The parallel fiber arrangement is
present in muscles shaped like a strap, or in parts of flat-shaped muscles.
In fusiform muscles, the muscle fibers are parallel to the longitudinal axis
of the muscle. In pinnate muscles, the fibers are oriented at an angle to the
tendon or aponeurosis. Because of their resemblance to a feather, these are
called pinnate or pennate. Both spellings are used. See Fig. 13.

Figure 13. Different arrangement of muscle fibers. Parallel fibers are


oriented longitudinally in the direction of the muscle action. Fusiform
fibers are tapered. Pinnate fibers are parallel but oriented at an angle to the
action of the muscle.
C. Pinnation increases muscle force but decreases muscle velocity

The consequence of orienting the muscle fibers at an angle with respect to


the tendon is to increase the effective cross-sectional area of the muscle
while reducing the distance the muscle can contract along the lines of the
tendons. To see this, I have calculated the force and velocity of two
different muscles: a strap muscle with parallel fibers having a volume of
300 cm3 and a length of 36.8 cm and a second muscle with pinnate
architecture with fibers 12 cm long and an overall length of 36.8 cm, with
the same volume as the strap muscle. The pinnate muscle has fibers
oriented at 15° to the line of action. The geometry of these two is shown in
Fig. 14.

Strap muscles such as the sartorius are composed of muscle fibers that do
not span the distance from tendon to tendon. The longest muscle fibers are

166
Skeletal Muscle Mechanics - Dr. Feher

about 12 cm. Such strap muscles are divided into compartments by fibrous
bands called inscriptions. The sartorius muscle has three inscriptions,
giving four compartments; the semitendinosus has three compartments,
and the biceps femoris and gracilis have two compartments each.

Without belaboring the calculations, the results are summarized in the


boxes in Fig. 14: the strap muscle has a high velocity but low force
compared to the pinnate muscle. The pinnate muscle allows more muscle
fibers to get into the act of producing force, while being constrained to its
anatomic location and volume. It pays for this force by being slower.
The effect of the pinnate architecture, then, is to increase muscle strength
at the expense of muscle speed.

Figure 14. Comparison of a strap muscle, such as the sartorius, with a pinnate muscle. Both muscles
are 36.8 cm long and both have a volume of 300 cm3. Because of the orientation of the muscle fibers,
the pinnate muscle generates more force at slower velocities than the comparably sized parallel-fibered
strap muscle.

167
Skeletal Muscle Mechanics - Dr. Feher

V. FATIGUE is the decrease in force upon repeated stimulation of a muscle.


A. Fatigue is produced within a few seconds of maximal stimulation

Everyday experience shows us that maximal effort can be sustained only


briefly. The more intense the effort, the faster one fatigues. From our
earlier sections it should be clear that intense efforts rely predominately on
fast twitch fibers. These are more easily fatigued than the slow twitch
fibers.

Figure 15. Fatigue of a maximally stimulated muscle (complete recruitment


and tetanus) during prolonged stimulation.
B. Fatigue of slowly contracting muscles takes longer to produce

Fatigue also occurs when muscles are repetitively used in less powerful
movements. This fatigue, however, takes longer to produce, and longer to
recover.

C. Muscles can be classified according to their contractile properties.

Burke’s system of classification recognizes four types of muscle fibers,


based on their contractile properties, include resistance to fatigue:
1. S = slow twitch fibers
2. FR = fast, fatigue-resistant
3. FI = fast, intermediate fatigue resistant
4. FF = fast, fatiguable

D. Most muscles are heterogeneous mixtures of the different muscle types

168
Skeletal Muscle Mechanics - Dr. Feher

Most muscles consist of thousands of muscle fibers. In general, muscles


contain all of the different fiber types, but differ in the percentage. For
example, the soleus muscle in the human consists predominately of slow
twitch fibers, whereas the gastrocnemius consists of predominately fast-
twitch fibers. Neither are entirely one type or another, however. There is
considerable individual variation in the percentages of different muscle
fibers that comprise any particular muscle.

V. PRACTICE QUESTIONS

1. An ocular muscle has a twitch time of about 25 ms. Stimulation at


maximum recruitment at 20 Hz ( I Hz = 1 cycle per second) for 0.5 s gives
A. An unfused or partial tetanus
B. Tetanus
C. A sequence of twitches separated by 50 ms
D. Slightly greater twitches separated by 20 ms
E. Fatigue

2. Passive tension

A. Is the increment in muscle force upon activation by its nerve


B. Varies linearly with muscle length
C. Increases to a maximum at L0 and then decreases
D. Increases less than linearly with muscle length
E. Is the force measured by stretching a muscle without activating its
nerve

3. Muscle A has a cross-sectional area of 10 cm2 and a maximum velocity of


20 cm s-1. Muscle B has a cross-sectional area of 20 cm2 and a maximum
velocity of 40 cm s-1. The ratio of their maximum power, PA / PB should
be about

A. 0.25
B. 0.5

169
Skeletal Muscle Mechanics - Dr. Feher

C. 1
D. 2
E. 4

4. After drinking (= alcohol) a little too much, your somewhat juvenile friend
decides to challenge you to a test of strength. The winner is the one who
can hold up the heaviest weight in the supine palm of the hand with the
elbow at a right angle. The game proceeds by adding successive copies of
Dr. Costanzo’s text, Physiology, 3rd edition. Remember, you’ve been
drinking. The contraction here is classified as

A. Isometric
B. Concentric
C. Eccentric
D. Isotonic
E. Isovolumic

5. The ability to hold the additional copies of Dr. Costanzo’s text in the
above question is achieved by

A. Increasing the frequency of activation of all motor units


B. Activating the large motor units first
C. Successively activating more and more motor units
D. Changing the length of the biceps brachii and brachialis muscle
E. Increasing the magnitude of the action potentials on all of the
muscle fibers

6. Mr. Universe’s biceps brachii and brachialis muscles have a total cross
sectional area of 160 cm2 whereas a representative puny medical student,
“Amy” has a total cross sectional area of only 40 cm2. Mr Universe says
he can flex his arm loaded with 25 Kg (!) faster than the student can flex
his arm loaded with 10 Kg. A third medical student, “Bill” feels the
competition is unfair and challenges them both when his load is 12.5 Kg.
His muscles are 100 cm2 in area. Your friendly professor measures peak
velocity of movement during a biceps curl. After the competition is over,

170
Skeletal Muscle Mechanics - Dr. Feher

what is the order of the finishes? Assume that the length of the arms and
positions of origin and insertions of the muscle are identical. (In short,
ignore mechanical advantage and focus on the muscles)

A. Universe, Bill, Amy


B. Bill, Amy, Universe
C. Bill, Universe, Amy
D. Universe, Amy, Bill
E. Amy, Universe, Bill
Answers:
1. C The twitch time is 25 ms = 0.025 s. This means that summation just begins
at 1/0.025 s = 40 Hz. At a stimulation frequency of 20 Hz the twitches are
separated by 50 ms, which occurs at baseline and therefore no summation occurs.
There is no partial tetanus, no tetanus, and sequence of twitches is separated by 50
ms. tetanus).
2. E The passive tension is the tension developed without activation of the nerve. It
varies progressively with distance, being near zero at rest length and increasing
faster than linear with length.
3. A Muscle power is proportional to force times velocity. Maximum muscle power
is proportional to maximum force, which is proportional to area., and the velocity
at maximum power is proportional to the maximum velocity (it occurs at about
one-third Vmax). Thus PA α AreaA VmaxA and PB α AreaB VmaxB so PA / PB
=(10 cm2 x 20 cm s-1) / (20 cm2 x 40 cm s-1) = 2/8 = 0.25. This ratio is inexact, but
it is closer than any other alternative answer.
4. A Isometric. The muscle is not moving when additional weight is placed on it.
Isometric contractions keep muscle length constant; eccentric contractions
lengthen the muscle; concentric contractions shortens it. Isotonic contractions
shorten but keep force constant. Isovolumic contraction refer to the heart’s
isometric contraction before the valves open.
5. C This is recruitment. Force is increased by activating more motor units. The
smaller ones are activated first (the size principle). Of course, to keep the books
held the muscles must be activated in trains of impulses.
6. C. The speed of contraction depends on the load. We can calculate the load per
unit cross sectional area: Mr Universe is 25 Kg / 160 cm2 = 0.156 Kg cm-2; Amy’s
is 10 Kg/40 cm2 = 0.25 Kg cm-2; Bill’s is 12.5 Kg /100 cm2 = 0.125 Kg cm-2. So
the load is lightest with Bill, intermediate with Mr. Universe and heaviest for
Amy. The fastest person should be Bill, followed by Mr. Universe and then Amy,
because the speed is inversely proportional to load.

171
Contractile Mech in Skeletal Muscle - Dr. Feher

Contractile Mechanisms in Skeletal Muscle


Joseph Feher, Ph.D.
LECTURE OUTLINE:
I. Introduction

A. How can we explain overall behavior of muscle?

II. Muscle Cells have a highly organized structure

A. Large multinucleated muscle cells are striated.


B. Striations are the most prominent feature of skeletal muscle.
C. Striations are organized longitudinally into myofibrils.
D. Striations are due to overlap of thin and thick filaments.
E. Thick and thin filaments form interdigitating hexagonal arrays.
F. The functional unit of muscle is the sarcomere.
G. Excitation is carried into the muscle by transverse tubules.
H. Myofibrils are covered by an internal membrane network, the
sarcoplasmic reticulum.

III. The sliding filament hypothesis explains the length-tension curve

A. I bands shorten while A band stay the same length during muscle
shortening.
B. The sliding filament hypothesis predicts that force depends on overlap of
the filaments.

IV. Force is produced by an interaction between thick and thin filament proteins

A. The thick filament consists primarily of myosin.

1. Myosin consist of 6 polypeptides


2. The giant protein titin is a template for thick filament assembly

172
Contractile Mech in Skeletal Muscle - Dr. Feher

3. Myosin forms a regular array of head groups on the thick filament.


4. The thick filament contains a “bare zone” in its middle with no
head groups.

B. The thin filament consists primarily of actin.

1. Globular actin polymerizes to form filamentous actin.


2. The giant protein, nebulin, sets the length of the thin filament.
3. Thin filaments also contain tropomyosin.
4. Troponin is a complex of three proteins on the thin filament.
5. Thin filaments are anchored at the Z-disk by actinin.

C. Cross bridges from the thick filament split ATP and cause shortening.

1. Myosin heads bind to actin filaments


2. Myosin is an actin-activated ATPase
3. Actomyosin ATPase activity occurs in the cross-bridge cycle.
4. ATP hydrolysis allows the myosin to "walk" along the actin
filament

V. Cross-bridge cycling rate explains the fiber type dependence of the force-velocity
curve

A. Force of contraction is related to the number of myofibrils in parallel


B. Speed of contraction is related to the number of sarcomeres in series
C. Rate of shortening in a sarcomere depends directly on the turnover rate of
the cross-bridges
D. Slow and fast twitch muscles have different myosin isoforms.
E. Muscles can be classified on the basis of their myosin staining.

VI. Force is transmitted outside the cell through the cytoskeleton and special
transmembrane proteins

173
Contractile Mech in Skeletal Muscle - Dr. Feher

A. Costameres are located at the Z-disk.


B. Costameres are a complex assembly of proteins.
C. Costameres may have multiple functions

VII. Practice Questions


OBJECTIVES:
1. In a drawing or electron micrograph, identify: myofibril, A band, I band, H band,
M line, Z line
2. Define the sarcomere and give its approximate resting length
3. List the major protein components of the thick filaments and the overall length of
thick filaments
4. List the major protein components of the thin filaments and the overall length of
thin filaments
5. Describe the origin of the length-tension curve on the basis of overlap of thin
filaments and force generators on the thick filaments
6. Indicate the part of the myosin molecule that hydrolyzes ATP
7. Indicate why cross-bridge cycling rate determines (in part) muscle speed
8. Describe how muscles can be classified on the basis of myosin isotype staining
9. Describe how force is thought to be transmitted outside of the muscle
Suggested Reading: Berne and Levy, pp. 223-227; 231-233
I. INTRODUCTION: from the last lecture we learned that...

• muscles are heterogeneous with respect to contractile properties


• muscle force can be graded by recruitment of motor units
• muscle force can be graded by increasing the frequency of motor neuron
firing
• muscle force can be graded by changing the length of the muscle
• muscle velocity is inversely related to force of shortening
• the power of a muscle peaks at about 1/3 maximal force and at about 1/3
maximal velocity

174
Contractile Mech in Skeletal Muscle - Dr. Feher

We seek an explanation of these overall behaviors in the sub-cellular and


molecular description of muscle.
II. MUSCLE FIBERS have a highly organized structure.

A. Large, multinucleated muscle fibers are striated

Muscle cells are large multinucleated fibers some 10-100 μm in diameter


and as long as 12 cm. In some muscles, they can span the distance
between insertion and origin. They need many nuclei to govern protein
synthesis and degradation in these large fibers. The nuclei are located near
the periphery of the cell and often are more highly concentrated near the
neuromuscular junction.

B. The striations are the most prominent feature of muscle cells.

Viewed under light microscopy, the most striking feature of muscle cells
is their stripes. These stripes, or striations, result from the highly
organized arrangement of proteins in the muscle fiber. The striations
consist of alternating A-bands and I-bands, named because the I-bands
are isotropic to polarized light (meaning that they appear the same from all
directions) whereas the A-bands are anisotropic to polarized light.

Figure 1. Microscopic appearance of skeletal muscle fibers. A bundle of frog sartorius


muscle fibers was teased out and viewed under phase contrast microscopy. Cross-striations
are readily apparent in these unstained muscle fibers.
C. The striations are organized longitudinally into myofibrils.

Muscle cells are also organized longitudinally into tiny threads called
myofibrils. These cylinders are composed of two kinds of filaments. The

175
Contractile Mech in Skeletal Muscle - Dr. Feher

thin filament contains actin and the thick filament contains myosin.
These myofibrils also show cross-striations that are due to the way in
which the filaments overlap each other. The myofibrils are kept in register
across the entire cell to give rise to the striated appearance of the fiber.

Figure 2. Electron micrograph of striated muscle. The spaces between myofibrils are filled with
membranes of the sarcoplasmic reticulum, mitochondria, and glycogen granules. The myofibrils are
bundles of myofilaments arranged longitudinally parallel to the long axis of the muscle fiber. The
various bands are named according to their position, appearance, or how they rotate the plane of
polarized light.
D. The striated appearance is due to overlap of thick and thin filaments

The anisotropy of the A bands is due to myosin in the thick filaments. The
thick filaments are 1.6 μm long, so the A-band is also 1.6 μm wide. The
thin filaments are about 1.0 μm long. Opposite thin filaments are
connected at the Z-line (from the German zwischen, meaning “between”).
Because the myofibrils are cylindrical, the Z-line is actually a disk and it is
also called the Z-disk. Thin and thick filaments typically overlap and the
distance between successive Z-disks is less than the A-band plus the
length of two thin filaments (= 1.6 + 2 x 1.0 = 3.6 μm). The H-zone (from
the German helles, meaning “clear”) is a clearer area in the middle of the
A band that shows where the thin filaments do not overlap the thick
filaments. Proteins at the M-line (from the German mittel, meaning
“middle”) connect the thick filaments and keep them in register.

E. The thick and thin filament form interdigitating hexagonal arrays (Fig. 3)

Both thick and thin filaments form hexagonal arrays. The array of the thin
filament is rotated 30° from the thick filaments. Each thick filament is in
the center of a hexagon of thin filaments, whereas each thin filament is in
the center of a triangle of thick filaments. Thus, there are 2 thin filaments

176
Contractile Mech in Skeletal Muscle - Dr. Feher

for every thick filament. Electron micrographs show cross-bridges


between the thick and thin filaments. The interaction of the filaments
through these cross-bridges produces either shortening or force.

F. The functional unit of contraction is the sarcomere.

The material found between successive Z-disks is called the sarcomere.


This is the functional unit of contraction or force production. The
myofibrils consist of thousands of these sarcomeres strung end to end. At
rest, the sarcomere is about 2.2 μm long.

Figure 3. Structure of the muscle fiber and myofibrils.


G. Excitation is carried into the muscle fiber by transverse tubules.

Long tubules come off the sarcolemma at regular intervals perpendicular


to the long axis of the muscle fiber. These are the transverse tubules or T-
tubules. They bring the excitation of the action potential on the
sarcolemma into the deepest parts of the interior of the cell.

177
Contractile Mech in Skeletal Muscle - Dr. Feher

H. The myofibrils are covered with an internal membrane network called the
sarcoplasmic reticulum (see Fig. 4).

The sarcoplasmic reticulum, or SR, forms an internal compartment


separate from the cytoplasm. It is functionally divided into a longitudinal
SR and terminal cisternae. The terminal cisternae form sacs that closely
appose the T-tubules, whereas the longitudinal SR are thin tubes of
membrane that connect terminal cisternae from one side of the sarcomere
to the other. The longitudinal SR and terminal cisternae form a single
enclosed space. In skeletal muscle the junction of one T-tubule and two
terminal cisternae forms a triad at the junction of the A-band and I-band,
so there are two triads per sarcomere.

Figure 4. Structure of the SR around the myofibril.


III. THE SLIDING FILAMENT HYPOTHESIS explains the length-tension
curve.

A. I bands shorten while A bands stay the same length during muscle
shortening.

Muscle shorten because the sarcomeres shorten. The Z-disks move closer
together but there is no change in the length of the A band: all of the
shortening appears to occur in the I-bands, which are the part of the thin
filaments that don’t overlap the thick filaments. A.F. Huxley and R.
Niedegerke first proposed that the thin (I-band) filaments slide past the
thick (A-band) filaments and that force arises from the interaction between
these filaments.

178
Contractile Mech in Skeletal Muscle - Dr. Feher

B. The sliding filament hypothesis predicts that force depends on the overlap
of thick and thin filaments.

At a sarcomere length of 3.65 μm, there is no overlap of the filaments


because the A-band is 1.6 μm and each of the two I-band filaments is 1.0
μm, At no overlap, there is no force. At progressively shorter lengths, the
A-band and I-band filaments overlap increasingly so the force
proportionately increases. At 2.25 μm there is maximum overlap and so
there is maximum force. This force does not decrease until the sarcomere
shortens to less than 1.95 μm. The reason for this is that there is still
maximal overlap of the force generators on the thick filament. The thick
filament has a bare central region (about 0.3 μm) that lacks the force
generators. When the sarcomeres shorten still further (below 1.95 μm), the
thin filaments begin to run into each other. Although the thin filaments can
slide past each other, they encounter resistance and so the active force
falls. At still shorter lengths, the thick filament begins to butt up against
the Z-disk. This occurs at about the length of the A-band, or 1.6 μm.
Further shortening seriously distorts the thick filament and force falls
precipitously with further shortening.

Figure 5. Dependence of tension on the degree of overlap of the thin and thick
filaments.

179
Contractile Mech in Skeletal Muscle - Dr. Feher

IV. FORCE IS PRODUCED by an interaction between thick filament proteins


and thin filament proteins.

A. The thick filament consists primarily of myosin

1. Myosin consists of 6 polypeptides.

Myosin is a complex of 6 proteins having a combined molecular


weight of 480,000 Da. It consists of two heavy chains of 200,000
Da each, and a total of 4 myosin light chains. The two heavy
chains have a long, rod-shaped tail and a globular “head” region
that contains the active site. An arm section attaches the head to
the tail. The arm and heads form the cross-bridges that interact
with the thin filament to produce force.

2. The giant protein, titin, serves as a template for the aggregation of


myosin to form the thick filament.

The giant protein titin, also called connectin, is the largest protein
known to date with a molecular weight of about 3.7 million Da. At
8-10% of the myofibrillar protein, it is the third most abundant
skeletal muscle protein. It spans the distance from Z-disk to M-line
and binds α-actinin, myosin and M-protein, the one responsible
for tying together the thick filaments at the M-line. Myosin binds
to titin to form a large aggregate, the thick filament.

3. Myosin forms a regular array of head groups on the thick filament.

Myosin binds to titin and to other myosin molecules along the


myosin tail region. The myosin heads project out of the thick
filaments at regular intervals of 14.3 nm. There appear to be 3
myosin molecules (6 heads) at each plane oriented at right angles
to the long axis of the filament. There is an identical repeat every
43 nm. Each thick filament has about 300 myosin heads projecting
from its surface. Fig. 6 shows the structure of the thick filament.

4. The thick filament is polarized, with head groups at each end and a
“bare zone” in the middle.

The myosin molecules lay themselves down along the titin


template so that the tail regions point toward the middle of the
filament. Because of this arrangement, the middle of the thick

180
Contractile Mech in Skeletal Muscle - Dr. Feher

filament contains only tails, and the heads are located at the ends
of the filament.

Figure 6. Structure of the thick filament.


B. The thin filament consists primarily of actin.

1. Globular actin (G-actin) polymerizes to form filamentous, or F-actin.

Globular actin, or G-actin, is a 41.8 KDa monomer with a diameter of


about 5.5 nm. It aggregates to form a filament, F-actin, consisting of
two strands of actin molecules wound around each other. About 7 G-
actin molecules forms each half-turn, giving a half-turn distance of
about 38.5 nm.

2. The giant protein, nebulin, sets the length of the actin filament.

Nebulin is another giant protein, with a molecular weight between


600,000 and 900,000 Da in different muscles. It spans the whole
length of the thin filaments and is anchored at the Z-disk. It contains a
string of about 200 actin binding domains. Nebulin may set the length
of the thin filament (normally about 1.0 μm in skeletal muscle).

3. Thin filaments contain tropomyosin.

Tropomyosin consists of two non-identical polypeptide chains, each


with a molecular weight of about 33,000 Da. They are long, rod-
shaped molecules, and the tropomyosin α and β wrap around each

181
Contractile Mech in Skeletal Muscle - Dr. Feher

other to form a supercoil. The tropomyosin complex is 38.5 nm long,


the same size as a half-turn of the F-actin helix. Along with troponin,
tropomyosin participates in the regulation of the active state of muscle.

4. Troponin is a complex of three proteins on the thin filament.

Troponin consists of troponin T, a 37,000 Da protein; troponin C of


18,000 Da and troponin I of 21,000 Da. These three derive their
names from their functions: TnT binds to tropomyosin; TnC binds
calcium ions (and thereby confers Ca2+ sensitivity onto the
myofilaments), and TnI inhibits the interaction between the thick and
thin filaments that cause force development or shortening. These three
proteins form a complex at one end of each tropomyosin molecule.
The troponin complex is responsible for the final regulation of the
contractile state of the myofilaments.

Figure 7. Structure of the thin filament.


5. Actin filaments of adjacent sarcomeres are joined at the Z-disk by α
actinin.

The Z-disk contains a number of proteins that can bind F-actin. The
precise way that these proteins align the thin filaments is not yet
known. One of these proteins, α actinin, consists of two subunits of
95,000 Da that is located in the Z-disk and anchors the thin filaments
there. A proposed arrangement of the Z-disks shown in Fig. 8
illustrates the complexity of this structure. Many forms of muscular
dystrophy are linked to mutations in one or another of these proteins.

182
Contractile Mech in Skeletal Muscle - Dr. Feher

Figure 8. Postulated linkage of thin and thick filaments at the Z-disk. The thin filaments
have opposite polarity at the Z disk. The Z-disk contains a variety of proteins that bind to
other proteins in the disk. Gamma filamin is a cytoskeletal protein that links the Z-disk to
the outside of the cell.
6. The actin filaments at the Z-disks have opposite polarity.

The thin filaments to the right of a Z-disk project towards a thick


filament to the right, whereas the thin filaments to the left of a Z disk
projects towards a thick filament to the left. Thus, the polarity of the
thin filaments at the Z-disk are opposite. The plus end, the end where
the filament grows, is capped by CapZ and anchored by it at the Z
disk.
C. Cross-bridges from the thick filament split ATP and generate force.

1. Myosin heads binds to actin filaments.

Proteolytic cleavage of myosin with papain separates myosin into the


long rod-shaped tail and two myosin subfragments called S1. These S1
fragments bind to actin filaments and can be seen as a lateral
projection on the thin filament that resembles an arrowhead in negative
stained electron micrographs. These arrowheads all point the same
way, revealing the F-actin polarity.

2. Myosin is an actin-activated ATPase.

Although myosin can split ATP all by itself, actin binding speeds it up

183
Contractile Mech in Skeletal Muscle - Dr. Feher

some 200-300 fold. The hydrolysis of ATP takes several steps: ATP
binds to the myosin head (the “business” end of the molecule), then it
is hydrolyzed to ADP and Pi, which remain non-covalently bound.
Then the myosin must release the products as free ADP and Pi. The
step that limits the speed of this reaction is the release of ADP and Pi.

3. Actomyosin ATPase activity occurs in the cross-bridge cycle.

Myosin splits ATP according to the simplified reaction cycle shown in


Fig. 9. The cycle involves sequential binding of ATP and conversion
of myosin into a form having low actin affinity, hydrolysis of ATP
without release of the product ADP and Pi, conversion of myosin into
a form having a high affinity for actin, ejection of the ADP and Pi and
subsequent change in conformation of the myosin head, and binding of
ATP again.
4. ATP hydrolysis allows the myosin head to “walk” along the actin
filament.

The main idea here is that ATP hydrolysis is linked to a sequential


change in the conformational state of the myosin head that allows
myosin to walk along the actin filament. Since myosin is connected to
the thick filament, this movement of the myosin head corresponds to a
movement of the thick filament relative to the thin filament.

5. Many cross-bridges produce macroscopic force.

The cross-bridges have been likened to so many oarsmen rowing a


large boat. The problem with the analogy is that usually oarsmen row
in concert, and the medium they pull against is completely fluid, as
opposed to the thin filament. Perhaps a better analogy is a group of
sailors pulling in a rope hand-over-hand. In our everyday experience,
this is not highly coordinated unless the load on the rope is great.

184
Contractile Mech in Skeletal Muscle - Dr. Feher

Figure 9. The acto-myosin cross-bridge cycle.

V. CROSS-BRIDGE CYCLING RATE explains the fiber type dependence of


the force-velocity curve

A. Force of contraction is related to the number of myofibrils (sarcomeres) in


parallel

The myosin heads tug on the actin filaments. These actin filaments are
connected at the Z-disk to titin, which is in turn connected to the thick
filament on the adjacent sarcomere. Thus the force developed in each
sarcomere is transmitted through the Z-disk to the adjacent sarcomere. It is
clear from this description that the tension in a single myofibril is the same
everywhere in the myofibril: the developed force is equal to the force
developed by each sarcomere. Each myofibril contributes force in relation
to the number of force generators, which is proportional to the number of
filaments in the myofibril. The total force developed by the muscle
depends on the number of its myofibrils. Taken to its final conclusion, we
should expect muscle force to be proportional to its cross-sectional area.

B. Speed of contraction is related to the number of sarcomeres in series.

The shortening of a muscle is the result of the shortening of its composite


sarcomeres. If one sarcomere shortens a distance Δx, and another
sarcomere in series with it shortens the same amount, the total shortening
is 2 x Δx. Since both of these sarcomeres shorten in the same time
interval, Δt, the velocity of shortening is Δx/Δt for one sarcomere and 2 x

185
Contractile Mech in Skeletal Muscle - Dr. Feher

Δx/Δt for two sarcomeres in series. Thus, the velocity of shortening is


directly related to the number of sarcomeres in series and the rate of
sacromere shortening.

Figure 10. Relation between


sarcomere
and muscle shortening.
C. Rate of shortening in a sarcomere depends directly on the turnover rate of
the cross-bridges.

Each cross-bridge cycle slides the thin filament about 7-10 nm past the
thick filament. Rapid turnover of the cross-bridge means that more of
these cycles occur per second, and therefore the thin filament slides past
the thick filament more quickly. Thus, the rate of shortening of each
sarcomere, and therefore the entire muscle, depends on the turnover rate of
the cross-bridges.

D. Slow and fast twitch muscles have different myosin isoforms.

Muscles make a variety of different isoforms of myosin which are


encoded by separate genes. In the adult there are two basic varieties: slow
myosin and fast myosin. The catalytic mechanisms for the two types are
similar, but the turnover number (the number of completed reactions per
second) is about 10 s-1 for the fast myosin and 3 s-1 for slow myosin.

186
Contractile Mech in Skeletal Muscle - Dr. Feher

E. Muscles can be classified on the basis of their myosin staining.

It is possible to stain for the different myosin isoform ATPase activity


based on the incubation conditions such as the pH. On this basis, a variety
of different staining patterns can be observed. Brooke has classified
muscles as type I, type IIA, type IIB and type IIC on the basis of myosin
staining. The type I is the slow type of fiber and type II refer to different
fast twitch fiber types. Figure 11 illustrates muscle heterogeneity as
evidenced by myosin staining.

Figure 11. Histological staining of myosin reveals muscle heterogeneity.


Myosin staining differentiates among various muscle types. This histological
section shows three well-defined classes of staining but several intermediate
fibers are also evident.
VI. FORCE IS TRANSMITTED outside the cell through the cytoskeleton and
special transmembrane proteins.

A. Costameres are located at the Z-disk.

The cytoplasmic face of the sarcolemmal membrane possesses a


cytoskeletal assembly of proteins in a discrete, rib-like lattice. These are
the costameres. They are located at the Z-disk where they bind to actin
filaments through γ-actin filaments.

B. Costameres are a complex assembly of proteins.

The structure of costameres is still being worked out, but many proteins

187
Contractile Mech in Skeletal Muscle - Dr. Feher

participate in it. These include dystrophin, a 427 KDa protein that is


absent in Duchenne muscular dystrophy. This protein is localized to the
periphery of the muscle fibers, on the cytosolic side of the sarcolemma,
and binds to cytoskeletal elements. It is concentrated at the costameres but
is not restricted to this location.

C. Costameres may have multiple functions.

The function of the costameres is not yet established. The possibilities


include: (1) to transmit force from the contractile filaments to the outside
of the cell; (2) to mechanically support the sarcolemma to protect it
against damage, particularly during eccentric contractions; (3) to maintain
uniform sarcomeric spacing between resting and active fibers of different
motor units.

Figure 12. Proposed arrangement of proteins in costameres of skeletal muscle.


VII. PRACTICE QUESTIONS

1. The Z-disk lies in the middle of the

A. A band
B. H zone
C. I band
D. Terminal cisternae
E. T-tubule

188
Contractile Mech in Skeletal Muscle - Dr. Feher

2. Which protein is not located in the Z disk

A. Actinin
B. Actin
C. Nebulin
D. Titin
E. Myomesin

3. The top of the length-tension curve is flat because

A. The actin filaments begin to interfere with each other, dropping


force
B. The myosin filament is longer than the actin filament
C. All of the muscle fibers are already recruited
D. The middle of the myosin filament contains a bare zone with no
force generators
E. Thin and thick filaments are maximally overlapped.

4. Troponin is located

A. On the thick filament


B. On the thin filament
C. Mainly in the Z-disk
D. In the M line
E. In the lumen of the SR

5. During concentric contractions

A. The A bands shorten and so does the H zone


B. The I bands shorten and so does the H zone
C. A and I bands shorten and so does the H zone

189
Contractile Mech in Skeletal Muscle - Dr. Feher

D. A bands and I bands shorten but H zone stays constant


E. The I bands shorten but the H zone stays constant

6. Costameres

A. Are sarcomeres in the intercostal muscles


B. Link sarcomeres together
C. Form a scaffold for the assembly of myosin
D. Link the myofilaments to the extracellular matrix
E. Are the units of force in muscle

Answers: 1C; 2E; 3D; 4B; 5B; 6D

190
Autonomic Nervous System 1 - Dr. Biber

Autonomic Nervous System 1


Margaret C. Biber, D.Phil.

OBJECTIVES:

Please note that these objectives pertain to ANS lectures I-IV.

At the end of these lectures you should know and understand the following material:

1. The relationship between the organization of the sympathetic and parasympathetic


divisions of the ANS to their overall physiological effects.
2. Anatomical and functional differences between the skeletal neuromuscular
junction and autonomic neuroeffector junctions.
3. Transmitters used at ganglionic and neuroeffector junctions and highlights of the
transmitter life cycle: Storage, release, biological inactivation, metabolism and de
novo synthesis for acetylcholine (Ach), norepinephrine (NE) and the hormone,
epinephrine (EPI).
4. Receptor types for Ach and the catecholamines, NE and EPI and their effects.
5. The mechanism of action of other transmitters/mediators including ATP, NO and
peptides.
6. The organization of autonomic reflexes.
7. The overall physiological effects of the parasympathetic and sympatho-adrenal
systems and the receptor types that mediate the responses.

Reading: Berne, Levy, Koeppen and Stanton: Physiology, 5th edition. 2004; Ch. 11,
Pages 206-215 –Table 11-1 is too detailed. Use Table in handout.
Costanzo: Physiology, 2006 Ch. 2, Pages 45-64

Note: Please follow the version in the handout wherever discrepancies exist between the
textbooks and the handout.

AUTONOMIC NERVOUS SYSTEM (ANS)

LECTURE 1 OUTLINE

INTRODUCTION
TARGETS OF INNERVATION
FUNCTIONS OF THE ANS
Regulation of visceral organs
Responses to environmental stimuli
CHARACTERISTICS OF AUTONOMIC CONTROL
Speed of onset
Tonic activity
Reflex control
GENERAL FEATURES OF ORGANIZATION
Pre and postganglionic neurons

191
Autonomic Nervous System 1 - Dr. Biber

Sympathetic and parasympathetic


ORGANIZATION OF THE SYMPATHETIC NERVOUS SYSTEM
Sympathetic chain
Collateral ganglia
Adrenal Medulla
Synaptic organization of sympathetic ganglia
Summary of organizational features of the sympathetic n.s.
ORGANIZATION OF THE PARASYMPATHETIC NERVOUS SYSTEM
Summary of organizational features of parasympathetic n.s.
ORGANIZATION OF AUTONOMIC NEUROEFFECTOR JUNCTIONS
COMPARISON OF SOMATIC AND AUTONOMIC NERVOUS SYSTEMS

INTRODUCTION

The ANS, in conjunction with the endocrine system, maintains man and adapts him to the
environment. By making continuous adjustments to organ system function in response to
both internal and external environmental changes, the ANS controls the internal
environment within narrowly defined limits (homeostasis) and permits the individual to
function and behave normally. This activity occurs automatically that is, without
conscious control.

The term ANS is often narrowly defined as the peripheral motor nerves innervating the
organ systems. This definition ignores the essential sensory input required for autonomic
regulation. However, it is useful for gaining an understanding of the layout and basic
functions of the system. According to this definition, the ANS consists of the sympathetic
and the parasympathetic divisions and the enteric nervous system that lies within the
walls of the gastrointestinal tract.

The ANS is anatomically distinct from the somatic motor system that innervates skeletal
muscle:

TARGETS OF INNERVATION:

192
Autonomic Nervous System 1 - Dr. Biber

FUNCTIONS OF THE ANS

ƒ REGULATION OF VISCERAL ORGAN FUNCTION (SOME EXAMPLES):

• Cardiovascular system: arterial blood pressure (regulates blood vessel


diameter and mechanical performance of the heart)
• Digestive system: coordinates gastrointestinal motility and secretions
(in conjunction with hormones)
• Urinary bladder: voiding
• Respiratory tract: secretions and airway resistance
(diameters of bronchi and bronchioles)

Important role in homeostasis

ƒ RESPONSES TO SPECIFIC ENVIRONMENTAL STIMULI:

• LIGHT
o constriction of the pupil to bright light (miosis)
o dilation of the pupil in low light (mydriasis)
o focus lens

• TEMPERATURE
o dilation of blood vessels of the skin (cutaneous vaso- dilation) and
sweating in a warm environment
o cutaneous vaso-constriction, goose pimpling and increased
metabolism of fat in the cold

• STRESS
o the ANS (mainly the sympathetic and the adrenal medulla)
mediates the FIGHT or FLIGHT response. This response allows the
body to undertake the severe physical exertion needed to survive
life threatening situations. It mobilizes and sustains physiological
and biochemical changes to meet the increased metabolic demands
of extreme physical effort.

The FIGHT or FLIGHT response has characteristic features:

• increase in heart rate and force of contraction (heart pounding)


• widely dilated pupils (wide eyed with fear)
• pallor (pale with fear; ashen faced) as blood is directed to the skeletal
muscle and away from other vascular beds, notably the skin, which
constricts
• goose pimpling: (hair standing on end)
• cold sweat (clammy hands)
• dry mouth

193
Autonomic Nervous System 1 - Dr. Biber

CHARACTERISTICS OF AUTONOMIC CONTROL

SPEED OF ONSET:

The ANS can produce rapid and dramatic changes in organ activity:

• heart rate can double within 3-5 seconds


• sweating can occur within a few seconds in response to a rise in
temperature
• goose pimpling appears in seconds in the cold
• blood pressure can rise or fall over seconds. A marked fall in blood
pressure can cause fainting.
• Fight or flight response is triggered in seconds

TONIC ACTIVITY:

• Most autonomic nerves fire low frequency bursts of action potentials


averaging around 1 impulse per sec (1 Hz). This basal activity is called
“tone” so there is “sympathetic tone” and “parasympathetic tone”.

• Tone or tonic activity establishes baseline contractile or secretory activity.

• Baseline activity can be increased or decreased simply by increasing or


decreasing impulse traffic in the autonomic nerves.

REFLEX CONTROL:

• ANS regulation occurs reflexly, i.e. without conscious control.

• Certain autonomic functions (urination, defecation) are brought under


voluntary control through participation of voluntary (skeletal) muscles.

• Autonomic reflexes are initiated in response to specific types of sensory


information. We are usually unaware of this sensory input.

• Autonomic reflexes are highly sensitive to emotional state. This


sensitivity arises through the same central circuitry that triggers the fight
or flight response.

194
Autonomic Nervous System 1 - Dr. Biber

GENERAL FEATURES OF ORGANIZATION

PRE AND POSTGANGLIONIC NEURONS

Two neurons carry information from the CNS to the effector organ. (Contrast this
2-neuron pathway with the single neuron of the somatic motor system). The 2
neurons of the ANS are:

Fig 1. Autonomic neuron

Spinal cord

preganglionic

postganglionic

Fine axons:
Conduct APs at low speeds (0.5 m/sec)
& at low frequency (~20 Hz)
smooth muscle
Control of target organ activity is slower & much less precise glands, heart
than control of skeletal muscle by somatic motor neurons

• Preganglionic neuron: The cell body lies in a brainstem nucleus, or in the


lateral gray of the spinal cord. A fine myelinated axon (2-4 u diameter)
travels toward the periphery and usually ends within a ganglion, where it
synapses with:

• Postganglionic neuron: The cell body is usually in a ganglion, where it


receives synaptic input from the preganglionic neuron. The postganglionic
axon travels out to the target organ.

SYMPATHETIC AND PARASYMPATHETIC DIVISIONS - a classification that reflects


differences in functional organization:

• Sympathetic preganglionic nerves lie in the thoraco-lumbar region of the


spinal cord.
• Parasympathetic preganglionic nerves lie in the cranial and sacral
regions of the CNS.

195
Autonomic Nervous System 1 - Dr. Biber

ORGANIZATION OF THE SYMPATHETIC N.S.

SYMPATHETIC CHAIN (FIG. 2)

• Preganglionic cell bodies lie in spinal cord lateral gray (intermediolateral


columns), in segments Tl through L3.

• Axons leave the spinal cord via ventral motor roots (VR) and then leave
VR via white rami communicans to enter a vertebral ganglion of the
sympathetic chain at the same segmental level (Fig. 3).

• Preganglionic axons can synapse on postganglionic cells within this


ganglion

• Or they can turn in a cranial or caudal direction to innervate ganglia at a


higher or lower level in the sympathetic chain. This permits innervation of
cervical, lower lumbar and sacral sympathetic ganglia. Synaptic contact
can occur in more than one ganglion.

• Postganglionic neurons in vertebral ganglia (ganglia of the sympathetic


chain) return their axons to the corresponding spinal nerve via the gray
rami communicans (gray because unmyelinated) (Fig. 3).

• They travel with other motor nerves to the peripheral target organs
(smooth muscle of blood vessels and hair follicles and the sweat glands).
Their axons are small, diameter 1.5 u.

• Sympathetic ganglia in the cervical region are fused; the largest is the
superior cervical ganglion

COLLATERAL (PREVERTEBRAL) GANGLIA (unpaired abdominal ganglia)

• Some preganglionic axons emerging from segments T5-L3 pass through


the ganglia of the sympathetic chain without synapsing and continue to an
unpaired prevertebral (or collateral) ganglion, a homologue of a
vertebral ganglion that is found in the abdominal cavity.

• There are 3 prevertebral ganglia (see Fig. 2):

o celiac
o superior mesenteric
o inferior mesenteric

196
Autonomic Nervous System 1 - Dr. Biber

Figure 2. AUTONOMIC NERVOUS SYSTEM

197
Autonomic Nervous System 1 - Dr. Biber

Figure 3. SYMPATHETIC NERVOUS SYSTEM: SCHEMATIC DETAIL

• Postganglionic neurons in these ganglia supply the visceral organs.


Targets include:

o smooth muscle of the visceral organs & blood vessels


o glandular tissue
o the enteric n. s. that lies within the wall of the GI tract.

198
Autonomic Nervous System 1 - Dr. Biber

ADRENAL MEDULLA

• The adrenal medulla is an endocrine organ made up of chromaffin cells


that liberate their products directly into the blood stream. The term
“chromaffin” refers to the colored reaction product seen in fixed tissue.

• Chromaffin cells are related embryologically to postganglionic


sympathetic neurons.

• The adrenal medulla is innervated by preganglionic fibers that pass


through vertebral ganglia of the sympathetic chain, continue through the
celiac ganglion and terminate on the adrenal medullary chromaffin cells.

SYNAPTIC ORGANIZATION OF SYMPATHETIC GANGLIA

Two important organizational features, referred to as divergence and


convergence, determine the functional properties of sympathetic ganglia:

• DIVERGENCE (Fig. 4) refers to the fact that preganglionic sympathetic


neurons innervate many postganglionic neurons (from 10 to 100) in a
given ganglion or in ganglia up to 3 or 4 segments away up or down the
sympathetic chain.

• Nerve impulse activity in sympathetic preganglionic neurons activates


multiple postganglionic neurons.

• This can be demonstrated by stimulating preganglionically at one


segmental level (white ramus) and recording postganglionic nerve activity
in grey rami at multiple segmental levels:

• The widespread effect of preganglionic activity matches the diffuse


distribution of sympathetic target tissues (sweat glands, hair follicles and
smooth muscle of blood vessels).

• CONVERGENCE (Fig. 5) refers to the innervation of individual ganglia or


individual postganglionic sympathetic neurons by many preganglionic
inputs originating from the same or different spinal segments. In other
words, many preganglionic inputs converge on a single sympathetic
ganglion or on a single postganglionic neuron:

• This can be demonstrated by electrical stimulation of preganglionic


neurons (white rami) at different spinal segmental levels. Impulse activity
set up in postganglionic nerves is recorded from the grey ramus at a given
segmental level:

199
Autonomic Nervous System 1 - Dr. Biber

Figure 4. DIVERGENCE OF PREGANGLIONICS

Figure 5. CONVERGENCE OF PREGANGLIONICS

• This arrangement means that the firing pattern of sympathetic


postganglionic neurons reflects the integration of information from
preganglionic sympathetic nerves at different spinal segmental levels.

200
Autonomic Nervous System 1 - Dr. Biber

SUMMARY OF ANATOMICAL CHARACTERISTICS OF SYMPATHETIC N.S.

• Thoraco-lumbar outflow
• Relatively short preganglionic axons
• Long postganglionic axon
• Anatomically distinct ganglion system with much divergence and
convergence of inputs

ORGANIZATION OF THE PARASYMPATHETIC N.S. (Fig. 6)

• Preganglionic axons (2-4 u diameter, myelinated) arise in specific cranial


nuclei or sacral segments S2 - S4) and travel out to the effector organs
where they synapse with postganglionic neurons that lie in or around the
effector organ tissue (Fig 6).

• With the exception of the parasympathetic postganglionic neurons in the


head, the parasympathetic postganglionic neuronal cell bodies are not
collected together in parasympathetic ganglia but are usually diffusely
distributed in the target tissue wall. Their axons are always short.

• Preganglionic parasympathetic nerves show little branching (divergence).


The ratio of pre to post ganglionic neurons is about 1:1 or 1:2.

• Parasympathetic action tends to be discrete and confined to single organs.

• Postganglionic parasympathetic nerves are just one among multiple types


of motor neuron found in the enteric nervous system that control gut
motility and secretory activity (see lecture IV, reflex control).

SUMMARY OF THE ANATOMICAL CHARACTERISTICS OF THE


PARASYMPATHETIC

• Cranio-sacral outflow
• Relatively long preganglionic axons
• Little branching of preganglionic axons (little divergence)
• Short postganglionic axons
• Except for head, no well organized parasympathetic ganglion system;
postganglionic neurons lie in or near target tissue

201
Autonomic Nervous System 1 - Dr. Biber

Figure 6. SCHEMATIC OF THE PARASYMPATHETIC NERVOUS SYSTEM

202
Autonomic Nervous System 1 - Dr. Biber

ORGANIZATION OF AUTONOMIC NEUROEFFECTOR JUNCTIONS (Fig. 7)

This differs strikingly from that of the skeletal NMJ. Important features include the
following:

• Extensive, highly branched networks of fine fibers that have a beaded or varicose
appearance. The small swellings (or varicosities) are the sites of transmitter
release.

• Variable distances from the varicose terminals and the plasmalemma of the target
effector tissues.

• Overlap among terminal networks from neighboring postganglionic neurons so


target tissues may be innervated by more than one postganglionic neuron (not one
to one as with the skeletal NMJ).

• Postsynaptic receptors widely scattered over the innervated organ. No specialized


region for postsynaptic receptors comparable to the skeletal muscle endplate.

• Receptors on effector cells are G-protein coupled, and can have inhibitory or
excitatory effects depending on the receptor subtype.

Figure 7:

203
Autonomic Nervous System 1 - Dr. Biber

COMPARISON OF SOMATIC AND AUTONOMIC NERVOUS SYSTEMS

SOMATIC AUTONOMIC
Target: skeletal muscle smooth muscle
heart
glands
Control: precise; rapid (msecs) less so (100 msecs to secs)

Anatomy: one neuron two neurons

Properties of motor wide diameter fine diameter


neurons: myelinated 2nd neuron unmyelinated
rapidly conducting slow conducting
15 - 50 m/sec 0.5 m/sec

Neuroeffector well defined & organized branching, diffuse terminals with variable
junction: nerve terminals close to distances from effector cell membrane
muscle endplate (15 nm (80 - 1000 nm)
gap) receptors not organized into a localized
Ach receptors tightly chemosensitive area
packed in endplate G-protein coupled signaling on the
ligand-gated ion channels effector cells

Transmitter: acetylcholine acetylcholine - parasympathetic


norepinephrine- sympathetic (exception
generalized sweat glands)

Receptors: Ligand-gated ion channel Ligand-gated ion channel on post


ganglionic neurons
G-protein-coupled signaling on the
effector cells

Effects: excitation excitation or inhibition


one to one relay modulatory effects

204
Autonomic Nervous System 1 - Dr. Biber

Figure 8. SCHEMATIC AUTONOMIC NERVOUS SYSTEM

205
Excitation-Contraction Coupling - Dr. Feher

Excitation-Contraction Coupling
Joseph Feher, Ph.D.
LECTURE OUTLINE:
I. Excitation-contraction coupling links the action potential to contraction.
A. Contraction begins with motor neuron excitation.
B. Action potential on the nerve leads to fusion of synaptic vesicles with the
nerve membrane.
C. Acetylcholine release from the vesicles changes the conductance of the
muscle membrane.
D. The muscle action potential is similar to the unmyelinated axon.
E. The muscle action potential is converted to an intracellular Ca2+ signal.

II. Calcium transients arise from T-tubule depolarization coupled to Ca2+ release
from SR stores.

A. T-tubule depolarization is required for activation.


B. A transient rise in Ca2+ follows the action potential but precedes force in a
muscle twitch.
C. The Ca2+ transient orginates from the sarcoplasmic reticulum.
D. The depolarization of the T-tubule is coupled to Ca2+ release through
specific proteins.

1. The dihydropyridine receptor in the T-tubule is a voltage sensor.


2. The ryanodine receptor in the SR terminal cisternae is the Ca2+
release channel.

E. The SR stores Ca2+ by binding it to calsequestrin in the lumen of the SR.


F. Release of Ca2+ from SR stores starts the Ca2+ transient and re-uptake ends
it.

1. RyR1 opens briefly.


2. The anatomic arrangement of the SR minimizes diffusion time.

206
Excitation-Contraction Coupling - Dr. Feher

3. The Ca2+ transient is returned to its baseline by Ca2+ pumping back


into the SR.

III. Cross-bridge cycling is controlled by myoplasmic [Ca2+].

A. Tropomyosin inhibits cross-bridge cycling in the relaxed muscle.


B. Troponin C binds Ca2+ cooperatively.
C. TnC binding of Ca2+ changes the configuration of TnC, TnI, TnT and
Tropomyosin on the thin filament.
D. TnC binding is a Ca2+ switch.

IV. Sequential SR release and summation of myoplasmic [Ca2+] explains summation


and tetany.

A. Ca2+ binding to TnC occurs later and lasts longer than the Ca2+ transient in
the twitch.
B. Force peaks later than the time course of Ca2+ binding to TnC.
C. The series elastic elements are responsible for the different time of force
and cross-bridges.

1. The series elastic element explains the delay in rise of force.


2. The series elastic element explains the delay in fall of force.

D. Repetitive stimulation releases more Ca2+ from remaining SR stores.


E. Summation of Ca2+ transients produces tetanus.

1. Usually all cross-bridges are activated in the twitch.


2. Summation of Ca2+ transients prolongs the time of TnC-Ca2+
binding.

V. Practice Questions

207
Excitation-Contraction Coupling - Dr. Feher

OBJECTIVES:
1. Describe the relative time courses of action potential, cytosolic [Ca] and force
during a twitch
2. Describe the origin of the cytosolic calcium transient during the twitch
3. Describe the anatomic location of the SR and identify terminal cisternae and
longitudinal SR
4. Describe the T-tubules with their location and their function in excitation-
contraction coupling
5. Name the voltage sensor on the T-tubule
6. Name the calcium release channel on the terminal cisternae of the SR
7. Name the protein that confers calcium sensitivity onto the contractile filaments,
and describe its location
8. Describe how the troponin complex regulates contraction through calcium
binding
9. Describe how the Ca transient returns to resting conditions
10. Describe how the twitch force lasts longer than the Ca transient or the fraction of
TnC occupied by Ca
11. Describe how repetitive stimulation produces greater force than the twitch
Suggested Reading: Berne and Levy, pp. 228-231; 236

I. EXCITATION-CONTRACTION COUPLING links the action potential to


contraction.

A. Contraction begins with motor neuron excitation

Lower motor neurons in the ventral horn of the spinal cord control skeletal
muscle fiber activation through the sequence of action potentials that reach
the neuromuscular junction. These action potentials on the nerve are
transformed into action potentials on the muscle surface membrane by
neuromuscular transmission.
B. Action potential on the nerve leads to fusion of synaptic vesicles with the
nerve membrane.

Depolarization of the pre-synaptic motor neuron membrane by an action


potential opens a Ca2+ channel on the presynaptic nerve membrane. This

208
Excitation-Contraction Coupling - Dr. Feher

begins a cascade of events that ends with the fusion of synaptic vesicles
with the presynaptic membrane, releasing acetylcholine into the cleft
between nerve and muscle.

C. Acetylcholine released from the vesicles changes conductance of the


muscle membrane.

Released acetylcholine diffuses to the post-synaptic membrane, the


sarcolemma of the muscle fiber, where it binds to the two α sub-units of
the acetylcholine receptor, which is also an ion channel consisting of 5
subunits. In response to acetylcholine binding, the ligand-gated channel
increases its conductance to both Na+ and K+, resulting in an inward
current carried by Na+ that depolarizes the cell, forming the end-plate
potential. The depolarization spreads passively, or electrotonically, to the
nearby patch of sarcolemma. When the end-plate potential reaches
threshold, it starts an action potential on the muscle membrane
D. The muscle action potential is similar to the unmyelinated axon.

The muscle cell membrane, or sarcolemma, contains voltage-sensitive Na+


and K+ channels. It has regular deep invaginations, the transverse or T-
tubules, that also are excitable. The end-plate potential produces an action
potential on the muscle cell membrane that propagates in both directions
away from the neuromuscular junction and is simultaneously conducted
into the interior of the cell along the T-tubules.
E. The muscle action potential is converted to an intracellular Ca2+ signal.

The depolarization conducted along the T-tubules into the interior of the
cell is not actually inside the cell because the T-tubule lumen is continuous
with the extracellular fluid. The depolarization of the T-tubules releases
Ca2+ from intracellular stores into the myoplasm, which transiently raises
the Ca2+ concentration. The increased intracellular Ca2+ concentration
activates the contractile filaments. The process by which depolarization of
the T-tubule is converted to an intracellular Ca2+ signal and the regulation
of contraction by Ca2+ is called excitation-contraction coupling, E-C
coupling.
II. CALCIUM TRANSIENTS arise from T-tubule depolarization coupled to
Ca2+ release from SR stores.

A. T-tubule depolarization is required for activation.

Many studies not described here have demonstrated that depolarization of


the T-tubule is an essential link in E-C coupling.

209
Excitation-Contraction Coupling - Dr. Feher

B. A transient rise in Ca2+ follows the action potential but precedes force in a
muscle twitch.

Ashley and Ridgway first demonstrated that Ca2+ transients follow the
action potential but precede force in barnacle muscle. They used a
luminescent probe, aequorin, that emits light when it binds Ca2+. The time
course of the Ca2+ is consistent with the action of Ca2+ as a second
messenger between action potential and contractile activity. (See Fig. 1)

Figure 1. Duration of muscle action potential, Ca2+ transient


and force in a muscle twitch. The action potential is shortest,
lasting some 2-5 ms. The Ca2+ transient follows the action
potential and force is developed later still.

C. The Ca2+ during E-C coupling originates from the sarcoplasmic reticulum

The rise in myoplasmic [Ca2+] upon depolarization of the sarcolemma in


skeletal muscle can occur in the absence of extracellular Ca2+. This Ca2+
originates from some intracellular store, the sarcoplasmic reticulum, or
SR. The SR is a network of a sac-like, membrane-bound organelle that
surrounds the myofibrils, contains loads of Ca2+, and makes close contact
with the T-tubules in structures called triads, consisting of one T-tubule
and two terminal cisternae. The terminal cisternae are a specialized
region of the SR that forms a continuous sac near its junction with the T-
tubule. The triads are found at the junction of the A-band and I- band in
skeletal muscles. In electron micrographs, foot structures can be seen
connecting the T-tubule with the terminal cisternae. The SR does not form
a continuous sleeve over the myofibrils. The SR has “windows” in its
surface to allow for transport of metabolites, such as ATP, ADP and Pi,
into and out of the myofibril.

210
Excitation-Contraction Coupling - Dr. Feher

Figure 2. Arrangement of the sarcoplasmic reticulum (SR) membranes around the myofibril. Typically the T-tubule
penetrates the muscle fiber at the level of the A-I junction and makes close contact with two terminal cisternae of the
SR. Together the junction of T-tubule and SR is called a triad. A large protein forms junctional “feet” structures, visible
in electron micrographs, between terminal cisternae and T-tubule. All myoplasmic surfaces of the SR, except that
facing the T-tubule, are covered with “bumps” visible in electron micrographs, and these are thought to represent the
major protein constituent of the SR, the SERCA1 Ca-ATPase pump.
D. The depolarization of the T-tubule is coupled to Ca2+ release through
specific proteins.
1. The dihydropyridine receptor in the T-tubule is a voltage sensor.

Depolarization of the T-tubule is the primary muscle signal for


contraction. A protein in the T-tubule called the dihydropyridine
receptor, abbreviated as DHPR, detects the T-tubule depolarization
and transmits its signal to the SR. The DHPR consists of 5 subunits;
one subunit forms a Ca2+ channel. The DHPR is associated with L-
type Ca2+ channel activity, but Ca2+ influx through the DHPR is
unnecessary for E-C coupling in skeletal muscle. There is close
physical contact between the DHPR and the ryanodine receptor
(RyR) in the SR terminal cisternae membrane. Thus, the DHPR is a
voltage sensor that triggers Ca2+ release from the SR. Present results
favors direct interaction of the DHPR with RyR.
2. The Ryanodine receptor in the SR terminal cisternae is the Ca2+
release channel.

The ryanodine receptor is named because it binds a plant alkaloid,


ryanodine, with high affinity. There are several different isoforms.

211
Excitation-Contraction Coupling - Dr. Feher

RyR1 is the type in skeletal muscle; RyR2 is in the heart and brain,
and RyR3 is present in epithelial cells, smooth muscle and also in the
brain. The RYR1 is a large protein of 565 KDa. It associates as a
tetramer in the SR membrane, forming a structure of about 2 million
Da. The purified RyR1 forms channels in lipid bilayers with large
conductances and gating characteristics consistent with its role in E-C
coupling. The DHPR interacts either directly with RyR1 or through
another protein to trigger the opening of RyR1 and subsequent release
of Ca2+ stored within the lumen of the SR.
E. The SR stores Ca2+ by binding it to calsequestrin in the lumen of the SR.

The SR is a membrane-bound organelle with a distinct inside and outside.


The outside faces the myoplasm; the inside forms and faces the lumen.
Inside the SR is a 63 KDa protein called calsequestrin. Calsequestrin
contains many acidic residues that bind Ca2+ with low affinity (Kd = 10-
3
M) but high capacity. Calsequestrin allows the SR to store a lot of Ca2+
at relatively low concentrations.
F. Release of Ca2+ from the SR starts the Ca2+ transient and re-uptake back
into the SR ends it.
1. RyR1 opens only briefly

The depolarization of the T-tubule is brief, and so is the time of Ca2+


release from the SR – just a few milliseconds. Ca2+ released from the
SR then diffuses through the myofibril.
2. The anatomic arrangement of the SR minimizes diffusion times.

Muscle contraction must occur rapidly after signaling from the nerve,
and all of the sarcomeres should be activated in unison. The delay is
minimized by the presence of the T-tubules to bring excitation into the
middle of the muscle fiber with minimal delay, and by the punctate
and distributed sites of Ca2+ release located close to the point of Ca2+
regulation on the myofilaments. Because there are two triads per
sarcomere, the largest distance from myofilaments to Ca2+ release sites
is typically 1 :m or less. Diffusion across this short distance requires
about 1 ms.

212
Excitation-Contraction Coupling - Dr. Feher

Figure 3. Cartoon schematic of the Ca2+ movements that cause the rise of Ca2+ in excitation of skeletal
muscle. The depolarization of the T-tubule is sensed by the dihydropyridine receptors (DHPR) on the T-tubule.
This information is passed on to the ryanodine receptors (RyR) that are in close physical contact with the
DHPR. The RyR is the Ca2+ release channel that opens briefly, allowing a pulse of Ca2+ to leave the SR. The
bulk of the released Ca2+ can bind to and activate the myofilaments because the SERCA pump on the SR pumps
Ca2+ relatively slowly compared to release.
3. The Ca2+ transient is returned to its baseline by Ca2+ re-uptake back
into the SR.

The Ca2+ transient begins by Ca2+ release from the SR. To relax the
muscle, the activator Ca2+ is taken back up into the SR by a calcium
pump. This is a membrane protein of 110 KDa that links the
hydrolysis of ATP to ADP and Pi with the pumping of 2 Ca2+ atoms
back into the SR lumen. This calcium pump is also referred to as the
SR Ca-ATPase and is also called SERCA, for smooth endoplasmic
reticulum calcium ATPase. There are several different isoforms in
different tissues. The one in fast twitch muscle (Type II) is called
SERCA1.

213
Excitation-Contraction Coupling - Dr. Feher

Figure 4. Cartoon schematic of the Ca2+ movements that bring an end to the Ca2+ transient and thereby bring
about relaxation. The RyR is closed shortly after it opens to release stored Ca2+. The SERCA calcium pump links
the hydrolysis of ATP to ADP and Pi, consuming free energy, to the active transport of Ca2+ from the myoplasm
to the SR lumen. The concentration of Ca2+ in the resting myoplasm is between 10-8 and 10-7 M, whereas [Ca2+]
in the SR lumen is between 10-3 and 10-2 M, so there is a tremendous concentration of Ca2+ within this organelle.
The rate of decline of the Ca2+ transient depends on the magnitude of the pulse of released Ca2+ , the rate of
pumping of each SERCA unit and the number of pump units.

III. CROSS-BRIDGE CYCLING is controlled by myoplasmic [Ca2+]

A. Tropomyosin inhibits cross-bridge cycling in the relaxed muscle.

During relaxation there is some overlap of the thin and thick filaments, but
there is no force development because tropomyosin is in the way of
myosin interacting with the actin filament. Recall the structure of the thin
filament: it is a double strand of actin molecules with a half-repeat of
about 7 actins spanning some 38.5 nm. Tropomyosin is a long rod-shaped
heterodimer molecule 38.5 nm long that lies along the filament. The
troponin complex of TnT, TnI and TnC resides at one end of the

214
Excitation-Contraction Coupling - Dr. Feher

tropomyosin. At low myoplasmic [Ca2+] the tropomyosin blocks the


myosin head from interacting with the actin on the thin filament, and there
is no cross-bridge formation.
B. Troponin C binds Ca2+ cooperatively.

Troponin C has four binding sites for Ca2+. Two of these are called high-
affinity or “Ca-Mg” sites. The other two sites have a lower affinity for
Ca2+ and a much lower affinity for Mg2+, and are therefore called “Ca-
specific sites.” The Ca-Mg sites are always occupied and therefore they
do not participate in regulation of TnC. Ca2+ binding to the Ca-specific
sites regulates TnC configuration.
C. TnC binding of Ca2+ changes the configuration of TnC, TnI, TnT and
Tropomyosin on the thin filament.

Binding of Ca2+ induces a change in the conformation of TnC. Because


TnC connects TnT with TnI, all three shift their position. Because TnT
binds tropomyosin, tropomyosin also shifts its position. Tropomoyosin
movement uncovers the binding site for myosin on the actin, allowing
cross-bridges to form. Binding to actin activates myosin ATPase activity
and cross-bridge cycling, producing force or shortening.

Figure 5. Ca-control of actomyosin ATPase activity or force. No force is


generated at resting myoplasmic [Ca2+] of 10-7 M, whereas maximum force is
generated at peak [Ca2+] of about 10-5 M.

215
Excitation-Contraction Coupling - Dr. Feher

D. TnC binding of Ca2+ acts as a Ca2+ switch.

The steep dependence of force on [Ca2+] (see Fig. 5) indicates that the
myoplasmic [Ca2+] switches the contractile filaments from an inactive state at
rest, when the myoplasmic [Ca2+] is less than 10-7 M, to an active state when the
Ca2+ transient raises the myoplasmic [Ca2+] to 5 x 10-6 M.

Figure 6. Calcium control of actin-myosin interaction. In relaxed muscle, tropomyosin blocks actin-myosin
interaction and is held in place by TnT, which binds tropomyosin, TnC, which binds calcium, and TnI, which inhibits
the interaction of actin and myosin. In the presence of micromolar Ca2+ brought about by release of Ca2+ from the SR,
TnC changes its conformation and re-arranges TnI and TnT so that tropomyosin moves out of the way of the myosin
binding site on actin. Thus, actin and myosin can interact and engage in cross-bridge cycling to consume ATP and
shorten or develop force.
IV. SEQUENTIAL SR RELEASE AND SUMMATION OF MYOPLASMIC
[Ca2+] explains summation and tetany.

A. Ca2+ binding to TnC occurs later and lasts longer than the Ca2+ transient in
the twitch.

Because it takes some time for Ca2+ to bind to TnC, there is a delay
between the rise in the Ca2+ transient and TnC binding with Ca2+ during a
single twitch. Since TnC binds after the free [Ca2+] increases, it seems
reasonable that the peak of TnC-Ca follows the peak in free [Ca2+].

Release of Ca2+ from TnC is also delayed from the fall in the free Ca2+
transient . The drop in the myoplasmic [Ca2+] is brought about by active
transport of Ca2+ back into the SR through the operation of the SERCA Ca
pump. This pump can transport only that Ca2+ that diffuses to it - the free
Ca2+. As free [Ca2+] drops, Ca2+ dissociates from the TnC-Ca2+ complex.

216
Excitation-Contraction Coupling - Dr. Feher

This dissociation reaction is even slower than the binding reaction.


Therefore, TnC remains bound with Ca2+ significantly longer than the
Ca2+ transient.
B. Force development peaks later than the time course of Ca2+ binding to
TnC.

If actin and myosin interaction result in cross-bridge cycling that produces


shortening or force, and these interactions depend on Ca2+ binding to TnC,
we ought to expect that the force developed by muscle will rise and fall
with Ca2+ binding to TnC. This, however, is not the case, as can be see in
Fig. 7 below.

Figure 7. Time course of Ca2+ transient, [TnC-Ca], the concentration of


TnC with bound Ca2+, and force during the muscle twitch. The force is
delayed due to the series elastic elements in the muscle.
C. Elastic properties of the muscle are responsible for the different time
course of force and cross-bridge cycling during the twitch.
1. The series elastic element explains the delay in the rise of force during
a twitch.

Rapid stretch of a fiber after contraction has begun produces a much


stronger force, and rapid release during contraction reduces the force
to zero. Therefore, the force-generators in the muscle appear to be in
series with a spring, as shown in Fig. 8., called the series elastic
element. Force developed by the contractile elements first stretches
the spring. The force transmitted to the outside of the muscle depends
on the length of the spring as determined by the length-tension
characteristics of this spring. Thus there is a delay in the force that is
transmitted from the cross-bridges to the outside of the muscle because
the force is conditioned by the series elastic elements.

2. The series elastic component explains the delay in the fall of force
during a twitch.

217
Excitation-Contraction Coupling - Dr. Feher

Similarly, the series elastic element remains stretched when Ca2+


dissociates from TnC and the cross-bridge cycling stops. Thus, the
force transmitted to the outside of the muscle remains elevated for a
time after the force generators have stopped producing force.

Figure 8. Series and parallel elastic components of muscle contraction. The muscle behaves as if there
are series elastic components in series with the contractile filaments and other elastic components
in parallel with it. The identification of the proteins responsible for these elastic behaviors is not known.
There is some evidence that the series elastic behavior resides in the contractile filaments themselves.
D. Repetitive stimulation causes repetitive Ca2+ release from the SR because
Ca2+ release during a single twitch does not exhaust SR Ca2+ stores.

The RyR opens very briefly to form a large conductance pathway for Ca2+
exit from the SR. Because of its large concentration gradient from the
lumen to myoplasm, Ca2+ comes rushing out. However, because of its
brief opening, only part of the SR Ca2+ store is released in a single
contraction. Since the depolarization of the T-tubule is even shorter than
the opening of the RyR, The RyR can release more Ca2+ from the SR
store by repetitive stimulation. This results in a transfer of the SR Ca2+
store to the myoplasm, with a summation and prolongation of the Ca2+
transients. The frequency of stimulation determines if this summation
forms a continuously high myoplasmic [Ca2+] or an oscillating
myoplasmic [Ca2+].
E. Summation of Ca2+ transients produces tetanus.
1. Usually all cross-bridges are activated in the twitch.

As indicated in Fig. 7, during a twitch usually all 70 :M of the TnC is


occupied by two Ca2+ at the regulatory sites, giving a total of some 140
:M Ca2+ bound to TnC. However, this Ca2+ does not stay bound for
long and there is insufficient time for the cross-bridges to fully stretch
the series elastic elements to produce maximal tension.

218
Excitation-Contraction Coupling - Dr. Feher

2. Summation of Ca2+ transients prolongs the time of TnC-Ca2+ binding.

The summation of the Ca2+ transients by repetitive stimulation of Ca2+


release from the SR exposes TnC to a more continuous high
myoplasmic [Ca2+]. This favors binding of Ca2+ to TnC and prolongs
the activation of the cross-bridges. This in turn allows more time for
cross-bridge cycling and more time to fully stretch the series elastic
elements. When the series elastic elements are stretched fully,
maximum tension is reached. Maximum tension is reached at the
tetanus. Thus, the origin of tetanus is in the summation of the Ca2+
transients (see Fig. 9).
3. Summation begins when stimulation frequency equals the frequency of
the twitch

Summation begins when the next stimulation of the muscle occurs just
before the muscle has completely relaxed. The time between initial
stimulation and complete relaxation is the twitch time. The frequency
of any cyclic event is the inverse of its period, p, which in this case is
the twitch time. Thus we should expect summation to begin when the
frequency of stimulation is just a bit higher than 1/p.

Figure 9. Summation of Ca2+ transients and tetanus. The Ca2+ transient is shorter than force
development in the twitch. Rapid repetitive stimulation can elicit a second release of Ca2+ from remaining
SR Ca2+ stores before the Ca2+ transient is finished. Although the amount of released Ca2+ in subsequent
releases may diminish, because of diminished SR Ca2+ remaining in the SR, the total myoplasmic [Ca2+]
increases. Rapid repetitive stimulations results in a transfer of Ca2+ from the SR to the myoplasm. During
the twitch the cross-bridges are fully activated, but there is insufficient time for force to stretch the series
elastic elements. During prolonged and rapid repetitive stimulation (tetanus) force has time to stretch
these elements and to fully transmit the force to the outside of the muscle. At intermediary stimulation
frequencies muscle force is partially fused.

219
Excitation-Contraction Coupling - Dr. Feher

V. PRACTICE QUESTIONS
1. Calsequestrin
A. Detects depolarization of the T-tubule
B. Opens the ryanodine receptor Ca2+ channel
C. Binds Ca2+ in the lumen of the SR
D. Binds Ca2+ in the cytoplasm of muscle fibers
E. Confers Ca2+ sensitivity on the myofilaments

2. Selected proteins involved in excitation-contraction coupling follow the


sequential order from first to last as (AChR = acetylcholine receptor;
DHPR = dihydropyridine receptor; RyR = ryanodine receptor; SERCA1a
= SR Ca-ATPase; TnC = troponin C;)

A. DHPR, AChR, RyR, TnC, SERCA1a


B. RyR, TnC, SERCA1a, DHPR, AchR
C. AChR, DHPR, RyR, TnC, SERCA1a
D. AChR, DHPR, SERCA1a, RyR, TnC
E. AChR, DHPR, RyR, SERCA1a, TnC

3. Inhibition of interaction of actin and myosin during resting muscle is


directly provided by

A. TnC
B. TnI
C. TnT
D. Tropomyosin
E. Actin

220
Excitation-Contraction Coupling - Dr. Feher

4. The Ca2+ transient during the twitch falls while muscle force is still rising
because

A. Occupancy of TnC with Ca2+ is falling


B. The series elastic components are still being stretched further
C. Ca2+ release from the SR continues
D. The Ca2+ transient tracks only free Ca2+ and not bound Ca2+
E. The series elastic elements

5. The force during a tetanus is

A. Much higher than the twitch force because the series elastic
elements are fully stretched
B. Much higher than the twitch force because all muscle fibers are
recruited
C. Much higher than the twitch force because all of the TnC is
saturated with Ca2+ only during tetanus
D. The same as the twitch force because all TnC is saturated with
Ca2+ even during a twitch
E. The same as the twitch force because muscle force is proportional
to cross-sectional area alone
Answers: 1C; 2C; 3D; 4B; 5A

221
Autonomic Nervous System 2 & 3 - Dr. Biber

Autonomic Nervous System 2 & 3: Physiology & Molecular


Mechanisms
Margaret C. Biber, D. Phil.

OBJECTIVES:

Please note that these objectives pertain to ANS lectures I-IV.

At the end of these lectures you should know and understand the following material:

1. The relationship between the organization of the sympathetic and parasympathetic


divisions of the ANS to their overall physiological effects.
2. Anatomical and functional differences between the skeletal neuromuscular
junction and autonomic neuroeffector junctions.
3. Transmitters used at ganglionic and neuroeffector junctions and highlights of the
transmitter life cycle: Storage, release, biological inactivation, metabolism and de
novo synthesis for acetylcholine (Ach), norepinephrine (NE) and the hormone,
epinephrine (EPI).
4. Receptor types for Ach and the catecholamines, NE and EPI and their effects.
5. The mechanism of action of other transmitters/mediators including ATP, NO and
peptides.
6. The organization of autonomic reflexes.
7. The overall physiological effects of the parasympathetic and sympatho-adrenal
systems and the receptor types that mediate the responses.

Reading: Berne, Levy, Koeppen and Stanton: Physiology, 5th edition. 2004; Ch. 11,
Pages 206-215 –Table 11-1 is too detailed. Use Table in handout.
Costanzo: Physiology, 2006 Ch. 2, Pages 45-64

Note: Please follow the version in the handout wherever discrepancies exist between the textbooks and the
handout.

LECTURES II & III OUTLINE

COMPARISON OF SYMPATHO-ADRENAL & PARASYMPATHETIC


Sympatho-adrenal:
diffuse targets
expenditure of energy
Parasympathetic:
discrete targets
conservation of energy
MOLECULAR BASIS FOR ACTIONS OF ANS:
TRANSMITTERS:
Identity & sites of release

222
Autonomic Nervous System 2 & 3 - Dr. Biber

Life cycle of ACh


Life cycle of NE in sympathetic varicosity
Evoked release of small transmitters and peptides
Adrenal medulla: storage, release, synthesis and metabolism
Pheochromocytoma
RECEPTORS
Cholinergic
Nicotinic & Muscarinic
Parasympathetic effects
Adrenergic
Alpha & Beta
Sympatho-adrenal effects
Summary of effects of sympathoadrenal and parasympathetic on target tissues (Table 1)

COMPARISON OF SYMPATHETIC AND PARASYMPATHETIC

The ANS maintains man and adapts him to his environment but the sympathetic and
parasympathetic nervous systems have rather different roles:

• The sympathetic nervous system mobilizes the body for activity and, in the
extreme case, in conjunction with the adrenal gland, allows the body to handle
threatening situations (FIGHT or FLIGHT).
• The parasympathetic plays a restorative role and conserves energy.
• Differences exist in the targets of these two systems:

The sympathetic innervates widely distributed systems.


The parasympathetic exerts a more discrete control.

It is easier to remember the actions of these two systems at the level of individual organs
and tissues if one first has an understanding of their overall function and can then relate
this to their organization.

SYMPATHETIC NERVOUS SYSTEM

ƒ Diffuse Target Tissues


• sweat glands
• smooth muscle of blood vessels supplying skeletal muscle, skin
• smooth muscle of hair follicles

In man, these target tissues do not have any parasympathetic innervation. The
sympathetic has excitatory actions on these tissues. Consequently, it regulates:

• blood pressure (blood vessels supplying the skeletal muscle are especially
important; the ANS innervation to heart also contributes)
• the distribution of blood among tissues and within tissues
• body temperature (cutaneous blood vessels; sweat glands)

223
Autonomic Nervous System 2 & 3 - Dr. Biber

Figure 1. AUTONOMIC NERVOUS SYSTEM

224
Autonomic Nervous System 2 & 3 - Dr. Biber

ƒ STRESS RESPONSE

Together with the adrenal gland, the sympathetic NS mobilizes the body to handle
threatening situations -- the fight or flight reaction. The effects of the sympatho-
adrenal enable the body to undertake severe physical exertion. The activity of organs that
are nonessential or counterproductive is inhibited. Targets of sympatho-adrenal action
include:

• Cardiovascular system:
o redistribution of blood, e.g., flow of blood to the skin and mesentery is
dramatically reduced, flow to skeletal muscle is enhanced.
o increase in cardiac output
• Respiratory system:
o relaxation of muscles of trachea and bronchi
• Digestive system:
o inhibition of motility and secretions
• Metabolism:
o mobilization of glucose,
o increased lipolysis,
o increase in basal metabolic rate

PARASYMPATHETIC NERVOUS SYSTEM

ƒ Conservation/Replenishment of Energy Supplies, Maintenance of the


Organism.
ƒ Discrete Control of Individual Target Tissues

Examples of parasympathetic regulation include:

• excitatory effects on the gastrointestinal tract (increases motility and secretory


activity in sequential fashion as digestion proceeds)
• stimulation of glandular secretions (but sympathetic controls sweat glands)
• slowing of the heart
• control of pupil diameter by the pupillary light reflex (regulates the amount of
light falling on the retina)
• accommodation of the lens for near vision
• voiding the urinary bladder (micturition)

225
Autonomic Nervous System 2 & 3 - Dr. Biber

SUMMARY OF SYMPATHO-ADRENAL AND PARASYMPATHETIC


EFFECTS BY TARGET TISSUE
HEART : sympatho-adrenal has excitatory effects: it increases
the rate of beating and the force of contraction
parasympathetic is inhibitory, slowing heart rate

SMOOTH sympatho-adrenal can excite or inhibit smooth muscle


MUSCLE: (adrenal medulla relaxes bronchial smooth muscle; sympathetic
constricts
vascular smooth muscle).
parasympathetic excites most of the smooth muscle it
innervates (e.g GI tract; urinary bladder)

GLANDS: parasympathetic stimulates glandular secretions


(sympathetic stimulates sweat glands)

METABOLIC
EFFECTS: mediated by the sympatho-adrenal system

To understand the basis for the different actions of the sympathetic and
parasympathetic NS on their target organs requires an understanding of the
molecular mechanisms that mediate these effects, including the transmitters
released at the neuroeffector junction, and the receptors with which they interact.

226
Autonomic Nervous System 2 & 3 - Dr. Biber

Figure 2. AUTONOMIC NERVOUS SYSTEM: TRANSMITTERS

MOLECULAR BASIS FOR PHYSIOLOGICAL ACTIONS OF ANS

TRANSMITTERS AND SITES OF RELEASE (FIG. 2)

ƒ Acetylcholine (Ach)
• Ganglion: Terminals of both parasympathetic and sympathetic
preganglionic nerves release ACh
• Neuro-Effector Junction: Terminals of parasympathetic postganglionic
nerves release ACh onto the target effector tissues
Terminals of sympathetic postganglionic nerves that supply the sweat
glands that cover the body (generally distributed sweat glands) also release

227
Autonomic Nervous System 2 & 3 - Dr. Biber

ACh.

ƒ Norepinephrine (NE)
• Neuro-Effector Junction: Terminals of sympathetic postganglionic
nerves release the catecholamine, NE onto target effector tissues, (except
in the case of generally distributed sweat glands).

HORMONE RELEASE from adrenal medulla

ƒ Epinephrine (EPI), a catecholamine (CA) closely related to NE, is the principal


CA released into the blood stream along with small amounts of NE.

ƒ The methyl substituent on the amine group accounts for EPI’s characteristic
properties that distinguish it from NE.

LIFE CYCLE OF TRANSMITTERS/HORMONES: ACETYLCHOLINE

• ACh is stored in small clear (agranular) vesicles that also contain high
concentrations of ATP (FIG 3).
• A few large dense cored vesicles that store peptides such as vasoactive intestinal
peptide (VIP) are also present. Peptide is released with high frequency stimulation
and augments the effects of Ach on the target organ.
• Released ACh interacts with receptors and is rapidly destroyed within msecs of its
release through metabolic breakdown by acetylcholinesterase (FIG 3), one of the
most rapidly acting enzymes in the body.
• Inhibition of acetylcholinesterase potentiates and prolongs the effects of ACh in
the ANS.
• ACh supplies are replenished by choline acetyltransferase from choline and
acetylCo A. There is a sodium dependent choline uptake mechanism into the
nerve terminal.

228
Autonomic Nervous System 2 & 3 - Dr. Biber

Figure 3. CHOLINERGIC NERVE VARICOSITY

NOREPINEPRINE (FIG 4)

• NE in sympathetic postganglionic nerve terminal varicosities is stored mostly in


small dense cored vesicles that contain NE, ATP and dopamine beta hydroxylase
(converts dopamine to NE)(FIG 4). Some large dense cored vesicles contain
enkephalin as well as all the other components.
• A separate population of large dense cored vesicles stores peptides such as
neuropeptide Y that is released with high frequency stimulation and enhances the
effect of NE.
• Released NE is biologically inactivated by reuptake by a sodium dependent
transporter molecule present in the membrane of postganglionic sympathetic
nerve terminal varicosities.

229
Autonomic Nervous System 2 & 3 - Dr. Biber

• Once inside the sympathetic nerve terminal, NE is either taken up into a storage
vesicle for subsequent release or metabolized.
• Inhibition of reuptake (e.g. with cocaine) potentiates the effects of NE.
• Supplies of NE are maintained by reuptake and by synthesis of new transmitter
(see below).

Figure 4. POSTGANGLIONIC SYMPATHETIC NERVE VARICOSITY

230
Autonomic Nervous System 2 & 3 - Dr. Biber

Figure 5. EVOKED RELEASE OF TRANSMITTER

231
Autonomic Nervous System 2 & 3 - Dr. Biber

EVOKED RELEASE OF TRANSMITTER

• Requires calcium entry into the nerve terminal and occurs by exocytosis.
• Involves the same quantal release mechanism described for the motor nerve at the
skeletal NMJ (see Dr DeSimone’s notes), BUT the probability of release of
quanta is much lower than for somatic motor nerves and hence the amount of
transmitter released by a single action potential is less.
• Substances stored in the same vesicle are released together. For example, ATP
and NE are coreleased from NE storing vesicles of postganglionic sympathetic
nerves.
• Peptides stored in separate large dense cored vesicles (e.g. VIP) are only released
in response to high frequency stimulation.
• The neuronal firing pattern (e.g. low frequency versus higher frequency of action
potentials) determines the mixture of transmitters released and hence the nature of
the chemical signal received by the target tissue.

THE ADRENAL MEDULLA

• In the adrenal medulla, EPI (or NE) is stored in large dense cored vesicles. These
vesicles also contain:
o high concentrations of ATP
o peptides, leu and met enkephalin
o high MW proteins including:
an enzyme, dopamine-beta-hydroxylase
highly acidic proteins, the chromogranins

• In response to splanchnic nerve activity, CA and other vesicle contents are


released directly into the blood stream from the adrenal medulla and act on targets
at a distance.
• The half life of circulating EPI is approximately 10 secs. To produce a prolonged
effect, the adrenal must continue to release amine.
• Stores of EPI (and NE) are replenished by resynthesis. The steps in CA synthesis
are as follows (Figure 6):

232
Autonomic Nervous System 2 & 3 - Dr. Biber

Figure 6. SYNTHESIS AND STORAGE OF CATECHOLAMINES

• Substrate, tyrosine, an aromatic amino acid, is made by hydroxylation of


phenylalanine by a liver enzyme, PHENYLALANINE HYDROXYLASE.
(Lack of phenylalanine hydroxylase causes phenylalanine to accumulate in the
blood stream. An elevated blood level of phenylalanine interferes with uptake of
other amino acids into the CNS and, if untreated, produces mental retardation.
The disorder is known as phenylketonuria (PKU), and is treated by dietary
restriction of phenylalanine). Tyrosine made from phenylalanine is taken up from
the circulation by sympathetic nerve terminals or adrenal chromaffin cells.
• TYROSINE HYDROXYLASE converts tyrosine to dopa (3,4-
dihydroxyphenylalanine). This enzyme reaction is highly regulated and hence is
rate limiting for CA synthesis. Tyrosine hydroxylase activity increases
dramatically with nerve stimulation. The increase in activity involves rapid
phosphorylation by protein kinase A.
• Prolonged stimulation of the sympatho-adrenal system (e.g by severe stress)
increases the levels of tyrosine hydroxylase protein. This adaptive response is
critical for maintaining NE levels in sympathetic nerves and EPI in the adrenal
medulla.

233
Autonomic Nervous System 2 & 3 - Dr. Biber

• Several additional steps follow the hydroxylation of tyrosine:


• Dopa is decarboxylated to dopamine by the cytoplasmic enzyme DOPA
DECABOXYLASE (also called AROMATIC AMINO ACID
DECARBOXYLASE)
• Dopamine is pumped into the storage vesicle where it is converted to NE by
DOPAMINE-BETA-HYDROXYLASE. Synthesis stops here in sympathetic
postganglionic nerve terminals.
• EPI is formed by methylation of NE on the amino group by
PHENYLETHANOLAMINE-N-METHYL TRANSFERASE (PNMT).
• PNMT is a cytoplasmic enzyme. NE leaves the storage vesicle and is N-
methylated in the cytoplasm. The resulting EPI is then pumped back into the
vesicle where it remains until released.
• GLUCOCORTICOID from the adrenal cortex is essential for EPI synthesis.
Glucocorticoid both induces and stabilizes PNMT. Glucocorticoids reach the
adrenal medulla from the venous outflow of the surrounding adrenal cortex.
• Extra-adrenal chromaffin tissue or tumors of the adrenal (pheochromo-cytomas)
are not exposed to high glucocorticoid levels and therefore make only NE.
• CA released into the circulation from the adrenal medulla are biologically
inactivated by metabolism in well vascularized tissues such as liver and kidney.
• Metabolism involves 3-O-methylation on the aromatic ring by catechol-O-
methyltransferase (COMT) and oxidative deamination. The amino group is
removed by monoamine oxidase (MAO) a mitochondrial enzyme. The resulting
aldehyde is converted to an acid by aldehyde dehydrogenase.
• The acid, 3-methoxy-4-hydroxymandelic acid (also called vanilmandelic acid or
VMA) is excreted in the urine.

234
Autonomic Nervous System 2 & 3 - Dr. Biber

EXCRETION OF CATECHOLAMINES (CA) AND METABOLITES

• Over a 24h period, approx. 90% of CA is excreted as the acid, VMA (2000 - 9000
ug/24h), 10% as O-methylated NE or EPI.
• Small amounts of NE released from sympathetic nerves supplying blood vessels
appear in urine (10-70 ug/24h).
• EPI and its O-methylated product, metanephrine come from the adrenal but make
up less than 10% of urinary CA or metabolites.
• PHEOCHROMOCYTOMAS are tumors of adrenal chromaffin tissue. They
may secrete high levels of CA and produce life threatening episodes of
hypertension (high BP). Elevated urinary levels of CA and their metabolites are
diagnostic.
• Treatment is by surgical removal or, when surgery is not possible, with alpha
blocking agents (see below).

RECEPTORS

• These are found on the post synaptic (ganglionic) or post junctional (effector) cell
membrane and interact with transmitters released from the nerve terminal.

• Receptors function as a coding system. They show a high degree of specificity.


Thus, receptor molecules which bind ACh do not bind NE and vice versa.
• The response produced (excitation, inhibition) is determined by the properties
of the receptor and its transduction mechanism.

CHOLINERGIC RECEPTORS RESPOND TO ACh

Two broad classes of cholinergic receptor exist, NICOTINIC and MUSCARINIC.


(FIG 7)

NICOTINIC RECEPTORS

Signaling by ACh in AUTONOMIC GANGLIA is mediated by NICOTINIC


RECEPTORS that lie on cell bodies of sympathetic and parasympathetic
postganglionic neurons.

• Nicotinic receptors are excited by ACh.


• They are also excited and then blocked by NICOTINE. Hence, their name,
cholinergic receptors of the nicotinic type or simply nicotinic receptors.
• Nicotinic receptors gate a cation selective ion channel. When ACh binds to the
receptor the channel opens, sodium and potassium ions pass down their
electrochemical gradients and the postganglionic neuron is depolarized.
• Transmission is FAST. Action potentials in the preganglionic nerve always
generate action potentials postganglionically.

235
Autonomic Nervous System 2 & 3 - Dr. Biber

• The ganglionic nicotinic receptors are not identical to those at the endplate of the
skeletal NMJ. They show differences in sensitivity to blocking drugs (antagonists)
and slightly different subunit structure.

MUSCARINIC RECEPTORS

Signaling by ACh at EFFECTOR CELLS is mediated by MUSCARINIC


RECEPTORS. These receptores are named after the alkaloid muscarine that mimics the
action of ACh.

• Muscarinic receptors are G-protein coupled. Numerous receptors subtypes have


been cloned.
• ACh produces excitation of smooth muscle and glands and inhibition of the
heart (slowing or bradycardia).
• The inhibitory receptor in the heart is coupled via a G protein to a potassium
channel that opens in the presence of Ach and slows the rate of depolarization of
the cardiac pacemaker, the sino-atrial (SA) node.
• Other inhibitory muscarinic receptors are negatively coupled to adenylate cyclase
via Gi, and decrease cyclic AMP production.
• The excitatory muscarinic receptors found on smooth muscle and glands
stimulate the production of inositol-1,4,5-trisphosphate (IP3) in response to ACh.
They are coupled to phospholipase C over Gq. The IP3 releases calcium from
internal stores in muscle or glandular tissue, causing contraction or secretion.

A summary of selected effects of activation of the parasympathetic nervous system is


given in Table 1.

236
Autonomic Nervous System 2 & 3 - Dr. Biber

Figure 7. ANS RECEPTORS

PHARMACOLOGY OF MUSCARINIC RECEPTORS


EFFECTS OF MUSCARINE (AGONIST) AND ATROPINE (ANTAGONIST)

Targets of parasympathetic activity are illustrated with muscarine poisoning. Muscarine


is not metabolized and hence produces a marked and prolonged stimulation of
muscarinic receptors:

• stimulation of secretory activity of all glands→ Salivation, Sweating, tearing


(Lacrimation), nasal and bronchial secretions;
• contraction of bladder → Urination;

237
Autonomic Nervous System 2 & 3 - Dr. Biber

• activation of smooth muscle possessing muscarinic receptors → increase in GI


motility → Diarrhea and vomiting (Emesis)
• slowing of the heart (bradycardia)
• constriction of bronchi → wheezing;
• pinpoint pupil (miosis) and blurred vision

Muscarinic receptors are blocked by the muscarinic antagonist, atropine, that was
originally obtained from the deadly nightshade, Atropa belladonna.
Atropine reverses the effects of muscarine poisoning.

ACTIONS OF ATROPINE

When given alone, atropine blocks the effects of any ongoing parasympathetic activity.
Readily detected effects include:

• inhibition of glandular secretions → dry mouth, dry eyes, dry nasal passages, dry
skin
• tachycardia due to the loss (or reduction) of ongoing vagal tone
• loss of the pupillary light reflex (pupil is dilated = mydriasis)
• loss of the ability to focus the lens for near vision (cycloplegia)

ADRENERGIC RECEPTORS

ADRENERGIC RECEPTORS RESPOND TO NE AND EPI

These receptors can be divided into two major classes, alpha and beta, that in turn are
further subdivided. All these receptors are G-protein coupled.

ALPHA1 RECEPTORS

• Alpha1 receptors are found on smooth muscle and glands and produce excitation.
• NE and EPI are about equally effective at alpha1 receptors (EPI is actually a bit
more potent than NE).

• Neuronally released NE is the physiological agonist for alpha1 receptors.


Alpha1 receptors require high concentrations of catecholamines to be excited.
Under physiological conditions, only NE released from sympathetic nerve
terminals is present in high enough concentrations to excite alpha1 receptors. Even
under stressful conditions, the EPI released from the adrenal does not reach high
enough levels in the blood to excite significant numbers of alpha1 receptors.

• Signal transduction: The alpha1 receptor activation by NE involves IP3


production, IP3-mediated Ca2+ release from the sarcoplasmic reticulum with
smooth muscle contraction or glandular secretion. Some alpha1, receptors may
also couple to a Ca2+ permeable ligand-gated cation channel.

238
Autonomic Nervous System 2 & 3 - Dr. Biber

ALPHA2 RECEPTORS

• Are inhibitory receptors found on nerve terminals (presynaptic).


• When present on sympathetic postganglionic nerve terminals, they are called
autoreceptors (see FIG 4). Activated when NE is released in large amounts
(under high frequency stimulation), they inhibit further NE release from the same
terminals (feedback inhibition). Autoreceptor activation conserves transmitter
under conditions of high utilization.
• Adrenal medullary chromaffin cells do not possess alpha2 receptors. Thus,
secretion of EPI is not restrained by feedback inhibition. Chromaffin cells can
become depleted of EPI when release is prolonged, for example in response to
profound, long lasting stress.

ALPHA2 HETERORECEPTORS

• Alpha2 receptors present on nerve terminals that do not secrete NE (nonadrenergic


terminals) are called hetero-receptors.
• Alpha2 hetero-receptors are present on the terminals of postganglionic
parasympathetic nerves (very important in the gastrointestinal tract).
• When activated by NE released from sympathetic nerves these alpha2 hetero-
receptors inhibit evoked release of Ach. In this way, the sympathetic exerts an
inhibitory effect indirectly, by reducing the output of excitatory transmitter. This
is the principal way in which the sympathetic inhibits activity of the
gastrointestinal tract.
• Signal transduction: Activation of alpha2 receptors reduces the magnitude of the
inward calcium current that passes into the nerve terminal through voltage
sensitive calcium channels that are opened by invading action potentials. As a
result, the release of transmitter is reduced.

BETA RECEPTORS

• Beta receptors exist as excitatory beta1 and inhibitory beta2 receptors. Beta
receptors are much more sensitive to CA than alpha receptors.
• The beta1 adrenergic receptor produces excitation in the heart.
• EPI and NE are equally effective at beta1 receptors and work at very much lower
concentrations than at alpha receptors.
• The beta2 adrenergic receptor produces relaxation of smooth muscle
(inhibition). It is found on tracheal and bronchial smooth muscle, in the GI tract
and in the smooth muscle of blood vessels supplying the skeletal muscle bed
(where it occurs along with alpha1 receptors).
• The beta2 receptor is preferentially activated by EPI. NE is not at all effective at
activating beta2 receptors.
• Signal transduction: All beta receptors are positively coupled to adenylate
cyclase, via Gs and thus generate cAMP with subsequent activation of protein
kinase A and phosphorylation of one or more proteins. The type of response

239
Autonomic Nervous System 2 & 3 - Dr. Biber

depends on the target protein phosphorylated. This material will be covered in


lectures on smooth muscle and heart muscle later in the course.

PHARMACOLOGY OF ADRENERGIC RECEPTORS

Selective blocking agents (antagonists) are available to block the various classes of
adrenergic receptors. Some of these drugs are useful clinically, e.g. Beta1 blockers are
used as antiarrythmics. Beta2 selective agonists will dilate the bronchi. They are useful in
asthma.

SUMMARY OF SYMPATHO-ADRENAL AND PARASYMPATHETIC


EFFECTS BY TYPE OF TARGET TISSUE
HEART : Sympathetic (NE) or adrenal (EPI) have excitatory effects, increasing
the rate of beating and the force of contraction via activation of
BETA1
receptors.

Parasympathetic has an inhibitory effect, slowing the rate of beating


via action of Ach on MUSCARINIC receptors (M2).

SMOOTH Sympathetic excites smooth muscle via ALPHA1 receptor activation


MUSCLE: (blood vessels, piloerector muscles);
Adrenal EPI activates BETA2 receptors and produces relaxation of
certain smooth muscle (e.g. airways, GI tract & blood vessels
supplying skeletal muscle)

Parasympathetic excites most of the smooth muscle it innervates and


responses are mediated by MUSCARINIC receptors (smooth muscle
of gastrointestinal tract)

GLANDS: Parasympathetic always stimulates glandular secretions, via


MUSCARINIC receptors.

Sympathetic stimulates sweating via MUSCARINIC ACTION of ACh


on generalized sweat glands and via alpha1 effects of NE on sweat
glands of palms of hands.

240
Autonomic Nervous System 2 & 3 - Dr. Biber

Review of Sympatho-Adrenal Actions on Organ Systems


(see TABLE 1)

TABLE 1. AUTONOMIC EFFECTS ON VARIOUS ORGANS OF THE BODY


Receptor Parasympathetic
Organ Sympathetic Stimulation
Type * Stimulation
Eye: Dilates (mydriasis) (radial Narrows (miosis)
Pupil
m. contracts) (sphincter contracts)
Contracts for near
Ciliary muscle Relaxes for far vision
vision
Tear glands:
Secretion

Heart: Increased rate Slowed rate


Muscle
Increased force Decreased force of atrial
of contraction contraction
Lung: Airways Relaxation Contraction
Glands Increased secretion
Systemic blood Abdominal
Constriction None
vessels: viscera
Constriction
Skeletal muscle None
Dilation +
Skin and
Constriction None
mucosa
Gastrointestinal Motility and #
tract: Decreased Increased peristalsis and tone
tone
Sphincters Contracted Relaxed
Secretions Inhibited # Stimulated
Gallbladder
and
Relaxation Contraction
bile ducts:

Urinary
Bladder: Contraction Inhibited Excited

Skin: Piloerector
Contraction None
muscles
palms Secretion None
Sweat
glands generalized Secretion cholinergic
muscarinic None

* Receptor type indicated where definitely known

+ Respond to circulating epinephrine released from adrenal. With the normally low
circulating concentrations that occur in response to stressors, you see vasodilatation

241
Autonomic Nervous System 2 & 3 - Dr. Biber

because receptors are activated preferentially; with high epinephrine concentrations,


seen only under pathological conditions, receptors are also activated and
vasoconstriction predominates.

# Sympathetic inhibition of gastrointestinal function takes place primarily as a result


of inhibition of release of Ach from parasympathetic postganglionic neurons and possibly
other excitatory transmitters from peptidergic motor neurons. This action is mediated by
NE released from sympathetic nerve terminals and acting at presynaptic alpha2
heteroreceptors. Inhibition of Ach release from preganglionic parasympathetic neurons
may also contribute.

The sympathetic innervation to the GI tract is continuously active, with low level basal
discharge (or tone), which reduces the effectiveness of basal excitatory parasympathetic
activity. Basal levels of motility and secretory activity reflect the interplay between the
tonic parasympathetic and sympathetic inputs.

The remaining effects listed below are important components of the stress response:
Alpha and beta effects are involved but are not well characterized in humans.

Sympathetic Parasympathetic
Organ
Stimulation Stimulation
Coagulation Increased None
Blood:
Glucose Increased None
Basal metabolism Increased up to 150% None
Liver Glucose released None
Adrenal cortical
secretion Increased None

Mental activity Increased alertness None


Increased glycogenolysis
Increased strength of
Skeletal Muscle None
contraction

Fat Increased lipolysis None

242
Autonomic Nervous System 2 & 3 - Dr. Biber

MUSCLES OF THE IRIS


DILATOR OR RADIAL SPHINCTER

SYMPATHETIC PARASYMPATHETIC
INNERVATION INNERVATION

Figure 8
Stimulation of sympathetic nerves causes Parasympathetic fibers are tonically active. They
contraction of the radial muscle and widening of regulate pupil diameter in response to light falling
the pupil (MYDRIASIS). Drugs which mimic the on the retina, by contracting the sphincter muscle
effect of NE (sympathomimetics) produce the (pupilary light reflex). Stimulation of the
same parasympathetic nerves reduces the pupil to a pin
effect when instilled into the eye. point (MIOSIS). A similar effect is obtained with
drugs that mimic the action of ACh at muscarinic
This response is seen in severe stress (fight or receptors or prolong the effects of ACh at the
flight response). neuroeffector junction.

Dilation of the pupil results when muscarinic


receptors are blocked by ATROPINE

243
Muscle Energetics and Fatigue - Dr. Feher

Muscle Energetics and Fatigue


Joseph Feher, Ph.D.
LECTURE OUTLINE:
I. Rate of ATP consumption by active muscle depends on the load.
A. The free energy of ATP hydrolysis is the chemical source of mechanical
work.
B. ATP hydrolysis is linked to mechanical work through the actomyosin
ATPAse.
C. The rate of cross-bridge cycling and ATPase activity depend on the load.
D. ATP hydrolysis is also needed for control reactions.

II. Rate and amount of ATP consumption varies with the intensity and duration of
exercise

III. Muscle consumption of ATP is always fast

A. Normal activation of muscle is by trains of impulses


B. In vigorous exercise, frequency of muscle activation increases
C. Each activation of the muscle requires fast ATP hydrolysis
D. Intensity of exercise changes the rest period for metabolism to recover
energy reserves

IV. Metabolism regenerates ATP in different time scales and capacities

A. Direct phosphorylation regenerates ATP fastest.

1. Creatine phosphate provides the first buffer for ATP in muscle, but
it increases Pi.
2. Myokinase can regenerate ATP from ADP.

B. Glycolysis provides a rapid but low capacity supply of ATP for fast twitch
fibers.

244
Muscle Energetics and Fatigue - Dr. Feher

C. Oxidative phosphorylation provides a slower but high capacity supply of


ATP.

V. The fuel used by muscle varies with intensity and duration of exercise

VI. At high intensity, glucose and glycogen is the preferred fuel for muscle

A. Carbohydrates are stored as glycogen and mobilized by glycogenolysis


B. Glycolysis uses muscle glycogen and blood glucose
C. Glycolysis produces pyruvate, ATP and NADH without requiring oxygen
D. Generation of lactic acid regenerates cytoplasmic NAD+

VII. Muscle and muscle fibers can be classified by metabolic properties

A. Muscles can be classified as SO, FOG and FG


B. Whole muscles are mixtures of muscle fiber types
C. Muscle fiber types differ in the isomers of many different proteins
D. Muscle fiber types differ in the relative amount of organelles
E. Muscle fiber types are more continuously distributed than suggested by
the classification schemes

VIII. Mismatch of lactic acid production and oxidation determines the rate of lactic acid
release into the blood

A. Blood lactate levels rise progressively with increases in exercise intensity


B. Mitochondria can metabolize lactic acid
C. Lactic acid forms a carrier system for NADH oxidation
D. Lactic acid is produced by fully oxygenated tissue
E. Three lactic acid shuttles remove lactate from its site of production

1. the intracellular shuttle removes lactate to the mitochondria


2. The cell-cell shuttle removes lactate to adjacent oxidative fibers

245
Muscle Energetics and Fatigue - Dr. Feher

3. The Cori cycle shuttles lactate to the liver for gluconeogenesis

F. The “anaerobic threshold” really has to do with mismatch of lactic acid


production and oxidation

1. The rest periods between contractions become shorter


2. The fast glycolytic fibers are increasingly recruited over oxidative
fibers
3. Sympathetic stimulation increases the rate of glyocogenolysis

IX. Exercise increases glucose transporters in the muscle sarcolemma

A. Glucose uptake by muscle depends on the number and activity of GLUT4


transporters
B. Insulin increases the number of GLUT4 transporters by recruiting latent
transporters
C. Exercise increases the number of active GLUT4 transporters independent
of insulin
D. Mechanism of GLUT4 recruitment is probably through AMPK and
CAMK

X. The site of Fatigue depends on the muscle and intensity of exercise.

A. Fatigue is a reduction in developed force.


B. Metabolic fatigue occurs with repetitive stimulation
C. Sites of fatigue could be in any process from brain to contractile element
D. Fatigue during maximum sustained contractions is not in the brain.
E. Pi and H+ in muscle interferes with force development by actomyosin
ATPase.
F. Fatigue at submaximal work rates depends on glycogen stores.

1. Fatigue accompanies glycogen depletion

246
Muscle Energetics and Fatigue - Dr. Feher

2. Fatigue can be delayed by glycogen supercompensation

XI. The importance of glycogen in exercise is evident from muscle glyogenoses

XII. Training regimens are designed to increase strength of endurance

A. The most rapid training involves training the brain


B. Training for strength induces muscle hypertrophy

1. Muscle strength depends on muscle size


2. Training sends a signal for muscle growth

a. Myostatin inhibits muscle growth


b. Insulin-like growth factor-1 and muscle growth factor also
influence muscle size

C. Endurance training uses repetitive movements

XIII. Our ability to switch muscle fiber types is limited

A. Muscle capabilities are determined by the mixture of muscle fiber types


B. Single muscle fibers can be hybrids
C. It is unlikely that muscle types interconvert in human training regimens

XIV. Practice Questions


OBJECTIVES:
1. Describe how intensity of exercise differs in heavy resistance vs. endurance
exercise
2. Explain why ATP hydrolysis in active muscle is always fast
3. List the sources of ATP generation in order of their rapidity
4. List the sources of ATP generation in order of their capacity
5. Describe how fuel use varies with intensity and duration of exercise

247
Muscle Energetics and Fatigue - Dr. Feher

6. Explain how lactic acid production allows for more rapid glycolysis
7. List the types of muscle fibers based on metabolic properties
8. List the metabolic fates of lactic acid
9. Explain why exercise reduces diabetic’s need for insulin
10. Identify the likely site of fatigue in high-intensity, short-duration exercise
11. Identify the likely site of fatigue in moderate-intensity, long-duration exercise
12. Describe the overall changes that occur during hypertrophy
13. Describe the function of myostatin and the consequence of its lack
14. Explain how muscle hybrid types can arise
15. Describe our limited ability to switch muscle types
Suggested Reading: Berne and Levy, pp. 237-241
I. MUSCULAR ACTIVITY CONSUMES ATP AT RATES DEPENDENT ON
LOAD

A. The free energy of ATP hydrolysis is the chemical source of mechanical


work.

In adenosine triphosphate ( ATP), energy is required to attach each of the


phosphates on the ribose moiety. These bonds are less stable than their
starting materials: The γ bond makes ATP less stable than ADP and Pi ;
the β Pi bond makes ADP less stable than AMP and Pi. When these bonds
are hydrolyzed, chemical energy becomes available for work with the
remainder dissipated as heat.

B. ATP hydrolysis is linked to mechanical work through the myosin ATPase.

The cross-bridge cycle links shortening or force production to the


hydrolysis of ATP through the kinetics of the actin and myosin
interactions. In an intact muscle, macroscopic force results from thousands
of these cross-bridges cycling at furious rates.

248
Muscle Energetics and Fatigue - Dr. Feher

Figure 1. Structures of ATP and ADP and the ATP hydrolysis reaction. Under the conditions of the myoplasm
[ATP] = 5 x 10-3M, [ADP] = 0.04 x 10-3 M, [Pi] = 5 x 10-3 M and pH = 7.0, hydrolysis of ATP liberates
approximately 57 kjoules mol-1

C. The rate of cross-bridge cycling and ATPase activity depend on the load.

Unlike smooth muscle, which can generate force without large amounts of
ATP hydrolysis, skeletal muscle hydrolyzes ATP whether or not it
shortens. Isometric contractions consume ATP because the cross bridges
continue to cycle even when shortening does not occur. Eccentric
contractions also require ATP hydrolysis to resist stretch and thereby
impart rigidity to the joints. But fewer muscle fibers are recruited in
eccentric contractions, because more force results from the eccentric
contractions. Thus, the rate of ATP consumption for whole muscles varies
with the speed and type of contraction.
D. ATP hydrolysis is also needed for control reactions.

Muscles also use ATP to re-accumulate activator Ca2+ into the SR, for
active ion pumping by the Na+-K+-ATPase , and for maintenance chores.
During activity, the actomyosin ATPase is the main cause of ATP
hydrolysis.
II. RATE AND AMOUNT OF ATP CONSUMPTION VARIES WITH THE
INTENSITY AND DURATION OF THE EXERCISE

The rate of ATP utilization by the aggregate muscles of the body depends on the
intensity of the exercise. The total amount of ATP used during a bout of exercise

249
Muscle Energetics and Fatigue - Dr. Feher

is its rate of utilization times the duration of the event. High intensity exercise can
be sustained only for short periods, whereas moderate intensity exercise can be
endured for long times. The relationship between intensity and sustainable effort
is not linear. Table 1 below lists the approximate rates and amounts of ATP
needed for different track events.
Rate of ATP consumption Amount of ATP needed
Event
(mol/min) (mol)
Rest 0.07 ----
100 m sprint 2.6 0.4
800 m run 2.0 3.4
1500 m run 1.7 6
42200 m marathon 1.0 150
Table 1. Rate and amount of ATP needed for different track events.
Adapted from Hultman, E. and Sjoholm, H. Biochemical causes of fatigue, in “Human Muscle Power” (1986)
Human Kinetics Publishers, Inc, Champaign Illinois.

III. MUSCLE CONSUMPTION OF ATP IS ALWAYS FAST WHEN THE


MUSCLE IS ‘ON’

A. Normal activation of muscle is by trains of impulses

The motor neuron carries a single code for activating its muscle fibers:
the temporal pattern of its action potentials. For particular movements,
motor units in groups of muscles are activated in a particular sequence to
coordinate the movement. Figure 2 shows an electromyogram (EMG) of
rat leg muscles obtained at a slow walking speed. The EMG records
electrical activity of the muscle, not force. Each muscle is activated at
appropriate times for a definite length of time, and this activity alternates
with periods of rest.
B. In vigorous exercise, frequency of muscle activation increases

The EMGs in Fig. 2 illustrate muscle activation in slow walking. In


vigorous walking, the frequency of activation increases and the duration of
each activation decreases. The duty cycle, the fraction of time that the
muscle is activated, increases with increased intensity. Many animals
engage different sets of muscle coordination, called gaits, to vary their
speed of locomotion. In humans, walking and running form distinctive
gaits, but running fast and running slowly differ mainly in the speed and
recruitment rather than the sequence of activation. Weight lifting differs
from running in that it activates a selected few muscles. Increasing the
intensity of weight lifting means increasing the weight, and this is
achieved by increasing recruitment of the muscle fibers until 100%

250
Muscle Energetics and Fatigue - Dr. Feher

recruitment is reached at the maximum weight. The maximum weight can


be lifted only slowly, so the muscles must be activated by a train of
impulses that last as long as it takes to lift the weight. This differs from the
activation pattern of repetitive exercise such as walking or running in that
in weight lifting there is no rest phase for the muscle fibers until the
exercise is over. This is the origin of the different kind of fatigue for
heavy resistance exercise versus endurance exercise.

Figure 2. Electromyogram of rat leg muscles at a slow walking speed, 1 mph, on a treadmill. Modified from
G.A. Brooks, T.D. Fahey, T.P. White and K.M. Baldwin, Exercise Physiology, Third Edition, McGraw Hill,
1999.
C. Each activation of the muscle requires fast ATP hydrolysis

Larger EMG amplitudes (recorded in volts) indicate greater number of


muscle fibers that are firing action potentials. Each muscle fiber, when
activated, is activated completely. The control of force for the entire
muscle is achieved by the temporal recruitment of the fibers: which fibers
are being activated and with what frequency and in what sequence. Each
activation of a muscle fiber entails fast rates of ATP hydrolysis because all
of the actomyosin cross-bridges are activated by the Ca2+ transient with
each action potential on the motor neuron.

D. Intensity of exercise changes the rest period for metabolism to recover


energy reserves

Increasing intensity of exercise increases the overall frequency at which


individual muscle fibers are activated. Each activation entails furious rates
of ATP hydrolysis. Increasing the frequency of activation necessarily
decreases the time that the muscle is not activated, and this is the time the
muscle has for metabolism to oxidize substrates produced during the
contractions. Thus, every muscle contraction requires rapid ATP
consumption and regeneration, but at low intensity exercise there is
more time for metabolism to recover the resting state.

251
Muscle Energetics and Fatigue - Dr. Feher

IV. METABOLISM REGENERATES ATP IN DIFFERENT TIME SCALES


AND CAPACITIES (See Figure 3 and Table 2)
A. Direct phosphorylation re-generates ATP fastest.
1. Creatine phosphate provides the first buffer for ATP in muscle,
but it increases Pi.

Muscles contain about 15-20 mM creatine phosphate. Creatine


phosphokinase (CPK) catalyzes the phosphorylation of ADP from
creatine phosphate to form ATP. This extremely rapid reaction
helps to “buffer” ATP concentrations near the normal 5 mM in the
muscle fiber myoplasm. When creatine phosphate is used to
regenerate ATP, myoplasmic [Pi] increases. Creatine
phosphokinase is located nearby ATP-utilizing reactions such as
the myosin ATPase and SR Ca-ATPase and may directly transfer
ATP to these enzymes. This is called substrate channeling. These
ATPase enzymes split ATP faster in the presence of CPK. All
contractions of muscle require creatine phosphate regeneration of
ATP for maximum force.
2. Myokinase can re-generate ATP from ADP.

Another enzyme, myokinase, converts two molecules of ADP into


ATP and AMP. AMP levels may be what is sensed by the “fuel
gauge” of muscle fibers. When it goes up, you are running out of
fuel.

B. Glycolysis provides a rapid but low capacity supply of ATP for fast twitch
fibers.

Glycolysis begins with glucose that may arise from glycogen or from the
blood, and ends with two pyruvate molecules. Glycolysis requires 2ATP
molecules and generates 4ATP, for a net gain of only 2 ATP molecules
per molecule of glucose. Additional ATP (4 or 6 per molecule of glucose)
can be generated from the NADH produced by the oxidation of
glyceraldehyde-3- phosphate during glycolysis.

C. Oxidative phosphorylation provides a slower but high capacity supply of


ATP.

Cytosolic pyruvate formed by glycolysis enters the mitochondria to be


converted to acetyl CoA by pyruvate dehydrogenase. In this process, 1
CO2 is released and reducing equivalents as NADH are produced. The
acetyl CoA is converted to 2 more CO2 molecules through the TCA cycle,
which also produces NADH, FADH2 and GTP. In the presence of oxygen,

252
Muscle Energetics and Fatigue - Dr. Feher

the NADH and FADH2 is oxidized through the Electron Transport Chain
(ETC). The ETC pumps H+ ions out of the mitochondrial matrix,
establishing a [H+] gradient and an electrical potential across the inner
mitochondrial membrane. This electrochemical gradient for H+ is used by
the mitochondrial ATP synthase to synthesize ATP from ADP and Pi. Net
ATP production from the complete oxidation of pyruvate is 30 ATP
molecules per molecule of glucose. Oxygen is needed as the final electron
acceptor from the ETC. Without oxygen, the ETC remains reduced and
everything backs up. The TCA stops for lack of NAD+, and beta oxidation
of fats stops for the same reason. However, lack of oxygen is pathological
rather than physiological. In normal physiology, the issue is how fast
oxidative phosphorylation is going with respect to ATP consumption.

V. THE FUEL USED BY MUSCLE VARIES WITH INTENSITY AND


DURATION OF EXERCISE

Muscles can use fats, carbohydrates and proteins as fuels. Which is used at what
rates depends on the type, intensity and duration of exercise. At rest, muscles use
mainly free fatty acids. At moderate exercise (< 50% maximum O2 consumption
(VO2)), muscle use blood glucose and free fatty acids. At higher intensities of
exercise (>50% VO2) the proportion contributed by glycogen becomes
increasingly important so that at 70-80% VO2 aerobic metabolism of glycogen is
predominant.

253
Muscle Energetics and Fatigue - Dr. Feher

Figure 3. Overall energy metabolism driving contraction in skeletal muscle. ATP is consumed in a variety of
reactions including the actomyosin cross-bridges and the SR Ca-ATPase pump. ATP is provided by a variety of
routes including glycolysis and complete oxidation of carbohydrates through the TCA cycle and electron
transport chain (ETC) in the mitochondria. The source of glucose for glycolysis can be muscle glycogen or
plasma glucose. The glucose is imported into the muscle by a glucose transporter, GluT4. Plasma glucose
originates from liver and extrahepatic tissues either through glycolysis (liver) or gluconeogenesis (liver,
kidneys, intestine). Fatty acids form acetylCoA through beta oxidation and the acetyl CoA is then completely
oxidized, in the presence of adequate oxygen, in the mitochondria. These fatty acids may derive from muscle or
from adipose stores. When glycolytic flux is rapid and myoplasmic NADH accumulates, glycolysis continues
by the regeneration of NAD+ by converting pyruvate to lactic acid by lactate dehydrogenase (LDH). Production
of lactic acid thereby allows glycolysis to continue. Lactic acid produced in this way is transported into the
blood and from there to the liver where it can be converted to glucose again. This cycle of muscle glucose to
lactate to liver lactate to glucose is the Cori cycle.

254
Muscle Energetics and Fatigue - Dr. Feher

Rate of ATP production Amount of ATP available


Source of energy
(mol/min) (mol)
ATP and creatine phosphate 4.4 0.7
Glycogen to lactate 2.4 1.6
Muscle glycogen to CO2 1.0 84
Liver glycogen to CO2 0.4 19
Fatty acids to CO2 0.4 4000
Table 2. Rate and amount of ATP available for contraction from various fuel sources
Table 2 shows the rates of ATP production and amounts of ATP available from various
sources. It is important to remember that every muscle contraction utilizes creatine
phosphate and glycogen to lactate, but at low frequency the oxidation of lactate or blood
glucose during the rest period pays to resynthesize glycogen.
VI. AT HIGH INTENSITY, GLUCOSE AND GLYCOGEN IS THE
PREFERRED FUEL FOR MUSCLE
A. Carbohydrates are stored as glycogen and mobilized by glycogenolysis.

Muscle cells burn glucose, but the amount of free glucose in the blood is
limited and cannot fuel muscle activity alone. Muscles and liver store
carbohydrates as glycogen. Glycogen is mobilized through glycogenolysis
to provide glucose for muscle activity. Glycogenolysis is controlled by
sympathetic nervous activity and circulating epinephrine. It is mediated by
phosphorylase, and is controlled by a Gs protein linked to adenylyl
cyclase, the production of cAMP and the activation of protein kinase A.
The rapid utilization of ATP in normal contractions appears to
require glycogenolysis. Glycogen is re-generated during the resting phase
of the muscle between trains of impulses.
B. Glycolysis uses muscle glycogen and blood glucose.

Glycogen stored in muscle is dedicated to glycolysis because muscle lacks


glucose 6-phosphatase that converts G-6P to glucose. Only glucose can
cross the cell membrane. Ionically charged G-6P cannot. Because it lacks
the enzyme to make free glucose, muscle cannot export significant
glucose. Liver and other tissues produce glucose that can travel to muscle
through the blood. Muscle tissues take up glucose by a transporter, Glut4,
that is sensitive to exercise. The Glut4 transporter is recruited to the cell
membrane by insulin, but exercise also recruits these transporters in the
absence of insulin. This is why diabetics cut back on their insulin when
they exercise.

C. Glycolysis produces pyruvate, ATP and NADH without requiring oxygen.

255
Muscle Energetics and Fatigue - Dr. Feher

Glycolysis produces a net gain of 2 molecules of ATP per molecule of


glucose. It also produces NADH from NAD+ as an obligatory cofactor for
the reaction of glyceraldehyde-3-phosphate to 1,3-diphosphoglycerate. If
the cell runs out of cytoplasmic NAD+, glycolysis will stop. Generating
ATP without requiring oxygen is called anaerobic metabolism.
D. Generation of lactic acid regenerates cytoplasmic NAD+

As noted above, glycolysis requires NAD+ in order to proceed.


Cytoplasmic NADH produced by glycolysis can be oxidized back to
NAD+ by the mitochondria through shuttle systems (the malate shuttle and
the glycerolphosphate shuttle) that transfer reducing equivalents (NADH)
into the mitochondrial matrix. Conversion of NADH to NAD+ requires an
oxidized electron transport chain. During rapid bursts of glycolysis, the
mitochondria cannot keep up with the NADH generated by glycolysis. In
these rapid bursts, both NADH and pyruvate concentrations momentarily
spike. Lactic dehydrogenase converts pyruvic acid to lactic acid,
simultaneously converting NADH to NAD+. This NAD+ can then be used
to allow glycolysis to proceed at the glyceraldehyde 3-phosphate step.
Thus, lactic acid production allows glycolysis to proceed during rapid
glycolytic bursts of ATP production, but the reaction goes faster in
exercise because the concentrations of its substrate, pyruvate, increases.
VII. MUSCLES AND MUSCLE FIBERS CAN BE CLASSIFIED BY THEIR
METABOLIC PROPERTIES
A. Muscles can be classified as slow oxidative (SO), fast glycolytic (FG) and
fast oxidative-glycolytic (FOG).

Muscles can be classified by their mechanical properties (see Skeletal


Muscle Mechanics) and by their myosin staining (see Contractile
Mechanisms). Muscle fibers can also be classified on the basis of their
metabolic capabilities. Peter and coworkers described three types of fibers:
slow oxidative (SO); fast glycolytic (FG) and fast oxidative-glycolytic
(FOG). The three main classification schemes are shown in Table 3.
Muscle Property Used to
Classification Scheme Fiber Types
Classify Types
Burke mechanical properties S, FR, FI, FF
Brooke myosin ATPase staining I, IIA, IIB, IIC
Peter metabolic capacity SO, FOG, FG
Table 3. Muscle Fiber Type Classification Schemes.
B. Whole muscles are mixtures of muscle fiber types.

Whole muscles consist of thousands of muscle fibers and these muscle


fibers are distributed among the various muscle fiber types. Specific

256
Muscle Energetics and Fatigue - Dr. Feher

muscles may be predominately one type or another, and the distribution of


fiber types in a given muscle varies between individuals.
C. Muscle fiber types differ in the isoforms of many different proteins.

Brooke’s classification scheme is based on the expression of different


myosin isoforms in skeletal muscle. Many other proteins also can be
expressed as one of several different isoforms. The SERCA Ca-ATPase
has a fast-twitch (SERCA1a) and slow-twitch isoform (SERCA2a);
calsequestrin within the SR lumen has at least two different isoforms, a
“fast” type and a “cardiac” type. The RyR has different isoforms (RyR1 in
skeletal muscle and RyR2 in cardiac muscle). TnC is expressed in
different forms in fast skeletal (TnC2) and slow twitch skeletal muscle and
cardiac muscle (TnC1). Why muscles have some of these isoforms is not
yet clear. Table 4 compares some of the different proteins expressed in
different muscle types and the relative abundance of selected organelles.
D. Muscle fiber types also differ in the relative amount of organelles.

Oxidative fibers contain a lot of mitochondria compared to glycolytic


fibers. The relative amounts of the SR also vary depending on the speed of
contraction. Slow twitch fibers generally have about one-half as many SR
Ca2+ pumps as fast twitch fibers. The myoplasmic parvalbumin content
correlates well with the speed of the fibers, whereas the myoglobin content
correlates well with its oxidative capacity.
Type I Muscle Type IIa Muscle Type IIb Muscle Cardiac Muscle
Twitch Slow Fast Fast
Fatigue Resistant Resistant Fatigable Resistant
Metabolism Oxidative Oxidative Glycolytic Oxidative
Mitochondria +++ ++++ + ++++
SR volume ++ +++ ++++ +
Glycogen + +++ ++++ ++
Myosin Heavy Chain MHC-I MHC-IIa MHC-IIb, -IIx MHC-α, MHC-β
Myosin Light Chain MLC-1aS, -1bS MLC-1f, -3f MLC-1f, -3f MLC-1v, -1a
SR Ca-ATPase SERCA2a SERCA1a SERCA1a SERCA2a
Phospholamban ++ - - +
Calsequestrin fast and cardiac fast fast cardiac
RyR RyR1 RyR1 RyR1 RyR2
Troponin C TnC1 TnC2 TnC2 TnC1
Myglobin +++ +++ - +++
Paravalbumin - + ++ -
Table 4. Comparison of different muscle types.

257
Muscle Energetics and Fatigue - Dr. Feher

VIII. MISMATCH OF LACTIC ACID PRODUCTION AND LACTIC ACID


OXIDATION DETERMINES THE RATE OF LACTIC ACID RELEASE
A. Blood lactate levels rise progressively with increases in exercise intensity

Progressive increases in exercise intensity causes a progressive rise in


blood lactic acid, as shown in Fig. 4. The increased circulating lactic acid
is caused by release of lactic acid from the working muscles. Originally
lactic acid was thought to be produced during anaerobic metabolism, so
that the increased levels in the blood was thought to represent increased
reliance on anaerobic metabolism. For this reason, the knee in the curve
was called the “anaerobic threshold”. Newer views suggest that the
tissue is not anaerobic, even though lactic acid production is increased.

Figure 4. Blood lactate concentration as a function of relative


work load. Lactate levels in blood increase only gradually until
about 60% of VO2 max is reached, and then lactate concentration
increases markedly with further increases in exercise intensity.
B. Mitochondria can metabolize lactic acid

Mitochondria possess a monocarboxylic acid transporter (MCT1) that


transports lactic acid into the mitochondria. Mitochondria also possess
lactic dehydrogenase, LDH, that converts lactic acid to pyruvate. Lactate
enters the mitochondria over MCT1 (Monocarboxylic acid transporter)
and is converted back to pyruvate by mitochondrial LDH. The pyruvate is
then oxidized by the TCA cycle.
C. Lactic acid forms a carrier system for NADH oxidation

Because lactic acid can enter the mitochondria and be converted back to

258
Muscle Energetics and Fatigue - Dr. Feher

pyruvate, it carries cytoplasmic reducing equivalents, as NADH, into the


mitochondria (Fig. 5). NADH is converted to NAD+ by LDH in the
cytoplasm, lactic acid crosses over to the mitochondria and converts
NAD+ to NADH in the matrix. The lactic acid, converted to pyruvate, is
then consumed by the mitochondria.

Figure 5. Lactic acid carries reducing equivalents into the mitochondria. Cytosolic NAD+ is an
obligatory requirement for glycolysis. Conversion of pyruvate to lactic acid in the cytoplasm
regenerates NAD+ so that glycolysis can continue. The lactic acid enters the mitochondria over the
MCT1 carrier (which also transports pyruvate) and is converted back to pyruvate in the
mitochondria by mitochondrial lactate dehydrogenase (LDH). The NADH generated in the
mitochondria can be oxidized back to NAD+ by the electron transport chain.
D. Lactic acid is produced by fully oxygenated tissue

The classical view is that lactate is produced only under anaerobic


conditions, when muscle PO2 falls below levels that fully energize
mitochondria. This view is now thought to be wrong. The main fact that
doesn’t fit is that lactate is produced by exercising muscles that are
fully oxygenated.

E. Three lactate shuttles remove lactate from the site of production

1. The intracellular shuttle removes lactate to the mitochondria in the


same cell

259
Muscle Energetics and Fatigue - Dr. Feher

Lactate produced in the cytosol moves into the mitochondria of the


cell where the lactate is converted to pyruvate and oxidized by the
TCA cycle coupled to oxidative phosphorylation.

2. The cell-cell shuttle removes lactate to adjacent oxidative cells

The fastest fibers produce lactate at the highest rate. They first become
unable to oxidize all of the lactic acid themselves, and the lactate
enters the blood. Neighboring oxidative fibers, which are generally
smaller than the large glycolytic fibers, take up some of this lactate
and oxidize it. This constitutes the cell-cell shuttle (Fig. 6.)

3. The Cori cycle shuttles lactate to the liver for gluconeogenesis

The liver takes up lactate that is released into the blood by the active
muscles. The liver either metabolizes the lactate for energy or uses it
to make new glucose through gluconeogenesis, and exports the
glucose into the blood. Muscles can then take up this glucose and use
it again for energy. This cycle of blood glucose to muscle lactate to
blood lactate to liver lactate and back to blood glucose is called the
Cori cycle.

F. The “anaerobic threshold” really has to do with mismatch of lactic acid


production and oxidation

The increases in blood lactate with intensity of exercise is caused by the


release of more lactic acid by the exercising muscles than can be
metabolized by the aggregate tissues of the body. Release outstrips
oxidation plus gluconeogenesis. This has the appearance of an increase of
anaerobic metabolism, and in one sense it is. Every muscle contraction
involves a period of “anaerobic” generation of ATP. In less intense
exercise, there is sufficient rest time for the lactic acid produced during
this period to be oxidized. When exercise intensity increases, three
different things happen:

1. The rest period between contractions becomes shorter


2. The fast glycolytic fibers are increasingly recruited over the oxidative
3. Sympathetic nervous system increases the rate of glycogenolysis,
further increasing the supply of pyruvate, and, by mass action, the
production of lactate

260
Muscle Energetics and Fatigue - Dr. Feher

Because of these, lactate release by the active muscles soars with


increased exercise intensity and outstrips the ability of the tissues to
metabolize the lactate. This occurs without gross anaerobic conditions.
The muscle are still fully oxygenated. In this sense there is no
anaerbiosis yet there is increased periods of lactate production by
anaerobic pathways.

Figure 6. The lactate shuttles. Lactate is produced in the cytoplasm by LDH acting on pyruvate and NADH.
Lactate can be shuttled from the cytoplasmic compartment to the mitochondrial compartment by importing the
lactate into the mitochondria and linking it to synthesis of pyruvate and generation of NADH in the
mitochondrial matrix. This is the intracellular lactate shuttle. Secondly, lactate can be exported into the blood
where it is taken up by adjacent oxidative muscle fibers and completely oxidized by its mitochondria. This is
the cell-to-cell lactate shuttle. Third, lactate released into the blood when lactic acid production is high can be
taken up by liver cells (also called hepatocytes). The hepatocytes resynthesize glucose from the lactate and
export it back into the blood where it can be taken up by the exercising muscle, for example. This is the Cori
cycle.

261
Muscle Energetics and Fatigue - Dr. Feher

IX. EXERCISE INCREASES GLUCOSE TRANSPORTERS IN THE MUSCLE


SARCOLEMMA

A. Glucose uptake by muscles depends on the number and activity of GLUT4


transporters

One of the important fuels for muscle is blood glucose, which originates
mainly from the liver, intestine and kidney through glycogenolysis or
gluconeogenesis. Blood glucose enters the muscle fibers through specific
GLUT4 transporters in the muscle fiber membrane. The rate of uptake
depends on the number of these transporters in the membrane and their
activity.

B. Insulin increases the number of GLUT4 transporters by recruiting latent


transporters

One of the most important effects of insulin is to increase the uptake of


glucose by the peripheral tissues, especially muscle. Insulin increases
glucose uptake by recruiting GLUT4 transporters from latent storage in
vesicles in the muscle fiber. This mechanism is shown in Fig. 7.

C. Exercise increases the number of active GLUT4 transporters independent


of insulin

Exercise itself exerts an insulin-like effect and increases glucose uptake


by increasing the number of GLUT4 transporters, but without increases in
insulin. Diabetic persons who inject insulin and then exercise could
experience too much glucose removal and possibly suffer from
hypoglycemia. For this reason, diabetics should reduce their injection of
insulin when they anticipate they will exercise.

D. Mechanism of GLUT4 recruitment is probably through AMPK and


CAMK

The mechanism by which exercise increases GLUT4 transporters is not


yet completely worked out. However, researchers believe than AMPK, a
protein kinase stimulated by AMP, and calmodulin-dependent protein
kinase, CAMK, may be involved. AMPK is stimulated by AMP, which is
produced from ADP by myokinase when ADP concentrations rise during
contractions. The AMPK is thought to be a kind of “fuel gauge” that
senses low fuel levels and then switches off ATP-consuming reactions and
turns on ATP-producing reactions. AMP acts in a negative feed-back
mechanism to restore ATP levels. CAMK, on the other hand, is activated
by Ca2+ when it rises to activate the myofilaments. If it simultaneously

262
Muscle Energetics and Fatigue - Dr. Feher

activates GLUT4 transporters, it acts as a feed-forward mechanism to


begin increasing ATP supply in anticipation of its need to support
contraction.

Figure 7. Insulin, AMPK and CAMK increase GLUT4 incorporation into the sarcolemma of muscle
fibers. All three increase the number of sarcolemma GLUT4 from a population of latent transporters
located in vesicles in the cell. The increased numbers of GLUT4 transporters increase glucose uptake and
generation of ATP through glycolysis and oxidation of pyruvate or lactate.
X. THE SITE OF FATIGUE DEPENDS ON THE MUSCLE AND INTENSITY
OF EXERCISE

A. Fatigue is a reduction in developed force.


Fatigue is a transient loss of work capacity resulting from preceding
work. Reduction in the maximal force from a muscle, compared to its
rested force, is viewed as fatigue. By this criteria, fatigue sets in very
soon after maximum effort. Usually a person can bench press a maximal
weight only once.
B. Metabolic fatigue occurs with repetitive stimulation.

A second kind of fatigue follows exercise at submaximal force for many


repetitions. After some time, we tire and eventually become unable to
develop even this submaximal force.

263
Muscle Energetics and Fatigue - Dr. Feher

C. Sites of fatigue could be in any process in the chain from brain to


contractile element.

In principle, impairment of any one of the chain of events starting with the
central nervous system and ending with the contractile elements of muscle
could reduce developed force. These events include excitatory drive to the
higher motor centers (motivation or effort), balance between excitatory
and inhibitory pathways in spinal motor neurons; conduction of action
potential on the motor neuron to the neuromuscular junction; transmission
across the neuromuscular junction; propagation of the muscle action
potential over the SL and into the T-tubules; excitation-contraction
coupling; generation of force at the actin and myosin filaments.

D. Fatigue during maximum sustained contractions in humans is not in the


brain.

In classic experiments, Merton tested the adequacy of central nervous


system activation of fatiguing muscles by directly stimulating the nerve
leading to a muscle during maximal voluntary contractions. If the
voluntary contraction was maximal, the electrical stimulation would not
increase the force. He found no increase in force upon external stimulation
and concluded that the CNS is not the cause of fatigue. Direct stimulation
of the muscle also did not increase force of fatigued muscle, suggesting
that the neuromuscular junction failure also does not cause fatigue.

E. Pi and H+ in muscle interferes with force development by actomyosin


ATPase.

In fast twitch fibers activated for short bursts, creatine phosphate


regenerates ATP from ADP. The terminal phosphate comes from the
creatine phosphate, so that regeneration of ATP increases Pi in the
myoplasm. At the same time, activation of anaerobic glycolysis produces
ATP with a build up of lactic acid and H+ ions. During exercise the pH of
muscle can fall from pH 7.0 to pH 6.0. Both Pi and H+ interfere with force
production, by directly inhibiting the acto-myosin ATPase or by making
the myofilaments less sensitive to activator Ca2+. The resulting reduction
in force is perceived as fatigue.

F. Fatigue at submaximal work rates depends on glycogen stores.

1. Fatigue accompanies glycogen depletion

The performance times at high but submaximal work loads depends on


the size of the glycogen stores before exercise. Fatigue appears when

264
Muscle Energetics and Fatigue - Dr. Feher

glycogen levels fall but before they are zero. This has led to the
hypothesis of the “glycogen shunt” in which glycogenolysis is
necessary to maintain ATP during contraction, and is resynthesized
during the rest period between contractions. When glycogen becomes
low, it can no longer sustain ATP levels during contraction and force
falls, even though glycogen is not completely used up.

2. Fatigue can be postponed by glycogen supercompensation

Glycogen stores can be increased by a combination of exercise and


carbohydrate consumption. This is referred to as carbohydrate
loading. It is usually accomplished by exhaustive exercise followed
within 2 hours by a high-carbohydrate meal. Under these conditions
the glycogen stores supercompensate and store larger than normal
amounts of glycogen. For serious athletic contests such as marathons
the race preparation is more complicated, taking place over the two
weeks prior to the race.
XI. THE IMPORTANCE OF GLYCOGEN IN EXERCISE IS EVIDENT
FROM MUSCLE GLYCOGENOSIS (Figure 8)

There are 11 well known hereditary disorders of glycogen or carbohydrate


metabolism that affect muscle alone of together with other tissues. These
disorders cause two major clinical syndromes:

A. acute, recurrent, reversible muscle dysfunction manifesting as exercise


intolerance, myalgia upon exercise, and cramps, often culminating in
muscle breakdown and myoglobinuria

B. fixed, often progressive weakness, sometimes simulating dystrophic,


inflammatory or even neurogenic processes
Fig. 8 illustrates the points in metabolism characterized by these glycogenoses.

265
Muscle Energetics and Fatigue - Dr. Feher

Figure 8. Muscle Glycogenoses.

XII. TRAINING REGIMENS ARE DESIGNED TO INCREASE STRENGTH


OR ENDURANCE

A. The most rapid training involves training the brain.

During initial training (the first few weeks) the maximal voluntary

266
Muscle Energetics and Fatigue - Dr. Feher

contraction increases whereas the maximal evoked contraction (produced


by direct and maximal stimulation of the motor nerve) does not. This
suggests that trainees learn to activate their muscles more fully or they
improve coordination of the voluntary contraction. Increases in maximal
evoked contraction require a longer training period.
B. Training for strength induces muscle hypertrophy.

1. Muscle strength depends on muscle size.

The maximum force that can be exerted by a muscle depends on its


cross-sectional area and architecture such as pinnation. Strength
training employs contractions against large resistances with few
repetitions. It is called resistance training.

2. Training sends a signal for muscle fiber growth.

Beginning training is associated with delayed onset muscle


soreness, or DOMS. Part of the soreness could be due to
microscopic tears in muscle fiber membranes or connective tissue
or to the stretching of sensory neurons from edema or mechanical
stretch. The exercise signals muscle hypertrophy: the diameter of
the fibers increases but the number of fibers stays the same.
Although the dogma states that training increases the size of
muscle fibers but not their number, some electron micrographs of
muscles appear to show muscle fibers splitting length wise. Some
limited amount of hyperplasia (increase in cell number) may also
occur. Both type I and type II fibers hypertrophy in response to
resistance training.

Hypertrophy occurs two ways: muscle fibers make more


myofibrils and satellite cells within the muscle are recruited to
fuse with existing muscle fibers to help control the extra
cytoplasm. During development, satellite cells are recruited to
form myotubes that further differentiate to become muscle fibers.

a. Myostatin inhibits muscle growth

Stretch, hypoxia, and intracellular [Ca2+] signal muscle cell


nuclei to produce a host of transcription factors that
increase synthesis of myofibrillar proteins. One product
that inhibits muscle differentiation and growth is
myostatin. Myostatin is an autocrine and paracrine
hormone produced by muscle cells that is a negative
regulator of muscle mass. Watch the newspapers for

267
Muscle Energetics and Fatigue - Dr. Feher

athletic abuse of myostatin inhibitors. People with non-


functioning myostatin mutations have gross muscle
hypertrophy (N Engl J Med 350:2682-2688, 2004).

b. Insulin-like growth factor-1 and muscle growth factor


(MGF) also influence muscle size

Other muscle growth factors include insulin-like growth


factor-1 (IGF-1) and muscle growth factor (MGF). Muscles
have receptors for IGF-1 that activate the cascade leading
from phosphatidyl inositol 3-kinase (PI3K) to activation of
protein kinase B (PCB-ACK) and mammalian target of
rapamycin (mTOR). This is inhibited by cAMP-dependent
protein kinase when AMP levels rise during hypoxia, for
example. Figure 9 illustrates the likely signaling pathways
for muscle hypertrophy.

3. Strength training decreases relative mitochondrial volume.

Heavy resistance training results in a reduction in the


mitochondrial volume density and the ratio of mitochondrial to
myofilament volume. Strength training appears to have no effect
on the muscle capillarity.

C. Endurance training uses repetitive movements.

Endurance training increases the capillarity of muscles and tunes the


muscles’ metabolic capabilities. Concentrations of myoglobin and TCA
cycle enzymes are increased as well as both the size and number of
mitochondria. Muscles of endurance trained subjects use fats as the
primary fuel for moderate exercise, thereby sparing glycogen for bursts of
high intensity activity.

268
Muscle Energetics and Fatigue - Dr. Feher

Figure 9. The signaling events in muscle hypertrophy. Muscle grows in response to stretch, increases in the
integrated cytoplasmic [Ca2+], androgens and glucocorticoids and other cytokines. Calcium activates
calcineurin, a protein phosphatase that dephosphorylates NFAT (nuclear factor of activated T cells) and
activates it. Myostatin produced by muscle inhibits satellite cell division and differentiation. Muscle cells also
respond to IGF-1 (insulin-like growth factor-1) and MGF (muscle growth factor).
XIII. OUR ABILITY TO SWITCH MUSCLE FIBERS TYPES IS LIMITED

A. Muscle capabilities are determined by the mixture of muscle fiber types

Earlier we saw that muscles are heterogeneous mosaics of different muscle


fiber types, and that the overall mechanical and metabolic performance of
the muscles is a consequence of the composition of that mosaic. One basis
of this heterogeneity is the expression of specific myosin isoforms. There
are at least 20 structurally distinct classes of myosin heavy chains. Eleven
of these are expressed in adult mammalian muscles, but some are specific
to one muscle. The most common isoforms are MHCIb, MHCIIa, MHCIIb
and MHC IId.

B. Single muscle fibers can be hybrids

269
Muscle Energetics and Fatigue - Dr. Feher

Immunohistochemistry reveals that some muscle fibers are “pure” types


that express only a single myosin heavy chain (MHC) isoform. However,
some muscle fibers contain two or more isoforms and are “hybrids”. How
can this be? Recall that each muscle fiber contains many nuclei, typically
located at the periphery of the cell near the sarcolemma, as shown in Fig.
10. Each nucleus controls a volume of cytoplasm or surface of the fiber
called its “nuclear domain”. A separate population of nuclei congregate
near the neuromuscular junction. During transitions between fiber types, it
is possible that some of these nuclei receive different signals than others,
and therefore transcribe different genes for the expression of myosin. In
this way, hybrid muscle fiber types arise. The existence of hybrid muscle
fibers allows a more continuous gradation between muscle types, as
shown in Table 5.

Figure 10. Nuclear domains in muscle fibers. Muscle fiber nuclei are located in the
periphery of the fiber, nearly aligned in rows with more or less regular spacing, with
some 35-80 nuclei per mm of fiber. Each nucleus controls protein expression in a volume
or surface element called its domain.

Myosin Heavy Chain


Muscle Fiber Type Muscle Fiber Description
Expression
Type I Pure Fiber MHCI Slow
MHCI>MHCIIa
Hybrid MHCIIa>MHCI
Type IIa Pure Fiber MHCIIa Fast Fatiguable
MHCIIa>MHCIIx
Hybrid MHCIIx>MHCIIa
Type IIb Pure Fiber MHCIIx Fast Fatiguable
Table 5. The muscle fiber type continuum. Hybrid muscle fibers allow transitional forms
intermediate between Types I, IIa, and IIb. Humans do not make MHCIIb as in experimental
animals, so the human form in fast fatiguable muscles is named MHCIIx.
C. It is unlikely that muscle types interconvert in human training regimens.

270
Muscle Energetics and Fatigue - Dr. Feher

Is it possible for humans to convert a type I slow fiber into a type II fast
fiber, or vice-versa? In cross-innervation experiments in animals, a fast-
twitch muscle is removed from its bed and transplanted to a slow-twitch
muscle bed, and a slow-twitch muscle is transplanted to a fast-twitch bed.
The muscles in these cases are converted part way from slow twitch to fast
twitch, and vice-versa. This demonstrates that it is the pattern of neural
stimulation that determines muscle type. Chronic low frequency
stimulation of fast twitch fibers increases the expression of proteins
normally expressed only by slow-twitch fibers. Denervation or muscle
unloading increase the levels of proteins normally expressed by fast-twitch
fibers. The evidence for human transformation of muscle types is
inconclusive. It appears that the stimulation of muscle necessary to
transform the fiber types is so severe that no human can train that hard.
The scientific consensus is that the transformation of muscle types is
limited in part by the original position of the muscle on the muscle fiber
type continuum. The transformation by exercise is always towards a
slower type of muscle, but frank conversion of Type IIB fiber to a Type I
fiber does not occur.

D. The mechanism of muscle type switching appears to involve calcineurin

Calcineurin is a calcium-dependent protein phosphatase that is located in


the Z disks of skeletal muscle. With continued activity, the integrated
[Ca2+] in muscle cells increases, which may signal calcineurin activation.
It cleaves a phosphate off NFAT (nuclear factor of activated T cells) that
migrates to the nucleus and begins switching off transcription of DNA
coding for MHCIIa mRNA, and switches on the gene for MHCI. The
result is a switching of muscle fiber types. (See Fig. 11).

271
Muscle Energetics and Fatigue - Dr. Feher

Figure 11. Mechanism of fiber type switching.

XIV. PRACTICE QUESTIONS

1. The main source of ATPase activity in exercising muscle is

A. The SR Ca-ATPase
B. The Na-K-ATPase
C. Acto-myosin ATPase
D. Myokinase
E. Creatine kinase

2. Because of lack of glucose 6 phosphatase

A. Muscles can import glucose from the blood


B. Liver cells can contribute to blood glucose homeostasis

272
Muscle Energetics and Fatigue - Dr. Feher

C. The Cori cycle can occur


D. Muscles cannot contribute substantially to blood glucose
homeostasis
E. Muscles cannot participate in glycolysis

3. Fatigue in high-intensity, brief exercise is thought to be caused by

A. A build up of Pi and H+ in the myoplasm


B. Lactic acid
C. Failure of T-tubule transmission
D. Depletion of glycogen
E. Failure of neuromuscular transmission

4. When people with insulin-dependent diabetes exercise, they reduce their


insulin shots because

A. Exercise increases glucose uptake by muscle


B. Exercise increases insulin secretion
C. Exercise increases glycogenolysis
D. Exercise increases lactic acid production
E. They do not do this, tricky question person

5. Lactic acid levels in blood

A. Increases dramatically at 60-70% VO2 max because muscles


become anaerobic
B. Increases only when blood supply to muscle is compromised
C. Is normally close to zero
D. Explains most of short-term, high intensity fatigue
E. Increases at 60-70% VO2 max because glycolysis increases
dramatically and lactic oxidation cannot keep up

273
Muscle Energetics and Fatigue - Dr. Feher

6. Persons with low levels of myostatin should

A. Have predominately Type I muscle fibers


B. Show exercise intolerance
C. Have predominately Type II muscle fibers
D. Have larger muscles than normal
E. Have smaller muscle than normal
Answers: 1C; 2D; 3A; 4A; 5E; 6D

274
Smooth Muscle - Dr. Karnam

Smooth Muscle I and II


S.M. Karnam, Ph.D.

Suggested Reading: Berne and Levy (5th Edition, 2003), pp 246-262


Linda S. Costanzo (3nd Edition, 2006), pp 38-43

Learning Objectives: After studying this material, the student should:

a) Analyze the contractile apparatus of smooth muscle and differences with striated
muscle
b) Identify the mechanism of smooth muscle contraction: the role of Ca2+ and
myosin light chain phosphorylation in mediating smooth muscle contraction
c) Differentiate the mechanism of Ca2+-independent contraction (Ca2+ sensitization)
d) Describe the mechanism of muscle relaxation

I. Structure of smooth muscle

Smooth muscle is widespread: it is a major component of the walls of the hollow


organs including airways, gut, urogenital tract and vasculature. Similarities and
dissimilarities with striated muscle are obvious.

The ultimate function of smooth muscle is to develop force or contract, to provide


motility or to alter the dimensions of an organ. For some of the functions contractions
must be phasic (contraction followed by relaxation) to allow the lumen to refill between
contractions. For others, contractions must be tonic (sustained contraction) to resist
continual distending forces.

In most organs, the orientation of smooth muscle is circular or circumferential so


that contraction decreases the intraluminal volume/diameter and thereby increasing
resistance to flow. In the gut, there is a second layer of muscle cells oriented in the
longitudinal axis (oral-to-anal) termed the longitudinal muscle layer. Network of
neurons, glial cells and interstitial cells of Cajal separate the circular and longitudinal
muscle layers.

Smooth muscle cells

Some structural properties are common to all smooth muscle cells: they are single
(uninuclear) cells; they have no transverse striations; they have no T-tubules; they have
abundant caveolae; and they have numerous cell-to-cell junctions. Other aspects of the
muscle structure vary from organ to organ and are related to the characteristic functions
of each muscle.

275
Smooth Muscle - Dr. Karnam

Smooth muscle cells are spindle-shaped. The characteristic shape seems dictated by
the insertion of the contractile apparatus over the entire cell surface and is essential for
mechanical properties of these cells. Most individual smooth muscle cells are very small
in comparison to skeletal muscle cells.

Figure 1. Organization of the contractile and cytoskeletal apparatus in smooth


muscle cells. Thin actin filaments emerge from the poles of the cytoplasmic dense bodies
and interdigitate with thick myosin filaments. Dense bands in the plasma membrane are
connected to dense bodies in the cytoplasm by intermediate filaments. When juxtaposed
dense bands from adjacent cells can form close intermediate junction (provides
mechanical coupling). Gap junction provides electrical coupling.

a) Caveolae are flask-shaped invaginations of the cell membrane. Caveolae


increase the surface area of the cell. More than one-third of the plasma membrane at the
cell surface is in the form of caveolae, while the rest constitutes the cell surface proper.
The exact function of caveolae, however, is not known.

276
Smooth Muscle - Dr. Karnam

b) Dense bands are structures associated with the cell membrane where
contractile apparatus and cytoskeleton are anchored to the cell surface. Dense bands are
sometimes coupled to each other in adjacent cells. Dense bodies are scattered throughout
the cytoplasm. The linkage with actin filaments is very evident both in dense bands and
dense bodies.

c) Cell-to-cell junctions are very abundant in smooth muscle, and they serve at
least two fundamental functions: mechanical coupling (transmission of force between
cells) and ionic coupling (transmission of excitation from cell to cell). The mechanical
coupling is supported by intermediate junctions. Ionic or electrical coupling is provided
by gap junctions.

d) Sarcoplasmic reticulum is well developed in smooth muscle. The smooth


sarcoplasmic reticulum can sequester calcium ions from the cytoplasm and is regarded as
a major storage site for calcium. The sarcoplasmic reticulum has inositol trisphosphate
(IP3) receptors and ryanodine receptors, which, when activated by appropriate second
messengers, allow the release of intracellular calcium needed for contraction. The
sarcoplasmic reticulum also contains a calcium ATPase or calcium pump that functions
to sequester calcium into sarcoplasmic reticulum following contraction.

e) Myofilaments: The three classes of filaments found in smooth muscle are


actin (thin, 6-8 nm) filaments, myosin (thick, 14 nm) filaments and intermediate (10 nm)
filaments.

Thin filaments contain actin polymer and tropomyosin as in striated muscle.


Compared to striated muscle, there is very large number of actin filaments in smooth
muscle. Smooth muscle lacks the regulatory protein, troponin, but contains two
unique proteins, calponin and caldesmon.

Thick filaments are large aggregates of myosin molecules and composed of two
heavy chain subunits (~ 200 kDa each) and two each of two types of light chains (two
20-kDa regulatory light chains and two 17-kDa essential light chains). The myosin
heavy chain dimers form globular head and coiled tail regions. The head region
contains distinct sites for actin binding, ATP hydrolysis and association of light chain
subunits.

Intermediate filaments-a cytoskeletal component present in most cell types-are


abundant in smooth muscle, and are far more abundant than in cardiac and skeletal
muscle. They are intermediate in size between and thick and thin filaments. The
main component of intermediate filaments in visceral muscles is desmin and in
vascular muscles is vimentin. Intermediate filaments are anchored in the dense
bodies and bands, and link to the cytoskeleton. These provide mechanical coupling
between force generating proteins (actin and myosin) and the cell membrane thereby
causing deformation and shortening of smooth muscle cells.

277
Smooth Muscle - Dr. Karnam

Figure 2. Contractile proteins in smooth muscle. Contractile proteins in


smooth muscle are organized into thick and thin filaments as schematically illustrated.
Thin filaments are composed of tropomyosin, caldesmon, and/or calponin bound to two
intertwined strands of polymerized actin. Myosin, composed of two heavy chains and two
pairs of light chains – essential light chains (ELC: 17-kDa) and regulatory light chains
(MLC: 20-kDa)- polymerizes into thick filaments.
II. Innervation of smooth muscle

The role of nerves in smooth muscle is somewhat special, because many muscles
have myogenic activity and can contract independently of nervous influence. Muscle

278
Smooth Muscle - Dr. Karnam

cells are often electrically coupled, so excitation can spread quickly from innervated cell
to cells that are not directly innervated.

There is wide diversity in the type and degree of innervation of smooth muscle
tissue in the body. For example, both parasympathetic and sympathetic branches of the
autonomic nervous system, as well as post-ganglionic nerves from the enteric nervous
system innervate smooth muscle of the gastrointestinal tract. On the other hand, many
blood vessels receive only sympathetic innervation. Neurotransmitters can be classified
in terms of their ability to cause contraction (excitatory transmitters) and relaxation
(inhibitory transmitters) of smooth muscle cells.

There are two categories of smooth muscle based on how the muscle cells are
stimulated to contract.

A. Single-unit (unitary) smooth muscle is found arranged in sheets and all the
cells in a sheet are able to contract as a single unit. Muscle of the stomach and
intestine behave like unitary type smooth muscle. Unitary smooth muscle is self-
excitable (exhibit spontaneous electrical activity) or myogenic and does not
require nervous stimulation for contraction. This type of smooth muscle is
sparsely innervated and muscle cells are electrically coupled by gap junctions so
that excitation can rapidly spread among cells resulting in unitary contraction.
Autonomic nervous system can modify the rate and strength of contractions.
Other factors that influence contractions are hormones and mechanical stretch.

Figure 3. Schematic of unitary (A) and multiunit (B) smooth muscle. Note the
difference between the two types of muscle cells in innervation and contact.

B. Multiunit smooth muscle is found in the walls of large blood vessels, in the
large airways of respiratory tract and in the eye muscles. Like striated muscle,
multiunit smooth muscle is neurogenic, that is, it requires nerve stimulus to
initiate contraction. Unlike skeletal muscle there are no neuromuscular junctions.

279
Smooth Muscle - Dr. Karnam

Neurotransmitters are released into extracellular fluid surrounding the smooth


muscle cells. Certain hormones and drugs can also influence contractions of
multiunit smooth muscle.

III. Electrical properties of smooth muscle

A. Resting Membrane Potential

The resting membrane potential, defined as the steady-state potential at which the
net flow of current (i.e., ions) across the plasma membrane is zero, varies from about -40
to -80 mV in muscle cells of the gut. Differences in resting membrane potential exist
between muscle cells in different regions of gastrointestinal tract, such as the fundus,
corpus, and antrum of the stomach

B. Gated Ion-Selective Channels

In addition to passive ion-selective channels, the plasma membrane contains ion-


selective channels that can be regulated by membrane potential (i.e., voltage-gated
channels) and by various humoral, hormonal, or neural agents (i.e., ligand-gated
channels). Ligands can activate channels directly and through G proteins in the
membrane. Ligands can also activate, inhibit, or modulate voltage-gated channels
through second messengers.

Voltage-gated Ca2+ channels carry the long-lasting inward Ca2+ current (L-type
2+
Ca channels) and are activated rapidly by depolarization of the plasma membrane but
are inactivated more slowly. Inactivation occurs as a result of Ca2+ influx and membrane
depolarization. Most of the voltage-gated Ca2+ channels are sensitive to the drug
dihydropyridine and are thus, known as dihydropyridine-sensitive Ca2+ channels.
Dihydropyridines are well known in pharmacology as L-type Ca2+ channel blockers.

Several types Voltage-Gated Potassium Ion Channels have been identified in


gastric and intestinal smooth muscle.83–95 The channels differ in their conductance, ranges
of voltage activation, and Ca2+ sensitivities. The most widely distributed is a high-
conductance, Ca2+-activated, voltage-sensitive K+ channel. During resting conditions,
when cytosolic Ca2+ concentrations are low (<10-7 M), relatively few channels are open.
On stimulation, the increase in calcium induces activation of large numbers of K+
channels, which carry an outward current, that drives the membrane potential to its
resting state. Ca2+ channels and Ca2+-activated K+ channels constitute the electrical
apparatus that sustains rhythmicity in smooth muscle. Activation of Ca2+ channels
induces an inward flow of Ca2+ ions that depolarizes the membrane and increases
cytosolic calcium. Depolarization and an increase in cytosolic calcium inactivate the
Ca2+ channels and activate the K+ channels, by inducing an outward flow of K+ ions.
Suppression of the inward flow of Ca2+ ions and enhancement of the outward flow of K+
ions restore the resting membrane potential.

280
Smooth Muscle - Dr. Karnam

C. Interstitial Cells of Cajal (ICC): Electrical pacemakers and mediators of


neurotransmission in gastrointestinal tract

In gastrointestinal muscle phasic contractions are caused by electrical activity


termed slow waves, which arise from defined pacemaker regions. Pacemaker areas
contain a variety of cell types, including enteric neurons, glial cells, smooth muscle cells,
immune cells and interstial cells of Cajal (ICC).

ICC are a distinctive population of enteric cells that express c-kit, the proto-
oncogene that encodes the receptor tyrosine kinase, Kit. The closeness of nerve fibers to
ICC, and ICC to smooth muscle cells led Cajal to propose that ICC were functionally
interposed between nerve terminals and smooth muscle cells. ICC and smooth muscle
cells are electrically coupled. The ICC are spontaneously active, generating and
propagating slow-wave depolarizations, and thus regulating phasic contractions in the
gastrointestinal tract.

Figure 4. Profile of a typical slow wave. A rapid upstroke is followed by partial


repolarization, a plateau potential of variable duration and complete repolarization.

A typical slow wave consists of the following sequence: rapid depolarization (i.e.,
upstroke), partial repolarization, a sustained plateau lasting several seconds, and complete
repolarization to the resting membrane potential. Slow waves originate in pacemaker
regions and propagate rapidly throughout its thickness. Propagation is rapid and is
facilitated by the network of ICC and the abundance of gap junctions between muscle
cells.

IV. Excitation-contraction coupling

One of the significant characteristics distinguishing smooth muscle from skeletal


muscle is the existence of multiple pathways for the activation and subsequent regulation
of contractile process. There are many differences between striated and smooth muscle

281
Smooth Muscle - Dr. Karnam

contraction. These include: a) mechanism of excitation-contraction coupling, b)


differences in the calcium regulation of actin-activated myosin-ATPase activity, 3)
differences in calcium sensitivity.

A. Crossbridge cycling

Contraction is initiated by an increase in intracellular free calcium (either


released from intracellular stores or by influx from extracellular stores) and ATP is
utilized to convert chemical energy into mechanical energy. The force and shortening of
the muscle are dependent on the interaction of thick and thin filaments, the formation and
cycling of crossbridges, and the sliding filament mechanism of contraction.

In its simplest form, the sliding filament mechanism requires:

1. Two types of filaments, one containing myosin and the other containing actin.
2. Overlap of myosin-containing thick filaments with actin-containing thin filaments as a
function of degree of shortening.
3. Neither type of filament is changing its length, regardless of the contractile state of the
muscle.
4. The force necessary for filaments to move relative to each other is provided by
crossbridges projecting from the thick filaments. The crossbridges have following
properties:

a) crossbridges go through cycles of attachment, force-production/movement and


detachment.
b) each crossbridge produces same amount of force
c) force production is proportional to the number of active crossbridges

B. Regulation of contraction by myosin light chain phosphorylation.

Myosin from smooth muscle differs from striated muscle myosin in that smooth
muscle myosin-ATPase activity cannot be actin-activated unless their 20-kDa myosin
light chains (MLC) are phosphorylated.

The discoveries that phosphorylation of smooth muscle myosin results in marked


stimulation of actin-activated myosin-ATPase activity, and that myosin light chain
(MLC) kinase, the enzyme responsible for myosin phosphorylation, requires calcium and
calmodulin for activity led to the development of a model for regulation of smooth
muscle contraction whereby calcium-dependent phosphorylation of myosin light chain
initiates smooth muscle contraction.
In this scheme activators of smooth muscle contraction lead to increase in
cytosolic calcium and the formation of calcium-calmodulin complexes that bind to and
activate the enzyme MLC kinase. Activated MLC kinase catalyzes the phosphorylation
of MLC, which results in dramatic increases in actin-activated myosin-ATPase activity of
the smooth muscle myosin and thereby initiates crossbridge cycling and mechanical

282
Smooth Muscle - Dr. Karnam

output. Decreases in cytosolic calcium brought by calcium extrusion or uptake into the
sarcoplasmic reticulum, result in inactivation of MLC kinase, MLC dephosphorylation by
myosin light chain phosphatase, and muscle relaxation. Thus smooth muscle contraction
is regulated by the relative activities of MLC kinase and MLC phosphatase

A considerable body of evidence supports a central role for MLC


phosphorylation-dephosphorylation in the regulation of contraction. This evidence is
briefly summarized as follows:

1. Correlation between MLC phosphorylation and actin-activated myosin-


ATPase activity.

2. Correlation between MLC phosphorylation and contraction.

3. Correlation between MLC dephosphorylation and relaxation

4. Inhibition of MLC phosphorylation and contraction by calmodulin antagonists


and MLCK inhibitors

5. Contraction induced by phosphatase inhibitors.

MLC MLC

MLCK MLCPase MLCK MLCPase


MLC-p MLC-p

Contraction Relaxation

Figure 5. The ratio of activities of myosin light chain kinase (MLCK) and
myosin light chain phosphatase (MLCPase) effects contraction and relaxation in
smooth muscle. Left: Increased cytosolic calcium activates MLCK resulting in increased
MLC phosphorylation (MLC-p) and contraction. Inhibition of MLC phosphatase also
increases MLC phosphorylation and contraction. Right: Decreased cytosolic calcium
inhibits MLC kinase, thereby decreasing MLC phosphorylation and leading muscle
relaxation. Stimulation of MLC phosphatase also decreases MLC phosphorylation
resulting muscle relaxation.
MLCK: myosin light chain kinase, MLCPase: myosin light chain phosphatase,
MLC-p: phosphorylated myosin light chain.

283
Smooth Muscle - Dr. Karnam

C. Source of Calcium

An essential step in smooth muscle contraction is phosphorylation of the 20-kDa


regulatory myosin light chain (MLC20) by a Ca2+/calmodulin-dependent MLC kinase. An
increase in cytosolic calcium is a prerequisite for stimulation of MLC kinase activity.
Two mechanisms lead to an increase in [Ca2+]i. In the first, interaction of a contractile
agonist with its receptor on the plasma membrane generates a messenger that causes the
release of Ca2+ from intracellular stores. In the second, interaction of the contractile
agonist with its receptor generates a messenger that induces depolarization of the plasma
membrane, which opens voltage-gated Ca2+ channels and causes Ca2+ influx, followed by
Ca2+-induced Ca2+ release from intracellular stores.

Smooth muscle cells, like other cells, possess efficient mechanisms to dispose of
the increase in calcium that occur during contraction. In the resting state, the cells
maintain low concentrations of Ca2+ in the cytosol despite large chemical (e.g., 2 mM
Ca2+ outside versus 100 nM Ca2+ inside the cell) and electrical (e.g., membrane potential
of -40 to -80 mV) gradients favoring the movement of Ca2+ into the cell. The disposal
mechanism in the plasma membrane include a Ca2+/Mg2+-ATPase, which acts as a high-
affinity Ca2+ pump sustained by ATP hydrolysis, and a low-affinity, high-capacity Na+-
Ca2+ exchanger. In the sarcoplasmaic reticulum, a high-affinity sarco-endoplasmic
reticulum Ca2+-ATPase pump (SERCA) participates in dissipating the increase in
cytosolic calcium.

D. Calcium-dependent contraction

The interaction between actin and myosin provides molecular basis for muscle
contraction. This interaction is regulated by calcium in all muscle types, but mechanisms
of regulation are fundamentally different for smooth muscle compared with skeletal and
cardiac muscle. Smooth muscle differs from skeletal muscle in having two mechanisms
for initiation of processes leading to contraction. Increases in cytosolic calcium may
occur by influx of calcium through calcium channels in the plasma membrane or by
release of calcium from sarcoplasmic reticulum.

Electro-mechanical coupling: Depolarization of the membrane electrical


potential leads to opening of voltage-gated calcium channels followed by elevation in
cytosolic calcium, which in turn activates the contractile proteins to initiate contraction.
This is called electro-mechanical coupling.

The resting membrane potential of smooth muscle is, like that of other cells,
negative (-40 to –70 mv, depending on the cell type). More positive potentials
(depolarization) can open voltage-gated calcium channels, causing calcium influx to
increase cytosolic calcium and trigger contraction.

The plasma membrane of smooth muscle contains a great variety of ion channels.
The distribution and properties of these channels vary among different (for example,

284
Smooth Muscle - Dr. Karnam

intestine, large and small vessels, uterus) tissues, contributing to the diversity of smooth
muscle. Voltage-dependent calcium channels and calcium-activated K+ channels are the
main channels involved in the regulation of cytosolic calcium. The activity of these
channels can be modulated by excitatory and inhibitory neurotransmitters.

The depolarization-induced rise in cytosolic calcium is due, in part, to the influx


of extracellular calcium through voltage-gated calcium channels, but this calcium also
releases additional calcium from sarcoplasmic reticulum, a process known as calcium-
induced calcium release.

Depolarization

VO channel

Ca2+ Influx

[Ca2+]i
+ MLC
RyR
SR MLCK MLCPase
MLC-p
[Ca2+]i
+

Contraction
Ca2+/CaM

Figure 6. The electro-mechanical pathway of excitation-contraction coupling


in smooth muscle. Depolarization of the membrane opens voltage-gated calcium
channels (VO channel) to induce calcium influx. The increase in cytosolic calcium
([Ca2+]i) binds calmodulin and activates myosin light chain kinase and increases myosin
light chain phosphorylation leading muscle contraction. The initial increase in [Ca2+]i
also releases additional Ca2+ via ryanodine receptors (RyR) (calcium-induced calcium
release).
VO channel: voltage-gated calcium channel, MLCK: myosin light chain kinase,
MLCPase: myosin light chain phosphatase, MLC-p: phosphorylated myosin light chain,
RyR: ryanodine receptors, SR: sarcoplasmic reticulum, Ca2+/CaM: calcium-calmodulin
complex, + denotes stimulation of activity.

285
Smooth Muscle - Dr. Karnam

Pharmaco-mechanical coupling: Smooth muscles have an additional


mechanism in which binding of contractile agonist to its receptor on the muscle
membrane leads to elevation in cytosolic calcium without any change in the membrane
electrical potential. This is called pharmaco-mechanical coupling.

Activation of receptors coupled to contractile agonists result in G protein-


dependent stimulation of phospholipase C-β isozymes, which hydrolyze membrane
phosphoinositides, mainly phosphatidylinositol 4,5-bisphosphate to generate two
messengers: 1-2-diacylglycerol, an activator of protein kinase C and inositol 1,4,5-
trisphosphate trisphosphate (IP3), the main calcium mobilizing messenger in smooth
muscle. Inositol 1,4,5-trisphosphate diffuses through the sarcoplasm and binds to the IP3
receptor located in the sarcoplasmic reticulum membrane. This receptor is also a Ca2+
release channel, allowing Ca2+ to diffuse down its concentration gradient from
sarcoplasmic reticulum lumen into sarcoplasm, thereby triggering contraction.

Agonist

Gαq

PLC-β1

IP3 DAG
MLC

IP3R
SR MLCK MLCPase
MLC-p
[Ca2+]i
+

Contraction
Ca2+/CaM

Figure 7. The pharmaco-mechanical pathway of excitation-contraction


coupling in smooth muscle. Binding of contractile agonist to its receptors on the cell
membrane activates a heterotrimeric G protein (Gαq), which in turn stimulates
phosphoinositide-specific phospholipase-C β1 (PLC-β1). PLC-β1 hydrolyzes
phophotidylinositol bisphosphate to generate inositol 1,4,5-trisphosphate (IP3) and
diacylglycerol (DAG). IP3 binds to its receptors (IP3R) on the sarcoplasmic reticulum
(SR) membrane to release calcium. The increase in cytosolic calcium ([Ca2+]i) binds

286
Smooth Muscle - Dr. Karnam

calmodulin and activates myosin light chain kinase and increases myosin light chain
phosphorylation leading muscle contraction.
MLCK: myosin light chain kinase, MLCPase: myosin light chain phosphatase,
MLC-p: phosphorylated myosin light chain, PLC-β1:phospholipase C-β1, IP3R: inositol
1,4,5-trisphosphate receptor, SR: sarcoplasmic reticulum, Ca2+/CaM: calcium-
calmodulin complex, + denotes stimulation of activity.

In addition to inositol 1,4,5-trisphosphate receptors, the sarcoplasmic reticulum


also contains ryanodine receptors. It has been shown that the ryanodine receptors
respond, as in cardiac muscle, to elevations in cytosolic calcium by calcium-induced
calcium release. The sarcoplasmic reticulum is the physiological intracellular source of
activator calcium and inositol 1,4,5-trisphosphate is the physiological calcium mobilizing
second messenger.

The sarcoplasmic reticulum in smooth muscle also contains phospholamban, a


phosphoprotein that regulates calcium uptake by the sarcoplasmic reticulum. The
calcium-storage capacity of the sarcoplasmic reticulum is enhanced by the intraluminal
calcium-binding proteins calsequestrin and calreticulin.

E. Calcium-independent contraction (calcium sensitization)

There is little doubt that calcium-dependent myosin light chain phosphorylation


represents a major pathway by which muscle tone is regulated. The question is whether
this is the only pathway involved. One of the recent developments in smooth muscle
contraction has been the identification of secondary mechanisms of regulation that can
modify, independently of cytosolic calcium, the levels MLC phosphorylation. The
mechanism(s) through which contractile agonists can increase force (i.e. MLC
phosphorylation) without necessarily increasing calcium is referred to as calcium-
independent contraction or calcium sensitization.

Increased MLC phosphorylation of smooth muscle can be affected not only by


increasing cytosolic calcium and, thereby, the activity of MLC kinase, but also by
inhibiting MLC phosphatase. MLC phosphatase is trimer containing a regulatory subunit,
a catalytic subunit and a subunit with unknown function. The inhibitory signal for
calcium sensitization is communicated by a small monomeric G protein RhoA to Rho-
kinase that phosphorylates MLC phosphatase regulatory subunit and inhibits the catalytic
activity of MLC phosphatase, resulting in increased MLC phosphorylation and
contraction. The RhoA/Rho kinase pathway plays an important physiological role in
smooth muscle contraction. Protein kinase C (PKC) activated by diacylglycerol can also
inhibit MLC phosphatase by phosphorylating and thereby activating CPI-17, an
endogenous inhibitor of MLC phosphatase.

287
Smooth Muscle - Dr. Karnam

Agonist

RhoA

Rho Kinase PKC


MLC

MLCK MLCPase CPI-17


MLC-p

Contraction

Figure 8. Calcium-independent (calcium-sensitization) contraction in smooth


muscle. The major calcium-independent pathway that increases myosin light chain
(MLC) phosphorylation and induces contraction is through activation of Rho-kinase by
RhoA. Activated Rho-kinase inhibits myosin light chain phosphatase (MLCPase) activity
resulting in increased levels of MLC phosphorylation leading to muscle contraction. The
other pathway that enhances MLC phosphorylation is through protein kinase C (PKC)-
mediated phosphorylation of CPI-17, an inhibitor of MLC phosphatase.
MLCK: myosin light chain kinase, MLCPase: myosin light chain phosphatase,
MLC-p: phosphorylated myosin light chain, - denotes inhibition of activity.

E. Regulation of contraction by thin-filament-associated proteins

Contractile regulation by thin-filament-associated proteins (for example,


caldesmon and calponin), and possibly through their phosphorylation, has been
intensively studied.

Caldesmon and calponin regulate smooth muscle contractility through their ability
to inhibit actin-activated myosin-ATPase activity and to inhibit contraction.
Phosphorylation of caldesmon and calponin reverses the inhibitory effect on myosin-
ATPase thus facilitating contraction. Biochemical studies of caldesmon and calponin
phosphorylation in intact muscle suggest important regulatory functions for these
proteins, but the details of signal transduction pathways and the molecular effects of
caldesmon and calponin on the crossbridge cycle are still unclear.

288
Smooth Muscle - Dr. Karnam

V. Molecular Mechanism of Smooth Muscle Relaxation

Smooth muscle relaxation has classically been described as a default process


resulting from the reduction in cytosolic calcium. Cytosolic calcium can be reduced,
causing relaxation, through several mechanisms. The rate and extent of calcium pumping
into the sarcoplasmic reticulum is sufficient to cause relaxation. Relaxation can also be
achieved by the active extrusion of calcium (via sarcolemmal Ca2+ ATPase) and by a
reduction in the sensitivity of the contractile system to calcium. Alternatively, inhibition
of Ca2+ influx through hyperpolarization can also reduce cytosolic calcium and cause
relaxation.

The intracellular second messengers involved in smooth muscle relaxation are the
cyclic nucleotides, cAMP and cGMP, generated by the activity of adenylyl cyclase and
guanylyl cyclase, respectively. They exert their intracellular effects by activating cAMP-
and cGMP-dependent protein kinases. cAMP is the dominant mediator of smooth muscle
relaxation stimulated by β-adrenergic drugs, whereas cGMP-mediated relaxation is
triggered by nitric oxide.

The nitric oxide-cGMP pathway is responsible, at least in part, for the vascular
smooth muscle relaxation produced by many agents, including nitrovasodilators and
acetylcholine. These agents stimulate the endothelial cell to produce nitric oxide and
nitric oxide-dependent cGMP.

It is generally accepted that cAMP and cGMP triggers relaxation of smooth


muscle by activating an intracellular molecular cascade, which revolves around the
activity of cAMP-dependent protein kinase (PKA) or cGMP-dependent protein kinase
(PKG). This cascade results in a reduction of cytosolic Ca2+. Two mechanisms for
smooth muscle relaxation exist: the first is the reduction of intracellular calcium, and the
second is the reduction of the sensitivity of the contractile system to the calcium.

A. Reduction of intracellular calcium concentration.

Cyclic nucleotides trigger a reduction in cytosolic calcium through the activation


of cAMP-dependent protein kinase and/or cGMP-dependent protein kinase. Activated
kinase phosphorylates several key target proteins, including ion channels, ion pumps,
receptors, and enzymes, all involved in the control of intracellular Ca2+ concentration.
Phosphorylation of these target proteins reduces intracellular Ca2+ and results in
relaxation of smooth muscle.

Activation of calcium-activated K+ channels: Calcium-activated K+ channels


are a family of channel proteins expressed in multiple cell types, including
smooth muscle cells. The efflux of K+ through these channels induces
hyperpolarization leading to decrease in calcium influx via voltage-dependent
calcium channels. Phosphorylation by PKA or PKG increases the channel
activity resulting in hyperpolarization, which in turn decreases calcium influx via
voltage-gated calcium channels.

289
Smooth Muscle - Dr. Karnam

Direct inhibition of membrane calcium channel activity: Membrane voltage-


dependent calcium channels, which normally open in response to a membrane
depolarization, can also be regulated by cAMP- and cGMP-dependent protein
kinases. Phosphorylation of membrane calcium channels by PKA or PKG
decreases intracellular calcium, leading to relaxation of smooth muscle

Activation of calcium-ATPase pump in the sarcoplasmic reticulum (SR):


Calcium-ATPase pump activation is responsible for the uptake of calcium into the
SR, actively decreasing intracellular calcium and thereby relaxing smooth muscle.
The activity of calcium-ATPase pump in the SR is regulated by the protein
phospholamban. Phosphorylation of phospholamban by PKG increases calcium-
ATPase pump activity and sequestration of calcium into the SR, thereby inducing
smooth muscle relaxation.

Inhibition of inositol 1,4,5-trisphosphate (IP3) generation: Because IP3 is the


principal compound responsible for releasing calcium from sarcoplasmic
reticulum; the inhibition of IP3 formation by PKA- or PKG-dependent
phosphorylation of phospholipase C-β is a potential mechanism of action of
cyclic nucleotide’s capacity to induce smooth muscle relaxation.

Inhibition of the IP3 receptor function (inhibition of calcium release): After


its generation by phospholipase C-β activity, IP3 acts as a second messenger for
smooth muscle contraction by binding to a specific receptor. The IP3 receptor is a
channel protein located in the sarcoplasmic reticulum, which opens up when
bound to IP3. The open IP3 receptor channel permits calcium release into the
cytoplasm, resulting in contraction of smooth muscle. Phosphorylation of IP3
receptor by PKA or PKG reduces the channel activity in response to IP3.

B. Reduction of contractile system calcium sensitivity (calcium desensitization).

Desensitization to calcium is defined as a decline in myosin light chain (MLC)


phosphorylation and force in the absence of proportional, or any, decline in cytosolic
calcium. The contractile state of smooth muscle is primarily dependent on the level of
MLC kinase and MLC phosphatase activity. Inhibition of MLC phosphatase could
produce contraction by increasing MLC phosphorylation in the absence of increased
intracellular calcium (calcium sensitization). Alternately, an increase in MLC
phosphatase activity could produce relaxation by decreasing MLC phosphorylation
without changing intracellular calcium (calcium desensitization).

290
Smooth Muscle - Dr. Karnam

K+ Ca2+

Plasma
membrane
PLC-β
+ - -
PKA/PKG
MLC -
+
+
MLCK IP3R
MLCPase
SR
MLC-p Ca2+ Ca2+

Relaxation

Figure 9. Mechanisms of action of cAMP-dependent protein kinase (PKA) and


cGMP-dependent protein kinase (PKG) to mediate smooth muscle relaxation. The
kinases decrease cytosolic calcium levels by 1) inhibiting the activity of phospholipase C-
β (PLC-β), thereby diminishing the synthesis of calcium mobilizing messenger, inositol
1,4,5-trisphosphate (IP3), 2) inhibiting calcium release from sarcoplasmic reticulum
stores (inhibiting IP3 receptor function), 3) stimulating calcium uptake by activating
sarcoplasmic reticulum Ca2+-ATPase pump, and 4) inhibiting the activity of
plasmalemmal Ca2+ channels either directly or indirectly (by activating K+ channels).
Decreased cytosolic calcium inhibits myosin light chain kinase activity, thereby
decreasing MLC phosphorylation. PKA and PKG also decrease myosin light chain
phosphorylation by stimulating myosin light chain phosphatase activity.
MLCK: myosin light chain kinase, MLCPase: myosin light chain phosphatase, MLC-p:
phosphorylated myosin light chain, PKA: cAMP-dependent protein kinase, PKG: cGMP-
dependent protein kinase, PLC-β: phospholipase C-β, SR: sarcoplasmic reticulum, IP3R:
inositol 1,4,5-trisphosphate receptor, + denotes stimulation, - denotes inhibition.

291
Smooth Muscle - Dr. Karnam

Table 1. Differences between smooth (unitary and multiunit) and skeletal


muscle

Feature Smooth Muscle Skeletal Muscle

Unitary Multiunit

Thick and Thin


Filaments Yes Yes Yes

Striations No No Yes

T-tubule No No Yes

Nucleus Single Single Many

Innervation Sparse Dense Each cell

Type of control Involuntary Involuntary Voluntary

Effect of nerve
stimulation Excitation/ Excitation/ Excitation
inhibition inhibition only

Gap junctions Many Some None

Spontaneous activity Yes No No

Source of Ca2+ Influx/ Influx/ Release (SR)


release (SR) release (SR)
Site of
action of Ca2+ Calmodulin Calmodulin Troponin

Speed of
contraction Very slow Very slow Fast/slow

292
Smooth Muscle - Dr. Karnam

Sample questions

1. Which of the following statements about smooth muscle is true?

(a) Smooth muscle is striated and involuntary


(b) Nuclei are peripherally located in the fibers
(c) Fibers are small and spindle shaped
(d) Branching fibers are a characteristic
(e) Contractions are rapid and forceful

Answer: C

2. Unitary smooth muscles have numerous

(a) Axon terminals


(b) Z-lines
(c) Gap junction
(d) T-tubules
(e) Intercalated discs

Answer: C

3. 17-kilodalton light chains are part of

(a) Myosin molecule


(b) Actin filaments
(c) Caldesmon
(d) Calponin
(e) Tropomyosin

Answer: A

4. The role of calcium in smooth muscle contraction is to

(a) Bind troponin and activate actomyosin interaction


(b) Bind calmodulin and activate myosin light chain kinase
(c) Bind directly to myosin light chain kinase
(d) Inhibit myosin light chain phosphatase
(e) Activate cAMP-dependent protein kinase

Answer: B

293
Smooth Muscle - Dr. Karnam

5. Which of the following events occur before inositol 1,4,5-trisphosphate (IP3)


generation in smooth muscle in the mechanism of excitation-contraction coupling?

(a) Release of calcium from ryanodine receptors


(b) Release of calcium from IP3 stores
(c) Activation of myosin light chain kinase
(d) Activation of cAMP-dependent kinase
(e) G protein-dependent stimulation of PLC-β

Answer: E

294
Autonomic Nervous System 4 - Dr. Biber

Autonomic Nervous System 4: Reflexes


Margaret C. Biber, D.Phil.

OBJECTIVES:

Please note that these objectives pertain to ANS lectures 1-4.

At the end of these lectures you should know and understand the following material:

1. The relationship between the organization of the sympathetic and parasympathetic


divisions of the ANS to their overall physiological effects.
2. Anatomical and functional differences between the skeletal neuromuscular
junction and autonomic neuroeffector junctions.
3. Transmitters used at ganglionic and neuroeffector junctions and highlights of the
transmitter life cycle: Storage, release, biological inactivation, metabolism and de
novo synthesis for acetylcholine (Ach), norepinephrine (NE) and the hormone,
epinephrine (EPI).
4. Receptor types for Ach and the catecholamines, NE and EPI and their effects.
5. The mechanism of action of other transmitters/mediators including ATP, NO and
peptides.
6. The organization of autonomic reflexes.
7. The overall physiological effects of the parasympathetic and sympatho-adrenal
systems and the receptor types that mediate the responses.

Reading: Berne, Levy, Koeppen and Stanton: Physiology, 5th edition. 2004; Ch. 11,
Pages 206-215 –Table 11-1 is too detailed. Use Table in handout.
Costanzo: Physiology, 2006 Ch. 2, Pages 45-64

Note: Please follow the version in the handout wherever discrepancies exist between the
textbooks and the handout.

LECTURE IV OUTLINE

AUTONOMIC REFLEXES
Schematic autonomic reflex
Comparison with somatic reflexes

Examples of reflex control:

Pupillary light reflex


Reflex activation of adrenal medullary secretion
Reflex control of gastrointestinal function

parasympathetically mediated excitation


sympathetically mediated inhibition

295
Autonomic Nervous System 4 - Dr. Biber

circuit involving CNS, collateral ganglia and ENS:


global sympathetic inhibition (fight or flight)

circuit involving collateral ganglia and ENS:


selective sympathetic inhibition
ongoing (tonic) sympathetic inhibition

NOVEL TRANSMITTERS

PEPTIDES
ATP
NITRIC OXIDE (NO)

REFLEXES

DEFINITION OF A REFLEX

Reflexes are stereotyped responses to specific sensory inputs

Sensory input over -----> Integrative -----> Efferent limb -----> Response elicited
afferent limb Center by effectors

CLASSIFICATION OF REFLEXES

1. Somatic reflex: mediated over skeletal muscle


2. Autonomic reflex: expressed by the effectors controlled by the ANS

COMPARISON OF SOMATIC AND AUTONOMIC REFLEXES:

SOMATIC REFLEX: tendon knee jerk

Figure 1.

• Integrative center lies in the spinal cord restricted to a specific segment


• Monosynaptic
• Somatic motor nerve (efferent) excites skeletal muscle
• Little modulation from higher centers

296
Autonomic Nervous System 4 - Dr. Biber

ANS REFLEX

Figure2

• Afferent limb: general visceral afferents but others used also as for somatic
reflexes
• Integrative center is not well localized in the CNS; can be in spinal cord, in brain
stem or not even in CNS
• Never monosynaptic
• Efferent limb includes a ganglion
• Efferent limb may produce excitation or inhibition of the effector
• Higher centers in the CNS (cognitive, emotional) exert profound influence on
ANS reflexes

EXAMPLES OF ANS REFLEXES

Pupillary light reflex


Stress activation of adrenal medulla
Reflex activation of the GI tract

Parasympathetic:

Vago-vagal activation of stomach


Gastro-colic reflex

Sympathetic:

Global inhibition (stress response)


Selective inhibition: Enterogastric reflex
Tonic inhibition

297
Autonomic Nervous System 4 - Dr. Biber

PUPILLARY LIGHT REFLEX:

This is an example of a reflex to a specific stimulus (light) that produces a very discrete
response that not subjected to much central modulation:

The circuitry of this reflex is well established.

Figure 3. PUPILLARY LIGHT REFLEX

LOCATION OF LESIONS WITHIN THE PATHWAY

Lesion to motor nerve of left eye:

Shine light in left eye → pupillary constriction in right eye but no response in left eye
Shine light in right eye → pupillary constriction in right eye but no response in left eye

Damage to optic nerve from left eye

Shine light in left eye → no response in either eye


Shine light in right eye → normal response in both eyes

REFLEX ACTIVATION OF THE ADRENAL MEDULLA

Reflex release of adrenal CA is normally triggered through activation of sympathetic


preganglionic nerves running in the splanchnic nerve that supplies the chromaffin cells.

298
Autonomic Nervous System 4 - Dr. Biber

Figure 4

WHAT TYPE OF STIMULI RELEASE ADRENAL MEDULLARY


CATECHOLAMINES?

Physiological stimuli for catecholamine release from the adrenal medulla are:

• psychological: emotional states (e.g. fear, anxiety, anger, "fight or flight


reaction")
• metabolic: hypoglycemia (the most powerful)
• hypothermia

Pathological stimuli include:

• severe hemorrhage
• heart attack
• pain

After severe trauma, the adrenal medulla may be depleted of CA and opioid peptides
(enkephalins) that are co-released with the amine.

IMPORTANT NOTE: ASTHMATIC ATTACKS DO NOT TRIGGER RELEASE OF


CA FROM THE ADRENAL MEDULLA. This is surprising given that EPI produces
relaxation of the airways via beta-2 receptor activation. Selective beta-2 agonists can be
used to treat asthmatic attacks acutely.

ANGIOTENSIN-II

Under certain pathological conditions (e.g. severe loss of blood), the hormone,
ANGIOTENSIN-II also produces CA secretion. Under these conditions, circulating
levels of angiotensin-II become markedly elevated and the angiotensin-II interacts

299
Autonomic Nervous System 4 - Dr. Biber

directly with angiotensin-II receptors on the chromaffin cells of the adrenal medulla to
produce CA release. You will learn about the production of angiotensin –II in the lectures
on renal physiology.

REFLEX CONTROL OF GI FUNCTION

PARASYMPATHETIC CONTROL (EXCITATORY)

• Reflex activation of the stomach: Food within the stomach stimulates chemico
and mechano receptors in the stomach wall and signals are conveyed to the CNS
over GVA fibers. Reflex activation of the vagus stimulates mechanical and
secretory activity in the stomach wall.
• Reflex activation of the bowel: The presence of food in the stomach or upper
part of the intestine triggers activity in sacral parasympathetic nerves innervating
the colon and produces a massive propulsive contraction. This is the gastro
(stomach) colic (large intestine) reflex.

The parasympathetic coordinates digestive activity along the length of the GI tract.

ENTERIC NERVOUS SYSTEM

The ENS is an independent nervous system comprised of local sensory neurons, local
interneurons and local motor neurons. Parasympathetic postganglionic nerves are just one
type of motor neuron in the ENS. It extends along the entire length of the GI tract.

The ENS exerts local reflex control over many digestive processes. It mediates many
stereotyped movements,
e.g. peristalsis.

300
Autonomic Nervous System 4 - Dr. Biber

Figure 5. REFLEX CONTROL OF GI FUNCTION


PARASYMPATHETIC CONTROL (EXCITATORY)

SYMPATHETIC CONTROL OF GI FUNCTION (INHIBITORY)

• Global inhibition (fight or flight response)


• Discrete inhibition of specific area (e.g. gastric emptying) during digestion
• Tonic inhibition of tonic parasympathetic tone

Figure 6

301
Autonomic Nervous System 4 - Dr. Biber

Sympathetic activity regulates GI function at two different levels:

• via activation over the CNS (global inhibition)


• via activation of postganglionic neurons within the collateral ganglia without any
CNS involvement (discrete inhibition and tonic inhibition)

Most sympathetic inhibition of GI function is mediated presynaptically over alpha-2


hetero-receptors that lie on the terminals of postganglionic parasympathetic nerves.
Inhibition of GI motility and secretory activity is therefore INDIRECT. NE released from
sympathetic postganglionic neurons inhibits the release of the excitatory transmitter, Ach,
thereby reducing the activation of the target tissues.

GLOBAL INHIBITION OF GI FUNCTION IN THE FIGHT OR FLIGHT


RESPONSE

The afferent limb of this reflex involves cognition (recognition of a threatening situation)
as well as CNS regions involved in autonomic control.

Sympathetic nerves innervating the GI tract are activated. Released NE produces

• Inhibition of the motility of GI smooth muscle and secretory activity of many


glands mediated over presynaptic alpha-2 hetero receptors that lie on the
terminals of the excitatory postganglionic parasympathetic nerves and inhibit
release of Ach.
• Reduction of blood flow via alpha 1 receptor activation on the smooth muscle of
blood vessels.
• Activation of sphincter muscles (alpha-1 receptors) to prevent food from moving
from one part of the gut to another.

Adrenal medullary EPI is released into the circulation and

• produces GI smooth muscle relaxation directly via beta-2 receptors

ENTERO-GASTRIC REFLEX

• Activated after the stomach has emptied some of its contents into the duodenum.
• Inhibition of further emptying allows the acid pH of the chyme to be raised so the
pancreatic proteases can work. Bicarbonate secreted by the exocrine pancreas
neutralizes the acid chyme from the stomach.
• The stomach is inhibited from further emptying until the pH of the chyme in the
duodenum has been raised.

302
Autonomic Nervous System 4 - Dr. Biber

This reflex involves:

• Activation by acid of LOCAL SENSORY NEURONS within the ENTERIC


NERVOUS SYSTEM in the wall of the duodenum
• Activation of POSTGANGLIONIC SYMPATHETIC NEURONS in the
COELIAC (COLLATERAL) GANGLION by these local sensory neurons
• Inhibition of the stomach motility (pyloric end). Inhibition is brought about by NE
released from sympathetic terminals acting at PRESYNAPTIC ALPHA-2
HETERO-RECEPTORS on the terminals of the excitatory parasympathetic
postganglionic nerves to reduce release of Ach.

This reflex does not involve the CNS. The efferent (motor) limb is the postganglionic
sympathetic nerve.

TONIC SYMPATHETIC INHIBITION

Ongoing activity of the GI tract is always restrained by tonic sympathetic activity.

• This inhibition is mediated by postganglionic sympathetic nerves within the three


collateral ganglia in response to activation of local sensory neurons within the
enteric NS.
• The NE released from the terminal branches of the sympathetic postganglionic
neurons acts on presynaptic alpha-2 hetero-receptors to inhibit release of
excitatory transmitter (ACh) from local motor neurons (parasympathetic
postganglionic neurons).

303
Autonomic Nervous System 4 - Dr. Biber

SELECTIVE SYMPATHETIC INHIBITION OF GI FUNCTION:

Figure 7. EXAMPLE: INHIBITION OF GASTRIC EMPTYING DURING


DIGESTION OF A MEAL (ENTERO-GASTRIC REFLEX)

304
Autonomic Nervous System 4 - Dr. Biber

Figure 8. ANS CIRCUITS CONTROLLING GI FUNCTION

NOVEL TRANSMITTERS

The ANS uses other signaling molecules besides NE and Ach. This is particularly true in
the case of the ENS that contains many PEPTIDERGIC NEURONS.

These novel transmitters fall into three types:

• PEPTIDES
• ATP
• NITRIC OXIDE, (NO)

PEPTIDE TRANSMITTERS

As noted earlier, a distinct class of dense cored vesicles found in the terminals of some
autonomic nerves store specific peptides. Thus, vasoactive intestinal peptide (VIP) occurs
in some postganglionic parasympathetic nerves, whereas neuropeptide Y (NPY) is found
in some sympathetic postganglionic nerves.

305
Autonomic Nervous System 4 - Dr. Biber

These peptides have particular characteristics:

• Release only occurs with high frequency nerve stimulation


• Exocytosis takes place at non-junctional membrane
• Effects are long lasting (mins) indicating that the biological inactivation by
peptidases is slow
• VIP is found in the parasympathetic innervation to salivary gland. It is released on
high frequency stimulation when it enhances the stimulatory effects of ACh on
salivary secretion
• NPY occurs in sympathetic nerves supplying blood vessels. It is released in
response to high frequency stimulation and augments the contractile response
elicited by co-released ATP and NE.
• Autonomic nerves can thus release multiple agents (ATP, small transmitters: NE
& ACh, neuropeptides) which have different time courses of action. The
composition of the mixture depends on the strength and duration of activation.
• Many neurons within the ENTERIC NERVOUS SYSTEM (see above) use
peptides exclusively as transmitters. Examples include enkephalin, substance P,
somatostatin, cholecystokinin, CGRP.

ATP - A TRANSMITTER IN THE SYMPATHETIC NERVOUS SYSTEM

• ATP is co-stored with NE in synaptic vesicles (see FIG 9) (4 ATP: 1 NE) and co-
released with NE on exocytosis of synaptic vesicle in response to nerve
stimulation.
• ATP has an excitatory transmitter role, along with NE, in smooth muscle of many
arterioles and small arteries and of the vas deferens. It produces a rapid
contraction of smooth muscle that precedes the contraction produced by NE. This
action is always present since ATP is always released along with the NE.
(Contrast this with the release of peptides).

Figure 9

306
Autonomic Nervous System 4 - Dr. Biber

• How does it work? ATP opens a ligand-gated cation channel, highly permeable to
calcium ions, by binding to a so called PURINERGIC RECEPTOR.
• The increase in internal calcium triggers calcium-induced calcium release from
the smooth muscle SR and a very rapid muscle contraction that precedes the
contraction produced by NE at alpha1 receptors.

NITRIC OXIDE (NO)

Parasympathetic stimulation can produce relaxation of certain smooth muscles (GI tract;
erectile tissue of the penis). An important mediator is the gas, nitric oxide (NO).

• NO is synthesized on demand and has a brief lifetime. It is made by calcium-


calmodulin-dependent NO synthetase which is activated by a rise in intracellular
free calcium and converts arginine into NO and citrulline
• NO diffuses freely across the nerve terminal membrane into the surrounding
smooth muscle where it stimulates soluble guanylyl cyclase and generates cGMP
• cGMP activates protein kinase G and causes relaxation of smooth muscle through
multiple pathways that all reduce phosphorylation of myosin light chain (see Dr
Karnam’s notes on smooth muscle relaxation)
• NO was first discovered as an agent generated by the endothelium of blood
vessels in response to various agents, hence the name ENDOTHELIUM
DERIVED RELAXING FACTOR or EDRF.

Figure 10

307
Autonomic Nervous System 4 - Dr. Biber

GLOSSARY OF TERMS FOR PHYSIOLOGY OF THE AUTONOMIC


NERVOUS SYSTEM

The following definitions will aid in the understanding of lecture and reading material on
the autonomic nervous system.

adrenergic - adjective pertaining to norepinephrine (NE); e.g., an adrenergic nerve


is one which liberates NE as a transmitter substance.

agonist - (stimulant) - drug which combines with a receptor to initiate a


response

anhydrosis - absence or deficiency of sweat

antagonist - (blocker) - drug which combines with receptor but does not initiate a
response; will block access of agonist to receptor

A-V node - atrio-ventricular node - area of specialized conduction tissue which


transmits electrical impulses from atria of heart to the ventricles

bradycardia - slowing of the heart rate

bronchoconstriction - a narrowing of the lumina of air passages in lungs caused by


contraction of smooth muscle in bronchi and bronchioles

catecholamine - a ß-phenylethylamine or ethanolamine in which the benzene ring


contains hydroxyl groups in the 3 and 4 positions.

dopamine

norepinephrine
pinephrine

cholinergic - of or pertaining to acetylcholine; e.g., a cholinergic nerve liberates


acetylcholine as a transmitter substance

cholinomimetic - a substance that mimics the effects of acetylcholine at cholinergic


receptors on a neurone or effector cell

chronotropic - affecting time or rate, more specifically rate of contraction of heart;


positive chronotropic effect = an increase in heart rate;
negative chronotropic effect = a decrease in heart rate

cycloplegia - paralysis of ciliary muscles in the eye resulting in loss of

308
Autonomic Nervous System 4 - Dr. Biber

accommodation for near vision, i.e. the ability to focus the lens.

depressor - lowering of blood pressure

depressor substance - agent that lowers blood pressure

effector cell - cell whose activity is regulated by nerve or hormone which produces
an effect, e.g. contraction, secretion.

emesis - vomiting

exocytosis - process whereby the membrane of an intracellular body fuses with the
plasma or cell membrane allowing expulsion of the contents of the
intracellular body to the extracellular space, but retention of the
membrane.

ganglia - collection of nerve cell bodies located outside the central nervous
system

glycogenolysis - the splitting of glycogen to liberate glucose

junction - general term to describe area where chemical transmission of impulses


occurs between 2 nerves or between a nerve and an effector cell as in
neuromuscular junction or neuroeffector junction.

lacrimation - secretion of tears

lipolysis - the breakdown of fat resulting in liberation of free fatty acids

miosis - constriction of the pupil

mydriasis - dilation of the pupil

myocardial - of or pertaining to heart muscle

nerve conduction - movement of electrical impulses along an axon

neurochemical transmission the process whereby impulses are transferred from one nerve to
- another or from a nerve to an effector cell by means of the release of a
chemical.

neuromuscular junction - the area where somatic motor nerves transmit impulses to skeletal
muscle fibers.

pressor - adjective referring to an increase in blood pressure;

309
Autonomic Nervous System 4 - Dr. Biber

pressor substance - an agent that increases blood pressure

S-A- node - sino-atrial node - area of specialized myocardial tissue in the right
atrium which spontaneously depolarizes giving rise to heart beat.

sympathomimetic - agonist which mimics the effects of activity of sympathetic division of


the autonomic nervous system.

synapse - junction between two nerves. The signal from the presynaptic (input)
neuron is conveyed to the postsynaptic (output) neuron: Conduction is
unidirectional.

tachycardia - rapid heart rate.

tone - normal state of tension or activity

vasoconstriction - constriction of blood vessels

vasodilatation - dilation of the blood vessels

xerostomia - dry mouth associated with decreased production of saliva

REVIEW QUESTIONS: Autonomic nervous system.

Pick the single best answer

Question 1. The sympathetic division of the autonomic nervous system has all the
following characteristics except :

A. the cell bodies of the preganglionic neurons lie in the intermediolateral columns
of the spinal cord
B. has a well-organized ganglion system
C. all postganglionic neurons release norepinephrine
D. the system participates along with the adrenal medulla in the fight and flight
response

A. This is a true statement and therefore not the correct answer. The cell bodies
of the preganglionic neurons lie in the intermediolateral column.
B. This is not the exception. The sympathetic nervous system is characterized by
its well-defined ganglion system.

310
Autonomic Nervous System 4 - Dr. Biber

C. This is the exception. The sympathetic postganglionic fibers innervating the


sweat glands release acetylcholine rather than norepinephrine. (Postganglionic
sympathetic neurons to the sweat glands of the palms of the hands do however
release norepinephrine and are involved in emotional sweating (clammy
hands).
D. This is a true statement and therefore not the answer.

Question 2. Parasympathetic stimulation causes all the following responses EXCEPT :

A. increased gastrointestinal motility


B. constriction of the pupil (miosis)
C. slowing of the heart (bradycardia)
D. constriction of blood vessels

A, B & C are true. Ans D.

D is an incorrect statement and therefore the correct answer to the question. The
smooth muscle of blood vessels is innervated by the sympathetic nervous system.
Stimulation of the sympathetic nerves causes constriction of blood vessels via an
alpha-1 adrenoceptor action of the released norepinephrine.

Question 3. Acetylcholine released from postganglionic parasympathetic nerves has all


the following characteristics EXCEPT:

A. excitatory or inhibitory effects depending on the site of action


B. action at the neuroeffector junction blocked by atropine
C. coreleased with peptides at some synapses
D. inactivated by diffusion from the site of action

A, B & C are true. Ans D. Acetylcholine is inactivated by acetylcholinesterase.

Matching: Answers may be used once, more than once or not at all

Direct activation of certain receptor subclasses by the relevant transmitter produces


certain responses. Match the appropriate receptors with the response.

A. Muscarinic receptors
B. Beta2 receptors
C. Alpha1 receptors
D. Nicotinic receptors

1. Increased motility of the GI tract


2. Narrowing of the pupil (miosis)
3. Relaxation of bronchial smooth muscle (bronchodilation)
4. An increase in blood pressure
5. Release of catecholamines from the adrenal medulla

311
Autonomic Nervous System 4 - Dr. Biber

6. Slowing of the heart (bradycardia)


7. Generalized sweating

Answers: 1. A; 2. A (pupillary sphincter has muscarinic receptors); 3. B (bronchial


smooth muscle has beta-2 receptors); 4. C (alpha1 receptors on the smooth muscle of the
blood vessels cause constriction of the blood vessel in response to released NE); 5. D (the
medullary chromaffin cells have nicotinic receptors); 6. A; bradycardia is mediated by
muscarinic receptors; 7. A (generalized sweating is mediated by muscarinic receptors
activated by Ach released from postganglionic sympathetic nerves)

REVIEW EXERCISES FOR THE AUTONOMIC NERVOUS SYSTEM

1. Indicate which of the following types of adrenergic receptors (alpha-1, beta-1 or


beta-2) are responsible for the following sympathetic effects:

a. mydriasis (dilation of the pupil)


b. vasoconstriction (contraction of vascular smooth muscle)
c. tachycardia (rapid heart beat)
d. increased myocardial contractile force
e. vasodilation in skeletal muscle
f. bronchodilation
g. sweating (emotional)

Answers:

a. alpha-1 receptors cause the radial muscle of the pupil to contract and
hence produce dilation of the pupil
b. alpha-1
c. beta-1
d. beta-1
e. beta-2 (effect of EPI)
f. beta-2 (effect of EPI)
g. alpha-1

2. Figure out how muscarine and atropine would affect:

a. pupillary diameter
b. salivation
c. G-I function
d. urinary bladder function
e. the heart
f. sweat glands

Answers: muscarine

a. miosis (constriction)

312
Autonomic Nervous System 4 - Dr. Biber

b. stimulation
c. enhanced motility and secretory activity of the GI tract
d. contraction of the bladder
e. slowing (bradycardia)
f. stimulation of sweating

atropine

a. pupillary dilation
b. inhibition of salivary secretions--dry mouth
c. inhibition of GI motility and secretory activity
d. difficulty urinating
e. tachycardia
f. inhibition of generalized sweating--dry skin

313
Cell Physiology Problem Solving - Dr. Costanzo

Cell Physiology Problem Solving


Linda S. Costanzo, Ph.D.

1. Child with spherocytosis

The mother of a five-year old child became concerned when the child developed a
persistent fever and jaundice. The pediatrician ordered blood tests that showed an
abnormally low hemoglobin level. Inspection of the child’s red blood cells in
isosmotic saline showed that most of them had a spherical shape. The pediatrician
ordered an osmotic fragility test on the blood. In this test a blood sample is
equilibrated in a series of NaCl solutions that range in osmolarity from isosmotic
(0.9%, i.e., 0.9 g/100 ml) to progressively more hyposmotic solutions, ending with
0.05%, i.e., 0.05g/100ml. In each solution the absorbance is read on a
spectrophotometer set at a wavelength of 540 nm, which allows for the detection of
hemoglobin in the solution. Hemoglobin appears in the solution when red blood cells
have lysed (called hemolysis). The % hemolysis for each solution was then
calculated.

The data from the child were compared with data from a normal subject by plotting %
hemolysis for each subject vs the NaCl concentration in g/100 ml (percent hemolysis
on the y-axis and NaCl concentration on the x-axis). It is conventional to start the x-
axis with the highest NaCl concentration at the origin (0.85%) with progressively
lower concentrations running to the right ending with 0.05%. The patient’s osmotic
fragility curve confirmed the physician’s provisional diagnosis.

Percent hemolysis (normal


NaCl concentration (g/100 ml) Percent hemolysis (patient)
subject)
0.85 0.00 0.00
0.65 0.17 21.89
0.60 0.12 39.74
0.55 0.08 58.88
0.50 0.00 88.22
0.45 49.07 95.68
0.40 90.03 96.86
0.35 99.17 96.66
0.30 99.20 98.23
0.05 100.00 100.00

A. Plot the osmotic fragility curves for both data sets on the same graph.

314
Cell Physiology Problem Solving - Dr. Costanzo

B. Compare the osmolarities (in mOsm/L) at which the percent hemolysis equals
5% between samples. Repeat for 50% hemolysis.

C. Why does the patient’s osmotic fragility curve differ from that of the normal
subject?

D. Why were the patient’s red blood cells spherical in isosmotic saline?

2. Hypokalemic periodic paralysis

Joe, a 20 year old decathlete at Penn State, has been feeling extremely weak after his
events; his legs feel “like rubber.” Eating, especially carbohydrates, makes him feel
worse. At a Penn State Invitational, he had to be carried off on a stretcher and taken
to the Emergency Department. As part of the work-up, Joe’s serum K+ concentration
was measured and found to be normal (4.5 mEq/L). However, after a strenuous
exercise treadmill test, his serum K+ was measured again and found to be severely
depressed (2.2 mEq/L). Joe was diagnosed with an inherited disorder called
hypokalemic periodic paralysis and was subsequently treated with K+
supplementation.

A. What is the normal distribution of K+ between ICF and ECF? Where is most of
the body’s K+ located?

B. Using whatever reference sources you have available (textbook, internet), identify
the major factors that can alter the distribution of K+ between ICF and ECF.

C. How does a decrease in serum K+ concentration lead to skeletal muscle


weakness?

D. Propose a mechanism whereby eating carbohydrates might worsen Joe’s


weakness?

315
Cell Physiology Problem Solving - Dr. Costanzo

E. How would K+ supplementation be expected to improve Joe’s condition?

F. Another inherited disorder, called primary hyperkalemic periodic paralysis,


involves an initial period of spontaneous muscle contractions (spasms), followed
by prolonged muscle weakness. Using your knowledge of the ionic basis for the
action potential, propose a mechanism whereby an increase in serum K+
concentration could lead to spontaneous contractions followed by prolonged
weakness.

3. Myasthenia gravis

Sylvia is a 32 year old mother of two small children. Over the past six months, she
has experienced strange symptoms, including eyestrain when she reads for more than
15 minutes and fatigue when she does simple things like chew her food, brush her
teeth, or dry her hair. She is having a very difficult time caring for her active children.
She was evaluated by her physician, who suspected myasthenia gravis. While
awaiting the results of a serum antibody test, the physician initiated a trial of
neostigmine, an acetylcholinesterase inhibitor. Sylvia felt better immediately while
take the drug, and her strength returned to almost normal. Meanwhile, the results of
the antibody test were positive and confirmed the diagnosis of myasthenia gravis.

A. What antibody do you think was measured in Sylvia’s serum? Against what
protein is this antibody directed?

B. Based on your knowledge of the steps in neuromuscular transmission, explain


why severe weakness occurs in myasthenia gravis?

C. How did neostigmine improve Sylvia’s muscle strength?

D. Of the following drugs listed below that act at various stages of neuromuscular
transmission, which one/s would be contraindicated in myasthenia gravis?

316
Cell Physiology Problem Solving - Dr. Costanzo

Botulinis toxin

Curare

Neostigmine

Hemicholinium

317
Review Session - Dr. Feher

Review Session
Joseph Feher, Ph.D.
I. SKELETAL MUSCLE MECHANICS

A. The motor unit is the sum of all muscle fibers innervated by a single
motor neuron.

B. The muscle twitch is the force developed upon a single stimulation of the
muscle.

1. The twitch time varies with the kind of muscle.

a. Slow twitch or Type I fibers


b. Fast twitch or Type II fibers

2. The action potential is shorter than the twitch: it is a trigger

C. There are 3 ways to “grade” muscle force:

1. Recruit more motor units.

a. This is done physiologically


b. Small motor units are recruited first: this is the size
principle.

2. Increase the frequency of firing.

a. Intensity of stimulation is encoded by the frequency


b. Summation of twitches gives rise to the tetanus.
c. Tetanic frequency depends on the muscle type.

3. Change the length of the muscle.

a. Muscles usually have a fixed rest length.

318
Review Session - Dr. Feher

b. Muscle length is a function of anatomy and is not subject to


physiological control.
c. Length-tension curve gives rise to the sliding-filament
hypothesis.

D. Force varies inversely with speed of contraction.

1. Maximum isometric force is at zero velocity.


2. Maximum velocity is at zero force.

E. Power is force times velocity.

1. Power is maximum at about one-third Fmax


2. Power is maximum at about one-third Vmax
3. Maximum power is 2-3 times greater in fast-twitch fibers (per unit
weight)

F. Most muscles are heterogeneous mixtures of muscle fiber types.

1. Muscles can be classified as Slow (S), Fast Fatiguable (FF), Fast


Intermediate (FI) and Fast Fatigue-Resistant (FR)

II. CONTRACTILE MECHANISMS IN SKELETAL MUSCLE

A. Muscle cells have a highly organized structure.

1. Muscles are large and multinucleated


2. Muscles contain myofibrils

a. Myofibrils are bundles of myofilaments


b. Myofibrils show striations
c. Thick filaments are mostly myosin and constitute the A
band

319
Review Session - Dr. Feher

d. Thin filaments are mostly actin and constitute the I band


e. The unit of contraction is the sarcomere: Z line to Z line
f. The myofilaments form interdigitating hexagonal arrays.
g. Actin filaments are anchored at the Z disk by actinin.

3. Myofibrils are surrounded by a membranous network, the


sarcoplasmic reticulum.

B. The sliding filament hypothesis explains the length-tension curve.

1. Force depends on the overlap of the A and I band filaments


2. The sliding-filament hypothesis predicts that force is low at low
overlap and high at high overlap
3. The sliding-filament hypothesis predicts that force is reduced when
filaments from adjacent Z lines interfere with each other.

C. The filaments produce force by cross-bridge cycling.

1. Myosin is an actin-activated ATPase


2. Force of contraction is related to the number of cross-bridges
3. Velocity of shortening is related to the turnover of cross-bridges

a. Fast and slow twitch muscles have different myosin


isoforms
b. The different turnover of myosin explains the different
velocity of shortening.

D. Muscles can be classified by myosin staining as I, IIA, IIB and IIC.

E. Force is transmitted to the outside of the muscle through the cytoskeleton


anchored to costameres.

320
Review Session - Dr. Feher

III. EXCITATION-CONTRACTION COUPLING

A. Excitation-Contraction coupling links the action potential to contraction.

B. Excitation on the nerve (the action potential) is converted to an action


potential on the muscle. This is neuromuscular transmission.

C. The action potential on the muscle cell membrane is converted to a Ca2+


transient.

1. The action potential travels along the sarcolemma and down the
T-tubules.
2. The T-tubules are invaginations of the surface membrane at the A-I
junction.
3. T-tubules make contact with the sarcoplasmic reticulum at triads
that consist of one T-tubule and two terminal cisternae from the
SR.
4. Depolarization of the T-tubule is sensed by dihydropyridine
receptors (DHPR) which are voltage sensors.
5. In skeletal muscles, the DHPR makes direct contact with
ryanodine receptors (RYR1) on the SR membrane.
6. The RyR1 receptors are Ca2+ channels. When the DHPR sense a
depolarization, the RYR1 channels open and Ca2+ stored in the
terminal cisternae is released.
7. Excitation on the T-tubule is therefore transformed into a rise in
[Ca2+] in the skeletal muscle cell.
8. The Ca2+ transient is ended by re-uptake into the SR by the SR Ca-
ATPase (SERCA1).

D. The Ca2+ transient regulates muscle contraction.

1. At rest, actin and myosin are prevented from interactin by


tropomyosin.
2. Tropomyosin and actin bind troponin, which consists of three
proteins.

321
Review Session - Dr. Feher

a. Troponin T binds tropomyosin.


b. Troponin C binds Ca2+
c. Troponin I inhibits the interaction between actin and
myosin filaments.

3. Ca2+ binding to regulatory sites on TnC causes a conformational


change
4. Tropomyosin moves out of its inhibitory position when TnC is
bound with Ca2+
5. Actin and Myosin can interact to produce force and split ATP.
6. When Ca2+ desorbs from TnC, the resting state is re-established.

E. Summation of twitches (tetanus) is caused by summation of the Ca2+


transients

1. Force follows TnC Ca2+ saturation because of the series-elastic


component.
2. Prolonged activation raises force because the series-elastic
elements are fully stretched
3. Repetitive stimulation causes repetitive Ca2+ release

IV. MUSCLE ENERGETICS AND FATIGUE

A. Hydrolysis of ATP is the chemical fuel for mechanical work.

B. Rate and amount of ATP consumption varies with intensity and duration
of exercise.

C. When activated, muscles use ATP fast

D. Metabolism regenerates ATP at different time scales and capacities

1. Direct phosphorylation

322
Review Session - Dr. Feher

a. Creatine phosphate
b. Myokinase

2. Glycolysis provides rapid but low-capacity supply of ATP


3. Oxidative Phosphorylation provides slower but higher capacity for
ATP supply

E. Lactic Acid is produced fasting during high intensity exercise.

1. Lactate production oxidizes NAD+ to NADH, allowing glycolysis


to proceed without mitochondrial oxidation.
2. Lactate can be metabolized within muscle fiber mitochondria
3. Lactate goes to liver where it can be re-packaged as glucose (Cori
cycle)

F. Muscles can be classified based on their metabolic properties.

1. Slow oxidative (SO), Fast Glycolytic (FG) Fast Oxidative-


Glycolytic (FOG)
2. Whole muscle are mixtures of muscle fiber types.

G. Mismatch of lactic acid production and oxidation determines rate of lactic


acid release into the blood

1. Muscles always make lactic acid


2. Muscles always oxidize lactic acid
3. During activity, making lactic acid outstrips oxidation
4. Lactate can be shuttled to

a. the mitochondria
b. oxidative fibers
c. liver

323
Review Session - Dr. Feher

H. Exercise increases glucose uptake by muscle independent of insulin

I. Fatigue is a reduction in developed force because of previous activity.

1. Cause of fatigue is usually not upper or lower motor neurons, nor


failure of neuromuscular transmission.
2. Early fatigue on maximal stimulation is probably due to Pi and pH.
3. Fatigue at submaximal work rates depends on glycogen stores.
4. People with glycogenoses often have exercise intolerance

J. Training can increase strength or endurance.

1. Rapid strength gains early on are neuronal training.


2. Muscle strength depends on muscle size.
3. Endurance training increases capillarity, mitochondria and
myoglobin.
4. It is unlikely that fiber types interconvert in human training
regimens.

324
Intro to Cardiovascular System - Dr. Baumgarten

Introduction to Cardiovascular System


Clive M. Baumgarten, Ph.D.

OBJECTIVES:

1. Describe organization of heart and vasculature


2. Describe the arterial pulse
3. Describe the factors regulating the cardiac pump
4. Identify components of the vasculature
5. Describe the cardiac cycle

I. OVERVIEW

A. Organization of the Cardiovascular System

1. Systemic Circulation –
Left heart and systemic
vasculature
2. Pulmonary Circulation −
Right heart and pulmonary
vasculature
3. Right and left heart are
coupled. Output of one is
input of the other. In the
steady state, the amount
pumped by right and left
hearts must be exactly
equal.
4. Blood flow to each organ is
arranged in parallel.
5. Blood flow to each organ is
controlled independently
by dilation or constriction
of arterioles leading to the
organ.

325
Intro to Cardiovascular System - Dr. Baumgarten

B. Arterial Pulse

1. Systole: LV contraction.
2. Diastole: LV relaxation.
3. Systolic pressure: Maximum pressure; during systole.
4. Diastolic pressure: Minimum pressure; at end of diastole.
5. Dicrotic Notch or Incisura: Marks end of systole and beginning of diastole.
6. Dicrotic wave: Pressure changes in diastole.
7. Arterial pressure rises during ascending limb (first part of systole) and falls
during descending limb.

II. THE PUMP

326
Intro to Cardiovascular System - Dr. Baumgarten

A. Cardiac output (CO): Amount of blood pumped per minute. CO is product of


stroke volume (SV), the amount pumped per beat, and heart rate (HR).

CO = SV × HR
Typical: CO = 5 L/min, SV = 70 mL/beat, HR = 72 beats/min

B. The heart pumps blood from low pressure (veins) to high pressure (arteries).

1. Pressure in the vena cava, the input to the heart, is near 0 mm Hg. Blood flows
from the veins, through the atria, into the ventricle to fill the heart. Central
venous pressure equals right atrial pressure (RAP or PRA).
2. Output of the heart is to the aorta. The contraction of the left ventricle must
increase ventricular pressure to aorta pressure (Paortic) to eject blood.
3. LV versus RV
a. LV free wall and intraventricular septum are thick because LV pumps
blood into high pressure aorta.
b. RV free wall is much thinner than LV free wall. Pressure in pulmonic
artery is much lower than pressure in the aorta.

C. Cardiac action potential (AP) originates in muscle and initiates contraction.

1. The electrical impulse arises spontaneously in the sino-atrial node (SAN), the
pacemaker of the heart, which is located in the right atrium.
2. Organized contraction and pumping of blood require conduction of the AP to
the rest of the heart over a specific pathway and with specific timing. The
electrical impulse first activates the atria and then spreads via the atrio-
ventricular node (AVN) to His-Purkinje system to the ventricles.
3. HR and conduction of AP are modulated by the autonomic ns.
4. Onset of contraction is delayed by ~50 ms after upstroke of AP.

D. Cardiac Function Curve − Describes overall function of heart as a pump.


Relationship between input to the heart, RAP, and output of the heart, CO.

III. THE VASCULATURE

A. Components

1. Arteries
Thick, muscular, and elastic walls. Distensiblity of arteries acts to smooth out
BP pulsations. Elastic recoil of large arteries during diastole helps maintain
propulsion of blood between heart beats.

327
Intro to Cardiovascular System - Dr. Baumgarten

2. Arterioles
Smallest branches of arteries. Undergo vasoconstriction and vasodilation to
regulate the blood flow by changing resistance to flow to an organ or through
the entire circulation. Arteriole diameter is regulated by autonomics,
circulating vasoactive hormones, and local metabolites. For the systemic
circulation as a whole, the resistance to blood flow is total peripheral
resistance (TPR).
3. Capillaries
Thin-walled, small diameter vessels specialized for exchange.
4. Venules
Smallest branches of veins. Thin-wall vessels with high compliance. (i.e.,
stretch easily). Undergo venoconstriction and venodilation to regulate the
volume of the vasculature (unstressed volume) and thereby central venous
pressure and cardiac filling. Regulated by autonomics.
5. Veins
Contain majority of blood volume and act as a reservoir for blood. Can
accommodate large volume change with small change in pressure (i.e., veins
are distensible or compliant)

B. Vascular Function Curve − Describes the function of the vasculature as the


relationship between RAP and venous return, the amount of blood returning to
the vena cava, which must equal CO in the steady-state.

IV. CARDIAC CYCLE

A. Cardiac Cycle = Systole + Diastole. Events are based on the activity of LV.

1. Systole
a. Cardiac contraction, typically ~350 ms.
b. Begins with closure of mitral valve, ends with closure of aortic valve
2. Diastole
a. Relaxation and filling of heart. Duration variable, depends on HR.
3. Timing of electrical activity sets timing of mechanical events
a. Opening and closing of valves, changes in cardiac and arterial
pressure, and heart sounds. Mechanical contraction follows membrane
depolarization by ~50 ms.

328
Intro to Cardiovascular System - Dr. Baumgarten

B. Valves:

1. Semi-lunar valves: Left, aortic valve; Right, pulmonic valve.


2. AV valves: Left, mitral valve; Right, tricuspid valve.
3. Valves open and close at different times to give blood flow in one direction.
There are no valves between atria and great veins.

C. Components Electrocardiogram (ECG)

1. P wave: Atrial depolarization


2. QRS complex: Ventricular depolarization
3. T wave: Ventricular repolarization

329
Intro to Cardiovascular System - Dr. Baumgarten

D. Events in Cardiac Cycle Are Based on LV.

1. Systole
a. Before systole begins, ventricular pressure is lower than arterial pressure
and lower than atrial pressure. Therefore, the semi-lunar valves are closed
(L − aortic, R − pulmonic) and the AV valves are open (L − mitral, R −
tricuspid). Systole begins with the onset of LV contraction.
b. Increasing pressure immediately causes AV valves to snap shut producing
the first heart sound (S1 − “lubb”). Both semi-lunar and AV valves are
closed trapping a volume of blood in the ventricle (end-diastolic volume).
c. Contraction of ventricle against fixed volume of blood (isovolumetric
contraction) increases LV pressure until LV pressure reaches Paortic.
d. Semi-lunar valves open.
e. Ejection of blood into arteries. Arterial pressure trace shows increase in
pressure during systole as blood is rapidly forced into aorta. The
maximum pressure is systolic pressure.
f. As contraction ends, LV pressure begins to fall. When LV pressure falls
below Paortic, the aortic valve snaps shut. The pulmonic valve. closes after
the aortic v. Closing of semi-lunar valves causes second heart sound (S2 −
“dup”)
g. The end of systole is marked by the dicrotic notch in the arterial pressure
trace.

330
Intro to Cardiovascular System - Dr. Baumgarten

2. Diastole
a. Initially, both the semi-lunar and AV valves are closed. Ventricular
pressure falls as muscle relaxes. The first part of relaxation occurs at a
constant volume (isovolumetric relaxation).
b. LV pressure falls to below atrial pressure and AV valves open.
c. Ventricle refills almost entirely by blood flowing from veins through the
open atria.
d. Atrial contraction provides the final portion of ventricular filling.
e. During diastole, arterial pressure initially increases then slowly falls
(dicrotic wave). The elastic recoil of arteries and reflections of the
pressure wave smooths out the pressure pulse and act maintain arterial
pressure during diastole. The minimum pressure is diastolic pressure.

V. REFERENCES

A. Koeppen, B.M. and Stanton, B.A. Berne & Levy Physiology, 6th Ed., 2008. pp.
289-291.
B. Costanzo, L.S., Physiology, 3rd Ed., Saunders, 2006, Chapter 4, pp. 111-115,
121-123, 144-146, 148-150 .

1-introduction to cardiovascular system-2009.doc


8/28/2008

331
Contraction of Cardiac Muscle - Dr. Baumgarten

Contraction of Cardiac Muscle


Clive M. Baumgarten, Ph.D.

OBJECTIVES:

1. Describe the basis for E-C coupling and contraction


2. Describe the regulation of intracellular Ca2+
3. Describe how contractile force is modified
4. Describe how heart rate modifies contractility
5. Describe length-tension curves and their basis
6. Describe how sarcomere length modulates contraction
7. Describe how length-tension curves predict the amount of shortening
8. Describe the force-velocity relationship

I. BASIS OF CONTRACTION

Basis for contraction in cardiac and skeletal muscle is similar but not identical. Action
potential duration is much longer in heart, but the duration of contraction is comparable.

A. Comparison of Cardiac and Skeletal Muscle Structure


1. Sarcomere structure is similar to skeletal muscle.
a. Thin filaments: f-actin, troponin, tropomyosin; 1.0 :m in length.
b. Thick filaments: myosin; 1.6 :m in length.
c. Titin: Connects Z-line to actin and actin to myosin
(1)

332
Contraction of Cardiac Muscle - Dr. Baumgarten

2. Fewer myofibrils in cardiac muscle. Force per cross sectional area is less.
3. Cardiac sarcoplasmic reticulum (SR) has a smaller volume than skeletal SR.
But cardiac muscle has more mitochondria (~25-30% of cell volume).
4. Transverse tubules (T-tubules)
a. At each Z-line in ventricle and are much wider than in skeletal muscle
b. Sparse in atria and absent in Purkinje fibers.
5. Junction of SR and sarcolemma differs
in heart and skeletal muscle.
a. Heart: Cisterna of SR form diads
with limited SR–T-tubule contact
b. Skeletal muscle: SR forms triads that
envelop T-tubules
6. Macroscopically, cardiac fibers are interwoven, whereas skeletal fibers run in
parallel. Cardiac muscle fibers form overlapping, helically arranged, sheets
that arise from the connective tissue surrounding the valves at the base of the
heart. At mid-myocardium, fibers run circumferentially, and their orientation
tilts ±45° as one moves towards both endocardial and epicardial surfaces.

B. Excitation-Contraction Coupling

1. Action potential spreads along the surface membrane and invades the T-
tubules which are located at the Z-line in mammalian cardiac muscle.
2. Ca2+ entry during the plateau of the action potential as ICa, the Ca2+
current. Ca2+ channels also are termed dihydropyridine (DHP) receptors
(DHPs are a family of Ca2+ channel blockers).
3. Ca2+-induced Ca2+ release. Entry of a small amount of Ca2+ is the trigger for
release of a large amount of Ca2+ from the sarcoplasmic reticulum (SR). i.e.,
Ca2+ release amplifies Ca2+ entry. SR Ca2+ release channel is called the
ryanodine receptor (RyR). Importantly, Ca2+-induced Ca2+ release is graded
by two factors:
a. amount of Ca2+ stored in the sarcoplasmic reticulum; and
b. size of the trigger, i.e., the amount of Ca2+ entry via ICa.
4. Intracellular Ca2+ concentration ([Ca2+]i) increases.
a. Primary source of Ca2+ responsible for the increase of [Ca2+]i is the
sarcoplasmic reticulum.
b. Force is function of [Ca2+]i between ~0.1 to ~10 :M; half-max. at ~3 :M.
c. Physiologically, the maximum force is never observed because [Ca2+]i
remains too low (i.e., <10 :M) and the Ca2+ transient is too brief for Ca2+
binding to reach equilibrium.

333
Contraction of Cardiac Muscle - Dr. Baumgarten

5. Ca2+ binds to troponin-C (TnC).


a. Ca2+ binding to TnC causes tropomyosin to shift on the thin filament,
unblocking the myosin binding site and allowing weak binding of actin.
b. Weak A-M complexes undergo a cooperative conformational change,
forming strong A-M complexes (cross-bridges) that rapidly split ATP and
generate force.
c. Formation of strong A-M complexes increases the Ca2+ affinity of TnC.
6. Contraction ensues, essentially as in skeletal muscle. There is a delay
between the increase in [Ca2+]i and the development of tension. The
actomyosin is an ATPase.
7. Ca2+ reaccumulation by SR. The is reaccumulated by the sarcoplasmic
reticulum Ca2+ pump, which is an ATPase.
8. Relaxation. [Ca2+]i returns to near resting level before relaxation begins; Ca2+
unbinding from TnC ends A-M ATPase activity and force production.

C. Regulation of Intracellular Ca2+ Concentration ([Ca2+]i)

1. Requirement for steady-state: Ca2+ efflux across the cell membrane


(sarcolemma) must exactly equal Ca2+ influx (ICa). Any alteration in influx
or efflux must cause a
corresponding change in [Ca2+]i.
2. Altered [Ca2+]i leads to:
a. Parallel changes in amount of
Ca2+ in sarcoplasmic reticu-
lum stores.
b. Modulates Ca2+ influx (and to
a lesser extent, Ca2+ efflux)
until influx and efflux are
again in balance creating a
new steady state.
3. Sarcolemmal Processes
Influx
a. Ca2+ Current: ICa via L-type Ca2+ channels is the primary means of Ca2+
influx.
Efflux
a. Na+-Ca2+ Exchanger: Primary means of Ca2+ efflux. Uses energy in
Na+ gradient to extrude Ca2+ during diastole (3 Na+ in → 1 Ca2+ out).
• Na+-Ca2+ exchanger is a high capacity, low affinity transporter. It
moves lots of Ca2+ but cannot get [Ca2+]i to very low levels.
• Increasing HR causes a slight decrease of Na+ gradient, slows Ca2+
efflux via Na+-Ca2+ exchange, and thereby, increases [Ca2+]i.

334
Contraction of Cardiac Muscle - Dr. Baumgarten

b. Sarcolemmal Ca2+ Pump: A sarcolemmal Ca-ATPase directly extrudes


Ca2+ from the cell using energy from splitting of ATP. (This pump is
different than the SR Ca pump).
• Sarcolemmal Ca-ATPase is low capacity, high affinity system. It
works when [Ca2+]i is low, but pumps a small amount of Ca2+.
4. Intracellular Organelles
a. Sarcoplasmic Reticulum: A Ca-ATPase pump is a major feature of the
sarcoplasmic reticulum membrane and is capable of accumulating Ca2+.
This process uses ATP. Ca2+ also is released from the SR. Increasing
cytoplasmic Ca2+ increases the amount of Ca2+ stored in the SR, and
decreasing cytoplasmic Ca2+ decreases the amount stored.
b. Mitochondria: Mitochondria can also accumulate Ca2+. Mitochondrial
Ca2+ uptake is slow and requires high cytoplasmic Ca2+ levels.
Consequently, this process does not have a physiological role in
contraction, but it is important in regulating mitochondrial ATP
production and in pathological states.
c. Intracellular organelles can transiently effect intracellular Ca2+ but cannot
maintain steady-state Ca2+ constant in the face of a mismatch between
sarcolemmal Ca2+ influx and efflux.

D. Action Potential Duration and Contraction

1. Action potential duration (APD) can influence contractile force. Figure


illustrates effect of shorting APD experimentally by voltage clamp.
2. Reduction of APD to less than ~150 msec results in contractions with
decreased force. After several abbreviated action potentials, action potentials
with a normal duration initially will give contractions with decreased force.
3. Decreasing APD reduces Ca2+ entry via ICa. Reduced ICa provides
a. smaller trigger for Ca2+ release from the sarcoplasmic reticulum
b. smaller Ca2+ influx, and thus, less Ca2+ is available to fill SR Ca2+ stores

Effect of experimentally decreasing


APD decreased by voltage clamp
0: Control APD and contraction
1: Shortening of APD without
change in force
2-4: Further shortening of APD with
graded decrease in force.

335
Contraction of Cardiac Muscle - Dr. Baumgarten

II. MECHANISMS MODULATING CONTRACTILE FORCE

A. No recruitment. The heart is electrical syncitium. Every fiber is activated.

B. No tetanic contraction. Action potential and contraction end at nearly the same
time. Refractory period prevents stimulation at high enough rate.

C. Length of muscle. Length-dependent changes in contractile force (muscle


mechanics) are observed.

D. Contractility. The intrinsic ability of the muscle to generate force independent of


muscle length. An increase in contractility is associated with increased peak
tension and increased rate of tension development (dT/dt) and is termed a
positive inotropic event. Decreased contractility is a negative inotropic event.

E. Contractile force is proportional to the number of cross-bridges that cycle,


splitting ATP. Number of cycling cross-bridges depends on:

1. number of strong actin-myosin complexes, a function of length;


↑sarcomere length → ↑strong actin-myosin complexes
2. fraction of troponin-C (TnC) bound to Ca2+, which depends on [Ca2+]i and
TnC Ca2+ affinity.

III. Mechanisms that alter contractility

A. Altered amount of Ca2+ released by sarcoplasmic reticulum.

1. size of trigger - magnitude of ICa


2. amount of Ca2+ in sarcoplasmic reticulum;
release is roughly proportional to amount stored.

B. Amount of Ca2+ in sarcoplasmic reticulum depends on:

1. rate of uptake by SR from cytoplasm.


2. rate of influx into cell via ICa.
3. rate of extrusion from cell via Na+-Ca2+ exchange.

336
Contraction of Cardiac Muscle - Dr. Baumgarten

IV. Factors that alter contractility

A. Autonomic nervous system


1. Sympathetics − (norepinephrine) − positive inotropy.
2. Parasympathetics − (acetylcholine) − negative inotropy.
B. Pharmacologic agents and ion concentrations in blood (e.g., serum Ca2+).
C. Heart Rate − Contractility is increased by elevating heart rate either by electrical
stimulation or physiologically by sympathetic stimulation.

V. Heart Rate. Heart rate affects Ca2+ homeostasis. The effect of increasing HR is
discussed below. Analogous arguments can be made for decreasing HR.

A. Increased HR increases the amount of Ca2+ that enters the cell per unit time
because ICa is activated more often. Physiologically, HR is increased by
norepinephrine (NE) which also directly increases ICa per beat. (Quantitatively,
Ca2+ entry is the integral of ICa/beat times HR. As HR increases, the gain in Ca2+
influx due to activating ICa more frequently and NE-induced stimulation of ICa far
outweighs the decrease in Ca2+ influx per beat due to decreased APD.)
B. Increased Ca2+ entry (IICa × HR) results in an increase in [Ca2+]i.
C. Increased [Ca2+]i results in greater Ca2+ efflux via Na+-Ca2+ exchange and the
sarcolemmal Ca2+ pump so that efflux equals the new higher influx. A new
steady-state is achieved.
D. Increased [Ca2+]i causes greater accumulation of Ca2+ by the SR.
E. When HR is increased by norepinephrine, the duration of contraction decreases.
Norepinephrine increases the rate of SR Ca2+ uptake by phosphorylation of
phospholamban and decreases the Ca2+ affinity of troponin C, allowing faster
reaccumulation of Ca2+.

337
Contraction of Cardiac Muscle - Dr. Baumgarten

VI. Examples of Effect of Heart Rate on Contractility

A. Ascending Staircase. The force-frequency relationship describes the steady-


state force generated at different stimulation frequencies. Several contractions are
necessary to obtain a new steady-state. This phenomenon is also termed positive
(Bowditch) staircase or Treppe. The force of the first beat on changing HR is
anomalous. This is due to the time required for Ca2+ accumulated by the SR to be
in position for release.

B. Rest potentiation. After a longer than normal interval between beats, the first
beat will exhibit greater than normal (potentiated) contractile force.

C. Post-extrasystolic potentiation. An early extrasystole (e.g., a beat interspaced


between regular beats) develops reduced contractile force. However, the
following regular beat exhibits greater than normal (potentiated) contractile force.

338
Contraction of Cardiac Muscle - Dr. Baumgarten

III. MECHANICS OF PAPILLARY MUSCLE

Mechanics considers the length-dependence of contractile force. Remember,


length and contractility are independent mechanisms for setting contractile force.
The basis for length-dependent changes in contractile force is that sarcomere
length sets the maximum number of cross bridges that can form if Ca2+ binds. (In
contrast, the actual level of [Ca2+]i during excitation, i.e., contractility, determines
what fraction of the maximum number of binding sites produces force.)

A. Length-Tension Relationships

1. Resting tension is developed passively by applying a preload. As preload


increases, the muscle elongates. Cardiac muscle has a greater stiffness (lower
compliance) than skeletal muscle in the resting state.
2. Total tension is measured during an isometric contraction. Tension is
dependent on muscle length at onset of contraction and, therefore, on preload.
3. Active tension (also termed developed tension or twitch tension) is equal to
total tension minus resting tension. The length at which maximum developed
tension is generated is Lmax.

4. Ascending Limb of Active Length-Tension Curve. In contrast to skeletal


muscle, there is a significant resting tension at Lmax in heart.

339
Contraction of Cardiac Muscle - Dr. Baumgarten

5. To provide the maximum possible developed tension, a substantial significant


preload is required to elongate the muscle.
6. Resting tension increases more rapidly with increasing muscle length in heart
than in skeletal muscle. This is equivalent to saying cardiac muscle is stiffer
than skeletal muscle. Resting stiffness depends in large part on titin, an
elastic protein that runs from z-line to the M-line (mid-point of sarcomere).
Different titin isoforms − with different elasticity − are found in cardiac and
skeletal muscle; this explains difference in resting stiffness.
7. Physiologically, these factors force cardiac muscle to operate on the
ascending limb of the active length-tension curve.

B. Developed tension as a function of resting sarcomere length

1. Maximum number of interac-


tion between thick and thin
filaments is at Lmax, a
sarcomere length of 2.2 :m.
2. Working length (i.e., at end
of diastole) is 1.9-2.0 :m.
3. Force falls to near 0 at 1.6
:m, the length of the thick
filament.
4. Tension is proportional to the
number of cross-bridges (i.e.,
strong A-M complex).
5. Factors responsible for the
length-dependence of twitch
(developed) tension:

a. Filaments overlap sets number of cross-bridges that can be made. (There


are a greater number of cross bridges near working length and Lmax than at
very short or very long lengths.)
b. Increasing sarcomere length favors formation of strong actomyosin
complexes (working cross-bridges) via positive cooperativity and, thereby,
increases the Ca2+ affinity of TnC (contributes to ascending limb).
c. Increasing sarcomere length increases SR Ca2+ release (ascending limb).

340
Contraction of Cardiac Muscle - Dr. Baumgarten

C. Contractions with Afterload

1. Length-tension relationships predict the amount of shortening that occurs


when a muscle contracts with an afterload.
2. Experimental design: the length of the muscle is adjusted with a preload.
Upon stimulation, the muscle contracts isometrically until enough force is
generated to lift the afterload off its support (A to B). Then, the muscle
shortens isotonically (B to C).

341
Contraction of Cardiac Muscle - Dr. Baumgarten

3. The amount of shortening depends upon:


a. Initial length or Preload
• ↑ preload → ↑ shortening
• ↓ preload → ↓ shortening
b. Afterload
• ↑ afterload → ↓ shortening
• ↓ afterload → ↑ shortening
c. Contractility
• ↑ contractility → ↑ shortening
• ↓ contractility → ↓ shortening

D. Force-Velocity Relationship

1. As in skeletal muscle, the velocity of shortening depends on the load.


Maximum velocity of shortening (Vmax) occurs under 0 load, and the velocity
is 0 when the load is equal to the isometric tension (Po) developed.
2. The force-velocity relationship is shifted upward and to the right by:
a. increased initial length
b. increased contractility
c. increased heart rate

IV. REFERENCES

A. Koeppen, B.M. and Stanton, B.A. Berne & Levy Physiology, 6th Ed., 2008.
pp.261-265, 317-321.
B. Costanzo, L.S. Physiology, 3rd Ed., Saunders, 2006. Chapter 4, pp. 137-142.
Review Chapter 1, pp. 32-38.
C. Katz, A.M. Physiology of the Heart, 3rd edition, Raven, New York, 1998,
Chapters 7-11, 13-14. Readable. Designed for medical students.

2-contraction of cardiac muscle-2009.doc 8/28/2008

342
Cardiac Function Curve - Dr. Baumgarten

Cardiac Function Curve


Clive M. Baumgarten, Ph.D.

OBJECTIVES:

1. Describe the cardiac function curve (CFC)


2. Describe he relationships between (a) length and volume and (b) tension and
pressure
3. Identify diastolic and systolic pressure-volume curves
4. Construct a pressure-volume loop and describe the determinants of stroke volume
5. Describe heterometric compensation for changes in afterload and contractility
6. Generate the CFC from changes in right atrial pressure
7. Describe the factors that modulate the CFC

I. DEFINITIONS

A. Systole: Active contraction of the heart. About 1/3 of the cardiac cycle.

B. Diastole: Relaxation and refilling of chambers.

C. Heart Rate (HR): Number of contractions per minute (beats/min).

D. Stroke Volume (SV): Amount of blood ejected during systole (ml/beat)

SV = EDV - ESV

1. End Diastolic Volume (EDV): Volume of blood in ventricle the instant


before systole begins.
2. End Systolic Volume (ESV): Volume of blood in ventricle at completion of
systole as marked by closure of the aortic and pulmonic valves.
3. Ejection Fraction = SV/EDV ≥ 0.55

E. Cardiac Output (CO) = SV X HR

SV typically is 1 ml/kg. For a 70 kg individual


70 ml/beat × 72 beats/min = 5,040 ml/min ≈ 5 liters/min

F. Cardiac Index (CI) = CO/body surface area

CI = 2.2 - 4.0 L/min/m2

343
Cardiac Function Curve - Dr. Baumgarten

II. CARDIAC FUNCTION CURVE

The cardiac function curve relates right atrial pressure (RAP or PRA), the factor
controlling blood flow into the heart, to cardiac output, the blood flow from the
heart. The basis for the curve is essentially a transformation of 1-dimensional length-
tension relationships into 3-dimensions. A change in length gives a change in
volume, and tension gives pressure.

A. 1-D to 3-D transformation: assuming heart is a thin-walled sphere.


1-Dimension 3-Dimensions Transformation
____________________________________________________________________________
3
Length (L) Volume (V) V = 4/3πr ; C = 2πr
3
= 4/3π(C/2π)
3
V∝C
____________________________________________________________________________

Tension (T) Pressure (P) P = 2HT/r


____________________________________________________________________________
r = radius C = circumference H = wall thickness

1. Geometry causes amplification of length changes: volume α (length)3


2. Law of Laplace relates tension and pressure. Remember that tension controls
sarcomere length.
P = 2HT / r
Tension and, therefore, sarcomere length is related to P, r and H
T P T r T 1/H
B. A more accurate representation of the heart is as an ellipsoid:
V = 4/3 × (Dap/2)(Dlat/2)(Dmax/2)

344
Cardiac Function Curve - Dr. Baumgarten

III. STROKE VOLUME

A. Determinants of End Diastolic Volume.

Atrial pressure is the preload on the intact heart. During diastole Patrial is initially
greater than Pvent. Blood flows into the ventricle until the pressures are equal
(ignoring the inertia of blood). The volume contained at this time is EDV; the
pressure is end diastolic pressure (EDP). The relationship between diastolic
pressure and diastolic volume is the Ventricular Diastolic Pressure Curve.

B. Determinants of End Systolic Volume.

Arterial (aortic) pressure is the afterload on the intact heart. Fiber shortening and
ejection of blood cannot occur until intraventricular pressure just exceeds aortic
pressure. For the right ventricle, pulmonary artery pressure is the afterload.
Diastolic pressure in left ventricle is greater than that in right ventricle.

C. Isovolumetric (Isometric) Systolic Pressure Curve

Relates maximum isovolumetric developed pressure to EDV. To obtain the


curve, the aorta is cross-clamped to prevent injection of blood and fiber
shortening. Upon excitation, the contraction is isovolumetric; pressure increases
from its diastolic value, set by EDV and EDP, to a maximum pressure.

Connecting the maximum isovolumetric developed pressures obtained at different


EDV gives the isovolumetric systolic P-V curve. This relationship also is known
as Starling's Law of the Heart or the Frank-Starling relationship. We usually
use the curves for the left heart, but analogous curves can be drawn for the right
heart. Therefore, scales for both right and left atrial pressure are shown.

345
Cardiac Function Curve - Dr. Baumgarten

D. Pressure-Volume Loop

A cycle of contraction, ejection, relaxation and refilling is termed a pressure-


volume loop. Combining the diastolic and systolic curves, allows determination
of SV (i.e., the amount of shortening) as a function of preload (EDV), afterload
(aortic pressure), and contractility.
These curves represent changes in only one variable while the others are held
constant. Physiologically, changes in one variable often cause a change in a
second.

1. Effect of preload. Initially (A), the left atrial pressure (~5 mm Hg) sets EDV
(140 ml). Upon excitation, the heart contracts isovolumetrically until (B),
when the aortic valve opens. Ejection of blood occurs, and ventricular
volume falls (B − C). Note, ventricular pressure is not really constant
during ejection because of the compliance of the aorta and the differing
rates of blood flow into and out of the aorta with time. At (C), the aortic
valve closes (ESP, ESV), and the ventricle relaxes isovolumetrically (C to D).
Intraventricular pressure (D) is now less than atrial pressure and filling begins
(D to A). If EDV is increased (A'), SV also increases (assume constant Paortic).
2. Similarly, the effect of changes in afterload and contractility on SV can be
determined from pressure-volume loops.
3. SV depends on:
• Preload (EDV or PRA)
↑ preload → ↑ SV and ↓ preload → ↓ SV
• Afterload (Paortic)
↑ afterload → ↓ SV and ↓ afterload → ↑ SV
$ Contractility
↑ contractility → ↑ SV and ↓ contractility → ↓ SV

346
Cardiac Function Curve - Dr. Baumgarten

E. Heterometric (Frank-Starling) Compensation

Pressure-volume loops and venous return combine to compensate for changes in


SV and help maintain CO. Compensation implies a return of SV towards
"normal" in response to a change in EDV, afterload or contractility.
The basis for compensation is a shift of blood volume (BV) between arterial and
venous circulation. Compensation operates over several beats and is incomplete.
(Because compensation is incomplete, physiologically HR is reflexively adjusted
to maintain CO and blood pressure.)

347
Cardiac Function Curve - Dr. Baumgarten

IV. CARDIAC FUNCTION CURVE

The cardiac function curve describes relationship between a right heart (input)
parameter (RAP) and a left heart (output) parameter (CO). To isolate effects of
RAP on CO, contractility, total peripheral resistance, and HR are held constant.
In contrast, afterload (aortic pressure) is allowed to vary as a function of flow (CO).

A. Linkage of right and left heart parameters follows from the series organization of
the system. Flow from the right ventricle (QR) is a function of RAP described by a
pressure-volume loop for the right heart. In turn, LAP is a function of QR. Finally,
LAP is a determinant of QL, i.e., CO.

B. To obtain the CFC, right atrium is attached to an infinite reservoir. The reservoir
height controls RAP. RAP is set to different values, and CO is determined at
each. At a constant HR, CO is a proportional to SV. Each pressure-volume loop
(see diagram) corresponds to a different RAP. Note, because TPR is held
constant, increased SV causes increased aortic pressure: Flow = ΔP/TPR .

348
Cardiac Function Curve - Dr. Baumgarten

Typical Values of P-V Loops (from previous Figure) for Constructing CFC Graph:
P-V Loops RAP EDV ESV SV
__________________________________________________________________
I 0 140 60 80
II 4 240 100 140
III 7 335 160 175
IV 10 510 335 175
__________________________________________________________________

Steepest part of cardiac function


curve is between RAP = 0 and 4
mm Hg; at RAP ≥7 mm Hg, the
CO reaches a plateau.

C. CO when RAP < 0. When RAP < 0, one might expect no flow into right atrium
and, consequently, a CO of 0. But, transmural pressure (PTrans = PIn - Pout) must
be considered. For practical purposes, the heart sits in the intrapleural space,
which has a negative pressure (about −4 mm Hg);. Note, pressures are measured
relative to atmospheric pressure (0 mm Hg). Thus, if RAP = −1, then PIn = −1
and PTrans = (−1) − (−4) = +3. Thus, heart will expand and fill with blood.

D. Plateau of Cardiac Function Curve. Several factors tend to limit CO as RAP


increases.

1. Increasing afterload − Increased afterload offsets increased preload (See


table above; compare EDV and SV at RAP of 7 and 10 mm Hg.)
• As SV increases at a constant TPR, aortic pressure (afterload) increases.
FLOW = ΔP/TPR
The increase in afterload tends to limit SV.
• The force-velocity relationship indicates that at high EDV, ejection of
blood is faster than at low EDV. With faster ejection of blood, Paortic
increases.
2. Pericardium − The pericardium is inelastic and limits EDV.

349
Cardiac Function Curve - Dr. Baumgarten

IV. MODULATION OF CARDIAC FUNCTION CURVE

A. Contractility:
$ ↑ contractility (positive inotropic intervention) → ↑ SV and ↑ CO.
$ ↓ contractility (negative inotropic intervention) → ↓ SV and ↓ CO.

B. Total peripheral resistance:


$ TPR → ↑ Paortic → ↓ SV and ↓ CO
$ TPR → ↓ Paortic → ↑ SV and ↑ CO

C. HR:
At a constant SV, an increase in HR
must increase CO. At HR > 100 bpm,
diastole is too brief for complete
filling, and EDV is reduced. HR × SV
peaks at ~125 bpm under the
conditions used to obtain the cardiac
function curve. Increases in contrac-
tility and vascular compensation can
give increases in CO up to HR of ~175
bpm in a healthy and fit patient.

D. Atrial contraction. During tachycard-


ia, atrial contraction ("atrial kick")
makes an important contribution to
ventricular filling (up to 20%). At the
resting HR, however, atrial contraction
has only a minor effect on EDV (5%).

350
Cardiac Function Curve - Dr. Baumgarten

E. The time for diastolic filling of ventricle is inversely proportional to HR. At HR


≤100 bpm, filling is complete. At HR > 100 bpm, filling is incomplete, and if
TPR and contractility are constant, CO falls at HR > 125 bpm.

F. Effect of Heart Rate on Cardiac Output and Stroke Volume

1. Curve 1 makes the unphysiologic assumption that SV is independent of HR,


and therefore, CO is directly proportional to HR.
2. Curve 2 considers only the effect of ventricular filling time on SV and CO.
At HR > 125/min, CO falls.
3. Curve 3 considers the intact system and includes physiologic responses
normally seen at elevated HR (increased contractility and decreased TPR).

V. REFERENCES

A. Koeppen, B.M. and Stanton, B.A. Berne & Levy Physiology, 6th Ed., 2008. pp
325-328, 376-380.
B. Costanzo, L.S. Physiology, 3rd Ed., Saunders, 2006. Chapter 4, pp. 141-146.
3-cardiac function curve-2009.doc 8/28/2008

351
Hemodynamics 1 & 2 - Dr. Pittman

Hemodynamics 1 & 2
Roland Pittman, Ph.D.

OBJECTIVES:

1. Understand the consequences of the parallel architecture of the systemic


circulatory network.
2. State the relationship among pressure difference, flow and resistance.
3. Apply the relationship between volumetric flow and the product of average blood
velocity and cross-sectional area.
4. State and apply Poiseuille’s law for blood flow in a single vessel.
5. Describe the factors that determine vascular diameter.
6. Describe the factors that determine blood viscosity.
7. Compare and contrast laminar and turbulent flow, in relation to Reynolds’
number.

SUGGESTED READING ASSIGNMENT:

R. M. Berne, B. M. Koeppen, M. N. Levy, and B. A. Stanton, Physiology, 5th Ed., St.


Louis: Mosby, pp. 265-270 and 341-354, 2004
L. S. Costanzo, Physiology, 3rd Ed., Philadelphia: W.B. Saunders, pp. 111-120, 2006

I. PRIMARY GOAL OF THE CIRCULATORY SYSTEM

A. The cardiovascular system is designed to ensure the survival of all cells of


the body at every moment or to keep the immediate chemical environment
of each cell in the body at a composition appropriate for its normal
function. The term "homeostasis" is used to denote the constancy of the
internal environment (Claude Bernard, 1866).

B. Single cells: Consider the simple case of a single cell supplied with
nutrients from the large volume of medium in which it resides. Oxygen
availability is often a limiting factor for cell survival and it is generally
supplied to a cell by passive diffusion. The critical size (radius) of a
spherical cell (for simplicity) that is just adequately supplied with oxygen
from the surrounding medium is about 1 mm.

C. Multicellular organism (human body, ~ 100x1012 cells): How to supply


with O2?

1. Bathing medium is water : Total body water = 60% of body weight


(assume 70 kg person).

352
Hemodynamics 1 & 2 - Dr. Pittman

Total body water is distributed in three compartments:

a. Intracellular = 23 liters (33% body weight)


b. Interstitial = 16 liters (22.5% b.w.)
c. Circulating plasma = 3 liters (4.5% b.w.)

2. Cells are bathed in interstitial fluid (ISF), but interstitial fluid


volume is a little more than half the intracellular fluid volume.
Thus, ISF can not be considered a large reservoir of fluid and its
composition is directly influenced by cellular metabolism.

Problem: How can the composition of ISF be maintained near its


normal value?

Solution: Continuously refresh the ISF by putting it in intimate


contact with "fresh, reconditioned" fluid (i.e., arterial blood).

The circulating blood must be brought close to the cells, since


nutrient and waste exchange takes place by passive diffusion, a
transport mechanism which is most efficient over short distances.
Thus, the cardiovascular system uses bulk flow to reduce the
effective distance between parts of an organism.

3. Two important conditions must be satisfied:

a. There must be adequate blood flow through capillaries in


the tissue.
b. The chemical composition of incoming blood must be
controlled to be that which is desired in the ISF.

4. The design and operation of the cardiovascular system fulfill these


conditions.
5. The cardiovascular system carries out three important functions:

a. Moves material (carrier is blood)


b. Moves heat (tissue metabolism generates heat that must be
brought from the body's core to its surface)
c. Transmits force (via erectile tissue)

II. DESIGN OF THE CARDIOVASCULAR SYSTEM

A. Pulmonary and systemic circulations

353
Hemodynamics 1 & 2 - Dr. Pittman

Figure 1. Schematic diagram of the parallel and series arrangement of the vessels composing
the circulatory system. The capillary beds are represented by the thin lines connecting the
arteries (on the right) with the veins (on the left). The crescent-shaped thickenings proximal
to the capillary beds represent the arterioles (resistance vessels). (Redrawn from Green HD:
In Glasser O. editor: Medical physics, vol 1, Chicago, 1944, Year Book Medical Publishers.)

Organs (tissues) are connected in parallel and the following statements are
consequences of this parallel architecture:

1. A given volume of blood ejected from the left ventricle passes through
only one organ before entering the veins.
2. Arterial blood entering each organ has the same composition.

354
Hemodynamics 1 & 2 - Dr. Pittman

3. Blood pressure at the entrance to each organ is the same.


4. Blood flow to each organ can be controlled independently (local
regulation of flow).

B. Types of tissues

1. Blood "reconditioners" - maintain ISF composition "constant." In


general, flows to these tissues exceed their metabolic needs.

a. Lung - gas exchange (oxygen, carbon dioxide)


b. Kidney - electrolyte composition, fluid balance
c. Gut - nutrient absorption
d. Skin - temperature regulation

2. "Essential" tissues: Flows to these tissues match their metabolic needs.

a. Heart - needs continuous energy supply for pumping activity


b. Brain - requires continuous nutrient supply and washout of
metabolites for clear thinking, etc.
c. Muscle - enormous blood flow requirements during exercise.

III. SOME DEFINITIONS AND RELATIONSHIPS AMONG PRESSURE,


FLOW AND RESISTANCE

A. Pressure: P = force/area

In order to make blood move, a force must be applied to it. Pressure is


force per unit area.

F = m a (Newton's 2nd law)

F = (ρ A h) g

P = F/A = ρgh

For mercury ρ = 13.6 g/cm3

g = 980 cm/sec2

If h = 0.1 cm, P = 1.33 x 103 dynes/cm2


or we say that, P = 1 mmHg = 1 Torr
Figure 2.

355
Hemodynamics 1 & 2 - Dr. Pittman

Pressure is usually measured relative to atmospheric pressure at sea level (1


atmosphere = 760 mmHg). As an example, a mean arterial pressure of 100
mmHg is equivalent to 13.6 x 100 mm of water or about 4.5 feet. That's why
penetrating an artery can be such a dramatic event!

B. Flow: Q = volume/time

Consider the situation in which we apply a force to a fluid and cause it to


move at average velocity, v. In a time interval t, a fluid moving with an
average velocity v will cover a distance x = v•t. The volume of fluid that
moves past a reference location in the tube or vessel is V = A•x = A•v•t. Since
flow, Q, is volume per unit time, Q = A•v. This relationship is true for steady
flow in a vessel of any arbitrary cross section and is a consequence of the
conservation of mass (i.e., the fluid is incompressible and flow is conserved).

Figure 3.

C. Resistance: R = pressure difference/flow

1. It is convenient for future discussions to define the term "resistance to


blood flow" in analogy with electrical resistance to current flow.

R ≡ Δ P/Q.

Blood "resists" being moved because friction develops between the


blood and the vessel wall, in addition to the friction that develops
between adjacent imaginary layers (laminae) of blood. It is this
inherent friction in the flow of blood through the vascular network that
leads to a progressive reduction in the energy contained in blood along
any pathway of blood vessels. The total energy in blood is mostly (~
90%) potential energy due to the blood pressure; kinetic energy due to
blood motion makes up the balance.

This definition is just the fluid mechanical analog of Ohm's Law: R =


V/I.

356
Hemodynamics 1 & 2 - Dr. Pittman

The analogy is made by identifying the following corresponding


quantities:

Pressure difference (ΔP) → Voltage drop (V)


Flow (Q) → Current (I)
Resistance to fluid flow (R) → Resistance to current flow (R)

2. Combining fluid mechanical resistances

There are two common ways in which resistance elements can be


combined: series and parallel. The rules for determining the effective
resistance of a given combination are simple and can be carried over
from electrical to fluid mechanical circuits.

a. Series = consecutive segments

Figure 4.

Rule: Resistances in series add

REff = R1 + R2

b. Parallel = branches

Figure 5.

Rule: Reciprocals of resistances in parallel add

1/REff = 1/R1 + 1/R2

c. Several resistances in parallel

We will have occasion in the future to discuss what happens


when there are several resistances in parallel (e.g., consider
several capillaries in a microcirculatory unit, several
microcirculatory units which comprise a vascular network in a

357
Hemodynamics 1 & 2 - Dr. Pittman

tissue, or the organs of the systemic circulation that are


connected in parallel).

Suppose there are N identical resistances, R (vessels or


vascular units), connected in parallel. The resistance of the
parallel combination, REff, is given by:

l/REff = l/R + l/R + ... +l/R (N terms) = N(l/R)

REff = R/N

Thus, increasing the number of parallel channels for blood flow


decreases the effective resistance to blood flow.

Another assertion which is not too difficult to realize is that the


effective resistance of a parallel combination is always smaller
than the smallest resistance in the combination.

IV. FLOW OF BLOOD IN CYLINDRICAL TUBES

A. Poiseuille's law for a viscous fluid

Figure 6.

Q = volume flow = π r 4ΔP/8ηl = ΔP/R

r = radius of tube (can vary in the circulatory system)

l = length of tube (generally a constant in the circulatory system)

ΔP = P1- P2 = pressure difference between the ends of the tube; also


driving pressure or perfusion pressure

η= viscosity of blood (property related to ease of flow)

358
Hemodynamics 1 & 2 - Dr. Pittman

Recall that Q = ΔP/R; thus, R = 8ηl/πr4

Figure 7. After Caro et al. 1978. Velocity profiles in steady laminar flow at the entrance to a
tube, showing the increasing width (δ) of the boundary layer, corresponding to the change
from an initially flat profile to a fully developed parabolic profile.

The radial dependence of velocity in a Newtonian fluid flowing through a


vessel or tube of circular cross section is called "parabolic" due to the
quadratic dependence of velocity on radial position, r: v(r) = v0 [1-(r/ri)2],
where ri is the internal radius of the tube and v0 is the maximum velocity
that occurs on the axis (r=0); the minimum velocity is zero at the wall
(r=ri; called the "no slip" condition).

Figure 8.

B. Influence of radius on flow

359
Hemodynamics 1 & 2 - Dr. Pittman

1. Flow is proportional to radius (or diameter) to the fourth power. Thus,


small changes in vascular caliber can produce profound changes in
blood flow through a vascular segment.
2. Small arteries and arterioles have the capability of dramatically
changing their caliber in response to neural, chemical or humoral
stimuli. Thus, these vessels can be the sites of large changes in
resistance to blood flow.
3. The next figure illustrates the profile of mean blood pressure along the
systemic vasculature from the left side of the heart to the right side.

Figure 9.

4. Note that the largest decrement in pressure occurs in the arterioles.


These vessels are collectively known as the "resistance vessels" and
regulation of blood flow takes place primarily at this site.

Since a capillary has the smallest radius of all the blood vessels, an
individual capillary has the highest resistance to blood flow (we ignore
the dependence of vessel length for this discussion). However, the
collective resistance to flow of all the capillaries is less than the
collective resistance to flow of all the arterioles. The resolution of this
apparent paradox is that there are many more capillaries than there are
arterioles, enough so that the parallel combination of arterioles has a
higher resistance than the parallel combination of capillaries. This
description can be summarized symbolically by:

rcap < rarteriole → R(single capillary) > R(single arteriole)

But, R(all capillaries) < R(all arterioles), since # capillaries » #


arterioles and R(single capillary)/# capillaries < R(single arteriole)/#

360
Hemodynamics 1 & 2 - Dr. Pittman

arterioles.

5. Blood vessel diameter is determined by the balance of distending


forces (blood pressure) and compressing forces (elastic recoil and
smooth muscle contraction).

a. The role of blood pressure is best described by considering the


Law of Laplace that describes the balance of forces at
equilibrium for a hollow, elastic cylinder. In the following
form the Law of Laplace is exact and independent of the
properties of the material making up the elastic tube.

T = P•ri/w, where T is circumferential wall tension, P is the


transmural pressure (intravascular minus extravascular
pressure; extravascular pressure is usually atmospheric and, by
convention, equal to zero), ri is the internal radius, and w is
wall thickness. The Law of Laplace is often presented in an
approximate form for a thin-walled elastic tube as T = P•ri,
where wall thickness is ignored (and, of course, T has different
units than for the exact thick-walled case).

Figure 10.

b. The passive elastic properties of a blood vessel for a given state


of vascular smooth muscle (VSM) contraction are given by the
following constitutive relation in graphic form. Altering the
level of VSM contraction (tone) shifts the T vs. ri curve to the
right (relaxation) or left (contraction) of the control curve.

361
Hemodynamics 1 & 2 - Dr. Pittman

Figure 11.

c. The equilibrium diameter of the vessel is determined by


superimposing the Law of Laplace line onto the elastic
constitutive relation. The unique intersection of the two graphs
gives the equilibrium radius and wall tension. VSM contraction
results in a constriction and VSM relaxation results in a
dilation, corresponding to our intuitive expectations.

Figure 12.

d. Note that both changes in blood pressure and changes in VSM


contractile state can produce immediate changes in vessel
diameter. The contractile state of VSM is under continuous
control by neural (i.e., sympathetic nerves), humoral (e.g.,
epinephrine) and local (myogenic and metabolic) mechanisms.

C. Influence of viscosity on flow - rheological properties of blood (R η)

Viscosity is a property of fluids which characterizes how easily a fluid will


flow. The higher the viscosity, the greater is the resistance to flow.

Viscosity can be thought of as a measure of the "resistance" to a deforming


force. A sustained force is needed to maintain fluid motion.

For most fluids (e.g., blood plasma) viscosity is independent of fluid velocity;
these fluids are called "Newtonian" fluids.

362
Hemodynamics 1 & 2 - Dr. Pittman

Whole blood, however, is a suspension of red cells in plasma and the viscosity
of blood does depend on fluid velocity. This behavior is termed "non-
Newtonian."

There are four major factors that determine the viscosity of blood. They are:

1. Temperature - Increases in temperature, such as occur in exercise or


fever, produce decreases in viscosity. Conversely, decreases in
temperature, such as occur in hypothermia, produce increases in
viscosity. Thus, changes in temperature produce opposite changes in
viscosity.
2. Hematocrit (Hct) - Hematocrit is the fractional volume of blood
occupied by red cells (i.e., the concentration of red cells). Viscosity
depends upon hematocrit according to the following graph. Hematocrit
can vary with the physiological state of the body (e.g., anemia,
polycythemia).

Figure 13. Viscosity of whole blood, relative to that of plasma, increases at a progressively
greater rate as the hematocrit ratio increases. For any given hematocrit ratio the apparent
viscosity of blood is less when measured in a biological viscometer (such as the hind leg of a
dog) than in a conventional capillary tube viscometer. (Redrawn from Levy MN, Share L:
Circ Res 1:247, 1953.)

In the capillary tube viscometer, one can determine apparent viscosity


by solving Poiseuille's equation for η:

η = π r4 ΔP/8 Q l

The value of η computed in this way is referred to as "apparent"

363
Hemodynamics 1 & 2 - Dr. Pittman

viscosity because Poiseuille's equation assumes that viscosity is


independent of velocity, whereas blood is a non-Newtonian fluid that
violates this assumption.

It is convenient to express the viscosity of blood relative to that of


plasma, a Newtonian fluid. From the equation above, viscosity is
inversely proportional to flow, all other things (i.e., r, l and ΔP) being
equal. Thus, we can define relative viscosity of blood at a given
hematocrit as:

ηrel = η(blood)/η(plasma) = Q(plasma)/Q(blood)

In the figure above, why is the in vivo (hind leg) viscosity consistently
lower than the in vitro (glass tube) value for all hematocrits? One
obvious difference is that the glass tube has a constant diameter,
whereas the hind limb is a complex branching network composed of
blood vessels of a wide range of diameters. This suggests that one
might look at diameter as a possible cause for the difference.

3. Tube diameter – As illustrated in Fig. 14, the relative viscosity of blood decreases
when tube (or vessel) diameter decreases below about 300 µm (corresponding to
the in vivo microcirculation).

Figure 14. Viscosity of blood, relative to that of water, increases as a function of tube
diameter up to a diameter of about 0.3 mm. (Redrawn from Fahraeus R. Lindqvist T: Am J
Physiol 96:562, 1931.)

364
Hemodynamics 1 & 2 - Dr. Pittman

The explanation for this phenomenon requires the recognition of several features of blood
flow discussed earlier. Red blood cells are quite flexible and this property causes them to
migrate away from the wall and toward the axis of the vessel due to geometric and fluid
dynamic effects. This migration creates a cell-poor layer (almost all plasma) next to the
tube or vessel wall that is about 4 μm thick. It is always present, but only becomes
noticeable in vessels smaller than about 300 μm in diameter. The resulting core region of
RBCs and plasma has a high concentration of RBCs (i.e., hematocrit). Fluid velocity is
higher near the axis of the vessel (recall the parabolic velocity profile), so that average
red cell velocity will be greater than average plasma (and blood) velocity since RBCs are
preferentially in the core region. A look at what is going on inside a small tube (see Fig.
15 from Barbee and Cokelet’s work) reveals that the hematocrit (i.e., RBC concentration)
inside the tube is smaller than what would be expected if the RBCs were distributed
uniformly across the lumen. Since viscosity depends on hematocrit, the lower hematocrit
present in small tubes or vessels will result in a lower viscosity.

Figure 15. The "relative hematocrit ratio" of blood flowing from a feed reservoir through
capillary tubes of various calibers as a function of the tube diameter. The relative hematocrit
is the ratio of the hematocrit of the blood in the tubes to that of the blood in the feed
reservoir. (Redrawn from Barbee JH, Cokelet GR: Microvasc Res 3:6, 1971.)

4. Shear rate is the relative velocity of one imaginary layer of fluid with respect to
that of the adjacent layers. In symbolic terms, shear rate is the negative of the
derivative of velocity with respect to radial position or dv/dr. If we use the
parabolic velocity profile for a viscous Newtonian fluid (v = v0 [1-r2/ri2]), shear
rate (γ) is:

γ = -dv/dr = 2v0r/ri2

Thus, shear rate is zero at the center of the vessel (r = 0) and attains a maximum
value of 2v0/ri at the wall (r = ri). Since mean (average) velocity is equal to half
the maximum velocity and vessel diameter is twice the radius, maximum shear
rate is often expressed as:

γmax = -dv/dr | max = 8vavg/d

It is also useful to express maximum shear rate in terms of blood flow, Q. This

365
Hemodynamics 1 & 2 - Dr. Pittman

can be done by noting that


Q = A•vavg = πri2•vavg. Thus shear rate is given by:

γmax = -dv/dr | max = 4Q/πri3

As the maximum shear rate (and flow) increases, the viscosity of blood decreases.
This is due to the breakup of aggregates of red cells which form at low flows (or
shear rates). (Red cells have a natural tendency to stick together.) Also, the
tendency for RBC aggregation is greatest at the center of a vessel where the shear
rate is lowest.

The following figure illustrates this effect (The shear rate referred to on the
horizontal axis should be interpreted as the maximum shear rate, since shear rate
varies across the vessel lumen from zero to the maximum value at the wall. Also,
the viscosity should be interpreted as the cross-sectional average viscosity since
that also varies over the cross-section).

Figure 16. Decrease in the viscosity of blood (centipoise) at increasing rates of shear. The
shear rate refers to the velocity of one layer of fluid relative to that of the adjacent layers and
is directionally related to the rate of flow. (Redrawn from Amin TM, Sirs JA: Q J Exp
Physiol 70:37, 1985.)

Because shear rate is highest at the wall of a vessel, there will be a large viscous drag on
the endothelial surface, trying to pull it along the direction of flow. This continuous shear
stress on the endothelium often results in damage to the intimal surface of a vessel.

366
Hemodynamics 1 & 2 - Dr. Pittman

Acute, transient increases in shear rate lead to vasodilation due to the shear-induced
release of a vasodilator (nitric oxide) from the endothelium. Chronic increases in shear
rate lead to remodelling of the vascular network (i.e., cell division) and increased vessel
diameter. These physiological responses appear to be part of a feedback system that
attempts to maintain a fairly constant shear at the vessel wall.

In low flow states (e.g., circulatory shock), low shear rates lead to increased viscosity that
tend to reinforce the low flow (positive feedback), resulting in a vicious cycle.

D. Laminar vs. turbulent flow (qualitative character of blood flow)

1. Laminar flow

a. ΔP = R•Q (linear dependence of pressure drop on flow)


b. Smooth, axial flow is usually present in the circulation.
c. Laminar flow is generally associated with smooth surfaces (i.e.,
endothelium).

Figure 17. In laminar flow of all elements of the fluid move in streamlines that are parallel to
the axis of the tube; movement does not occur in a radial or circumferential direction. The
layer of fluid in contact with the wall is motionless; the fluid that moves along the axis of the
tube has the maximal velocity.

367
Hemodynamics 1 & 2 - Dr. Pittman

2. Turbulent flow

a. ΔP = aQ + bQ2 (additional quadratic dependence of pressure drop on


flow). This implies that one needs a higher driving pressure to obtain a
given flow; thus, the energy cost is greater to achieve a given flow when
turbulence is present.
b. Irregular motion is usually present with radial and circumferential
components of velocity.
c. Turbulent flow is generally associated with rough surfaces (e.g., damaged
endothelium), bifurcations (branches, especially in large vessels with high
blood velocity), and valves (i.e., between heart chambers).

Figure 18. In turbulent flow the elements of the fluid move irregularly in axial, radial, and
circumferential directions. Vortices frequently develop.

3. Reynolds number (1883): Quantify the distinction between laminar and turbulent
flow

a. NR = ρdv/η, where ρ is fluid density (usually constant for blood), d is


diameter, v is mean velocity and η is viscosity. NR is a dimensionless
number.
b. NR is the ratio of inertial forces to viscous forces
c. For values of the Reynolds number above about 2,000 flow becomes
unstable and turbulence can develop (i.e., there is a tendency for turbulent
flow to develop, but it does not necessarily occur). Conditions that
promote turbulent flow are large diameter, high velocity and low viscosity
(as occurs in anemia). Endothelial damage becomes more likely in regions
of turbulent flow. A consequence of endothelial damage is the release of
more thromboxanes and less prostacylin, a condition that will promote

368
Hemodynamics 1 & 2 - Dr. Pittman

platelet aggregation (thrombus formation).

STUDY QUESTIONS

1. Consider two identical vessels, A and B, that converge to form vessel C. If vessel
C has twice the cross-sectional area of vessel B, then as blood flows from vessels
A and B into vessel C, the blood velocity in C:

A. Becomes twice as great as it was in A.


B. Remains the same as it was in A.
C. Decreases to 1/2 what it was in A.
D. Decreases to 1/4 what it was in A.
E. Decreases to 1/8 what it was in A.

ANSWER: B

You need to realize three things: (1) flow in A and B are the same; (2) flow in C is
equal to the sum of the flows in A and B; and (3) flow equals cross sectional area
times average velocity (Q = A x v).

QA = QB

QA + QB = QC

2 x (AA x vA) = AC x vC ; since AC = 2 x AB , vC = vA

2. If the blood viscosity, length, and radius of a blood vessel are simultaneously
doubled, how would the resistance to blood flow change?

A. It would increase by a factor of 2.


B. It would decrease by a factor of 4.
C. It would increase by a factor of 4.
D. It would decrease by a factor of 16.
E. It would increase by a factor of 16.

ANSWER: B

One needs to know that resistance to blood flow is proportional to viscosity and
length, and inversely proportional to radius to the fourth power. Thus the new
resistance will be 2 x 2 / 24 = ¼ times the original resistance.

3. The viscosity of blood in the in vivo microcirculation is increased by increases in


which of the following?

369
Hemodynamics 1 & 2 - Dr. Pittman

1. Hematocrit
2. Shear rate
3. Vessel diameter
4. Temperature

ANSWER: B (1 & 3)

You just need to remember that increases in hematocrit and vessel diameter (below
about 300 :m) lead to increases in viscosity, whereas the opposite is true about
shear rate and temperature.

4. Which of the following is a consequence of the parallel architecture of the


cardiovascular system?

A. Cardiac output is less than the sum of blood flows through all the organs.
B. The composition of blood entering each organ is the same.
C. The arterial pressure at the inflow of each organ is proportional to the size of
the organ.
D. Each organ always receives the same fixed fraction of the cardiac output.
E. A given volume of blood ejected from the left ventricle passes through more
than one organ.

ANSWER: B

You should remember that the stroke volume of the left ventricle is distributed to all
organs (blood only passes through one organ) in inverse proportion to their
respective resistances to blood flow. The composition of the blood entering each
organ is the same, as is the arterial pressure at the inflow to each organ (there is
very little resistance to blood flow in the large arteries, hence only a small
decrement in pressure).

5. Consider a segment of aorta that has a constricted region. The diameter of the
normal region is twice that of the constricted region. Which of the following
statements is TRUE about the normal and constricted regions?

A. Circumferential wall tension is greater in the constricted region.


B. Volumetric flow is smaller through the constricted region.
C. Average blood velocity is greater in the constricted region.
D. Cross-sectional area is greater in the constricted region.
E. Shear rate at the wall is greater in the normal region.

ANSWER: C

Circumferential wall tension is expressed by the Law of Laplace (T = Pr/w). Since P

370
Hemodynamics 1 & 2 - Dr. Pittman

is relatively constant in this segment of a large vessel (resistance and pressure drop
are small), T will depend mainly on r. Thus, T is smaller in the constricted region.
Since volumetric flow is the same at every point along the segment (conservation of
flow) and cross-sectional area is smaller in the constricted region, the average
velocity of blood is greater in the constricted region (Q = A v). Wall shear rate is
proportional to the average velocity divided by the internal radius so that it is
greater in the constricted region.

371
Cardiac Oxygen Consumption - Dr. Baumgarten

Cardiac Oxygen Consumption and the Working Heart


Clive M. Baumgarten, Ph.D.

OBJECTIVES:
1. Describe the factors that influence myocardial oxygen consumption
2. Describe measurement of oxygen consumption and cardiac output using the Fick
principle
3. Describe the measurement of cardiac work and how cardiac work influences
oxygen consumption
4. Describe the application of the above to therapies for angina pectoris

I. MYOCARDIAL OXYGEN CONSUMPTION ( M V O 2 )

A. The rate of oxygen consumption per minute ( M V O2• ) is a measure of the total
energy production and expenditure of the heart. M V O2 is proportional to both
the amount of ATP synthesized and the amount utilization.
B. Metabolic substrates. Cardiac muscle is an omnivore; it can utilize fatty acids,
carbohydrates (glucose, pyruvate, lactate), amino acids, and ketones. Normally,
fatty acids are the primary source of energy. After a carbohydrate meal or
during high demand, the fraction of energy derived from carbohydrates increases.
C. Substrates have nearly the same enthalpies when expressed per liter of O2
consumed, ~4.8 kcal/L O2. This means that nearly equal amounts of ATP are
produced per liter of O2 consumed independent of the substrate.
Substrate % Oxygen Used Enthalpy
Fatty Acids 60 − 65% ~9 kcal/g ≈ 4.7 kcal/L O2
Carbohydrates 30% ~4 kcal/g ≈ 5.0 kcal/L O2
Amino Acids 5 − 10% ~9 kcal/g ≈ 4.7 kcal/L O2

D. ATP Use.
1. Contraction − actomyosin ATPase
2. Relaxation − sarcoplasmic reticulum ATPase
3. Basal processes − Na+-K+ pump, protein synthesis, phosphorylation, etc.
E. ATP Storage.
creatine + ATP ↔ creatine phosphate + ADP
Enzyme: creatine phosphokinase (CPK)
F. In the steady-state, energy production

equals utilization. O2 consumption is
proportional to energy production.

M V•O 2 is a measure of both energy utilization
and production. Basal M V O 2 and M V O 2 of the beating heart are:

Basal M V O2 = 2 mL/min/100 g of heart

Beating Heart M V O2 = 8-10 mL/min/100 g of heart

372
Cardiac Oxygen Consumption - Dr. Baumgarten

II. FICK PRINCIPLE − Measurement of O2 Consumption or Cardiac Output



A. Measurement of O2 Consumption. M V O2 of an organ can be measured in situ
utilizing the Fick Principle. The basis for this method is the law of conservation
of mass, in this• case O2. The amount of oxygen consumed by an organ per unit
time (e.g., M V O2 ) must equal the difference between the amount entering the
organ via the arterial supply and the amount leaving via the veins.
1. Definitions:
Q(organ) = blood flow (mL/min/100g). Note that: QA = QV
[O2] = mL O2/mL (i.e., O2 conc.)


B. Fick Equation ( M V O2 and Q are expressed per 100 g of tissue):

M V O 2 (organ) = {QA × [O2]A} − {QV × [O2]V}
= Q × [{[O2]A - [O2]V}

M V O 2 (organ) = flow × AV O2 difference

units for M V O2 : mL O2/min/100 g =
mL/min/100 g × mL O2/ml

For the heart (300 g):


Q = QA = QV = 180 mL/min/300 g
[O2]A = 0.20 mL O2/ml & [O2]V = 0.05 mL O2/mL

M V O 2 (organ) = Q × {[O2]A − [O2]V}
= 180 mL/min/300 g × 0.15 mL O2/mL

M V O2 = 27 mL O2/min/300 g = 9 mL O2/min/100 g
• For the heart, Q is blood flow in the coronary arteries, [O2]A is
measured in the aorta, and [O2]V in the coronary sinus. Coronary flow
can be measured with flow probes or a tracer (e.g., 131Xe); thermodilution,
the most commonly used technique, measures left ventricular coronary
flow rather than total coronary flow.
• For other organs, the values for the respective artery and vein are used.

373
Cardiac Oxygen Consumption - Dr. Baumgarten

C. Measurement of cardiac output.

The Fick Principle also can be used


to calculate cardiac output.
Taking advantage of the fact that
blood flow (Q) through the lungs
must equal cardiac output and the
law of conservation of mass:

QPV × [O2]PV = QPA × {[O2]PA + qtotal}


qtotal = O2 consumption (mL O2/min)
CO = Q = QPV = QPA
CO = qtotal/([O2]PV − [O2]PA)

III. CARDIAC WORK

Work is force × distance. For the heart, the force is pressure, and the distance is CO.
A. Stroke Work = P × SV; more precisely, SW = ∫P dV
B. Minute Work = P × SV × HR = P × CO
C. Work (of left ventricle) is
measured as the area within the
pressure-volume loop (A) and is
called Pressure-Volume Work.
D. Work also is done by the inertia
of venous return and the left
atria (B, area between the P-V
loop and the baseline).
E. Kinetic work done in accelerat-
ing blood is a minor component
of LV work and is ignored in this
diagram.
2
Wkinetic = ½ mv
[m = mass of blood; v = velocity]

374
Cardiac Oxygen Consumption - Dr. Baumgarten


IV. DETERMINANTS OF M V O 2

A. Tension is directly related to actomyosin crossbridge formation and cycling, and


therefore,

to ATP utilization. An increase in tension causes an increase in
M V O2 .
1. Afterload is related to tension by the Law of Laplace.
2. Radius is related to tension by the Law of Laplace.

B. Pressure versus Flow (Volume) Work. The work done •


to increase pressure
within the ventricle is far more costly in terms of M V O2 than work done to
increase stroke volume. Pressure and flow work are sometimes called internal
and external work, respectively.

Effect of increasing work from a to b •by increasing pressure or increasing


flow. Increasing pressure increases M V O 2 more than increasing CO.

$ Increase Pressure at constant CO → Large increase in M V O2

$ Increase CO at constant Pressure → Small increase in M V O2

C. Efficiency
Efficiency = P-V work/energy equivalent of O2 consumed.
Typically, cardiac efficiency is 5 − 20%. The rest of the energy appears as heat.

Because altering pressure and CO have different effects on M V O2 , increasing
pressure or CO from a to b must have different consequences for efficiency.
$ Increase Pressure at constant CO → Efficiency decrease
$ Increase CO at constant Pressure → Efficiency increases

D. Contractility. Increased contractility augments both developed tension and the


maximum velocity of shortening.
1. A faster turnover of crossbridges increases ATP consumption.
2. Increased [Ca2+]i requires the sarcoplasmic reticulum to reaccumulate more
Ca2+, using ATP.
3. The efficiency of the heart decreases.

375
Cardiac Oxygen Consumption - Dr. Baumgarten

E. Heart Rate. The effects of heart rate largely result from its effects on
contractility and tension.
1. Tension. At a higher heart rate, the heart is developing tension more
frequently. The Time-Tension Index (TTI) measures the area under the
ejection phase of the ventricular pressure curve per minute (or sometimes• per
beat). TTI is easy to measure clinically and is roughly proportional to M V O2 .
However, this is not always the case. For example, norepinephrine increases
contractility but shortens the duration of contraction and, thus, the duration of
ejection.

Consequently, norepinephrine can decrease the TTI while increasing
M V O2 .

2. Contractility. Increased heart rate itself results in increased contractility. In


addition, heart rate physiologically is increased by norepinephrine released
from sympathetic nerves. Norepinephrine causes an increase in contractility,
even at a constant heart rate.

V. CLINICAL APPLICATION OF M V O 2

A. Atherosclerosis of the coronary arteries exhibits lipid deposits/calcification in the


intima, narrowing of the internal diameter, and reduction in blood flow. The
capacity to deliver oxygen to the heart is reduced, and during periods of exercise
or stress, cardiac oxygen demand exceeds supply. The patient experiences chest
pain, angina pectoris, because of the myocardial hypoxia. The two approaches to
this problem are:
1. Increase coronary blood flow
2. Decrease oxygen demand by:
• reducing developed tension
• reducing heart rate, and/or
• reducing contractility.

376
Cardiac Oxygen Consumption - Dr. Baumgarten

B. Therapeutic approaches

1. Propranolol is a β adrenergic blocker that inhibits the positive inotropic and


chronotropic effects of the sympathetic nervous system. It also has a direct
negative inotropic effect at higher doses.
Propranolol decreases O2 demand by decreasing (a) developed tension,
(b) heart rate, and (c) contractility.
2. Nitroglycerin is a vasodilator. However, an increase in coronary flow is not
the principal mechanism of action. Vasodilatation reduces arterial pressure
and, consequently, afterload. Pooling of blood in the veins reduces venous
return, end diastolic volume, and radius. Both effects decrease tension during
ejection of blood.
Nitroglycerin decreases O2 demand by decreasing developed tension.
3. Digitalis is a positive inotropic agent. At first, such drugs would appear to be
contraindicated. However, patients with angina often have low contractility
(due to hypoxia), cardiac dilatation, and compensatory increases in heart rate.
Digitalis increases SV and, consequently, decreases end diastolic volume and
radius. As SV improves, heart rate tends to slow.

Digitalis decreases oxygen demand by decreasing (a) developed tension •


and
(b) heart rate. Note.

there is a tradeoff between this decrease in M V O 2 and

the increase in M V O2 associated with increased •contractility. In the normal


patient, the increased contractility will increas M V O2 .

VI. REFERENCES

A. Koeppen, B.M. and Stanton, B.A. Berne & Levy Physiology, 6th Ed., 2008. pp
417-421, 318-321.
B. Costanzo, L.S. Physiology, 3rd, Saunders, 2006. Chapter 4, pp. 146-147.

4-cardiac oxygen consumption-2009.doc 8/28/2008

377
Control of Cardiac Output 1 - Dr. Ford

Control of Cardiac Output 1


George D. Ford, Ph.D.

OBJECTIVES:

1. Discuss the consequences of having a closed circulatory system.


2. Discuss how the right heart and left heart are coupled physiologically.
3. Know the three major factors governing venous return to the heart. Be able to
show these graphically as well as how variations in each factor affect this
graphical representation. Specifically:

a. Describe the role that the ratio of arterial to venous compliance and total
peripheral resistance plays in the graphical description of the venous
return.
b. Discuss the notions of mean systems pressure and stressed and unstressed
volume and the effect variations in these parameters have on the graphical
representation above.

4. Graphically display the relationship between venous return and cardiac output.
5. Translate the concepts discussed into clinically observable parameters.
6. Discuss at least three other factors which could influence venous return but are
not easily assimilated into the graphical analysis used for the other concepts
above.

I. OVERVIEW

We will begin with a discussion of the basic problem, the need for steady state
flow to be equal everywhere in an enclosed system, i.e. cardiac output and venous
return must be equal. We will then develop a relationship between venous return
and right atrial pressure in terms of three parameters; total peripheral resistance,
the ratio of arterial to venous compliance, and the mean systems pressure. This
latter is a new variable which depends largely on total blood volume. This
relationship is known as the vascular function curve. Next we will discuss how
the system flow is determined by the intersection of the vascular function and
cardiac function curves. We will then see how perturbations in these parameters
affect the function curves and thus the cardiac output or system flow. Finally we
will look at some factors influencing venous return that don’t readily fit in the
Guyton analytical approach.

II. IN A STEADY-STATE, CARDIAC OUTPUT MUST EQUAL VENOUS


RETURN

This statement, which is true for the cardiovascular system when the heart is
connected to the vasculature as shown in Fig. 1, has created a theoretical
conundrum for physiologist since the year 1628 when William Harvey published
his observations on the circulation of the blood.

378
Control of Cardiac Output 1 - Dr. Ford

Figure 1. Conceptual model of the cardiovascular systems showing the right atrium (RA) connected
to the right ventricle (RV), pulmonary vessels, left atrium (LA), left ventricle (LV), arteries, arterioles
and capillaries, and veins. The structures in this figure are connected in series. Cardiac output in liters
per minute, is defined as flow from the right atrium through the left ventricle. Venous return in liters
per minute, is defined as flow from the aorta to the right atrium. It is very much like the model shown
in Fig. 22-2 on p. 397 of Berne, et al, “Physiology”, 5th edition or Figure 4-1 on p. 112 of L. Costanzo,
“Physiology”, 3rd Edition.

Let’s say the right heart output increases by 1% while the left heart output
remains constant at 5 L/min. In just an hour, 3 L of fluid (50 mL/min differential
x 60 minutes) would accumulate in the pulmonary circulation. That’s some
congestion. But this doesn’t happen. Instead the increased right heart output
causes an increase in left ventricular EDV that, in turn, causes an increase in the
left heart output. Thus the system increases its flow until it again equal
everywhere. This matching of the two hearts through the classic Frank-Starling
mechanism happens so rapidly and thoroughly that the right heart, pulmonary
circulation, and left heart can be treated conceptually as one big “pump” as shown
in Fig. 2 below.

Figure 2. Same as Figure 1 only the right heart, pulmonary circulation, and left heart have
been coalesced into one big “pump”.

379
Control of Cardiac Output 1 - Dr. Ford

III. THE CARDIAC OUTPUT, AND VENOUS RETURN, ACHIEVED BY THE


INTACT CIRCULATORY SYSTEM IS DETERMINED BY THE
PROPERTIES OF BOTH THE HEART AND CIRCULATORY SYSTEM.

Until 1955, physiologists argued vehemently about whether flow through the CV
(cardiovascular) system is determined by the function of the heart or the function
of the vascular system. Arthur Guyton deserves the credit for providing a
framework for physiologists to attack the question of the role of the heart and
vasculature in regulating cardiac output. This framework is used in this and
subsequent lectures. The approach is described in detail in a book by Guyton,
Jones, and Coleman entitled "Circulatory Physiology: Cardiac Output and its
Regulation" Second Edition, 1973, published by W.B. Saunders Co. Philadelphia.
This analysis shows that when a normal heart is in the circulation it is primarily
the properties of the vascular system which determine cardiac output. (The
normal heart can pump whatever the vascular system returns to it.) With heart
failure, both the heart and vascular system determine cardiac output.

IV. THE CARDIAC OUTPUT OF THE SYSTEM SHOWN IN FIG. 2 CAN BE


DETERMINED IF: 1) THE DEPENDENCE OF THE OUTPUT OF THE
HEART ON RIGHT ATRIAL PRESSURE (CARDIAC FUNCTION
CURVE) AND 2) THE DEPENDENCE OF FLOW IN THE VASCULAR
SYSTEM ON RIGHT ATRIAL PRESSURE ARE KNOWN.

The dependence of flow in the vascular system on right atrial pressure is shown in
Fig. 3:

Figure 3. Vascular flow and right atrial pressure. This is analogous to fig. 22-3, p.398 of Berne, et
al, “Physiology”, 5th Edition or figure 4-26 on p. 151 of L. Costanzo, “Physiology”, 3rd edition.

The theoretical basis for this curve is not obvious. The curve may be derived
mathematically and for those who like mathematical rigor that derivation is
strongly recommended. In the lecture, we will follow the mathematical approach.
For those who prefer a more intuitive approach, the following bit of the syllabus
is one such approach.

380
Control of Cardiac Output 1 - Dr. Ford

The desired curve can be derived intuitively by carefully examining the time
course of changes in: right atrial pressures, mean arterial pressures and flows
through the vascular system upon stopping the heart. This record is shown in Fig.
4.

Figure 4. Pressures and flows in the vascular system upon stopping the heart.
Pa = arterial pressure, Rap = right atrial pressure (central venous pressure).

By answering the following questions about the curves in Fig. 4 the origin of the
vascular function curve can be discerned.

A. Why does arterial pressure fall? (The pressure difference causes a flow
from the arteries to the veins.) Why does it fall fastest immediately after
stopping the heart? (The pressure difference is the greatest then.)
B. Why does venous pressure rise? (Because the volume of blood in the veins
increases.) Why does it rise less than arterial pressure falls? (Because the
veins have a higher compliance than the arteries.)
C. Why doesn't pressure in the arteries and veins fall to zero? (Because there
is still an enclosed volume.) This is very important point. It is known as
the mean circulatory pressure or mean systems pressure, i.e. the pressure
in the system at zero flow.

Using the data in Fig. 4 and knowing that the R = 20 mm Hg x minutes/liters it is,
possible to construct the Vascular function curve shown in Fig. 3.

381
Control of Cardiac Output 1 - Dr. Ford

In simplest terms this figure demonstrates that as Pra (right atrial pressure)
increases vascular flow rate decreases. (Clinically the term "central venous
pressure" is often used in lieu of the term right atrial pressure. Central venous
pressure is determined by measuring right atrial pressure.)

I believe it is easier to see the how the various factors influence the shape of the
vascular function curve if you take a little more analytical approach.

Let’s start the “pump” and produce a flow of 1 L/min. At steady state, this means
there also must be a flow from the arteries to the veins of 1 L/min. The only way
to produce a flow is to have a pressure difference and the only way to produce a
pressure difference is by a volume change. The pressure difference is being
produced by the “pump” taking a volume from the veins and putting it in the
arteries, thus:

In general, of course Flow = (Part - Pven)/Total resistance

Part increases by the increment ΔPart = ΔVol/Ca and

Pressure in the vein decreases by the increment ΔPven = -ΔVol/Cv

where Ca = arterial compliance and Cv = venous compliance

But since the volume increment is the same, i.e. the same volume is taken from
the veins and put into the arteries:

ΔPart x Ca = - ΔPven x Cv or ΔPart = -(Cv/Ca) x ΔPven

Now these increments of pressure must be on top of the means system pressure,
so:

Part = ΔPart + Pms and Pven = ΔPven + Pms

We can eliminate ΔPart as a variable by noting our 2 relationships show:

ΔPart = Part - Pms and ΔPart = -(Cv/Ca) x ΔPven

We further use the relation ΔPven = Pven - Pms so:

ΔPart = Part - Pms and ΔPart = -(Cv/Ca) x (Pven - Pms)

Hence:

Part - Pms = -(Cv/Ca) x Pven + (Cv/Ca)Pms which rearranges to:

Part = -(Cv/Ca) x Pven + [ 1 + (Cv/Ca)] x Pms

382
Control of Cardiac Output 1 - Dr. Ford

If we substitute this into our general expression for flow, we have:

Flow = (Part - Pven) = -(Cv/Ca) x Pven + [ 1 + (Cv/Ca)] x Pms - Pven


Total Resistance Total Peripheral Resistance (TPR)

Since flow must be the same everywhere, Flow = Venous Return and we have the
following desired relationship between venous return and right atrial pressure:

Venous Return = (Pms - Pven) x [(Cv/Ca) + 1]


TPR

V. THE SHAPE OF THE VASCULAR FUNCTION IS DETERMINED BY


RESISTANCE TO FLOW THROUGH THE VASCULAR SYSTEM AND
MEAN SYSTEMIC PRESSURE.

The relationship we have derived has a couple of notable findings. The first, and
perhaps not so intuitive, is that the driving force for venous return is (Pven - Pms).
The second is that the there is linear relationship between this driving force and
venous return, i.e. the relationship is of the form y = mx + b. The slope of this
curve is given by - [(Cv/Ca) + 1]/TPR and the x intercept (zero flow) occurs when
Pven = Pms. We will show that the TPR and Pms are dynamic physiological
variables. Ca/Cv is really not a dynamic variable. It may change slightly over the
course of many years (ageing) and recent studies have shown that it is possible to
change Ca over the course of several weeks of intensive exercise training which
may be a factor in the cardiovascular conditioning of highly trained athletes.
But the Ca/Cv ratio is not felt to be subject to daily regulation. Note the negative
sign of the slope, i.e. it slants to the left. If you substituted realistic values for
Ca/Cv and TPR, say 19 and 20 mmHg/(L/min), you would obtain the linear
portion of the vascular function curve shown in Fig. 3.

VI. THE “KNEE” OF THE VASCULAR FUNCTION CURVE IS A


CONSEQUENCE OF THE NEGATIVE PRESSURES IN THE THORACIC
CAVITY

Figure 3 is not linear at low right atrial pressures (< -2 mmHg). Our relationship
does not predict this non-linearity. Is our relationship wrong? No, the veins are
merely thinned walls structures that collapse when subjected to even small
intramural pressure differences such as those experienced in the thoracic cavity as
a result of respiratory activity. In common words, they collapse like straws when
sucked on too hard. Thus the TPR effectively becomes infinite and no flow is
possible.

VII. CHANGES IN RESISTANCE TO FLOW CHANGE THE SLOPE OF


VASCULAR FUNCTION CURVES.

383
Control of Cardiac Output 1 - Dr. Ford

Fig. 5 shows vascular function curves with different resistances to venous return.

Figure 5. Vascular function curves with different resistances to flow. Curve A is


the vascular function curve for R = 20 mm Hg x min/liters. Curve B is the vascular
function curve for R = 10 mm Hg x min/liters. Curve C is the vascular function curve
for R = 40 mm Hg x min/liters. Analogous to Fig. 22-7 on p. 401 of Berne, et al,
“Physiology”, 5th Edition or Fig. 4-30 on p. 155 of L. Costanzo, “Physiology”. 3rd
Edition.

Vascular resistance to flow and hence the slope of the vascular function curve is
subject to change by the sympathetic nervous system and by vasodilator and
vasoconstrictor substances. The resistance of the vascular system to flow is
largely, but not entirely, determined by the caliber of the arterioles. The caliber of
the arterioles, in turn, is changed by the contraction and relaxation of the vascular
smooth muscle of the arterioles. This, in turn, is often a function of the
sympathetic nerve activity at this level although vasoactive agents and blocking
drugs can influence their activity as well. Decreases in vascular resistance
(vasodilation) will increase flow in the vascular system for any given right atrial
pressure. Conversely increases in resistance (vasoconstriction) will decrease flow
for any given value of right atrial pressure.

VIII. Pms, MEAN SYSTEM PRESSURE, IS DETERMINED BY STRESSED


VOLUME AND CAPACITANCE OF THE STRESSED VOLUME.

Any elastic structure will exert a pressure on its enclosed volume when its walls
are sufficiently stretched to exert a tension, i.e. the walls are strained. If you start
with an empty structure and slowly fill it, no pressure develops until there is
sufficiently stretch in the walls to produce a strain. This volume is the unstressed

384
Control of Cardiac Output 1 - Dr. Ford

volume. As you add further volume, a pressure results. This is the stressed
volume. This notion is pictured in Fig. 6 below:

Figure 6. Effect of blood volume on Pms. Pms = mean system pressure (mm Hg), UV = unstressed volume (liters),
SV = stressed volume (liters). This is similar to Fig. 4-27 On p. 152 of L. Costanzo, “Physiology”, 3rd edition.

This figure shows that blood is partitioned into vascular compartments as follows:

BV = UV + SV

BV = blood volume, UV = volume of unstressed vascular compartment,


SV volume of stressed vascular compartment.

Note: Do not confuse the compartment volumes, UV and SV, with blood volume.

The slope of the curve for the stressed volume is the compliance of the stressed
system. Pms can be determined graphically from the extrapolation shown or from
the relationship Pms = Stressed Volume / Compliance of the stressed volume.
Since we rarely know a precise value for the stressed volume, we will use the
extrapolation technique in further discussion.

IX. CHANGES IN STRESSED VOLUME CHANGE Pms

There are two ways to change the stressed volume. One is by changing the blood
volume. Increases in blood volume (over transfusion) increase the stressed
volume and thus increase Pms. Decreases in blood volume (hemorrhage) decrease
the stressed volume and thus decrease Pms.

The other way to change stressed volume is to change the amount of blood
subjected to stress. Since most of the blood (~70%) is found in the veins, this is
accomplished by venoconstriction. Increasing venous tone increases the stressed

385
Control of Cardiac Output 1 - Dr. Ford

volume. Effectively shifting the curve in Fig. 6 to the left. Assuming blood
volume does not change, this causes an increase in Pms. Conversely a decrease in
venous tone would decrease the stressed volume, shift the curve in Fig. 6 to the
right, and cause a decrease in Pms.

The slope of the relation between Pms and blood in the SV is Csv (compliance of
the stressed volume). Physiologically this variable is not subject to change in
short term, nor is it regulated: thus we do not have to worry about it changing.

386
Control of Cardiac Output 2 - Dr. Ford

Control of Cardiac Output 2


George D. Ford, Ph.D.

OBJECTIVES:

1. Discuss the consequences of having a closed circulatory system.


2. Discuss how the right heart and left heart are coupled physiologically.
3. Know the three major factors governing venous return to the heart. Be able to
show these graphically as well as how variations in each factor affect this
graphical representation. Specifically:
a. Describe the role that the ratio of arterial to venous compliance and total
peripheral resistance plays in the graphical description of the venous
return.
b. Discuss the notions of mean systems pressure and stressed and unstressed
volume and the effect variations in these parameters have on the graphical
representation above.

4. Graphically display the relationship between venous return and cardiac output.
5. Translate the concepts discussed into clinically observable parameters.
6. Discuss at least three other factors which could influence venous return but are
not easily assimilated into the graphical analysis used for the other concepts
above.

I. REVIEW OF THE HIGHLIGHTS FROM LAST LECTURE IN THIS


SEQUENCE:

A. In the steady-state, flow must be equal everywhere in the closed


cardiovascular system.
B. The Vascular Function Curve is the relationship between venous return
and right atrial pressure. It depends on 3 things:

1. The mean systems pressure (Pms) which determines the zero


intercept, i.e. pressure in the system with no flow. The mean
systems pressure is primarily a function of the effective circulating
blood volume.
2. The venous to arterial compliance ratio (Cv/Ca) and total
peripheral resistance (TPR) which determine the slope of the
vascular function curve.
3. Intrathoracic pressure which determines the “knee” of the
vascular function curve.

II. CARDIAC OUTPUT IN THE STEADY STATE IS DETERMINED BY


THE PROPERTIES OF THE HEART, VASCULAR SYSTEM, AND
BLOOD VOLUME. THESE DETERMINE THE SHAPES OF CARDIAC
FUNCTION AND VASCULAR FUNCTION CURVES. CARDIAC
OUTPUT WILL BE IN A STEADY STATE WHEN THE VASCULAR

387
Control of Cardiac Output 2 - Dr. Ford

FLOW RATE AND CARDIAC OUTPUT ARE EQUAL AT THE SAME


RIGHT ATRIAL PRESSURE.

Since flow must be equal everywhere in the system, the system can only exist at
flows compatible with both functions, i.e. the intersection of the two curves as
shown in Fig. 1 below.

Figure 1. Superposition of normal cardiac and vascular function curves. Pretty much like Fig. 22-8 on p. 401
of Berne, et al, “Physiology”, 5th Edition or Fig. 4-26 on p. 151 of L. Costanzo, “Physiology”, 3rd edition.

It is important to note here that the heart is an organ composed primarily of


striated muscle, that the vascular system is composed primarily of smooth muscle
and elastic elements, and that the vascular system is filled with blood. The cardiac
and vascular function curves are generated by the properties of these two
muscular systems and blood. It follows from this that to change the shape or
position of the cardiac and vascular function curves requires either that the
contractility of cardiac muscle change, the contraction of vascular muscle change,
or that blood volume changes.

III. CHANGES IN CARDIAC OUTPUT IN CARDIOVASCULAR SYSTEMS


ARE ACHIEVED BY MODIFYING THE VASCULAR FUNCTION
CURVE, THE CARDIAC FUNCTION CURVE, OR BOTH.

A. Positive changes in the cardiac function curve under resting


circumstances produce little change .

Figure 2 shows that increases in the inotropic (↑ contractility) state of the


heart will have little effect on cardiac output. Under normal conditions the
cardiovascular system equilibrium point is near the heel of the vascular

388
Control of Cardiac Output 2 - Dr. Ford

function curve and the improved ability of the heart to increase cardiac
output can not lead to an increased cardiac output. (That the equilibrium
point for the cardiovascular system is close to a right atrial pressure of 0
mm Hg indicates that normal heart has successfully transferred almost as
much blood to the arterial reservoir as possible and therefore created as
great a pressure gradient for flow as possible. Any additional transfer of
blood to the arteries would collapse the veins.)

B. Negative changes in cardiac function lead to elevated right atrial


pressures.

On the other hand, decreases in cardiac contractility (negative inotropic


state) can have fairly profound effects as seen in curve C of Fig. 2. Note
the elevated right atrial pressure (RAP) at the new steady state. The
negative inotropic state makes the heart less able to transfer blood from
the veins to the arteries and hence the higher right atrial pressure and the
lower vascular flow rate. Elevated RAPs are strong clinical indicators of
heart failure.

Figure 2. The effect of inotropic state on cardiac output and Pra. Curve b control. Curve a, a cardiac function
curve shifted by a positive inotropic intervention, Curve c, a cardiac function curve shifted by a negative
inotropic intervention. See Fig. 22-12 on p. 404 of Berne, et al, “Physiology”, 4th Edition or Fig. 4-28 on p. 153
of L. Costanzo, “Physiology”, 3rd edition.

C. Changes in blood volume cause changes on cardiac output.

Fig. 3 shows the effect of an increase or decrease in blood volume on


cardiac output/venous return and right atrial pressure. Note that changes in
blood volume affect only Pms. Otherwise the cardiac function curve is
virtually unaffected and the slope of the vascular function curves are also
relatively unaffected.

389
Control of Cardiac Output 2 - Dr. Ford

Figure 3. Effect of blood volume changes on cardiac output/venous return and right atrial pressure. Curve b is
the control condition, curve a is produced by-an increase in blood volume, curve c is produced by a decrease in
blood volume. The transfusion part is shown in Fig. 22-11 on p. 403 of Berne, et al, “Physiology”, 5th Edition or
Fig. 4-29 on p. 154 in L. Costanzo, “Physiology”, 3nd edition.

D. Changes in stressed volume also effect change in cardiac output and


right atrial pressure.

Increases in stressed volume produce changes in the vascular function


curve identical to that of a blood volume increase. Curve A of Fig. 3 could
be produced by an increase in stressed volume (or a decrease in unstressed
volume). Curve C of Fig. 3 could be produced by a decrease in stressed
volume (or an increase in unstressed volume).

E. The effect of the sympathetic nervous system on cardiac output and


right atrial pressure.

As a quick review, fibers from the sympathetic nervous system innervate,


among other structures, veins, arterioles, and myofibers of the cardiac
ventricles. The neurotransmitter released from these fibers is
norepinephrine. In vascular smooth muscles this transmitter binds to
alpha1 receptors and causes the muscle fibers to contract. The effect on
veins is to decrease the unstressed volume and thereby to increase Pms.
The effect on arterioles is to decrease the caliber of the arterioles, to
increase total peripheral resistance (TPR), and thereby to rotate the
vascular function curve counterclockwise. The myofibers in the
ventricular walls of the heart increase their contractility (positive inotropic
response) in response to an increase in the release of neurotransmitter
from the sympathetic nervous system. The norepinephrine binds to beta
receptors to produce this increase in cardiac contractility. The

390
Control of Cardiac Output 2 - Dr. Ford

consequence of these three effects of sympathetic stimulation on the


cardiovascular system is shown in Fig. 4.

Figure 4. The effect of sympathetic stimulation on cardiac output/venous return and right atrial pressure. Figs.
22-9 on p. 402 and 29-13 on p. 468 of Berne, et al, “Physiology”, 4th Edition, have some of this same
information.

There are two points which need to be made about this figure and the
response of the cardiovascular system to increases in sympathetic nervous
system activity. 1) The effect of the decrease in unstressed volume
(contraction of the veins) is sufficiently great so that the vascular function
curve produced in response to sympathetic nervous system activation
always lies to the right and above the control curve. The counterclockwise
rotation of the vascular function curve due to contraction of arterioles does
not bring the curve produced by sympathetic stimulation below the control
curve. 2) It is difficult to predict precisely how right atrial pressure will
change. If there is a change in it will be small.

IV. DURING THE TRANSIENT STATE VENOUS RETURN MAY DIFFER


FROM CARDIAC OUTPUT

When the equilibrium condition (steady-state) changes from one value to another
the cardiovascular system goes through a transient-state in which cardiac output
differs from venous return. An example of such a transient-state is the response of
the cardiovascular system to a sudden addition of fluid into the cardiovascular
system. The response is plotted in Fig. 5.

391
Control of Cardiac Output 2 - Dr. Ford

Figure 5. The transient effects of intra arterial infusion on venous return and cardiac output.

After rapid infusion of fluid into an artery a new venous return curve is created
because of the increase in Pms from 7 mm Hg to 10 mm Hg produced by the
infusion of fluid. Note that at the time of infusion the equilibrium condition was a
right atrial pressure of 0 mm Hg and a cardiac output of 5 liters/min. After
infusion the venous return at 0 mm Hg is, however, 7.5 liters per minute. This is
greater than cardiac output. With venous return greater than cardiac output,
venous volume and hence venous pressure will increase. The increase in venous
pressure will increase right atrial pressure. In this example the increase is first to 1
mm Hg and then to 2 mm Hg. At a right atrial pressure of 2 mm Hg the cardiac
output and venous return become equal at 7.0 liters/min, the new equilibrium
(steady-state) condition.

V. OTHER FACTORS INFLUENCE VENOUS RETURN

A. THE SKELETAL MUSCLE “PUMP”

Standing upright has a potential price to pay with regard to venous return.
Consider the force of gravity acting on a column of fluid represented by
the blood in our feet leading to our heart. This is a column of say 130 cm
of H2O or the equivalent of 10 cm of Hg. This could be a damper on
venous return, a condition referred to as orthostatic hypotension. This
doesn’t usually happen for two reasons. First there is a corresponding
effect on the arterial side which tends to push with nearly equal force.
Think of what happens in an U-tube. Increasing the pressure has an equal
effect on both sides. More physiologically though, the veins have valves

392
Control of Cardiac Output 2 - Dr. Ford

which prevents any significant back flow. In addition, as the veins course
through the skeletal muscle systems of our legs and thighs, they are
exposed to the rhythmic contractions of our postural muscles. In effect this
acts like a peristaltic pump, rather like squeezing a tube of tooth paste.
This action, combined with the unidirectionality conferred by the valves
acts as an effective pump for venous return from the lower extremities.
This is generally enough to offset the influence of gravity. How does this
get reconciled with the Guyton approach? I’m sure Dr. Guyton would
argue this effect is equivalent to an increase in stressed volume.

B. THE RESPIRATORY “PUMP”

During inspiration the pressure within the thorax and around the heart
decreases. This decrease increases ventricular transmural pressure and
increases the flow of blood into the ventricles. This is most important for
the right ventricle. Thus inspiration increases venous return to the right
heart and increases right heart output.

During expiration the pressure within the thorax increases and reduces
venous return. However venous return during apnea (cessation of
breathing) is lower than during normal rates of respiration indicating the
normal breathing facilitates venous return more than impeding it.

There are some times when the respiratory effect is more prominent. For
example, the Valsalva maneuver is used clinically to test the competence
of the baroreceptor reflex. (This reflex will be discussed in a later lecture.)
During the Valsalva maneuver intrathoracic pressure is increased to very
high pressures by contracting the thoracic muscle when the glottis is
closed. This very high intrathoracic pressure can bring venous return to the
right heart to zero. Coughing, defecation, heavy lifting, and the playing of
brass instruments are modifications of this maneuver and can produce high
intrathoracic pressures which impede venous return.

Another possible ramification is in the application of artificial respiration.


Positive pressure inspiration with normal pressure expiration, as done in
some types of positive pressure ventilation, impedes venous return
markedly. If positive pressure inspiration is followed by negative pressure
expiration venous return is increased.

VI. RECOMMENDED READING

Berne, R. M., Levy, M. N., Koeppen, B.M. & Stanton, B.A. "Physiology", 5th
Edition, Mosby Year Book, Boston, 2004. Chapter 22, pp 395 -412.

Guyton, Jones, and Coleman, "Circulatory Physiology: Cardiac Output and its
Regulation", 1973, W.B. Saunders, Philadelphia, Chapter 14, pp 237-248.

393
Control of Cardiac Output 2 - Dr. Ford

Costanzo, Linda S., “Physiology”, 3rd Edition, 2006, Saunders, Philadelphia,


Chapter 4, pp. 151-156.

VII. SAMPLE QUESTIONS

Instructions: Identify all correct answers.

1. If the mean circulatory pressure in an isolated non-flowing vascular


system is 10 mm Hg, the unstressed volume is 3.5 liters and the
capacitance of the stressed volume is 100 ml/mm Hg, the total volume of
fluid in the vascular system is?
A. 3.0 liters
B. 3.5 liters
C. 4.0 liters
D. 4.5 liters
E. 5.0 liters

2. A reduction in the amount of norepinephrine released by the sympathetic


nervous system would produce which change or changes in a vascular
function curve?
A. decrease mean system pressure
B. decrease unstressed volume
C. decrease total peripheral resistance
D. rotate the slope of the vascular function curve counterclockwise
E. rotate the slope of the vascular function curve clockwise

3. Which of the following changes would increase right atrial pressure in a


cardiovascular system?
A. increase in cardiac contractility
B. increase in blood volume
C. increase in total peripheral resistance
D. decrease in unstressed vascular volume
E. decrease contraction of veins

4. Which of the following changes would decrease cardiac output in a


cardiovascular system?
A. increase in total peripheral resistance
B. decrease in cardiac contractility
C. decrease in mean system pressure
D. increase in unstressed vascular volume
E. increase contraction of veins

394
Control of Cardiac Output 2 - Dr. Ford

ANSWERS

1. D (Calculation: volume = unstressed + stressed = 3.5 L + 10mmHg x


100ml/mmHg = 3.5 + 1 L = 4.5 L)
2. A, C, E (yes A, ↓NE means ↓ venoconstriction and hence ↓stressed
volume and ↓Pms; not B, ↓stressed volume means ↑unstressed; yes C,
↓NE means ↓ vasoconstriction; not D, see explanation for E; yes E, ↓
vasoconstriction means ↓ TPR which causes ↑slope)
3. B, D (not A, shift of CFC to left causes ↓ RAP; yes B, ↑BV means ↑Pms
means VFC shifts upward; not C, ↑TPR means VFC shifts downward; yes
D, ↓ UV means ↑stressed volume which means ↑Pms as in D; not E,
venodilation means ↑unstressed and ↓stressed volume hence ↓Pms)
4. A, B, C, D (really same question as one above asking which parameters
would shift either the VFC or CFC in the right direction to move the
intersection of the two curves upward; yes D because ↑unstressed and
↓stressed volume hence ↓Pms; not E because venoconstriction leads to
↑Pms)

395
The Arterial and Venous Systems - Dr. Pittman

The Arterial and Venous Systems


Roland Pittman, Ph.D.

OBJECTIVES:

1. State the primary characteristics of the arterial and venous systems.


2. Describe the elastic properties of arteries in terms of pressure, volume and
capacitance.
3. Define systolic, diastolic, pulse and mean arterial pressures.
4. Describe the relationship among arterial pulse pressure, stroke volume and arterial
capacitance.
5. State the characteristics of the venous system in terms of blood volume and
capacitance.
6. Describe the ways in which mean circulatory (or systemic) pressure can be
changed in terms of blood volume and venous contraction.

SUGGESTED READING ASSIGNMENT:

R. M. Berne, B. M. Koeppen, M. N. Levy, and B. A. Stanton, Physiology, 5th Ed., St.


Louis: Mosby, pp. 355-365, 2004

L.S. Costanzo, Physiology, 3rd Ed., Philadelphia: W.B. Saunders, pp. 120-125; 152,
2006.

I. THE WHOLE SYSTEM

A. Overall schematic diagram showing:

1. Parallel architecture of the vascular network


2. Unidirectional movement of blood (valves in veins and heart)

Figure 1.

396
The Arterial and Venous Systems - Dr. Pittman

B. Some facts about the arteries and veins

1. About 80% of the resistance to blood flow is located on the arterial


side of the circulation.
2. About 70% of the blood volume is located on the venous side of the
circulation.
3. The veins and heart have one-way valves to ensure the unidirectional
flow of blood.
4. The arteries and veins are in a state of partial constriction due to the
continuous release of norepinephrine, a vasoconstrictor, from the
sympathetic nerve terminals located in the walls of these vessels. In
addition, the vasodilator nitric oxide (NO) is continuously released
from the endothelial cell lining of all blood vessels.

II. THE ARTERIAL SYSTEM

A. Elastic recoil of arteries

1. The heart ejects blood only during about 1/3 of the cardiac cycle.
2. The potential energy stored in the elastic arteries maintains the
pressure that keeps blood flowing during the rest of the cardiac
cycle.

B. Arterial elasticity

1. Static pressure/volume relationship

a. The volume of blood in the arteries determines arterial


pressure. An increase (decrease) in arterial blood volume
produces an increase (decrease) in arterial pressure. This
general principle, that the blood pressure is determined by the
blood volume, is true for any segment of the vascular system.
b. The distensibility of the arterial wall generally decreases
with increasing age (i.e., the arteries of older individuals
are generally stiffer than those of younger persons).

397
The Arterial and Venous Systems - Dr. Pittman

Figure 2. Pressure-volume relationships for aortas obtained at autopsy from humans in the
different age groups (denoted by the numbers at the right end of each of the curves).
(Redrawn from Hallock P. Benson IC: J Clin Invest 16:595, 1937).

2. Definition of capacitance (also compliance or distensibility)

Ca= dV/dP or ΔV/ΔP

This equation quantifies the passive elastic properties of the arteries by


defining the capacitance of the arteries (or any other vessel) as the
change in volume divided by the associated change in pressure, i.e., it
is the slope of the above pressure - volume relationship.

C. Determinants of arterial pressure

1. Pressures depend on the volume of blood in the large arteries at each


instant of time during the cardiac cycle.

a. Diastolic pressure: Pd (min art. press. during cardiac cycle)


b. Systolic pressure: Ps (max art. press. during cardiac cycle)
c. Mean arterial pressure: Pa ≈ Pd + (Ps - Pd)/3
d. Pulse pressure: Pp = Ps - Pd

398
The Arterial and Venous Systems - Dr. Pittman

Figure 3. Arterial systolic, diastolic, pulse and mean pressures. The mean arterial pressure
(Pa) represents the area under the arterial pressure curve (shaded area) divided by the cardiac
cycle duration (t2 - t1).

D. There are two determinants of the volume of blood in the large arteries (and
hence the pressure in the large arteries) at any given instant during the cardiac
cycle. These are:

1. The rate at which blood enters the large arteries from the left ventricle
of the heart (i.e., the cardiac output);
2. The rate at which blood flows through all of the organs of the body
(i.e., blood flow through the total peripheral resistance, sometimes
called vascular runoff).
3. Thus, the rate of change of blood volume in the large arteries is just
the rate at which blood flows into the arteries minus the rate at which
blood flows out of the arteries. Symbolically this can be represented
as:

dVa/dt = Inflow from heart (t) - Outflow through organs (t)

Thus, as Pa increases during systole, blood must be entering the


arteries faster than it is flowing through the organs; during diastole, the
opposite is true.

E. Determinants of pulse pressure

1. Stroke volume: Pp ≈ stroke volume/Ca

This relationship is fairly accurate (we ignore vascular runoff for the
moment) and comes from (1) rearranging the equation that defines Ca
and (2) identifying the change in volume as the stroke volume of the

399
The Arterial and Venous Systems - Dr. Pittman

left ventricle and the resulting change in pressure as the pulse pressure.

For simplicity, the graph below shows a linear pressure - volume


relationship, a simplifying approximation that retains the essential
feature of the elastic property of the arteries.

Figure 4. Effect of a change in stroke volume on pulse pressure in a system in which arterial
compliance remains constant over the range of pressures and volumes involved. A larger
volume increment (V4 - V3 as compared with V2 - V1) results in a greater mean pressure (PB as B

compared with PA) and a greater pulse pressure (P4 - P3 as compared with P2 - P1).

2. Arterial capacitance: Ca

The effect on pulse pressure of a change in arterial capacitance can be


seen by noting that Pp is inversely proportional to Ca. Thus, increases
in Ca (i.e., increasing distensibility) produce lower pulse pressure,
whereas decreases in Ca (i.e., decreasing distensibility) produce higher
pulse pressure.

Figure 5. For a given volume increment (V2 - V1) a reduced arterial compliance (curve B as
compared with curve A) results in an increased pulse pressure (P4 - P1 as compared with P3 -
P2).

400
The Arterial and Venous Systems - Dr. Pittman

3. Total peripheral resistance: TPR = (Pa - Pv or ra)/CO

The dependence of pulse pressure on total peripheral resistance (TPR)


is less obvious than the dependence on stroke volume and capacitance,
because TPR does not explicitly appear in the equation for pulse
pressure.

What happens when TPR increases? By rearranging Ohm's law for the
flow of blood through the total peripheral resistance, one sees that:

Pa = Pv or ra + TPR • CO

Since Pv (central venous pressure or blood pressure in the large veins)


is close to atmospheric pressure (i.e., about 0 mm Hg), then we see
that:

Pa ≈ TPR • CO

to a good approximation. At a constant CO, increasing TPR will result


in an elevated Pa (this could be called hypertension if Pa is elevated
high enough). Panel A of the figure below shows a linear pressure -
volume relationship and an elevated Pa does not produce any change in
pulse pressure for the same stroke volume (recall that we are keeping
CO constant) that we had at the lower Pa. However, when we consider
the more accurate nonlinear pressure -volume relationship shown to
the right in panel B, the elevated Pa puts us on a portion of the curve
that has a smaller slope (and hence lower distensibility and Ca). So,
since a lower value of Ca is associated with the elevated arterial
pressure, pulse pressure will be higher than at the lower mean arterial
pressure. Thus, one would expect a hypertensive individual to have a
higher pulse pressure for a given stroke volume than would a person
with normal arterial pressure. Since hypertension is often more
common in older individuals, the elevated mean arterial pressure,
combined with their less compliant large arteries, exacerbates the
condition of higher pulse pressures, as well as higher systolic and
diastolic pressures.

401
The Arterial and Venous Systems - Dr. Pittman

Figure 6. Effect of a change in TPR (volume increment remaining constant) on pulse pressure when
the pressure-volume curve for the arterial system is rectilinear, A, or curvilinear, B.

III. THE VENOUS SYSTEM

A. Distensibility

Veins are about 20 times as distensible as arteries! Thus, the capacitance


or compliance for veins is about 20 times its value for the arteries.
Symbolically this is expressed as:

Cv ≈ 20 Ca

B. Most (about 70%) of the blood volume resides in the veins. It is


convenient to define the terms unstressed volume and stressed volume
for our future discussions of the vascular function curve.

1. Unstressed volume can be thought of as the "slack" volume in the


cardiovascular system under a given level of vascular tone (or
contraction). It is the amount of blood you would have to introduce
into a previously empty vascular system before there would be
elastic recoil of the vessels. Thus, the blood pressure would be 0 mm
Hg in an unperfused circulatory system until the blood volume
exceeded the unstressed volume.
2. Stressed volume can be thought of as the difference between total
blood volume and the unstressed volume. It is the increment in blood
volume that produces the recoil pressure within the circulatory
system.

402
The Arterial and Venous Systems - Dr. Pittman

3. Blood volume (VB) = unstressed volume (VU) + stressed volume


(VS)

C. Static pressure - volume relationship for the isolated, unperfused


circulatory system

1. The figure below shows the relationship between the blood pressure
(Pmc, mean circulatory filling pressure) in an isolated, unperfused
circulatory system, as a function of the volume of blood contained in
it. Note that this pressure is produced by the elastic recoil of the
arteries and veins against the blood contained within them. Pmc does
not rise above 0 mm Hg until the unstressed volume has been filled.
One can think of this recoil pressure as reporting the degree to which
the circulatory system is filled with blood.

Figure 7.

2. The two most common ways to change Pmc (by changing the stressed
volume) are to change:

a. Blood volume

Increases (decreases) in blood volume produce increases


(decreases) in Pmc, because of the elastic recoil of the stressed
volume.

b. Venous contraction

Recall that most of the blood volume resides in the veins and
that the veins are in a constant state of partial constriction due
to their stimulation by the sympathetic nerves. Increasing

403
The Arterial and Venous Systems - Dr. Pittman

venous smooth muscle contraction produces constriction,


resulting in a smaller diameter and, hence, smaller unstressed
volume. A smaller unstressed volume with no change in overall
blood volume will lead to an increase in stressed volume and,
hence, in Pmc. Graphically, this corresponds to a shift to the left
in the static pressure - volume relationship. A decrease in
venous contraction will lead to the opposite results. The reason
why venous, rather than arterial, contraction is the important
factor here is because most of the blood volume is located in
the veins, so that any change in their contractile state will have
a larger impact on the distribution of blood within the
circulatory system.

STUDY QUESTIONS

1. If the volume of blood in an isolated nonflowing vascular system is 6 liters, the


unstressed volume is 4.5 liters and the capacitance of the stressed volume is 150
ml/mm Hg, the mean circulatory pressure will be:

A. 5 mm Hg
B. 6 mm Hg
C. 7 mm Hg
D. 10 mm Hg
E. 14 mm Hg

ANSWER: D

You need to remember the graph relating mean circulatory pressure to blood
volume and that only the volume of blood identified as being in the "stressed
volume" contributes to the vascular recoil that produces the mean circulatory
pressure.

The stressed volume in this question is 6 minus 4.5 liters, or 1.5 liters. You also need
to remember that the capacitance is defined as the change in volume divided by the
associated change in pressure. In this case, the change in volume is the stressed
volume and the change in pressure (change above the pressure existing when blood
volume equals unstressed volume) is the mean circulatory pressure. Thus, Pmc =
1500/150 = 10 mm Hg.

2. If heart rate is increased, but total peripheral resistance, stroke volume and
compliance remain unchanged, which of the following will be true? (Assume a
linear pressure-volume relation for the arteries.)

404
The Arterial and Venous Systems - Dr. Pittman

1. Diastolic pressure will increase.


2. Pulse pressure will be unchanged.
3. Systolic pressure will increase.
4. Cardiac output will increase.

ANSWER: E (1, 2, 3, & 4)

This question is challenging, but you should be able to reason out the answer. You
need to remember that the factors which can change pulse pressure are stroke
volume of the left ventricle, compliance of the large arteries and total peripheral
resistance. Thus, response 2 is correct. If heart rate increases and stroke volume is
constant, then cardiac output must also increase, so that response 4 is correct.
Deciding whether responses 1 and/or 3 are true is harder for most people to
determine. Since cardiac output is increased while total peripheral resistance
remains constant, mean arterial blood pressure will increase. Since pulse pressure
remains constant (see reasoning above), both systolic and diastolic pressure must
increase. This means that there must be a new and increased steady state volume of
blood in the large arteries.

3. Against the advice of his physician, a student takes too much of the experimental
drug Loosenup, resulting in increased distensibility of the aorta. Assuming that
stroke volume and heart rate are unchanged in the presence of the drug, which of
the following would be expected to decrease?

1. Systolic pressure in the aorta.


2. Diastolic pressure in the aorta.
3. Pulse pressure in the aorta.
4. Mean arterial pressure.

ANSWER: B (1 & 3)

If the aorta becomes more distensible (i.e., more compliant), then the recoil pressure
on the same stroke volume will be less, so pulse pressure and systolic pressure both
fall. Mean arterial pressure will not change since cardiac output and total
peripheral resistance remain unchanged (no mention is made of a change in TPR, so
you should assume that it remained the same; also the resistance vessels would have
to be affected by the drug and it is stated that only the aorta is affected). Since mean
arterial pressure remains unchanged and pulse pressure falls, diastolic pressure
must rise.

4. Pulse pressure:

A. is the difference between systolic and diastolic pressures.


B. decreases when stroke volume increases.
C. increases when arterial compliance increases.
D. decreases when total peripheral resistance increases.

405
The Arterial and Venous Systems - Dr. Pittman

E. increases when heart rate increases at constant cardiac output.

ANSWER: A

Pulse pressure is defined as the difference between systolic and diastolic pressures.
Pulse pressure increases when stroke volume increases (more stretch and elastic
recoil of the large arteries); it decreases when arterial compliance increases (large
arteries would be more distensible and thus accommodate the stroke volume with
less recoil); and it increases when TPR increases (the large arteries would be
operating at the higher pressure end of their static pressure-volume relationship).
An increased heart rate at constant cardiac output implies that stroke volume must
have decreased, thereby lowering pulse pressure.

406
Electrical Activity of the Heart - Dr. Baumgarten

Electrical Activity of the Heart: Action Potential, Automaticity, and


Conduction 1 & 2
Clive M. Baumgarten, Ph.D.

OBJECTIVES:

1. Describe the basic characteristics of cardiac electrical activity and the spread of
the action potential through the heart
2. Compare the characteristics of action potentials in different parts of the heart
3. Describe how serum K modulates resting potential
4. Describe the ionic basis for the cardiac action potential and changes in ion
currents during each phase of the action potential
5. Identify differences in electrical activity across the tissues of the heart
6. Describe the basis for normal automaticity
7. Describe the basis for excitability
8. Describe the basis for conduction of the cardiac action potential
9. Describe how the responsiveness relationship and the Na+ channel cycle modulate
cardiac electrical activity

I. BASIC ELECTROPHYSIOLOGIC CHARACTERISTICS OF CARDIAC


MUSCLE

A. Electrical activity is myogenic, i.e., it originates in the heart. The heart is an


electrical syncitium (i.e., behaves as if one cell). The action potential spreads
from cell-to-cell initiating contraction. Cardiac electrical activity is modulated by
the autonomic nervous system.
B. Cardiac cells are electrically coupled by low resistance conducting pathways gap
junctions located at the intercalated disc, at the ends of cells, and at nexus, points
of side-to-side contact. The low resistance pathways (wide channels) are formed
by connexins. Connexins permit the flow of current and the spread of the action
potential from cell-to-cell.
C. Action potentials are much longer in duration in cardiac muscle (up to 400 msec)
than in nerve or skeletal muscle (~5 msec). Action potential characteristics vary
in different portions of the heart.

407
Electrical Activity of the Heart - Dr. Baumgarten

II. ELECTRICAL ANATOMY OF THE HEART

A. The pacemaker of the heart is the sino-atrial node (SAN). The SAN is located
under the endocardium in the right atrium at the superior vena cava. Action
potentials arise in the SAN spontaneously and propagate to the rest of the heart
with a specific sequence and specific timing.

B. The rate of spontaneous firing of the SAN determines normal heart rate.
Parasympathetic (vagus n.; acetylcholine) and sympathetic (norepinephrine)
nerves enervate the SAN and modulate heart rate. Slowing of the HR
(parasympathetic) is termed a negative chronotropic effect, and increasing HR
(sympathetic) is termed a positive chronotropic effect.

C. The SAN action potential spreads into atria in all directions. The atrial
internodal tracts are preferential conduction pathways from SAN to AVN and LA
(of little physiological significance). Internodal tracts are also called the atrial
specialized conducting system. Conduction velocity is ~1 m/sec.

D. The AV node is the electrical link between the atria and ventricles. It is located
under the endocardium in the floor of the right atrium at the septum. The speed of
conduction of the action potential slows dramatically (~20 to l00-fold) in the AV
node. Normally, AV node conduction velocity is 0.01 to 0.05 m/sec. This gives
rise to the AV delay. The AV delay allows for the time needed to complete filling
of the ventricle before ventricular contraction begins.

Conduction velocity in AVN is modulated by autonomic tone. Sympathetics


speed conduction velocity, termed a positive dromotropic effect, and
parasympathetics slow conduction velocity, a negative dromotropic effect.

408
Electrical Activity of the Heart - Dr. Baumgarten

E. After passing through AV node, excitation enters the ventricular specialized


conducting system (His-Purkinje system) at the Bundle of His (Common
Bundle), proceeds through right and left bundle branches, their divisions, and
terminal Purkinje fibers to ventricular muscle. Conduction velocity is 2-4 m/sec
in the His-Purkinje system, the fastest conduction velocity in the heart. The His-
Purkinje system rapidly distributes the cardiac impulse; this allows contraction to
proceed from apex to base, as needed for efficient ejection of blood.
F. In the ventricular muscle, the action potential spreads at ~1 m/sec. (Details of
sequence of ventricular activation are covered in the ECG lecture.)
G. Normal sinus rhythm (NSR) is the normal rhythm of the heart.
Normal sinus rhythm requires that:
1. the impulse that initiates the beat must arise in the SAN;
2. the rate must be 60 to 100 bpm and regular;
3. excitation must occur in a normal sequence (given above) with appropriate
timing of excitation of various portions of the heart.

III. THE ACTION POTENTIAL

A. Phases of the Action Potential


Phase 0 – upstroke
Phase 1 − initial repolarization
Phase 2 – plateau
Phase 3 – repolarization
Phase 4 − electrical diastole

409
Electrical Activity of the Heart - Dr. Baumgarten

B. Resting Potential

In atria, ventricle and Purkinje fibers, the membrane potential is normally


constant during phase 4 and is termed the resting potential (Em). Em is largely
determined by the transmembrane K+ gradient and approaches EK, the K+
equilibrium potential. Changes in serum K+ over the clinically observed range
significantly alter Em. Em of SA and AV nodes is essentially insensitive to serum
K+ (below ~20 mM). In SAN and AVN, where phase 4 is not constant, the most
negative potential attained is termed the maximum diastolic potential (MDP)
rather than Em.

(Note: Alterations in serum K+ must be electroneutral, e.g., opposite


changes in K+ and Na+ or matching changes in K+ and Cl-; effects of
altered serum K+ on Em are due to changes in EK. Students sometimes
erroneously assume that the altered serum K+ changes simply increases
the number of positive charges outside the cell.)

C. Action Potential Duration

Action potential duration (APD) varies inversely with rate. As heart rate
increases, the APD shortens.

410
Electrical Activity of the Heart - Dr. Baumgarten

D. Characteristics of the action potential vary in different portions of the heart.

1. Em is about −85 mV (inside neg.), except at SAN and AVN, Em is −60 mV.
2. APD: 150 to 400 msec
PKJ > Vent > Atria > AVN ≈ SAN
3. Upstroke is rapid except in SA and AV nodes.
4. Some tissues can spontaneously initiate action potentials. They are said to
exhibit automaticity.
5. Phases 2 and 3 run together in SAN and AVN and phase 1 is absent.

IV. IONIC BASIS FOR THE CARDIAC ACTION POTENTIAL

A. The movement of charge across the membrane (current flow) causes changes in
membrane potential. Inward movement of positive charge (inward current; e.g.,
influx of Na+ and Ca2+) causes depolarization. Outward movement of positive
charge (outward current; e.g., efflux of K+ or influx of Cl-) causes
hyperpolarization. A constant membrane potential means inward and outward
currents are equal.

411
Electrical Activity of the Heart - Dr. Baumgarten

B. The magnitude of current carried by an ion (I) is equal to the product of the
conductance (g) of the membrane for that ion and the driving force (i.e., how far
from electrochemical equilibrium is the distribution of the ion).
For K+: IK = gK (Em − EK)
For Na+: INa = gNa (Em − ENa)
1. Conductance is the reciprocal of electrical resistance; it is related to but not
identical to permeability and reflects the ability of ions to cross the membrane.
High conductance implies that it is easy for ions to cross the membrane.
2. Driving force is the difference between Em and the Nernst equilibrium
potential for the ion. Driving force is the electrochemical gradient, the sum
of the electrical and concentration (chemical) gradients, expressed in terms of
volts. The electrochemical gradient drives diffusion of ions across the
membrane.
3. Direction of current. By definition: (1) the direction of current flow is the
direction of flow of cations (K+ efflux = outward current; Cl– efflux = inward
current); (2) positive current is outward current (repolarizing or
hyperpolarizing) and negative current is inward current (depolarizing).
• The sign and direction of current are set by the sign of the driving force.
• K+: Em always is positive to EK. This means driving force (Em − EK) is
positive, and K+ current (IK) is positive (outward), corresponding to K+
efflux.
• Na+: Em always is negative to ENa. This means the driving force
(Em − ENa) is negative, and Na+ current (INa) is negative (inward),
corresponding to Na+ influx.

4. Nernst equilibrium potential represents the membrane potential at which the


electrical and chemical gradients are equal and opposite. At that potential,
influx and efflux of an ion are equal, i.e., there is no net flux and no current.
If Em were positive to EK, the electrical gradient drawing K+ in would be less
than the concentration gradient pushing K+ out, and a net K+ efflux (positive
current) would be observed. The Nernst equation is:
Equilibrium Potential = −2.303 RT/zF log [C]i/[C]o = −61/z log [C]i/[C]o
where R is the Gas Constant, T the temperature (°K), F Faraday's constant,
and z the valance (e.g., Na+ = +1, K+ = +1, Cl- = −1, Ca2+ = +2).
The approximate values of the equilibrium potentials for K+, Na+ and Ca2+ and
how changes in intra or extracellular concentrations affects the equilibrium
potentials are important.
EK = −61 log [K+]i/[K+]o = −61 log (150 mM / 4 mM) ≈ −100 mV
ENa = −61 log [Na+]i/[Na+]o = −61 log (15 mM / 150 mM) ≈ +60 mV
ECa = −61/2 log [Ca2+]i/[Ca2+]o = −61/2 log (10-7 M / 2×10-3 M) ≈ +110 mV

412
Electrical Activity of the Heart - Dr. Baumgarten

C. Membrane Conductances. Electrical activity is controlled by the voltage and


time- dependence of membrane conductance. Several types of channels with
differing selective ionic permeabilities have been identified; each channel type is
a different protein molecule. Each channel type has its own conductance term.
Current Effect on Em
Current Conductance Ion Equilib. Pot. During AP During AP
INa gNa Na+ +60 mV Inward Depolarizing
IK gK K+ −100 mV Outward Repolarizing
2+
ICa gCa Ca +100 mV Inward Depolarizing
IK1 gK1 K+ −100 mV Outward Repolarizing
Ito gto K+ −100 mV Outward Repolarizing
_______________________________________________________________

K+ Current Nomenclature. Some texts use 'IK' to include IK and IK1, but
separate membrane proteins are responsible. IK is similar to K+ current (delayed
rectifier) in nerve. Rapid (IKr), slow (IKs), and ultra-rapid (IKur, atria only)
activating components of delayed rectifier with distinct drug sensitivity are
present. IK1 behaves differently than IK in nerve and also is termed the inward-
going rectifier.
Electrogenic ion transport. Na+-K+ pump and Na+-Ca2+ exchange maintain the
ionic gradients. Both processes are electrogenic, meaning they produce a net
current. At Em, the Na+-K+ pump produces an outward (hyperpolarizing) current
(3 Na+ out/2 K+ in) and Na+-Ca2+ exchange an inward (depolarizing) current (1
Ca2+ out/3 Na+ in). The contribution of these currents to the action potential
configuration is relatively modest, but over the long term, maintenance of the
ionic gradients is critical.
D. Ionic Basis for Phases of Ventricular Action Potential. (See next Figure)
$ Phase 4 − Em is constant. Thus inward currents and outward currents are
equal. The potassium conductance (gK1) is high; other conductances are low.
(To see quantitatively why inward and outward currents are equal, remember
that the driving force for K+ is small and those for Na+ and Ca2+ are large.)
$ Phase 0 − Depolarization causes an increase (activation) of gNa. Because
driving force on Na+ is large and inwardly directed (Em − ENa = −85 − 60 =
−145 mV), Na+ rushes into the cell. This large INa causes further
depolarization. In turn, depolarization increases gNa even more, leading to a
further increase in INa. A rapid, regenerative (self-sustained) depolarization
results, and Em approaches ENa.
By late in phase 0, INa begins to decrease because: (1) depolarization causes
inactivation (decrease) of gNa, and (2) driving force on Na+ is reduced. At
the action potential peak, Em ≈ +40 mV, and (Em − ENa) = +50 − 60 = −10,
less than 10% of its value during phase 4.

413
Electrical Activity of the Heart - Dr. Baumgarten

$ Phase 1 − A rapid repolarization results when outward currents exceed


inward currents. INa continues to decrease because of inactivation (reduced
gNa). IK1 is slightly increased because of the large increase in K+ driving force
and despite the voltage-dependent decrease in gK1. At the peak of the action
potential, (Em− EK) is +140 mV, while during phase 4, (Em − EK) is +15 mV.
An additional outward current, Ito, contributes to phase 1 repolarization in
some parts of the heart including epicardial ventricular muscle and Purkinje
fibers. These tissues have a prominent phase 1 and a distinct notch between
phase 1 and 2. Ito is a K+ current. There also is evidence for a repolarizing
Cl- current (Cl- influx) during phase 1. .

414
Electrical Activity of the Heart - Dr. Baumgarten

$ Phase 2 − Inward and outward currents are nearly equal. Depolarization


causes:
1. activation (increase) followed by inactivation (decrease) of gCa; ICa is
much smaller than INa, about 1/50 of the amplitude. L-type Ca2+
channels are mainly responsible for the main plateau Ca2+ current.
2. slow activation (increase) in gK; IK is outward current.
During the plateau, total current becomes more outward (ICa decreases, IK
increases) and membrane potential slowly repolarizes.
$ Phase 3 − Outward K+ currents are significantly greater than inward
current (primarily ICa). gK1 increases rapidly giving the maximum outward
current observed. By the end of phase 3, gK1 has increased to its resting level
and gK slowly deactivates (decreases) to its resting level. As phase 3
progresses, the outward K+ currents tend to decrease in magnitude because of
decreases in driving force and conductance.

E. Important Tissue Differences in the Ionic Basis of the Action Potential.

1. SA and AV Nodes
a. INa is NOT responsible for phase 0. Instead, depolarization is caused by
regenerative activation of ICa (L- and T-type). Because the amplitude of
the current is smaller, the rate of depolarization is slower. T-type Ca2+
channels are more prominent in SAN than in other parts of the heart.
b. Automaticity is due to the sum of the currents becoming inward during
diastole. An increasing inward current carried primarily by Na+ is
responsible. This current is the pacemaker current, If. If is turned off by
depolarization during AP and is turned on by hyperpolarization (phase 4).
Depolarization initiated by If reduces a background K+ current and begins
to activate ICa as threshold potential is approached. If is prominent in both
SA and AV nodes.
2. His-Purkinje System
Em is normally constant during diastole in the His-Purkinje system. In the
absence of a sinus beat, however, Em can spontaneously depolarize causing
automaticity. If is responsible for normal automaticity in the His-Purkinje
system.

415
Electrical Activity of the Heart - Dr. Baumgarten

F. Role of currents in action potential

1. INa − Responsible for upstroke; Turns on during phase 0 and off during phases
0 & 1.
2. ICa − Responsible for inward current during plateau. Turns on early in phase
2 and slowly turns off in phase 2. Inactivation of ICa helps set action potential
duration. Note that ICa reflects the activity of 2 distinct channel types, T
(transient) and L (longer lasting). Most effects ascribed to Ca2+ channels are
due to L channels. L-type Ca2+ channels are modulated by classical Ca2+
channel agonists (norepinephrine) and antagonists (verapamil, acetylcholine).
T-type Ca2+ channels are more important in SAN than in other parts of the
heart.
3. IK1 − Responsible for resting potential. Decrease in gK1 upon depolarization
limits increase in IK1 otherwise caused by increased K+ driving force during
plateau.
4. IK− Slowly turns on during phase 2. Slow increase in IK helps set action
potential duration. There are several types of IK channels in heart. IKr
activates more rapidly than slowly activating IKs, and ultra-rapid IKur speeds
atrial repolarization. Some drugs distinguish between types.
5. Ito − Contributes to phase 1 repolarization, especially in epicardial ventricular
muscle and Purkinje fiber. Also contributes to heart rate-dependence of APD
in these tissues. Expression of Ito varies in different parts of the heart. At
least two different channels with different kinetics contribute to Ito.
6. ICl - At least 3 distinct, Cl– currents have been identified in heart. A Ca-
dependent Cl- current contributes to Ito (Ito2). These currents can modulate
action potential configuration.

Action potential duration, primarily the length of phase 2, is set by the slow turn
on (activation) of IK and slow turn off (inactivation) of ICa. These events occur
during phase 2 and initiate the rapid repolarization of phase 3. The outward K+
currents are greatest during phase 3 when the rate of repolarization is greatest.
APD decrease as HR increases because of slow changes in several currents
including IKr, IKs, and ICa-L.

Why is there a plateau in heart but not in nerve? (1) ICa provides the inward
current to maintain the depolarization. ICa is absent or insignificant in nerve; and
(2) the decrease in gK1 on depolarization limits outward current and makes it
easier to maintain a plateau. In nerve, gK1 does not decrease. Thus, outward IK1 is
much larger and repolarization occurs within msec.

416
Electrical Activity of the Heart - Dr. Baumgarten

V. IMPULSE FORMATION

A. Normal automaticity is the intrinsic ability of specialized cells of the heart to


spontaneously depolarize during diastole (also termed phase 4 depolarization)
and to initiate an action potential.

B. Cells exhibiting normal automaticity are found in:


1. SAN
2. Certain specialized peri-nodal atrial fibers.
3. AVN (N-H region, i.e., that portion adjacent to His Bundle)
4. Bundle of His
5. Bundle Branches
6. Purkinje fibers.
C. The pacemaker with the fastest rate will control heart rate.
Normally, the hierarchy is: SAN (fastest), AVN, Bundle of His, Bundle Branches,
Purkinje fibers (slowest). That is, as one moves distally in the sequence of
activation, the intrinsic rate of automaticity slows. However, disease processes
and/or autonomic tone can modify this order. Upon blocking or abolishing the
SAN impulse, the AVN usually takes over.
D. Cells that do not normally exhibit normal automaticity are termed latent
pacemakers (B(2) to B(6), above). Their automaticity is suppressed by the
normal heart beat or by electrical stimulation. This suppression of latent
pacemakers is termed overdrive suppression. Latent pacemakers are also referred
to as ectopic pacemakers because the site of initiation of the heart beat is outside
SAN.

417
Electrical Activity of the Heart - Dr. Baumgarten

E. To initiate a spontaneous beat, a pacemaker cell must depolarize from the


maximum diastolic potential to the threshold potential. That means that the rate
of firing of pacemakers is dependent on three factors that control
1. Maximum diastolic potential.
2. Threshold potential.
3. Rate of diastolic (phase 4) depolarization.

The rate of firing reflects diastolic events and threshold potential. Because APD is much
shorter than phase 4 (diastole), changes in APD do not significantly alter automaticity.
Threshold potential. Potential at which membrane generates a net inward current (i.e.,
the inward current becomes larger than the outward current), and the depolarization
becomes self-sustained and gives rise to the upstroke.
Some characteristics of threshold potential. Threshold potential is NOT the voltage at
which Na+ (or Ca2+ in SAN or AVN) channels suddenly switch on. Rather, Na+ (and
Ca2+) channels begin to open at voltages far negative to the threshold potential. The
fraction of resting channels that open increases gradually as the membrane is
depolarized. This implies that interventions that alter the number of Na+ channels in the
resting state also alter threshold potential. For example, if an intervention reduced the
number of Na+ channels in the resting state, a greater fraction of the remaining Na+
channels must open to overcome the outward current; a greater depolarization would be
required, and threshold potential would shift to a more positive potential. Because Em
controls the number of resting Na+ (e.g., in Purkinje fibers) and Ca2+ channels (in
SAN and AVN), threshold potential usually moves in the same direction as Em.

418
Electrical Activity of the Heart - Dr. Baumgarten

VI. EXCITABILITY

A. Excitability reflects the ability of heart muscle to initiate an action potential in


response to depolarizing current. Threshold is the amount of current needed to
depolarize from Em to the threshold potential.

B. Excitability changes during the action potential.

1. Absolute Refractory Period (ARP): Stimulation is not possible.


2. Relative Refractory Period (RRP): Begins after ARP. An action potential
can be elicited, but a larger current is needed than at rest. During the first part
of the RRP, within the ERP, action potentials generate a local response but
fail to conduct.
3. Effective Refractory Period (ERP): Longer than ARP. A conducted action
potential cannot be elicited.
4. Supranormal Period (SNP): Current required for excitation is less than
required at rest. Threshold potential returns to control value just before
repolarization is complete. The SNP is most prominent in Purkinje fibers.

C. Recovery of the ability to generate an inward current is essential to recovery of


excitability. Recovery of the Na+ channel (or Ca2+ channel in SAN or AVN) from
the Inactivated (I) to the Closed but Available (C) state requires repolarization.
During the cardiac cycle, changes in membrane potential, threshold potential, and
membrane resistance contribute to the time-dependence of excitability. Except
for SAN and AVN, recovery of excitability closely follows repolarization. In
SAN and AVN recovery of excitability lags further behind repolarization because
the slow kinetics of Ca2+ channels.

419
Electrical Activity of the Heart - Dr. Baumgarten

VII. IMPULSE CONDUCTION

A. Conduction in heart occurs via local circuit currents as in unmyelinated nerves.

rm = membrane resistance
i
r = intracellular resistance
for cell-to-cell current flow

1. Inward current generated by INa (or ICa in SAN or AVN) depolarizes the
membrane.
2. Depolarizing current spreads down the fiber from cell-to-cell causing distant
membrane to depolarize Depolarization alters the charge stored by membrane
capacitance.
3. Current loop is completed in the extracellular space.
4. When a distant patch of membrane is sufficiently depolarized, it also
generates inward current, and conduction continues. This requires
depolarization to threshold potential.
5. Conduction velocity is NOT related to action potential duration (APD).
Conduction velocity reflects the time it takes for excitation to spread. APD
reflects the time depolarization persists once a cell is excited.

B. Determinants of Conduction Velocity

1. Amplitude of inward current that generates local circuit current flow.


Smaller inward currents will depolarize distant membrane more slowly and,
thereby, slow conduction velocity.
2. Passive properties - the resistive and capacitive properties of the membrane
and cell-to-cell junctions determine the spread of current. Increased gap
junction resistance (Ri), referred to as electrical uncoupling, makes it more
difficult to depolarize distant membrane and, thereby, slows conduction
velocity.
Rm = membrane resistance
Cm = membrane capacitance
Ri = intracellular (cell-to-cell) resistance
3. Excitability − the amount of current required to reach threshold potential.
Decreased excitability means a larger amplitude current (or the same current
for a longer time) is required to depolarize to threshold potential; decreased
excitability slows conduction velocity.

420
Electrical Activity of the Heart - Dr. Baumgarten

VI. RESPONSIVENESS

A. The maximum rate of rise of phase 0 of the action potential (dV/dt or V max) is
dependent on the membrane potential at the moment of excitation.

B. dV/dt is a rough measure of the amplitude of the inward current during phase 0
(INa in atria, ventricle and Purkinje fibers or ICa in SA and AV node).
C. After depolarization, recovery to a more negative potential is necessary to reprime
Na+ (or Ca2+) channels. Depolarization can again elicit an increase in
conductance only after recovery.

D. All other things being equal, dV/dt is roughly directly proportional to both
conduction velocity and excitability.
E. The voltage-dependence inactivation also explains why changes in Em cause
threshold potential to shift in the same direction.

421
Electrical Activity of the Heart - Dr. Baumgarten

IX. REFERENCES

A. Koeppen, B.M. and Stanton, B.A. Berne & Levy Physiology, 6th Ed., 2008. pp
292-300.
B. Costanzo, L.S. Physiology, 3rd, Saunders, 2006. Chapter 4, pp. 125-136.

5-electrical activity of the heart-2009.doc 8/28/2008

422
Microcirculation - Dr. Pittman

Microcirculation
Roland Pittman, Ph.D.

OBJECTIVES:

1. Associate the structure and function of the microcirculation.


2. State the determinants of diffusion of small solutes across the capillary wall.
3. Compare and contrast the permeability of the capillary wall to water-soluble and
lipid-soluble solutes.
4. State the four pressures that determine capillary exchange of water, according to
Starling’s hypothesis of fluid exchange.
5. Quantify the exchange of fluid between the intravascular and interstitial spaces
according to Starling’s hypothesis.
6. Identify the characteristics of the lymphatic system, including the determinants of
lymph flow.

SUGGESTED READING ASSIGNMENT

R. M. Berne, B. M. Koeppen, M. N. Levy, and B. A. Stanton, Physiology, 5th Ed., St.


Louis: Mosby, pp. 368-379, 2004.

L. S. Costanzo, Physiology, 3rd Ed. Philadelphia: W.B. Saunders, pp. 163-166, 2006.

I. STRUCTURE AND FUNCTION OF THE MICROCIRCULATION

A. Structure: Types of microvessels

423
Microcirculation - Dr. Pittman

Figure 1. Composite schematic drawing of the microcirculation. The circular structures on


the arteriole and venule represent smooth muscle fibers; branching solid lines represent
sympathetic nerve fibers. The arrows indicate the direction of blood flow.

B. Function

The primary function of the circulatory system is to exchange substances


between blood and tissue. The exchange processes take place in the
microcirculation. The classes of vessels playing a role there are the arterioles
(resistance vessels which regulate flow), capillaries (the primary exchange
vessels) and venules (exchange and collecting vessels). The amount of flow
through the capillaries appears to be regulated to maintain adequate tissue
oxygenation. This regulation appears to be accomplished in part by the
elaboration of tissue metabolites which affect the flow of blood through
precapillary vessels.

II. TRANSCAPILLARY EXCHANGE OF SOLUTES

A. Diffusion

This passive mechanism of transport is a rapid and efficient mode of

424
Microcirculation - Dr. Pittman

exchange over the small distances (tens of :m) between the blood supply
(capillaries) and tissue cells.

Figure 2.

Fick's first law describes the net rate of transfer of a substance from a
location of higher concentration to one of lower concentration:

ΔN/Δt = DA (Δc/Δx) = PA Δc

ΔN/Δt = number of moles of substance exchanged per unit time

D = diffusion coefficient for substance through the capillary wall

A = surface area available for diffusion (α number of perfused capillaries)

Δc = concentration difference across capillary wall = c(blood) - c(ISF)

Δx = thickness of the capillary wall (~ 1 :m)

P = permeability of the capillary wall defined as D/Δx

B. Permeability characteristics of the capillary wall

1. The wall is composed of a single layer of endothelial cells about l :m


thick.
2. For lipid soluble substances (e.g., oxygen), the entire wall surface is
available for diffusion.
3. For water soluble substances (e.g., glucose), there are small aqueous
pathways equivalent to cylindrical pores 80 to 90 Å in diameter
through which they may pass. Total pore area is about 1/1000 (i.e.,
0.1%) of the surface area of a capillary.
4. The permeability of the wall to a particular substance depends upon
the relative size of the substance and the pore ("restricted" diffusion).

425
Microcirculation - Dr. Pittman

Figure 3.

C. The amount of a substance which is exchanged can be increased by the


opening of more capillaries - this increases the surface area available for
exchange. Normally only a fraction (about 1/3 to 1/2) of the capillaries in a
given tissue are being perfused at any given moment. During times of
increased demand for nutrients (e.g., heart and muscle tissue during exercise)
more can be opened. Whether a given capillary is open or closed depends on
the contractile state of a region of smooth muscle (probably a terminal
arteriole) located near the entrance to a capillary.

III. TRANSCAPILLARY EXCHANGE OF WATER

A. The processes whereby water passes back and forth across the capillary
wall are called filtration and absorption. The flow of water depends upon
the relative magnitude of hydraulic and osmotic pressures across the
capillary wall.

B. Fluid compartments of "average" adult (70 kg person)

1. Extracellular space (19 L)

a. Plasma space (3 L)
b. Interstitial space (16 L)

2. Intracellular space (23 L)


3. Compartmental exchanges

426
Microcirculation - Dr. Pittman

Figure 4.

B. Why doesn't all the water leak out of the capillaries?

1. The effective "diameter" of a water molecule is about 2 Å, whereas the


effective "diameter" of the transendothelial pathways is about 80 Å.
2. The mean hydraulic pressure inside a capillary is about 25 mmHg
higher than that outside the capillary.

Figure 5.

In this diagram Ra and Rv represent the precapillary and postcapillary


resistances to blood flow, respectively. Since Rv/Ra is about 1/5, a given
change in Pv has much more impact on Pc than the same change in Pa.

3. Since the facts noted above in items 1 and 2 suggest that water should
leave the vascular system, why in reality is there no large net flow of
water from the vascular space to the interstitial space?

427
Microcirculation - Dr. Pittman

4. Proteins in plasma (primarily albumin and globulins) are too large to


cross the capillary wall which for them behaves like a semipermeable
membrane (i.e., the reflection coefficient, σ, is near 1) separating blood
from interstitial fluid. Thus, there is a net osmotic pressure established
between plasma and interstitial fluid and this tends to prevent the
leakage of water from the vascular compartment.
5. Note that the osmotic pressure of either plasma or interstitial fluid can
be written as the sum of three terms:

Π(total) = Π(electrolytes) + Π(nonelectrolytes) + Π(proteins)

Although small electrolytes and nonelectrolytes are osmotically active


particles, they can readily pass across the capillary wall (σ ≈ 0) and
their concentrations are approximately equal on both sides of the
capillary. Thus, no net osmotic pressure difference is created by these
substances and only the plasma proteins are responsible for the
observed net osmotic pressure. This quantity is generally referred to as
the colloid osmotic pressure or oncotic pressure.
6. Starling's hypothesis (1896) combines all this information and states
that in the steady state there is a delicate balance between hydraulic
and osmotic pressures which leads to little or no net flow of water.
Algebraically this is expressed by the following equation:

F = K {(Pc - PISF) - (Πpl -ΠISF)} = K {ΔP-ΔΠ}

F = rate of fluid flow across the capillary wall

K = capillary filtration coefficient, or hydraulic conductance


(α permeability to water x perfused capillary surface area)

Pc = capillary hydraulic pressure (32 mmHg to 15 mm Hg).

PISF = hydraulic pressure in ISF (0 mmHg)

Πpl = osmotic pressure due to plasma proteins (28 mmHg)

ΠISF = osmotic pressure due to proteins in ISF (5 mmHg)

If F > 0, water is filtered from blood into ISF.

If F < 0, water is absorbed into blood from ISF.

428
Microcirculation - Dr. Pittman

Figure 6. Schematic representation of the factors responsible for filtration and absorption
across the capillary wall and the formation of the lymph.

IV. THE LYMPHATIC SYSTEM

A. In 24 hours more fluid is filtered than is reabsorbed; 20 L/day are filtered


and 16 L/day are reabsorbed by the capillaries. The overflow is carried
back to the vascular system (via the superior vena cava) by the lymphatic
circulation.

B. Characteristics of the lymphatic system

1. There are a large number of small vessels whose ends are closed.
2. Flap valves (similar to those in veins) provide for unidirectional
flow back to the cardiovascular system.
3. The smallest (terminal) vessels are very permeable, even to
proteins which occasionally leak from systemic capillaries.

C. Lymph flow is determined by:

1. Interstitial fluid pressure (↑ PISF → ↑ Qlymph)


2. The lymphatic "pump" (moves fluid from the extremities to the
central circulation)

a. One-way flap valves (produce unidirectional flow)


b. Skeletal muscle contraction (periodically squeezes fluid in
lymphatics)
c. Tissue compression (squeezes fluid in lymphatics)

429
Microcirculation - Dr. Pittman

d. Periodic (~ 5/min) lymphatic smooth muscle contraction in


response to stretch

D. Control of ISF protein concentration is one of the most important


functions of the lymphatic system.

E. Edema formation: If more net fluid is filtered than can be handled by the
lymphatics, the volume of interstitial fluid increases. This fluid
accumulation is called edema. This circumstance is important clinically
since solute exchange (e.g., oxygen) decreases due to the increased
diffusion distances produced when the accumulated fluid pushes the
capillaries, tethered to the interstitial matrix, away from each other.

STUDY QUESTIONS

1. Which of the following will produce increased capillary hydraulic pressure?

1. Increased venous pressure.


2. Decreased venous resistance.
3. Increased arterial pressure.
4. Increased arterial resistance.

ANSWER: B (1 & 3)

You need to remember the four factors that determine capillary hydraulic pressure:
arterial and venous pressures and resistances. You can determine the effect of each
one either by the equation from class or by thinking intuitively about what would
happen as a consequence of each change.

2. Which of the following statements is/are true about capillary exchange of solutes?

1. Passage of water-soluble substances is generally restricted to small channels


between adjacent endothelial cells.
2. The aqueous channels that allow the passage of substances like glucose
comprise a surface area of about 10% of the capillary wall.
3. Lipid soluble substances can generally pass through the entire capillary wall.
4. Proteins generally pass across the capillary wall by active transport.

ANSWER: B (1 & 3)

You need to remember that small water-soluble molecules are restricted to pass
through aqueous channels between adjacent endothelial cells that make up about
0.1 % of the surface area of the capillary wall. Lipid soluble substances have access
to the entire capillary wall and proteins are generally restricted to the intravascular
space.

430
Microcirculation - Dr. Pittman

3. Which of the following will produce increased lymph flow?

A. Decreased venous pressure.


B. Decreased venous resistance.
C. Decreased plasma protein concentration.
D. Increased arterial resistance.
E. Decreased interstitial fluid pressure.

ANSWER: C

Decreased venous pressure and decreased venous resistance will both result in lower
capillary hydraulic pressure, and hence less filtration and lower lymph flow.
Decreased plasma protein concentration will produce lower osmotic pressure in the
plasma and will produce more filtration and hence increased lymph flow. Increased
arterial resistance will lower capillary hydraulic pressure and lead to less filtration
and lymph flow. Decreased interstitial fluid pressure (the driving force for fluid
entry into the lymphatics) will lead to decreased lymph flow.

4. Consider a typical capillary in which the following values are observed:

hydraulic pressure at the beginning of a


= 30 mm Hg
capillary
hydraulic pressure at the end of a capillary = 15 mm Hg
interstitial hydraulic pressure = 3 mm Hg
interstitial colloid osmotic pressure = 1 mm Hg.

For what value of plasma colloid osmotic pressure will there be absorption of fluid all
along the capillary?

A. 20 mm Hg.
B. 22 mm Hg.
C. 24 mm Hg.
D. 26 mm Hg.
E. 28 mm Hg.

ANSWER: E

In order to have fluid absorption all along the capillary, the net pressure at all
points along the capillary must be < 0. The pertinent combination of pressures is
[(PC – PISF) – (ΠPL - ΠISF)]. (PC – PISF) represents the net filtration pressure due to
the hydraulic pressures inside and outside the capillary, respectively. (ΠPL - ΠISF)
represents the net absorption pressure due to the protein osmotic (oncotic)
pressures inside and outside the capillary, respectively. To ensure that there will be
absorption all along the capillary, it is necessary to make sure that the net pressure
at the entrance to the capillary (arterial end) is < 0.

431
Microcirculation - Dr. Pittman

Plugging in the numbers, we require that (30 – 3) – (ΠPL – 1) = 27 + 1 - ΠPL < 0. So,
if ΠPL > 28, there will be absorption all along the capillary.

432
Peripheral Circulation and its Control - Dr. Pittman

Peripheral Circulation and its Control


Roland Pittman, Ph.D.

OBJECTIVES:

1. Compare and contrast global and local regulation of blood flow.


2. Define autoregulation of blood flow, reactive hyperemia and active hyperemia.
3. Describe the myogenic hypothesis/mechanism of local blood flow regulation.
4. Describe the sequence of events in response to a change in blood pressure in the
resistance vessels according to the myogenic mechanism.
5. Describe the metabolic hypothesis/mechanism of local blood flow regulation.
6. Describe the sequence of events in response to a change in blood flow or
metabolic activity according to the metabolic mechanism.
7. List five mediators of the metabolic mechanism of local flow control.

SUGGESTED READING ASSIGNMENT

R. M. Berne, B. M. Koeppen, M. N. Levy, and B. A. Stanton, Physiology, 5th Ed., St.


Louis: Mosby, pp. 380-387, 2004

L.S. Costanzo, Physiology, 3rd Ed. Philadelphia: W.B. Saunders, pp. 166-168, 2006.

I. REGULATION OF BLOOD FLOW

A. The cardiovascular system controls blood flow to individual organs (1) by


maintaining arterial pressure within narrow limits by the mechanisms
designed to regulate Pa and (2) by allowing each organ to adjust its vascular
resistance (R) to blood flow to an appropriate value. The cardiac output (CO)
is distributed among the various organs according to their respective
resistances so that flow (Q) in an organ is given by:

Q = (TPR/R) CO

B. There are three major mechanisms that control the function of the
cardiovascular system: local, neural and humoral. They can work
independently of each other, but there are also interactions among them. It is
important to recognize that the vasoregulation occurs in the resistance vessels.

II. LOCAL REGULATION OF BLOOD FLOW

A. General

433
Peripheral Circulation and its Control - Dr. Pittman

1. In addition to neural and humoral mechanisms for regulating the


function of the cardiovascular system, there are mechanisms intrinsic
to the various tissues which can operate independently of
neurohumoral influences.
2. Local regulatory processes allow each tissue in the body some measure
of autonomy to satisfy its current and particular requirement.
3. Because the various organs and tissues of the body are connected in
parallel, it is fairly easy to see that the cardiac output can be
redistributed among the tissues should their relative needs change.

B. Site of local regulation: the microcirculation

1. The microcirculation is comprised of a network of blood vessels --


arterioles, capillaries and venules -- whose functions are regulation of
tissue perfusion and exchange of substances between blood and tissue.
2. Although the topology of vascular networks is in general quite
complex, as a first approximation we can think of most networks in
terms of a collection of microcirculatory "units" connected in parallel,
where each unit is composed of a feeding arteriole, several capillaries
and a collecting venule.
3. Because of the parallel structure of the network, it is possible to
redistribute blood flow from one region to another within a tissue to
accommodate any alterations in local needs.

C. Examples of local control processes

1. Autoregulation of blood flow

The term "autoregulation" in this context refers to the tendency for


organ blood flow to remain constant in the face of local changes in
arterial pressure. Autoregulation is observed in virtually every vascular
bed. It is most pronounced in brain and kidney and is prominent in the
heart, skeletal muscle, intestine and liver. Recall that flow (Q) equals
perfusion pressure (ΔP = Pa-Pv) divided by vascular resistance (R) so
that as ΔP rises through the autoregulatory range (Pa = 80-160 mmHg
in brain and kidney), R must increase to maintain constant flow. This
relation is diagrammed below.

434
Peripheral Circulation and its Control - Dr. Pittman

Figure 1. Pressure-flow relationship in the skeletal muscle vascular bed of a dog. The closed
circles represent the flows obtained immediately after abrupt changes in perfusion pressure
from the control level (point where lines cross). The open circles represent the steady-state
flows obtained at the new perfusion pressure. (Redrawn from Jones RD. Berne RM: Circ Res
14:126, 1964.)

2. Reactive hyperemia

Hyperemia is literally an excess of blood in a region. Reactive


hyperemia refers to the elevated blood flow observed in an organ
following a period of circulatory arrest (i.e., occlusion of blood
supply). The magnitude of the hyperemia is related both to the
duration of the occlusion period and to the preocclusion blood flow.
An example of this phenomenon is diagrammed below.

Figure 2. Reactive hyperemia in the hind limb of the dog after 15-, 30-, and 60-second
occlusions of the femoral artery.

435
Peripheral Circulation and its Control - Dr. Pittman

3. Active (or functional) hyperemia

Active hyperemia is the increase in blood flow which accompanies an


increase in the metabolic activity of an organ. It has been described in
skeletal and cardiac muscle, brain, intestine, stomach, salivary glands,
kidney and adipose tissue. The name of the hyperemia depends upon
the specific function of the tissue (e.g., contraction hyperemia for
muscle or secretory hyperemia for various glands).

D. Theories of local regulation

1. Two major hypotheses/mechanisms have been proposed to account for


the local regulatory phenomena described above. Although proponents
of these hypotheses have often presented them as competing theories,
in any given tissue some combination of the two mechanisms is
probably operative.

2. Myogenic hypothesis/mechanism:

The myogenic hypothesis/mechanism, in essence, states that vascular


smooth muscle actively contracts in response to stretch. Thus, blood
vessels exposed to an increased intravascular pressure will become
passively distended. The smooth muscle in the vessel wall responds by
active contraction (constriction) which tends to return vascular caliber
to below its original value.

There are several pieces of evidence which support the myogenic


hypothesis/mechanism:
a. Brief periods of arterial occlusion ultimately lead to substantial
vasodilation. The sequence of events is the following. First, the
arterial occlusion causes intravascular pressure to fall
downstream of the occlusion. The resulting passive reduction
in vascular caliber causes "active" compensatory dilation (i.e.,
reduction in VSM activity).
b. Reduction of the external pressure around an organ ultimately
leads to sustained vasoconstriction. The sequence of events is
the following. First, reduction in external pressure causes the
transmural pressure (PT= Pintravascular - Pextravascular) to increase,
producing passive dilation. This dilation, in turn, leads to active
smooth muscle contraction and vessel constriction.
c. Elevation of venous pressure ultimately leads to a sustained
increase in precapillary resistance in some organs. The
sequence of events is the following. First, the elevated venous
pressure is transmitted through the capillary network to the
arterioles where active VSM contraction occurs in response to
the passive distension.

436
Peripheral Circulation and its Control - Dr. Pittman

d. It has been postulated that specialized stretch or tension


receptors exist in the walls of resistance vessels. After the
stretch (either increase or decrease) is sensed, this information
is communicated to the vascular smooth muscle cells and they
respond to maintain constant wall tension.
e. The following figures show the basic elements in the myogenic
hypothesis/mechanism and how they can be used to describe
autoregulation and reactive hyperemia.

Figure 3.

437
Peripheral Circulation and its Control - Dr. Pittman

Figure 4.

3. Metabolic hypothesis/mechanism

The metabolic hypothesis/mechanism states that there is a link


between blood flow and tissue metabolism. It has usually been more
specialized to suggest a link between oxygen supply and demand
according to the diagram below.

438
Peripheral Circulation and its Control - Dr. Pittman

Figure 5.

Tissue cells continuously utilize ATP as an energy source to sustain


cellular function. The two most common ways in which ATP can be
produced are by oxidative phosphorylation and glycolysis. "Ox-phos"
is the preferred way to generate ATP, so that cells have a continuous
need for oxygen. In the presence of an adequate supply of oxygen
(normoxia), the ADP produced from hydrolysis of ATP is
rephosphorylated as part of oxidative phosphorylation, and the
contribution of glycolysis to ATP production is negligible.

When the supply of oxygen decreases below normal (hypoxia), not all
of the ADP is rephosphorylated by ox-phos and some is degraded
further to AMP and then to adenosine. Adenosine is a powerful
vascular smooth muscle relaxant (i.e., produces vasodilation) and the
amount of it produced is tightly linked to the degree of hypoxia.
During hypoxia, glycolysis is stimulated and some of the lost ATP
production is made up through this metabolic pathway. The end
product of glycolysis, lactic acid, dissociates into hydrogen ion and
lactate, both of which also have vasodilator properties. A general
principle then is that cells continuously produce metabolic wastes
(e.g., adenosine, hydrogen ion, lactate) many of which are vasoactive
(usually vasodilator). Metabolite production occurs at a low level,
even under normoxic conditions.

There is a close linkage between metabolite production and tissue


oxygen concentration, [O2]T, as illustrated in the following diagram.
As [O2]T decreases, there is a large increase in metabolite production.

439
Peripheral Circulation and its Control - Dr. Pittman

The main reason responsible for the decrease in metabolite production


with increases above normal in [O2]T is that some small fraction of
most tissues are slightly hypoxic at any moment, but temporal
variations in the regional distribution of tissue perfusion do not allow
situations of chronic hypoxia to develop. Under normal conditions
there is a balance between oxygen supply and demand, but imbalances
give rise to local adjustments in blood flow that bring supply back in
register with demand.

Figure 6.

The following oxygen-linked metabolites have been implicated as


potential chemical mediators in the metabolic hypothesis/mechanism:
adenosine (from ATP hydrolysis: ATP → ADP → AMP → adenosine)
and hydrogen and lactate ions (from lactic acid generated by
glycolysis). Their levels are increased when there is a reduction in
oxygen supply relative to demand (i.e., tissue hypoxia). The
production of more carbon dioxide as a result of increased tissue
activity (leading to increased oxidative metabolism) leads to
vasodilation through increased H+. Increased potassium ion and
interstitial fluid osmolarity (i.e., more osmotically active particles)
cause vasodilation under physiological conditions associated with
increased tissue activity.

Two of the other boxes in the block diagram on the previous page
deserve further description. "Oxygen Delivery, QO2 = Q [O2]a" refers
to the convective flow of oxygen in the arterial blood. Thus, oxygen is
delivered by bulk flow of blood (flow = Q) to the exchange vessels
(i.e., capillaries) by virtue of its presence in the blood at concentration
[O2]a. Hence, increasing blood flow will increase the delivery of
oxygen via the blood to the tissues.

"Metabolite Washout" is the other box that plays a key role in


determining the concentration of vasodilator metabolites in the
interstitium, [Vasodilator]ISF. The concept of metabolite washout can
be appreciated by considering the movement of the vasodilators

440
Peripheral Circulation and its Control - Dr. Pittman

produced in tissue cells. They diffuse away from their sites of


production, through the interstitial fluid, and across the walls of the
nearby capillaries (these molecules are generally small enough to pass
through the aqueous channels in the capillaries; and most cells have at
least one capillary near them). Once the metabolite enters the capillary,
it is "washed away" by the blood flowing through the capillary, hence
the term "metabolite washout."

It is the summation of metabolite production and metabolite washout


that determines the concentration of vasodilators in the ISF that is in
contact with the nearby arterioles that control blood flow. Increases in
metabolite concentration thus cause vascular smooth muscle
relaxation, lowering the resistance to blood flow.

Consider exercising skeletal muscle as an example. With the onset of


exercise, metabolite production and oxygen requirements increase.
The metabolites diffuse away from the sites of production and reach
the vasculature. Vasodilation ensues, lowering resistance to blood
flow. The resulting increase in blood flow increases the oxygen supply
and finally a new steady state is achieved in which oxygen supply and
demand are matched. This scenario operates for other tissues in which
metabolic activity changes.

The following figures show how the metabolic hypothesis describes


autoregulation, reactive hyperemia and active hyperemia. The
vertically oriented arrows near each box indicate whether the quantity
in the nearest box increases (up arrow) or decreases (down arrow).

Figure 7.

441
Peripheral Circulation and its Control - Dr. Pittman

Figure 8.

442
Peripheral Circulation and its Control - Dr. Pittman

Figure 9.

STUDY QUESTIONS

1. In the diagram below the systemic arterial pressure and the blood flow through an
organ are plotted as a function of time. Which of the following statements is/are
true about this situation?

443
Peripheral Circulation and its Control - Dr. Pittman

1. Between 1 and 2 minutes, there is a passive dilation of the arterioles.


2. Between 1 and 2 minutes, there is an accumulation of vasodilator metabolites,
according to the metabolic hypothesis of flow control.
3. After 4 minutes, the diameters of arterioles begin to increase, according to the
myogenic hypothesis of flow control.
4. At 3 minutes, resistance to blood flow is lower than it was between 0 and 1
minute.

ANSWER: C (2 & 4)

You should recognize this as the example of reactive hyperemia. After a brief period
of occlusion, organ blood flow rises above control before returning to the control
baseline value. During the occlusion, there is passive reduction in vessel diameter
and vasodilator metabolites accumulate in the tissue. When flow is restored by
releasing the occlusion, vasodilation occurs (resistance to flow decreases) and after 4
minutes (in this example) resistance vessel (arteriolar) diameter decreases toward
control.

2. According to the metabolic hypothesis of blood flow control, which of the


following events is/are associated with the local regulatory response to increased
organ perfusion pressure?

1. Increased washout of vasodilator metabolites.


2. Increased arteriolar wall tension.
3. Increased supply of oxygen to tissue cells.
4. Constriction of resistance vessels due to increased sympathetic activity.

ANSWER: B (1 & 3)

This question describes the example of autoregulation: approximate maintenance of


flow despite increased perfusion pressure. The explanation of this phenomenon
offered by the metabolic hypothesis is that increased flow washes out vasodilator
metabolites produced by tissue metabolism and increases the supply of oxygen to

444
Peripheral Circulation and its Control - Dr. Pittman

the tissue.
Although wall tension passively increases in response to the pressure increase, wall
tension is not a part of the explanation in the context of the metabolic hypothesis (it
is for the myogenic hypothesis, however). Sympathetic nervous activity does not
play a role in these locally mediated responses.

3. Which of the following substances would be expected to reduce arteriolar


resistance?

1. Angiotensin II
2. Adenosine
3. Norepinephrine
4. Lactate

ANSWER: C (2 & 4)

Adenosine and lactate are resistance vessel relaxants, thereby reducing arteriolar
resistance. Angiotensin II and norepinephrine are vasoconstrictors.

4. Consider an organ in which the perfusion pressure suddenly decreases from 100
to 60 mmHg. Which of the following statements about regulation of blood flow is
correct?

A. According to the metabolic hypothesis, there will be an increased


accumulation of vasodilator metabolites.
B. According to the myogenic hypothesis, the initial passive response to the fall
in pressure will be a rise in resistance vessel wall tension.
C. The endothelial cells should release a vasoconstrictor in response to the initial
fall in flow.
D. If there is no change in the number of blood vessels perfused, then the
velocity of blood in the capillaries should remain unchanged.
E. The situation depicted in this question is a classic example of reactive
hyperemia.

ANSWER: A

The sudden decrease in perfusion pressure (or arterial pressure) should produce an
immediate fall in blood flow and is the scenario of autoregulation. The metabolic
hypothesis predicts that there will be: (1) a decrease in oxygen delivery (leading to
an increase in the production of vasodilator metabolites) and (2) a decrease in the
washout of vasodilator metabolites from the interstitium. Either of these events will
lead to an accumulation (increase) in the concentration of vasodilators in the vicinity
of the arterioles. In the context of the myogenic hypothesis, the reduction in
perfusion pressure will lead initially to a passive decrease in arteriolar diameter
(less pressure to keep the vessel expanded) and reduction in wall tension (T = Pr/w).
The endothelial cells should release less nitric oxide (a vasodilator) in response to

445
Peripheral Circulation and its Control - Dr. Pittman

the fall in flow. The reduced blood flow through the same number of capillaries
implies that the velocity of blood through them will be smaller.

446
Autonomic Reg of Card Function - Dr. Baumgarten

Autonomic Regulation of Cardiac Function


Clive M. Baumgarten, Ph.D.

OBJECTIVES:

1. Describe the autonomic innervation of the heart


2. Describe the ionic basis for chronotropic, dromotropic, and inotropic effects in
various cardiac tissues
3. Describe the role of adrenergic receptors and G proteins in adrenergic (β1) and
muscarinic regulation of cardiac function

I. AUTONOMIC INNERVATION AND EFFECTS

A. Parasympathetic innervation by the vagus nerve (X cranial n.) is primarily


supraventricular, that is, to the SAN, atria, and AVN. (A few parasympathetic
fibers also innervate ventricular muscle and Purkinje fiber innervation.)
Acetylcholine (ACh) released by postganglionic parasympathetic nerves terminals
interacts with muscarinic (M2) receptors in the heart. These receptors are
blocked by atropine.

B. Sympathetic innervation via the superior cervical ganglion supplies all of the
heart. Norepinephrine (NE) is released by post-ganglionic nerve terminals and
interacts with β1 receptors. β1 receptors are blocked by non-selective β-blockers
such as propranolol and cardioselective (i.e., β1) blockers including metoprolol
and practolol.

C. Sympathetic and parasympathetic nerves are usually antagonistic in their


modulation of cardiac activity. For example, a decrease in parasympathetic tone
usually has the same effect as an increase in sympathetic tone. In some cases,
however, one branch dominates. At rest the heart is under parasympathetic
control; blocking both autonomic inputs results in an increase in heart rate.
Functionally, parasympathetic withdrawal (i.e., a decrease in parasympathetic
tone) is a prompt and important regulator of cardiac function. The main
autonomic effects are on:
1. Heart Rate − Chronotropic Effect
2. Conduction Velocity of Action Potential − Dromotropic Effect
3. Muscle Contractility − Inotropic Effect
Positive effects imply increases (i.e., in heart rate), while negative effects imply
decreases.

447
Autonomic Reg of Card Function - Dr. Baumgarten

D. Action of autonomics largely can be explained by considering the effects of ACh


and NE on three ionic currents (+, increase; –, decrease):

Current NE ACh
IK-ACh 0 +
ICa (L-type) + −
If + −

IK-ACh is an IK1-like current activated by ACh and adenosine.

II. CHRONOTROPIC EFFECTS

A. SA Node

1. Sympathetic Stimulation (NE) increases HR = + Chronotropic effect:


a. Increased If causes increased rate of phase 4 depolarization.
b. Increased ICa makes threshold potential more negative.
2. Parasympathetic Stimulation (ACh) slows HR = – Chronotropic effect.
a. Decreased If causes decreased rate of phase 4 depolarization.
b. Increased IK-ACh causes hyperpolarization of MDP.
c. Decreased ICa makes threshold potential more positive
d. Parasympathetic withdrawal strongly increases HR.

448
Autonomic Reg of Card Function - Dr. Baumgarten

B. AV Node

The AV node normally does not exhibit automaticity when the heart beat
originates from the SAN. Nevertheless, it is capable of initiating the heart beat,
that is, it is a latent pacemaker. The effects of autonomics on SA and AV nodes
are the same. Under pathophysiologic conditions, the intrinsic rate of the AV
node can exceed that of the SA node and AV nodal automaticity can be
expressed.

C. His-Purkinje System

NE enhances automaticity in the His-Purkinje system also. This automaticity


usually remains latent, however. Block of the SA and AV nodes will result in
ventricular escape, heart beats arising from Purkinje fibers due to their
automaticity. The name 'ventricular' escape is a misnomer. Sympathetic tone
modulates the frequency of ventricular escape beats.

III. DROMOTROPIC EFFECTS

A. AV Node
The AV node is the most important site of control of conduction velocity (CV) by
the autonomic nervous system.

1. Sympathetic Stimulation (NE) Increases CV = + Dromotropic Effect


a. Increased ICa causes increased inward current and gives rise to increased
dV/dt of phase 0 and more rapid conduction. Decreases AV delay.
b. Refractory Periods. An increase of ICa decreases the duration of the ARP
and ERP and shifts the RRP earlier in the cardiac cycle.
Mechanism: With more Ca2+ channels (i.e., increased ICa), a smaller
fraction of the channels needs to recover from inactivation to support
excitability.
Implication: Decreased ERP means that the AVN can faithfully conduct
action potentials at higher heart rates.

2. Parasympathetic Stimulation (ACh) Decreases CV = – Dromotropic Effect.


a. Increased IK-ACh slows CV by opposing ICa.
b. Decreased ICa reduces inward current, dV/dt, and slows CV.
c. Increased IK-ACh and decreased ICa prolong the ARP and ERP

449
Autonomic Reg of Card Function - Dr. Baumgarten

B. Atrium

1. Parasympathetic Stimulation (ACh) Increases CV = + Dromotropic Effect


Increased IK-ACh causes hyperpolarization of atrial muscle. The
responsiveness relationship predicts increased dV/dt and increased CV
because more Na+ channels move from the inactivated to resting available
state. This indirect effect on Na+ channels is much more important in atria
than the direct effect of increased IK-ACh, which by itself would tend to slow
conduction velocity.

IV. INOTROPIC EFFECTS

A. Sympathetic Stimulation (NE) Increases Contractility = + Inotropic Effect


1. Increased force is produced by both atrial and ventricular muscle at a given
length. Several changes in the twitch are noted:
a. increased peak tension
b. increased rate of tension development (dT/dt)
c. increased rate of relaxation.
d. decreased twitch duration (not required for +inotropic effect)

450
Autonomic Reg of Card Function - Dr. Baumgarten

2. In the intact heart, increased contractility implies more efficient pumping of


blood and a shift of the systolic pressure-volume curve (Frank-Starling Curve)
and greater cardiac output and decreased twitch duration maximizes time for
ventricular refilling at high HR.

3. Increased ICa results in a greater trigger for Ca2+-induced Ca2+ release and
greater filling of the sarcoplasmic reticulum Ca2+ stores. Thus, increased ICa
during the plateau results in a more rapid increase and higher levels of
cytoplasmic Ca2+ and, consequently, enhanced force production.
4. The basis for faster relaxation is:
a. Increased rate of Ca2+ accumulation by sarcoplasmic reticulum due NE-
induced phosphorylation of phospholamban, an SR protein that regulates
the SR Ca2+ pump.
b. Decreased affinity of TnC for Ca2+. More rapid release of Ca2+ from
myofilaments makes it available for uptake by SR. By itself, this effect
would decrease contractility.
B. Vagal Stimulation (ACh) Decreases Contractility = − Inotropic Effect
1. ACh has a strong negative inotropic effect on atrial muscle.

2. ACh decreases atrial contractility by increasing in IK-ACh and decreasing ICa.


a. Low doses: Increase IK-ACh causing hyperpolarization and decreased
action potential duration. Abbreviation of the action potential indirectly
decreases Ca2+ entry and contractility.
b. High doses: ACh increases IK-ACh and directly decreases ICa. This gives
rise to a profound negative inotropic effect.
3. ACh has a significant negative inotropic effect in ventricle only in the
presence of high sympathetic tone. Thus parasympathetic stimulation
counteracts sympathetic stimulation.

451
Autonomic Reg of Card Function - Dr. Baumgarten

V. RECEPTOR MECHANISMS

A. G Proteins

1. β1 and muscarinic receptors activate membrane bound target proteins


(including channels and membrane bound enzymes) through a family of
proteins referred to as G proteins. G proteins are heterotrimers with α, β, and
γ subunits (i.e., 3 different subunits). The α subunits bind guanosine
triphosphate (GTP) or guanosine diphosphate (GDP). The α subunits of
various G proteins are distinct. A number of G proteins share common β and
γ subunits, while others have distinct β and γ subunits.
2. Binding of a ligand to its receptor leads to activation of a G protein.
Activation causes dissociation of the GTP-bound α subunit from βγ. Both α
and βγ remain within the membrane, and one or both act on target proteins.
The messenger system deactivates in a reaction that reforms the heterotrimer
while GTPase activating proteins assist the dephosphorylation of GTP to
GDP. After dissociation of GDP and rebinding of another GTP to the α
subunit, the G protein is once again capable of activation.
3. G proteins can stimulate or inhibit channels (e.g., K+ and Ca2+) and second
messenger systems via, for example, adenylate cyclase. Stimulatory G
proteins include Gs and inhibitory G proteins include Gi and GK. GK is
named for its stimulatory effect on K+ channels. In some cases, effects of G
proteins on channels are via cyclic AMP and protein kinase A. Direct effects
on channels also have been noted. (Additional classes of G-proteins are
known.)

452
Autonomic Reg of Card Function - Dr. Baumgarten

B. β1 Adrenergic Receptor Activation

Gs couples the β1 receptor to adenylate cyclase, a membrane bound enzyme that


converts ATP to 3',5'-cyclic AMP (cAMP). Activation of the receptor increases the
rate of cAMP formation and raises the cAMP level in the cell. cAMP in turn
activates cytoplasmic cAMP-dependent protein kinase (protein kinase A, PKA),
which phosphorylates target proteins. Dephosphorylation of the target (or in some
cases a second phosphorylation) terminates the effect.

Examples of targets include:


1. PKA targets
a. ICa channel − phosphorylation via PKA increases ICa (stimulation)
b. phospholamban − phosphorylation increases the rate of Ca2+ uptake by
the sarcoplasmic reticulum (stimulation)
c. Troponin − phosphorylation decreases the Ca2+ sensitivity of the
myofilaments. (aids relaxation)
2. Gs targets
a. adenylate cyclase (stimulation)
b. Ca2+ channels (stimulation)

C. M2 Muscarinic Receptor Activation

GK (a type of Gi) couples the muscarinic receptor to adenylate cyclase. GK


inhibits adenylate cyclase and antagonizes the effects of β1 receptor activation.
GK appears to directly activate K+ channels.

453
Autonomic Reg of Card Function - Dr. Baumgarten

IV. REFERENCES

A. Koeppen, B.M. and Stanton, B.A. Berne & Levy Physiology, 6th Ed., 2008. pp
370-372, 380-382.
B. Costanzo, L.S. Physiology, 3rd, Saunders, Philadelphia, 2006, Chapter 4, pp. 134-
135, Chapter 2, pp. 47-63.
6-Autonomic Regulation of Cardiac Function-2009.doc 8/28/2008

454
Special Circulations 1 - Dr. Pittman

Special Circulations 1
Roland Pittman, Ph.D.

OBJECTIVES:

1. Describe the distribution of cardiac output among the various organs in the
transition from rest to various states of exercise.
2. Describe the interplay between autonomic and metabolic factors in local
regulation of coronary blood flow.
3. Describe the blood brain barrier and its function.
4. Describe the influence of the autonomic nerves on cerebral blood flow.
5. Compare and contrast the influence of changes in arterial PCO2 and PO2 on
cerebral blood flow.

SUGGESTED READING ASSIGNMENT:

R. M. Berne, B. M. Koeppen, M. N. Levy, and B. A. Stanton, Physiology, 5th Ed., St.


Louis: Mosby, pp. 413-421 and 424-426, 2004.
L.S. Costanzo, Physiology, 3rd Ed., Philadelphia: W.B. Saunders, pp. 168-169, 2006.

I. BLOOD FLOW REGULATION IN THE CIRCULATION


(INTRODUCTION)

The distribution of the cardiac output to the various organ systems of the body is
indicated in the table below.

Figure 1.

455
Special Circulations 1 - Dr. Pittman

As can be seen from the table, blood flow depends on the organ, as well as the
level of activity of the body.

Note how blood flow through the various body regions changes as the human
subject begins to exercise. In particular, note how cerebral blood flow is closely
regulated, and how blood flow increases in the coronary, skeletal muscle and skin
circulations, while it decreases in the splanchnic and renal circulations as exercise
continues.

In the following sections we will discuss the circulations of several regions of the
body and review the extrinsic and intrinsic factors which play a role in blood flow
regulation in each particular region.

II. THE CORONARY CIRCULATION

A. Anatomy

The entire blood supply of the myocardium is provided by the right and
left coronary arteries which arise at the root of the aorta behind the right
and left cusp of the aortic valve. (See Figure 2.)

The right coronary artery supplies, for the most part, the right ventricle
and atrium. The left coronary artery supplies principally the left ventricle
and atrium. Most of the venous blood returns to the right atrium through
the coronary sinus, however, a smaller amount returns via the anterior
coronary veins. In addition, there exist some vascular channels between
the vessels of the myocardium and the cardiac chambers.

456
Special Circulations 1 - Dr. Pittman

Figure 2. Anterior and posterior surfaces of the heart, illustrating the location and distribution
of the principal coronary vessels.

B. Regulation of Coronary Flow

1. Neural (Extrinsic) Control

In the intact animal the autonomic nervous system appears to play a


minor role in coronary blood flow regulation when compared with
local, metabolic factors. When the sympathetic nerves are stimulated, a
vasoconstriction of the coronary vessels does occur. However, the
concomitant increase in myocardial metabolic rate results in the
production of vasodilator substances which overshadow this

457
Special Circulations 1 - Dr. Pittman

vasoconstrictor tendency.

The use of experimental preparations such as isolated coronary arteries


and potassium-arrested hearts in which metabolism has been stopped
has enabled investigators to look more closely at the effects of neural
input on the coronary circulation. Histological studies have confirmed
that adrenergic nerves innervate the coronary vessels, and, in addition,
it has been demonstrated that two adrenergic receptors, alpha and beta,
exist in coronary blood vessels.

2. Local (Intrinsic) Control

Blood flow autoregulation occurs in the coronary circulation over a


wide range of perfusion pressure (see Figure 3.)

Figure 3. Pressure-flow relationships in the coronary vascular bed. At a constant aortic


pressure, cardiac output, and heart rate, coronary artery perfusion pressure was abruptly
increased or decreased from the control level indicated by the point where the two lines
cross. The closed circles represent the flows that were obtained immediately after the change
in perfusion pressure, and the open circles represent the steady-state flows at the new
pressures. There is a tendency for flow to return toward the control level (autoregulation of
blood flow); this is most prominent over the intermediate pressure range (about 60 to 180
mm Hg).

458
Special Circulations 1 - Dr. Pittman

The dominant factor that controls coronary blood flow appears to be


the metabolic rate of the myocardium. It has been shown by many
investigators that any increase in the metabolic activity of the heart
results in an increase in coronary blood flow. Whereas a myogenic
mechanism cannot explain the changes in blood flow that accompany
alteration in tissue metabolic rate, the metabolic mechanism appears to
account for changes in blood flow quite well.

Any sudden increase in the metabolic rate of the heart initially


produces tissue hypoxia, which refers to a state in which oxygen
delivery by the blood is insufficient to meet the oxygen demand of the
tissue. Hypoxia has been shown to be a potent stimulus for coronary
vasodilation. It is still unknown whether the blood vessel wall is
directly sensitive to oxygen (i.e., direct effect of oxygen) or whether
hypoxia of the parenchymal tissue (i.e., indirect effect of oxygen)
results in the increased production of a vasodilator metabolite that
elicits arteriolar dilation. Recent evidence seems to indicate that the
latter proposal is more likely. In fact, adenosine, a potent vasodilator
metabolite, has been shown to rapidly increase in concentration when
the oxygen delivery to the myocardium is inadequate.

Other vasodilator substances that have been considered are H+, K+ and
prostaglandins, although none of these substances are as potent a
vasodilator as adenosine when infused into the coronary circulation. At
present it appears that coronary blood flow is closely linked to the
metabolic needs of the myocardium with oxygen and adenosine
playing key roles in the chain of events that bring about blood flow
regulation.

III. CEREBRAL CIRCULATION

A. Anatomy

Blood is supplied to the brain by the carotid artery (which feeds the circle
of Willis) and by the basilar artery (which feeds the vertebral arteries).
The circle of Willis supplies blood to the anterior, middle and posterior
cerebral arteries. Blood from both the carotid arteries and the vertebral
arteries can freely communicate through the circle of Willis. The
beneficial aspect of this anatomy is that blood flow to the brain will not
cease despite an occlusion of one of these vessels.

The capillaries of the brain have a unique feature that has been referred to
as the blood-brain barrier. The walls of brain capillaries exhibit a low
permeability that prevents the passage of most substances. However, gases
(e.g., O2 and CO2) and certain nutrients (e.g., glucose and certain amino
acids needed for neurotransmitter synthesis with special carrier

459
Special Circulations 1 - Dr. Pittman

mechanisms) can pass with little difficulty between blood and cerebral
tissue.

B. Regulation of Cerebral Blood Flow (CBF)

1. Neural (Extrinsic) Control

The arteries on the brain surface and the larger arterioles inside the
brain are supplied with sympathetic and parasympathetic fibers.
However, maximal stimulation of the sympathetic nerves reduces
CBF by only 5-10%. The cerebral veins also receive a sympathetic
nerve supply and constrict by 10-20% when the nerves are
maximally stimulated. Similar stimulation of the parasympathetic
fibers elicits only a modest response. It remains to be seen what
role, if any, the constriction of cerebral veins by the sympathetic
nerves may play in regulating intracranial pressure. Further
research is required to determine the functional importance of the
autonomic nervous system in regulation of CBF.

2. Local (Intrinsic) Control

The rate of CBF is regulated primarily by the concentration of


carbon dioxide in the cerebral tissues. Acute increases in the level
of CO2 result in marked vasodilation and an increase in CBF (see
Figure 4). If CO2 tension (pressure) is reduced, vasoconstriction
occurs, bringing about a decrease in CBF. Experimental results
indicate that CO2 does not act directly on vascular smooth muscle
but, rather, changes in CO2 lead to changes in pH in the fluid that
bathes the vessels and it is this change in pH that elicits alterations
in vascular caliber. It is unclear how pH changes cause variations
in smooth muscle tone. However, it has been suggested that the pH
inside smooth muscle cells is the important factor and that changes
in the concentration of calcium ion may be involved.

460
Special Circulations 1 - Dr. Pittman

Figure 4. Chemical control of CBF: cerebral blood flow variations induced by acute changes
of arterial PCO2. The effect is mediated by pH changes in the brain extracellular fluid (CSF)
surrounding the arterioles; probably it is pH inside the smooth muscle cells that (via Ca++
changes?) causes the active changes in vascular tone (1).

When dealing with cases in which chronic changes of CO2 are


involved, one must consider the effect the blood-brain barrier may
have on the pH of the cerebrospinal fluid (CSF). The blood brain
barrier prevents the bicarbonate molecule from passing between
the blood and the CSF. However, since CO2 is freely permeable, it
can pass across the blood brain barrier. In a case where CO2
tension is chronically reduced, initially arterial hypocapnia and
alkalosis occur. Next alkalosis of the CSF also occurs due to the
reductions of CO2 tension. However, with time (24-36 hours) the
bicarbonate concentration of the CSF is adjusted so that the pH of
the CSF returns to normal. In this case CBF is normal since the
adaptation in CSF pH parallels (and probably causes) the CBF
adaptation.

Minor alterations in the normal O2 tension of the arterial blood do


not cause significant changes in CBF. However, with more severe
hypoxia (PO2 < 50 mmHg), CBF increases dramatically. (See
Figure 5)

461
Special Circulations 1 - Dr. Pittman

Figure 5. Chemical Control of CBF: cerebral blood flow variations induced by changing
arterial Po2.

Since severe hypoxia induces brain-tissue lactacidosis (due to


lactic acid production), it has been suggested that chemical control
of CBF by CO2 and O2 are virtually the same (i.e., via pH change).
The answer, however, remains unsettled since some investigators
feel that cerebral vasodilation in moderate hypoxia cannot be
explained by tissue acidosis in the initial stages.

In the intact animal the effects of hypoxia are felt to be less


pronounced than effects of CO2 on the CBF. This is due to the
hyperventilation following hypoxia that, in turn, results in
hypocapnia. Hypocapnia would inhibit the vasodilatory actions of
hypoxia.

Similar to the coronary circulation, autoregulation of the cerebral


circulation occurs over a wide range (60-180 mmHg) of arterial
blood pressure.

462
Special Circulations 1 - Dr. Pittman

STUDY QUESTIONS

1. During light exercise (i.e., walking up the stairs from Dr. Costanzo’s office to the
Sanger Hall 13th floor Health Club), blood flow to which of the following organs
or tissues is likely to increase above resting levels?

1. Coronary
2. Skin
3. Skeletal Muscle
4. Cerebral

ANSWER: A (1, 2, & 3)

You need to recognize that any level of exercise will cause blood flow to the heart,
skin, and active muscles to increase. Blood flow to the brain generally does not
change during exercise.

2. Which of the following is/are thought to play a major role in control of the
coronary circulation?

1. Stimulation of alpha-adrenergic receptors on the coronary resistance vessels.


2. Stimulation of beta-adrenergic receptors on myocytes.
3. Stimulation of beta-adrenergic receptors on the coronary resistance vessels.
4. Production of adenosine by myocytes.

ANSWER: C (2 & 4)

The major determinant of coronary blood flow is the metabolic state of the
myocardium. Stimulation of the beta-adrenergic receptors on the myocytes will lead
to increased contractility of the heart and, hence, to an increased energy demand
that is met by increased blood flow. Adenosine, a product of ATP degradation and a
powerful vasodilator, appears to be the primary chemical mediator of imbalances
between energy supply and demand. Although there are both alpha- and beta-
adrenergic receptors on the coronary resistance vessels, they have minor influence
on coronary blood flow and certainly play no major role in the control of the
coronary circulation.

3. Which of the following statement(s) is/are true about the cerebral circulation?

1. Cerebral arteries and veins receive sympathetic innervation.


2. Cerebral blood flow is regulated primarily by carbon dioxide.
3. Cerebral blood flow is relatively insensitive to changes in arterial PO2 above
50 mmHg.
4. A reduction in cerebral carbon dioxide levels leads to vasodilation.

463
Special Circulations 1 - Dr. Pittman

ANSWER: A (1, 2, & 3)

The vasculature of the cerebral circulation receives sympathetic innervation, but


these nerves have very little influence on cerebral blood flow. The cerebral
circulation is under the control of chemical influences, primarily carbon dioxide.
Unless the arterial blood PO2 drops below about 50 mmHg, CBF is insensitive to
changes in blood oxygenation. A reduction in cerebral carbon dioxide levels leads to
vasoconstriction, not dilation.

4. Which of the following statements is true about the cerebral circulation?

A. Cerebral arteries and veins do not receive sympathetic innervation.


B. Cerebral blood flow is regulated primarily by arterial [H+].
C. Cerebral blood flow is very sensitive to changes in arterial norepinephrine
concentration.
D. A reduction in cerebral carbon dioxide levels leads to vasoconstriction.
E. Cerebral blood flow falls during exercise.

ANSWER: D

Cerebral arteries and veins do receive sympathetic innervation, but the vasomotor
response to sympathetic stimulation is minor. The blood brain barrier protects the
chemical environment of the cerebral tissue and effectively insulates it from external
influences, such as blood-borne [H+] and norepinephrine. Carbon dioxide, through
its conversion to H+ on the brain tissue side of the blood brain barrier, is the agent
to which the cerebral circulation is most responsive; decreasing CO2 leads to
vasoconstriction. Cerebral blood flow remains relatively constant during exercise,
despite changes in arterial pressure (exquisite autoregulation), and the fact that
arterial CO2 is well controlled during exercise by the respiratory system.

464
Cardio Phys Problem Solving 1 - Dr. Baumgarten

Cardiovascular Physiology Problem Solving 1


Clive M. Baumgarten, Ph.D.

1. How would the following alter conduction velocity in the heart?


A. Depolarization of the His-Purkinje system.

B. Lowering serum K+ from 4 mM to 1 mM.

C. Blocking Ca2+ channels with verapamil.

D. Increasing the rate of an implanted atrial pacemaker from 70 to 150 bpm.

2. Using pressure-volume loops, illustrate Frank-Starling compensation in response to a


decrease in contractility. After compensation is complete, how are the following
parameters altered as compared to their values before the decrease in contractility.
A. EDV

B. ESV

C. EDP

D. SV

465
Cardio Phys Problem Solving 1 - Dr. Baumgarten

3. An otherwise normal person is given an isovolemic transfusion of plasma so that her


systemic hematocrit is decreased from 40% to 25%. The following items relate to the
consequences of this intervention.
A. Considering the passive hemodynamic effects of the intentional hemodilution,
what would you expect to happen to total peripheral resistance? Why?

B. What will happen to cardiac output and central venous pressure?

C. What happens to mean blood velocity and wall shear rate?

D. Is the tendency toward turbulence more or less compared with the normal
hematocrit situation?

E. Suppose that the arterial pressure is regulated so that during hemodilution it


remains at the value observed with normal hematocrit. How might this occur?

4. Identify single changes in either the cardiac function curve (CFC) or vascular
function curve (VFC) that would account for altered cardiac output (CO) and right
atrial pressure (RAP) in a cardiovascular system composed of a pump and vascular
system. Changes to be identified are in the CFC or a vascular system composed of an
unstressed volume (UV), a stressed volume (SV), a blood volume (BV), and a total
peripheral resistance (TPR).
Change Cause
↓ CO and ↓ RAP (identify two)

↓ CO and ↑ RAP (identify one

↑ CO and ↓ RAP (identify one)

↑ CO and ↑ RAP (identify two)


11-Cardiovascular Physiology Problem Solving 1-2009.doc 8/8/2008

466
Special Circulations 2 - Dr. Pittman

Special Circulations 2
Roland Pittman, Ph.D.

OBJECTIVES:

1. Describe the role of sympathetic nervous activity on blood flow in skeletal


muscle.
2. Describe the role of the myogenic and metabolic mechanisms on blood flow in
skeletal muscle.
3. Describe the anatomical characteristics of blood flow in the skin, including
nutritive vessels and the subcutaneous venous plexus.
4. Compare and contrast the importance of neural and local control of the skin
circulation.
5. Describe the features of the “triple response” in the skin due to minor trauma.

SUGGESTED READING ASSIGNMENT:

R. M. Berne, B. M. Koeppen, M. N. Levy, and B. A. Stanton, Physiology, 5th Ed., St.


Louis: Mosby, pp. 421-424, 2004.

L.S. Costanzo, Physiology, 3rd Ed., Philadelphia: W.B. Saunders, pp. 169, 2006.

I. SKELETAL MUSCLE CIRCULATION

A. Anatomy

Small arteries enter muscles and subsequently branch into numerous


smaller vessels called arterioles. The arterioles can be grouped into larger
flow-controlling arterioles (> 50 µm in diameter) or smaller (< 25 µm in
diameter) arterioles that determine the number of capillaries with blood
flow. An interesting characteristic of the larger arterioles is their tendency
to anastomose with one another to form an arcading network of blood
vessels throughout the muscle. Similar vascular arcades are found in other
tissues (omentum, mesentery, etc.). One benefit of such arcading systems
is the maintenance of blood flow to tissue regions despite the occlusion of
an upstream branch.

The smaller vessels do not anastomose with each other, but rather give rise
to increasingly smaller arterioles that finally terminate into capillary
networks. The capillaries have an average diameter of 4.7 µm and tend to
become slightly larger at their venular end (i.e., 5.9 µm). Capillaries
anastomose with other capillaries either from the same arteriole or from
different arterioles. Precapillary sphincters do not appear to exist in
skeletal muscle, and recent evidence suggests that the smaller arterioles

467
Special Circulations 2 - Dr. Pittman

upstream from the capillary bed may be responsible for regulating flow
through the capillary bed.

Figure 1. Arrangement of microvasculature of skeletal muscle.

B. Regulation of Skeletal Muscle Blood Flow

1. Neural (Extrinsic) Control

Sympathetic fibers innervate most of the arterial network of skeletal


muscle. The veins also receive some sympathetic input, but the density
of innervation is much less than for the arteries. The smooth muscle
cells have two different receptor types, α- and β2-adrenergic. When
these receptors are activated, the smooth muscle cells respond by
contraction (vessel constriction) and relaxation (vessel dilation),
respectively.

At low stimulation frequencies vasoconstriction is more pronounced


on the venous side than on the arterial side. At higher frequencies of
stimulation, vascular resistance increases 500 to 800% and unstressed
volume falls 30 to 35% (see Figure 2).

468
Special Circulations 2 - Dr. Pittman

Figure 2. Comparison of capacitance and resistance vessel responses in the hind quarters of
the cat during sympathetic nerve stimulation. Shaded areas indicate range of responses seen
at various stimulation frequencies. (Redrawn from Mellander.)

Sympathetic cholinergic fibers have been found to exist in cat and dog
skeletal muscle, however, they do not appear to exist in humans.

The response of blood vessels to the sympathetic transmitter,


norepinephrine, is generally vasoconstriction. However, when an α-
adrenergic blocking agent (phentolamine) is administered, a
vasodilator influence acting through β2- receptors is unmasked. When
β2-receptors are blocked with propranolol (while phentolamine is still
present), norepinephrine has no effect.

The effect of infused epinephrine on the circulation is related to the


dose given. If physiologic or therapeutic doses of epinephrine are
administered, vascular resistance may decrease (vasodilation).
However, larger doses of epinephrine can cause marked
vasoconstriction.

2. Local (Intrinsic) Control

Blood flow through skeletal muscle is autoregulated in the pressure


range between 20-160 mmHg. The mechanism responsible for the
maintenance of blood flow, despite changes in perfusion pressure, is
hotly debated. It appears most likely that both the myogenic and
metabolic mechanisms participate, with the metabolic mechanism
dominating when the ratio of metabolism to flow is high.

In resting skeletal muscle, a decrease in the arterial perfusion pressure


leads to dilation of the arterioles. This dilation can be abolished if
more oxygen is supplied to the muscle. These results suggest that
hypoxia plays an integral role in the autoregulatory response. It is not
known conclusively whether hypoxia of the vascular smooth muscle
(i.e., direct effect of oxygen) or hypoxia of the skeletal muscle fibers

469
Special Circulations 2 - Dr. Pittman

(i.e., indirect effect of oxygen) is primarily responsible for arteriolar


dilation secondary to hypotension. Experiments in which an oxygen-
sensing microelectrode was used to measure periarteriolar O2 tension
or tissue O2 tension indicate that PO2 in the tissue falls more
drastically than it does at the arteriolar wall. This result suggests that
an indirect effect of oxygen is more likely. According to this proposal,
the tissue cell becomes hypoxic, resulting in the production of
vasodilator substances (such as adenosine, lactic acid or inorganic
phosphate). These vasoactive substances then diffuse to the arteriolar
wall and cause relaxation. At the present time researchers are still
trying to determine which vasodilator substances may be involved.

In exercising muscle, the local increase in blood flow is probably


caused by several different factors all operating at the same time.

a. The amount of dissolved O2 in the tissues decreases rapidly


secondary to the increase in oxygen consumption of the
muscle. The decrease in oxygen would lead to a switch from
the oxidative metabolism of glucose to the glycolytic
degradation of glucose to lactic acid. The production of lactic
acid will result in pH alterations in the tissue that may result in
relaxation of the vasculature.

b. Exercising muscle consumes large quantities of ATP and


produces ADP as a breakdown product. This ADP may be
further degraded into the potent vasodilator adenosine.
c. The exercising skeletal muscle cell releases large quantities of
K+ into the extracellular fluid upon excitation of the cell
membrane. K+ is known to act as a vasodilator and therefore
may play a role in vascular relaxation accompanying exercise.

Other substances have also been implicated as vasodilator metabolites


(e.g., acetate, CO2, inorganic phosphate). However, it remains to be
determined how many and which specific substances are primarily
involved.

II. SKIN CIRCULATION

A. Anatomy

The circulatory apparatus of the skin is characterized by two major types


of vessels: (1) the usual nutritive arteries, capillaries and veins and (2) an
extensive subcutaneous venous plexus that holds large quantities of blood
that can heat the surface of the skin. Also, in some skin areas (feet, hands,
lips, nose, and ears) arteriovenous anastomoses exist. These are large

470
Special Circulations 2 - Dr. Pittman

direct vascular communications between the arteries and the venous


plexus. (See Figure 3)

Figure 3. Arrangement of cutaneous blood vessels combining features which are normally
characteristic of specific regions. SP, subpapillary plexus layer; VP, venous plexus; AP, arteriolar
plexus; SCP, cutaneous plexus; A-VA, arterio-venous anastomosis; GA, arterio-venous glomus.

The subcutaneous venous plexus and the arteriovenous anastomoses serve


in the regulation of body temperature by conducting heat from the internal
structures of the body to the skin, so that heat can be removed from the
body by radiation to the environment. When body heat needs to be
conserved, the sympathetic nerves innervating the arteriovenous
anastomoses cause constriction in these vascular channels and thereby
substantially reduce blood flow into the venous plexus. In addition, the

471
Special Circulations 2 - Dr. Pittman

arteries and veins of the extremities run in parallel allowing countercurrent


exchange of heat.

B. Regulation of the Skin Circulation

1. Neural (Extrinsic) Control

The principal function of blood flow through skin is to control body


temperature, and since this function in turn is regulated by the nervous
system, the blood flow through the skin is principally regulated by
nervous mechanisms rather than local regulation. A control center
which regulates body temperature is located in the anterior
hypothalamus. When this region is heated, the skin vessels vasodilate
and sweating occurs. Cooling of the area causes vasoconstriction and
cessation of sweating.

The vasoconstriction of skin blood vessels is brought about by


stimulation of the sympathetic vasoconstrictor fibers which release
norepinephrine. At normal body temperature the vasoconstrictor
nerves keep these anastomoses almost totally closed. When the body
becomes overheated, sympathetic input is greatly reduced and blood
flow to the skin is greatly increased.

When body temperature is excessive and sweating begins, skin blood


flow is increased further by "active" vasodilation. The mechanism
behind "active" vasodilation is unknown; however, two interesting
hypotheses are presented here:

a. Since blood flow increases rapidly after the onset of sweating,


it has been proposed that kallikrein is released from the sweat
glands and produces bradykinin. Bradykinin would act on the
arterioles to cause vasodilation.
b. Some researchers feel that a sympathetic vasodilation system
may exist. These fibers would release acetylcholine from their
nerve endings and thereby produce vascular relaxation.
Whether or not such a system exists is in question because
blood flow increases only in the presence of sweating.

2. Local (Intrinsic) Control

The metabolism of the skin is very low so there is virtually no


metabolic autoregulation of blood flow as compared with other tissues
(e.g., muscle, coronary, cerebral). Myogenic participation in the
regulation of skin blood flow is still under investigation. As the
forearm is lowered relative to the heart, flow through the skin remains
constant, suggesting that increased vascular tone (myogenic response)

472
Special Circulations 2 - Dr. Pittman

may act to offset the increased transmural and perfusion pressures.

The response of the skin to mild trauma is of interest. When the skin is
stroked firmly with a pointed object, three local reactions called the
"triple response" occur. The triple response is comprised of (1) a red
line, (2) a red flare and (3) a raised wheal. The red line is caused by
local damage to the tissues and most likely results in release of
histamine or other vasodilator substances from damaged cells. The red
flare develops in 20 to 40 seconds after trauma and is caused by an
axon reflex (see Figure 4).

Figure 4. Schematic representation of the axon reflex in response to a scratch on the skin
surface with a sharp instrument. Arrows indicate the pathways of impulses in a sensory nerve
from the site of stimulation to adjacent arterioles to produce local vasodilation (flare).

The axon reflex is caused by excitation of pain fibers. The impulse


travels up a pain fiber and back down the same fiber. An unknown
vasodilator substance is released from the nerve fiber and causes
vasodilation for 2 to 3 cm on each side of the stroke line. The flare can
last up to 10-30 minutes. The raised wheal develops for up to 1 cm on
each side of the stroke line within 1 minute after injury. This wheal
may be due to high sensitivity of the tissue to histamine which acts to
dilate the arterioles, constrict the veins and thereby increase capillary
hydraulic pressure and fluid flow into the tissue, leading to local
edema formation.

473
Special Circulations 2 - Dr. Pittman

STUDY QUESTIONS

1. Which of the following statement(s) is/are true about the skeletal muscle
circulation?

1. Vascular smooth muscle cells have only alpha-receptors.


2. Adrenergic receptors on the veins respond to sympathetic stimulation by
producing vasoconstriction.
3. Resistance vessels respond to sympathetic stimulation over a more narrow
range of frequencies than do capacitance vessels.
4. The vasomotor response to epinephrine can be dilation or constriction,
depending on the concentration.

ANSWER: C (2 & 4)

Vascular smooth muscle cells in skeletal muscle have both alpha- and beta-
adrenergic receptors. The adrenergic receptors on the venous side of the circulation
receive norepinephrine in response to increased sympathetic stimulation, resulting
in contraction of the venous smooth muscle and vasoconstriction. The resistance
vessels respond to sympathetic stimulation over a much wider range than do the
capacitance vessels (i.e. reach maximum response at higher stimulation frequencies).
Low (physiologic) concentrations of epinephrine generally lead to dilation (response
of beta receptors > response of alpha receptors), whereas high (pharmacologic)
concentrations lead to constriction.

2. Which of the following statement(s) is/are true about local control of blood flow
in skeletal muscle?

1. Both metabolic and myogenic mechanisms appear to be operative.


2. A decrease in perfusion and pressure leads to vasoconstriction.
3. Exercising muscle cells release large quantities of potassium.
4. The supply of oxygen appears to play a minor role.

ANSWER: B (1 & 3)

The control of blood flow in skeletal muscle can be accounted for by a combination
of metabolic and myogenic mechanisms. Either mechanism predicts that a decrease
in perfusion pressure leads to vasodilation, not vasoconstriction. At the beginning of
exercise, release of potassium ions by the contracting muscles appears to be the
major determinant of the rapid, locally mediated vasodilatory response. The supply
of oxygen to exercising muscle is a major determinant of blood flow in exercise.

3. Which of the following statement(s) is/are true regarding the cutaneous


circulation?

474
Special Circulations 2 - Dr. Pittman

1. Metabolic autoregulation of blood flow is prominent.


2. Increased temperature leads to vasoconstriction.
3. Part of the response to minor trauma is the release of bradykinin from mast
cells.
4. Blood flow is regulated primarily by sympathetic nerve stimulation.

ANSWER: D

Blood flow to the skin (cutaneous circulation) is primarily under control of the
sympathetic nerves and local phenomena, such as autoregulation, are generally not
very prominent. Increased temperature leads to vasodilation, not constriction. The
response to minor trauma (triple response) is mediated, in part, by release of
histamine from mast cells.

4. Which of the following statements is true about the circulation of the skin?

A. Metabolic autoregulation of blood flow is important.


B. Increased skin temperature leads to vasodilation.
C. Part of the response to minor trauma is the release of adenosine from mast
cells.
D. Blood flow decreases on going from rest to moderate exercise.
E. Arteriovenous anastomoses are primarily under metabolic control.

ANSWER: B

Blood flow to the skin is primarily under control of the sympathetic nerves, so that
local phenomena, such as metabolic autoregulation, are generally not very
important. Increased temperature leads to vasodilation. The response to minor
trauma (triple response) is mediated, in part, by release of histamine, not adenosine,
from mast cells. Blood flow in the skin increases from rest to moderate exercise. The
arteriovenous anastomoses are primarily under sympathetic nervous control.

475
Regulation of Arterial Blood Pressure 1 - Dr. Ford

Regulation of Arterial Blood Pressure 1


George D. Ford, Ph.D.

OBJECTIVES:

1. State why it is important to maintain arterial pressure at its normal value.


2. Name the 8 blocks which make up the renal-body fluid blood pressure regulating
system; show how they are coupled together; and describe the function of each
block.
3. Define the following abbreviations: Q or CO, Pa, E, Pms, TPR, dEo/dt, dEi/dt,
dE/dt.
4. Describe how changes in renal function (increase or decrease) affect: E, Pms, Q,
and Pa.
5. Describe the anatomical connections in the arterial baroreceptor reflex.
6. Describe how pressure is transduced by baroreceptors.
7. Describe how the information generated by baroreceptors is processed by the
central nervous system.
8. Describe the cardiopulmonary baroreflexes.

I. OVERVIEW

There are three levels of regulation, neural, humoral, and intrinsic. The latter is
often referred to volume regulation in most textbooks. They operate in very
distinct time frames. Traditionally, they are covered in the order listed above, but
these two lectures will reverse that order.

II. (MEAN) ARTERIAL PRESSURE IS THE PRODUCT OF CARDIAC


OUTPUT AND TOTAL PERIPHERAL RESISTANCE.

Pa = C.O. x TPR

In this equation Pa = arterial pressure (mm Hg), C.O. = cardiac output (liter/min),
and TPR total peripheral resistance (mm Hg x min/liters).

This seductively simple equation may be one of the most dangerous equations in
cardiovascular physiology because C.O. and TPR are not, as suggested by the
equation, independent variables. Unfortunately changes in TPR strongly affect
C.O. as we have just finished discussing. Changes in C.O. also affect TPR. Thus it
cannot be stated that a doubling of TPR will double Pa at the steady state.

III. THE INDEPENDENT VARIABLES REGULATING ARTERIAL BLOOD


PRESSURE ARE: 1) CONTRACTILITY OF VASCULAR SMOOTH
MUSCLE, 2) CONTRACTILITY OF THE HEART, AND 3) RENAL
FUNCTION.

476
Regulation of Arterial Blood Pressure 1 - Dr. Ford

Regulation of these three independent variables determines arterial blood


pressure. The levels at which these variables are regulated are: 1) intrinsic, 2)
neurogenic, and 3) hormonal. The regulation of arterial blood pressure will be
presented at these three levels and in that order. The intrinsic level is taught first
because the neurogenic and hormonal mechanisms operate on top of and regulate
components of the intrinsic system. The intrinsic system is often called the Renal
Body-fluid system.

IV. INTRINSIC REGULATION OF ARTERIAL BLOOD PRESSURE


INVOLVES INTERACTIONS BETWEEN: HEART, VASCULAR
SYSTEM, AND KIDNEYS. WHEN LINKED TOGETHER THEY
BECOME THE BODY FLUID SYSTEM.

It is particularly the view of Arthur Guyton and many others that the kidney,
heart, and vascular system interact, even in the absence of neural or hormonal
control, in such a way that a nearly normal blood pressure can be achieved. These
components (heart, kidney, and vascular system) possess intrinsic properties
which allow them to adjust blood pressure at a slow rate (days to years). Neural
and endocrine factors increase the rate of response markedly (minutes to days).
With the neural mechanisms, responses occur after about 10 seconds.

In the intrinsic system the kidney, heart and vascular system are coupled in series.

Figure 1. Diagram of the intrinsic Renal Body-fluid blood pressure regulating system. For explanation see the
following discussion.

477
Regulation of Arterial Blood Pressure 1 - Dr. Ford

An explanation of this diagram starts with blocks that are already familiar.

A. Block 1. The blood volume-mean systemic pressure relationship.

This block shows the relationship between blood volume (Vb) and means
system pressure (Pms). Blood volume is one determinant of mean system
pressure. The other determinants are the unstressed volume and the
capacitance of the stressed volume.

B. Block II. The mean system pressure-cardiac output relationship.

Mean system pressure anchors the vascular function curve and the intersection
of this curve with the cardiac function curve determines cardiac output (Q).
The curves shown are for the intrinsic contractility of the heart and vascular
system.

C. Block III. The cardiac output-total peripheral resistance relationship.

The major determinant of total peripheral resistance is the caliber of arterioles.


This caliber is determined by the contraction of vascular smooth muscle. The
contraction of vascular smooth muscle is subject to influences which can be
categorized as: a) intrinsic or b) extrinsic (neural or hormonal).

The Intrinsic autoregulation of total peripheral resistance [TPR] is a concept


that I hope Dr. Pittman has already discussed with you. The next paragraph is
intended only to be a quick review of his more thorough discussion.
Arterioles, if completely devoid of neural or hormonal regulation still have the
capacity to alter their caliber. The phenomenon thereby produced is termed
autoregulation of blood flow. The mechanism(s) responsible are not well
established. At least two hypotheses have been proposed and each may play a
role. The two hypotheses are: 1) the myogenic hypothesis and 2) the metabolic
hypothesis. These hypotheses, in brief, are:

1. The myogenic hypothesis states that arterioles contract in response to


pressure induced stretch (tension). This decreases the caliber (radius)
of the arteriole and increases resistance to flow. (Recall that
Poiseuille's Law states that resistance to flow is inversely related to the
fourth power of the radius of the vessel).
2. The metabolic hypothesis states that increases in flow through an
arteriole decrease the concentration of metabolically produced
vasodilators, in particular nitric oxide (NO). The decreased
concentration of vasodilators causes contraction of arteriole smooth
muscle. Contraction of the arteriole smooth muscle decreases arteriole
caliber and increases resistance to flow (Poiseuille's Law).

478
Regulation of Arterial Blood Pressure 1 - Dr. Ford

Note in block III that TPR remains constant until Q reaches a certain value.
After that value of Q is reached TPR increases as Q increases.

D. Block IV. Pa = CO x TPR

This block states that arterial pressure (Pa) is the product of cardiac output
(CO) and total peripheral resistance (TPR).

E. Block V. The arterial pressure-renal excretion of extracellular fluid


(dEo/dt) relationship. (Renal Function Curve)

Increases in Pa increase the rate at which extracellular fluid (E) is lost


(dEo/dt). Note that dEo/dt is zero until Pa reaches 60 - 80 mm Hg.

The kidney does not excrete extracellular fluid per se. Rather, it excretes
components of extracellular fluid. The component of most importance is
sodium. The regulation of the extracellular fluid volume takes place primarily
by the regulation of the sodium content of the body. We are interested in
extracellular fluid volume because one compartment of extracellular fluid is
blood. Interstitial fluid is the second major compartment.

F. Block VI. Net Change in the dE/dt is the sum of inputs and outputs

This block states that the algebraic sum of the rate of input of extracellular
fluid (dEi/dt) and the rate of loss of extracellular fluid (dEo/dt) determines
whether the rate of gain of E, dE/dt, may be positive or negative. The rate of
input of extracellular fluid (dEi/dt) is determined by the rate at which sodium
and fluid enters the body, i.e., dietary choices.

G. Block VII. The volume of the extracellular fluid is the integral (algebraic
sum) of all gains and losses of E.

H. Block VIII. The blood volume/extracellular fluid volume relationship.

The volume of extracellular fluid (Ve) is the sum of blood plasma volume
(Vp) and interstitial fluid volume (Vi).

Ve = Vp + Vi

Blood is normally composed of about 42 per cent red blood cells and 58 per
cent plasma. Thus plasma volume is a determinant of blood volume (Vb).
Typically Vb is about 1/3 of Ve. Later we will consider conditions which can
shift interstitial fluid into or out of the vascular space.

479
Regulation of Arterial Blood Pressure 1 - Dr. Ford

V. THE RENAL BODY-FLUID SYSTEM COMES TO A STEADY-STATE


WHEN dEo/dt = dEi/dt.

It is important to recognize that the renal body-fluid system can only stop
changing, i.e., come to a steady-state, when dEo/dt = dEi/dt. Whenever they
differ, E will either be increasing or decreasing. Changes in E cause changes in
Vb, Pms, Q, TPR, and Pa. Thus, none of these can come to a steady-state until
dEo/dt equals dEi/dt.

VI. PATHOLOGICAL INTERVENTIONS CAN CHANGE THE VALUES OF


Q (CARDIAC OUTPUT), Pa (ARTERIAL BLOOD PRESSURE), Vb
(BLOOD VOLUME), E (EXTRACELLULAR FLUID VOLUME), TPR
(TOTAL PERIPHERAL RESISTANCE), dEo/dt (THE RATE OF LOSS OF
EXTRACELLULAR FLUID), OR Pms (MEAN SYSTEM PRESSURE).

By way of example consider the effect of cardiac failure on E, CO, and Pa. The
effects on each block would be as follows:

A. Block II. Cardiac failure shifts the cardiac function curve to the right. This
will increase right atrial pressure (RAP) and decrease Q.

B. Block III. The decrease in Q will decrease TPR.

C. Block IV. The decrease in Q and TPR will decrease Pa.

D. Block V. The decrease in Pa will decrease dEo/dt.

E. Block VI. The decrease in dEo/dt will increase dE/dt.

F. Block VII. The increase in dE/dt will increase E.

G. Block VIII. The increase in E will increase Vb

H. Block I. The increase in Vb will increase Pms (mean system pressure).

I. Block II. The increase in Pms will shift the vascular function curve to the
right and shift the equilibrium point (the intersection of the vascular
function curve and cardiac function curve )upward on the cardiac function
curve. If the heart is not severely failed, CO, Pa, and dEo/dt will be
returned to nearly normal. When dEo/dt is nearly returned to normal, it
will equal dEi/dt and the system will be in a new steady-state. Values
which will have changed are E, Vb, Pms and Pra. The elevated Pra is a
strong clinical indication of heart weakness even in the face of near
normal blood pressure and cardiac output.

480
Regulation of Arterial Blood Pressure 1 - Dr. Ford

Renal failure is a pathological condition which can affect blood pressure.


When this condition rotates the renal function curve clockwise, a greater
Pa is required to produce an equal dEo/dt. You should determine what
would happen following such a change. The outcome would be the
following: There would be increases in E, Pms, right atrial pressure (Pra),
Q, TPR and Pa. When Pa becomes high enough to make dEo/dt equal to
dEi/dt a steady-state is achieved.

VII. NEUROGENIC CONTROL OF ARTERIAL BLOOD PRESSURE.

The neurogenic control of blood pressure involves baroreflexes, chemoreflexes


and higher neural centers. These mechanisms of control are extrinsic mechanisms.
Of these extrinsic mechanisms the baroreflexes are believed to be the most
important to the minute by minute stabilization of arterial blood pressure.

A quick review of the general principles of reflex and hormonal control systems
of the heart and circulation might be in order. All physiological control systems
operate on the principal of negative feedback.

A disturbance (e.g., a fall in blood volume produces a fall in arterial blood


pressure) is sensed (e.g., the carotid sinus baroreceptors) and compared to some
reference point (SET POINT) (e.g., in the solitary tract nucleus in the medulla of
the central nervous system). The difference between the set point and the
controlled variable (e.g., blood pressure), is the error signal that is used to return
the controlled variable (blood pressure) toward its set point. The term negative
feedback is applied to this system because the compensation (e.g., an increase in
pressure) is in a direction opposite to the initial change in the controlled variable
(in this case a fall in blood pressure).

VIII. THE ARTERIAL BARORECEPTOR REFLEX CONTROL OF BLOOD


PRESSURE

The arterial baroreflex is a negative feedback control system regulating blood


pressure over short periods of time, seconds to minutes. The current view is that it
stabilizes blood pressure around its normal value. (The long term regulation of
blood pressure involves hormonal regulation of renal function.) As a feedback
system, it needs an error detector, some set point for comparison, and an effector
system to return the system toward normal. Involved in this reflex are afferent
neurons, the central nervous system, and efferent neurons.

The detectors are baroreceptors, i.e., nerve endings sensitive to arterial blood
pressure. They are found in the carotid sinus and the aortic arch, see Fig. 2. below.

481
Regulation of Arterial Blood Pressure 1 - Dr. Ford

Figure 2. Localization of carotid sinus and aortic baroreceptors. Figs. 21-8 and 21-99 on p. 388-389 of Berne
et al, “Physiology”, 5th Edition, is a much more detailed depiction.

Arterial baroreceptors, also called mechanoreceptors, are pressure sensitive


transducers. Studies have shown that they detect pressure indirectly through
stretch. Increases in pressure increase stretch and the increase in stretch produces
action potentials carried by afferent nerve fibers to the central nervous system. If
baroreceptor sensitivity to stretch is altered, experimentally or by disease, the
response of the receptors is also altered. Prolonged hypertension leads to a
decrease in stretch in response to pressure and a decrease in neural response to
high blood pressure.

Baroreceptors respond to both pressure and rate of change of pressure as shown if


Fig. 3.

482
Regulation of Arterial Blood Pressure 1 - Dr. Ford

Figure 3. Relationship between phasic arterial blood pressure in the carotid sinus and aorta and frequency of
nerve traffic (represented by single vertical spikes). Panel A shows nerve traffic at several different levels of
mean aortic pressure. Panel B shows nerve activity at the same mean arterial pressure but two different pulse
pressures. Just like Fig. 21-10 on p. 390 of Berne et al, “Physiology”, 5th Edition.

Note in panel A and B that the rising phase of pressure is a stronger stimulus for
action potential production than pressure itself. As mean arterial pressure reaches
very high levels, e.g., 200 mm Hg, the firing rate is maximal and continuous.

A. Afferent components of the reflex. (see Fig. 2)

Sensory information generated by the carotid baroreceptors is transmitted


from the carotid sinus over the fine carotid sinus nerve which joins the
glossopharyngeal nerve and carries nerve traffic to the cardiovascular centers.
Information from the aortic baroreceptors is carried over the depressor nerve
and ascends via the vagus nerve.

There are two types of afferent fibers; large myelinated A fibers and small,
unmyelinated C fibers. The A fibers have a lower threshold, greater
sensitivity, and narrower response range. They are more responsive in the
hypotensive range. The C fibers have higher thresholds, lower sensitivity, and
a wider range. They are more responsive in the hypertensive range. Combined
the overall response range is nearly linear from 50 up to 200 mmHg. See
figure 4:

483
Regulation of Arterial Blood Pressure 1 - Dr. Ford

Figure 4.

B. Efferent components of the reflex. (see Fig. 2)

Efferent parasympathetic information is carried over the vagus nerve to the


sinoatrial (SA) node and atrioventricular (AV) node. Efferent information is
also carried down the spinal cord and out the ventral root of the spinal cord to
sympathetic ganglia. The postganglionic fibers go to arterioles, veins, and the
ventricles of the heart.

C. Central connections of the arterial baroreceptor reflex. (Fig. 5.)

484
Regulation of Arterial Blood Pressure 1 - Dr. Ford

Figure 5. Central connections of the baroreceptor reflex. This is sort of like Fig. 21-13 on p. 393 of Berne et
al, “Physiology”, 5th Edition. Fig. 4-31 in Costanzo, “Physiology”, 3rd Edition, p. 157 is a block diagram of
these connections.

Afferent nerve traffic from the baroreceptors is carried to the solitary tract
nucleus (STN) in the medulla (brain stem). At the solitary tract nucleus
information is divided; some goes to the vasomotor center, and some to the
vagal nucleus. Neural impulses from the solitary tract inhibit the neural
activity of the vasomotor center and stimulate the neural activity of the vagal
nucleus. The plus (+) signs in circles mean that the neural activity produces an
excitatory effect on the structure innervated. The minus (-) signs indicate an
inhibitory effect. The outflow from the vagal nucleus goes via vagal
preganglionic fibers to the atrioventricular (AV) node and the sinoatrial (SA)
node.

D. Operation of the baroreflex.

The operation of the baroreflex can most easily be described by a specific


example. The example given is that produced by a fall in blood pressure. (A
rise in arterial pressure will produce opposite effects.) The effects (responses)
in brief are:

485
Regulation of Arterial Blood Pressure 1 - Dr. Ford

1. A fall in arterial pressure reduces the firing rate (rate of impulse


production) of the baroreceptors in the carotid sinus and aortic arch.
2. A decrease in afferent neural traffic from these structures decreases the
neural input to and output from the solitary tract nucleus.
3. A decrease in neural output from the solitary tract,

a. decreases the neural input to the vagal nucleus, and


b. decreases the neural input to the vasomotor center.

4. The decrease in neural input to the vagal nucleus decreases the neural
output from the vagal nucleus and decreases parasympathetic activity
to the SA and AV node.
5. The decrease in parasympathetic activity to the SA node increases
heart rate. This is the most important effect of the decrease in
parasympathetic activity to the heart. The decrease in parasympathetic
activity to the AV node increases conduction velocity through the
node.
6. The decrease in neural input to the vasomotor center increases the
output of neural traffic from the vasomotor center. (Recall that the
effects of fibers from the solitary tract are inhibitory on the vasomotor
center.)
7. The increased neural activity to the ventral horn cells increases neural
activity in the preganglionic, and ultimately postganglionic
sympathetic fibers to veins, arterioles, SA node, AV node, atrial and
ventricular muscle.

The sympathetic efferent activity increase results in four important sites of action
and effects. They are: 1) the SA-node, where the effect is to increase heart rate; 2)
ventricular myocardium, where the effect is to increase cardiac contractility; 3)
arterioles, where the effect is to increase the contraction of vascular smooth
muscle and thereby to decrease the caliber of the arteriole and increase the
resistance to blood flow; and 4) veins, where the effect is to increase the
contractility of vascular smooth muscle and thereby to decrease the caliber of the
veins which decreases unstressed volume and increases Pms (mean system
pressure).

This sympathetic activity affects 3 of the blocks in the renal body-fluid


mechanism of blood pressure regulation. The two obvious blocks are: block I (the
Pms block), and block II (the CO block). Less obvious is the effect on block V III
(the Vb block). The effect of activating the sympathetic system on block I is to
decrease unstressed volume and thus to increase Pms; on block II is to shift the
vascular function curve to the right (Pms is increased) and to rotate the slope of
the curve counterclockwise, the cardiac function curve is shifted upward (a
positive inotropic response); and the effect on block VIII is to rotate the line
relating Vb to E counterclockwise. (Contraction of arterioles reduces the pressure

486
Regulation of Arterial Blood Pressure 1 - Dr. Ford

within capillaries and allows interstitial fluid to move from the interstitial space
into the vasculature.) These effects on blocks I, II, and VIII are shown in Fig. 6.

Figure 6. The effect of activating the sympathetic system on blocks I, II, and VIII of the renal body-fluid
system.

The solid lines show the condition of the system before activation of the sympathetic
system. The dotted lines show the condition of the system after its activation.

IX. RESETTING OF THE BARORECEPTORS

Resetting of the baroreceptors can be done either centrally or peripherally. The


advantage of peripheral resetting is that it’s rapidly reversible. This occurs during
inspiration. More sustained central resetting is seen during exercise.

Peripheral resetting begins to occur after the arterial pressure has been elevated
for 15 minutes. Essentially the stimulus response curve shifts to the right so that
the new arterial pressure is at the lower end of the curve. The mechanism for
resetting is unknown but it effectively renders the baroreceptor system of little use
in reducing chronically elevated arterial pressure (hypertension).

487
Regulation of Arterial Blood Pressure 1 - Dr. Ford

X. CARDIOPULMONARY BARORECEPTORS

There are a wide variety of other receptors, including some baroreceptors. But
others respond to volume or chemical signals. The precise physiological role of
these systems is not well understood although each may be of importance in
understanding select clinical syndromes. The general consensus is that the signals
from all these systems are integrated at the level of the hypothalamus to determine
the degree of sympathetic and parasympathetic tone appropriate for a given set of
conditions.

An example of other such receptors are the cardiopulmonary receptors found in


the atria and great veins. One class of these is known as the A receptors. They are
veno-atrial stretch receptors activated by tension during atrial contraction (atrial
systole). Another class is known as B receptors. They are activated by stretch
either of the great veins or the atrial chamber during filling and are generally
considered volume receptors.

The impulses from these receptors travel via vagal afferent fibers to the vagal
nucleus. The output of the vagal nucleus produces numerous effects: a) it inhibits
the vasomotor efferent output to the kidney and increases renal blood flow, b)
there is a general inhibition of vasomotor output which depresses the sympathetic
tetralogy and reduces blood pressure, c) it inhibits nerve activity to the SA node
and increases heart rate, d) the increase in renal blood flow may be responsible for
inhibition of angiotensin formation and aldosterone secretion (see Regulation of
Arterial Blood Pressure II), e) there is also a centrally mediated depression of
vasopressin secretion.

Perhaps the most famous demonstration of these receptors is the Bainbridge


Reflex where a bolus of fluid delivered rapidly to the great veins produces marked
tachycardia.

Other vagal afferent fibers arise from the ventricles and the coronaries. The
coronary fibers are chemosensitive and respond to capsaicin, bradykinin, and
some prostaglandins. These most likely play a major role in the perception of pain
associated with angina and myocardial ischemia/infarction.

XI. REFERENCES

1. Shepherd and Vanhoutte, "The Human Cardiovascular System: Facts and


Concepts" Raven Press, 1979, Chapter 5, pp. 129-135.

2. Mohrman, David E. and Heller, Lois Jane. “Cardiovascular Physiology”,


4th Edition, McGraw-Hill, 1997, Chapter 10, pp. 151 - 173.

488
Regulation of Arterial Blood Pressure 1 - Dr. Ford

3. Berne, R. M., Levy, M. N., Koeppen, B.M., and Stanton, B.


A.“Physiology”, 5th Edition, 2004, pp. 359-366 & 380-394

4. Costanzo, L. S., “Physiology”, 3rd Edition, 2006, pp. 156-163

XII. PRACTICE PROBLEMS

One or more of the answers to the following questions may be true. Mark each
correct answer with a T.

1. An increase in dEi/dt in a cardiovascular system regulated only by the


renal body-fluid system, will cause which changes when the system comes
to a steady-state?

A. an increase in dEo/dt
B. a decrease in Pa
C. an increase in Q
D. a decrease in TPR

2. A decrease in cardiac contractility will produce which of the following


changes in a cardiovascular system regulated only by the renal body-fluid
system when the system achieves the steady-state?

A. a decrease in Q
B. an increase in Pms
C. an increase in Pa
D. an increase in E

3. A rise in arterial pressure will produce which of the following responses


through the action of the baroreceptor reflex?

A. an increase in baroreceptor firing rate


B. a decrease in neural output from the vasomotor center
C. an increase in neural output from the vagal nucleus
D. a decrease in heart rate

ANSWERS

1. a. T, b. F, c. T, d. F

2. a. T, b. T, c. F, d. T

3. a. T, b. T, c. T, d. T

489
Electrocardiogram 1 and 2 - Dr. Baumgarten

Electrocardiogram 1 and 2
Clive M. Baumgarten, Ph.D.

OBJECTIVES:

1. Identify the components of the Lead II electrocardiogram and how the compon-
ents correspond to cardiac electrical activity
2. Describe the basis for unipolar and bipolar ECG recordings
3. Describe the Dipole approach and Einthoven's triangle
4. Identify the frontal plane limb leads and the electrode arrangement used to record
each
5. Identify how representations in Einthoven's triangle can be converted to the
triaxial lead or hexaxial lead system
6. Predict (a) ECG deflections that result from specified dipole vectors and (b)
dipole vectors that result from specified deflections in limb leads
7. Describe the sequence of ventricular activation and how this gives rise to the QRS
complex
8. Describe and calculate the mean electrical axis

490
Electrocardiogram 1 and 2 - Dr. Baumgarten

I. THE ELECTROCARDIOGRAM

A. A typical Lead II ECG recording. Standard ECG Calibration: 25 mm/se 0.1


mV/mm; 1 mm/box; darker lines every 5th box. This means 1 small box = 40 ms.

B. Components of the ECG

1. P-wave: 0.11 s Depolarization of atrial muscle.


2. PR-interval: 0.12 − 0.20 s
First atrial depolarization to first ventricular depolarization.
Approximately 50% reflects AV nodal conduction time.
Varies with HR and age.
3. QRS interval: 0.10 s
Ventricular depolarization. Reflects spread of excitation through ventricle and
most of conduction in His-Purkinje system.
4. T-wave: Ventricular repolarization
5. ST-segment: All of ventricle depolarized
6. QT-interval: ~0.30 − 0.40 s (10% longer in females than males.)
First depolarization to last repolarization of ventricle. Very dependent on HR.
QT-interval “corrected” to HR of 60 is called QTc
1/2
QTc = QT/(RR-interval)
QTc is <0.39 s in males and <0.43 s in females.
7. RR-interval: 0.6 − 1 s (also termed Cycle Length)
Time between successive R waves.
HR [bpm] = 1/(RR-interval) [bps] x 60 [s/min]

491
Electrocardiogram 1 and 2 - Dr. Baumgarten

C. Relationship of ECG to timing of action potentials.

II. BASIS FOR THE ECG

A. ECG measures voltage between two points on the body surface. Two types of
recordings are made:
1. Unipolar: Voltage at one exploring electrode is measured versus a distant or
derived ground point, which is taken as 0 volts (V = VA - 0).
2. Bipolar: Voltage is recorded as the difference between the voltages at two
electrodes (V = VA - VB) that may be close together or widely spaced.

492
Electrocardiogram 1 and 2 - Dr. Baumgarten

B. Extracellular electrodes indirectly detect differences between the intracellular


membrane potentials (Em) of different sections of the heart.

1. Differences in Em may arise because of normal electrical activity (the spread


of depolarization or repolarization) or, in disease, because of differences in
resting Em.

2. Differences in Em cause current flow through the cytoplasm and the


extracellular space. Current always must flow in a closed loop.
3. An electrode placed in the extracellular space adjacent to the resting (-85 mV)
section would detect a positive potential, and simultaneously, an electrode
placed adjacent to the active (+50 mV) section would detect a negative
potential. Thus the deflection recorded depends on electrode position.

III. DIPOLE APPROACH

A. A dipole is a vector giving the magnitude and direction of the difference in the
extracellular potential (or equivalently, the separation of charge). It is drawn
pointing towards the positive potential (charge).

B. Isopotential Line: When cells have the same Em, there is no net current flow.
Consequently, the extracellular space is isopotential (at the same voltage
everywhere) and the potential difference recorded by the ECG is exactly 0 volts
and is constant. At isopotential, the magnitude of the dipole is 0.

493
Electrocardiogram 1 and 2 - Dr. Baumgarten

C. Einthoven’s Triangle. Conventions for


recording the ECG were first established
by Einthoven who viewed the heart as a
dipole in a homogeneous conducting
media. The right arm, left arm, and left
leg were considered to form an
equilateral triangle in the frontal plane
with the heart in the center. This is
called Einthoven's Triangle. Recording
from any where on a limb is electrically
equivalent to recording from where the
limb meets the body.

D. Standard limb leads (Leads I, II and III) result from Einthoven's view of the
heart. Each lead represents the difference in the voltage recorded at two of the
limbs. They are bipolar recordings in the frontal plane.
Lead I = LA − RA
Lead II = LL − RA
Lead III = LL − LA
The axes of these leads define the sides an equilateral triangle. Each lead views
the electrical activity of the heart (dipole) from a different perspective.

E. Augmented limb leads (aVR, aVL, aVF) record one limb minus a derived
ground. They are unipolar recordings.
aVR = RA − 0
aVL = LA − 0
aVF = LL − 0
The axes of the augmented limb leads are perpendicular to the axes of the
standard limb leads (i.e., perpendicular to the sides of the equilateral triangle) and
are in the frontal plane.

494
Electrocardiogram 1 and 2 - Dr. Baumgarten

F. Amplitude and sign of lead voltage depend on:


1. magnitude and direction of the dipole vector
2. angle between the lead and the dipole vector
Each lead "views" the electrical event from a different direction and will record a
different voltage at the same instant in time. The expected deflection can be
calculated from the projection of a vector onto a line representing the axis of
the lead. (Draw perpendicular lines from axis of lead to the vector.)
$ RULE 1: The deflection is positive if the vector points in the direction of the
lead, that is, towards its positive terminal.
$ RULE 2: The deflection is negative if the vector points away from the
direction of the lead, that is, away from its positive terminal.
$ RULE 3: The deflection is 0 if the vector is perpendicular to the lead.
$ RULE 4: The deflection is maximum if the vector is parallel to the lead.

495
Electrocardiogram 1 and 2 - Dr. Baumgarten

G. Triaxial and Hexaxial format. Leads can be drawn as Einthoven's triangle, or


keeping the angles the same, redrawn as a triaxial (I, II, III) or hexaxial (I, II, III,
aVR, aVL, aVF) reference system.
1. Definition: By agreement, the positive end of lead I (LA) is at 0°, and angles
are labeled from 0 to +180° going clockwise and from 0 to −180° going
counter-clockwise. In the triaxial system, positive and negative lead axes
occur at 60° increments, and in the hexaxial system, at 30° increments.
2. Advantage of hexaxial system: Direction of vector can be estimated to within
~15° by finding lead(s) with greatest magnitude deflection.
Figures: Positive end of lead line is labeled with name (i.e., I, II, III, aVR, aVL,
aVF) and angle is shown in parentheses.

3. Geometric considerations REQUIRE the following relationships:


II = I + III and aVR + aVL + aVF = 0.
If these relations are not true, leads are hooked up incorrectly!

496
Electrocardiogram 1 and 2 - Dr. Baumgarten

IV. ECG PROBLEM SET

The following diagrams practice:


$ Predicting the ECG deflection in each Lead from the dipole vector generated by
cardiac electrical activity. (Dipole → Deflection in Lead)
$ Identifying the dipole vector (and thus, cardiac electric activity) from the ECG
deflections. (Deflection in Lead → Dipole)
$ Calculating the dipole vector axis (angle). (Deflection in Lead → Axis Angle)

497
Electrocardiogram 1 and 2 - Dr. Baumgarten

498
Electrocardiogram 1 and 2 - Dr. Baumgarten

499
Electrocardiogram 1 and 2 - Dr. Baumgarten

V. PATTERN OF VENTRICULAR ACTIVATION − BASIS FOR THE QRS


COMPLEX

A. Septal Activation: The intraventricular septum is activated first by fibers from


left bundle branch. A few msec later, excitation begins at the right septal surface.
The mean vector in the frontal plane is shown at right. The vector is directed to
the right and superiorly.

B. Spread of Activation Towards Apex: Excitation spreads over the septum, and
excitation of the free wall is initiated at subendocardial sites. Excitation spreads
out and coalesces so that some of the endocardial surface is depolarized. The
wavefront moves from the endocardial to epicardial surface and in a caudad
direction. The mean vector is directed to the left and inferiorly; its magnitude is
large because there is a large boundary between depolarized and resting tissue.

500
Electrocardiogram 1 and 2 - Dr. Baumgarten

C. Activation of Base: Excitation reaches the epicardial surface, and spreads


towards the base. Finally, the only wavefront remaining is near the base of the
heart and is moving in a cephalad direction. The mean vector is directed to the left
and superiorly. The left side dominates because the left side is thicker than the
right.

D. Consider the dipoles generated during the three main phases of ventricular
depolarization. Plot the mean vector (shown above) on Einthoven's triangle, and
draw the projection of the vector on a lead (e.g., lead II). The ventricle initially is
uniformly polarized, no current flows in the extracellular space, and the voltage
recorded by the ECG is 0. Over time, the mean vector generated by the heart goes
from 1 − 3. This gives rise to the QRS complex. The ECG returns to 0 potential
when all of the ventricle is depolarized. Try to predict the morphology of the
QRS in the other frontal plane leads.

501
Electrocardiogram 1 and 2 - Dr. Baumgarten

E. Mean Electrical Axis


1. Represents orientation of the vector in the frontal plane during the QRS
complex.
2. It can be calculated from the amplitude of the net deflections in any two leads,
which is measured as the difference between the absolute values of the most
positive and most negative deflections of the QRS in those leads.
3. Normally the mean electrical axis is between +105° and -30°. Recall, the
positive terminal of lead I points towards 0°, and positive terminal of aVF
points towards 90°.
4. Example of Calculation

$ Assume the following QRS deflections in Lead I and aVF


I: Q= = −2, R= +5, S= −1
aVF: Q= −1, R= +6, S= −3
$ Net deflections (|max Vpos| − |max Vneg|) are:
I: |R| − |Q| = 5 − 2 = +3
aVF: |R| −-|S| = 6 − 3 = +3
$ Question: What is the vector that gives a deflection of +3 in Lead I
and +3 in aVF?
$ Plot +3 units on Lead I and aVF, and draw perpendiculars from the
lead lines.
$ Answer: Vector drawn to intersection of the perpendiculars points to
45o. The mean electrical axis in this example is 45o.

502
Electrocardiogram 1 and 2 - Dr. Baumgarten

F. Why is the T-wave upright? The sequence of repolarization depends on the


timing of depolarization and the duration of the AP. Endocardium tends to have
longer action potentials than epicardium. Consequently, endocardium
depolarizes first but repolarizes last. The dipole during repolarization is the same
direction as during depolarization and the T wave and QRS point in the same
direction in lead II.

QRS and T are approximated by simply subtracting the epicardial action potential
(shorter APD) from the endocardial action potential (longer APD). This reflects
the fact that extracellular ECG waveforms arise from differences in the
intracellular potentials; the QRS occurs between the two upstrokes and the
T wave between the two repolarizations (dashed lines).

VI. REFERENCES

A. Koeppen, B.M. and Stanton, B.A. Berne & Levy Physiology, 6th Ed., 2008. pp.
310-313.
B. Costanzo, L.S. Physiology, 3rd Ed., Saunders, 2006, Chapter 4, pp. 125-126, 136-
137.

7-Electrocardiogram 1 and 2-2009.doc 8/28/2008

503
Regulation of Arterial Blood P ressure 2 - Dr. Ford

Regulation of Arterial Blood Pressure 2


George D. Ford, Ph.D.

OBJECTIVES:

1. Describe the Central Nervous System Ischemic Response.


2. Describe chemical sensitivities of arterial and cardiopulmonary chemoreceptors,
and the responses to these chemicals.
3. Describe the role of hypothalamus in blood pressure regulation.
4. Describe the role of stress relaxation and reverse stress relaxation in the regulation
of blood pressure.
5. Describe the role of the Renin-Angiotensin-Aldosterone system in the regulation
of arterial blood pressure.
6. Describe the role of Atrial natriuretic factor (ANP) in the regulation of arterial
blood pressure.
7. Describe the mechanisms which maintain, arterial blood pressure normal in
response to varying intakes of sodium (dEi/dt).

I. OVERVIEW

We will begin by looking at some receptors that can influence the cardiovascular
system that are not baroreceptors but instead are chemoreceptors. Then we will
discuss how two different hormonal systems, the renin-angiotensin-aldosterone
system and the atrial natriuretic peptide (ANP) system can influence the renal
function curve and thereby play a major role in the regulation of blood pressure.

II. ARTERIAL PERIPHERAL CHEMORECEPTORS MONITOR PLASMA


OXYGEN AND CARBON DIOXIDE PRESSURES IN ARTERIAL
BLOOD.

Peripheral receptors for plasma pO2, and pCO2, are found in close proximity to
the carotid sinus baroreceptors and aortic baroreceptors and are named the carotid
bodies and aortic bodies respectively. The major function of these receptors is to
increase respiration in responses to decreases in oxygen tension (pO2) and
increases in carbon dioxide tension (pCO2). Activation of these receptors also
activates the vasomotor center. This activation is, however, weak as compared to
the central ischemic response to be described later. If the chemoreceptors are
activated at the same time that the baroreceptors are responding to a decrease in
blood pressure the actions of the two signals synergize; i.e., the blood pressure
response is greater than the sum of the actions of the arterial baroreceptor and
arterial chemoreceptor.

III. THE CENTRAL NERVOUS SYSTEM (CNS) ISCHEMIC RESPONSE,


WHICH OPERATES IN RESPONSE TO CENTRAL
CHEMORECEPTORS, IS THE MOST POWERFUL ACTIVATOR OF
THE SYMPATHETIC TETRALOGY.

504
Regulation of Arterial Blood P ressure 2 - Dr. Ford

When blood flow to the vasomotor center is decreased to the point of ischemia,
the vasomotor center becomes strongly excited (the excitation may be due to a
failure to remove CO2, rather than a failure to supply nutrients, particularly O2).
This leads to strong activation of the sympathetic tetralogy which produces a
strong increase in CO (cardiac output), TPR (total peripheral resistance), and Pa
(arterial pressure). Mean arterial pressure may rise as high as 270 mmHg. The
effect of activating the sympathetic tetralogy on the kidney may be so strong that
blood flow to this organ ceases. This reflex may be activated when Pa falls to 60
mmHg or less. The Pa must be low enough to cause low blood flow to the
vasomotor center.

The Cushing reaction is a special case of the Central Ischemic Response. In the
Cushing reaction an increase in cerebrospinal fluid pressure is the cause of the
increases in intracranial pressure. The increase in intracranial pressure decreases
the caliber of capillaries and veins thereby increasing the resistance to blood flow
in the brain. The decreased cerebral blood flow activates the vasomotor center and
sympathetic tetralogy. The increase in Pa produced by the activation of the
vasomotor center will persist as long as the increase in cerebrospinal fluid
pressure persists. This cerebrospinal fluid pressure/CNS ischemic response is
called the Cushing Reaction after the surgeon who first described it.

IV. THE INTEGRITY OF THE HYPOTHALAMUS IS ESSENTIAL TO THE


PROPER FUNCTIONING OF LOWER BRAINSTEM AREAS AND
PRODUCES EFFECTS OF ITS OWN ON BLOOD PRESSURE.

Behavioral and emotional effects on blood pressure are generally processed


through the hypothalamus. Being scared into a dead faint for example involves
CNS-induced depression of the vasomotor center. Experimentally general
stimulation of the anterior hypothalamus produces hypertension and bradycardia
while general stimulation of the posteriolateral region of the hypothalamus
produces hypertension and tachycardia.

V. THE PLASTICITY OF THE VASCULATURE ALSO PROVIDES SOME


PROTECTION AGAINST BLOOD PRESSURE CHANGES INDUCED BY
RAPID INCREASES AND DECREASES IN BLOOD VOLUME.

Rapid changes in blood volume can lead to rapid changes in Pms, and thus in CO,
and finally in Pa. Such changes in blood volume can occur as the result of
hemorrhage or the over transfusion of blood.

Stress relaxation. When blood volume is increased, smooth muscle passively


stretches increasing unstressed vascular volume and decreasing stressed volume.
Stressed volume is sometimes referred to effective circulating volume in some

505
Regulation of Arterial Blood P ressure 2 - Dr. Ford

texts. There is a resultant decrease in Pms, CO, and Pa. The increase in unstressed
volume, due to passive stretch of vascular smooth muscle is called stress
relaxation. Stress relaxation can provide about a 50 per cent compensation for the
blood pressure increase produced by an increase in blood volume.

Reverse stress relaxation. When blood volume is decreased, smooth muscle


passively decreases in length and vascular unstressed volume decreases. This
compensatory decrease in unstressed volume, termed reverse stress relaxation,
produces a compensatory increase in vascular stressed volume, and a
compensatory increase in Pms, Q, and Pa. The degree of decrease in blood
volume which can be partially compensated by this mechanism is much lower
than the compensation which can be achieved for increases in blood volume.

VI. HORMONAL REGULATION OF ARTERIAL BLOOD PRESSURE: THE


RENIN, ANGIOTENSIN, ALDOSTERONE SYSTEM AND ATRIAL
NATRIURETIC PEPTIDE.

Baroreceptors are the primary regulators of blood pressure over the short term.
Over the longer term the renin-angiotensin-aldosterone (RAA) system is thought
to be the primary regulator of arterial blood pressure. As with the baroreceptors,
this system operates on components of the renal-body fluid system. The
components primarily affected are: 1) the kidney which regulates the volume of
extracellular fluid (E) and hence CO, and 2) TPR. Renin is an enzyme released by
the kidney. Renin enzymatically cleaves angiotensin-I from angiotensinogen.
Angiotensin-I is converted to angiotensin-II (the active peptide) by the action of
the angiotensin converting enzyme (ACE). Angiotensin-II stimulates the release
of aldosterone, a steroid hormone from the adrenal cortex. This is the second
active principle in the system.

A. RENIN RELEASE

Three signals, all of which can arise as a consequence of a fall in arterial


blood pressure, are primarily responsible for the release of renin. They are:
1) a decrease in renal perfusion pressure (renal arterial pressure) sensed by
renal baroreceptors in renal afferent arterioles (see Fig. 1), 2) a decrease in
the volume, or sodium composition of fluid reaching the macula densa
(see Fig. 1), and 3) an increase in β- receptor -mediated sympathetic neural
activity to Juxtaglomerular cells (see Fig. 1). (Recall that decreases in Pa
increase sympathetic neural activity.)

506
Regulation of Arterial Blood P ressure 2 - Dr. Ford

Figure 1. Schematic of the relation between Macula Densa cells, Juxtaglomerular cells, glomerulus, and
afferent and efferent arterioles of the kidney. (See Figs. 34-4, p. 627 and 36-5, p. 676 in Berne et al,
“Physiology”, 5th Edition). See also Fig. 4-33 in Costanzo, “Physiology”, 3rd Edition, p. 160.

In response to one or more of three signals renin is released from


Juxtaglomerular cells lining the afferent arterioles of the kidney. The
signal is believed to arise as the product of a decrease in the sodium load

507
Regulation of Arterial Blood P ressure 2 - Dr. Ford

to the kidney (sodium load = plasma sodium concentration x glomerular


filtration rate). A decrease in sodium load can be due either to a decrease
in glomerular filtration rate or a decrease in plasma sodium concentration,
or both. All of these signals can arise as a consequence of a fall in
extracellular fluid volume because extracellular fluid volume is a major
determinant of cardiac output and hence arterial pressure.

B. ATRIAL NATRIURETIC PEPTIDE (ANP) AND THE RELEASE OF


RENIN

Atrial Natriuretic Factor, also called atrial natriuretic peptide or (ANP), is


a peptide which can suppress the release of renin. ANP is found in the
atria of the heart and released by increases in atrial pressure. The signal is
generated by an increase in stretch on the atrial fibers. Stretch on atrial
fibers, for example by over expansion of extracellular fluid volume, blood
volume, or volume buildups secondary to decreases in cardiac contractility
release ANP

The physiological functions of ANP are not fully understood. The current
view is that ANP is a potent inhibitor of smooth muscle contraction. A
particularly important site of this action is the afferent arteriole of the
kidney. Relaxing the afferent arteriole produces an increase in pressure on
renal baroreceptors and increases glomerular filtration rate. Both of these
effects inhibit the release of renin. Evidence exists that ANP can also
block the release of renin by a direct action. ANP has also been shown to
increase the renal excretion of sodium by direct inhibition of renal tubular
sodium transport mechanisms.

C. ANGIOTENSIN-I (A-I) AND ANGIOTENSIN II (A-II) FORMATION


Renin, a proteolytic enzyme converts angiotensinogen, a plasma protein
released from the liver, to angiotensin-I (A-1), Fig.1. A-I is converted to
A-II by angiotensin converting enzyme (ACE) found in vascular
endothelium of the lung and other tissues. The rate limiting step in the
formation of A-II is the release of renin. Thus, the levels of A-II in the
blood correlate directly with levels of renin.

D. ACTIONS OF ANGIOTENSIN-II

Angiotensin-II produces two important effects. The first, and perhaps most
important effect is to decrease the rate of loss of extracellular fluid
(dEo/dt) through the kidney. This can be diagramed as a shift in the renal
function curve to the right. That is for any given Pa the rate of loss of
extracellular fluid (dEo/dt) is decreased. The second effect is to constrict
arterioles, including the arterioles of the kidney. A-II is not a potent
constrictor of veins. The effect of the generalized constriction of arterioles
is to increase TPR (total peripheral resistance). A-II is one of the most

508
Regulation of Arterial Blood P ressure 2 - Dr. Ford

potent naturally occurring vasoconstrictors known.

E. ALDOSTERONE SECRETION AND ITS EFFECTS

Aldosterone is synthesized and released from glomerular cells of the


adrenal cortex primarily in response to A-II and increases in plasma
potassium.

Effects of aldosterone. The important action of aldosterone is to decrease


renal excretion of sodium and thus extracellular fluid (dEo/dt). This is
equivalent to a shift in the renal function curve to the right. See Figure 2
below.

Figure 2. shows two renal function curves: one at zero angiotensin-II concentration and one at an angiotensin
level 2.5 times normal. (Note: aldosterone produces similar shifts in renal function curves.) The rate of
extracellular fluid excretion (dEo/dt) or sodium intake (dEi/dt), in units of times normal, is shown on the
ordinate. Arterial pressure is shown on the abscissa. The straight horizontal line is drawn for a sodium intake,
dEi/ dt, 1 times normal. The arterial pressures required to excrete sodium (dEo/dt) at the rate of sodium intake
are the pressures at which the horizontal lines intersect the two renal function curves.

VII. INFLUENCE OF ATRIAL NATRIURETIC PEPTIDE ON RENAL


FUNCTION

By its very name, one would predict that a “natriuretic” factor would have an
opposite effect to the renal-angiotensin-aldosterone axis, particularly a factor such
as ANP which also acts as a vasodilator as well. Indeed its overall effect is to
promote the loss of extracellular fluid (↑ dEo/dt). This would effective shift the

509
Regulation of Arterial Blood P ressure 2 - Dr. Ford

renal function curve to the left as shown in Figure 3 below.

Notice I have included ADH on this figure. ADH is involved with a system
primarily regulating plasma osmolarity or water content. Naturally that can’t be
divorced from Na+ regulation since Na+ is the major osmotically active substance
in our body. The primary action of ADH, as its name implies, is antidiuresis To
retain water, one must retain Na+. Hence ADH has basically the same effect as the
RAA system.

Figure 3. The effects of both the RAA, ADH, and ANP systems on renal function curves.

VIII. COMPENSATION FOR CHANGES IN DIETARY INTAKE OF SODIUM


IS ACHIEVED BY THE COMBINED EFFECTS OF ANGIOTENSIN-II
AND ALDOSTERONE ON RENAL EXCRETION OF SODIUM.

When the renal-body fluid system for regulating blood pressure was discussed
little attention was paid to the consequences of changes in dEi/dt, i.e. the input of
sodium. Without the intervention of some control system increases in dEi/dt can
only be compensated by increases in Pa until dEo/dt = dEi/dt.

At this point we start a discussion of how a constant blood pressure can be


maintained in the face of changing inputs of sodium (dEi/dt). Recall that changes
in E (extracellular fluid) are regulated primarily by regulating the excretion of
sodium. Secondary reflexes maintain osmolarity constant by excreting or
conserving water as necessary. Recall also that in the steady state the rate of loss
of extracellular fluid (sodium) must equal the rate of gain of extracellular fluid.
The solution to this problem involves the release of renin and formation of A-II
and release of aldosterone in a reciprocal inverse relationship to sodium intake.

510
Regulation of Arterial Blood P ressure 2 - Dr. Ford

Figure 4. Arterial pressures, renal function curves, and angiotensin-II (aldosterone) levels at different rates of
sodium intake.

The problem before us is to discern how the body manages to maintain arterial
blood pressure normal in the face of changing sodium input. To understand the
answer two principles need to be recalled: 1) increases in sodium load to the
kidney decrease renin secretion and therefore blood levels of angiotensin-II , and
2) decreases in angiotensin-II shifts the renal function curve to the left (Fig. 2).
Figure 4 shows the 4 levels of angiotensin-II (in units times normal) which are
produced by 4 levels of sodium intake (in units times normal). Sodium intake is
shown by the horizontal lines within the graph.

Note that as sodium intake increases angiotensin-II levels decrease. Fig. 4 also
shows the 4 renal function curves which are produced by the 4 levels of
angiotensin-II. Note that as angiotensin-II levels decrease the renal function
curves are shifted to the left. The arterial pressures produced by the intersection of
the sodium input lines and the renal function curves identify the arterial pressures
required to balance sodium input with sodium output. Note that over a 40 fold
range of sodium intake (0.25 - 10 times normal) the arterial pressure (Pa) required
to match sodium excretion to sodium intake does not change because the levels of
angiotensin-II are changed to match the changing sodium intake. Subjects unable
to make these shifts are classified as salt-sensitive.

IX. ALTERNATE LEARNING RESOURCES

1. Berne, R. M., Levy, M. N., Koeppen, B. M. , and Stanton, B. A.


"Physiology", 5th Edition, 2004. The renin, angiotensin, aldosterone

511
Regulation of Arterial Blood P ressure 2 - Dr. Ford

system, and atrial natriuretic peptide (ANP) are covered on pages 672 -
683. Also blood pressure changes in response to signals from: peripheral
chemoreceptors, hypothalamus, cerebrum, skin and viscera, pulmonary
reflexes, and central chemoreceptors are covered on page 391-392.

2. Guyton, "Textbook of Medical Physiology", W.B. Saunders, 8th Edition,


1991. Chapter 19, pp 205-220.

3. Scher, A. in "Textbook of Physiology", Patton et al. editors, W. B.


Saunders Co., Philadelphia, 1989, Volume 2, Chapter 51, pp. 972-990.

4. Costanzo, L. S. , “Physiology”, 3rd Edition, 2006, pp. 159-163.

X. PRACTICE PROBLEMS

Identify each true answer. More than one answer may be true. Abbreviations:
Pms, mean systemic pressure; TPR, total peripheral resistance; Q, cardiac output;
A-II angiotensin-II; E, extracellular fluid; dEo/dt, rate of loss of extracellular
fluid.

1. A rise in arterial blood pressure would produce which changes in the


plasma concentrations of these hormones?

A. increase vasopressin, increase norepinephrine, increase A-II


B. increase vasopressin, decrease norepinephrine, increase A-II
C. increase vasopressin, decrease norepinephrine, decrease A-II
D. decrease vasopressin, decrease norepinephrine, decrease A-II

2. A fall in arterial blood pressure would produce which set of changes?

A. increase renin, increase aldosterone, shift the renal function curve


to the right
B. decrease renin, increase aldosterone, shift the renal function curve
to the right
C. decrease renin, decrease aldosterone, shift the renal function curve
to the right
D. decrease renin, decrease aldosterone, shift the renal function curve
to the left

3. An increase in salt and water intake (dEi/dt) would produce which


changes in a cardiovascular system under neuronal and hormonal control?

A. decrease A-II decrease aldosterone, shift renal function curve to


right
B. decrease A-II, decrease aldosterone, shift renal function curve to
left

512
Regulation of Arterial Blood P ressure 2 - Dr. Ford

C. increase A-II decrease aldosterone, shift renal function curve to


right
D. increase A-II, increase aldosterone, shift renal function curve to
left

ANSWERS

1. D
2. A
3. B

513
Exercise and Hemorrhage - Dr. Ford

Exercise and Hemorrhage


George D. Ford, Ph.D.

OBJECTIVES:

1. Identify the parameters in the Fick diffusion equation which the cardiovascular
system modifies to increase oxygen delivery to exercising muscle.
2. Describe the 3 phases of the response of the cardiovascular system to exercise and
identify for each phase the major changes taking place and the mechanisms by
which these changes take place.
3. Describe how blood flow to muscle and cardiac output are changed during
exercise.
4. Describe the three major phases in the responses to hemorrhage and identify the
mechanisms operating in each phase.

I. EXERCISE

A young adult male or female at rest has a cardiac output of about 5 liters per
minute. When this individual exercises strenuously, using a large fraction of
his/her muscle mass, cardiac output increases to about 23 liters per minute, a 4.6
fold increase in cardiac output. Blood flow to muscle increases from about 1 liter
per minute at rest, to 18 liters per minute during heavy exercise. During this very
large increase in cardiac output blood pressure increases only moderately, if at all,
from 10 to 60 percent depending on the type and severity of exercise. To maintain
this flow heart rate will increase nearly 3fold, stroke volume will increase nearly
75%, and the ejection fraction will increase nearly 50%. We will discuss the
mechanisms accounting for these changes.

A. THE PROBLEM DURING EXERCISE IS TO MATCH SUBSTRATE


SUPPLY TO SUBSTRATE UTILIZATION

During exercise sufficient flow of oxygen and metabolic substrates must


be delivered to muscle to provide energy at a rate equal to the rate of its
utilization. When enough oxygen is delivered, sufficient substrates are
probably also provided. The problem is to provide sufficient oxygen to
maintain oxidative phosphorylation at a rate equal to the rate of ATP
hydrolysis. Figure 1 below shows the excellent correlation between
cardiac output and metabolism that does occur:

514
Exercise and Hemorrhage - Dr. Ford

Figure 1. Change in cardiac output as a function of oxygen uptake during


exercise. As oxygen consumption increases from its normal value of 250ml/min to
close to 5 L/min with increasing exertion, cardiac output increases from 5 L/min to
almost 30 L/min. (Taken from Åstrand, et al, J. Appl. Physiol. 19:268-274, 1964.)

B. THE SOLUTION TO THE OXYGEN DELIVERY PROBLEM IS TO


OPTIMIZE THE PARAMETERS IN THE FICK DIFFUSION
EQUATION.

The Fick equation identifies the parameters which determine the rate at
which substrates diffuse. In the case of the cardiovascular system we are
concerned with the rate of diffusion of substrate, e.g., oxygen, from the
lumen of capillaries to mitochondria. This rate must match the rate of
oxygen consumption. The equation relating these two, the rate of oxygen
consumption to the rate of oxygen delivery is:

M O2 = rate of oxygen utilization


AO2 = area of oxygen diffusion pathway (capillary surface area)
DO2 = diffusion coefficient for oxygen
XO2 = distance from capillary to mitochondria
pO2 capillary = oxygen tension in capillary blood
pO2 cell = oxygen tension at mitochondria

515
Exercise and Hemorrhage - Dr. Ford

The rate at which oxygen diffuses from capillaries to tissues is increased


by increasing the number of open capillaries. This changes the following
variables in the Fick diffusion equation: increase AO2, decrease in
effective XO2, and increase pO2capillary. This latter increase may not be
obvious. It is due to the increased flow that is largely due to the increased
cardiac output. Hence the excellent correlation shown in Fig. 1.

C. THERE ARE 3 MAJOR PHASES IN THE RESPONSE OF THE


CARDIOVASCULAR SYSTEM TO EXERCISE.

1. Phase 1. (mechanical phase)

During this phase the muscles of the body are tensed. Those of the
abdomen are of greatest importance because the increase in abdominal
pressure compresses the abdominal veins and thereby decreases
unstressed vascular volume. The consequent increases in Pms (mean
system pressure) causes an increase in cardiac output.

2. Phase 2. (central-command/reflex phase)

The origin of the drive for the changes which take place during this
phase is uncertain. There probably are reflexes from lungs, joints,
muscles, and baroreceptors and central drive from the motor cortex to
the vasomotor center. The earliest effect is an increase in heart rate due
to withdraw of vagal tone. Next there is a strong increase in
sympathetic tone to the heart and vasculature. During the progress of
exercise there is also an increase in venous return because the
contraction of skeletal muscles between venous valves propels blood
back to the heart (increase in stressed volume).

3. Phase 3. (active hyperemia)

Phases 1 and 2, which take place during the first about 30 seconds of
exercise, produce about a 50 percent increase in cardiac output. This is
far less than the approximate 460 percent increase in cardiac output
observed during heavy exercise. The remaining 410 percent increase in
cardiac output is largely due to a 60 to 75 per cent decrease in total
peripheral resistance. The decrease in total peripheral resistance is due
to the active hyperemic response of arterioles in contracting muscle,
and may also be due in part to a decrease in sympathetic tone to the
vasculature of exercising muscles. Also included in the decreased TPR
is the increase in the number of open capillaries. Recall that resistance
decreases as branching increases.

Fig. 2 shows in graphic form the changes in cardiac and vascular


function which occur during the three phases of exercise. It is

516
Exercise and Hemorrhage - Dr. Ford

important to recognize that the events initiated during phase 1 and 2


continue operating during phase 3 of the response. That is the
mechanical and sympathetic phases continue to contribute to the
increase in cardiac output during the third (hyperemic) phases of
exercise.

Figure 2. Response of the cardiovascular system to exercise.


Curve A shows the vascular function and cardiac function curves for the system at rest.
Curve B shows the vascular function curve following the activation of phase 1.
Curve C shows the vascular and cardiac function curves following activation of phase 2.
Curve D shows the vascular function curve following the activation of phase 3.

D. BLOOD FLOW IS PARTITIONED DURING EXERCISE.

Fig. 3 shows the partition of blood flow during exercise.

517
Exercise and Hemorrhage - Dr. Ford

Figure 3. The partition of blood flow during exercise. Exercise intensity is directly
proportional to the rate of oxygen consumption. (Redrawn from Ruch, H. P. and Patton, T. C.:
Physiology and Biophysics, 20th Ed., 1974). It is also Fig. 24-1, p. 434 of Berne et al
“Physiology”, 5th Edition, 2003.

Muscle shows the greatest change in blood flow as exercise intensity


increases. In this particular study muscle blood flow increased from 1
liter/min to 19 liters/min.

Blood flow to the viscera decreases from 3 liters/min to about 0.5


liters/min because of increased sympathetic stimulation to these organs.

Blood flow to the brain is virtually unchanged during the increase in


exercise intensity.

Coronary blood flow increases because the amount of work done by the
heart increases with increasing cardiac output and increased arterial blood
pressure. With increased work there is an increase in the production of
vasodilator metabolites.

Initially blood flow to the skin decreases. As exercise intensity increases


the heat input to the body increases. Blood flow to the skin is increased,
except at the highest levels of exercise, to augment heat loss.

E. ARTERIAL BLOOD PRESSURE RISES DESPITE THE DECREASE IN


TPR.

At first one would conclude that blood pressure must fall during exercise

518
Exercise and Hemorrhage - Dr. Ford

due to the decreased TPR. This would be counter productive since you
would lose perfusion pressure for the flow to the exercising muscles. But
remember the “dangerous equation”: Part = C.O. x TPR. The fall in TPR
is more than counterbalanced by the increase in cardiac output. Blood
pressure typically rises 10 to 60% depending on the subject’s fitness and
duration of exercise.

You may also ask why the baroreceptor mechanism is not invoked by this
increase in blood pressure. It is reset centrally just as the body temperature
set point.

F. LIMITS TO EXERCISE INTENSITY.

Severe exercise increases body temperature and increases lactic acid


accumulation thus decreases pH. The decrease in muscle pH is probably
the cause of pain in exercising muscle and limits the level of exercise
tolerated. The need for some blood flow to the skin to dissipate the heat
may be a major limiting factor in how much of the cardiac output is
actually available to the exercising muscles.

In total body exercise, in contrast to exercising one limb, exercise is


limited by the limits to cardiac output. Cardiac output limits exercise
intensity because it limits the delivery of oxygen. Oxygen supply is the
ultimate limit on exercise intensity.

G. THE EFFECT OF TRAINING

As a general rule of thumb, training leads to small increases (10 to 25%)


in normal stroke volume. How this is accomplished depends on the type of
training. Static training such as weight lifting generally increases the
myocardial wall thickness and mass allowing for decreased end systolic
volumes. Dynamic training such as running or swimming is usually
accomplished by chamber enlargement. Thus work is done higher on the
length-tension curve.

II. HEMORRHAGE

The responses of the cardiovascular system to hemorrhage is frequently divided


into two major types, hemorrhage which is successfully defended, called
hemorrhagic shock, and hemorrhage which is not successfully defended, called
irreversible hemorrhagic shock. We will spend most of our time on the successful
defense. Hemorrhage, like exercise, proceeds in 3 stages.

519
Exercise and Hemorrhage - Dr. Ford

A. THE IMMEDIATE EFFECTS OF HEMORRHAGE ARE A RAPID


REDUCTION IN CARDIAC OUTPUT AND ARTERIAL BLOOD
PRESSURE

The decrease in blood volume decreases mean systemic pressure. The


decrease in mean systemic pressure decreases cardiac output and to a
minor extent total peripheral resistance by autoregulatory mechanisms.
The severity of the effect of hemorrhage depends upon the per cent of
blood volume lost.

Figure 4. Effect of blood loss on cardiac output and arterial pressure.

Two points are apparent from this figure: 1) pressure is maintained better
than cardiac output, and 2) when 30-35 per cent of blood volume is
removed a plateau in pressure and output is reached. The reflexes which
maintain pressure are primarily the baroreceptor reflexes. It is believed
that the plateau, at low pressure, is achieved by the entrance of
chemoreceptor reflexes and the central ischemic responses. With reflexes
intact about 35 per cent of blood volume can be removed. In the absence
of these reflexes only about 17 per cent of the blood volume can be
remove before cardiac output drops to zero.

B. THE COMPENSATORY RESPONSES TO HEMORRHAGE.

It is convenient to describe the responses of the cardiovascular system to


hemorrhage in terms of the rapidity of the response.

520
Exercise and Hemorrhage - Dr. Ford

1. The fast (seconds to minutes) responses to hemorrhage are reflex or


neural in origin.

Table I lists the fast responses, the arterial pressures at which they
operate, and their gain. The higher the gain the more complete the
compensation.

Table 1. Reflex responses to changes in arterial pressure.


Pressure Range
Reflex Gain
(mmHg)
Baroreceptor 60-180 7
Chemoreceptor 40-80 4
CNS Ischemic 20-40 11
Vasopressin ? ?

The baroreceptor, chemoreceptor, and CNS ischemic receptors


activate the vasomotor system and thus the sympathetic tetralogy.
The sympathetic tetralogy includes the following responses, all
mediated by the sympathetic nervous system: a) increase in heart
rate (also mediated by a decrease in parasympathetic drive to the
heart), b) decrease in unstressed volume and consequent increase
in mean system pressure, c) increase in total peripheral resistance
(TPR), and d) increase in cardiac contractility.

2. Intermediate responses (minutes to hours).

Three responses take place during this interval.

a. Reverse stress relaxation.

When blood is removed from the vascular system the


unstressed volume slowly decreases, a phenomena termed
"reverse stress relaxation". Reversed stress relaxation increases
mean system pressure. This increases cardiac output.

b. Renin-angiotensin mechanism.

The fall in arterial blood pressure: decreases renal arteriolar


pressure, decreases filtered sodium load, and increases
sympathetic activation of juxtaglomerular cells. Each of these
changes contributes to the release of renin from
juxtaglomerular cells. The gain of the renin-angiotensin-II
system is about 3 and it does not, therefore, provide strong
compensation for the loss of blood volume.

521
Exercise and Hemorrhage - Dr. Ford

c. Capillary fluid shift

During hemorrhage plasma and red blood cells are lost. Plasma
is one of the compartments composing extracellular fluid. The
other compartment is interstitial fluid. Extracellular fluid can
shift across capillaries from plasma to interstitial fluid or from
interstitial fluid to plasma, depending upon hydraulic and
oncotic pressures across the capillary. Recall from the Starling-
Landis equation that the flow of fluid across the capillary
(radial flow, Jv) is determined by a filtration coefficient (K),
the hydrostatic pressures in the capillary (Pc) and in the
interstitium (Pi), and the oncotic pressures in plasma (Πp) and
interstitial fluid (Πi), as shown in the following equation:

Jv = K[(Pc - Pi) - (Πp -Πi)]

Normally Jv, is positive, that is fluid flows from blood to


interstitium. Following hemorrhage Pc drops for three reasons:
1) venous pressure (P) falls because of the fall in blood
volume, 2) arterial pressure (Pa) falls because of the fall in
cardiac output, and 3) the ratio (Rv/Ra) of venous resistance
(Rv) to arteriolar resistance (Ra) falls because of sympathetic
nervous system activation of arteriolar smooth muscle. The
precise relationship between these factors is given by the
following equation:

Pc = (Rv/Ra)Pa + Pv
I + Rv/Ra

The increase in blood volume produced by a shift of interstitial


fluid into blood helps to restore blood volume. The shift of
interstitial fluid to plasma is the cause of the decrease in
hematocrit which follows an episode of hemorrhage. The gain
of this compensatory mechanism is 2, and it is not a strong
compensatory system.

The shift of interstitial fluid to blood increases the ratio of


blood volume to extracellular fluid volume. This is represented
in block 8 of the renal-body fluid mechanism as a
counterclockwise rotation of the ratio of blood volume to
extracellular fluid volume as show in Fig. 5.

522
Exercise and Hemorrhage - Dr. Ford

Figure 5. The effect of the fluid-shift mechanism on the ratio between blood volume
and extracellular fluid volume. The solid line shows the relation before hemorrhage.
The dashed line shows the relation after the fluid shift.

d. Vasopressin.

With a sharp decline in arterial pressure vasopressin is released


from the posterior pituitary. This potent constrictor of vascular
smooth muscle contracts both veins and arterioles, decreasing
unstressed volume and increasing TPR respectively.

3. The slow (hours to days) renal-body fluid response promoted by


angiotensin-II and aldosterone returns blood volume to normal.

All of the compensatory mechanisms described above have gains of


less than infinity and cannot completely compensate for hemorrhage.
The renal-body fluid system has a gain of infinity and is capable of
returning blood pressure to normal.

C. OTHER COMPENSATORY RESPONSES TO HEMORRHAGE.

1. Restoration of red blood cells.

When blood is lost all the components of blood are lost. Thus red and
white cells and plasma are lost. To return blood to its original
composition requires the production of red and white blood cells as
well as plasma proteins. As sodium, other salts, and water are returned
to extracellular fluid the concentration of plasma proteins and blood
cells are reduced. The low red blood cell concentration is detected by
the kidney which releases the hormone, erythropoietin. Erythropoietin
is transported to bone marrow where it stimulates the maturation and
release of red blood cells.

523
Exercise and Hemorrhage - Dr. Ford

2. Restoration of plasma proteins.

The reduction in plasma proteins is detected by the liver which is


thereby stimulated to increase their production.

D. PARTITIONING OF RESPONSES.

The intensity of the different compensatory mechanisms; fast,


intermediate, and slow; which respond to hemorrhage, change with time.
Initially the reflex mediated responses dominate. As time elapses the
intermediate responses begin and the intensity of the reflex mediated
responses diminish. As the slow, renal-body fluid mechanism, is brought
into action the intensity of the intermediate and fast response systems are
diminished. There continues, however, to be overlap in the response
mechanisms until blood pressure is returned to normal.

E. IRREVERSIBLE HEMORRHAGIC SHOCK.

If hemorrhagic shock persists the condition can become irreversible. This


can occur if too much time elapses between hemorrhage and transfusion,
even if large amounts of blood are transfused. Immediately following the
transfusion of these large volumes of blood, blood pressure can return to
normal. After several hours, however, arterial pressure declines again and
death ultimately ensues. It is clear that during the period of hemorrhagic
shock pathological processes are initiated which can not be reversed by
blood transfusion. The pathophysiology of irreversible shock is not well
understood. The following observations have been made on this
extensively studied problem.

1. Cardiac failure.

In animal models of irreversible hemorrhagic shock, low blood


pressure and strong reflex mediated vasoconstriction of visceral organs
produces severe ischemia in these organs. It is hypothesized that these
organs release a substance tentatively called "myocardial depressant
factor" which produces myocardial failure. It is believed that this heart
failure may then worsen because of the low cardiac output associated
with it. There is thought to be a positive feedback loop in which the
situation worsens for each turn of the loop, e.g. a fall in cardiac output
of 10 per cent produces a fall in cardiac output of 20 per cent which
then produces a fall of 40 per cent and so on.

2. Acidosis.

The decreased cardiac output associated with hemorrhagic shock


decreases the rate at which the kidney removes hydrogen ion from the

524
Exercise and Hemorrhage - Dr. Ford

body. Hydrogen ion therefore accumulates. This hydrogen ion relaxes


vascular smooth muscle, both veins and arterioles, and decreases their
sensitivity to circulating catecholamines. These actions increase
unstressed volume and decrease TPR, which further decreases cardiac
output and arterial pressure.

3. Central nervous system depression.

When brain blood flow is sufficiently depressed brain function is


depressed. The resultant decrease in vasomotor function decreases the
intensity of the sympathetic tetralogy and all of its accompanying
sequellae.

When the adrenal medulla is activated by the sympathetic nervous


system enkephalins are released along with catecholamines. The
enkephalins depress the brainstem. Naloxone, an enkephalin
antagonist, is used in the treatment of hemorrhagic shock.

4. Reticuloendothelial system.

The reticuloendothelial system is composed of tissues; spleen, lung,


Kuffer cells, lymph nodes, microglial cells; containing fixed
macrophages. The activity of this system is dependent upon a complex
of plasma proteins know as complement. Complement activates
macrophages. A depression in the activity of macrophages, due to a
decrease in the blood levels of complement, allows bacteria from the
gut and elsewhere to multiply and gain access to the internal organs of
the body. A bacterial cell wall lipopolysaccharide (LPS), also called
endotoxin, is released during this invasion by bacteria. Endotoxin
produces most of the symptoms of hemorrhagic shock and was once
believed to be the primary cause of irreversible shock. The observation
that aseptic animals also develop hemorrhagic shock weakened that
hypothesis. In the literature you will see reference to two kinds of
shock, hemorrhagic and septic. The pathophysiology is quite similar
but in the latter case endotoxin is known to be present.

III. RECOMMENDED READING

1. Exercise
Berne, R. M., Levy, M. N., Koeppen, B. M., Stanton, B. A., "Physiology",
5th Edition, 2004, pp. 433 - 442.

Guyton, Jones and Coleman, "Circulatory Physiology; Cardiac Output and


its Regulation", 1973, W.B. Saunders, Philadelphia, pp 436-450.

525
Exercise and Hemorrhage - Dr. Ford

Costanzo, L. S., “Physiology”, 3rd Edition, 2006. pp 171-176.

2. Sports Physiology

Guyton, "Textbook of Medical Physiology", 8th Edition, 1991, W.B.


Saunders, Philadelphia, pp. 940 - 949. This is an interesting chapter
dealing with many aspects of exercise. Included are: diet, energy sources,
oxygen debt, training, cardiovascular system, fluids and salts, muscle
types, exercise types, and drugs. If you are into running or other types of
exercise you will find the information in this chapter of interest.

3. Hemorrhage
Berne, R. M., Levy, M. N., Koeppen, B. M., and Stanton, B. A.,
"Physiology", 5th Edition, 2004: pp 433 - 442 but especially 437 - 441.

IV. PRACTICE PROBLEMS

Identify each true answer. There may be more than one.

1. The cardiovascular adjustments which take place in response to exercise


optimize the parameters of the Fick diffusion equation. These adjustments
include which of the following changes?

A. an increase in the diffusion constant for oxygen


B. an increase in the diffusion distance for oxygen
C. an increase in muscle arteriolar contraction
D. an increase in the area for capillary oxygen diffusion

2. Which of the following changes occur during exercise?

A. Arterial pressure is increased in part by an increase in sympathetic


nervous system activity.
B. Resistance to blood flow in exercising muscle decreases 10 - 20
fold.
C. Arterial baroreceptor reflexes are involved in increasing cardiac
output.
D. Release of vasodilator substances relaxes muscle arterioles which
increases cardiac output.

3. Exercise provides excellent examples of which of the following?

A. the effect of total peripheral resistance on cardiac output


B. metabolic regulation of organ blood flow
C. local regulation of blood flow
D. neural regulation of blood flow

526
Exercise and Hemorrhage - Dr. Ford

4. A fall in blood pressure, as seen following severe hemorrhage, will cause


which of the following changes, either directly or indirectly as a
compensatory mechanism (all changes are relative to the pre-hemorrhage
condition)?

A. a decrease in baroreceptor firing rate


B. an increase in the rate of secretion of renin
C. a decrease in unstressed vascular volume
D. a decrease in net filtration of fluid out of capillaries

5. The response to hemorrhage includes which of the following changes?

A. a increase in the release of atrial natriuretic factor


B. a shift of interstitial fluid into blood
C. an increase in the rate of secretion of renin
D. a decrease in the activity of the sympathetic nervous system

ANSWERS

1. D (not A, the diffusion coefficient is a constant; not B, opening more capillaries


actually decreases the mean distance for diffusion; not C, the active hyperemia
would tend to dilate, not constrict arteriolar smooth muscle; yes D, opening more
capillaries will increase the surface for diffusion)

2. A, B, C, D (yes A, phase 2; yes B, phase 3; yes C, without this blood pressure


would fall due to the massive decrease in TPR and perfusion would be impaired;
yes D, active hyperemia in phase 3)

3. A, B, C, D (yes A, phase 3; yes B, active hyperemia in phase 3;yes C and D, see


distribution shifts in Figure 3)

4. A, B, C, D (yes A because BP falls; yes B because renal perfusion falls; yes C


because volume is loss and the sympathetic activity causes an unstressed to
stressed shift; yes D becuse the reflex constriction at the arteriolar level will
decrease the pressure of blood entering into the capillary

5. B, C (not A, ANF release is inhibited; yes B, see option D in question above; yes
C, see option B above; not D, the baroreceptor reflex will increase sympathetic
activity)

527
Cardio Phys Problem Solving 2 - Dr. Pittman

Cardiovascular Physiology Problem Solving 2


Roland Pittman, Ph.D.

1. A patient comes to your office for a routine physical. You find that his blood
pressure is 185/135 mm Hg, and his heart rate is 75 bpm. In the process of getting
a history, you discover that this patient has a high dietary intake of NaCl. In order
to diagnose the cause of the high blood pressure, you send him for laboratory
studies which show the following:

A. cardiac output is higher than normal,


B. total peripheral resistance is normal,
C. blood volume is above normal,
D. urinary and blood catecholamines are normal, and
E. plasma renin, angiotensin II and aldosterone are normal.

Suggest and support a tentative diagnosis for these blood pressure readings. What
types of drugs would you prescribe for this patient?

2. Starling's Law of the heart ensures that the outputs of the two hearts (right and
left) are equal. Following a decrease in right heart output, e.g., after right heart
failure, how does Starling's Law ensure that the output of right and left hearts will
be equal?

3. Consider an individual engaging in light to moderate exercise. The following


items are related to blood flow in several vascular beds during the exercise period.

A. Describe how blood flow changes in the following circulations: cerebral,


coronary, muscle, and skin.

B. What are the mechanisms responsible for these changes in blood flow?

4. Lymph flow in an organ is observed to decrease. Give four possible reasons why
this might occur.

528
Cardiac Cycle - Dr. Baumgarten

The Cardiac Cycle


Clive M. Baumgarten, Ph.D.

OBJECTIVES:

1. Describe periods comprising cardiac cycle and events within each period
2. Describe the temporal relationships between pressure, blood flow, ventricular
volume, the venous pulse, heart sounds and the electrocardiogram
3. Describe the temporal differences between right and left heart events
4. Describe venous A-, C-, and V-waves and pulmonary capillary wedge pressure
5. Describe changes in the arterial pulse as the pressure wave moves from the aorta
to the femoral artery
6. Describe heart sounds and their basis

I. EVENTS OF THE CARDIAC CYCLE

A. The cardiac cycle represents one cycle of electrical and mechanical activity of the
heart. The cycle is divided into periods primarily based on the activity of the left
ventricle.
1. Atrial Systole − 110 msec
2. Isovolumetric Ventricular Contraction − 60 msec
3. Rapid Ventricular Ejection − 120 msec
4. Reduced Ventricular Ejection − 170 msec
5. Isovolumetric Ventricular Relaxation − 90 msec
6. Rapid Ventricular Filling − 120 msec
7. Reduced Ventricular Filling or Diastasis − 160 msec
Approximate durations are listed for a HR of 72 bpm. The major effect of HR
is to alter the duration of diastasis; increasing HR decreases diastasis. The
duration of systole is also inversely related to HR. However, changes in the
duration of systole are much less pronounced than changes in diastasis.

B. Temporal relationships between physiological variables including the ECG,


mechanical activity, pressure in the chambers of the heart, aorta and veins, and
cardiac sounds are considered for the cardiac cycle in the Wiggers diagram.

529
Cardiac Cycle - Dr. Baumgarten

530
Cardiac Cycle - Dr. Baumgarten

II. GENERAL RELATIONSHIPS BETWEEN ELECTRICAL AND


MECHANICAL ACTIVITY

A. Atrial electrical activity begins with the P-wave and ends during the QRS
complex. Because the SA node is on the right side, RA events preceded LA
events. Ventricular electrical activity begins with the Q-wave and ends at the end
of the T-wave.
B. Contraction lags behind the corresponding electrical events by 20 - 50 msec.
For the ventricle, mechanical relaxation is complete just after full repolarization.
For the atria, relaxation occurs as much as 40 msec after repolarization. The atrial
action potential is much shorter in duration than the ventricular action potential,
although the contractions are nearly equal in duration.
C. The exact relationships between electrical and mechanical activity depend on
heart rate. With increasing heart rate, APD shortens more than the duration of
contraction.

III. VENTRICULAR SYSTOLE

A. Isovolumetric Contraction
1. Isovolumetric contraction begins after the onset of the QRS-wave. Its
beginning is marked by the first increase of ventricular pressure following the
completion of atrial contraction. This is often approximated by the peak of
the R-wave.
2. At the end of diastasis, the AV valves are tensed and nearly closed. They are
fully closed immediately by the pressure increased caused by isovolumetric
contraction. The aortic and pulmonic values are closed throughout this
period.
3. Ventricular pressure rises rapidly, and ventricular volume remains constant.
No blood is ejected.
4. Closure of AV valves corresponds to the first heart sound, S1.
B. Rapid Ventricular Ejection
1. When ventricular pressure exceeds that in the aorta and in the pulmonary
artery, the semilunar valves open marking the beginning of rapid ejection.
2. There is a rapid blood flow from LV to aorta (and RV to pulmonary artery).
The peak of aortic flow precedes the peak of ventricular pressure. By the time
ventricular pressure reaches it peak, aortic flow has decreased to about 70% of
its maximum value.
3. Ventricular volume decreases sharply. Approximately 60% of the SV is
ejected.
4. Atrial pressure falls below central venous pressure, and atrial filling begins.
5. The onset of the T-wave marks the end of rapid ventricular ejection.

531
Cardiac Cycle - Dr. Baumgarten

6. Eddy currents keep the semilunar valve leaflets away from the arterial walls.
They prevent the leaflets from blocking the coronary ostia which are located
within the Sinuses of Valsalva behind the left and right aortic valve leaflets.

C. Reduced Ventricular Ejection


1. Ejection of blood is slower, and ventricular volume decreases at a slower rate.
The muscle fibers have come close to their minimum length. Repolarization
during the T-wave signals that contraction is ending.
2. Ventricular and aortic pressure begin to fall from their peak values. Aortic
pressure falls because runoff of blood to the peripheral vasculature is greater
than blood flow from the ventricle to the aorta.
3. Aortic pressure very slightly exceeds ventricular pressure during most of this
period. Ejection continues at a slow rate despite the reversed pressure
gradient because of the momentum of blood. The energy available for
ejection is the sum of the potential energy (pressure gradient) and kinetic
energy (momentum).
4. Atrial pressure remains below central venous pressure, and atrial filling
continues.

IV. VENTRICULAR DIASTOLE

A. Isovolumetric Relaxation

1. As contraction ends, left ventricular pressure falls below aortic pressure, and
the aortic valve closes. The pulmonic valve closes slightly after the aortic
valve. This delay is increased by inspiration and decreased by expiration.
Since ventricular pressure remains higher than atrial pressure, the AV-valves
remain closed.
2. Semilunar valve closure occurs because of both the pressure gradient and
eddy currents. At the beginning of this period, before closure of the values are
complete, there is there is a very slight retrograde flow. This appears as a
brief negative aortic flow in the Wiggers diagram.
3. During isovolumetric relaxation, ventricular pressure falls rapidly, and
ventricular volume is constant since all values are closed.
4. Closure of the semilunar valves corresponds to the second heart sound, S2.
5. Following closure of the aortic valve, aortic pressure increases a second time
in the absence of flow into the aorta. This creates the dicrotic notch or
incisura in the aortic pressure trace. The second pressure increase is caused,
in part, by elastic recoil of the aortic valve and aorta. The pressure wave
passing down the arterial tree is reflected at branch points. Reflections
propagate upstream and also cause aortic pressure after the dicrotic notch.
6. Isovolumetric relaxation occurs after completion of the T-wave.

532
Cardiac Cycle - Dr. Baumgarten

B. Rapid Ventricular Filling

1. Isovolumetric relaxation ends when ventricular pressure falls below atrial


pressure, the AV valves open, and rapid ventricular refilling begins. In the
Wiggers diagram, the left ventricular and atrial pressure are shown with mitral
value opening.
2. Relaxation of the ventricle assists refilling by sucking blood into the
ventricle.
3. Blood flow from the aorta to the peripheral arteries continues and aortic
pressure falls towards its diastolic level.
4. Rapid blood flow into the ventricle and/or rapid relaxation of the muscle sets
up vibrations giving the third heart sound, S3, which occurs near start of this
period.

C. Diastasis

1. Ventricular filling continues at a slower rate. The point of transition from


rapid filling to diastasis is somewhat arbitrary.
2. The duration of diastasis is inversely proportional to heart rate. Increasing
heart rate markedly decreases the time for ventricular refilling (both diastasis
and rapid refilling). At HR > 100 bpm, refilling is incomplete.

D. Atrial Systole

1. Atrial contraction follows the P-wave near the end of diastasis.


2. Atrial systole increases intra-atrial pressure by ~5 mm Hg owing to atrial
ejection of blood. This contributes up to 20% of ventricular filling at high HR
but only 5% at slow HR, and ventricular volume and pressure rise
corresponding amounts.
3. Atrial systole causes the A-wave in the atrial and central venous pressure
traces.
4. After atrial contraction is completed, blood flow back towards the atria from
the ventricles starts to close the AV-valves.
5. At slow HR, the effect of atrial systole on ventricular pressure (atrial kick) is
obvious in recordings of ventricular pressure. At high heart rates, the
contribution fuses with the rapid increase in ventricular pressure and volume
(i.e, atrial systole occurs during rapid ventricular filling; diastasis is virtually
eliminated). Absence of a clear step in ventricular pressure does not mean
that atrial contraction is unimportant at high heart rates. To the contrary, atrial
contraction is quite important and makes a significant contribution to
ventricular filling at high heart rates because the time for passive refilling is
reduced.
6. The QRS complex marks the end of atrial systole.

533
Cardiac Cycle - Dr. Baumgarten

V. COMPARISON OF LEFT AND RIGHT HEART

Critical differences in timing of right and left heart events can be summarized from the
following observation: Isovolumetric contraction and relaxation of the right heart
occur total within the corresponding periods for the left heart. This results from the
physiology of electrical conduction and differences in pressures. Differences include:
A. RA contraction occurs before LA contraction because the SAN is in the RA.
B. LV contraction starts before RV contraction, and the mitral valve closes before
the tricuspid valve. The RV lags because the right bundle branch of the His-
Purkinje system is longer and, hence, conduction time is greater.
C. LV ejection starts last and ends first. (1) Pulmonic valve opens before the aortic
valve because it takes longer to develop the higher pressure needed on the left
side. (2) LV ejection ends before RV ejection because high aortic pressure
induces valve closure first.
D. Mitral valve opens after tricuspid. More time is needed for the higher pressure on
the left side to fall.

534
Cardiac Cycle - Dr. Baumgarten

2-Cork Mnemonic − Gives left−right differences in timing of valve motion

VI. VENOUS PRESSURE

A. Venous Pulse − A Measure of Right Atrial Pressure

Cervical neck veins communicate with the right atrium without intervening
valves. Consequently, alterations in RAP are transmitted to the neck veins and
are easily distinguished on physical examination. The morphology of left and
right atrial and central venous pressure traces are quite similar. The major waves
are represented in recordings from each, and the same terminology is applied.
1. A-wave
a. Venous pressure slowly increases during diastasis as RA volume and
pressure slowly increase.
b. The maxima of the A-wave is caused by atrial systole.
2. C-wave
a. A second maxima closely follows the A-wave. Venous pressure rises
during isovolumetric contraction period reaching a maxima early in the
rapid ejection period.
b. The C-wave is caused by the bulging of the tricuspid valve into the right
atria due to right ventricular contraction. It was named erroneously
because it was initially thought that it was an 'artifact' of carotid artery
pulsation and /or of gross cardiac movement during systole.
3. V-wave
a. Venous pressure reaches a minima at the end of the rapid ejection period.
It rises to a peak at the end of isovolumetric relaxation, and then falls.
b. The V-wave reflects blood flow from the great veins into the right atria
increasing RAP (rising phase) and from RA to RV decreasing RA pressure
during rapid ventricular filling (falling phase).
On physical examination, A and C-waves merge. Thus, findings are often
expressed in terms of A and V-waves only.

535
Cardiac Cycle - Dr. Baumgarten

B. Pulmonary Capillary Wedge Pressure − A Measure of Left Atrial Pressure

Information on left atrial pressure is often obtained in clinical situations as


Pulmonary Capillary Wedge Pressure. A catheter is placed into a peripheral
vein and is advanced through the superior vena cava, right atrium, right ventricle,
and pulmonary artery until it wedges in the distal portion of the pulmonary
arterial bed. Since there are no valves between the left atrium and the pulmonary
capillaries, the contours of pulmonary wedge pressure directly reflect LAP,
although the numerical values are slightly different.
A, C, and V-waves correspond to the morphology of the venous pulse. On left
side, the mitral valve bulges into the left atria because of LV contraction.

VIII. ARTERIAL PRESSURE

The pressure pulse generated by LV contraction moves down the arterial tree with a
high velocity, ~5 m/sec. Its velocity increases in the periphery as vessel compliance
decreases. In contrast, the velocity of blood flow is only ~0.1m/sec. High frequency
components of the pressure pulse are damped by vessel walls. The pressure pulse is
partially reflected at branch points, and the reflected wave adds to the pressure.
Result: systolic and pulse pressures are higher in femoral a. and abdominal aorta
than in thoracic aorta! Mean pressure decreases distally from heart and controls flow.

536
Cardiac Cycle - Dr. Baumgarten

VIII. HEART SOUNDS

Auscultation of the heart means ascertaining its condition by listening to its sounds
with a stethoscope. The vibrations (sounds) are brief, low frequencies (30-250 Hz),
and low amplitude. Vibrations are caused by disturbances in flow; normally, flow
through open valves is silent. A phonocardiogram is a record of electronically
amplified sounds. It significantly extends the range of detectable frequencies.

A. First Heart Sound (S1) - 'lubb'

S1 occurs during isovolumetric contraction and results from the sudden tension
and recoil of the AV valves and adjacent structures. Although normally fused
(i.e., heard as one sound), the mitral valve closes before the tricuspid. The sounds
may be split (i.e., heard as two sounds) on inspiration.

B. Second Heart Sound (S2) - 'dup'

S2 occurs on closure of the semilunar valves at onset of isovolumetric relaxation.


This initiates vibrations in the column of blood and tensed vessel walls. S2 is
higher in pitch, shorter in duration, and lower in intensity than S1; it is described
as a snapping sound.
Inspiration causes splitting of S2. Inspiration:
1. Decreases intrathoracic pressure
2. Increases RV end-diastolic volume
3. Increase RV stroke volume and ejection time Thus, the pulmonic valve
closure is delayed relative to the aortic valve.

537
Cardiac Cycle - Dr. Baumgarten

C. Third Heart Sound (S3)

S3 is caused by blood vibrating the ventricular walls as it passively flows from


atria to ventricle during the rapid phase of ventricular refilling. It is a very soft,
low-pitch, muffled sound. S3 is physiologically normal in young individuals.
When it returns, it often reflects decreased compliance of the ventricular wall
(e.g., in congestive failure).

D. Fourth Heart Sound (S4)

S4 is caused by rapid filling of the ventricle during atrial systole. It occurs at the
peak of atrial contraction, immediately before S1. It is always inaudible in
normal adults.

IX. REFERENCES

A. Koeppen, B.M. and Stanton, B.A. Berne & Levy Physiology, 6th Ed., 2008. pp.
321-325.
B. Costanzo, L.S., Physiology, 3rd Ed., Saunders, 2006, Chapter 4, pp. 148-151.

8-Cardiac Cycle-2009.doc 8/28/2008

538
Body Temperature Regulation - Dr. Ford

Body Temperature Regulation


George D. Ford, Ph.D.

OBJECTIVES:

1. List the body temperatures, high and low, at which temperature regulation is lost.
2. Distinguish between Fever and Hyperthermia.
3. Distinguish between Heat Stroke and Heat Exhaustion.
4. List sources of heat input and mechanisms for heat loss.
5. Describe how heat input and heat output are regulated.
6. Describe the anatomical location and sensitivities of the temperature sensors.
7. Describe how skin and hypothalamic temperatures are combined to make
appropriate responses to changes in core and skin temperature.
8. Describe the Set-Point theory of Body Temperature Regulation.
9. Describe how infections produce fevers.

I. INTRODUCTION AND OVERVIEW

The Physician and the Thermometer.

The normal constancy of body temperature and its deviation from this value
produced by disease makes the measurement of body temperature one of the most
common, non-invasive clinical procedures. In this lecture, we will try to examine
the mechanisms which ensure precise body temperature regulation and then
examine the mechanisms by which disease increases body temperature. We will
begin by looking at the range of temperatures the body might obtain and the
definition and measurement of what we mean by body temperature. Next we will
examine briefly the mechanisms by which the body can gain or lose heat and how
these mechanisms may be regulated. Finally, we examine the Set-Point theory of
temperature regulation and how this accounts for the increase in body temperature
associated with many bacterial and viral infections.

TWO NEW TERMS:

Poikilotherm - Describes a creature that allows its body temperature to vary with
the environmental temperature.

Homeotherm - Describes a creature that tries to keep its body temperature


constant despite variations in the environmental temperature.

Although Man is considered a homeotherm, He is actually a hybrid of sorts. He


allows his surface (skin) temperature to vary while trying to keep his body core
temperature constant.

539
Body Temperature Regulation - Dr. Ford

II. OPERATIONAL RANGE OF BODY TEMPERATURE

Figure 1. Effect of core temperature on ability to respond to


abnormal temperature.

Failure of Temperature Regulation has severe consequences. Note that the


body only survives in a fairly narrow range of body temperatures as compared to
the temperature ranges found in our environment. Between 95°F and 105°F (34 to
40° C) temperature regulation is not in jeopardy. Above 106°F (40° C) and below
84°F (34° C) temperature regulation is lost. (See Fig. 1). Too hot is more acutely
dangerous than too cold, but don’t tell that to Arctic explorers.

III. BODY CORE TEMPERATURE

The temperature of mixed venous blood reflects the thermal status of all of the
body tissues and is therefore, the best representation of the body core temperature.
Since very little heat transfer takes place in the lungs and venous blood is well
mixed as it enters the pulmonary artery, pulmonary arterial blood temperature is
close to mixed venous blood temperature. Pulmonary arterial blood temperature is
estimated by taking esophageal temperature at the level of the heart. This
approach is often used during surgery.

IV. PRACTICAL MEASUREMENT OF BODY CORE TEMPERATURE

The next best estimate of core temperature is obtained by measuring rectal


temperature, which is usually a few tenths of a degree centigrade higher than
arterial blood temperature. Vaginal temperature is virtually identical to rectal
temperature. Tolerated in infants but often causes discomfort in adolescents and
adults.

540
Body Temperature Regulation - Dr. Ford

Oral temperature, which tends to be 0.5-1.5° C below the prevailing rectal


temperature, is most frequently used because this approach is well tolerated by
patients.

Modern technology has brought the technique of measuring temperature by infra-


red radiation detection to the textbook and bedside. Today there are such
instruments for reading body temperature from the radiation of heat from the
capillary blood flow in the ear. This technique has the advantage of not requiring
the conscious participation by the patient.

V. THE PRECISION AND POWER OF TEMPERATURE REGULATION

Body temperature is maintained within very narrow limits in the face of very wide
ranges in the rate of heat gain or loss. This power is shown in the figure below
which shows core (rectal) temperature of a naked individual exposed to the
indicated temperatures for 3 hours.

Figure 2. Core temperature of naked individual after exposure


to indicated temperature for 3 hours.

The thermal neutral zone is the ambient temperature range over which normal
body temperature is achieved without activation of metabolic and evaporative
processes. For a nude adult this zone is between 27° C and 33° C. For adults clad
in a more socially acceptable manner and slightly more active, such as listening to
this lecture, that range is lowered to 21 to 28° C.

VI. TEMPERATURE IS A MEASURE OF HEAT CONTENT

Heat content is determined by the kinetic energy of the molecules of a body. To


raise or lower temperature, energy must by added or reduced respectively. The
relation between heat and temperature is given by the equation:

541
Body Temperature Regulation - Dr. Ford

The heat content of a 70 Kg man with a body temperature of 98.6° F (37° C) is:

H = 70 Kg X 0.85 Kcal X (273° + 37°)


Kg X °T
H = 18,445 Kcal

The point is that to keep temperature constant a balance between mechanisms by


which the body gains heat and the body loses heat must be maintained.

VII. SOURCES OF HEAT INPUT (Hi)

A. Environment, e.g., Ambient temperature above body temperature.

Obviously this mechanism is greatly dependent on the external


environment.

B. Metabolism - The major and constant source of heat gain by the body.

To appreciate how much energy must be added to the body to change core
temperature consider the following problem: How long would it take for
resting metabolism, which consumes 250 ml O2/min, to raise core
temperature 1°C?

You will need to know that 5 Kcal of heat are produced per L 02,
consumed.

Using the relationship: H = M x SH x T given in section 6:

Increasing the temperature 1°C in a human weighing 70 Kg requires:

H = 70 x 0.85 x 1 = 59.5 Kcal.

Resting metabolism produces heat at the rate of:

0.25 L O2/min x 5 Kcal/L 02 = 1. 25 Kcal/min.

To produce 59.5 Kcal at 1.25 Kcal/min would require 47.6 min


(59.5/1.25).

Note: During exercise, 02 consumption can be as high as 2.5 L/min.


Therefore, 4.76 minutes of exercise at this rate would raise core
temperature 1°C assuming no compensation takes place.

542
Body Temperature Regulation - Dr. Ford

The point is that metabolism is a constant source of heat gain even at rest.
It can become critical during exercise particularly if heat loss mechanisms
are compromised.

VIII. MECHANISMS OF HEAT LOSS (Ho)

A. Radiation (Hr)

Physicists refer to this as black body radiation. Biophysically it is


expressed as:

Hr = Kr Ar (Ts - Te)

Kr = proportionality constant for radiation loss


Ar = skin area involved in radiant energy exchange
Ts = skin temperature
Te = environmental temperature

Simply put, the rate of heat loss by radiation is proportional to exposed


skin area and the temperature difference between skin and environment.
Note this could be a source of heat gain in the appropriate environment.

B. Convection/Conduction (Hc)

Conduction occurs by direct contact and exchange of thermal energy by


molecular collision. Convection implies movement. The usual sequence is
conduction between the body and the layer of air in immediate contact
with the skin. Then this layer moves either by thermal stirring or general
air movement (wind) causing convection. This is the usual route of heat
loss (~60%) in the thermal neutral zone. Again biophysically:

Hc = Kc Ac (Ts - Ta)

Ta = air temperature next to the skin


Ac = skin area involved in convection
Kc = proportionality constant for convection/conduction

C. Evaporation (He)

This is the process of sweating. It is potentially the greatest mechanism for


heat loss. It is routinely activated in exercise but usually makes a minimal
contribution in resting conditions. Of course there is always a little
evaporative heat loss due to evaporation from the oral cavity but this is
generally referred to as insensible loss as it plays no role in normal
temperature regulation. Biophysically the relationship is similar but the
parameters are slightly different:

543
Body Temperature Regulation - Dr. Ford

He = Ke. Aw (pH2Os- pH2Oa)

Ke = proportionality constant for evaporative loss


Aw = area of wet skin
pH2Os = vapor pressure of water at skin temperature
pH2Oa = vapor pressure of water in the ambient air

IX. ADJUSTMENTS IN HEAT GAIN

A. Thyroid hormone

The thyroid hormones T3 and T4 can increase metabolic rate and heat
production as much as10-25 per cent. In man, variations in thyroid
hormone levels and adjustments in basal metabolism (generally by
changing basal Na+-K+ ATPase levels) are generally chronic or adaptive
responses. Levels are low in heat acclimation and elevated in cold
adaptation. Interestingly the cardiac myosin isoform changes from fast to
slow in heat acclimation, probably in response to changes in thyroid
hormone levels.

B. Sympathetic nervous system

In man an important method for increasing tissue metabolic rate is


activation of sympathetic neurons innervating the beta receptors of many
tissues, especially the lipolysis of the brown fat of adipose tissue. This
response to the cold in man is well documented and can increase
metabolic rate by as much as 25 per cent.

C. Shivering

Shivering is the most potent mechanism for increasing heat production.


(Metabolic rate can increase up to 500 percent). Shivering is the rhythmic
contraction of skeletal muscle. It is produced by skeletal muscle
contractions which are interrupted by stretch induced inhibition of
contraction. The response is organized from the posterior hypothalamus
and involves alpha and gamma motor neurons. This is a semiconscious
mechanism. Higher levels of neural activity can suppress it.

D. Exercise

This is of course a voluntary process. Stomping your feet and clapping


your hands when you are cold are obvious examples.

544
Body Temperature Regulation - Dr. Ford

X. ADJUSTMENTS IN HEAT LOSS

A. Conduction/Convection

The rate of heat loss by radiation and conduction/convection is


proportional to Ts, skin temperature and exposed surface area. The
mechanisms regulating heat loss by these routes involve both conscious
and reflex levels mechanisms to regulate skin temperature. Consciously
skin surface area is regulated by adding or removing clothing as
appropriate. Skin temperature is regulated by regulating skin blood flow.
As with shivering this response is organized from the posterior
hypothalamus. Skin blood flow is regulated by changes in sympathetic
tone. Increases in sympathetic tone decrease blood flow, decreases in
sympathetic tone increase blood flow. In discussing blood flow a
distinction is made between apical skin (fingers, hands, toes, feet, ears,
nose, and lips) and nonapical skin (skin over the rest of the body).

In the cutaneous vasculature, branches from subcutaneous arteries


penetrate the dermis and form an arterial plexus in the deep dermis. The
vessels within this plexus run parallel to the surface and give rise to
arterioles that penetrate to the subpapillary region, where a subpapillary
plexus is formed. Here, capillaries connect to the subpapillary venous
plexus and single capillary loops ascend to each papilla. The descending
portion of the capillary loop ascend to each papilla. The descending
portion of capillary loop joins the subpapillary venous plexus, which
drains into the deeper cutaneous venous plexus. In apical regions of skin
muscular arteriovenous anastomoses form non-nutrient connections
between arterioles and venules. Arterioles, arteriovenous anastomoses, and
venues all are innervated by the sympathetic nervous system. Only alpha
receptors are seen in these vessels. The venous plexi contain variable
amounts of blood which can be significant under extreme conditions. The
arteries and veins run in parallel and close together which allows for some
heat exchange between warm arterial blood and cooled venous blood.
Blood flow is regulated by sympathetic tone. A decrease in tone increases
flow through arterioles and, more important for temperature regulation
increases arteriovenous shunting of blood to venous plexi. With these
venous plexi close to the surface of the skin there is a heating of skin and
an increase in heat loss by radiation, conduction/convection, and an
assistance in evaporative loss of heat. (Warm skin evaporates water more
rapidly than cold skin.) These anatomical arrangements are shown in Figs.
3 & 4.

545
Body Temperature Regulation - Dr. Ford

Figure 3. Functional arrangement of skin vasculature.

B. Heat loss by evaporation.

The most effective method for producing heat loss is to evaporate water
from the surface of the skin. The rate of evaporative heat loss depends
upon exposed surface area, skin temperature, skin wetness, and ambient
air relative humidity. Skin temperature is regulated as described above.
Skin wetness is dependent upon the activity of sweat glands (exocrine
glands) controlled by the sympathetic nervous system. This portion of the
sympathetic nervous system is unique because the transmitter is
acetylcholine. The receptor is muscarinic. Sweat glands form a primary
secretion from which salts are reabsorbed as the secretion travels along the
tubule leading to a pore as shown in Fig. 4. The evaporation of one liter of
water removes 580 Kcal from the body. Under optimal conditions the
body can evaporate about 1.5 liters of sweat per hour, which is about 12
times the basal rate of heat production.

546
Body Temperature Regulation - Dr. Ford

Figure 4. Mechanism for sweat formation.

C. Radiation

Heat loss (or gain) by radiation is most potently controlled consciously,


i.e., adding or removing clothing or other garments to control the exposed
surface area.

XI. THERMAL RECEPTORS

To maintain normal core temperature requires error sensors and appropriate


adjustments in the rate of heat production and heat loss.

Two major classes of sensors have been identified: skin temperature sensors, and
hypothalamic (core temperature) sensors.

547
Body Temperature Regulation - Dr. Ford

A. Skin Temperature Receptors

Skin receptors show 3 peaks of activity:

Figure 5. Relative firing rates of skin temperature sensors.

B. The anterior hypothalamus receptors

The hypothalamic receptors show 2 areas of activity. Notice how much


narrower the response range is for hypothalamic receptors. Some feel
there are also thermal receptors in the great veins and thoracic cavity.
They probably contribute but the hypothalamic ones seem to predominate.

Figure 6. Relative firing rates of hypothalamic temperature sensors.

548
Body Temperature Regulation - Dr. Ford

XII. THE NOTION OF SET POINT

The information received from skin and hypothalamic sensors are compared to a
set-point temperature in the hypothalamus.

Responses to information received from temperature sensors are organized within


the preoptic/anterior hypothalamus. The diagram below, Fig. 7, shows
temperature information coming into the hypothalamus from the skin and
hypothalamus and being compared to set point temperature. If that information
shows the sum of core and skin temperature to be below some preset, desired
level known as the set point temperature, heat loss mechanisms are inactivated
and heat production mechanisms activated. Heat production → shivering and
increases in metabolic rate. Heat loss → vasodilation and sweating. To decrease
heat loss there is active vasoconstriction. The circle shows that the temperature
and set point information are algebraically summed. If the sum is 0, there is no
response. If the sum is positive, i.e., if the sum of input, except set-point
temperature, is below set-point temperature; heat conservation and heat
production are initiated. If the sum is negative, heat production is inhibited and
heat loss mechanisms are activated.

Figure 7. Integration of temperature information.

Abbreviations: AH - anterior hypothalamus, PH - posterior hypothalamus, SW -


sweat, SH - shiver, VC - vasoconstriction, M - metabolism, R - reference point =
set point

XIII. SET POINT CAN BE CHANGED BY EITHER PERIPHERAL OR


CENTRAL INFORMATION

A. Peripherally

Decreases in skin temperature shift the set point temperature upward. This
resetting seems to anticipate the central response to the stimulus. The body
will react as if it is cold already, i.e., the normal body temperature is below
the new set point. Shivering and cutaneous vasoconstriction will result.

549
Body Temperature Regulation - Dr. Ford

You will see this result in lab. It is the basis of the cold pressor test for
normal sympathetic neural activity.

B. Centrally

The most important example of central resetting of set point is exercise.


The set point in exercise is centrally elevated to at least 39°, presumably to
accommodate the tremendous increase in heat production due to the
increased metabolic activity of the exercising muscles. Other central
resetting influences are hormonal such as the change in body temperature
during the female ovulation cycle and also the circadian clock whereby
body temperature is lowered at night.

XIV. SET POINT CAN ALSO BE CHANGED BY EXOGENOUS SIGNALS


(PYROGENS)

Fever is the situation where the set point is elevated such that body temperature is
maintained at a higher level than normal. Fever accompanies some trivial and
some very serious clinical conditions. Whatever the etiology, the pathophysiology
of fever appears to be the same whenever fever occurs. The sequence is outlined
below:

The role of the immune system in fever. The response of the body to an invasion
by any foreign body or organism is to activate the immune system. The stepwise
responses are:

A. Macrophages or B-lymphocytes phagocytize foreign organisms, virally


infected cells, or tissue debris. Activation of these cells includes activation
of the gene for interleukin-1 (IL-1). Also, IL-6 and IL-8 have recently
been shown to have similar properties.

B. IL-1 diffuses into the vascular space and is carried by blood to the anterior
hypothalamus. It is important to note that IL-1 is lipid soluble enough to
penetrate the blood-brain barrier.

C. At the anterior hypothalamus phospholipase A2 is activated by IL-1 and


arachidonic acid is released from plasma membranes.

D. Cellular cyclooxygenase initiates a series of reactions which produce


prostaglandins. One of these prostaglandins, prostaglandin E2, is believed
to be the agent which shifts the temperature set-point, in the anterior
hypothalamus, to a higher than normal value.

E. The response of the body to this increase in set-point is to initiate


responses which decrease heat loss (e.g., vasoconstriction) and increase
heat production (e.g., shivering). When body temperature reaches this new

550
Body Temperature Regulation - Dr. Ford

set-point temperature the body will maintain this temperature until the set
point is returned to normal.

XV. RESETTING THE SET POINT

One obvious method for returning the set-point to normal is to eliminate the
infection. A second method for returning the set-point temperature to normal is to
stop the production of prostaglandin E2. This is done clinically by inactivating
cyclooxygenase, which can be done either by prescribing antipyretics such as
acetylsalicylic acid (aspirin) or acetaminophen (e.g., Tylenol). Note: these drugs
do not produce hypothermia, they merely return the set-point to normal.

XVI. OTHER CONDITIONS OF LOSS OF PRECISE BODY TEMPERATURE


REGULATION

A. Fever vs. Hyperthermia

Many people use the term fever to describe any rise in body temperature.
There is a physiological difference between fever and other hyperthermias.
In all hyperthermias, body core temperature rises. Only in fever, however,
is there no tendency for compensatory mechanisms to restore body
temperature to a normal level. In fever body temperature is actively
defended at the febrile level; exposure of a febrile patient to a cold
environment will excite vasoconstriction and shivering even though body
temperature is elevated. Moreover, the febrile patient consciously prefers a
high body temperature and so feels cold when exposed to a cold
environment, even though the thermometer shows him clearly to be hot.
Physical cooling is an appropriate therapy in hyperthermias other than
fever, but during fever cooling will be resisted physiologically and will
distress the patient.

B. Heat Exhaustion

Excess sweating produces dehydration and vasodilation, which reduces


blood volume, decreases unstressed volume and decreases cardiac output.
The subject will be light-headed and feeling weak and may even
experience symptoms as severe as syncope. The skin will be pale and
clammy. There is usually no significant hyperthermia although body
temperature will generally be in the 38°to 40° range (remember they were
probably exercising too strenuously or in a hot environment). Placing the
subject in a cool environment and administering water or fluids orally is
usually sufficient therapy.

C. Heat Stroke

When large inputs of heat; e.g., during exercise in a hot environment,

551
Body Temperature Regulation - Dr. Ford

produce core temperatures in excess of 40°C there is a risk of heat stroke.


At core temperatures of above 42°C heat stroke is almost certain to take
place. These high core temperatures destroy the function of the brain,
liver, and other organs and death ensues rapidly. The severity of pathology
depends upon the temperature of the core and the duration of the high core
temperature. Treatment consists of rapid cooling. This needs to be done
under supervision to avoid producing hypothermia. If supervision is not
immediately available then application of water to naked skin and
increased evaporation of the water by movement of air can be quite
successful in reducing core temperature.

D. Hypothermia

Hypothermia is defined as a condition in which the core temperature is


less than 35°C. At core temperatures of about 27°C respiration stops and
death ensues if there is no external intervention. Hypothermia is more
commonplace because ambient temperature is often far below core
temperature. It is more frequency seen in the elderly patient.

XVII. RECOMMENDED READING

Berne, R. M., Levy, M. N., Koeppen, B. M., and Stanton, B. A. "Physiology",


Mosby, 5th Edition, 2004. Temperature regulation is not given a separate
treatment by this text. Instead there is a brief description of the action of the
hypothalamus in the regulation of body temperature on page 217. Also there is a
brief discussion of the role of skin blood flow in temperature regulation on pp.
421 - 423. There is also a brief discussion on the interplay of temperature
regulation and exercise on pages 435-436

Schafer, James A., in “Essential Medical Physiology”, Johnson, L. R., Editor,


Lippincott-Raven, 1998. Chapter 62, pp. 815 -824.

“Physiology and Pathophysiology of Temperature Regulation”, Clark M. Blatteis,


Ed. World Scientific Publishing, Singapore, 1998.

Costanzo, L. S., “Physiology”. 3rd Edition, 2006. pp 169-171.

552
Body Temperature Regulation - Dr. Ford

XVIII. SAMPLE PROBLEMS

Instructions: Identify the correct answers (more than one may be correct).

1. A patient is observed to have cold wet skin. Which actions of the


autonomic nervous system best account for this condition?

A. increased parasympathetic activation of vascular beta receptors and


increased activation of parasympathetic innervation of sweat
glands
B. decreased sympathetic activation of sweat glands and increased
sympathetic activation of vascular beta receptors
C. decreased parasympathetic activation of sweat glands and
increased parasympathetic activation of vascular alpha receptors
D. increased sympathetic activation of sweat glands and increased
sympathetic activation of vascular alpha receptors

2. Which set of changes, all taken individually, could increase (body) core
temperature?

A. ↑ muscle contraction, ↑pH2Os, ↓ pH2Oa


B. ↑ muscle contraction, ↓pH2Os, ↑ pH2Oa
C. ↑ muscle contraction, ↓pH2Os, ↓ pH2Oa
D. ↓ muscle contraction, ↓pH2Os, ↓ pH2Oa
E. ↓ muscle contraction, ↓pH2Os, ↑ pH2Oa

Where pH2Os = partial pressure of water at the skin and pH2Oa = partial
pressure of water in ambient air.

3. An increase in skin temperature increases heat loss (either directly or


indirectly) by?

A. radiation
B. convection
C. evaporation
D. pyrogens

4. A patient is observed to stop shivering. This could be due to which of the


following changes?

A. A decrease in air temperature


B. An increase in hypothalamic temperature
C. An increase in interleukin-1
D. Administration of a cyclooxygenase inhibitor

553
Body Temperature Regulation - Dr. Ford

ANSWERS

1. D (cold implies low skin blood flow, hence arteriolar vasoconstriction via
α1-receptor activation and wet implies sweating, hence sympathetic
cholinergic activation of the sweat glands)

2. B (↑ muscle contraction means ↑metabolism and hence heat, the other


factors deal with evaporative heat loss which is to be minimized by ↓ in
the gradient for heat loss, i.e., ↓pH2Os, ↑ pH20a)

3. A, B, C (hopefully obvious)

4. B, D (If a person is shivering, one should assume the body temperature is


less than the set point. If the shivering stops, one assumes the body
temperature has risen to the set point OR that the set point has fallen down
to the body temperature. Options A and C raise the set point while option
D lowers it albeit not below normal. Option B would activate the central
thermoreceptors so that the body would think it is warm, not cold, and
stop shivering.)

554
Lab Group Assignments

Lab Group 1
Monday, February 16, 2009
9 a.m. – 12 noon

Thursday, March 19, 2009


8 – 10 a.m.

Room 204, Egyptian Faculty: Dr. Gea-Ny Tseng

Abedi, Scott Sina


Aboulhosn, Lara
Adam, Jason David
Al-Hashimi, Saba Kamel
Alexander, Nebu
Ali Imtiaz, Mubdiul
Ally, Ryan Inshan
Anderson, Bradley Barrett
Anthony, Craig Lincoln
Armstrong, Amy Elizabeth

Room 206, Egyptian Faculty:Dr. Shyama Masilamani

Artrip, Sarah Beth


Avula, Danielle
Baig, Kimberlyn Maravet
Balasubramaniam, Saranya Chellam
Bauer, Clayton Thomas
Beckta, Jason Mark
Bennion, Nathan Ronald
Berhanemeskel, Mahlet Girma
Bharucha, Grishma

Room 207, Egyptian Faculty:Dr. Jack Grider

Bhowmik, Nirjhor Mondril


Binyamin, Tamar Rebecca Mya
Black, Scott Bayles
Broughton, Robert Oliver
Brown, Ashley Landon
Brueckner, Jessica Colleen
Bui, Linh Ngoc
Buquo, Lauren Elizabeth
Burk, Spencer Mark
Burrer, Sandra K

555
Lab Group Assignments

Room 208, Egyptian Faculty: Dr. Clive Baumgarten

Campbell, Heather Sterling


Carter-Loebig, Nicole Sherron
Causey, Stephen Merrill
Chande, Neha Dilip
Chang, Andrew Seung
Chang, Sherry
Chen, Hsuan-Chih
Chen, Renee Tran
Collins, Lindsay Ann

Room 209, Egyptian Faculty:Dr. Joseph Feher


Colton, Adrianne Michelle
Dante, Siddhartha Allen
Davidson, Donald Dale JR
Davis, Timothy Hamilton
Delacruz, Panfilo Castro
Dholaria, Kevin
DiPasquale, Kathleen Diana
Disharoon, Steven James
Dockery, Lauren Elizabeth
Drake, Nicholas Matthew

Room 210, Egyptian Faculty: Drs. Ivo and Luciana Torres

Egbert, Bryce Larre


Emejuru, Jason Uchechi
Emmanuel, Joann Rohini
Eskildsen, Scott Martin
Faramand, Rawan Ghassan
Filler, Robert
Forbes, Michael
Fox, Benjamin D
Garnett, Katherine Simms

Room 211, Egyptian Faculty: Dr. Scott Walsh

Gibson, Peter Desmond


Gillen, William Spencer
Givens, Nathan Campbell
Godshall, Elizabeth
Goeden, Marcie Jean
Gokli, Ami
Goldman, Ashton Howard
Goldstein, Shira Kate
Goodell, Cara Amy Raphael
Grant, Mai Sorrel

556
Lab Group Assignments

Lab Group 2

Thursday, February 12, 2009


9 a.m. – 12 noon

Friday, March 20, 2009


8 – 10 a.m.

Room 204, Egyptian Dr. Raphael Witorsch

Grewal, Sharleen Kaur


Haines, Amanda Kathryn
Hamadani, Lara
Hamdani, Tarik James
Haney, Joshua William
Hanger, Michael William
Hardenbrook, Spencer James
Harrison, Mary Virginia
Heitz, Christopher Todd

Room 206, Egyptian Dr. Roland Pittman

Hendi, Aditi S.
Herczyk, Matthew David
Hillenbrand, Karl David
Hoang, Valerie Magdelena
Hou, Angela Yingchun
Hoyt, Jennifer Renee
Hsu, David William
Humsi, Michael Kamil
Imbery, Terence Edward
Jahanshahi, Pooya

Room 207, Egyptian Dr. Martin Mangino

Janovski, Alexander John


Joyce, John Michael
Junkin, Andrew Ryan
Kanel, Jason Alan
Kelberg, Matthew Edward
Kim, Michael Youn-Il
King, Jamie Leigh
Kolber, Marcin Konrad
Koors, Paul Daniel

557
Lab Group Assignments

Room 208, Egyptian Dr. Wayne Barbee

Kye, Cecilia
Le, Anh Kim
Le, John Chuong Quang
Lee, Ran
Leonard, Rachel Ann
Lo, Patricia Wai Yin
Loken, Erik Kristen
Lung, Tina Kathy
Maldonado, Michael Damian
Mann, Nathaniel

Room 209, Egyptian Dr. Ramzi Ockaili

Manson, Mellisa Serwah


Martinovic, Maryann Elizabeth
Mateer, Erin Abbott Street
Mays, Mary Katherine
Mba, Mba Uzoma Udo
McCaughan, Michael Scott
Meliagros, Pete Dennis
Miller, Devin Tatcher
Modi, Parth Chandrakant

Room 210, Egyptian Dr. Richard Costanzo

Mohan, Shiva C.
Morehouse, Bethany Caroline
Mulye, Anita Diwakar
Muqri, Aceela
Nardone, Vincent John
Nelson, Mary Ann
Newton, Daniel Henry
Nguyen, Eric N
Nguyen, Monika Dao
Nottingham, Charles Upshur

Room 211, Egyptian Dr. Vijay Lyall

O'Connor, James Patrick


Oh, Kimberly Sun
Ohene-Baah, Nana Yaw
Painter, Matthew David
Parikh, Nishant Bharat
Patel, Jay Mukesh
Patel, Rushita Mrugendra
Patel, Vaibhav
Peterson, Sarah Rose

558
Lab Group Assignments

Lab Group 3

Friday, February 13, 2009


9 a.m. – 12 noon

Friday, March 20, 2009


1 – 3 p.m.

Room 204, Egyptian Dr. Linda Costanzo

Peterson, Timothy E
Poliquin, Rachel C
Poll, Milt Grover
Potts, Andrew J
Powell, Tanisha Michole
Powelson, Palen Ann
Raghavan, Rahul Veera
Rajendran, Bipin
Rajkumar, Jennifer Jo-Ann
Ramireddy, Archana

Room 206, Egyptian Dr. Fadi Salloum

Rankin, Trevan Monteil


Repic, Adrian
Rich, Melissa Anne
Richey, Justin Dan
Riggs, Emily Kim
Robertson, Ryan Nexsen
Rose, Hannah Iris
Rowlett, Robert Mitchell
Ruhi, Emre
Saffouri, George Bassam

Room 207, Egyptian Dr. Mohammed Kalimi

Sasinowski, Maciek
Scott, Chantal Devaru
Sethi, Ashish
Shafer, Sarah San Young
Sherwood, Alex Berry
Shoemaker, Rebecca Ryan
Shou, James Young
Siegel, Julia Anne
Sienkiewicz, Lisa

559
Lab Group Assignments

Room 208, Egyptian Dr. Margaret Biber

Singh, Malkit Kaur


Sirkin, Maxwell Richard
Smith, Cameron Reid
Smith, Christina Marie
Spetzler, Karli Ann
Spivey, Steven James
Sponholz, Luke Richard
Spradlin, Elizabeth Ann
Stempel, Hilary Elyse
Stranix, John Timothy

Room 209, Egyptian Dr. Alexander Golub

Stuart, Adam Michael


Swift, Jesse Scott
Talreja, Neha
Taormina, Yula Alexandria
Tarasidis, George Stratos
Tiangco, Noreen Enriquez
Titchner, Timothy Joseph
Titerence, Rachel Helen
Toensing, Christopher Cody

Room 210, Egyptian Dr. Helena Carvalho

Tozer, Jordan Travis


Trang, Vinh Quang
Trujillo, John Francisco
Turner, Patrick Lawton
Washington, Monica Michaelle
Watson, Sara Elizabeth
Watterson, Christopher Travis
Way, Serena
Wilkinson, Kathryn Elise

Room 211, Egyptian Dr. Murthy Karnam

Wilson, Robert Travis


Wilson, Sean Patrick
Wise, Elizabeth Anne
Wu, Chun-Yu
Yang, Daniel Tien
Yester, Jessie Wettig
Zedler, Kathryn Cecile
Zimmerman, William Fairchild

560
Cardiovascular Physiology Lab - Dr. Baumgarten

Cardiovascular Physiology Laboratory


Clive M. Baumgarten, Ph.D.

To pump blood, the heart has a rhythmical sequence of both electrical and mechanical
events, the cardiac cycle. The electrical activity, recorded as an electrocardiogram
(ECG), initiates the mechanical activity of the heart (contraction and relaxation of atria
and ventricles). Ejection of blood from the left ventricle into the aorta increases arterial
pressure from diastolic to systolic pressure and produces a pressure wave that passes
down the arterial tree.

This lab comprises four exercises that are designed to:


1. Measure the propagation velocity of the pressure wave
2. Examine the effect of postural changes on heart rate
3. Visualize the Korotkoff sounds used to measure systolic and diastolic blood
pressure
4. Examine the effect of exercise and recovery from exercise on heart rate, pulse
pressure, and components of the ECG.
To accomplish this, you will:
1. Record the Lead II ECG
2. Estimate pressure pulse by plethysmography
3. Record systolic and diastolic blood pressure and Korotkoff sounds with a
sphygmomanometer and stethoscope microphone.

A. EXPERIMENT 1: MEASURE THE PROPAGATION VELOCITY OF THE


PRESSURE WAVE

1. Overview
The pressure pulse is initiated by ejection of blood from the LV into the aorta. The
pulse propagates down the arterial tree as a sound wave moving through a liquid.
The velocity of propagation of the pressure pulse is much faster than the velocity of
the flow of blood. The arterial walls are elastic and are distended by the passing of
the pressure pulse. The compliance of the vessel walls acts to slow the velocity of
propagation of the pressure pulse as compared to that in a rigid tube. Nevertheless,
the elastic recoil of the vessel serves to maintain diastolic pressure and flow.

When does ejection of blood begin? The events in the ECG reflect the timing of
electrical activity of the heart rather than the mechanical events. The QRS complex
corresponds to depolarization of the ventricle. The ejection of blood occurs about 50
ms after the end of the QRS complex because (1) contraction follows electrical
activity and (2) LV pressure must increase from end-diastolic pressure to aortic
pressure (isovolumetric contraction) before the aortic valve opens and ejection
occurs.

561
Cardiovascular Physiology Lab - Dr. Baumgarten

B. Set up of computer and


physiological sensors
1. Make sure the BIOPAC
MP30 sensor interface is
turned OFF (switch on the
back left side)
2. Turn the Computer ON.
3. Plug in the sensors as
follows:
CH 1: Electrode lead
(SS2L)
CH 2: Pulse transducer
(SS2LA or SS4L)
4. Turn the BIOPAC ON.

C. Attach electrodes to record Lead II ECG


1. Peel off an electrode holding the tab.
Try not to touch the adhesive. The
sponge at the center of the electrode
contains a conductive gel.
2. Place electrodes on right arm, left leg,
and right leg as shown in the diagram.
3. Connect the cables. The pinch connect-
ors should be oriented with the metal
contacts facing the electrode, and the
cables should be connected according to
the following color code:
RED Left Leg (+ for Lead II)
WHITE Right Arm (– for Lead II)
BLACK Right Leg (ground)
Lead II (LA-RA) is recorded; Right leg is ground.

D. Attach the plethysmography pulse


transducer
1. Wrap the pulse transducer around the tip
of your index finger with the window
facing the finger pad and secure the
transducer with the velcro strap, as shown
in the diagram. Contact between the
transducer and finger should be firm
without compressing the finger.

562
Cardiovascular Physiology Lab - Dr. Baumgarten

E. Start data acquisition program


1. Double click on BCL2 Lessons.
2. Select Lesson 7 (L7-ECG&P-1), and click OK.
3. Type in a filename (e.g., student’s name), and click OK.

F. Calibration of System
1. The subject should relax so that signals from skeletal muscle electrical activity
do not corrupt the ECG.
2. Click on Calibrate in the upper left corner of the Setup window. The calibra-
tion recording will stop automatically after 8 s.
3. Check the calibration data (see example in figure). There should be an ECG
with reduced amplitude and a relatively flat baseline in the upper band (ECG),
and a recording that looks like the arterial pressure waveform in the lower
band (Pulse).

4. The scale for each trace is automatically adjusted by the computer so that
recordings will occupy nearly the full height of the trace. If the record is
different, redo the calibration. If it is similar, proceed to Data Recording.

G. Data Recording
WARNING: Experimental directions on the screen should be ignored. Follow
directions in the handout.
1. Click on Record (upper left) and record for about 15 s. Then click on Suspend
and review the data.
2. If the recording doesn’t look right (see figure), click on Resume and obtain a
second tracing. (Warning: Clicking on Redo will erase the data).
3. After a satisfactory record is obtained, remove the electrode cable pinch
connect- ors, peel off the electrode patches, and throw them away. Wash the
electrode gel residue from the skin. The electrodes may leave a slight ring on
the skin for a few hours.

563
Cardiovascular Physiology Lab - Dr. Baumgarten

H. Data Analysis
1. Measure the time for pressure pulse to move from the heart to the finger tip.
2. Measure the distance between the heart and finger tip.
3. Calculate the propagation velocity of pressure pulse by dividing the distance
by the time.
4. Report the result in m/s.
I. Making the Measurements
1. Using the tape measure, measure the distance from the spine at heart level to
the shoulder and out to the finger tip. The distance should be about 1 meter.
2. The pressure pulse is initiated by ejection of blood from the LV into the aorta.
Ejection of blood occurs about 50 ms after the end of the QRS complex
because (1) contraction follows electrical activity and (2) LV pressure must
increase from end-diastolic pressure to aortic pressure (isovolumetric
contraction) before the aortic valve opens and ejection occurs. Therefore,
measure the time from the end of the QRS to the beginning of the pressure
pulse at the finger tip and subtract 50 ms to account for the delay between
the QRS and ejection of blood.
3. Select a section of the record to measure using the horizontal scroll bar.
4. Expand the time scale by clicking on Horizontal (Time) Scroll Bar or by
using the Zoom Tool.
5. Click on I-beam cursor (lower right). Using the cursor, highlight a section of
record from the end of the QRS to the beginning of the pressure pulse.

564
Cardiovascular Physiology Lab - Dr. Baumgarten

6. Measurement Boxes are located near the top of the screen. Each
“measurement” has three sections: a channel number box, a measurement
pull-down menu box that will display when you click on it, and a results box,
that reports the designated measurement.
7. Select ΔT and channel 1 for the first box. ΔT is the difference in time (in s)
between the end and beginning of the selected area.
8. Repeat measurement on 3 successive beats. Average and subtract 50 ms.
9. The Zoom Previous and Autoscale Horizontal tools can be used to restore the
time scale to its original value. The horizontal scroll bar can be used to select
a different section of the record.
10. The propagation velocity of the pressure pulse was: ______________ m/s

II. EXPERIMENT 2: CHANGES IN HEART RATE ON ALTERING POSTURE.

A. Overview
We will observe the effect of
postural changes on resting heart
rate. In the supine position, arterial
pressure is nearly independent of
location. Immediately on moving
to the standing position, blood
pressure in the arteries above the
level of the heart decreases and
blood pressure below the level of
the heart increases (see figure).
The reason for this is the
hydrostatic pressure due to the
force of gravity on the column of
fluid (blood) depends on body
location when standing. In
contrast, there is no difference in
hydrostatic pressure in the supine
position. Note that the density of
Hg is 13.6 g/cm3.
Reflexive control of HR. The
large change in arterial pressure
upon standing is immediately
sensed by the carotid baroreceptors initiating the baroreceptor reflex. Within seconds,
there are autonomic actions on both the vasculature and heart. Vasoconstriction occurs
on both the arterial and venous sides of the circulatory network due to increased
sympathetic stimulation to the walls of the vessels. Heart rate is increased due to
withdrawal of parasympathetic and enhanced sympathetic traffic to the SAN.
Sympathetic stimulation of ventricular muscle and the increased heart rate lead to
enhanced contractility. Both the enhanced contractility and increased right atrial pressure
(due to vasoconstriction) cause augmentation of stroke volume. In turn, the elevated
stroke volume results in increased pulse pressure.

565
Cardiovascular Physiology Lab - Dr. Baumgarten

B. Set up computer and physiological sensors: No changes from Experiment 1.


C. Experimental Protocol
The subject lies on the table for 3-5 min and then rapidly moves from the supine to
standing position with a minimum of exertion. With assistance of a second student,
the subject should rotate their body horizontally, so that their legs are off the table,
and then rapidly stand with assistance. A second assistant should make sure the wires
do not become entangled during the change of posture. Heart rate will be recorded:
(1) at steady state while supine (e.g., after 3-5 min), (2) immediately upon standing,
and (3) at 30 s intervals for the next 3 min. (Practice the supine to standing maneuver
with leads attached to insure a smooth transition without excessive exertion.)
D. Data Acquisition
1. After 3-5 min in the supine position, begin recording by clicking on Record.
2. Obtain at least 30 s of baseline data.
3. Continue the recording without interruption during the posture change and
for 3-5 min in the standing position.
4. Then click on Suspend to stop the recording and review the data.
E. Data Analysis
Measure heart rate in bpm:
1. In the steady state (e.g., after 3-5 min) in the supine position
2. Immediately upon standing
3. At 30 s intervals for the next 3 min
F. Making the Measurement
1. Select the section of the record corresponding to the last 10 s in the supine
position using the horizontal scroll bar.
2. Expand the time scale by clicking on the Horizontal (Time) Scroll Bar or by
using the Zoom Tool (lower right).
3. Click on I-beam cursor.

4. Using I-beam cursor, highlight a section of record from one R wave to next.
5. Select BPM and channel 1 for 2nd measurement box. BPM first calculates
the difference in time between the end and beginning of the area selected by
the I-Beam tool (same as ΔT) and converts to bpm.
6. Repeat the measurement for 3 consecutive beats and average results.
7. Repeat the determination of heart rate (1) immediately upon standing and
(2) at 30 s intervals for 3 min.
8. Plot your results as heart rate as a function of time.

566
Cardiovascular Physiology Lab - Dr. Baumgarten

III. EXPERIMENT 3: MEASUREMENT OF BP AND VISUALIZATION OF


KOROTKOFF SOUNDS

A. Overview
You will record systolic and diastolic
blood pressure and visualize the
Korotkoff sounds. The following
figure represents a recording of
systemic arterial blood pressure
measured directly by inserting a
catheter into an artery and attaching
the catheter to a pressure measuring
and recording device.
Four pressures can be measured:
Systolic Pressure: The maximum arterial pressure (during systole).
Diastolic Pressure: The minimum arterial pressure (during diastole).
Mean Arterial Pressure: Estimated as Diastolic Pressure + 1/3 Pulse Pressure
Pulse Pressure: Systolic Pressure – Diastolic Pressure
By convention, systemic arterial BP is expressed as a ratio: systolic pressure/diastolic
pressure. For example, if systolic and diastolic pressures are 135 and 80 mm Hg, BP
would be expressed as 135/80. For these values, pulse pressure would be 55 mm Hg and
mean arterial pressure would be 98 mm Hg. Pulse pressure is directly related to stroke
volume of the heart. For example, with exercise the increase in stroke volume results in a
large increase in systolic pressure, while diastolic pressure either increases slightly,
remains constant, or falls slightly. The large increase in systolic pressure with only small
changes in diastolic pressure results in increased pulse pressure during exercise.
Systemic arterial blood pressure is commonly measured with indirect methods because
direct methods are invasive and neither practical nor convenient for routine use. The
most common method, auscultatation (listening to sounds made by internal organs), uses
a stethoscope and a sphygmomanometer. The
sounds detected when measuring blood pressure
are referred to as Korotkoff Sounds.
B. Method for Measuring Blood Pressure
Arterial pressure is determined by placing an
inflatable rubber cuff attached to a pressure
gauge around the arm, inflating it to collapse the
underlying artery, and listening with a
stethoscope positioned over the vessel and
below the cuff. Sound is created by the
turbulent flow of blood through the partially
compressed vessel. When cuff pressure exceeds
systolic arterial pressure, the artery is collapsed,
blood flow ceases, and no sound is produced.

567
Cardiovascular Physiology Lab - Dr. Baumgarten

Systolic Pressure: As cuff


pressure is slowly reduced,
blood flow through the
artery begins when cuff
pressure falls just below
systolic arterial pressure.
At this point, a sharp
tapping sound (first sound
of Korotkoff) is heard.
Cuff pressure when this
sound is first heard is taken
as an approximation of
systolic pressure.
Diastolic Pressure: As cuff
pressure is further reduced,
sound intensity increases,
then suddenly becomes
muffled (second sound of
Korotkoff) at the level of
diastolic pressure, and
finally disappear. Sounds
disappear when the vessel
is no longer compressed by
the cuff and normal non-
turbulent blood flow
resumes. Because it is easier to determine when the sound disappears than when it
becomes muffled, and because only a few mm Hg pressure differential exist between the
two points, the disappearance of sound is commonly used as an indicator of diastolic
pressure.
The diagram above shows the temporal relationship between the ECG, Korotkoff sounds,
cuff pressure, the blood pressure pulse waveform (at the arm), and the condition of the
brachial artery under the cuff. The pulse waveform represents the brachial pressure in the
artery above the cuff. The shaded area represents the blood flow that can pass below the
cuff as soon as the arterial pressure exceeds the cuff pressure.
Timing of Korotkoff sounds relative to the ECG is important. The first sound appears
during the T-wave. This sound occurs at the time of peak pressure (systole), which, if
measured at the heart, would occur earlier during the S-T segment. The delay in
detecting the sound at the arm is due to the time it takes the pressure wave (sound) to
reach the arm. The time between the R wave and the first sound should be consistent.
Using this fact, you can distinguish actual Korotkoff sounds from extraneous noise.
NOTE: If your blood pressure as determined from this exercise is “high,” you should not
be too concerned. A mistake may have been made in the measurement, or there may be
other factors that resulted in a temporarily high reading. If you are concerned, please
consult your physician, but do not try to diagnose or treat yourself based on these
laboratory blood pressure readings.

568
Cardiovascular Physiology Lab - Dr. Baumgarten

C. Set up of Computer and Physiological Sensors


Note: It is necessary to restart both the program and the BIOPAC sensor interface.
1. Make sure BIOPAC interface is turned OFF (switch on the back left side)
2. Exit the program. Click on X in upper right.
3. Plug the sensors as follows:
4. CH1: BP Cuff (SS19L)
CH3: Stethoscope (SS30L)
CH4: Electrode lead set (SS2L)
5. Turn BIOPAC back ON
6. Start the BSL2 Lessions and choose Lesson 16 (L16-BP-1).
7. Type the filename, typically a different name than before, and click OK.
D. Attaching and Preparing the Sensors
1. Place three ECG electrodes on the subject as before.
2. Release all pressure from the cuff by opening the cuff valve and rolling the
cuff in on itself, then press to flatten and close the valve tight.
E. Calibration
1. With cuff not on the subject but closed in loop with velcro, click on Calibrate
(upper left corner) and inflate cuff to 100 mmHg according to the dial gauge.
You may have to pump 10-12 times to have enough pressure in the cuff for
any gauge reading. After having reached 100 mmHg, click on OK.
2. Deflate the cuff pressure to 40 mmHg using the pressure release valve. Then
click OK. When you click OK the calibration of the microphone will begin.
3. After the calibration recording begins, lightly tap the stethoscope diaphragm
twice. The recording stops automatically after 8 s. If the record is similar to
the following, proceed to Data Recording. If different, Redo the calibration.
4. The pressure should be constant at 40 mmHg during the calibration period. If
it declines or has large spikes, the valve is not fully closed or the hose
connections are not tight. If the ECG is excessively noisy or drifts even
though the subject was relaxed, one or more of the electrodes are probably not
making good electrical contact with the skin.

569
Cardiovascular Physiology Lab - Dr. Baumgarten

F. Using the Sphygmomanometer


1. To obtain accurate measurements, it is important to release the cuff
pressure at a rate of 2-3 mmHg per second. Practice before putting the
sphygmomanometer on the subject.
2. To practice, open the valve and roll the cuff in on itself, flatten the cuff to get
out the air, and close the valve. Pump the bulb until the pressure dial reads
160 mmHg. Slowly turn the valve counter-clockwise to begin releasing the
cuff pressure. Have someone time how long it takes pressure to decline from
160 to 100 mmHg. Open the valve slowly so that you don’t have a large
pressure drop, and try to maintain an even release. To keep the release rate
constant, you may have to open the valve more as the cuff pressure gradually
diminishes. It should take 20-30 s for pressure to drop 60 mmHg.
G. Setting Up Sphygmomanometer and Stethoscope microphone

1. Place the cuff on subject’s LEFT arm so


that the “artery” label is over the brachial
artery (with arrow on label pointing down).
2. Position the cuff with the lower edge 1.5 to
2 inches above the antecubital fossa and
high enough to avoid covering any part of
the stethoscope diaphragm.
3. Wrap the cuff evenly and snugly around the
subject’s arm. The velcro wrap should hold
the cuff in place, but you may wish to inflate
the cuff slightly (10-20 mmHg) so that it will stay in place.
4. Position the sphygmomanometer pressure dial so you can read the face of the
dial straight on. A strap on the cuff above the “artery” label that allows the
dial indicator to be clipped on.
5. Position the subject’s arm at heart level. Palpate the brachial artery between
the antecubital fossa and the lower edge of the cuff to find the greatest pulse
and align the “artery” label of the cuff with the pulse point.
6. Place the stethoscope over the artery and apply firm but not excessive contact
pressure.

570
Cardiovascular Physiology Lab - Dr. Baumgarten

H. Experimental Protocol
1. One student inflates and deflates the cuff, while another operates the
computer. By auscultation, mark the time at which Korotkoff sounds begin
by pressing the F9 key (An arrowhead appears above the record).
2. Start the recording by clicking on Record, then inflate the cuff to 160 mmHg,
and click OK.
3. After clicking OK, release cuff pressure at a 2-3 mmHg/second and call out
when the Korotkoff sounds first appear (systolic). Insert a marker by
pressing F9.
4. Continue to release cuff pressure and call out when sounds completely
disappear (diastolic). Insert a marker by pressing F9, and click on Suspend.
5. After the sounds disappear, cuff pressure should be deflated as quickly as
possible to reduce venous congestion. Never leave an inflated pressure cuff
on the subject’s arm.
6. Record the BP as measured by auscultation and reading pressure dial.
7. Review the data. Record should look like the figure.
• Pressure should
decrease over time.
• Korotkoff sounds
should be observed.
• ECG trace should
not have excessive
noise.
8. If something went
wrong, click Redo
to repeat the recording.
I. Making the Measurement
1. Select the section of the record beginning at the first Korotkoff sound or
marker for systolic pressure using the horizontal scroll bar.
2. Expand the time scale by clicking on the Horizontal (Time) Scroll Bar or by
using the Zoom Tool (lower right).
3. Click on I-beam cursor (lower right).
4. Sequentially position the I-beam cursor at each time point of interest (first
and second Korotkoff sounds and markers for systolic and diastolic pressure
by auscultation).
5. Select Value and
channel 1 for the
third measurement
box. Value
displays the
calibrated BP for
that time point.
6. Repeat for the
additional time
points of interest.

571
Cardiovascular Physiology Lab - Dr. Baumgarten

J. Data Analysis
1. What was BP based on auscultation (reading dial)? ____/____
2. What was BP based on auscultation (markers on chart)? ____/____
3. What was BP based on visualization of Korotkoff sounds? ____/____

IV. EXPERIMENT 4: THE EFFECT OF EXERCISE

A. Overview
The purpose of this experiment is to examine the effect of exercise and recovery from
exercise on heart rate, the pressure pulse, and components of the ECG. During exercise,
metabolic demands increase, sympathetic tone is augmented, and parasympathetic tone is
withdrawn. As a result, heart rate and cardiac contractility are increased, arterioles
serving muscle beds vasodilate, whereas vascular beds serving certain other “non-
essential”organs (e.g., GI system) constrict, and venoconstriction elevates right atrial
pressure and enhances cardiac filling. Overall, increases in cardiac output, stroke
volume, pulse pressure, systolic pressure, and mean arterial pressure are observed.
Increases in heart rate are accompanied by
changes in the ECG. First, the times
between successive P waves, representing
depolarization of the atria from the SAN,
and between successive QRS complexes,
representing depolarization of the
ventricle, are decreased. Cycle length, the
time between heart beats, is usually
measured as the R-R interval, and the
inverse of cycle length is heart rate. The
autonomic effects responsible for
increased heart rate also modulate the
speed at which action potentials conduct
through the AVN and decrease the
effective refractory period of the AVN, so it is ready to conduct a second action potential
more sooner. Conduction time through the atria and AVN is measured as the P-R
interval, about half of which is attributed to conduction through the AVN. An
abbreviation of the P-R interval accompanies increases in heart rate during exercise.
Heart rate also alters action potential duration. Action potential duration is reflected in the
Q-T interval, which is inversely proportional to heart rate. In contrast, the duration of the
QRS complex, which reflects the spread of depolarization throughout the ventricle,
normally is unaffected by heart rate.
SUBJECT EXCLUSIONS
This laboratory procedure asks the subject to perform a standard brief amount of
strenuous exercise. You should not be a subject if:
1. You are not able to climb 3 flights of stairs without a marked shortness of breath or
chest discomfort;

572
Cardiovascular Physiology Lab - Dr. Baumgarten

2. You have a medical history of angina, myocardial infarction, congestive heart failure,
cardiac valve disease, arctic or mitral stenosis, mitral valve prolapse, endocarditis,
pulmonary embolus, deep venous thrombosis, sick sinus syndrome, atrial
fibrillation/flutter/tachycardia, conduction system disease, an abnormal ECG,
supraventricular or ventricular tachycardia, a family history of sudden cardiac death,
implanted pacemaker or defibrillator, heart valve replacement, coronary artery bypass
or angioplasty, congenital heart disease, are on any cardiac medication, or have a
history of asthma, or hypertrophic cardiomyopathy; or
3. If any of the following occur during the pre-exercise period: resting heart rate <45
bpm or >90 bpm; premature ventricular contractions; PR interval > 200 ms or <100
ms; QRS interval > 110 ms; QT interval >450 ms; blood pressure <95/50 or > 140/85.
SUBJECTS FOR THE EXERCISE PORTION OF THE LABORATORY MUST
READ AND UNDERSTAND THE ABOVE EXCLUSIONS.
B. Set up of Computer and Physiological Sensors. No change from Experiment 3.
To be able to record blood pressure, the pressure pulse, and the ECG immediately
after exercise, the sensors should be left in place during exercise. An assistant
should hold the cables during the exercise procedure to avoid entanglement. The
sphygmomanometer cuff and stethoscope may need to be adjusted after exercise.
C. Experimental Protocol
1. Target Heart Rate
There are several guidelines for determining a subject’s age-predicted
maximum heart rate. A common method is:
Target HR = (220 – AGE) × (% of maximum selected)
In this formula, the maximum heart rate is (220 – AGE). Typically, the
aim is to increase HR to 75-85% of the calculated maximum.
2. An alternative, the Karvonen formula, is reported to better correlate with
oxygen consumption. The Karvonen formula should be used for the
laboratory exercise. The calculation is:
Heart Rate Maximum = 220 – AGE
Heart Rate Reserve = Heart Rate Maximum – Resting Heart Rate
Target Heart Rate =
Heart Rate Reserve × (% of maximum selected) + Resting Heart Rate
Use 75% of the Heart Rate Maximum for this exercise
Calculation for a 24 year-old subject with a resting heart rate of 72.
Heart Rate Maximum = 220 – AGE = 196
Heart Rate Reserve = Heart Rate Maximum – Resting Heart Rate
= 196 – 72 = 124
Target Heart Rate =
Heart Rate Reserve × (% of maximum selected) + Resting Heart Rate
= (124 × 75%) + 68 = 93 + 68 = 161

573
Cardiovascular Physiology Lab - Dr. Baumgarten

3. Exercise Regimen
We will use a variation of the YMCA Three-Minute Step Test. This entails
rapidly stepping on and off of a bench.
Bench Height = 8 - 12 inches (2 pink blocks on each side)
Step Rate = 24 steps/min Duration = 3 - 5 minutes
Medical Warning – The subject should stop exercising immediately and lie down if
any of the following happen during or after exercise:
dizziness widening of PR interval to > 200 ms
shortness of breath widening of QRS to > 120 ms
chest discomfort ventricular activity (PVCs or worse)
abdominal discomfort supraventricular tachycardia
HR >85% of predicted maximum atrial fibrillation/flutter
sudden drop in heart rate failure to record a good pressure pulse
D. Recording the Data
1. Data will be recorded in 2 parts. First, a baseline resting recording will be
made. Then, after the exercise is completed, the recording will be restarted
and continued until heart rate returns to normal (about 3 min).
2. To obtain the baseline resting recording, click on Resume. Determine the
resting blood pressure as before. Click on Suspend after the blood pressure
has been determined.
3. Exercise for 3 -5 min. Do not exceed the Target Heart Rate.
4. Immediately after stopping exercise,
a. Click on Resume to begin to record the ECG, sounds, and pressure pulse.
b. Immediately determine blood pressure
c. Repeat the blood pressure determination at 1 min intervals until heart rate
has returned to its pre-exercise resting level.
5. Click on Suspend to end the recording.
6. Remove the electrode cable pinch connectors, peel off the electrodes and
throw them away (BIOPAC electrodes are not reusable). Wash the electrode
gel residue from the skin. The electrodes may leave a slight ring on the skin
for a few hours.
E. Data Analysis
1. Record HR, systolic and diastolic BP, mean BP, and the amplitude of the
pulse pressure on the chart and graph heart rate.
2. Use the I-beam cursor and the ΔT measurement box to determine P-R
interval, QRS interval, and Q-T interval at each time point. See diagram of
ECG for the definitions of these intervals.
3. To approximate the time for ventricular filling, measure from the end of the
T wave (marks beginning of isovolumetric relaxation) to end of next QRS
(marks beginning of isovolumetric contraction) and subtract 90 ms (duration
of isovolumetric relaxation at HR of 72 bpm). (This approximation ignores
small changes in the duration of isovolumetric relaxation during exercise that
result from altered aortic, LV, and LA pressures.)

574
Cardiovascular Physiology Lab - Dr. Baumgarten

4. Plot HR as a function of time.

10-cardiovascular physiology laboratory_2009.doc 8/6/2008

575
Cardio Practice Exam & Review - Dr. Baumgarten

Cardiovascular Practice Exam and Review Questions


Clive M. Baumgarten, Ph.D.

CARDIOVASCULAR REVIEW QUESTIONS

1. Identify the steps in excitation-contraction coupling that modulate cardiac


contractility.

2. Use length-tension curves to estimate the amount of shortening during an isotonic or


isovolumetric contraction. What factors modulate the amount of shortening? What
factors represent afterload and preload for the working ventricle?

3. What factors limit the increase in cardiac output as right atrial pressure is increased?

4. What factors alter the velocity of action potential conduction in the heart?

576
Cardio Practice Exam & Review - Dr. Baumgarten

5. List the principle membrane conductances and their role(s) in cardiac electrical
activity. What characteristics are important for determining the action potential
configuration?

6. Identify differences in the electrophysiologic properties of ventricle, atrial, and AV


node.

7. Explain the morphology of the QRS complex and T-wave in lead II.

8. The P-wave is positive in aVF and not detectable in lead I. Describe the vector
produced by atrial activation.

577
Cardio Practice Exam & Review - Dr. Baumgarten

CARDIOVASCULAR PRACTICE EXAM QUESTIONS


(Annotated answers follow. Try the questions before looking at the answers.)

SELECT: A if 1, 2 and 3 only are correct.


B if 1 and 3 only are correct.
C if 2 and 4 only are correct.
D if 4 only is correct.
E if ALL are correct.

1. The cardiac action potential:

1. is longer in duration in Purkinje fibers than in atrial muscle.


2. spreads from cell-to-cell by transmitter release at intercalated disks.
3. conducts most rapidly in the His-Purkinje system.
4. is initiated in the SA node by norepinephrine release.

2. During phase 4:

1. Em in ventricle approaches EK because gK1 is greater than other conductances.


2. increasing the intracellular concentration of K+ would make atrial Em more
positive.
3. Em is more negative in Purkinje fibers than in AV node.
4. If activates (turns on) slowly and increases the net outward current in SA node.

3. Which of the following would tend to reduce the duration of a Purkinje fiber action
potential? Asume that ionic gradients are not changed.

1. slowing the inactivation (turn off) of ICa.


2. decreasing the rate of ion transport by the Na+-K+ pump.
3. slowing the activation (turn on) of gK.
4. increasing heart rate from 60 to 120 bpm.

4. During the upstroke of an atrial action potential:

1. Depolarization is regenerative.
2. INa decreases because depolarization inactivates Na+ channels.
3. INa increase because depolarization activates Na+ channels.
4. An ACh-induced decrease of Ca2+ current will slow the rate of depolarization.

578
Cardio Practice Exam & Review - Dr. Baumgarten

5. Normal automaticity is:

1. increased by interventions that make maximum diastolic potential more


positive.
2. suppressed in latent pacemakers by normal sinus node activity.
3. increased by interventions that make the threshold potential more negative.
4. found in SA and AV nodes, His-Purkinje system and ventricular muscle.

6. Which of the following statements regarding conduction velocity is/are true?

1. Conduction velocity in AV node can be made the same as in ventricle by


sufficiently hyperpolarizing the AV node.
2. A decrease in the number of gap junctions will slow conduction velocity
because membrane resistance would be increased.
3. Action potential duration is roughly inversely proportional to conduction
velocity in Purkinje fibers.
4. Increasing serum K+ from 3 to 20 mM will slow conduction velocity in
Purkinje fibers by inactivating Na+ channels.

7. Which of the following statements regarding excitation-contraction coupling in


mammalian ventricular muscle is/are true?

1. Peak cytoplasmic Ca2+ concentration depends on ICa during the action potential
that initiates the twitch.
2. Interventions that slow the rate of Na+-Ca2+ exchange will increase the force of
contraction.
3. Peak cytoplasmic Ca2+ concentration depends ICa during several action
potentials preceding the twitch.
4. ICa supplies the majority of the Ca2+ that activates the myofilaments.

8. Contractility can be increased by:

1. increasing the rate of firing of post-ganglionic sympathetic neurons.


2. increasing the resting length of the muscle before excitation.
3. increasing the cytoplasmic Ca2+ concentration during the twitch.
4. tetanic stimulation of the heart.

579
Cardio Practice Exam & Review - Dr. Baumgarten

9. During isometric contraction of ventricular muscle:

1. increasing afterload increases the amount of shortening


2. increasing contractility increases the maximum preload
3. the initial velocity of shortening is increased by increasing preload
4. lmax corresponds to a resting sarcomere length of 2.2 μm

10. Contractility of the heart suddenly decreases. As compared to measurements made


immediately after the decrease in contractility, Frank-Starling compensation will
result in:

1. increased stroke volume


2. increased right atrial end diastolic pressure
3. increased left ventricular end diastolic volume
4. decreased left ventricular end systolic volume

11. A shift of the cardiac function curve from curve A to curve B in the above diagram
may be caused by:

1. increased left ventricular end diastolic volume.


2. increased total peripheral resistance.
3. increased heart rate from 125 to 175 bpm.
4. increased contractility.

580
Cardio Practice Exam & Review - Dr. Baumgarten

12. Assuming the ventricle is accurately modeled by a thin-walled sphere, which of the
following relationships is/are correct for the intact ventricle at the end of diastole?

1. sarcomere length α 1/ventricular radius


2. sarcomere length α end diastolic pressure
3. sarcomere length α wall thickness
4. sarcomere length α ventricular radius

13. Acetylcholine:

1. decreases atrial contractility


2. increases atrial conduction velocity.
3. decrease AV node conduction velocity
4. decreases automaticity of the AV node

14. Increased firing of cardiac sympathetic neurons causes:

1. increased twitch tension in ventricular muscle.


2. decreased twitch duration in ventricular muscle.
3. increased conduction velocity in AV node.
4. increased automaticity of Purkinje fibers.

15. Which of the following statements regarding coupling of cardiac receptors to the
final physiologic effect is/are correct?

1. β1 receptors activate adenylate cyclase via GK.


2. ACh increases Ca2+ channel phosphorylation by the following pathway:
ACh binding → ↑ GK → ↑ cAMP → ↑ Ca2+ channel phosphorylation
3. G proteins directly convert ATP to 3',5'-cyclic AMP.
4. G proteins are heterotrimers that remain within the cell membrane.

581
Cardio Practice Exam & Review - Dr. Baumgarten

ANNOTATED ANSWERS

1. B
1. The action potential duration is longest (350-400 ms) in the His-Purkinje
system; ~250 ms in ventricle and 150 ms in atria, AVN and SAN.
2. Wrong. Impulse conduction is electrical.
3. The impulse conducts at 4 m/s in His-Purkinje system. In comparison,
conduction velocity is ~1 m/s in atria and ventricle and extremely slow in
AVN and SAN.
4. Wrong. Impulse initiation is myogenic, i.e., it spontaneously arises in the
SAN. NE modulates the rate of firing (chronotropic effect) but does not initiate
the action potential.

2. B
1. The membrane is most permeable to K+. gK1 is the major conductance at the
resting potential.
2. Wrong. Increase intracellular K+ increases the K+ gradient. This shifts EK and
Em towards more negative potentials. Remember: EK = -61 log([K+]i/[K+]o).
3. Em in AVN and SAN is about −60 mV; it is about−85 mV in the rest of the
heart.
4. Wrong. If activates slowly during phase 4, but it carries an increasing inward
current.

3. D
1. Wrong. More inward current would increase APD.
2. Wrong. The Na+-K+ pump produces an outward current. Decreasing an
outward current would prolong APD.
3. Wrong. Again, this is decreasing an outward current.
4. APD is inversely related to heart rate.

4. A
1. Yes. Beyond threshold, the depolarizing inward current increases gNa and
results in a further increase of inward current. It is self-sustaining or
regenerative.
2. Yes. Na+ channels are closed (inactivated by depolarization as the upstroke
continues.
3. Yes. Na+ channels are opened (activated) by depolarization. (Very nasty
question, but depolarization really both opens Na+ channels during most of the
phase 0, but also starts to closes Na+ channels as the depolarization gets more
and more positive.)
4. ACh decreases ICa, but phase 0 of an atrial action potential is unaffected
because it is mediated by INa.

582
Cardio Practice Exam & Review - Dr. Baumgarten

5. A
1. Yes. Decreases the amount of depolarization needed to reach threshold
potential.
2. Yes. Latent pacemakers normally do not express their automaticity.
Stimulation by sinus beats causes overdrive suppression.
3. Yes. Decreases the amount of depolarization needed to reach threshold
potential.
4. No. Ventricular muscle does not exhibit normal automaticity.

6. D
1. No. There are fewer inward current carrying channels in AVN, and the
maximum ICa in AVN is far less than INa in ventricle. Nevertheless,
hyperpolarization will modestly increase AVN conduction velocity.
2. No. Decreasing the number of gap junctions will slow conduction velocity,
but gap junctions contribute to ri not rm.
3. No. APD is independent of conduction velocity; APD is inversely related to
heart rate.
4. Yes. Increasing serum K+ depolarizes Purkinje fibers. One moves down the
responsiveness curve (decreased dV/dt), Na+ channels inactivate, and thus
conduction velocity is slowed.

7. A
1. Yes. ICa is the trigger of Ca2+ release from the SR. Release is roughly
proportional to the size of the trigger.
2. Yes. Slowing the extrusion of Ca2+ by Na+-Ca2+ exchange will allow the SR to
accumulate more Ca2+. Release is roughly proportional to the size of the SR
Ca2+ stores.
3. Yes. ICa not only releases Ca2+ from the SR but also provides Ca2+ to reload
the SR Ca2+ stores. Ca2+ influx by ICa must exactly balance efflux by Na+ −
Ca2+ exchange and the sarcolemmal Ca2+ pump for the cell to be in a steady
state (i.e., constant twitch).
4. No. The SR is the proximate source of most of the Ca2+ that activates the
myofilaments.

8. B
1. Yes. Sympathetic stimulation increases contractility.
2. No. This would increase the force of contraction because of more optimal
overlap of thin and thick filaments. Reflects muscle mechanics not
contractility.
3. Yes. Ca2+ activates the myofilaments.
4. No. The heart cannot be stimulated tetanically. The long refractory period
prevents this. Tetanic stimulation increases contractile force in skeletal
muscle, however.

583
Cardio Practice Exam & Review - Dr. Baumgarten

9. D
1. No. There is no shortening during an isometric contraction.
2. No. Preload reflects the passive properties of the muscle and is not affected by
contractility.
3. No. There is no shortening during an isometric contraction. (This would be
true for an isotonic contraction, however.)
4. Yes. This is the sarcomere length at the peak of the twitch tension curve. The
normal working (resting) length of cardiac sarcomeres is 1.9 μm.

10. A

1. Yes
2. Yes
3. Yes
4. No

11. D
1. No. Would shift along curve,
not from one curve to another.
2. No. Shifts from B to A
3. No. Shifts from B to A.
Increasing heart rate up to 125
bpm would shift from A to B.
At greater HR, decreased time
for ventricular filling limits SV
and CO.
4. Yes. Greater contractility gives
increased shortening and
increased SV from any muscle
length (i.e., EDV).

584
Cardio Practice Exam & Review - Dr. Baumgarten

12. C
1. The Law of Laplace for a thin-walled sphere is: P = 2HT/r, where P is
pressure, H is wall thickness, T is wall tension, and r is radius. Remember that
sarcomere length is set by wall tension and that EDV and thus r is proportional
to EDP.
2. No. T α r
3. Yes. T α r
4. No. T α 1/H
5. Yes. T α r

13. E
1. Yes. Decreases ICa, the trigger for SR Ca2+ release, and thus contractility.
2. Yes. Increases IK1, hyperpolarizes Em, and thus increases CV.
3. Yes. Decreases ICa, the current underlying the AVN upstroke.
4. Yes. ACh suppresses If. Further, effects on both IK1, ICa tend to slow
automaticity.

14. E
1. Yes. Sympathetics stimulation (NE) causes positive inotropy. NE increases ICa
and the twitch.
2. Yes. NE speeds reaccumulation of Ca2+ by the SR and makes it easier for the
relaxation to occur by reducing the affinity of troponin-c for Ca2+. This
shortens the twitch duration and helps preserve adequate time for ventricular
refilling.
3. Yes. NE increase ICa, the basis of the AVN upstroke.
4. Yes. NE enhances If causing more rapid phase 4 depolarization.

15. D
1. No. β1 receptors act via Gs.
2. No. Muscarinic receptors act via GK, but inhibit adenylate cyclase and
decrease cAMP levels.
3. No. This is done by adenylate cyclase.
4. Yes. G proteins are membrane bound and have 3 different components - α, β
and γ. The α subunit dissociates from βγ upon activation. Both α and/or βγ
have effects on targets.

9-cardiovascular practice exam and review questions-2009.doc 8/7/2008

585
Body Fluids 1 and 2 - Dr. Costanzo

Body Fluids 1 and 2


Linda Costanzo, Ph.D.

OBEJCTIVES:

After studying this lecture, the student should understand:

1. The distribution of water between the major body fluid compartments.


2. How to measure the volumes of body fluid compartments using marker
substances.
3. The differences in composition between the major body fluid compartments.
4. The pathophysiology of the major fluid shift examples, including the predicted
changes in osmolarity, ECF and ICF volume, and hematocrit.
5. How to calculate new osmolarity and new ECF and ICF volumes following a
fluid shift.

I. BODY FLUID COMPARTMENTS

Water content (total body water, or TBW) comprises about 60% of body weight.
The percentage varies between 50-70%, depending on gender and amount of
adipose tissue. Males tend to have a higher percentage of water than females.
Water content is inversely correlated with adipose tissue. Infants have up to 75%
body weight as water, which is why severe diarrhea can be life-threatening.

Water is distributed between two major compartments: intracellular fluid (ICF)


and extracellular fluid (ECF), which are separated from each other by cell
membranes. ICF is 2/3 of TBW and ECF is 1/3 of TBW. ECF is further sub-
divided into two compartments, the interstitial fluid and plasma compartments,
which are separated from each other by capillary walls. Interstitial fluid is 3/4 of
ECF, and plasma water is 1/4 of ECF. Lymph, which is part of the ECF, is
interstitial fluid that is collected in the lymphatic vessels and then returned to the
plasma compartment.

An additional minor compartment is the transcellular fluid, which is not part of


ICF or ECF. Transcellular fluids are separated from the rest of the body fluids by
a layer of cells, and they include gastrointestinal, peritoneal, pleural, and
cerebrospinal fluids. Collectively, the volume of transcellular fluids is small, so
they are ignored in the above summary numbers.

586
Body Fluids 1 and 2 - Dr. Costanzo

Figure 1. Body fluid compartments. Total body water


is distributed between intracellular fluid and
extracellular fluid. Water as a percentage of body
weight is indicated for the major compartments.

A simple tool is the 60-40-20 rule. Approximately 60% of body weight is water
(TBW), 40% of body weight is ICF, and 20% is ECF. (ICF is 2/3 of TBW, i.e.,
40% of body weight; ECF is 1/3 of TBW, i.e., 20% of body weight.)

II. MEASUREMENT OF BODY FLUID COMPARTMENT VOLUMES

The volumes of body fluid compartments are measured with a method based on
the principle of dilution.

A. Method

1. A marker substance is selected, whose physical characteristics


are such that it distributes only in the body fluid compartment
whose volume you wish to calculate. For example, isotopic water
(e.g., D20) distributes throughout the TBW, and thus is a marker
for TBW. Mannitol, a large sugar, distributes throughout the ECF,
but does not cross cell membranes, and thus is a marker for ECF
volume. Radioactively labeled albumin distributes wherever

587
Body Fluids 1 and 2 - Dr. Costanzo

albumin is located, and thus is a marker for plasma volume. See


below for list of the marker substances.

2. A known amount of the marker is given. Wait for equilibration,


then measure the concentration of the marker. Correct for any
losses of marker that occurred during the equilibration period (e.g.,
excretion in urine).

3. Knowing the amount present in the body (amount given any minus
loss during equilibration) and the measured concentration,
calculate the volume of distribution of the marker substance (the
volume it was dissolved in). This is the volume of that body fluid
compartment, e.g., volume of distribution of D20 is volume of
TBW, etc.

Volume = amount
concentration

or, more specifically

Volume = amount given - amount lost during equilibration


concentration

B. Marker substances

Direct measurements and their marker substances:

Marker Substances
D20
TBW HT0
Antipyrene
Mannitol
ECF Inulin
Radioactive sulfate
Radioiodinated serum albumin (RISA)
Plasma Evan’s blue (dye that binds to serum
albumin)

Indirect measurements (there is no unique marker substance for these


compartments):

ICF TBW - ECF


Interstitial ECF - plasma

588
Body Fluids 1 and 2 - Dr. Costanzo

C. Example

A 70 kg male is injected with 1.5 g of mannitol. During the equilibration


period, 5% of the mannitol was excreted/hour. After two hours of
equilibration, the plasma concentration of mannitol was measured as 9
mg/100 ml. What body fluid compartment is being measured, and what is
its volume (i.e., what is the volume of distribution of mannitol)? Is this a
reasonable number?

Volume = 1500 mg - 150 mg


9 mg/100 ml

= 1350 mg
90 mg/L

= 15 L (volume of distribution of mannitol, or ECF volume)

An ECF volume of 15 L is reasonable for a 70 kg male. (70 kg ≈ 70 L.


20% of 70 L = 14 L......close enough for an approximation.)

III. COMPOSITION OF BODY FLUID COMPARTMENTS

The major difference in composition of body fluid compartments is between ICF


and ECF, which are essentially “mirror images” of each other – what’s high in
concentration in ECF is low in ICF, and vice versa. Transporters in cell
membranes create and maintain most of these differences in composition. There
are also small differences in composition between plasma and interstitial fluid
(which are both ECF); these differences in composition are due to the Gibbs-
Donnan effect of plasma protein, whereby interstitial fluid has a slighter higher
concentration of small anions (e.g., Cl-) and a slighter lower concentration of
small cations (e.g., Na+) than plasma.

Solute Plasma Interstitial Intracellular


Na+, mmol/L 144 140 15
K+, mmol/L 4.8 4.5 120
Ca2+, mmol/L (ionized) 1.3 1.2 10-7M
Cl- , mmol/L 100 109 20
HCO3- , mmol/L 24 25 15
Protein, g/dL 7 ---- 30
Osmolarity, mOsm/L 290 290 290

A few tidbits on units. Please save for reference!

1. Concentrations in body fluids are often expressed in molarity, such as


mmol/L.

589
Body Fluids 1 and 2 - Dr. Costanzo

2. For electrolytes, we sometimes use equivalents, such as mEq/L, which


is the concentration in mmol/L x charge on the ion. Thus, for univalent
ions, mEq/L = mmol/L; for divalent ions, mEq/L = 2 x mmol/L. That
is, a Na+ concentration of 1 mmol/L = 1 mEq/L; a Ca2+ concentration
of 1 mmol/L = 2 mEq/L.

3. Osmolarity is total solute concentration, expressed in units of


mOsmoles/liter. Osmolarity is concentration of solute particles, or
concentration in mmol/L x number of particles that dissociate in
solution. The number of particles that dissociate in solution is called
“g,” the osmotic coefficient. For example, the osmolarity of 150
mmol/L NaCl = 150 mmol/L x 2 = 300 mOsm/L (since NaCl
dissociates into two particles in solution, i.e., g = 2). Osmolality is
virtually the same thing as osmolarity, but expressed as mOsmoles/kg
H20. Plasma osmolarity can be approximated
as 2 x [Na+]. I will show you a more precise estimate of plasma
osmolarity in a subsequent lecture.

4. Substances like proteins are conventionally expressed in g/dL, where a


dL (deciliter) is 100 ml and is also called “%.”

a. % can mean “g per 100 ml.” For example, 0.9% NaCl is 0.9 g
NaCl/100 ml. It’s weird, but that’s what it means.
b. mg % means “mg per 100 ml.” For example, 5 mg% KCl
means 5 mg KCl/100 ml.

IV. FLUID SHIFTS - QUALITATIVE

A. Definitions and rules for fluid shifts

1. Osmolarity is the concentration of solute particles, in units of


mOsm/L.

2. Osmolarities of ECF and ICF are always equal in the steady state
(see Table above).

3. H2O shifts freely across cell membranes to establish and maintain


this equality. (This is the “fluid shift” we’ll be talking about.)

4. If a disturbance causes a change in ECF osmolarity, thus producing


a transient difference in ECF and ICF osmolarity, H2O shifts
between ECF and ICF until the osmolarities are equal again; once
the fluid shift has occurred, this is called the new steady state.

5. For purposes of discussion, we assume that NaCl, NaHCO3, and


mannitol are “extracellular” solutes; that is, they are confined to

590
Body Fluids 1 and 2 - Dr. Costanzo

ECF because they do not cross cell membranes.

6. Fluid shift disturbances are categorized according to whether they


involve an increase or decrease in ECF volume:

a. Volume contraction means a decrease in ECF volume;


also called volume depletion. Volume contraction causes
decreased blood volume and decreased blood pressure (Pa).
b. Volume expansion means an increase in ECF volume.
Volume expansion can cause increased Pa and edema.

7. Fluid shift disturbances are also categorized according to whether


they cause a change in body fluid osmolarity:

a. Isosmotic means no change in body fluid osmolarity


b. Hyperosmotic means body fluid osmolarity is increased
c. Hyposmotic means body fluid osmolarity is decreased

B. Method for analyzing fluid shift problems – do it this way every time!

1. Read the problem or case scenario and determine clearly what was
gained or lost. For example, if a person eats dry NaCl, then NaCl
was gained. If a person sweats profusely on a hot day, then NaCl
and water were lost.

2. Assume that any gain or loss from the body affects the ECF first.

3. Predict whether the gain or loss would cause a change in ECF


osmolarity. For example, if a person eats dry NaCl, then NaCl is
added to ECF and causes an increase in ECF osmolarity.

4. If there is a predicted change in ECF osmolarity, determine which


way water must shift to make the ECF and ICF osmolarities equal
again.

5. Finally from your analysis above, predict the directional changes in


the new steady state (after any fluid shift has occurred) for: ECF
and ICF osmolarities, ICF volume, ECF volume, and TBW. Also
predict whether there will be a change in hematocrit and plasma
protein concentration. (Reminder: hematocrit is the fractional
blood volume occupied by red blood cells [RBCs].)

C. Examples (see following table and figure)

1. Loss of isosmotic NaCl (isosmotic volume contraction) – e.g.,


diarrhea. A person who has diarrhea loses isosmotic (and isotonic)

591
Body Fluids 1 and 2 - Dr. Costanzo

fluid from the GI tract. The loss causes no change in ECF


osmolarity since the fluid lost has the same osmolarity as the body
fluids. Since there is no change in ECF osmolarity, no fluid shift is
required. Thus, in the new steady state, ECF and ICF osmolarities
are unchanged, ECF volume is decreased (due to the loss of
isosmotic fluid, and ICF volume is unchanged. TBW is decreased
because ECF volume is decreased. In considering the effects of the
disturbance on plasma protein concentration and hematocrit,
remember that plasma is part of ECF; if ECF volume decreases,
then plasma volume also decreases. Plasma protein concentration
is increased by a concentrating effect (the fluid lost in diarrhea
does not contain plasma proteins). Hematocrit is also increased by
a concentrating effect, because the same number of RBCs are
“dissolved” in a smaller plasma volume.

2. Loss of water (hyperosmotic volume contraction) – e.g. water


deprivation and diabetes insipidus (lack of ADH, antidiuretic
hormone). For example, a person with a high fever loses
“insensible” water. If this water is not replaced, there will be an
increase in ECF osmolarity (water is lost from ECF, solute is left
behind and becomes concentrated). Thus, transiently, ECF
osmolarity is higher than ICF osmolarity. The body will not permit
this inequality, and water shifts from ICF into ECF until ECF and
ICF osmolarities are again equal, and both higher than normal. In
the new steady state, ECF and ICF osmolarities are increased. ECF
volume is decreased (because of the initial loss of water). ICF
volume is decreased (because of the water shift). TBW is
decreased. Plasma protein concentration is increased (because the
loss of ECF volume concentrates the plasma proteins). Hematocrit,
it seems, would also be increased. However, hematocrit is
unchanged because of two offsetting effects. (1) The loss of ECF
and plasma volume “concentrates” the RBCs (same number of
RBCs in a smaller volume), which tends to increase hematocrit. (2)
RBCs are cells. In this example, there is a water shift out of cells,
causing the RBCs to shrink, and therefore occupy a smaller
fractional volume, which tends to decrease hematocrit.

3. Loss of NaCl (hyposmotic volume contraction) – e.g, adrenal


insufficiency. In adrenal insufficiency, there is lack of aldosterone,
the hormone that promotes renal Na+ reabsorption. When
aldosterone is lacking, there is excess urinary excretion of NaCl
and net loss of NaCl from the body. When NaCl is lost from ECF,
there is a decrease in ECF osmolarity. Thus, transiently, ECF
osmolarity is lower than ICF osmolarity. Water shifts from ECF to

592
Body Fluids 1 and 2 - Dr. Costanzo

ICF until the osmolarities are equal again, and both lower than
normal. In the new steady state, ECF and ICF osmolarities are
decreased. ECF volume is decreased (due to the water shift), ICF
volume is increased (due to the water shift), and TBW is
unchanged. Plasma protein concentration is increased due to
concentration of plasma proteins. Hematocrit is increased both due
to “concentration” of RBCs and due to the shift of water into RBCs
(causing them to swell).

4. Gain of isosmotic NaCl (isosmotic volume expansion) -- e.g.,


infusion of isosmotic saline. A person is infused with an isotonic
(and isosmotic) saline (NaCl) solution. The infusion would cause
no change in ECF osmolarity, since the infused solution has the
same osmolarity as the body fluids. Since there is no change in
ECF osmolarity, no fluid shift is required. Thus, in the new steady
state, ECF and ICF osmolarities are unchanged, ECF volume is
increased (due to the addition of the infused solution, and ICF
volume is unchanged. TBW is increased because ECF volume is
increased. Plasma protein concentration is decreased by dilution
(the infused solution contained no protein). Hematocrit is also
decreased because the same number of RBCs are “dissolved” in a
larger volume.

5. Gain of NaCl (hyperosmotic volume expansion) – e.g., high NaCl


intake. NaCl is added to ECF, and there is an increase in ECF
osmolarity. Thus, transiently, ECF osmolarity is higher than ICF
osmolarity. Water shifts from ICF to ECF until the osmolarities are
equal again, and both higher than normal. In the new steady state,
ECF and ICF osmolarities are increased. ECF volume is increased
(due to the water shift), ICF volume is decreased (due to the water
shift), and TBW is unchanged. Plasma protein concentration is
decreased due to dilution of plasma proteins. Hematocrit is
decreased both due to “dilution” of RBCs and due to the shift of
water out of RBCs (causing them to shrink).

6. Gain of water (hyposmotic volume expansion) – e.g., excess


water-drinking and SIADH (syndrome of inappropriate ADH).
Water is first added to ECF and there is a decrease in ECF
osmolarity. Transiently, ECF osmolarity is lower than ICF
osmolarity. The body will not permit this inequality, and water
shifts from ECF into ICF until ECF and ICF osmolarities are again
equal, and both lower than normal. In the new steady state, ECF
and ICF osmolarities are decreased. ECF volume is increased
(because of the initial addition of water). ICF volume is increased

593
Body Fluids 1 and 2 - Dr. Costanzo

(because of the water shift). TBW is increased. Plasma protein


concentration is decreased (because the increased ECF volume
dilutes plasma proteins). Hematocrit, it seems, would also be
decreased. However, hematocrit is unchanged because of two
offsetting effects. (1) The increase in ECF and plasma volume
“dilutes” the RBCs (same number of RBCs in a larger volume),
which tends to decrease hematocrit. (2) RBCs are cells, and in this
example, there is a water shift into cells, causing the RBCs to
swell, and therefore occupy a larger fractional volume, which tends
to increase hematocrit.

Disturbances of Body Fluids


ECF ICF Plasma
Type Example Osmolarity Hematocrit
Volume Volume [protein]
Isosmotic volume
contraction
Diarrhea ↓ N.C. N.C. ↑ ↑
Sweating;
Hyperosmotic volume
contraction
fever; diabetes ↓ ↓ ↑ N.C. ↑
insipidus
Hyposmotic volume Adrenal
contraction insufficiency
↓ ↑ ↓ ↑ ↑
Isosmotic volume Infusion of
expansion isotonic NaCl
↑ N.C. N.C. ↓ ↓
Hyperosmotic volume High NaCl
expansion intake
↑ ↓ ↑ ↓ ↓
Hyposmotic volume
expansion
SIADH ↑ ↑ ↓ N.C. ↓
ECF, Extracellular fluid; ICF, intracellular fluid; NaCl, sodium chloride; N.C., no change; SIADH syndrome
of inappropriate antidiuretic hormone.

594
Body Fluids 1 and 2 - Dr. Costanzo

Figure 2. Shifts of water between body fluid compartments. Normal extracellular fluid
(ECF) and intracellular fluid (ICF) osmolarity are shown by solid lines. Changes in volume
and osmolarity in response to various disturbances are shown by dashed lines. SIADH,
Syndrome of inappropriate antidiuretic hormone.

V. FLUID SHIFTS - QUANTITATIVE

A. How to analyze and calculate. Fluid shift problems can also be analyzed
quantitatively. That is, in addition to the qualitative approach above (e.g.,
whether osmolarity is increased or decreased, and whether ECF volume is
increased or decreased), we also can calculate the exact values for new
steady state osmolarity and body fluid volumes. That’s what I mean by
“quantitative.” To work these problems correctly and reliably, you must
perform the following steps in the following order. In the next section of
Examples, you will see how to work problems using these steps.

1. First, determine clearly what was gained or lost in the problem.


From the case scenario, calculate the number of osmoles
(mosmoles) gained or lost and the volume (L) gained or lost.
2. Next, calculate the new osmolarity of TBW in the new steady state.
We do this step next because we know that the new steady state
osmolarity will be the same throughout the body fluid

595
Body Fluids 1 and 2 - Dr. Costanzo

compartments (TBW). The calculated value of TBW osmolarity


will be the value for ECF and ICF osmolarities used in Step 3.
3. Finally, using the new, calculated TBW osmolarity (per Step 2),
calculate the new ECF and ICF volumes.

B. Examples

1. A man with a TBW of 40 L, ICF volume of 26.4 L, ECF volume of


13.6 L, and plasma osmolarity of 290 mOsm/L drinks 3 L of water.
In the new steady state, what is his plasma osmolarity, TBW, ECF
volume, and ICF volume?

What was gained or lost?


Gain = 3 L of water

New TBW osmolarity?


Old TBW osmoles = 40 L x 290 mOsm/L = 11,600 mOsm
New TBW = 40 L + 3 L = 43 L
11,600 mosmoles/43 L = 269.8
New TBW osmolarity =
mOsm/L

New ECF volume?


Old ECF osmoles = 13.6 L x 290 mOsmles/L
= 3944 mOsm
New ECF volume = 3944 mosmoles/ 269.8 mOsm/L
= 14.6 L

New ICF volume?


Old ICF osmoles = 26.4 L x 290 mOsm/L
= 7656 mOsm
New ICF volume = 7656 mOsm/269.8 mOsm/L
= 28.4 L

2. A woman with an ICF volume of 28 L, ECF volume of 14 L, and


plasma osmolarity of 295 mOsm/L, eats a bag of potato chips that
contains 300 mmoles of NaCl. Assuming that the osmotic
coefficient of NaCl is 2, in the new steady state, what is her plasma
osmolarity, TBW, ECF volume and ICF volume? How much water
shifted, and in which direction?

What was gained or lost?


Gain = 300 mmoles of NaCl
= 600 mosmoles (300 x 2)

596
Body Fluids 1 and 2 - Dr. Costanzo

New TBW osmolarity?


Old TBW = 28 L + 14 L
= 42 L
Old TBW osmoles = 42 L x 295 mOsm/L
= 12,390 mOsm
New TBW osmoles = 12,390 mOsm + 600 mOsm
= 12,990 mOsm
New TBW osmolarity= 12,990 mOsm/42L
= 309.3 mOsm/L

New ECF volume?


Old ECF osmoles = 14 L x 295 mOsm/L
= 4130 mOsm
New ECF osmoles = 4130 mOsm + 600 mOsm
= 4730 mOsm
New ECF volume = 4730 mOsm/309.3 mOsm/L
= 15.3 L

New ICF volume?


Old ICF osmoles = 28 L x 295 mOsm/L
= 8260 mOsm
New ICF volume = 8260 mOsm/309.3 mOsm/L
= 26.7 L

How much water shifted, and in what direction?


1.3 L, from ICF to ECF

3. A man with a TBW of 40 L, ECF volume of 13 L, ICF volume of


27 L, and plasma osmolarity of 300 mOsm/L is infused with 2 L of
a solution of 0.45% NaCl. After the infusion, in the new steady
state, what is his plasma osmolarity, ECF volume, and ICF
volume? (For NaCl, assume molecular weight is 58 g/m and
osmotic coefficient is 2.0.)

What was gained or lost?

2 L of 0.45% NaCl
0.45 g/100 ml x 2000 ml ÷ 58
= 0.155 moles
g/mole
0.155 moles x 2 = 0.310 osmoles
= 310 mosmoles

Gain = 310 mosmoles


and
2L

597
Body Fluids 1 and 2 - Dr. Costanzo

New TBW osmolarity?


Old TBW osmoles = 40 L x 300 mOsm/L
= 12,000 mOsm
New TBW osmoles = 12,000 mOsm + 310 mOsm
= 12,310 mOsm
New TBW osmolarity= 12,310 mOsm/42 L
= 293 mOsm/L

New ECF volume?


Old ECF osmoles = 13 L x 300 mOsm/L
= 3900 mOsm
New ECF osmoles = 3900 mOsm + 310 mOsm
= 4210 mOsm
New ECF volume = 4210 mOsm/293 mOsm/L
= 14.4 L

New ICF volume?


Old ICF osmoles = 27 L x 300 mOsm/L
= 8100 mOsm
New ICF volume = 8100 mOsm/293 mOsm/L
= 27.6 L

VI. PRACTICE QUESTIONS

1. Woman with ECF volume= 15 L, ICF volume =25 L, and plasma


osmolarity = 300 mOsm/L runs a marathon on a hot day. She loses 3 L of
sweat that has an osmolarity of 200 mOsm/L, and replaces all volume lost
by drinking pure water.

Her plasma osmolarity in new steady state?


New ECF volume?
New ICF volume?
New TBW?
Hct inc, dec, or unchanged?
New plasma Na concentration (inc, dec, or unchanged)?

2. Man with TBW= 45L, ECF volume=17L, and plasma osmolarity= 300
mOsm/L eats some yummy Sunchips (original) containing 450 mOsmoles
of NaCl. Being on a tight budget, he washes them down with 1.5 L of
water.

New osmolarity?

598
Body Fluids 1 and 2 - Dr. Costanzo

Direction of water shift?


Plasma protein inc, dec, or unchanged?
Hct inc, dec, or unchanged?
New plasma Na concentration (inc, dec, or unchanged)?

3. Woman has TBW=42 L, ECF volume = 15 L, ICF volume=27 L, and


plasma osmolarity=290 mOsm/L. She develops an infection with high
fever and loses 2 L of water in insensible losses. She is unable to drink or
eat.

New plasma osmolarity?


Approximate (value) new plasma Na concentration?
New TBW?
New ICF volume?

4. A man is injected with 2000 :Ci of tritiated water (HTO) and 4000 mg of
inulin. After equilibration, a plasma sample had an HTO concentration of
4 :Ci/100 ml and an inulin concentration of 16 mg/100 ml. During
equilibration, 20% of the inulin injected was excreted in the urine, and 2%
of the HTO injected was excreted. What are the man’s TBW, ECF, and
ICF volumes?

VII. ANSWERS

For Questions 1-3, I intentionally gave numerical answers or directional changes


without the step-by-step solutions. That way, if you get it wrong, you can try
again without having seen clues in the solution – you’ll learn better that way. If
you’re stumped, come by and see me.

1. Her plasma osmolarity in new steady state? 285 mOsm/L


New ECF volume? 13.68 L
New ICF volume? 26.32 L
New TBW? 40 L
Hct inc, dec, or unchanged? Inc
New plasma Na concentration? Dec

2. New osmolarity? 300 mOsm/L


Direction of water shift? None
Plasma protein inc, dec, or unchanged? Dec
Hct inc, dec, or unchanged? Dec
New plasma Na concentration? No change

599
Body Fluids 1 and 2 - Dr. Costanzo

3. New plasma osmolarity? 304.5 mOsm/L


Approximate new plasma Na concentration? Approx 152 mEq/L
New TBW? 40 L
New ICF volume? 25.7 L

4. TBW = 2000 :Ci - 40 :Ci


4 :Ci/100 ml
= 49 L

ECF = 4000 mg - 800 mg


16 mg/100 ml
= 20 L

ICF = TBW - ECF


= 49 L - 20 L
= 29 L

600
Intro to Gastro Phys & GI Hormones - Dr. Grider

Introduction to Gastrointestinal Physiology and GI Hormones


Jack Grider, Ph.D

OBJECTIVES:

1. Describe the general organization of the gut.


2. Identify the levels of neural control and describe the interaction between these
components.
3. Compare and contrast the neural, hormonal, and paracrine control mechanisms.
4. List the stimuli for the release of the main hormones.
5. Differentiate the physiological and pharmacological actions of the main gut
hormones.

Suggested Reading: Berne & Levy pp. 539-543, 545-548

I. FUNCTION OF THE GI TRACT

The main function is the conversion of ingested material to a form that is easily
absorbed by cells that line the gut (enterocytes). Optimal conditions for this
require regulation of the movements of smooth muscle (Motility); addition of
digestive enzymes, fluids, and electrolytes to the lumen to insure proper dilution
and regulation of pH (Secretion); and movement of digested end products across
the cell membrane for delivery to the rest of the body (Absorption). Undigested
residual material is excreted as feces. The process requires coordination of many
cells/tissues; this coordination is accomplished by the activity of nerves,
hormones, paracrine agents and autocrine agents.

II. FUNCTIONAL ANATOMY

A. Basic structure (fig 1):


A long muscular tube with several functional segments.

1. Mouth: chewing, saliva added, formation of bolus, initiate digestion


2. Esophagus: conduit
3. Stomach: major function is storage.

a. Other functions include physical grinding of solids,


acidification, some digestion, regulation of flow into intestine.
b. Formation of chyme, the term for a mixture of food particles,
enzymes, and gut secretions.

4. Small Intestine: three division- duodenum, jejunum, ileum

601
Intro to Gastro Phys & GI Hormones - Dr. Grider

a. main function is digestion and absorption

5. Large Intestine: three division: cecum, colon, rectum

a. main functions include drying of chyme into feces, storage of


feces, final absorption of water and electrolyte.

B. Sphincters: (figure 2)
Between each major region, there is a thickening of the muscle (circular)
layer called a sphincter. These function to regulate flow.

1. Upper & Lower Esophageal


2. Pyloric
3. Oddi
4. Ileo-cecal
5. Anal (internal & external)

Figure 1. Summary of Functions

602
Intro to Gastro Phys & GI Hormones - Dr. Grider

Figure 2. Sphincters

C. Associated structures
These serve to facilitate digestion and absorption by adding fluid, electrolyte,
and digestive enzymes and co-factors .

1. Salivary glands
2. Liver
3. Gallbladder
4. Pancreas

D. Organization of the gut wall (Figure 3)


From outside to inside there are multiple concentric layers

603
Intro to Gastro Phys & GI Hormones - Dr. Grider

Figure 3.

III. MECHANISM OF CONTROL

A. A large number of regulatory cells and mechanisms are needed to organize


and control the functions of the gut.

1. There are 108 neuronal cell bodies in the gut.


2. There are >20 identified hormones.
3. 70 to 80% of all of the body's immune cells are found in the gut
(GALT or Gut Associated Lymphatic Tissue).
4. Note that even with all these cells, there are still 10 non-mammalian
cells for every mammalian cell. Most of these non-mammalian cells
are bacteria (>500 species in colon alone).

B. Neural: Fine control of discrete regions.


Mediated by both extrinsic (cell bodies outside of the gut) and intrinsic
(cell bodies within the wall of the gut) nervous systems. The latter is
referred to as the Enteric Nervous System (ENS).

604
Intro to Gastro Phys & GI Hormones - Dr. Grider

C. Hormonal/Endocrine Control is more generalized. The Hormone


is released into blood which then carries it to distant target site where it
acts on cells expressing the appropriate receptors. The gut is largest
endocrine organ in body.

D. Paracrine agent: These are released into extracellular space, diffuse short
distances to act on adjacent cells with appropriate receptors. Paracrine
cells often have a cytoplasmic projection so as to release paracrine agents
over long distances.

E. Autocrine agent: These are released into the extracellular space and act
on cells of its origin. This is an important mode of action for growth
factors.

Figure 4.

IV. NEURAL CONTROL OF THE GUT

A. The Extrinsic nervous system is part of the autonomic nervous system


and shares many of its general characteristics. (Figure 5)

605
Intro to Gastro Phys & GI Hormones - Dr. Grider

Figure 5.

606
Intro to Gastro Phys & GI Hormones - Dr. Grider

Figure 5a.

1. Parasympathetic nervous system (Figure 6)

a. Preganglionic fibers to the esophagus through the transverse


colon originate in nuclei of the brain stem (Dorsal Motor
Nucleus of the Vagus (DMV)) and travel in the vagus nerve
(Xth cranial nerve). Preganglionic fibers to the remainder of
the colon, rectum, and anus originate in the sacral division of
the spinal cord (S2-4) and travel in the pelvic nerve.
b. Preganglionic neurotransmitter is acetylcholine.
c. Postganglionic cell bodies are in the ganglia of the enteric
nervous system.
d. Postganglionic transmitter may be acetylcholine or one of
many neurotransmitters released by neurons of the enteric
nervous system (see B below)
e. Generally parasympathetic stimulation leads to an overall
excitation of the gut (e.g. increased motility, increased
secretion)
f. Vagus nerve contains many axons. There are about 90%
afferent fibers and 10% efferent. These relatively few efferent
fibers (about 5000) innervate many (108) enteric neurons,
therefore parasympathetic activation of gut leads to widespread
activation of the gut via the enteric nervous system.

607
Intro to Gastro Phys & GI Hormones - Dr. Grider

Figure 6.

2. Sympathetic nervous system (Figure 6)

a. Preganglionic fibers originate in the thoracic and lumbar


portions of the spinal cord and pass through the sympathetic
paravertebral (chain) ganglia to synapse with postganglionic
neuronal cell bodies located in the prevertebral ganglia
(celiac ganglia, superior and inferior mesenteric ganglia). The
preganglionic transmitter is acetylcholine.
b. Postganglionic fibers originate in prevertebral ganglia.
Neurotransmitter is primarily norepinephrine.
c. Most postganglionic fibers terminate on other neurons in the
enteric nervous system. Stimulation of sympathetic fibers
usually results in a general inhibition of gut function (e.g.
decreased motility) as a result of an inhibition of neural activity
within the enteric nervous system. This is mainly the result of
presynaptic inhibition of acetylcholine release from enteric
neurons. This inhibition is mediated by activation of alpha-2
receptors on presynatpic terminal of the cholinergic neuron.
d. Some fibers terminate on blood vessels where they cause
vasoconstriction, some terminate on glands where they cause
secretion and some terminate on smooth muscle cells of

608
Intro to Gastro Phys & GI Hormones - Dr. Grider

sphincters where they cause contraction. This contractile effect


on sphincteric smooth muscle is opposite to the general
inhibitory effect of sympathetic stimulation of gut motility.
Both, however, prevent movement of chyme through the gut.
(Figure 7)

Figure 7.

3. Afferent Nerves (Figure 8).

a. Afferent nerves detect sensory information from various layers


of the gut (mucosa, muscle and serosa). There are 3 types.

1.Vagal afferent fibers which originate in the nodose ganglion


and connect the gut to the Vagal Complex (Nucleus of the Tractus
Solitarius (NTS) and Dorsal Motor Nucleus (DMN)) in the brain
stem.

2. Extrinisc Primary afferent neurons which originate in


Dorsal Root Ganglion and connect the gut to the prevertebral
ganglion and spinal cord.

3. Intrinisic Primary afferent neurons (IPANs) which


originate in the myenteric and submucosal neural plexuses of the
enteric nervous system

609
Intro to Gastro Phys & GI Hormones - Dr. Grider

Figure 8.

4. Long Arc Reflexes

a. Both the parasympathetic and sympathetic nervous system


participate in reflex arcs that coordinate activity over long
distances and between different regions of the gut. These are
termed Long Arc Reflexes. In these reflexes, both the afferent
and efferent neurons are extrinsic to the gut although their
effect is mediated through the enteric (intrinsic) nervous
system.
b. Long arc reflexes using the parasympathetic nervous system
are generally excitatory VagoVagal Reflexes (both afferent
and efferent neurons are in the vagus nerve. (Figure 9). The
vagal efferent ends on neurons of the enteric nervous system.

610
Intro to Gastro Phys & GI Hormones - Dr. Grider

Figure 9.

c. Long arc reflexes using the sympathetic nervous system are


generally inhibitory. The afferent component is usually an
extrinsic primary afferent neuron and the efferent neuron is a
sympathetic neuron arising in the interomediate lateral horn of
the thoracic-lumbar spinal cord and in the prevertebral
ganglion. The efferents end on neurons of the enteric nervous
system. The prevertebral ganglia neurons receive input from
passing primary afferent neurons and from viscerofugal or
intestinofugal neurons arising in the enteric nervous system
(Figure 10) . These mediate reflexes through the prevertebral
ganglia and modify sympathetic efferent neurons. Sympathetic
reflexes can pass through the spinal cord and can also pass
between sympathetic prevertebral ganglia. An example is the
Intestino-Intestinal Reflex which mediates inhibition between
regions of the intestine (Figure 11).

611
Intro to Gastro Phys & GI Hormones - Dr. Grider

Figure 10.

Figure 11.

B. Enteric Nervous System (ENS) or Intrinsic Nervous System

1. General Comments

a. Neuronal cell bodies and fibers are contained within the wall of
the gut. The number of neurons in ENS is about equal to the
number in the spinal cord (108 ).
b. Often called the Mini-brain.
c. ENS is comprised of two ganglionated plexuses and
interconnecting fibers: the myenteric plexus and submucosal
plexus (Figure 12).

612
Intro to Gastro Phys & GI Hormones - Dr. Grider

Figure 12.

d. ENS controls local function via reflexes that operate


independently of the central nervous system. These are called
Short arc reflexes because they control local activity and do
not extend more than a few cm from the stimulus. Afferent and
Efferent neurons are wholly within the enteric nervous system.
The Peristaltic Reflex is an example. These reflexes can be
modified by extrinsic nervous system. (Figure 13.)

Figure 13.

613
Intro to Gastro Phys & GI Hormones - Dr. Grider

e. ENS neurons act as postganglionic neurons of the parasympathetic


nervous system and as the end organ of sympathetic nervous
system.
f. ENS integrates inputs from afferent nerves, sympathetic and
parasympathetic nerves, and nerves in the enteric nervous system
(mini-brain). Most all information enters ENS system for
processing and determination of final response of gut tissues.
g. Sensory or afferent neurons for local arc reflexes are contained in
the enteric nervous system and are called Intrinsic Primary
Afferent Neurons or IPANs (see figure 8).
h. There are also enteric neurons that project axons to the prevertebral
ganglia (intestinofugal or viscerofugal neurons). These are
involved in long arc reflexes and in regulating sympathetic input to
the enteric neurons.
i. A variety of neurotransmitters are found in enteric neurons. Most
neurons contain and release more than one neurotransmitter.
Examples include:

i. Acetylcholine (ACh): The main transmitter in the enteric


nervous system. Ach stimulates both increased smooth
muscle contraction and secretions from cells and glands.
Found in excitatory motor neurons and interneurons.
ii. Enkephalins (opioid peptides): Generally inhibitory and
restrain release of ACh from enteric neurons. Found mainly
in interneurons.
iii. Tachykinins (TK) (substance P and neurokinin A):
major contractile transmitter mediating non-cholinergic
responses. Found in excitatory motor neurons and
interneurons.
iv. Vasoactive intestinal peptide (VIP): Major relaxant
transmitter for smooth muscle and excitatory transmitter for
secretory cells. Found in inhibitory motor neurons and
excitatory secretomotor neurons.
v. Gastrin releasing peptide (GRP); Found in long
interneurons of the intestine and in gastric enteric neurons
that stimulate acid secretion and gastrin secretion.
vi. Somatostatin; Major inhibitory transmitter of the gut.
Found in interneurons where it regulates the release of
other neurotrasnmitters.
vii. Nitric Oxide: relaxant transmitter; A gaseous transmitter
made on demand from L-Arginine by Nitric Oxide
Synthase (NOS).

j. Many other neurotransmitters are contained in enteric neurons.


k. Most tachykinin neurons also contain acetylcholine and account
for 40-50% of all ENS neurons. All VIP neurons contain NOS and

614
Intro to Gastro Phys & GI Hormones - Dr. Grider

account for 20-30% of all ENS neurons. There is no overlap


between TK/ACh neurons and VIP/NOS neurons.

1. Myenteric Plexus

a. This plexus is located in the muscularis externa between the


longitudinal and circular muscle layers. (Figure 9).
b. Fibers project mainly into the underlying muscle layer, but also
into submucosal plexus and mucosa. Activity primarily
responsible for control of muscle activity.

2. Submucosal Plexus

a. Located in submucosa.
b. Fibers project into underlying submucosa and mucosa
(including muscularus mucosa), but also to myenteric plexus.
In humans and large animals, these neurons also project to the
innermost layers of circular muscle.
c. Primarily responsible for control of secretion

V. HORMONAL CONTROL OF THE GUT

A. General comments

1. The gut is largest endocrine organ in the body.


2. Hormones are released from enteroendocrine cells located in
mucosa of gut. These cells extend from the lumen to basement
membrane. The Lumenal (apical) membrane has receptors for
chemical components of chyme and, if conditions are appropriate,
the cell releases hormone into blood from the basal side (see Figure
4).
3. The hormone is distributed throughout the body by the circulation
and cells which contain appropriate receptors for the hormone
respond.
4. Hormonal mechanisms are relatively more important in stomach and
small intestine and associated structures than in esophagus and
colon.
5. All gut hormones are polypeptides and all are amidated on the C-
terminal end.
6. To be accepted as a hormone, candidate must be shown to be present
in the gut, synthesized by an endocrine cell in the gut, the structure
identified, released by an appropriate physiological stimulus,
produce an appropriate physiological effect when infused
exogenously at the appropriate blood level, and to be blocked by
appropriate antagonists or antisera.

615
Intro to Gastro Phys & GI Hormones - Dr. Grider

7. There are many candidates which have met many but not all of these
criteria. Hormones which have met all criteria include gastrin,
cholecystokinin, secretin, glucose-dependent insulinotrophic
peptide (GIP). Stimuli for release and important physiological
actions are listed in Tables 1 and 2.

Table 1. Releasers of Gastrointestinal Hormones


Gastrin CCK Secretion GIP
Protein S S N S
Fat N S S- S
Carbohydrate N N N S
Acid I S- S N
Distension S N N N
Vagal Stimulation S N N N
S=stimulation; S- = of secondary importance; I=inhibition; N=no effect.

Table 2. Important PHYSIOLOGICAL Actions of Gastrointestinal Hormones


Hormones
Action Gastrin CCK Secretin GIP
Acid Secretion S I I
Pancreatic HCO3 Secretion S
Pancreatic Enzyme Secretion S
Bile HCO3 Secretion S
Gallbladder Contraction S
Gastric Emptying I I
Insulin Release S
Mucosal Growth S
Pancreatic Growth S S

B. Gastrin

1. Exists in several molecular sizes: main form is G-17, but also exists as
G-14, G-34 and larger forms.
2. C-terminal end is active as a fragment but has only 1/6 the potency of
G17.
3. Exists in sulfated and nonsulfated forms.
4. Found in G cells located in gastric antrum.
5. Released by:

a. mechanical distension of antrum,


b. vagal stimulation,

616
Intro to Gastro Phys & GI Hormones - Dr. Grider

c. presence of amino acid/peptides in antrum.

6. Release is inhibited when the antral pH drops below pH of 2 (a feed


back loop involving somatostatin mediates this effect).
7. Main physiological actions:

a. stimulate acid secretion,


b. stimulate growth of gastric mucosa.

C. Cholecystokinin (CCK)

1. Main form is CCK-33, but also exists as CCK-58 and CCK-5 (may be
neural form).
A fragment of the C-terminal end is active but is much less potent than
CCK-33.
2. Exists in only the sulfated form.
3. Located in I cells of the upper small intestine.
4. Released by amino acids/peptides and by monoglycerides/fatty acids
in upper small intestine.
5. Main physiological actions:

a. stimulates enzyme secretion by pancreas,


b. stimulates pancreatic growth,
c. stimulates gallbladder contraction,
d. inhibits gastric emptying.

Gastrin and Cholecystokinin make up a family of hormones. The C-terminal end


(last 5 amino acids Gly-Trp-Met-Asp-Phe) is identical in these hormones. This C-
terminal end is also responsible for receptor binding. This means that at high
concentrations, CCK and Gastrin can bind to each others cognate receptor and
produce the same effects, albeit with lower potency and efficacy. Sulfation of gastrin
is required for it to bind to the CCK receptor and mimic CCK.

At pharmacological levels or in pathological (tumor) conditions (see Table 3), CCK


and Gastrin can interact with each others receptors, but they are weaker stimulants. If
the weaker agent prevents the more potent agent from binding the receptor, there is a
net lower level of stimulation. This property is called: competitive antagonism. It is
the N-terminal end of the full molecule that makes each more potent at its own
receptor.

D. Secretin

1. Only one molecular form: 27 AA. The whole sequence is needed for
activity (no active fragment).
2. Located in S cells in upper small intestine.

617
Intro to Gastro Phys & GI Hormones - Dr. Grider

3. Released mainly by presence of acid (i.e. low pH) and to a lesser


extent monoglyceride/fatty acid in duodenum.
4. Main physiological actions:

a. stimulates secretion of bicarbonate from duodenal glands,


pancreas and bilary system,
b. stimulates pancreatic growth,
c. inhibits acid secretion.
d. inhibits gastric emptying

E. Glucose-dependent Insulinotropic Peptide (GIP)

1. Only one molecular form: 42 AA. The whole sequence is needed for
activity.
2. Located in K cells of upper small intestine.
3. Released by presence of monoglyceride/fatty acid, amino
acid/peptides, or carbohydrates in duodenum.
4. Main physiological actions:

a. stimulates insulin secretion by the pancreas


b. inhibits acid secretion by parietal cells of stomach.

Secretin and GIP are components of the Secretin Family which also includes
Glucagon, Vasoactive Intestinal Peptide (VIP), and Peptide Histidine Isoleucine
(PHI). The latter two are neurotransmitters. In this family, there is sequence
homology throughout structure, but no active fragment; the whole sequence is needed
for activity. Competitive antagonism occurs between members of this family.

TABLE 3: Pharmacological Actions (Actions at supraphysiological serum


levels)
Hormones
Action Gastrin CCK Secretin GIP
Acid Secretion S S I I
Gastric Emptying I I I I
Pancreatic HCO3 Secretion S S S N
Pancreatic Enzyme Secretion S S S N
Bile HCO3 Secretion S S S N
Gallbladder Contraction S S S ?
Gastric Motility S S I I
Intestinal Motility S S I I
Insulin Release S S S S
Mucosal Growth S S I I
Pancreatic Growth S S S ?

618
Intro to Gastro Phys & GI Hormones - Dr. Grider

S= stimulates; I=inhibits; N= no effect; ?= not tested

F. Other Important Candidate Hormones


1. Enteroglucagon: released from intestinal enteroendocrine cells in
response to a decrease in blood glucose; causes liver to increase
glycogenolysis and gluconeogenesis as well as increase lipolysis.
2. Pancreatic Polypeptide Family: Pancreatic polypeptide (PP)
released from pancreas in response to meal (especially amino acids)
and Peptide YY (PYY) released from ileum in response to lipids. Also
released by neural stimulation or by other hormones. Physiological
effect is inhibition of pancreatic enzyme secretion and an indirect
relaxation of gallbladder. PYY may play a role in feeding
behavior/satiety. A third member of family found in enteric nerves is
called neuropeptide Y (NPY). Since this is a family of related
peptides, they can all mimic each other’s actions at high
concentrations.
3. Somatostatin: released from intestinal cells in response to meal and
acid as well as by neural and hormonal factors. Inhibits secretion,
motility and hormone release throughout gut. Also found in nerves of
gut and brain, in endocrine cells of pituitary, and paracrine cells of the
gastric mucosa & pancreas.
4. Motilin: released from intestine by neural stimulation and maybe by
presence of biliary fluid in duodenum. Causes increased motility and
may have a role in induction of migrating myoelectric complex
(MMC) in stomach and upper intestine. A recently identified relative,
ghrelin, may play a role in feeding/satiety behaviors. Motilide
antibiotics act on motilin receptors to increase motility and are under
consideration as agents with which to treat decreased intestinal and
colonic transit.
5. Many other candidate hormones are presently being studied however
physiological characteristics for these have yet to be identified.

VI. PARACRINE CONTROL OF THE GUT

A. Release of paracine peptide into extracellular space where it diffuses to


adjacent or nearby cells. Those cells with appropriate receptors are excited
or inhibited.
B. Release of somatostatin by antral D cells diffuses to nearby antral G
cells where it inhibits the release of gastrin by the G cell. (Note:
somatostatin can thus be a neurotransmitter, hormone and a paracrine
agent).

VII. STUDY QUESTIONS

1. Short Arc reflexes

619
Intro to Gastro Phys & GI Hormones - Dr. Grider

A. involve the parasympathetic nerves.


B. usually only regulate a few cm of activity.
C. control mainly gastric secretion.
D. do not involve the submucosal plexus.
E. are mediated by the paravertebral ganglia.

2. Gastrin

A. Is release by cells of the duodenum.


B. Is a member of the secretin family of hormones.
C. Is present in nerves and endocrine cells.
D. Is released by vagal nerve stimulation.
E. Is released into the gastric lumen.

3. At physiological concentrations, secretin

A. Stimulates gastric emptying.


B. Inhibits acid secretion.
C. Causes VIP release.
D. Causes gallbladder contraction.
E. Simulates salivary secretion.

4. Most gut hormones are

A. Secreted from nerve terminals


B. Released in response to components of the chyme.
C. Complex carbohydrate molecules.
D. Located in cells of the stomach.
E. Present in the serosal layer of the gut.

5. Sympathetic nerves to the gut

A. Mainly synapse on enteric neurons.


B. Usually cause smooth muscle contraction.
C. Are rare in the intestine.
D. Release acetylcholine as neurotransmitter.
E. Originate in the brain stem.

VIII. Answers: 1 = B; 2 = D; 3 = B, 4=B, 5=A

620
Reg of Body Fluids: Na+ & Water - Dr. Costanzo

Regulation of Body Fluids: Na+ and Water


Linda Costanzo, Ph.D.

OBJECTIVES:

After studying this lecture, the student should understand:

1. Why body sodium content determines ECF volume and the relationships between
sodium content and arterial pressure.
2. The concepts of sodium balance and positive and negative sodium balance.
3. The functions of the major systems that regulate ECF volume, including the RAA,
the sympathetic nervous system, natriuretic hormones, and ADH.
4. How to estimate plasma osmolarity.
5. Regulation of ADH secretion, including osmotic and volume stimuli.
6. The major actions of ADH.

Separate (but related) systems control the amount of Na+ in the body and the amount of
water in the body. As you will learn, body Na+ content is the major determinant of ECF
volume. On the other hand, body water content is the major determinant of body fluid
osmolarity.

Note: The purpose of this lecture is to introduce the “players” that regulate Na+ and H2O
balance. To introduce the players, we will need to use a few terms that are not yet
familiar. Don’t worry -- these will be explained in detail in subsequent lectures.

I. REGULATION OF NA+ BALANCE (i.e., regulation of ECF volume)

A. Why body Na+ content determines ECF volume. This key idea can
seem strange at first. The reasoning goes like this. (1) Na+ and its
accompanying anions, Cl- and HCO3-, are the major solutes of ECF.
Furthermore, most of the body Na+ is in the ECF. (2) You’ve already
learned that when solute moves, H2O follows. (There are very few
exceptions to this rule, and you will see those few exceptions later in renal
physiology. For now, we will ignore the exceptions.) Therefore, the
amount of Na+ in ECF determines the amount of water in ECF, which is
the ECF volume.

B. Relation between Na+ content, ECF volume, and arterial pressure


(Pa). Recall that plasma volume is part of the ECF volume. Thus, ECF Na+
content determines ECF volume, which determines plasma volume and
blood volume, which determines arterial pressure (Pa). An increase in Na+
content (called positive Na+ balance) causes increased ECF volume, an
increase in blood volume and usually an increase in Pa. A decrease in Na+
content (called negative Na+ balance) causes decreased ECF volume,
blood volume, and Pa.

621
Reg of Body Fluids: Na+ & Water - Dr. Costanzo

C. Na+ balance. In the steady state, Na+ intake equals renal Na+ output plus
losses by extrarenal routes (e.g., skin). Normally, extrarenal Na+ losses are
negligible, although in disease, they can be significant (e.g., diarrhea,
excessive sweating, burn).

1. Normally, the body maintains Na+ balance by renal mechanisms.


That is, Na+ intake will be matched by urinary Na+ output. For
example (see figure below), if a person abruptly increases their
Na+ (and water) intake, initially, Na+ intake is greater than Na+
excretion by the kidneys; the Na+ content of the body increases,
and the person is in positive Na+ balance. During this period of
positive Na+ balance, the extra Na+ was added to the ECF, causing
an increase in ECF volume. There is increased body weight due to
the increased ECF volume. After several days, the kidneys detect
the increased ECF volume and initiate mechanisms that cause
excretion of the extra Na+; during this compensatory period, the
kidneys excrete more Na+ than is being ingested, and the person
loses weight, reflecting the decrease in ECF volume. If renal
compensatory mechanisms are normal, all of the extra Na+
originally ingested will be excreted in the urine, and ECF volume
and body weight return to normal.

Figure 1.

2. The opposite example (not shown in the figure) is a person who


has severe diarrhea for several days. Na+ is lost in diarrhea fluid,
causing a decrease in Na+ content, ECF volume, and body weight.
During this period, Na+ loss from the body is greater than Na+

622
Reg of Body Fluids: Na+ & Water - Dr. Costanzo

intake (negative Na+ balance). After several days, the kidneys


detect the decrease in ECF volume and initiate mechanisms that
decrease Na+ excretion by the kidneys until the person is returned
to Na+ balance; during this compensatory period, Na+ excretion is
less than Na+ intake.

In sum, the body detects changes in Na+ content (by detecting


changes in ECF volume), and then makes appropriate adjustments
in Na+ excretion to bring Na+ content back to normal.

3. A new concept that we will introduce here is effective arterial


blood volume (EABV). EABV cannot be identified anatomically,
but it is the “functional” arterial blood volume, which determines
tissue perfusion. Usually, changes in EABV parallel changes in
ECF volume (since effective arterial blood volume is part of the
blood volume, which is part of the ECF volume). However, there
are important situations (e.g., congestive heart failure) where ECF
volume is drastically increased, but EABV is decreased; the reason
for the dissociation is that the excess ECF volume is located in the
interstitial fluid (edema), not in the circulation. In this case, the
body responds to decreased EABV, causing Na+ retention,
increasing ECF volume further, and worsening the edema – a
dangerous vicious cycle.

D. Mechanisms for regulating ECF volume (i.e., Na+ balance). Four


systems detect changes in ECF volume (or EABV) and adjust Na+
excretion. By adjusting Na+ excretion, they are trying to restore body Na+
content and ECF volume to normal. These systems, as you will see, have
overlapping effects.

1. Renin-angiotensin II-aldosterone system (hereafter, the


“RAA”). The RAA is the major system for regulating body Na+
content and ECF volume. You are familiar with it from
cardiovascular physiology.

623
Reg of Body Fluids: Na+ & Water - Dr. Costanzo

Figure 2. The renin-angiotensin II-aldosterone system. The system is


described in terms of the response to a decrease in Pa. TPR, Total peripheral
resistance.

Briefly, a decrease in Pa causes a decrease in renal perfusion


pressure, which is sensed by renal afferent arterioles. Prorenin is
converted to renin in juxtaglomerular cells. Renin is an enzyme
that catalyzes conversion of angiotensinogen to angiotensin I.
Angiotensin I is converted to angiotensin II, by the action of
angiotensin converting enzyme (ACE), primarily in the lung.
Angiotensin II has several actions, including: increasing
production and secretion of aldosterone in the adrenal cortex,
constriction of arterioles, stimulating Na+ reabsorption in the
proximal tubule, and stimulating thirst. Aldosterone is the major
Na+-regulating hormone; it acts on the principal cells of late distal
tubule and collecting duct to induce the synthesis of Na+ channels,
thereby increasing Na+ reabsorption, ECF volume, blood volume,
and Pa.

624
Reg of Body Fluids: Na+ & Water - Dr. Costanzo

The tables below list the major stimulants and inhibitors of renin,
angiotensin II, and aldosterone secretion.

Renin Secretion
Stimulation Inhibition
Decreased perfusion pressure Increased perfusion pressure
Increased sympathetic Decreased sympathetic
+
Decreased distal delivery of Na Increased distal delivery of Na+
Prostaglandins ANP

Angiotensin II secretion
Stimulation Inhibition
Increased renin ACE inhibitors

Aldosterone secretion
Stimulation Inhibition
Increased angiotensin II Decreased angiotensin II
Decreased Pa (via RAA) Increased Pa (via RAA)
Hyperkalemia Hypokalemia

2. Sympathetic nervous system. You have already learned the major


effects of the sympathetic nervous system on the organ systems. It
also participates in Na+ balance as follows. When there is
decreased body Na+ content, ECF volume, and Pa, the decrease
in Pa is detected by the carotid sinus baroreceptors. This leads to
activation of the sympathetic nervous system (which will attempt
to restore Pa back to normal). In the kidney, sympathetic activation
has two effects: (1) constriction of afferent arterioles (thus
decreasing GFR) and (2) stimulation of Na+ reabsorption in
proximal tubule. You will learn that these two effects cause
decreased Na+ excretion and help restore body Na+ content, ECF
volume, and Pa back toward normal.

The opposite occurs when there is increased body Na+ content,


ECF volume, and Pa. There is inhibition of the sympathetics,
leading to dilation of afferent arterioles (increasing GFR) and
decreased Na+ reabsorption, leading to increased Na+ excretion,

625
Reg of Body Fluids: Na+ & Water - Dr. Costanzo

thus decreasing body Na+ content, ECF volume, and Pa back


toward normal.

3. Natriuretic hormones (e.g., atrial natriuretic peptide, ANP).


ANP is released from the atria in response to stretch. When there
is increased blood volume, there is increased volume in the veins
and the atria and increased atrial pressure, which causes secretion
of ANP; these stretch receptors are called “low-pressure” sensors,
because they respond to changes in blood volume and pressure on
the venous (low pressure) side of the circulation (in contrast to
“high-pressure” sensors for Pa on the arterial side that are used in
the RAA and sympathetic nervous system). ANP then has several
actions on the kidneys that lead to increased Na+ excretion: (1)
dilation of afferent arterioles and constriction of efferent arterioles,
which leads to increased GFR and (2) inhibition of Na+
reabsorption in late distal tubule and collecting ducts. The result,
“natriuresis,” tries to decrease ECF Na+ content, ECF volume, and
blood volume back to normal.

ANP Secretion
Stimulation Inhibition
Increased ECF Volume Decreased ECF volume
Increased atrial pressure Decreased atrial pressure

Brain natriuretic peptide (BNP) is structurally related to ANP.


First discovered in brain, BNP is secreted by the ventricles in
response to increased pressure. Its actions are the same as ANP’s,
but even stronger. Recombinant BNP (nesiritide) has been used
clinically to produce a large increase Na+ excretion in order to
reduce ECF volume (e.g., in heart failure).

Urodilatin is also structurally related to ANP. It is secreted by the


distal nephron and causes a local inhibition of Na+ reabsorption,
leading to increased Na+ excretion. It is secreted by and then acts
in the distal nephron.

4. Pressure natriuresis is the phenomenon whereby an increase in


arterial pressure per se (i.e., independent of hormonal effects)
causes increased Na+ excretion. The mechanisms underlying
pressure natriuresis are not fully understood, but include
“washout” of factors that normally promote Na+ reabsorption;
when “washed out,” Na+ reabsorption is inhibited, and Na+
excretion is increased.

626
Reg of Body Fluids: Na+ & Water - Dr. Costanzo

5. Antidiuretic hormone (ADH) is primarily important for water


balance and regulation of body fluid osmolarity. However, ADH is
also involved in regulating ECF volume and blood volume as
follows. You will learn that one stimulus for secretion of ADH is a
decrease in blood volume (hypovolemia). In turn, the major action
of ADH is to increase water reabsorption. Therefore, decreased
blood volume turns on both the RAA and ADH secretion. The
activated RAA leads to increased Na+ reabsorption and increased
ECF Na+ content. The increased ADH leads to increased water
reabsorption and increased body water. For example, following
hemorrhage, the RAA and ADH systems are co-activated and work
in complimentary fashion to increase ECF volume and blood
volume back toward normal. More on ADH in the next section.

II. REGULATION OF WATER BALANCE (i.e., regulation of body fluid


osmolarity).

Body fluid osmolarity (osmolar concentration) is normally held within a very


tight range, that is approximately 290 mOsm/L. (Recall that, in the steady state,
osmolarity of all body fluid compartments is the same.)

We will digress slightly here so that you can learn how to estimate the value of
plasma osmolarity, as it is done clinically. You learned earlier that plasma
osmolarity is approximately 2 x [Na+], since Na+ (and the associated anions)
represent most of the solute in ECF and plasma. To be more precise, though, we
include glucose and urea in the estimate as follows:

Plasma = 2 x [Na+] + glucose + BUN


osmolarity 18 2.8

where:
plasma osmolarity = total osmolar concentration (in mOsm/L)
Na+ = plasma Na+ concentration (in mmol/L)
glucose = plasma glucose concentration in mg/dL
BUN = blood urea nitrogen concentration in mg/dL

The Na+ is multiplied by 2 because Na+ must be balanced by an equal


concentration of anions (in plasma, these are Cl- and HCO3-). Glucose
concentration in mg/dL is converted to mOsm/L when it is divided by 18. The
BUN in mg/dL is converted to mOsm/L when it is divided by 2.8.

The value of body fluid osmolarity is kept constant by adjusting body water
content, rather than by adjusting body solute content. What does that mean? Note
that the units of osmolarity are mosmol/L. The body keeps osmolarity constant by
adjusting volume in the denominator. Two systems are involved in regulating
water balance: thirst and ADH.

627
Reg of Body Fluids: Na+ & Water - Dr. Costanzo

A. Thirst. When body fluid osmolarity rises by as little as 1%, there is


immediate stimulation of thirst and drinking behavior. Osmoreceptors in the
hypothalamus detect the increase in osmolarity, trigger drinking behavior,
which brings water into the body and decreases body fluid osmolarity back to
normal. Incidentally, these osmoreceptors for thirst are nearby, but distinct
from, the osmoreceptors involved in ADH secretion (see below).

B. ADH (also called vasopressin) is the major regulator of body fluid osmolarity
(by altering water reabsorption in late distal tubule and collecting ducts).

1. Synthesis of ADH. Briefly, ADH is a peptide hormone that is


synthesized in hypothalamic neurons, packaged in vesicles, and
transported down the axons of those neurons to the posterior pituitary,
where it is stored until there is a stimulus for its release.

2. Regulation of ADH secretion. The major, physiologic stimulus for


ADH secretion from the posterior pituitary is an increase in plasma
osmolarity. As little as a 1% increase in plasma osmolarity is sufficient
to trigger ADH secretion. Osmoreceptive neurons in the hypothalamus
(nearby the ADH-synthesizing neurons) are sensitive to changes in
plasma osmolarity. When plasma osmolarity increases, water shifts out
of these neurons, they shrink, and then depolarize; depolarization leads
to action potentials, which project to and activate the ADH-secreting
neurons.

There are other (non-osmotic) stimuli for ADH secretion. The most
important is a decrease in blood volume (hypovolemia). If blood
volume decreases by 10%, ADH secretion is stimulated. Decreased
blood volume is associated with decreased venous blood volume;
decreased venous volume causes decreased venous and atrial pressure,
which is detected by low-pressure sensors. These low-pressure sensors
send information via the vagus to the ADH-secreting hypothalamic
neurons.

Note that the response of ADH to hypovolemia is less sensitive than


the response to increased osmolarity; that is, to stimulate ADH secretion,
it takes a larger decrease in blood volume (10%) than an increase in
osmolarity (1%). However, when evoked, the hypovolemic stimulus is
more powerful and will “override” osmotic stimuli, as shown in the
figure. Thus, hypovolemia (volume contraction) stimulates ADH
secretion, even when plasma osmolarity is lower than normal.
Conversely, hypervolemia (volume expansion) inhibits ADH secretion,
even when plasma osmolarity is higher than normal.

628
Reg of Body Fluids: Na+ & Water - Dr. Costanzo

Figure 3.

Other stimuli for ADH secretion are pain, nausea, angiotensin II,
hypoglycemia, nicotine, and opiates. Other inhibitors of ADH secretion
are ethanol and ANP.

ADH Secretion
Stimulation Inhibition
Increased plasma osmolarity Decreased plasma osmolarity
Hypovolemia (volume contraction)Hypervolemia
Pain ANP
Nausea Ethanol
Angiotensin II
Hypoglycemia
Nicotine
Opiates

3. ADH actions. ADH has two major actions:

a. Increased water permeability in principle cells of late distal


tubule and collecting ducts, via insertion of water channels
(aquaporin 2, AQP2) in the luminal membrane. The receptor
on the principal cells is a V2 receptor, that acts via the
adenylyl cyclase (cyclic AMP) mechanism and utilizes a Gs
protein. The increase in water permeability leads to increased
water reabsorption.

b. Contraction of vascular smooth muscle, or vasoconstriction.


The receptor on blood vessels is a V1 receptor, that acts via
the phospholipase C (IP3/Ca2+) mechanism. Vasoconstriction

629
Reg of Body Fluids: Na+ & Water - Dr. Costanzo

of arterioles leads to increased TPR and Pa.

c. Other actions of ADH include increasing the Na+- K+ - 2Cl-


transporter of thick ascending limb and increasing urea
permeability of inner medullary collecting ducts. These actions
will be discussed in the lectures on concentration of the urine.

III. PRACTICE QUESTIONS

1. Over a two-day period, a man ate 4 g of Na+. During the two-day period,
his total urine volume was 2.5 L, and the urinary Na+ concentration was
100 mg/100 ml. Ignoring extrarenal losses of Na+, is the man in normal,
positive, or negative Na+ balance?

2. A person with hypertension has left renal artery stenosis and an elevated
renin in blood from the left kidney. What change would you expect in the
blood level of the following?

Angiotensin II
Aldosterone
Renin in blood from the right kidney

3. A person with an aldosterone-secreting tumor has hypertension. What


change would you expect in each the following?

Aldosterone level
Renin level
Angiotensin II level
Na+ balance

4. Estimate the plasma osmolarity in a person with a plasma Na+


concentration of 140 mEq/L, plasma glucose concentration of 100 mg/dL,
and BUN of 8 mg/dL.

5. Which of the following would be expected to have increased ADH


secretion? (Indicate all that are correct)

Person with serum osmolarity of 275 mOsm/L


Person with diarrhea for three days that caused a 20% loss of ECF volume
Person who drank 1 liter of beer
Person with severe nausea
Person with volume expansion

630
Reg of Body Fluids: Na+ & Water - Dr. Costanzo

ANSWERS

1. Positive Na+ balance. Intake = 4 g. Urinary excretion = 2.5 L x 100


mg/100 ml = 2500 mg = 2.5 g. Intake > excretion.

2. Angiotensin II – increased
Aldosterone – increased
Renin in blood from the right kidney – decreased (because increased Pa is
sensed by right kidney)

3. Aldosterone level – increased


Renin level – decreased (due to increased Pa)
Angiotensin II level –decreased (due to decreased renin)
Na+ balance –increased (due to increased aldosterone)

4.

Plasma
= 2 x 140 + 100/18 + 8/2.8
osmolarity
= 280 + 5.56 + 2.86
= 288.4 mOsm/L

5. Person with serum osmolarity of 275 mOsm/L – no


Person with diarrhea for three days that caused a 20% loss of ECF volume
– yes
Person who drank 1 liter of beer – no
Person with severe nausea – yes
Person with volume expansion – no

631
Gastrointestinal Motility 1 - Dr. Grider

Gastrointestinal Motility 1: Oral, Esophageal, and Gastric Motility


Jack Grider, Ph.D.

OBJECTIVES:

1. Contrast the two types of electrical activity of the gut.


2. Explain the interaction of the muscle layers.
3. Describe the neural and muscular events that mediate swallowing.
4. Compare the electrical activity in different regions of the stomach and describe
how they lead to different physiological role of each region.
5. Describe the gastric motor activity during filling, digestion and emptying.
6. Explain the role of the duodenum in regulating gastric function.

Suggested Reading: Berne & Levy pp.543-563

I. CHARACTERISTICS AND REGULATION OF GUT SMOOTH MUSCLE

A. General Characteristics

1. Smooth muscle cells are coupled to each other by means of gap


junctions between cells. This allows electrical events to spread rapidly
from cell to cell. These gap junctions also allow second messengers
(e.g. IP3/cAMP) and ions (e.g. Ca2+ ) to spread from cell to cell.
2. Because of this coupling, the contractile unit is not a single muscle cell
but rather it is a sheet of muscle. This sheet that acts as a single
contractile unit is called a functional synctium. This type of smooth
muscle is know as Unitary type of Smooth Muscle.
3. Resting membrane potential is not stable but rather shows spontaneous
oscillations (slow waves). Also absolute level and rate of
depolarization varies in different areas of the gut.
4. Two types of electrical activity can be identified.

a. Slow waves (also called basic electrical rhythm or control


potentials)
b. Spikes or muscle action potentials.

B. Electrical Activity

1. SLOW WAVES: (Figures 1 & 2) Membrane potential is not stable


but rather shows slow cycles of depolarization and repolarization.

632
Gastrointestinal Motility 1 - Dr. Grider

These cycles differ in speed, amplitude, frequency, and duration in


different regions of the gut. The wave consists of a rapid upstroke
(depolarization), a partial repolarization, a sustained plateau lasting
several seconds, and a complete repolarization to resting level. The
slow wave spreads electrically in the longitudinal and circular
directions as well as through the thickness of the muscle. All gut
muscle cells appear to be capable of generating slow waves however,
usually cells are coupled via gap junctions to pacemaker cells called
Interstitial Cells of Cajal (ICC). These are specialized cells whose
membrane potential oscillates at a higher frequency than the intrinsic
frequency of smooth muscle cells. There are many subtypes of ICC
that perform other actions such as acting as intermediates between
nerves and muscle. Pacemaker ICC are located at the myenteric level
where circular and longitudinal muscle intersect and at submucosal
level where circular muscle and submucosa intersect. Intramusclar
ICC are located throughout the thickness of the circular muscle and
have gap junction contacts with muscle cells and neurons.

Figure 1.

633
Gastrointestinal Motility 1 - Dr. Grider

Figure 2.

a. Slow waves are the result of influx of Ca2+ , and possibly other
ions, causing initial depolarization. The Ca2+ current slowly
inactivates and is balanced by an outward potassium current.
Eventually the potassium current dominates leading to
repolarization.
b. The amplitude and duration of plateau phase are determined by
the magnitude of the Ca2+ influx. Hormones and transmitters
can modify the plateau. Excitatory agents (e.g. acetylcholine,
substance P, CCK) increase the duration and amplitude, of the
plateau potential . If the plateau potential exceeds the
threshold level for voltage-activated calcium channel opening,
then more Ca2+ enters and contraction occurs (Figure 3);
inhibitory agents (e.g. vasoactive intestinal peptide) lower
amplitude and duration of the plateau phase thereby decreasing
the likelihood of contraction.

634
Gastrointestinal Motility 1 - Dr. Grider

Figure 3.

2. SPIKES (Action Potentials): These occur when membrane potential


is above threshold for opening voltage-dependent Ca2+ channels on
smooth muscle membrane and is usually seen in intestinal muscle.
Opening of these channels results in rapid and complete depolarization
of the cell. The rise in intracellular Ca2+ is rapid but voltage sensitive
Ca2+ channels rapidly inactivate as K channels open. Inactivation of
inward calcium currents and activation of outward potassium currents
lead to reversal of membrane potential (repolarization). The spike
potential are always accompanied by but are not necessary for
contraction. The amplitude of contraction is determined by the number
of spikes superimposed on the slow waves. (Figure 4)

Figure 4.

II. INNERVATION AND MOTILITY PATTERNS

635
Gastrointestinal Motility 1 - Dr. Grider

A. Innervation of muscle layers

1. The thinner longitudinal smooth muscle contains few nerve fibers.


This muscle is contracted primarily by acetylcholine which diffuses
into the longitudinal muscle layer after being released from neurons at
the junction of the myenteric plexus and the longitudinal muscle. This
layer also contains a very sparse innervation by inhibitory nerve fibers
so that relaxation is mainly due to inhibition of the release of the
excitatory transmitter, acetylcholine.
2. The thicker circular muscle layer contains many neurons of a variety
of types. The neurons do not make true synapses with muscle cells but
rather release transmitter from varicosities along the length of the
axon. The excitation or inhibition then spreads throughout the sheet of
muscle via gap junctions. The predominate innervation is by relaxant
(VIP and nitric oxide synthase-containing) nerves.
3. There is recent evidence that in some regions and species, the
interstitial cells of Cajal are also innervated by neurons of the
myenteric plexus. In this manner the contractile activity of the muscle
and electrical potential of the cell can be coordinated.

B. Coupling between muscle layers

1. In general, the circular and longitudinal muscle contractions are out of


phase. That is, the circular muscle layer is inhibited (relaxes) when the
longitudinal muscle layer contracts and visa versa. This prevents the
two layers from contracting at the same time and is the result of
reciprocal innervation of the two layers by inhibitory neurons of the
myenteric plexus.
2. Contraction of the longitudinal layer coincident with relaxation of the
circular muscle shortens and widens the gut. This has the effect of
creating a segment that is relaxed and easy to push material into. In
contrast, contraction of the circular layer and relaxation of the
longitudinal layer decreases the radius and lengthens the gut. This
results in an increase in the intra luminal pressure which is the driving
force to move material through the gut.
3. These two events occur in a coordinated manner as controlled by the
enteric nervous system.
4. Stimulation of the gut by chemical components of chyme, mechanical
movement of mucosa, or by distension results in contraction of the
circular muscle and relaxation of longitudinal muscle above (orad) the
site of distension. This increases intraluminal pressure and tends to
push the material anally. At the same time, the circular muscle below
(caudad) the site relaxes and the longitudinal layer contracts providing
a relaxed region into which the material can be easily propelled. The
contraction of longitudinal muscle pulls the gut up and over the
material also aiding in the anal propulsion. (Figure 5). The coordinated

636
Gastrointestinal Motility 1 - Dr. Grider

response orad and caudad to a site of stimulation is known as the Law


of the Intestine. This basic law defines movement through out the gut,
and is an example of a Short Arc Reflex called the Peristaltic reflex.

Figure 5.

5. Contraction of circular muscle above the site of distension is mediated


by cholinergic neurons and tachykinin containing (substance P)
neurons. Relaxation below the site of distension is mediated by
VIP/Nitric oxide synthase-containing neurons.

C. Patterns of motility (based on above law and wholly mediated through


neuronal circuits of myenteric plexus)

1. Tonic or sustained contraction: this type of contraction is seen in


sphincters, proximal stomach, and gallbladder. Muscle in one area
remains contracted over time. Neural innervation is usually inhibitory
and serves to relax the muscle.
2. Segmental contraction: mixing contractions in local area which are
not propagated (Figure 6). Common in small intestine and colon.
3. Peristaltic contractions: propulsive, propagated contractions which
move lumenal contents towards the anus (Figure 6). Basically, this is
a moving ring of contraction preceded by a moving ring of relaxation.
This is found throughout the gut (except proximal stomach) in regions
concerned with movement of chyme.

637
Gastrointestinal Motility 1 - Dr. Grider

Figure 6.

III. SWALLOWING AND ESOPHAGEAL MOTILITY

A. Chewing

1. Can be voluntary initially but becomes reflexive.


2. Begins to break down food mechanically and mixes food with salivary
secretions.

B. Swallowing: Initial events

Swallowing is initiated voluntarily, but then proceeds reflexively.


The initial stimulus is mediated by activation of sensory nerve endings in
the pharynx that respond to touch. These sensory nerves then activate the
swallowing center located in the lower pons and medulla of the brain
stem. Once activated, the swallow begins by activation of cranial nerves
followed by activation of the vagal nerve to the esophagus.

The swallow can be divided into three phases.

1. Oral or voluntary phase

a. The tongue separates a portion of the food in the mouth and


isolates it by elevating against the hard palate. The tongue then
pushes the bolus into the pharynx

2. Pharyngeal phase

a. Stereotyped, reflexive sequence of events.


b. Soft palate elevates to seal off nasopharynx.
c. Epiglottis closes over the larynx to close opening to
trachea.(respiration is inhibited).

638
Gastrointestinal Motility 1 - Dr. Grider

d. The upper esophageal sphincter relaxes and the pharynx


contracts to propel the bolus into the esophagus ( following the
law of the intestine).
e. The upper esophageal sphincter contracts behind the bolus
(Figure 7).

Figure 7.

3. Esophageal phase (Figure 8 )

a. Bolus distends the wall of the esophagus which initiates a


peristaltic contraction orad and relaxation caudad, as described
above.
b. Peristaltic contraction-relaxation complex which follows
pharyngeal and oral phase is called primary peristalsis. Often
residual food or a pill caught in the esophagus distends the
esophagus and initiates a peristaltic contraction not preceded
by the oral and pharyngeal phases. This is called secondary
peristalsis.
c. The afferent and efferent neurons mediating peristalsis in the
esophagus are vagal fibers. The afferents project to the brain
stem nucleus ambiguus and the vagal motor complex (Nucleus
of the tractus solitarius and the Dorsal motor nucleus). The
efferent fibers to the striated part of the esophagus directly
innervate and contract the skeletal muscle via release of
acetylcholine. The efferent fibers to the smooth muscle portion

639
Gastrointestinal Motility 1 - Dr. Grider

innervate enteric cholinergic neurons that cause contraction


and VIP/nitric oxide (NO) neurons that cause relaxation (Fig.8)

Figure 8.

4. Lower esophageal sphincter (LES)

a. Sphincter at junction of esophagus and stomach relaxes as


bolus approaches.
b. This relaxation is mediated by the release of vasoactive
intestinal peptide and nitric oxide from myenteric neurons.
c. Relaxation is followed by contraction (rebound) as bolus
passes into stomach. Thus the LES follows the same
stereotyped sequence described for peristalsis. (Figure 8).

IV. GASTRIC MOTILITY

A. Functional anatomy

640
Gastrointestinal Motility 1 - Dr. Grider

1. The stomach may be divided into sections based on anatomy and


function (Figure 9). The major functions of stomach are:

a. act as a reservoir,
b. grind and mix food with secretions,
c. regulate the delivery of nutrients to small intestine.

2. The upper (Proximal) portions are the fundus and orad corpus.
These regions are tonically contracted at rest due to a low resting
membrane potential (-48mv) that allows Ca2+ channels to remain open.
They have no slow waves or myoelectrical rhythm (Figure 10). This
region of the stomach is non-propulsive but rather serves as the major
storage site for a meal.
3. The corpus (mid and caudad) serves as both storage and mixing sites.
The muscle is thicker than that of the fundus and demonstrates slow
waves (Figure10). The meal is slowly mixed with gastric secretions by
contractions in this area over several hours.

Figure 9.

641
Gastrointestinal Motility 1 - Dr. Grider

Figure 10.

4. The antrum (distal stomach) has a very thick circular muscle layer
and demonstrates slow waves with superimposed action potentials or
spikes. (Figure 10). Contractions in this area are very strong and serve
to grind solid particles and to propel material into the small intestine.
Often referred to as the antral pump. The resting membrane potential
here is about -75 mv at its nadir.
5. Final area at junction of antrum and duodenum is the pylorus or
pyloric sphincter. This is tonically contrqcted and its activity is often
coordinated with that of the antrum.

642
Gastrointestinal Motility 1 - Dr. Grider

B. Reservoir Function

1. The initial motility of the stomach can be divided into 3 phases as


indicated in Figure 11. These phases are the result of motility effects in
the proximal (fundus and orad corpus) portions of the stomach

Figure 11.

2. Phase I. At rest before a meal begins, the stomach is flaccid and


empty because of the dominant inhibitory tone of the enteric nervous
system and because there are no stimuli to induce motility.
3. Phase II. As a meal begins, swallowing delivers food from the oral
cavity as a bolus. As the bolus approaches the Lower Esophageal
Sphincter, the proximal stomach relaxes to allow the bolus to pass into
the stomach and to allow the stomach to expand without a significant
increase in pressure. This is called Receptive Relaxation (Figure 12
& 13). This is mediated by a vago-vagal long arc reflex initiated by
vagal afferents fibers in the esophagus and vagal efferent fibers that
innervate inhibitory motor neurons of the enteric nervous system. The
latter release vasoactive intestinal peptide and nitric oxide causing
relaxation of the proximal stomach.
4. Phase III. As the meal progresses, the increased delivery of material
distends the wall of the stomach more, activating both vagal afferent
fibers and local intrinsic primary afferent neurons. These initiate
additional vago-vagal long arc and short arc reflexes that activate the
same inhibitory motor neurons leading to greater relaxation of the
proximal stomach. This is called Adaptive Relaxation. (Figure 12 &
13). This also prevents the distension of the stomach from causing an
increase in pressure as evident from the rise in pressure following
vagotomy (Figure 13).

643
Gastrointestinal Motility 1 - Dr. Grider

Figure 12.

Figure 13.

644
Gastrointestinal Motility 1 - Dr. Grider

C. Gastric Motility

1. Pacemaker cells (Interstital Cells of Cajal) located in the corpus


along the greater curvature initiate slow waves at a rate of 3/min.
These induced a moving Peristaltic or Propagating Contraction
wave that moves distally towards to the antro-pyloric region with
increasing strength and speed.
2. Movement of luminal contents depends on four events or forces
(FIGURE 14)

a. Propagating Peristaltic contraction


b. Gastric tone
c. Gastric emptying
d. Retropulsion

3. Retropulsion is the process of pushing larger particles back into the


proximal stomach as the propagating contraction pushes liquid and
smaller (few mm) particles distally.

Figure 14.

D. Gastric Emptying (FIGURE 15)

1. This requires the coordination of actions of all parts of stomach:


Corpus, antrum, and pylous as well as duodenal bulb and duodenum.
2. As the peristaltic wave pushed material distally, the pylorus/pyloric
sphincter is closed (Phase of propulsion). The peristaltic wave enters
the antral region where it becomes forceful and rapid. The contraction
crushes large particles and sends some liquid and small particles
through the pylous, which is beginning to open. As the peristaltic wave
reaches the pylous, it opens wide allowing chyme to flow into the
duodenum (Phase of Emptying). About 10 ml passes with each
peristaltic wave. The pylorus then closes (as does the duodenum)
preventing the movement of chyme into the duodenum. At this point,

645
Gastrointestinal Motility 1 - Dr. Grider

the antropyloric contraction further crushes material and propels


material back into the corpus (Phase of retropropulsion).
3. The process also acts as a sieve to insure that only small particles enter
the small intestine.

Figure 15.

4. There is a difference in the rate of gastric emptying of liquids and


solids (Figure 16). Since liquids empty without the need to be reduced
in size by antral grinding, an isosmotic non-nutrient fluid meal begins
to empty immediately. The rate of emptying is proportional to the
square root of the volume remaining in the stomach. The rate of
emptying of a solid, isosmotic, non-nutrient meal, demonstrates a
delay or lag time for grinding of solid to a size that will pass through
the pylorus. Emptying then proceeds as described for liquids. Nutrient
meals have additional controls as outlined below.

646
Gastrointestinal Motility 1 - Dr. Grider

Figure 16.

E. Regulation of Gastric Emptying of Chyme

1. Gastric emptying rate is influenced by both neural and hormonal


mechanisms.
2. Mechanoreceptors, activated by distension, are present in the wall of
the stomach and duodenum.
3. Distension of the stomach accelerates emptying whereas distension of
the duodenum slows emptying of the stomach.
4. Distension-induced effects are most likely neurally-mediated reflexes
involving both enteric neurons (short arc reflex) and vagal afferent
(sensory) and efferent (motor) neurons. The latter is called a
vagovagal reflex and is an example of a long arc reflex.
5. Chemoreceptors in mucosa of upper small intestine (esp. duodenum)
are sensitive to components of the chyme and activate neural reflexes
which slow gastric emptying.
6. Components of chyme can also slow gastric emptying as a result of the
release of gut hormones from mucosal (and pancreatic)
enteroendocrine cells.

F. Components of Chyme which Slow Gastric Emptying

1. Acid (esp. HCI)-release of secretin.


2. Lipid digestion products (e.g. fatty acids)-release of cholecystokinin
and GIP (and possibly Pancreatic Polypeptide (PP) from pancreatic
islets.
3. Protein digestion products (e.g. amino acids) -release of CCK and
gastrin.

647
Gastrointestinal Motility 1 - Dr. Grider

4. Osmolarity-both hypo- and hyperosmolar solutions slow gastric


emptying. Digestion of starch (i.e., carbohydrate) cause rapid rise in
osmolarity of chyme in duodenum.
5. Lipid digestion products are most effective at slowing gastric
emptying. This may be related to higher caloric content of fat vs
protein and carbohydrate. High caloric meals empty slower than low
caloric meals.
6. Recent evidence suggests a novel cite of action of CCK on sensory
mechanoreceptors in wall of stomach. CCK activates a vagovagal
reflex resulting in release of VIP from enteric neurons in the gastric
fundus. VIP causes relaxation of fundus (contractile cholinergic
neurons inhibited at same time) which leads to a reduction in gastric
emptying.

G. Mechanisms of Slowing of Gastric Emptying.

1. The slowing of gastric emptying by components of chyme is mediate


by changes in gastric tone (decreased tone), antral contraction
(decrease), pyloric opening time (decreased) and duodenal activity
(decreased peristalsis and increased non-propulsive contractions).
(Figure 17)

Figure 17.

648
Gastrointestinal Motility 1 - Dr. Grider

V. STUDY QUESTIONS

1. Electrical slow waves

A. Occur at the same frequency through out the gut.


B. Cause an increase in the influx of sodium ions.
C. Normally originate in the Interstital Cell of Cajal.
D. Are present in unitary and multi-unit smooth muscle.
E. Cause the release of second messengers within muscle cells.

2. Vasoactive intestinal peptide

A. Is a hormone that causes muscle contraction.


B. Causes contraction of sphincteric muscle.
C. Mediates relaxation caudad (anal) to a distension.
D. Is structurally related to gastrin and cholecystokinin.
E. Increases the number of spikes on intestinal slow waves.

3. Secondary peristalsis of the esophagus

A. Is a hormonally mediated wave of contraction.


B. Is a peristaltic wave that moves from lower to upper esophageal
sphincter.
C. Begins in the pharynx instead of the oral cavity.
D. Does not require relaxation of the lower esophageal sphincter.
E. Occurs independent of primary esophageal peristalsis.

4. The gastric slow wave with the longest duration occurs in the

A. Fundus.
B. Orad Corpus.
C. Middle Corpus.
D. Orad antrum
E. Caudad antrum

Answers : 1=C, 2=C, 3=E, 4=E

649
Clearance, RBF and GFR - Dr. Costanzo

Clearance, RBF, and GFR


Linda Costanzo, Ph.D.

OBJECTIVES:

After studying this lecture, the student should understand:

1. The concepts of renal clearance and clearance ratio.


2. How RBF is autoregulated, including the myogenic mechanism and
tubuloglomerular feedback.
3. How RPF is measured with PAH.
4. The difference between true RPF and effective RPF.
5. How to calculate RBF.
6. Characteristics of the glomerular capillary barrier and the factors that restrict
filtration.
7. How Starling forces across the glomerular capillary determine GFR.
8. How changes in Starling forces alter GFR.
9. Effects of vasoactive substances on afferent and efferent arterioles and their
expected effects on RPF and GFR.
10. How GFR is measured with clearance of inulin.
11. How filtration fraction is calculated.

I. INTRODUCTION TO RENAL PHYSIOLOGY, THE NEPHRON, AND ITS


BLOOD SUPPLY

Figure 1. Sagittal and coronal sections of the kidney.

650
Clearance, RBF and GFR - Dr. Costanzo

Figure 2. Segments of a superficial and a juxtamedullary nephron.

Note the following segments of the nephron: proximal convoluted tubule,


descending limb and thick ascending limb of loop of Henle, distal convoluted
tubule, and collecting ducts. Superficial nephrons have their glomeruli in the
cortex, and juxtamedullary nephrons have their glomeruli near the junction of
cortex and medulla. Juxtamedullary nephrons have much longer loops of Henle.

The juxtaglomerular apparatus is a specialized region with three major


components: (1) juxtaglomerular cells surround the afferent arterioles and

651
Clearance, RBF and GFR - Dr. Costanzo

secrete renin; (2) macula densa is a specialized region of the early distal tubule
that comes in close contact with its own glomerulus; and (3) mesangial cells line
the glomerulus, and afferent and efferent arterioles. There are two important
functions of the juxtaglomerular apparatus: secretion of renin by juxtaglomerular
cells (discussed in the previous lecture) and tubuloglomerular feedback, which
will be discussed in this lecture.

The blood supply to the kidney consists of the following components listed (in
the direction of blood flow): renal artery, progressively smaller arteries, afferent
arterioles, glomerular capillaries site of glomerular filtration), efferent arterioles,
peritubular capillaries (surround nephrons, provide nutrient flow and site of
reabsorption and secretion), small veins, and renal vein.

The basics of urine production include: (1) glomerular filtration of 180 L/day,
which produces an ultrafiltrate of plasma; (2) modification of this ultrafiltrate by
subsequent reabsorption and secretion to produce the final urine of 1-1.5 L/day.

II. RENAL CLEARANCE

A. Measurement of renal clearance

Cx = Ux V/Px

where Cx is clearance of any substance, Ux is urinary concentration of the


substance, V is urine flow rate (volume per unit time), and Px is plasma
concentration of the substance.

Clearance of x is the volume of plasma cleared of substance x per unit


time. Thus, units of clearance are volume/time (e.g., ml/min or L/min or
L/hour or L/day).

Special note: the numerator of the clearance equation (Ux V) is equal to


excretion rate (see next lecture).

B. Clearances of various substances

Substances with the highest clearances are filtered and secreted.


Substances with the lowest clearances are filtered and reabsorbed.
Substances that are filtered only (inulin) are glomerular markers and have
a clearance equal to the glomerular filtration rate.

C. Clearance ratio

Clearance ratio = Cx/Cinulin

Clearance ratio has no units and expresses the clearance of substance X

652
Clearance, RBF and GFR - Dr. Costanzo

relative to the clearance of inulin (a glomerular marker). Thus, the


clearance ratio gives information about the renal handling of substance X.
A clearance ratio of 1.0 means that substance X is also a glomerular
marker. If the clearance ratio is < 1.0, the substance is either not freely
filtered, or is filtered and subsequently reabsorbed. If the clearance ratio is
> 1.0, the substance is both filtered and secreted. For future reference,
clearance ratio is also called fractional excretion (fraction of the filtered
load excreted in urine); this statement will make sense only after we’ve
covered reabsorption and secretion.

III. RENAL BLOOD FLOW

A. Autoregulation of RBF

Figure 3. Autoregulation of renal blood flow and glomerular filtration


rate. Pa, Renal artery pressure.

Renal blood flow (and GFR) are kept constant over a wide range of arterial
pressures by changing the resistance of the afferent arterioles, a phenomenon
called autoregulation. Two mechanisms explain autoregulation:

1. Myogenic hypothesis is familiar from cardiovascular physiology. It states


that when arterioles are stretched (i.e., by an increase in Pa), they contract,
leading to an increased resistance. The increased resistance prevents an
increase in blood flow that would have otherwise occurred in response to
the increased Pa. In the kidney, it is the afferent arterioles that are

653
Clearance, RBF and GFR - Dr. Costanzo

stretched, and then contract, in response to increased Pa. Thus, an increase


in Pa results in constriction of afferent arterioles, which causes an increase
in afferent arteriolar resistance that decreases RBF (and GFR) back to
normal (i.e., keeps it constant).

2. Tubuloglomerular feedback

Figure 4. Mechanism of tubuloglomerular feedback. GFR, Glomerular filtration


rate; RBF, renal blood flow.

Tubuloglomerular feedback in an autoregulatory mechanism unique to the


kidney. When Pa increases, both RBF and GFR transiently increase. The
increase in GFR leads to increased delivery of solutes and water to the
macula densa of the distal tubule. The macula densa senses a component
of the increased delivered load (either luminal Na+ or Cl- concentration)
and, in response, secretes a vasoactive substance (probably adenosine, not
renin) that causes local constriction of nearby afferent arterioles. This
vasoconstriction then decreases RBF and GFR back to normal, i.e.,
autoregulation.

654
Clearance, RBF and GFR - Dr. Costanzo

B. Measurement of renal plasma flow (RPF) and renal blood flow (RBF). Renal
plasma flow (RPF) is measured with a specific marker substance. Renal blood
flow (RBF) is then calculated from the measured RPF (by using hematocrit).

1. True renal plasma flow. Renal plasma flow (RPF) is measured by


applying the Fick principle of conservation of mass. The amount of a
substance entering the kidney (via the renal artery) must be equal to the
amount leaving the kidney via the renal vein + ureter (assuming that
substance is not synthesized or metabolized by the kidney). The substance
that is used to measure RPF is an organic acid, para-aminohippuric acid
(PAH). PAH is both filtered and secreted by the kidney, thus it has a very
high renal clearance; as you will appreciate, its high renal clearance makes
it an appropriate marker for measuring RPF. (Incidentally, a constant PAH
concentration in blood is maintained by an infusion that replaces what is
excreted in urine.)

Figure 5. Measurement of renal plasma flow by the Fick principle.


PAH, Para-aminohippuric acid; [RA], concentration in renal artery;
[RV], concentration in renal vein; [U], concentration in urine; RPF,
renal plasma flow.

Amount of PAH entering the kidney = RPF x PAH concentration in renal


artery.

655
Clearance, RBF and GFR - Dr. Costanzo

Amount leaving the kidney = RPF x PAH concentration in renal vein +


urine flow rate (V) x PAH concentration in urine.

Applying the basic premise that PAH in = PAH out, rearranging, and
solving for RPF:

“True” RPF = [U]PAH x V


[RA]PAH - [RV]PAH

2. Effective renal plasma flow (clearance of PAH). Two simplifications


can be applied to the equation for RPF above to make it “user-friendly” in
humans. (1) Because PAH is both filtered and secreted, it has an extremely
high renal clearance. Thus, in one pass of blood through the kidney,
almost all PAH that entered in renal arterial blood is excreted in the urine,
leaving very little in the renal vein. If we assume that there is no PAH in
the renal vein, then we don’t need to measure PAH in the renal vein. (2)
We also don’t need to measure PAH in the renal artery because the PAH
concentration in renal artery is identical to PAH concentration in systemic
venous blood (except for renal venous blood); therefore, we can simply
measure PAH in venous plasma, which is called PPAH. By applying these
two simplications, we arrive at:

“Effective” RPF = [U]PAH x V = CPAH


[P]PAH

3. Notice that effective RPF is also the clearance of PAH! “Effective” RPF
underestimates “true” RPF by about 10% because about 10% of RPF
supplies portions of the kidney (e.g., adipose) that have nothing to do with
filtration and secretion of PAH; PAH in that small portion of RPF is not
excreted in urine, and ends up in the renal vein. In other words, renal vein
PAH is not exactly zero (as we had assumed), but it is nearly zero. As a
student, you are naturally wondering: when do I calculate effective RPF vs
true RPF. If you are given all the values needed for the true RPF equation,
then use them. Otherwise, calculate effective RPF.

4. Calculating renal blood flow

RBF = RPF
1 - Hct

Hematocrit is the fractional blood volume occupied by blood cells. Thus,


1-Hct is the fractional blood volume occupied by plasma.

656
Clearance, RBF and GFR - Dr. Costanzo

5. Example

If renal artery PAH concentration is 1.1 mg/ml, renal vein PAH


concentration is 0.1 mg/ml, urine PAH concentration is 650 mg/ml, urine
flow rate is 1 ml/min, and hematocrit is 0.45, what is the true renal blood
flow? (Answer = 1182 ml/min)

6. What’s the relationship between RPF and RBF? (To help you picture
“things.”)

Renal plasma flow (RPF) is to renal blood flow (RBF) as plasma is to


whole blood. Simple as that. Plasma is the fluid part of whole blood;
plasma is 93% water (called plasma water) and 7% plasma proteins. The
rest of whole blood is the cells. Fractional volume occupied by RBCs =
Hct; fractional volume occupied by plasma = 1 - Hct.

Why would we want to know renal plasma flow? After all, it's whole
blood that flows into the renal artery, not just plasma. The reason we want
to know RPF is that's the parameter we can measure with PAH. PAH (the
marker for RPF) is dissolved only in plasma, not in RBCs. So....we
measure RPF with PAH, and we calculate the RBF from the RPF, by
knowing the hematocrit. If we could put a flowmeter on the renal artery,
we could measure RBF directly......but we can't, so we use the PAH
method.

IV. GLOMERULAR FILTRATION

Glomerular filtration is the first step in urine formation. Substances in glomerular


capillary blood are filtered across the glomerular capillaries into Bowman’s space
(the first part of the proximal tubule). This ultrafiltrate contains water and the
small solutes of plasma, but, because of size and charge restrictions of the
glomerular barrier, no protein or blood cells.

657
Clearance, RBF and GFR - Dr. Costanzo

A. Glomerular capillary barrier consists of three layers: an endothelial cell


layer of 70-100 angstroms, which has filtration pores; a basement
membrane; an epithelial cell layer with podocytes that attach to the
basement membrane by foot processes, interspersed with narrow filtration
slits. There are fixed negative charges on the filtration slits and the
basement membrane that impede the filtration of large negatively charged
solutes like plasma proteins.

Figure 6. Structure of the glomerular capillary wall.

Sieving coefficient expresses the ability of a particular solute to be filtered


across the glomerular capillary barrier. A sieving coefficient of 1.0 means
that there is no restriction, i.e., the substance is freely filtered. A sieving
coefficient < 1.0 means the substance does not freely cross the barrier, i.e.,
is restricted; the lower the coefficient, the more restricted the crossing.

Substance Molecular weight (daltons) Sieving coefficient


Na+ 23 1.0
K+ 39 1.0
Cl- 36 1.0
Water 18 1.0
Urea 60 1.0
Glucose 180 1.0
Inulin 5,200 1.0
Myoglobin 16,900 0.75
Hemoglobin 68,000 0.03
Serum albumin 69,000 < 0.01

Molecular weight (and molecular radius) is a major factor that determines


filterability. Substances with molecular weight <5500 are freely filtered
across glomerular capillaries. For example, even inulin (a large fructose

658
Clearance, RBF and GFR - Dr. Costanzo

polymer) with molecular weight of 5200 makes the size “cut-off” and is
freely filtered. Above 5500, with increasing molecular weight, filtration is
more and more restricted; for example, myoglobin has limited filtration,
and normally there are only traces of hemoglobin and albumin in the
glomerular filtrate. The other factor that determines filterability is charge
on the solute. Because the glomerular barrier is lined with fixed negative
charges, solutes that are also negatively charged, such as albumin, are
further restricted from filtration. Importantly, some glomerular diseases
are characterized by loss of the fixed negative charge on the glomerular
barrier, allowing albumin to be filtered and thus present in the urine.

B. Starling forces determine GFR

GFR = Kf x net ultrafiltration pressure


= Kf x [(PGC - PBS) - πGC]

Starling forces (pressures) across glomerular capillaries are analogous to


Starling forces in other capillaries. The nomenclature is changed slightly
as follows. Instead of Pc for capillary hydrostatic pressure, we use PGC.
Instead of πc for capillary colloidosmotic pressure, we use πGC. The fluid
in Bowman’s space is analogous to interstitial fluid. Finally, there are
three, rather than four, Starling forces in glomerular capillaries – assuming
no protein is filtered, colloidosmotic pressure in Bowman’s space is zero
and can be ignored.

The equation above states that the sum (net) of the Starling forces
determines the net ultrafiltation pressure, which is the driving force.
Net ultrafiltration pressure, multiplied by the water permeability (Kf) of
glomerular capillaries is the GFR. In glomerular capillaries, the net
ultrafiltration pressure always favors filtration (never absorption).

659
Clearance, RBF and GFR - Dr. Costanzo

Figure 7. Starling forces across the glomerular capillaries. A, Net filtration; B,


filtration equilibrium. Arrows show the direction of the Starling pressures; numbers
are the magnitude of the pressure (mm Hg); + signs show pressures favoring
filtration; - signs show pressures opposing filtration. PGC, Hydrostatic pressure in
the glomerular capillary; PBS, hydrostatic pressure in Bowman's space; π GC,
oncotic pressure in the glomerular capillary.

Panel A is a snapshot of the Starling pressures as blood has just entered


the glomerular capillary from the afferent arteriole (the pressures are
‘spread out’ across the capillary diagram for readability). At this point, the
net ultrafiltration pressure is +16 mm Hg, favoring filtration.

Panel B is a shapshot of the pressures as the blood is about to leave the


glomerular capillary via the efferent arteriole (again, spread out for
readability). What happens along the capillary, i.e., between A and B?
When blood enters the capillary, the driving force strongly favors

660
Clearance, RBF and GFR - Dr. Costanzo

filtration. Filtration occurs, fluid and small solutes (but not protein or
blood cells) are lost from the capillary, causing the protein concentration
and oncotic pressure (πgc) of the glomerular capillary blood to increase.
The increase in πgc opposes filtration and eventually, there is no net
ultrafiltration pressure, no driving force, and glomerular filtration stops
(filtration equilibrium). Note that blood leaving the glomerular capillary
has a high πc which will be an important fact when we discuss proximal
tubule reabsorption.

C. Changes in Starling forces change GFR.

Figure 8.

1. Constriction of the afferent arteriole

2. Constriction of the efferent arteriole

661
Clearance, RBF and GFR - Dr. Costanzo

3. Increasing plasma protein concentration

4. Constriction of the ureter

5. Simultaneous constriction of afferent and efferent arterioles

Figure 9.

Some substances have effects on both afferent and efferent arterioles.


Their overall effect on GFR depends on which effect is stronger (i.e.,
whether PGC increases or decreases). For example, angiotensin II
constricts both afferent and efferent arterioles, but preferentially constricts
efferents. At low concentrations, angiotensin II constricts efferent
arterioles more than afferents; thus, RPF decreases, but PGC and GFR
increase (or are maintained). This preferential effect of angiotensin II is

662
Clearance, RBF and GFR - Dr. Costanzo

important clinically, as it “protects” GFR during high vasoconstrictor


states (e.g., response to hemorrhage). ACE inhibitors are used to treat
hypertension because they dilate arterioles; in the kidney, they
preferentially dilate efferent arterioles, thus increasing RPF, but
decreasing GFR (potentially dangerous, therefore, in renal failure).

Summary of effects of vasoactive substances on renal arterioles

• Sympathetic nervous system ( α1 receptors)

Vasoconstriction of afferent and efferent


More α1 receptors on afferent
Decreases RBF
Decreases GFR

• Angiotensin II

Vasoconstriction of afferent and efferent


Efferent more sensitive than afferent
Decreases RBF
Increases or maintains GFR

• ANP

Vasodilation of afferent
Vasoconstriction of efferent
Increases RBF
Increases GFR

• NO

Vasodilation of afferent and efferent


Increases RBF

• Prostaglandins

Vasodilation of afferent and efferent


Released locally (in kidney) in high vasoconstrictor states
Modulate, or offset, vasoconstriction by sympathetic and Angiotensin
II
Protect RBF and GFR

• Dopamine (low dose)

Vasodilation of afferent and efferent

663
Clearance, RBF and GFR - Dr. Costanzo

D. Measurement of GFR (clearance of inulin)

GFR= [U]inulin x V
[P]inulin

Inulin is a "glomerular marker," and its clearance measures GFR. Being a


glomerular marker means that inulin is filtered, but neither reabsorbed nor
secreted. Inulin is not an endogenous substance, thus it must be
administered to measure GFR. Creatinine, however, is endogenous, and it
is a near-perfect glomerular marker; therefore, creatinine clearance can
also be used to measure GFR.

Serum creatinine concentration and blood urea nitrogen (BUN) can be


used to estimate changes in glomerular filtration rate. Both creatinine and
urea are normally filtered and then excreted. If GFR decreases, there is
less filtration and less excretion of creatinine and urea, and their blood
concentrations increase.

E. Why is the clearance of PAH the RPF (effective RPF), while the
clearance of inulin is the GFR?

First, different substances have different renal clearances based on how


the substance is handled in the kidney. A substance that is filtered only
will have a mid-range clearance and be called a glomerular marker; a
substance that is filtered and reabsorbed will have much lower clearance; a
substance that is filtered and secreted will have a much higher clearance.
Clearance is ml/min of plasma cleared, or ridded, of that substance.

To illustrate the difference between what is measured by the clearances of


PAH and inulin, let's put some PAH and inulin into the plasma and have
that plasma flow into the renal artery and then into the afferent arterioles.
20% of that RPF (containing inulin and PAH) is filtered across glomerular
capillaries; the inulin and PAH in that 20% of the RPF will be excreted,
i.e., that portion of the RPF is cleared of its inulin and its PAH. The
remainder (80%) of the RPF that is not filtered flows into the efferent
arterioles and then into peritubular capillaries. The PAH (but not the
inulin) in that portion of RPF is secreted and then excreted; thus that
portion of the RPF is cleared of its PAH, but not of its inulin.

Bottom lines. The entire (effective) RPF is cleared of its PAH by a


combination of filtration and secretion; that is why the renal vein PAH is
nearly zero. Only 20% of the RPF is cleared of its inulin by filtration; that
20% is the GFR; the renal vein inulin is NOT zero.

664
Clearance, RBF and GFR - Dr. Costanzo

V. FILTRATION FRACTION

Filtration fraction = GFR


RPF

VI. Filtration fraction is approximately 0.2 or 20%, meaning that 20% of the renal
plasma flow is filtered across the glomerular capillaries. The remaining 80%
leaves by the efferent arterioles and becomes the peritubular capillary blood flow.

Clearance of inulin = GFR = ≈120 ml/min. Clearance of PAH = effective RPF =


≈600 ml/min. Filtration fraction = 120/600 = 20%; that's the 20% that is filtered.
It works!

VII. PRACTICE QUESTIONS

1. In a urine sample, inulin concentration is 120 mg/ml and PAH


concentration is 1220 mg/ml. In plasma sample, inulin concentration is 1
mg/ml and PAH concentration is 2 mg/ml. Urine flow rate is 1 ml/min.
What is GFR, effective RPF, and filtration fraction?

2. If a drug is administered that causes selective vasodilation of the efferent


arteriole, what effects would you predict on GFR, RPF, filtration fraction?

3. If a drug is administered that causes selective vasodilation of the afferent


arteriole, what effects would you predict on GFR, RPF, filtration fraction?

4. Which of the following would cause an increase in RPF, increase in GFR,


and increase in filtration fraction?

Angiotensin II
ACE inhibitor
Activation of sympathetic nervous system
ANP

ANSWERS

665
Clearance, RBF and GFR - Dr. Costanzo

1. GFR, 120 ml/min; effective RPF, 610 ml/min; filtration fraction, 0.197

2. Decrease, increase, decrease

3. Increase, increase, no change

4. ANP (dilates afferent, which increases RPF and GFR; constricts efferent
which increases GFR; thus, GFR will increase more than RPR, which
increases filtration fraction)

666
Reabsorption and Secretion 1 and 2 - Dr. Costanzo

Reabsorption and Secretion 1 and 2


Linda Costanzo, Ph.D.

OBJECTIVES:

After studying this lecture, the student should understand:

1. Definitions of reabsorption and secretion.


2. How to calculate filtered load, excretion rate, and reabsorption or secretion rate.
3. The glucose titration curve.
4. Causes of glucosuria.
5. The pattern of urea transport along the nephron, and the relationship between
urine flow rate and urea excretion.
6. The principle of non-ionic diffusion and its application to the excretion of weak
acids and bases.

The nephron is lined by a single layer of epithelial cells that serve the functions of
reabsorption and secretion. Reabsorption and secretion occur after filtration, modify the
glomerular filtrate, and ultimately determine how much of each substance will be
excreted in the urine. Reabsorption is the transport of a substance from the glomerular
filtrate (or tubular fluid) into the peritubular capillary blood; in this way, a substance that
was filtered is returned to the blood. Secretion is the transport of a substance from
peritubular capillary blood into the tubular fluid.

Many substances are reabsorbed including: water, Na+, Cl-, HCO3-, glucose, amino acids,
Ca2+, phosphate, and urea. A few substances are secreted including: organic acids (e.g.,
PAH, salicylates), organic bases (e.g., morphine), H+, and K+.

Most transport processes involved in reabsorption and secretion are carrier-mediated and
include: primary active transport, secondary active transport (cotransport and
countertransport), and facilitated diffusion. A few substances are transported by simple
diffusion (non-carrier-mediated). Water movement occurs by osmosis.

667
Reabsorption and Secretion 1 and 2 - Dr. Costanzo

I. MEASUREMENT OF REABSORPTION AND SECRETION RATES

Figure 1. Processes of filtration, reabsorption, and secretion in


a nephron. The sum of the three processes is excretion.

Filtered load = GFR x [P]x **

Excretion rate = V x [U]x

Reabsorption or secretion rate = Filtered load - excretion rate

If the filtered load > excretion rate, there has been net reabsorption. If the filtered
load < excretion rate, there has been net secretion. Units of filtered load, excretion
rate, reabsorption or secretion rate are amount/time (e.g., mg/min, mmoles/min,
mmoles/hour, mmoles/day)

**Note: for freely filtered substances, filtered load is calculated with the equation
shown above. However, for substances bound to plasma proteins (i.e., not freely
filtered), the filtered load calculation must be modified by the % free (unbound).
For example, if a substance is 60% bound to plasma proteins (40% free), filtered
load = GFR x total plasma concentration x 40%.

668
Reabsorption and Secretion 1 and 2 - Dr. Costanzo

II. GLUCOSE REABSORPTION (Titration Curve) – Carrier Mediated,


Cotransport

Figure 2. Cellular mechanism of glucose reabsorption in the


early proximal tubule.

The cellular mechanism of glucose reabsorption is a Na+-glucose cotransporter in


the luminal membrane of early proximal tubule. Glucose is freely filtered and,
therefore the filtered load increases as the plasma glucose concentration increases.
At low plasma glucose, all of the filtered glucose is reabsorbed and excretion is
zero. At higher plasma glucose concentrations, the transporters for reabsorption
become saturated and reabsorption levels off at the Tm level. Once there is
saturation of reabsorption, any additional glucose filtered is excreted in the urine.
Threshold is the point on the excretion curve where glucose first appears in the
urine. Notice that threshold occurs at a lower plasma glucose concentration than
does the Tm, a phenomenon called splay. Splay occurs because of the low affinity
of the Na+-glucose cotransporter and because of nephron heterogeneity (different
nephrons have a different Tm).

Glucosuria (increased glucose in the urine) can occur in diabetes mellitus when
an increased level of plasma glucose results in an increased filtered load that
exceeds the reabsorptive capacity of the nephron. In pregnancy, glucosuria can
occur because increased GFR leads to increased filtered load of glucose. In renal

669
Reabsorption and Secretion 1 and 2 - Dr. Costanzo

glucosuria, there is a decreased Tm due to decreased number or affinity of the


Na+-glucose cotransporter (i.e., renal defect).

III. PAH SECRETION (Titration Curve) – Carrier Mediated, Cotransport

Figure 3. PAH titration curve. PAH (para-aminohippuric acid)


filtration, secretion, and excretion are shown as a function of
plasma PAH concentration. Tm, Tubular transport maximum.

PAH is a prototype of a secreted substance. As with glucose, filtered load is


linearly related to plasma concentration. Secretion is a carrier-mediated process in
the proximal tubule, transporting PAH from peritubular capillary blood into
tubular fluid (urine). At low plasma concentrations of PAH, secretion increases
steeply with increasing concentration because plenty of carriers are available. As
the concentration increases, the carriers saturate and reach Tm. Excretion is the
sum of filtration and secretion; thus, excretion increases steeply at low plasma
concentrations and then less steeply once secretion is saturated.

Think about the following question. To measure RPF with PAH, would you
choose a plasma PAH concentration below Tm or above Tm?

IV. EXAMPLE OF UREA – Simple and Facilitated Diffusion

Urea is freely filtered across the glomerular capillaries, and then reabsorbed and
secreted by the nephron by passive mechanisms (facilitated diffusion and simple

670
Reabsorption and Secretion 1 and 2 - Dr. Costanzo

diffusion). Reabsorption of urea is greater than secretion, so there is net


reabsorption (about 40% of the filtered urea is excreted in the urine). The sites
for urea reabsorption are the proximal tubule and inner medullary collecting
ducts. The site for urea secretion is the thin descending limb of Henle’s loop.

All urea transport in the nephron is passive. Whether reabsorption or secretion,


urea is always moving down its concentration gradient. For urea transport to
occur, there must be a concentration gradient and that portion of the nephron must
be permeable to urea.

A. Proximal tubule. 50% of the filtered urea is reabsorbed in the proximal


tubule by simple diffusion, down its concentration gradient. Where does
this concentration gradient come from? The urea concentration in
glomerular filtrate is identical to the urea concentration in blood (i.e., in
Bowman’s space, there is no concentration gradient). As water is
reabsorbed in the proximal tubule (the subject of later lectures), urea lags
slightly behind, causing the urea concentration in the lumen to become
slightly higher than the urea concentration in blood – this slightly higher
luminal urea concentration becomes the driving force for urea
reabsorption. The corollary is that the more proximal water reabsorption,
the more urea reabsorption.

B. Thin descending limb of Henle. Because 50% of the filtered urea is


reabsorbed in the proximal tubule, 50% remains in the lumen to enter the
loop of Henle. Now, unexpected things happen. You will learn in later
lectures that very high concentrations of urea are established in the
interstitial fluid of the inner medulla (part of the corticopapillary osmotic
gradient). The thin descending limbs of Henle pass through this high-urea
region and, because they are urea- permeable, urea is secreted, down its
concentration gradient, from the interstitial fluid and blood of that region
into the nephron. More urea is secreted into thin descending limbs than
was reabsorbed in the proximal tubule, and approximately 110% of the
filtered load is now present in the lumen and enters the ascending limb of
Henle’s loop.

C. Thick ascending limb of Henle, distal tubule, cortical and outer


medullary collecting ducts. These segments are impermeable to urea,
so no urea transport occurs. However, in the presence of ADH, water is
reabsorbed in the late distal tubule and the cortical and outer medullary
collecting ducts. If water is reabsorbed in these segments, urea is “left
behind,” and the luminal urea concentration becomes very high (you will
apply this fact in the next step).

D. Inner medullary collecting ducts. 110% of the filtered urea arrives at the
inner medullary collecting ducts. Here, there is a specific transporter for
facilitated diffusion of urea, UT1, that is turned on by ADH. In the

671
Reabsorption and Secretion 1 and 2 - Dr. Costanzo

presence of ADH, urea is reabsorbed by the UT1, down its concentration


gradient. About 70% of the filtered urea is reabsorbed, leaving 40% to be
excreted. Urea that is reabsorbed by inner medullary collecting ducts in
the presence of ADH, is added to the corticopapillary osmotic gradient in
a process called urea recycling (i.e., urea that would otherwise have been
excreted, is “recycled” into the corticopapillary gradient). In the absence
of ADH, urea is not reabsorbed by inner medullary collecting ducts.

E. Relationship between urine flow rate and urea excretion. The higher
the urine flow, the higher the urea excretion (see figure below). Why?
Since all urea transport is passive, it depends on concentration difference.
Therefore, urea reabsorption is related to water reabsorption as follows.
Higher water reabsorption (lower urine flow rate) leaves more urea behind
and creates higher luminal urea concentration, which drives higher urea
reabsorption (lower urea excretion). Conversely, lower water reabsorption
(higher urine flow) means less urea reabsorption (higher urea excretion).

Figure 4.

V. NON-IONIC DIIFUSION – Weak Acids and Bases

Many substances secreted by the proximal tubule are weak acids (e.g., PAH,
salicylic acid) or weak bases (e.g., quinine, morphine). Weak acids and bases
exist in two forms, charged and uncharged, and the relative amount of each form
depends on pH. Weak acids have an acid form, HA, and a conjugate base form,
A-. At low pH, HA predominates; HA is uncharged. At high pH, A- predominates;
A- is charged. For weak bases, the base form is B and the conjugate acid is BH+.
At low pH, BH+ (charged) predominates; at high pH, B (uncharged)
predominates.

With respect to renal excretion of weak acids and bases, the relevant points are:
(1) relative amounts of charged and uncharged species in the urine vary with

672
Reabsorption and Secretion 1 and 2 - Dr. Costanzo

urine pH, and (2) only the uncharged species (“non-ionic”) can diffuse across the
cells.

A. Weak acids. To illustrate the role of non-ionic diffusion in excretion of


weak acids, consider the example of salicylic acid. (For simplicity, we will
call both forms, HA and A-, salicylate.) Like PAH, salicylate is filtered
and then secreted by the organic acid secretory mechanism in proximal
tubule. Consequently, the urine concentration of salicylate becomes higher
than the blood concentration, and there is a concentration gradient across
the cells. In urine, there is both the HA and A- forms, but only HA
(uncharged) can "back-diffuse" across the cells, from lumen to blood,
down this concentration gradient. At acid urine pH, HA predominates,
there is more “back-diffusion,” and the clearance of salicylate decreases
(see figure below). At alkaline urine pH, A- predominates, there is less
“back-diffusion,” and the clearance of salicylate increases.

The principle is used for treating aspirin overdose. By intentionally


alkalinizing the urine, more A- is created in the urine, and total salicylate
excretion is increased.

B. Weak bases. The effect of non-ionic diffusion on excretion of weak bases


is the opposite. The weak base is filtered and secreted, causing a higher
urine concentration than blood concentration. In the urine, there is both
BH+ and B forms, but only B (uncharged) can back-diffuse across the
cells, from lumen to blood, down this concentration gradient. At alkaline
urine pH, B predominates, there is more back-diffusion from urine to
blood, and the clearance of the weak base is decreased. At acid urine pH,
BH+ predominates, there is less back-diffusion, and the clearance of weak
base is increased.

Figure 5.

VI. PRACTICE QUESTIONS

1. The plasma concentration of inulin is 2 mg/ml, the plasma concentration


of X is 2 mg/ml, the urine concentration of inulin is 260 mg/ml, the urine

673
Reabsorption and Secretion 1 and 2 - Dr. Costanzo

concentration of X is 120 mg/ml, and the urine flow rate is 1 ml/min. Is


there net reabsorption or net secretion of X and what is the rate? (Assume
that X is not protein-bound.)

2. Glomerular filtration rate is 110 ml/min. The plasma concentration of Y is


100 mg/ml and Y is 70% bound to plasma protein. Urinary excretion of Y
is 4 g/min. Is there net reabsorption or secretion of Y and what is the rate?

3. In untreated type I diabetes mellitus, glucose is excreted in the urine.


Based on the titration curve for glucose, how would you explain the
glucosuria?

4. To measure renal plasma flow with the clearance of PAH, should the
plasma concentration of PAH concentration be below the Tm or above the
Tm for PAH secretion?

5. A person takes an overdose of morphine (a weak base). To increase the


urinary excretion of morphine, would you acidify or alkalinize the urine?

ANSWERS

1. Net reabsorption of X; 140 mg/min


2. Hint non protein-bound or ultrafilterable concentration of Y in plasma is
30 mg/ml. Net secretion; 700 mg/min
3. Plasma glucose concentration is higher than the renal threshold for glucose
excretion
4. Below Tm (To measure RPF with clearance of PAH, renal vein PAH is
assumed to be zero. For this to be true, or nearly true, all PAH in the RPF
must be cleared by a combination of filtration and secretion, and secretion
must not be saturated.)
5. Acidify the urine (Shift equilibrium toward increased BH+ in urine,
decreased B, less back-diffusion from urine to blood, increased excretion)

674
Gastrointestinal Motility 2 - Dr. Grider

Gastrointestinal Motility 2: Intestinal and Colonic Motility


Jack Grider, Ph.D.

OBJECTIVES:

1. Contrast the types of motility in the small intestine.


2. Describe the neural circuits that mediate peristalsis.
3. Differentiate the migrating motor complex from the peristaltic type of motility.
4. Compare colonic motility with that of the small intestine.
5. Describe the process of defecation.

Suggested Reading: Berne & Levy pp. 543- 563

I. MOTILITY OF THE SMALL INTESTINE


A. Motility Patterns.

1. There are three normal motility patterns that occur in the small
intestine (Figure 1):

a. Propagating contractions or Peristalsis. These are spatially


and temporally coordinated
b. Stationary contractions or Segmentation. These occur at
random in a region and do not propagate.
c. Clustered contractions. These are groups of contractions that
may be stationary or may propagate.

Figure 1.

675
Gastrointestinal Motility 2 - Dr. Grider

2. There are two abnormal motility patterns (Figure 2)

a. Antiperistaltic contractions. These move in the anal to oral


direction and may be indicative of pathology.
b. Aboral Power Contractions. The are high amplitude
contractions that move in the normal oral to anal direction.

Figure 2.

B. Postprandial (after eating) motility consists mainly of two types of motility,


peristalsis (dashed lines in Figure 3) and segmentation (arrow heads in
Figure 3)

Figure 3.

676
Gastrointestinal Motility 2 - Dr. Grider

C. Peristalsis (Figure 4A&4B)

1. As described earlier peristalsis is a traveling wave of contraction


preceded by a traveling wave of relaxation. The process is mediated
through neurons of myenteric plexus and is therefore a Short Arc
Reflex.
2. A sensory stimulus initiates the peristaltic reflex, a dual reflex
consisting of the "ascending excitatory reflex and descending
inhibitory reflex". Different stimuli may activate the reflex via
different mechanisms described below although once activated, the
efferent limb of the reflex is identical for all stimuli and is mediated
wholly by intrinsic or enteric neurons. The reflex results in contraction
of circular muscle and relaxation of longitudinal muscle orad or above
the site of stimulation (ascending excitatory reflex) and a relaxation of
circular muscle and contraction of longitudinal muscle caudad or
below the site of stimulation (descending inhibitory reflex). (the Law
of the Intestine). Contraction of both muscle layers is mediated by
acetylcholine/tachykinin excitatory motor neurons. Relaxation of
circular muscle is mediated by VIP/NOS inhibitory motor neurons.
Relaxation of longitudinal muscle is mediated by inhibition of
cholinergic/tachykinin motor neurons. The reflex arc likely involves
several interneurons that release a variety of neurotransmitters such
as enkephalins, somatostatin, Neuropeptide Y, and
acetylcholine/tachykinin.
3. Sensory (Afferent) limb: Short chain fatty acids, HCl, Bile acids, and
mecahnical distortion of the villi of the mucosa activate an intrinsic
primary afferent neuron (IPAN) indirectly. These stimuli cause the
release of serotonin (5-hydroxytryptamine)from enterochromaffin
cells of the mucosa. These are a subtype of enteroendocrine cell.
Local paracrine release of serotonin activates a 5-HT4 receptor on the
IPAN. The IPAN then releases the sensory transmitter calcitonin
gene-related peptide (CGRP) and substance P to activated the
interneurons in the reflex arc. This mucosal reflex is the most likely
physiological stimulus for intestinal peristalsis. Physical distention of
the gut wall activates an extrinsic afferent CGRP neuron originating in
the dorsal root ganglion. An axon collateral of this extrinsic neuron has
a branch that activates the same interneurons and motor neurons of the
peristaltic reflex pathway activated by mucosal stimulation. This is
called an Axon Collateral or Distension Reflex and probably is
activated in pathological conditions. It may also be activated when
there is a need for more powerful peristalsis since the mucosal reflex
and the distension reflex potentiate each other.
4. Peristalsis becomes a moving wave of the peristaltic reflex
(contraction preceded by a moving wave of relaxation) that pushes
chyme caudad (anally). It is the primary propulsive type of
contraction in the small intestine.

677
Gastrointestinal Motility 2 - Dr. Grider

5. Normally, these peristaltic contractions only travel a few centimeters


before dying out because of dissipation of the stimulus.

Figure 4A.

Figure 4B.

D. Segmentation

678
Gastrointestinal Motility 2 - Dr. Grider

1. As described earlier this is a local cycle of relaxation and contraction.

a. This type of motility is non-propagating and is non-


propulsive by itself.

2. Segmental contractions are the main mixing movements of the small


intestine. Chyme is pushed orad and caudad to expose more of it to the
mucosa of the gut where it is absorbed. This movement also mixes
chyme with digestive enzymes and secretions to increase digestion.
3. The maximal rate of contraction is set by rate of the electrical slow
waves because in the intestine the contraction is determined by the
spike potential that occur on the plateau of the slow wave. This is
determined by local pacemaker cells (Interstitial cells of Cajal) and is
characteristic of the region of the intestine. The slow wave rate is
about 12/min in the duodenum and decreases anally to about 8/min in
the ileum so that there is a net frequency gradient which slowly moves
chyme anally towards the ileocecal sphincter (Figure 5).

Figure 5.

E. Migrating Myoelectric (or Motor) Complex (MMC)

1. During the interdigestive period, a distinct pattern of electrical and


contractile activity spreads through the gut. This pattern is called the
migrating myoelectric complex (MMC), migrating motor complex
(MMC) or the interdigestive housekeeper.
2. The pattern consists of three phases: a period of quiescence (Phase I)
lasting 45-60min, followed by a phase of increasing contractile activity
where slow waves have increasingly more action potentials (Phase II),
followed by a short period (about 10 min) during which almost every
slow wave has action potentials and the muscle is contracting intensely
(Phase III) (Figure 6).

679
Gastrointestinal Motility 2 - Dr. Grider

Figure 6.

3. This cycle originates in the stomach and migrates anally throughout


the small intestine. A new MMC begins in the stomach every 75-90
min (Figure 7). This type of motility allows large particle to leave the
stomach and enter the small intestine.
4. The function of the MMC seems to be to continually sweep through
the small intestine and keep it clear of secretions and chime between
meals.
5. The mechanism is not fully understood however neural reflexes are
involved. The hormone motilin cycles in the same manner as the
MMC and may also have a role in initiating or regulation the MMC.
Opioid peptides and somatostatin have also been implicated.
6. On ingestion of a meal, the MMC is interrupted by continuous
contractile activity throughout the small intestine. This continuous
activity is of the segmental, non-propulsive type (Figure 7).

Figure 7.

F. Intestinal reflexes

1. The activity of the intestine is coordinated over long distances and


between other digestive organs by long arc reflexes.

680
Gastrointestinal Motility 2 - Dr. Grider

2. The intestino-intestinal reflex coordinates regions of the intestine


such that distension of one area results in inhibition of motility in other
area. This ensures adequate digestion in all regions and prevents
blockage of movement.
3. The gastro-ileal reflex coordinates activity in the stomach and ileum
so that increase activity in the stomach causes increased activity in the
ileum to clear it of contents.
4. Even though these are long arc reflxes, remember that they act finally
through the enteric nervous system.

G. Ileocecal sphincter

1. The sphincter at the ileocecal junction ensures the continued


movement of chyme in the anal direction. The sphincter extends
several cm orally in the distal ileum and extends into the cecum where
it has a fold or flap that prevents movement from cecum into ileum. In
addition, neural reflexes exist such that an increase in intracecal
pressure results in an increase in the sphincteric pressure.

II. COLONIC MOTILITY

A. Functional anatomy (Figure 8)

1. Longitudinal muscle gathered into three bands called tenia coli.


2. Circular muscle forms pouches called haustra.

Figure 8.

681
Gastrointestinal Motility 2 - Dr. Grider

B. Motility patterns

1. Mixing movements or local contractions:

a. These mix lumenal contents and are analogous to segmentation


in the small intestine. Often those of the ascending colon are
antiperistaltic (anal to oral direction) and function to delay
movement of feces into the transverse colon.
b. These push feces towards the cecum, which is a storage site
somewhat analgous to the stomach, and also aid in the removal
of water and electrolyte.

2. Mass movements: Major propulsive contraction which usually is


initiated in transverse colon and propels feces long distances into
sigmoid colon and rectum., Usually occurs 1-2 times per day.

C. Colonic reflexes

1. Colonic long arc reflexes are similar to those of the intestine.


2. The colono-colonic reflex insures that distension of one region of the
colon causes relaxation of other regions. This is mediated by the
sympathetic nervous system.
3. The gastrocolonic reflex: distension of the stomach caused increased
motility in the colon and increases the frequency of the mass
movements in order to prepare the way for material to arrive in the
colon.

D. Defecation Reflex (Figure 9)

1. Mass movement pushed feces distally, filling and distending the


rectum and activating stretch receptors in muscle wall.
2. A reflex is initiated through enteric and pelvic nerves to smooth
muscle and through the pudendal nerve to the skeletal muscle (external
anal sphincter).
3. The rectum contracts pushing feces towards the anus.
4. At the same time, the internal anal sphincter relaxes and the external
anal sphincter contracts.
5. Neural inputs from the central nervous system then either reinforce or
inhibit these signals. If reinforced, defecation is aborted; if inhibited,
defecation proceeds. Thus, defecation is under voluntary control.

682
Gastrointestinal Motility 2 - Dr. Grider

Figure 9.

6. Defecation requires additionally regulation of muscles of the pelvic


floor in coordination with the internal and external anal sphincter. The
Puborectalis muscle normally is contracted thereby pulling the anal
canal at a 90-degree angle. This hinders the passage of feces.
Defecation includes relaxation of the puborectalis and elevation of the
levator ani muscle to increase the anal canal angle to about 110-
degrees. In addition contraction of the diaphragm increases intra-
abdominal pressure that adds to pressure generated by contraction of
the rectum.

Figure 10.

683
Gastrointestinal Motility 2 - Dr. Grider

III. STUDY QUESTIONS

1. The rate of intestinal segmentation

A. Is dependent of the concentration of gut hormones.


B. Decreases from duodenum to ileum.
C. Is set by the number of smooth muscle cells contracting.
D. Depends on the density of innervation of a region by the vagus
nerve.
E. Is highest in the Sphincter of Oddi.

2. The peristaltic reflex following mucosal stimulation is initiated as a result


of the release of

A. Substance P.
B. Acetylcholine.
C. Serotonin (5-HT).
D. Nitric Oxide.
E. Vasoactive Intestinal Peptide.

3. Distension of the rectum

A. Always leads to a defecation reflex.


B. Causes only a passive contraction of the rectal circular muscle.
C. Causes contraction of both the internal and external anal
sphincters.
D. Causes a mass movement of the transverse colon.
E. Causes relaxation of the internal anal sphincter and contractions of
the external anal sphincter.

Answers: 1=B, 2=C, 3=E.

684
GI Secretion 1 - Dr. Grider

GI Secretion 1: Salivary and Gastric Secretion


Jack Grider, Ph.D.

OBJECTIVES:

1. List the volumes of secretion by various regions.


2. Predict the components of salivary secretion at different flow rates.
3. Describe the neural regulation of salivary secretion.
4. List the secretions and cells of origin for gastric secretions.
5. Explain the cellular steps involved in acid secretion in the stomach.
6. Discuss the interaction between neural, hormonal and paracrine regulatory
mechanisms regulating acid secretion at different phases of a meal.

Suggested Reading: Berne & Levy; pp. 566-580

I. GENERAL CHARACTERISTICS

A. Types of secretion
1. The table below indicates the volume of secretions and ingested fluids
entering the GI tract per day. (About 8.5 liters/day secretion).

Source Volume

Oral Intake 1500ml


Salivary Glands 1500
Stomach 2500
Bile 500
Pancreas 1500
Intestine 1000

TOTAL 8500ml

2. Of this, all but about 200/day ml is reabsorbed (mostly by the


intestine).
3. Secretions are of two types:serous and protein. Often these
components are derived from different cells within the structure of the
gland and its ducts.
4. The serous component is composed of water and electrolytes. The
nature of the electrolyte varies with the gland and functions to provide
the optimal environment for digestion.
5. The protein component is composed of enzymes necessary for the
digestion of nutrients, and mucus (mucopolysaccharides and
glycoproteins) which lubricates the chyme and provides some
buffering of pH in food and in chyme leaving stomach.

685
GI Secretion 1 - Dr. Grider

B. Types of glands

1. Mucous glands: These are single cells (goblet cells) interspersed with
the epithelial lining cells in the mucosa. These secrete mucus which
adheres to and protects the mucosa. (Figure 1a.)
2. Tubular glands: These are invaginations of the epithelium which
contain secretory cells. Secretions enter the lumen of the tube and flow
up and out into the lumen of the gut.

a. Gastric pits and Intestinal crypts are examples. (Figure 1a.)

3. Compound glands: These glands are outside of the gut wall. They are
composed of two parts (acini and ducts) arranged like a cluster of
grapes. The acini are composed of the secretory cells which secrete
into the center of the acinus. This secretion is called the primary
secretion. The primary secretion then flows towards the gut through a
system of increasingly larger ducts. The cells lining the ducts often
modify the electrolyte and water component of the secretion. The
modified secretion or secondary secretion is then emptied into the
lumen of the gut. The salivary gland and exocrine pancreas are
examples of compound glands. (Figure 1b.)

Figure 1a.

686
GI Secretion 1 - Dr. Grider

Figure 1b.

C. Intracellular mediators of secretion

1. Secretory cells have receptors for a variety of hormones and


neurotransmitters.
2. Some of these receptors are coupled to activation of adenylyl cyclase.
The net effect is to increase intracellular cyclic AMP which leads to
secretion. B-adrenergic agents, VIP and members of the secretin
family of peptides, and cholera toxin act through this mechanism.
3. Other secretary agents act by causing the hydrolysis of
phosphatidylinositol-(4,5) bisphosphate in the membrane to diacyl
glycerol (DAG) and inositol-(1,4,5) trisphosphate (IP3). The latter
is a water soluble product that enters the cytoplasm and causes a rise in
intracellular calcium via release of calcium from intracellular stores in
the endoplasmic reticulum. The rise in calcium results in secretion.

687
GI Secretion 1 - Dr. Grider

Agents acting through this mechanism include : acetylcholine, CCK,


tachykinins.

II. SALIVARY SECRETION

A. Three Glands

1. Parotid gland: secretion is only serous.


2. Submaxilary gland: secretion is serous and mucous.
3. Sublingual gland: secretion is serous and mucous, but mostly
mucous.

B. Functions of saliva

1. Mucous component lubricates food and also keeps the pH neutral.


2. Enzyme component has ptyalin which is an alpha amylase, and a
lipase, lingual lipase. These begin the digestion of carbohydrates and
lipids in oral cavity.
3. Aids in taste, speech and preventing oral infections.

C. Primary secretion (Figure 2)

1. Elaborated by acinar cells.


2. Composed of water and electrolytes, mucus and enzyme (ptyalin and
lingual lipase).
3. Electrolytes: Na+, K +, Cl -, and HCO3- secreted into acinar lumen at
nearly isotonic concentrations.

D. Secondary secretion (Figure 2)

1. The secondary secretion is the result of modification of the primary


secretion by cells lining duct system.
2. Duct cells absorb Na+ and Cl- from the solution and secrete K+ and
HCO3- . More electrolyte is removed than is added because the cells
lining the distal ducts are not fully water permeable so less water
follows electrolyte than expected. This results in a secondary secretion
that is hypotonic.

E. Effect of flow rate (Figure 3)

1. The rate of flow determines the amount of time the primary secretion
is in contact with the cells of the duct. The slower the flow, the longer
the contact time and the greater the opportunity for exchange of
electrolytes. At rapid flow rates, there is less time for exchange of
electrolytes so the final secretion resembles the primary secretion more
closely.

688
GI Secretion 1 - Dr. Grider

2. At slow flow rates, the saliva is low in NaCl and HCO 3- Saliva is
very hypotonic.
3. At rapid flow rates, the saliva is closer to that described for the
primary solution and is nearly isotonic.

Figure 2.

689
GI Secretion 1 - Dr. Grider

Figure 3.

F. Control of secretion

1. Hormonal

a. None

2. Neural

a. Parasympathetic: increases secretion. Nerve fibers release


both Ach and VIP. Both of these stimulate acinar cells directly
(VIP increases cAMP and Ach increases Ca2+ ).
Parasympathetic-stimulated secretion is high in ptyalin and the
serous (water & electrolyte) component since the blood vessels
around the gland are dilated by VIP increasing blood flow.
b. Sympathetic: increases secretion. Nerve fibers release nor-
epinephrine which activates both alpha and beta adrenergic
receptors (alpha increases Ca2+ and beta increases cAMP).
Sympathetic nerve activation constricts blood vessels limiting
blood flow so that the serous component is decreased resulting
in a high mucous concentration.
c. Note: both parasympathetic and sympathetic stimulate
secretion but the former is more potent than the latter.

G. Protein component:

690
GI Secretion 1 - Dr. Grider

1. Ptyalin or salivary amylase is active mainly in an alkaline


environment (pH optimum:7.0; pH Range= 4-11). As a swallowed
bolus enters the stomach, it encounters an acid environment which
begins to inactivate this amylase. However, amylase attached to its
substrate is protected from inactivation as is amylase in the core of a
bolus where the pH remains alkaline. Thus, a portion of salivary
amylase continues to function while the meal is in the stomach. Any
salivary amylase surviving passage to the intestine will continue to
function in the alkaline environment of the intestine. Thus, this
amylase plays an important role in the digestion of carbohydrate.
2. Lingual lipase: begins initial digestion of long chain triglycerides.
Since its pH optimum is 4.0 (Range of 2 to8) it continues to digest
lipid while the meal is in the stomach. No co-lipase is needed with
lingual lipase. This lipase preferentially removes the fatty acid in
position 3 of triglyceride.

III. GASTRIC SECRETION

A. Types of secretion

1. Hydrogen ion: secreted by parietal (oxyntic) cells located in


oxyntic glanda of the fundus and corpus of the stomach. H+ needed
for conversion of pepsinogen to pepsin. H+ is needed to provide
pH optimum for protein digestion by pepsin, and to prevents
bacterial growth.
2. Pepsinogen: secreted by chief cells of oxyntic gland. Cells secrete
an inactive form of the proteolytic enzyme which is then converted
to the active form (pepsin) by H+ in lumen.
3. Lipase: A gastric lipase is secreted from the chief cell. It is unlike
pancreatic lipase but similar to lingual lipase. Gastric lipase liberates
a single free fatty acid and a diglyceride from triglycerides and has
an acidic pH optimum.
4. Intrinsic factor (IF): secreted by oxyntic cells. IF is needed for
vitamin B12 absorption in ileum.
5. Mucus: Secreted by mucous cells in surface epithelium and in neck
of oxyntic gland. Mucus adheres to the surface and protects stomach
from autodigestion by acid and pepsin (neutralization of acid).
(Figure 4.)

691
GI Secretion 1 - Dr. Grider

Figure 4.

B. Functional anatomy

1. Oxyntic gland mucosa (Figure 5)

a. Epithelial cells of mucosa invaginate into mucosa to form


gastric pits.
b. These pits then form epithelial or tubular glands.
c. In the fundus and corpus, these oxyntic glands contains oxyntic
(parietal) cells which secrete HCl and intrinsic factor, Chief
cells which secrete pepsinogen, and mucous neck cells which
secrete mucus. Parietal cells are more numerous at base of the
gland.
d. Glands are surrounded by an extensive network of blood
vessels and capillaries that parallel the gland.

692
GI Secretion 1 - Dr. Grider

Figure 5.

C. Hydrogen ion secretion by parietal cell (Figure 6a)

1. Oxyntic cells have invaginations of the apical surface called canaliculi


which increase dramatically immediately after cell is stimulated. The
cell secretes H+ ion via a H+/K+ ATPase that is located on the apical
membrane so that the increase in apical membrane generated by the
canaliculi results in a greater the amount of active H+/K+ ATPase.
This allows the cell to secrete acid rapidly to achieve a final gastric pH
of less than 1.

693
GI Secretion 1 - Dr. Grider

2. H+ is generated within the cell from H2O. The resulting OH- is


neutralized by reaction with CO2, derived from the blood and from
cellular metabolism, to form HCO3-. This reaction is catalyzed by
carbonic anhydrase.
3. The HCO3- passively diffuses into the blood in exchange for Cl -.
4. At the canalicular surface (gastric lumen) H+ is actively pumped out of
the cell in exchange for K by the H/K ATPase. This is also called the
Proton Pump and is the site of action of Proton Pump Inhibitors
(PPIs) such as Omeprazole used extensively to treat gastric ulcer
disease and gastroesophageal reflux disease (GERD)
5. The net reaction is thus:H2O+ CO2+NaCl ----- > NaHCO3 (blood) +
HCl (Lumen).
6. At rest or at low levels of stimulation, NaCl is mainly secreted and at
high rates of stimulation, mainly HCl is secreted (Figure 6b).

Figure 6a.

694
GI Secretion 1 - Dr. Grider

Figure 6b.

D. Control of Acid Secretion

1. Neural: (Figure 8)

a. Enteric: Nerves of the enteric nervous system can simulate the


parietal (and peptic) cell directly. These effects are mainly
mediated by ACh (Cholinergic)
b. Vagus (Parasympathetic): Vagal activation during the
cephalic and gastric phases (via long arc reflex) leads to
increase in activity of local enteric excitatory neurons to
release ACh. The vagus can also activate local enteric neurons
innervating enterochromaffin-like cells (ECL cell) in the
stomach. These secrete histamine, which also stimulates the
parietal cell. Finally, vagal stimulation activates an enteric
neuron that innervates the antral G cell and causes gastrin
secretion. This enteric neuron uses a peptide transmitter called
GRP (Gastrin Releasing Peptide).

695
GI Secretion 1 - Dr. Grider

Figure 7.

2. Hormonal (Figure 8)

a. Gastrin: Gastrin is released into the blood from G-cells


located in antral mucosa. Release is stimulated by gastric
distension, presence of protein digestion production in the
chyme, or vagal stimulation (see above). At the parietal/oxyntic
cell, gastrin receptors (Also called - CCK-B receptors) are
stimulated, intracellular Calcium is elevated and HCl secretion
occurs. CCK can also stimulate this receptors but is of lower
potency then gastrin. If CCK concentrations are high, then
CCK can act as a competitive antagonist.

Figure 8.

3. Paracrine (Figures 8&9)

a. Histamine: Released from enterochromaffin-like cells in


lamina propria in response to vagal stimulation, or local

696
GI Secretion 1 - Dr. Grider

inflammation. Histamine acts in a paracrine manner to diffuse


through the extracellular space and activate H2 receptors on
parietal cells. This results in an increase in intracellular cyclic
AMP and increased HCl secretion. H2 receptor blocking
drugs such as cimetidine are used to treat gastric ulcer disease.
b. Somatostatin (Sts): Released from paracrine cells in mucosa.
There are cell located in the fundus that release Sts onto
parietal cells where it acts on Sts receptors. This results in a
decrease in intracellular cAMP and an inhibition of HCl
secretion. Similar cells in the antral mucosa act on the G cells
to inhibit gastrin secretion. The nNet effect is to decrease HCl
secretion. (Figure 9)

Figure 9.

E. Phases of acid secretion

1. Cephalic phase: Sight or smell of food activates neural (vagal) reflex


resulting in stimulation of the parietal cell directly as well as inducing
the release of gastrin from the G-cell. This phase is in preparation for a
meal.

697
GI Secretion 1 - Dr. Grider

Figure 10.

2. Gastric phase: presence of a meal in the stomach causes maximal


stimulation of acid secretion. Neural (local and vagal) mechanisms
directly stimulate the parietal cell. G-cell release of gastrin is
stimulated by distension of antrum, protein digestion products in
antrum, and neural activity. (Figure 11)

Figure 11.

3. Intestinal phase: The presence of chyme in the intestine and acid in


antrum begin to inhibit acid secretion. A fall in pH of the antrum
below 2 inhibits gastrin release via activation of somatostatin release.
Presence of food and acid in duodenum initiate neural reflexes which
inhibit acid secretion. Also low pH directly inhibits acid secretion.

698
GI Secretion 1 - Dr. Grider

a. Presence of food and acid in duodenum causes release of


hormones (GIP, CCK, secretin, enterogastrone) which inhibit
acid secretion.

4. During the course of a meal each phase occurs in sequence to cause a


rise then a fall in acid secretion. (Figure 12).

Figure 12.

F. Pepsinogen and Lipase secretion

1. Stimulated by vagal and local nervous mechanisms.


2. Usually secreted at same time as acid and flows up gastric gland lumen
towards surface with HCl.
3. Acid in lumen converts pepsinogen to activate pepsin by cleavage of
short peptide fragment from pepsinogen.

IV. SUMMARY

A. As a result of gastric and salivary secretory, motor and digestive activity,


the ingested material (now termed “Chyme”) had been significantly
altered to a state more conductive to further digestion and absorption.
Alterations include:

1. Emulsification of fat/lipid material


2. Initial conversion of triglyceride to FFA and diglyceride
3. Initial digestion of protein to oligo/polypeptides and few Amino
Acids
4. Initial digestion of large carbohydrates & starches to produce some
oligosaccharides
5. Some absorption of water (about 500ml) and ions

699
GI Secretion 1 - Dr. Grider

6. Acidification of chyme
7. Large increase in osmolarity of chyme

B. Some of these changes are beneficial, some present a challenge to the


intestine if uncorrected, and some create intraluminal molecules necessary
for release of hormones in the duodenum and for the activation of local
and long-arc reflexes in the small intestine.

V. STUDY QUESTIONS

1. The largest total amount of fluid secretion into the gut occurs in the

A. Oral cavity.
B. Esophagus.
C. Stomach.
D. Small intestine.
E. Colon.

2. During the intestinal phase of acid secretion

. Gastrin causes maximal acid secretion.


A. Pancreatic polypeptide causes gastrin release.
B. Secretin inhibits acid secretion.
C. Vagal stimulation is maximal.
D. Secretion from the Chief cells is stimulated.

3. Salivary gland secretion is stimulated by

. Secretin.
A. Gastrin.
B. Ptyalin.
C. Parasympathetic nerve stimulation.
D. Somatostatin.

4. Stimulation of the vagus nerve

. inhibits the secretion of pepsinogen.


A. occurs mainly during the intestinal phase of acid secretion.
B. causes contraction of the gastric fundus.
C. initiates the secretion of acid and enzymes.
D. inhibits antral contractions.

Answers 1=D,2=C,3=D, 4=D

700
GI Secretion 2 - Dr. Grider

GI Secretion 2: Pancreatic, Biliary and Intestinal Secretion


Jack Grider, Ph.D.

OBJECTIVES:

1. Describe the components of pancreatic secretion.


2. Describe the effect of flow rate on the components of pancreatic secretion and
compare to the effects of flow rate on salivary secretion.
3. List the mechanisms that prevent the pancreas from auto digestion.
4. Discuss the neural and hormonal mechanisms that regulate pancreatic secretion.
5. Explain the cellular processes that lead to synergism between stimulants of
pancreatic secretion.
6. Define the role of the CFTR in intestinal secretion.
7. Describe the regulation of intestinal secretion and the role of intrinsic and
extrinsic mechanisms.

Suggested Reading: Berne & Levy pp. 580-584,592-593, 595-608

As chyme leaves the stomach, it is acidic and usually very hyperosmotic. One of the
main functions of the Gastro-duodenal junction region is to reverse this by adding a large
volume of hyposmotic fluid containing bicarbonate ions. This secretion is derived from
duodenal glands lining the upper small intestine and as part of the biliary and pancreatic
secretion which enters the upper small intestine via the Sphincter of Oddi.

Figure 1.

I. PANCREATIC SECRETION

A. Functional Anatomy

701
GI Secretion 2 - Dr. Grider

1. Endocrine Portion: Islets of Langerhans-secrete Insulin,


Somatostatin, Glucagon and Pancreatic Polypeptide- into blood
(enterohepatic and local pancreatic portal systems important here).
2. Exocrine Portion: Enzyme, water, bicarbonate secretion into
duodenum via the pancreatic duct and common bile duct. The
secretion enters the duodenum at the ampula through the Sphincter
of Oddi. Enzyme secretion originates from the acinar cells, and
water and bicarbonate originate from ductal cells. (Fig. 2) Net
secretion is high in enzyme and is isotonic to mildly hypotonic and
alkaline.

Figure 2.

B. Secretion of water and electrolyte

1. Secretions function to dilute and neutralize the chyme as it enters the


duodenum. Osmolarity falls rapidly in duodenum (see fig 1) and
secretions are important in maintaining fluid balance (dumping
syndrome), providing optimal environment for digestive enzymes, and
for protection of mucosa.
2. Influence of flow rates: The final composition of secretion varies with
flow rate. Whereas the proximal duct cells secrete water and
bicarbonate, the more distal ducts reabsorb water and bicarbonate. At
high rates, the net secretion is high in bicarbonate and low in chloride.
This is reflective of the primary secretion. The reverse is true at low
flow rates, which are reflective of the secondary secretion. (Fig 3)

702
GI Secretion 2 - Dr. Grider

Figure 3.

C. Secretion of enzymes

1. Storage: All enzymes are stored in membrane bound vesicles called


zymogen granules. Proteolytic enzymes are stored as inactive
enzymes (-ogen, pro- enzymes) and the zymogen granules contain a
trypsin inhibitor which prevents trypsin from being activated within
the granule. These mechanisms protect the pancreas from being
digested by its own enzymes (autodigestion). In addition, the enzymes
need to be exposed to enterokinase and trypsin in the intestinal lumen
before becoming activated (Figure 4). Finally, these enzymes require
the neutral pH of intestine to be optimally active whereas the zymogen
granule is acidic.

Figure 4.

2. Release: Enzymes are released by fusion of the granule with the apical
membrane of the acinar cell. The granule contains all types of enzymes
so the whole complement of enzymes is secreted rather than a single

703
GI Secretion 2 - Dr. Grider

type (i.e. the enzyme complement is not matched to the nutrient


content of a meal.

D. Types of enzymes

1. Lipolytic: digestion of lipids

a. Lipase: Lipase is an esterase which splits triglycerides into


glycerol and free fatty acids, usually generating two fatty acids
from 1- and 3- positions and a 2-monoglyceride. Lipase
requires an oil/water interface, bile salts, and co-lipase (also
secreted by pancreas) to function optimally.
b. Cholesterol ester hydrolyase
c. Phospholipase

2. Amylases: digestion of carbohydrates

a. Pancreatic amylase: alpha-amylase which cleaves alpha-1,4-


linkage of starch to generate maltose, maltotriose, and other
alpha-limit dextrins.

3. Proteolytic: digestion of proteins

a. Trypsin(ogen): endopeptidase active at basic amino acids.


Trypsin is responsible of activation of the other proteolytic
enzymes in gut lumen.
b. Chymotrypsin(ogen):endopeptidase active at aromatic amino
acids.
c. (Pro)carboxypeptidase A & B: exopeptidase active at C-
terminal end of peptide. A acts at aromatic and neutral amino
acids and B at basic amino acids.

4. Others: Ribonuclease, deoxyribonuclease, elastase

E. Regulation of Secretion

1. Neural

a. Vagal stimulation: Stimulation of vagal preganglionic nerve


fibers initiate secretion via stimulation of neurons in the enteric
nervous system innervating the acinar cells. Local enteric
neurons release ACh, VIP (vasoactive intestinal peptide), and
GRP (Gastrin releasing peptide). (Figure 5)
b. Sympathetic: Indirect inhibition via vasoconstriction of blood
supply to pancreas.

704
GI Secretion 2 - Dr. Grider

Figure 5.

2. Endocrine (More important than neural)

a. Secretin: Major stimulant of water and bicarbonate.

i. The presence of acid (pH<4) in the duodenal lumen


causes the release of secretin into the blood from
mucosal cells. Secretin then binds to its receptor on the
duct cells and stimulates the secretion of water and
bicarbonate. Secretin also stimulates bicarbonate and
water secretion from duodenal glands and hepatocytes,
and inhibits acid secretion from stomach. The net effect
is to rapidly return duodenal pH towards neutral levels,
thereby removing the stimulus for its own release
(negative feedback loop).

b. Cholecystokinin: Major stimulant of enzyme secretion.

i. The presence of food in the lumen, especially fat


digestion products and amino acids, can stimulate the
release of CCK from mucosal cells into the blood. CCK
can also be release by neural (vagal) stimulation.
ii. CCK acts on CCK-A receptors on acinar cells to
stimulate enzyme secretion directly and also acts on
vagal afferent neural fibers to stimulate a long arc
reflex (Vagovagal). This stimulates enzyme secretion
indirectly (Figure 6). CCK also stimulates gallbladder
contraction and sphincter of Oddi relaxation (Fig 7).
These biliary effects are both neurally mediated (Figure
6&7). The contraction is the result of the activation of
enteric neurons that release ACh and the relaxation is
the result of the activation of enteric neurons that
release VIP/NO.

705
GI Secretion 2 - Dr. Grider

Figure 6.

Figure 7.

c. Pancreatic Polypeptide: Major inhibitory regulator.

i. PP is released from endocrine cells of pancreatic islets


into the pancreatic portal system in response to vagal
stimulation. PP acts to inhibit Ach release from enteric
neurons indirectly by inhibiting vagal output from the
CNS (negative feed back). (Fig 8) A similar peptide
PYY is released from the ileum in response to

706
GI Secretion 2 - Dr. Grider

intraluminal lipids and may mediate the "Ileal Break".


(Figure 9)

Figure 8.

Figure 9.

3. Regulation of pancreatic secretion

a. Pancreatic secretion can be divided into cephalic, gastric and


intestinal phases just as acid secretion was divided into
phases. As indicated in the table below, cephalic and gastric
phases are primarily neurally regulated whereas as the

707
GI Secretion 2 - Dr. Grider

intestinal phase in primarily, but not exclusively regulated by


hormones.

Table 1. The Three Phases of Postprandial Pancreatic Secretion

% of Pancreatic
Phases Stimulants Mediators
Response
Sight, Smell, Tasting,
Cephalic 25% Vagal
Eating
Gastric 10% Distension Vagal Cholinergic
CCK, Secretion,
Amino Acids, Fatty
Enteropancreatic
Intestinal 50-75% Acids, Ca++, H+,
Reflexes, Other
Distension
Hormones

4. Intracellular messengers

a. Just as excitation-contraction coupling was discussed for


smooth muscle cells, secretory or acinar cells demonstrate
“stimulus-secretion coupling” between membrane receptor
activation, generation of intracellular messengers and secretion
of protein. (Fig 10)
b. Both cAMP and Ca2+ can cause acinar cell secretion and are
usually synergistic.
c. Ach and CCK/Gastrin responses are mediated by an increase in
intracellular Ca++.
d. Secretin and VIP responses are mediated by an increase in
cAMP.
e. The effects of these second messengers is to activate protein
kinases which phosphorylate (or conversely phosphatases
which de-phosphorylate) cellular proteins (e.g. cytoskeletal
proteins) that cause vesicles containing the secretory product to
move to the luminal membrane. Fusion of the vesicle
membrane and the cell membrane releases the secretory
product into the lumen.

708
GI Secretion 2 - Dr. Grider

Figure 10.

II. BILIARY SECRETION

A. The liver secretes fluid and bile acids, and its ducts secrete fluid and
bicarbonate. These will be discussed primarily in the section on digestion
and absorption of lipids. As a component of the gastro-duodenal junctional
cluster, this secretion is important to the reduction in osmolarity and to the
neutralization of the acidic chyme.

709
GI Secretion 2 - Dr. Grider

III. INTESTINAL SECRETION (1000ml/day by enterocytes)

A. General Considerations

1. Cells of mucosal epithelium (enterocytes) secrete mucus, water, and


electrolyte.
2. Mucus serves to protect cells along mucosa from damage and also
provides lubrication.
3. In addition to the enterocytes, there are tubular glands in submucosa
of duodenum (Duodenal glands) that secrete a serous (water &
electrolyte) fluid which is similar to plasma except that it is higher in
HCO3-. These secretions enter invaginations of epithelium
(Intestinal crypts of Lieberkuhn) which empty into lumen of
duodenum.

B. Regulation of Secretion

1. Secretin increases the secretion from duodenal glands. This is


important to neutralize the acid being delivered into the duodenum
from the stomach.
2. Local neural mechanisms (components of the enteric nervous system
especially the submucosal plexus) activate intestinal enterocytes and
glands (mediated by vasoactive intestinal peptide (VIP), substance P,
calcitonin gene-related peptide ( CGRP) and Ach) and stimulate
sectretion.

C. Mechanism of secretion by enterocytes

1. The intestinal mucosa is folded into evaginations called villi ( a


villus; see figure next lecture) and invaginations call crypts (of
Lieberkuhn). The cells are continuous and originate from stem cells
in the base of the crypt and migrate towards the tip of the villus,
where they die and fall off ( total cell turnover time about 3-6 days).
During this migration, the cells change their characteristics and
expression of cell surface proteins (receptors and transporters).
While in the crypts, the enterocyte is generally thought of as a net
secretory cell, but as it moves up the villus it is transformed into a
net absorptive cell and no longer secretes large amounts of water
and electrolyte. (Figure 11.)

710
GI Secretion 2 - Dr. Grider

Figure 11.

2. The secretion of water by the crypt enterocyte is generally considered


to be coupled to the secretion of chloride ion via a Cl ion channel, the
cystic fibrosis transmembrane regulator (CFTR) (Figure 12). This
is the same protein that is defective in the clinical entity, cystic
fibrosis.
3. The process is illustrated below. The enterocyte actively transports 2
Cl ions into the basolateral side of the cell by the
sodium/potassium/chloride ion cotransporter. Chloride ion is then
secreted at the apical (lumenal) side via an electrogenic Cl ion channel
(CFTR). The sodium ion moves into the lumen via the paracellular
route and is driven by the electronegative potential gradient. Water
follows the movement of these ions into the lumen. The potassium
ions are removed by transport across the basolateral surface of the cell.

711
GI Secretion 2 - Dr. Grider

Figure 12.

4. The secretory process is greatly enhanced by hormones (e.g. secretin)


and neurotransmitters (VIP) that elevate cAMP. It is believed that this
effect is mediated by phosphorylation of the CFTR by cAMP-
dependent protein kinase (PKA). Cholera toxin is able to irreversibly
activate the G protein (Gs) coupled to adenylate cyclase, thereby
increasing cellular cAMP and maintaining this channel open. The
result is an intense secretion of Cl/Na and water leading to life-
threatening diarrhea. World-wide, this is one of the leading causes of
death.

D. Neural Regulation of secretion.

1. Just as the smooth muscle was regulated by enteric neurons of the


myenteric plexus, the secretion of the enterocytes is regulated by
enteric neurons primarily from the submucosal plexus. These neurons
receive input from extrinsic nerves (vagus) and from other neurons in
the myenteric plexus and project axons that terminate close to the
enterocytes. (Figure 13)

712
GI Secretion 2 - Dr. Grider

Figure 13.

2. The neural regulation of enterocyte secretion is governed by a


secretory reflex analogous to the peristaltic reflex (Figure 14).
Mechanical or chemical stimuli are sensed by a sensory neuron
(IPAN: Intrinsic Primary Afferent Neuron) with a cell body in the
submucosal plexus. The neurotransmitter of this neuron is Calcitonin
gene-related peptide (CGRP). The IPAN may actually be activated
by a paracrine agents such as serotonin (5-HT) and Prostaglandins
released in response to the luminal stimulus.

The IPAN activates a reflex involving excitatory cholinergic neurons


in the myenteric plexus which are connected cholinergic interneurons
in the submucosal plexus and to Secretomotor neurons in the
submucosal plexus which innervate the enterocytes. There are two
types of secretomotor neurons: one contains primarily Vasoactive
Intestinal Peptide (VIP) and one primarily contains Acetylcholine

713
GI Secretion 2 - Dr. Grider

(Ach). The components of the reflex are modified by excitatory input


from preganglionic vagal neurons and inhibitory input from enteric
interneurons neurons containing somatostatin and opioid peptides
and from extrinsic sympathetic postganglionic neurons.

Figure 14.

IV. COLONIC SECRETION

A. Mostly mucus secretion.

B. Small amount of serous secretion which is high in HCO- and K+.

V. INTERDIGESTIVE SECRETION

714
GI Secretion 2 - Dr. Grider

A. Just as described for myoelectrical activity recorded from smooth muscle,


there is a similar pattern of secretion of gastric, biliary and pancreatic fluid
and enzymes, periodically during the interdigestive period. This functions
to help keep the gut clear and clean during this period as well as ready to
receive the next meal. (Figure 15)

Figure 15.

VI. STUDY QUESTIONS

1. A meal high in carbohydrates results in the secretion of

A. amylase enzymes only.


B. lipase enzymes only.
C. Protease enzymes only.
D. Hydrochloric acid only.
E. A mixture of all pancreatic enzymes.

2. Cholecystokinin

A. is secreted by cells of the pancreas.


B. can be released by vagal stimulation.
C. causes contraction of gallbladder and Sphincter of Oddi.
D. stimulates the release of salivary amylase.
E. is located in cells of the gastric antrum.

3. The bicarbonate secretion of the pancreas is

715
GI Secretion 2 - Dr. Grider

A. increased at low flow rates.


B. stimulated mostly by CCK.
C. occurs primarily from duct cells.
D. acts to further stimulate the release of secretin.
E. is important to the conversion of pepsinogen to pepsin.

4. In the intestinal crypt, the enterocyte is mainly

A. an endocrine cell.
B. a paracrine secreting cell.
C. an absorptive cell.
D. a secretory cell.
E. a bile producing biliary cell.

Answers 1=E, 2=B, 3=C, 4=D

716
Na+ Transport 1 and 2 - Dr. Costanzo

Na+ Transport 1 and 2


Linda Costanzo, Ph.D.

OBJECTIVES:

After studying this lecture, the student should understand:

1. The terminology applied to single nephron function, including the meaning of


TF/P and the double ratio.
2. How to calculate fractional water reabsorption using TF/Pinulin
3. The pattern of sodium reabsorption along the nephron.
4. Features of and transporters involved in sodium reabsorption in the early proximal
tubule, late proximal tubule, thick ascending limb, early distal tubule, and late
distal tubule and collecting ducts.

I. TERMS TO DESCRIBE SINGLE NEPHRON FUNCTION

Up to this point, we have discussed primarily whole kidney function (e.g., GFR,
urine, clearance, excretion). Now we will turn our attention to the nephron, which
is the functional unit of the kidney. There is a specific vocabulary of the nephron,
with terms analogous to that of the whole kidney (e.g., tubular fluid is analogous
to urine).
• [TF]x is the concentration of substance X in tubular fluid. (Tubular fluid is
the fluid inside the nephron......also called luminal fluid.)

• [P]x is the concentration of substance X in plasma and is considered to be


constant.

• SNGFR is the single nephron glomerular filtration rate.

717
Na+ Transport 1 and 2 - Dr. Costanzo

• [TF/P]x is the concentration of substance X in tubular fluid relative to the


concentration in plasma. There are three possibilities for the value of this
ratio, which are explained as follows:

• [TF/P]X = 1.0. X has not been reabsorbed or secreted (all freely


filtered substances in Bowman's space), or X is reabsorbed in
proportion to water (e.g., Na in proximal tubule). For example,
[TF/P]X =1.0 for all freely filtered substances in Bowman’s space
(no reabsorption or secretion has taken place yet). For another
example, [TF/P]Na = 1.0 throughout the proximal tubule because
Na+ is reabsorbed in exact proportion to water.

• [TF/P]X < 1.0. X is reabsorbed more than water, causing the


concentration of X in tubular fluid to fall below that in plasma. For
example, [TF/P]glucose starts at 1.0 in Bowman’s space, but then
falls below 1.0 along the proximal tubule as glucose is reabsorbed
more than water.

• [TF/P]X > 1.0. X is reabsorbed less than water or X is secreted. If


X is reabsorbed less than water (or if X is secreted into tubular
fluid), the concentration of X in tubular fluid rises above that in
plasma. For example, [TF/P]urea is > 1.0 in cortical collecting ducts
in the presence of ADH because water is reabsorbed and urea is
not. For another example, [TF/P]K is > 1.0 in cortical collecting
ducts because this part of the nephron secretes K+.

718
Na+ Transport 1 and 2 - Dr. Costanzo

• [TF/P]inulin is the concentration of inulin in tubular fluid relative to the


concentration of inulin in plasma. This specific [TF/P]X ratio is used to
measure water reabsorption since inulin, once filtered, is "inert" (i.e., is
neither reabsorbed or secreted). Thus, the amount of inulin in tubular fluid
is constant along the nephron (because inulin is not reabsorbed or
secreted) but the concentration of inulin in tubular fluid is determined by
how much water remains; as water is reabsorbed, the tubular fluid
concentration of inulin increases. For example, if 50% of the filtered water
has been reabsorbed, then the tubular fluid inulin concentration doubles
and the [TF/P]inulin = 2.0. (Don't forget that the "P" in TF/P is always
assumed to be constant.) Calculate water reabsorption with this ratio as
follows:

Fraction of filtered water reabsorbed = 1 - 1


[TF/P]inulin

For example, if tubular fluid is sampled at the end of the proximal tubule,
and the [TF/P]in ratio is measured as 3.0, what fraction of the filtered water
has been reabsorbed up to that point? What fraction of the filtered water
remains in the lumen of the nephron?

Fraction of filtered water reabsorbed = 1 - 1


[TF/P]in
= 1 - 1/3
= 2/3, or 66.7% reabsorbed

If 2/3 of the filtered water has been reabsorbed, then 1/3 of the filtered
water remains in the lumen of the neprhon.

• [TF/P]x
[TF/P]inulin, or the "double ratio" is the fraction of the filtered load of a
substance remaining in the nephron at any point. If the "double ratio" is
0.3, then 30% of the filtered load of the substance remains in tubular fluid,
and 70% of the filtered load must have been reabsorbed.

If you’re wondering why the double ratio corresponds to fraction of


filtered load of a substance remaining in the nephron, here’s the
derivation (in italics, just FYI...):

719
Na+ Transport 1 and 2 - Dr. Costanzo

% of filtered load remaining = excretion rate of x at any point in nephron


divided by filtered load of x in nephron

Excretion rate at any point in nephron = [TF]x x V

SNGFR = single nephron GFR= [TF]inulin xV


[P]inulin

Filtered load of X = SNGFR x [P]x = [TF]inulin x V/[P]inulin x [P]x

Substituting, and cancelling V:

% of filtered load remaining = [TF/P]x/[TF/P]inulin

We will use [TF/P]X and the double ratio together to describe how
substances are handled in the nephron. For example, if, at the end of the
proximal tubule, the double ratio for Na+ is 0.33 and [TF/P]Na is 1.0, we
would say that 33% of the filtered Na+ remains in the nephron at the end
of the proximal tubule, that 67% of the filtered Na+ was reabsorbed by the
proximal tubule, and that this Na+ reabsorption must have been in exact
proportion to water reabsorption (since [TF/P]Na was 1.0).

II. OVERALL NA+ BALANCE

Na+ is the major ECF cation and, with accompanying anions Cl- and HCO3-,
constitutes the major ECF solute. As we have already discussed, the amount of
Na+ in ECF determines ECF volume and therefore also determines blood volume
and blood pressure. Thus, regulation of Na+ balance is the most important
function of the kidneys. On an average daily diet of 150 mEq of Na+ ingested, the
kidneys must excrete 150 mEq of Na+ to keep us in Na+ balance (neglecting small
non-renal losses such as via sweat). If the kidneys excrete less Na+ than is
ingested, then we are in positive Na+ balance; if the kidneys excrete more Na+
than is ingested, we are in negative Na+ balance.

720
Na+ Transport 1 and 2 - Dr. Costanzo

Figure 1. Na+ handling in the nephron. Arrows show locations of Na+


reabsorption; numbers are percentages of the filtered load reabsorbed or excreted.

Na+ is reabsorbed along the nephron as follows: 67% of the filtered load in the
proximal tubule, 25% in the thick ascending limb of Henle, 5% in the early distal
tubule, and 3% in late distal tubule and collecting duct. Cumulatively, this is more
than 99% of the filtered load reabsorbed, leaving less than 1% of the filtered load
to be excreted. (For a person in Na+ balance, 1% of the filtered load excreted
corresponds to the daily Na+ excretion that would equal daily Na+ ingestion.)

III. PROXIMAL CONVOLUTED TUBULE: EARLY AND LATE PROXIMAL

The entire proximal convoluted tubule reabsorbs 67% or 2/3 of the filtered Na+. A
major feature of proximal Na+ reabsorption (and total solute reabsorption as well)
is that it is linked directly to water reabsorption. Thus, Na+ (and solute)
reabsorption is proportional to water reabsorption and we call the process
isosmotic. The basis for isosmotic reabsorption will be explained later in the
lecture.

Proximal tubule is divided between an "early" part (first half, nearest the
glomerulus) and "late" part (second half). The cellular mechanisms for Na+
reabsorption are different in the two parts, so they will be discussed separately.

721
Na+ Transport 1 and 2 - Dr. Costanzo

A. Early proximal tubule

Figure 2. Cellular mechanisms of Na+ reabsorption in the early proximal tubule. The transepithelial potential
difference is the difference between the potential in the lumen and the potential in blood, -4 mV. ATP,
Adenosine triphosphate.

Early proximal tubule has the following features:

• Na+-glucose, Na+-amino acid, and Na+-phosphate cotransporters in


the luminal membrane
• Na+-H+ exchange in the luminal membrane (linked to filtered
HCO3- reabsorption, which will be covered in the acid-base portion
of the course)
• Preferential reabsorption of HCO3- over Cl- (as the anion
accompanying Na+ reabsorption)
• Na+- phosphate cotransport is inhibited by parathyroid hormone
(PTH) and responsible for the “phosphaturic” effect of PTH.
• Lumen-negative transepithelial potential difference due to Na+-
glucose cotransport

722
Na+ Transport 1 and 2 - Dr. Costanzo

• Always isosmotic reabsorption


• [TF/P]Na and [TF/P]osm = 1.0

B. Late proximal tubule

Figure 3. Cellular mechanisms of Na+ reabsorption in the late proximal tubule.


The transepithelial potential difference is +4 mV. ATP, Adenosine triphosphate.

Late proximal tubule has the following features:

• High luminal Cl- concentration (created by preferential


reabsorption of HCO3- in early proximal)
• Cl- reabsorption by Cl--formate exchange in luminal membrane
and by Cl- diffusion between cells (down Cl- concentration
gradient)
• Lumen positive transepithelial potential difference created by Cl-
diffusion
• Always isosmotic reabsorption
• [TF/P]Na and [TF/P]osm = 1.0

IV. PROXIMAL TUBULE: ISOSMOTIC REABSORPTION,


GLOMERULOTUBULAR BALANCE

Solute (mainly Na+, HCO3-, Cl-, glucose and amino acids) and water reabsorption
are always proportional to each other in proximal tubule. They are linked
mechanistically, so the reabsorption process is isosmotic. A consequence of this

723
Na+ Transport 1 and 2 - Dr. Costanzo

proportional, isosmotic process is that, along the entire proximal tubule,


[TF/P]Na = 1.0 and [TF/P]osm = 1.0. In other words, the concentration of Na+ and
total osmolarity do not change in proximal tubule fluid, even though lots of
sodium and total solute are reabsorbed. The mechanism of isosmotic
reabsorption is explained in the figure below.

Figure 4. Mechanism of isosmotic reabsorption in the proximal


tubule. Dashed arrows show the pathways for reabsorption;
circled numbers correspond to the text. πc, Peritubular capillary
colloid osmotic pressure.

Solute crosses the luminal membrane (as described for the early and late proximal
tubule). The luminal membrane is permeable to water, so water follows solute
into the cell. Na+ is pumped out across the lateral membranes into the lateral
intercellular space by the Na+-K+ pump; again, water follows. In the lateral space,
an isosmotic fluid accumulates. From that point on, reabsorption of this fluid is
driven by Starling forces across the peritubular capillary. The major Starling force
driving proximal tubule reabsorption is the high oncotic pressure, πc, of
peritubular capillary blood (recall how this high πc is created!).

Glomerulotubular balance is a regulatory feature of the proximal tubule. It says


that glomerular filtration of Na+ (and solute and water) are balanced by
reabsorption (i.e. by the tubule), resulting in constant fractional reabsorption of
67%. This balance or constancy ensures that ECF Na+ content, and therefore ECF
volume, will be maintained. The physiologic parameter that maintains
glomerulotubular balance is the oncotic pressure of peritubular capillary blood
(πc). Thus, if for some reason GFR were to spontaneously increase, there would
be increased filtration of fluid out of glomerular capillaries, increased πc in

724
Na+ Transport 1 and 2 - Dr. Costanzo

peritubular capillary blood, and increased driving force for proximal reabsorption,
i.e., constant fractional reabsorption.

Normally, glomerulotubular balance prevails and reabsorption is 67% of the


filtered load. However, changes in ECF volume (volume contraction and volume
expansion) can "override" and change the percentage reabsorbed. In ECF volume
expansion, fractional reabsorption is decreased (excretion is increased) and in
ECF volume contraction, fractional reabsorption is increased (excretion is
decreased).

Figure 5. Effects of ECF volume expansion (A) and ECF volume contraction (B) on isosmotic fluid
reabsorption in the proximal tubule. Changes in Starling forces in the peritubular capillary blood are responsible
for the effects. πc, Peritubular capillary colloid osmotic pressure; Pc, peritubular capillary hydrostatic pressure.

V. THICK ASCENDING LIMB OF HENLE'S LOOP

Thick ascending limb of Henle's loop reabsorbs 25% of the filtered Na+. In
contrast to the proximal tubule, which always reabsorbs isosmotically, thick
ascending limb reabsorbs solute without water. Because it is impermeable to
water, it also is called the diluting segment (if solute is reabsorbed and water is
left behind, the tubular fluid becomes dilute). Therefore, both [TF/P]Na and
[TF/P]osm are <1.0 in the thick ascending limb.

725
Na+ Transport 1 and 2 - Dr. Costanzo

Figure 6. Cellular mechanism of Na+ reabsorption in the thick ascending limb of the loop of
Henle. The transepithelial potential difference is +7 mV. ATP, Adenosine triphosphate.

Thick ascending limb has the following features:

• Na+-K+-2Cl- cotransporter in luminal membrane


• Cells are impermeable to water and tubular fluid is diluted as Na+ and
solute are reabsorbed
• Also called diluting segment
• Lumen positive transepithelial potential difference due to action of
Na+-K+-2Cl- cotransporter. This feature is puzzling at first, because it
seems that the cotransporter would be electroneutral. However, note on the
figure that some K+ diffuses from cell back into lumen, and consequently,
net negative charge enters cell, leaving the lumen with a positive charge.
• Furosemide and other loop diuretics inhibit the Na+-K+-2Cl- cotransporter
and thereby inhibit
Na+ reabsorption, the basis for their diuretic action
• [TF/P]Na and [TF/P]osm < 1.0
• Important for countercurrent multiplication (another lecture)
• Ca2+ and Mg2+ are reabsorbed between the cells, driven by the lumen-
positive potential. Loop diuretics abolish the lumen-positive and inhibit
Ca2+ reabsorption

VI. DISTAL TUBULE AND COLLECTING DUCTS

Together, the distal tubule and collecting ducts reabsorb 8% of the filtered Na+. In
terms of cellular mechanisms, these segments are divided into early distal tubule
and late distal tubule/collecting duct.

726
Na+ Transport 1 and 2 - Dr. Costanzo

A. Early distal tubule

Early distal tubule reabsorbs about 5% of the filtered Na+. Like thick
ascending limb, the cells are impermeable to water and it is called the
cortical diluting segment.

As a result both [TF/P]Na and [TF/P]osm are < 1.0.

Figure 7. Cellular mechanism of Na+ reabsorption in the early distal tubule. The
transepithelial potential difference is -10 mV. ATP, Adenosine triphosphate.

Early distal tubule has the following features:

• Na+-Cl- cotransporter in luminal membrane


• Cells are impermeable to water and tubular fluid is diluted as
solute is reabsorbed
• Called cortical diluting segment
• Thiazide diuretics (chlorothiazide; hydrochlorothiazide) inhibit
the Na+-Cl- cotransporter, which is the basis for their diuretic
action
• Ca2+ reabsorption is active and stimulated by parathyroid hormone
(PTH) and thiazide diuretics

B. Late distal tubule/collecting duct

Together, the late distal tubule and collecting duct reabsorb 3% of the
filtered Na+. There are two cell types interspersed along these segments,

727
Na+ Transport 1 and 2 - Dr. Costanzo

the principal cells and the intercalated cells. For completeness, both cell
types will be described here, although it is the principal cells that reabsorb
Na+. These segments are notable in that they are the site of action of
important hormones: aldosterone (regulates Na+ reabsorption, K+
secretion, and H+ secretion) and ADH (regulates water reabsorption)

1. Principal cells

Figure 8. Cellular mechanism of Na+ reabsorption in the principal cells of the late
distal tubule and collecting duct. The transepithelial potential difference is -50 mV.
ATP, Adenosine triphosphate.

Principal cells have the following features:

• Na+ channels in luminal membrane (called EnaC, for epithelial


Na+ channel)
• Aldosterone induces these Na+ channels and thereby increases Na+
reabsorption
• Na+ reabsorption is fine-tuned in the principal cells by the action of
aldosterone
• Water reabsorption is variable, via aquaporin2 (AQP2) in the
luminal membrane that are regulated by ADH
• K+ secretion via K+ channels (discussed in next lecture)
• K+-sparing diuretics such as spironolactone and amiloride inhibit
the action of aldosterone, block the Na+ channels, decrease which
is the basis for their diuretic action

2. Intercalated cells have the following features

728
Na+ Transport 1 and 2 - Dr. Costanzo

• Reabsorption of K+ via the H+- K+ ATPase (under conditions


of low K+ diet)
• Secretion of H+ via the H+ ATPase, which is increased by
aldosterone

Figure 9

VII. PRACTICE QUESTIONS

1. If [TF/P]Na+/[TF/P]inulin is 0.33 at the end of the proximal convoluted


tubule, what fraction of the filtered load of Na+ has been reabsorbed?

729
Na+ Transport 1 and 2 - Dr. Costanzo

2. If [TF/P]inulin = 3.0, what percentage of the filtered H2O has been


reabsorbed?

3. How will an increase in filtration fraction alter isosmotic reabsorption in


the proximal tubule and why?

4. How do changes in ECF volume alter isosmotic reabsorption in the


proximal tubule?

5. How does the Na+-K+-2Cl- cotransporter produce a lumen-positive


transepithelial potential difference in the thick ascending limb?

6. How does reabsorption of solute (NaCl) in the thick ascending limb cause
dilution of tubular fluid? What is [TF/P]osm of fluid leaving the thick
ascending limb?

7. What are major differences between the transport functions of the early
distal tubule and the late distal tubule/collecting duct?

8. Which of the following exhibits increased Na+ reabsorption in the late


distal tubule and collecting ducts?
Person with syndrome of inappropriate ADH
Person with aldosterone-secreting tumor
Person taking a thiazide diuretic
Person taking spironolactone

ANSWERS

1. 67%
2. 67%
3. Increases it (increased filtration from glomerular capillaries increases πc)
4. Increased ECF volume decreases isosmotic reabsorption; decreased ECF
volume increases isosmotic reabsorption
5. See notes
6. In thick ascending limb, more solute is reabsorbed than water, diluting the
tubular fluid. [TF/P]osm <1.0
7. Early distal, Na+-Cl- cotransport, no H2O; late distal/collecting duct, Na+
channels, H2O depends on ADH, aldosterone stimulates Na+ reabsorption
and K+ secretion
8. Person with syndrome of inappropriate ADH – no
Person with aldosterone-secreting tumor – yes
Person taking a thiazide diuretic – no
Person taking spironolactone – no

730
Digestion and Absorption 1 - Dr. Grider

Digestion and Absorption 1


Jack Grider, Ph.D.

OBJECTIVES:

1. Compare the specializations leading to increased intestinal surface area.


2. Compare and contrast the factors along the intestinal wall that influence intestinal
absorption.
3. Discuss the mechanism of carbohydrate digestion and absorption.
4. Compare the characteristics of protein absorption with those of carbohydrate
absorption.

Suggested Reading: Berne & Levy: 585-592;608-620

I. SPECIALIZED ANATOMICAL STRUCTURES

A. Specializations to increase surface area

1. There is no absorption in esophagus and little absorption in stomach.


The vast majority of absorption occurs in the small intestine which has
specialized structures to increase absorptive surface area of mucosa.
Most nutrients are absorbed before chyme reaches the ileum. The
colon is responsible mainly for final removal of electrolyte and water,
although there is also absorption of short chain fatty acid produced by
colonic bacteria.

731
Digestion and Absorption 1 - Dr. Grider

Figure 1. Amplification of the intestinal surface area by intestinal folds (plicae


conniventes), villi and microvilli. The numbers indicate the factor by which the
surface area is amplified over a flat surface. All together they amplify the
surface area by approximately 600 fold.

2. Intestinal specializations (Figure 1).


3. Folds in mucosa and submucosa called Circular Folds increase
surface area 3 fold.
4. Mucosa itself has folds called villi which increase surface area another
10 fold . Epithelial cells have folds on lumenal surface called
microvilli which increase surface area 20 fold. This is the major
specialization.
5. Net increase in surface area is 600 fold.

B. Structure of Villus (Figure 2)

1. An extensive capillary network in the villus removes absorbed


nutrients rapidly so as to maintain concentration gradients from lumen
to blood.
2. An extensive lymphatic network of lacteals removes lipids rapidly to
maintain gradient a for lipid absorption.
3. The innervation provides a mechanism to regulate secretion by
epithelial cells.

732
Digestion and Absorption 1 - Dr. Grider

4. Smooth muscle cells contract to allow villi to slowly move in the


lumen.
5. As noted earlier, there is a gradient of cell function in the villus.
Enterocytes on the tip of the villus are net absorptive cells whereas
those in the crypt are net secretory cells.

C. Brush border

1. Epithelial cells have enzymes (Brush Border enzymes) on the outer


surface of microvilli.
2. These enzymes and the mucous which lines the wall of the intestine
make up the brush border.
3. This structure serves to protect the epithelial cells and also perform
final digestion of carbohydrates and proteins into monosaccharides and
amino acids.

Figure 2.

D. General Characteristics of Absorption

1. Nutrients are digested in the lumen and must be absorbed from the
lumen, across the mucosa and epithelial cells, and into blood or lymph
for distribution to the rest of the body.
2. One of the major barriers to absorption of nutrient and ions is the
brush border area and the Unstirred Layer (Figure 3). This unstirred
layer is adjacent to the brush border and consists of mucous, secretory

733
Digestion and Absorption 1 - Dr. Grider

IgA, and brush border enzymes. This region is not mixed well by
segmental contractions, is thick (viscous), hydrophobic, and has an
alkaline pH.

Figure 3.

3. Movement of nutrient across the mucosa can be by the Paracellular


Route (between enterocytes) or by the Transcellular route (across
the enterocyte) (Figure 4). The paracellular route is passive and
depends on the leakiness of the intestine. The proximal intestine
(duodenum) is very leaky and becomes tighter distally. The role of the
paracellular route follows a similar gradient (more important proximal
and less so distally).

734
Digestion and Absorption 1 - Dr. Grider

Figure 4.

4. The Transcellular route can be active or passive movement of


nutrient or ions and occurs by multiple mechanism including
Channels, Carrier molecules, Exchangers, or Pumps in the Apical
or Basolateral Membrane. (Figure 5)

Figure 5.

735
Digestion and Absorption 1 - Dr. Grider

5. Most absorption occurs in Duodenum and Jejunum. The ileum has


specialized absorptive mechanism for Bile Salts, and vitamin B12-
intrinsic factor complex. Indigestible material, starch in the form of
Fiber, water and electrolytes enter the colon for final processing.

II. DIGESTION AND ABSORPTION OF CARBOHYDRATES

A. Dietary carbohydrates

1. Most ingested carbohydrates are in the form of starch. This is a


polymer of glucose molecules linked by alpha-1,4-linkages and by
alpha-1,6 linkages at branch points.
2. Lesser amounts ingested as sucrose (fructose and glucose) and
lactose (glucose and galactose).
3. Cellulose is a glucose polymer of beta-1,4-linkages. These are not
digestible by enzymes of gut.

Figure 6.

B. Digestion

1. Ptyalin: Salivary amylase begins the process in oral cavity. This


alpha-amylase breaks starches into smaller polymers of glucose.
Ptyalin has a neutral pH optimum and is inactive at acidic pH. Some
CH2O digestion continues in the stomach in the center of bolus or
wherever pH remains neutral. (Figure 6)
2. Pancreatic amylase: This is also an alpha-amylase which acts at
alpha-1,4-linkages. Most digestion is a result of the action of this
enzyme. The result is fragments of 2 glucose molecules (maltose), 3
glucose molecules (maltotriose), and larger polymers containing the
branch points (alpha-limit dextrins).
3. Brush border enzymes: responsible for final digestion to
monosacharides. Maltase converts maltose and maltotriose into
glucose, sucrase converts sucrose into glucose and fructose, and
lactase converts lactose into glucose and galactose. Isomaltase

736
Digestion and Absorption 1 - Dr. Grider

converts limit-dextrins into glucose, maltose and oligosaccharides.


(Figure 7)

C. Absorption (Figure 7)

1. Glucose: absorption is by a Na+-Linked carrier in the membrane of


Epithelial Cells. The carrier protein is expressed only in mature
epithelial cells (upper villus) and is termed SGLT1 (Sodium-Glucose
Linked Transport Protein 1). The SGLT1 has a binding site for Na+ .
Once Na+ binds, glucose then binds and both are transported into the
cell. Na+ is then actively pumped out of the basolateral membrane by
a Na/K ATPase pump. Glucose is removed at basal membrane by
facilitated transport using a carrier protein termed GLUT2 and
removed by diffusion into capillaries of villus. Overall absorption is
active due to Na+ transport out of epithelial cell and is energy
dependent.
2. Galactose: Uses the same Na+ linked carrier on the apical membrane
(SGLT1) and the same basolateral carrier (GLUT2) as glucose.
3. Fructose: Enters the epithelial cell by facilitated diffusion which uses
a different carrier (termed GLUT5) than glucose/galactose and which
is not coupled to Na+ . Once in cell, fructose leaves the basal side by
facilitated diffusion using the same carrier as glucose and galactose
(GLUT2). This is not energy dependent.

D. Indigestible Starch

1. Indigestible Starch (Fiber) passes into the colon along with any
undigested starch. This is fermented by colonic bacteria to form Short
Chain Fatty Acids, especially acetate, propionate, and butyrate.
These are easily absorbed passively by colonic mucosal cells. It is
noteworthy that butyrate is the main energy source for coloncytes.

737
Digestion and Absorption 1 - Dr. Grider

Figure 7.

III. DIGESTION AND ABSORPTION OF PROTEIN

A. Dietary protein

1. Protein is polymer of amino acids.


2. Protein in the gut is derived from food, mucus, enzymes, and
desquamated cells.
3. Up to 60gram/day protein digested and absorbed by gut.

B. Digestion (Figure 8)

1. Pepsin: initial digestion of protein is by pepsin in stomach. This


enzyme has an acidic pH optimum and becomes inactive in neutral

738
Digestion and Absorption 1 - Dr. Grider

pH of intestine. Pepsin converts large peptides into smaller


peptones and polypeptides.
2. Proteolytic enzymes of pancreas: as listed earlier, these include
endopeptidases (trypsin & chymotrypsin) and exopeptidases
(carboxypeptidases & aminopeptidases). These continue the
digestion, converting protein to small peptides and amino acids.
3. Brush border peptidases: these are located in the apical
membrane/brush border and convert small peptides to
oligopeptides (di-, tri-, and tetra-peptides) and amino acids.

Figure 8.

C. Absorption

1. Small peptides: Di- and tripeptides are absorbed into the epithelial
cell by a carrier mediated transport system. This is the major method
of absorption and is ultimately energy dependent (active), as it
depends upon maintaining an electrochemical Na gradient. The actual
carrier is however a membrane protein that binds H+ and the di/tri-
peptide. Once in the epithelial cell, peptidases in the cytosol hydrolyze
peptides to amino acids. The AAs then passively diffuse (may be
carrier assisted transport) across the basal membrane and are removed
into capillaries of the villus. (Figure 2)
2. Amino acids: These are absorbed by membrane bound carrier
systems. There are multiple carriers which are active and coupled to
Na+, as described for the carbohydrate carrier. There are carriers for
neutral amino acids, imino acids (proline & hydroxyproline), for
phenylalanine & methionine, etc. In addition, there may be Na-
independent carriers for basic amino acids and for neutral amino acids.
(Figures 9 & 10)

739
Digestion and Absorption 1 - Dr. Grider

Figure 9.

Figure 10.

IV. BLOOD FLOW AND REMOVAL OF NUTRIENTS

As a result of the increased secretory activity, increased motility, and increased


absorption of water soluble nutrients, there is a general shift in blood flow away
from other areas of the body and into the gut (fig 8). The effect is mediated by
both neural reflexes and hormonal influences.

740
Digestion and Absorption 1 - Dr. Grider

Figure 11.

This is also necessary to increase delivery of nutrient rich blood from the
intestinal villi to the liver where carbohydrate and amino acids will be removed
and stored as glycogen (enterohepatic portal circulation). The process is
enhanced by the actions of glucose-dependent insulinotrophic factor (GIP).
GIP is release into the blood by the presence of glucose in the duodenal lumen.
GIP acts on pancreatic endocrine cells to secrete insulin which then acts to
increase uptake of nutrient glucose from the blood thereby blunting the rise in
blood glucose following a meal (Figure 12).

Figure 12.

V. STUDY QUESTIONS

1. Acinar cells

A. Are located in all compound glands.


B. Secrete ptyalin.
C. Contain granules in their apical cytoplasm.
D. Are stimulated to secrete bicarbonate by secretin.

741
Digestion and Absorption 1 - Dr. Grider

E. Synthesize bile acids.

2. Enterokinase

A. Digests carbohydrates.
B. Is concentrated in the gallbladder.
C. Prevents trypsin from being activated.
D. Is located in duodenal enterocytes.
E. Is absorbed by a carrier mediated mechanism.

3. Brush border enzymes

A. Are secreted by acinar cells.


B. Are located in or on intestinal enterocytes.
C. Are released in response to secretin.
D. Are mainly present in gastric chief cells.
E. Digest large starch molecules.

Answers: 1=C, 2=D, 3=B

742
Digestion and Absorption 2 - Dr. Grider

Digestion and Absorption 2


Jack Grider, Ph.D.

OBJECTIVES:

1. Describe the role of bile in lipid absorption.


2. Compare digestion and absorption of lipid with that of carbohydrate and protein.
3. Describe the mechanism of absorption of the major ions.
4. Differentiate the mechanisms of water and sodium absorption during
interdigestive and postprandial periods.

Suggested Reading: Berne & Levy pp. 585-592, 608-620

I. DIGESTION AND ABSORPTION OF LIPIDS

A. Required structures and secretions

1. The digestion and absorption of lipid presents a special problem. The


environment in the lumen of the intestine is an aqueous environment.
Lipids are not soluble in this environment. To make lipids soluble,
bile is added to the small intestine at the level of the duodenum. Bile
then acts as a detergent to make lipids soluble.
2. Bile is composed of bile acids, water & electrolytes, cholesterol,
phospholipids, proteins and wastes intended for excretion (eg.
Bilirubin, drug metabolites).
3. Bile acids are synthesized by the liver, concentrated in the
gallbladder and modified in the intestinal lumen as described below.

B. Functional Anatomy

1. The liver

a. The liver has a unique arrangement of parenchymal cells and


blood vessels. The functional unit is called the liver lobule and
is illustrated in figure 1.
b. The lobule is arranged around a central vein. At the periphery
of the lobule is the hepatic artery (arterial blood) and the portal
vein (nutrient-rich venous blood from the intestine). Blood
flows from these two vessels into sinusoids which are bounded
by hepatocytes that are 1-2 cells thick. Because the sinusoids
are fenestrated, there is direct contact and exchange between
the blood and the hepatocyte. This arrangements accounts for
the ability of the hepatocyte to remove large amounts of
material from the blood. Between adjacent hepatocytes is a

743
Digestion and Absorption 2 - Dr. Grider

network of openings called biliary canaliculi into which the


hepatocytes secretes liver bile which contains bile acids,
metabolites for excretion in bile such as bilirubin, and an
isotonic aqueous solution of Na+, K+, Cl-. This liver bile flows
through the canaliculi and empties into the bile duct at the
periphery of the lobule.

Figure 1.

2. The biliary system

a. The bile ducts from each lobule join into larger ducts forming
the bile or cystic duct. The ductal cells that line the early ducts
exchange HCO3- for Cl-. The secretion of HCO3- is increased
by the action of the hormone, secretin. Eventually, the bile duct
joins the pancreatic duct to form the common bile duct that
eventually enters the duodenum at the Sphincter of Oddi.
(Figure 2). This sphincter is contracted at rest causing bile to be
diverted into the gallbladder. The gallbladder has a highly
absorptive mucosa that removes water and can concentrate the

744
Digestion and Absorption 2 - Dr. Grider

bile about 10 to 20-fold during storage.

3. The enterohepatic circulation of bile. (Figure 2a & b)

a. On ingestion of a meal, the gallbladder contracts and the


Sphincter of Oddi relaxes to allow bile to flow into the
intestine. Both of these actions are mediated by neural (local
and vagal) reflexes as well as the hormone CCK which is
released by the presence of lipid and protein digestion products
in the duodenum.
b. Bile acts in the lumen of the upper small intestine to keep lipids
in solution.
c. The bile salts are then reabsorbed actively in the terminal ileum
and proximal colon. The enterocytes of the terminal ileum have
specialized sodium-linked carrier mechanisms for the
reabsorption of bile salts. In addition, they are reasbsorbed
passively throughout the intestine because they are lipid
soluble. The bile salts enter the portal venous blood and are
transported to the liver where they are then removed by the
hepatocytes and resecreted into the liver bile. During a normal
meal the entire bile salt pool (2.5g) is re-circulated twice. This
is known as the enterohepatic circulation of bile and is
illustrated in Figure 2. About 20% of the bile salts are lost into
the feces daily and are replaced by de novo synthesis of bile
acids by the hepatocytes.

745
Digestion and Absorption 2 - Dr. Grider

Figure 2a. Figure 2b.

C. Bile Acids and bile salts

1. The bile acids which are synthesized by hepatocytes from cholesterol


are called Primary Bile Acids: There are two of these: Cholic acid
and Chenodeoxycholic acid. (Figure 3)
2. In the intestine/colon, these primary bile acids are modified by bacteria
which dehydroxylate them. These modified bile acids are called
secondary bile acids. Cholic acid is converted to deoxycholic acid
and chenodeoxycholic acid is converted to lithocholic acid. (Figure 3)

Figure 3.

746
Digestion and Absorption 2 - Dr. Grider

3. Bile acids are usually also conjugated to an amino acid, either taurine
or glycine This is the result of the formation of a peptide bond between
the amino group of the amino acid and the carboxyl group of the bile
acid (figure 3). Thus, bile contains 4 bile acids conjugated to one of
these amino acids. Finally, the bile acids are usually found as sodium
salts and are often referred to as bile salts. For example the bile salt
‘Sodium taurocholate’ would be the sodium salt of cholic acid
conjugated to taurine.
4. Bile salts are amphipathic having both hydrophillic and hydophobic
portions. The sterol nucleus is hydrophobic and the hydroxyl groups,
peptide linkage and the amino acid conjugates are hydrophillic.
Conjugated bile salts are more hydrophillic than unconjugated forms.
The more hydroxyl groups (primary acids) the more hydrophillic and
therefore the better in keeping lipids in solution.

D. Micelles

1. In solution, bile salts form micelles spontaneously at any concentration


above a defined point called the Critical Micelle Concentration,
which is specific for each bile salt. In the intestine during a meal, the
concentration is usually above this level.
Micelles are molecular aggregates formed by orienting the bile salts so
that the hydrophobic parts are in the center and the hydrophillic parts
point out into the aqueous solution. In this way, lipids and non water
soluble molecules may be dissolved in the center of the micelle.
(Figure 4)

Figure 4.

E. Digestion of lipid

1. Dietary lipids: Most lipid ingested is in the form of triglycerides


(glycerol esterified to three fatty acids). Less is ingested as
phospholipid and sterol.
2. Upper gut: In the human, there is a small amount of lipase in saliva
(lingual lipase) and in gastric juice (gastric lipase). This only

747
Digestion and Absorption 2 - Dr. Grider

accounts for minor digestion. Although gastric motility does emulsify


lipid to some degree, the lipids tend to separate from the aqueous
portion of meal and empty from the stomach slower than the rest of the
meal.
3. Intestine: most digestion occurs here.(Figure 5A) As lipid enters the
duodenum it is emulsified into small droplets (0-5-1.0 micron) which
are stabilized by bile salts. Once emulsified, pancreatic lipase in
conjunction with co-lipase begins to act at the oil/water interface to
hydrolyze the ester linkages between glycerol and fatty acid. The 1 and
3 linkages are attacked first yielding 2 free fatty acids and a 2-
monoglyceride. These free fatty acids, 2-monoglyceride, phospholipid,
lysolecithins, cholesterol, and bile salts then combine to form micelles
(5nm diameter). As described earlier this particle solublizes the lipid
digestion products. The micelle then moves to the gut wall where the
free fatty acids, monoglycerides, lysolecithins, cholesterol, and
phospholipids diffuse out of the micelle, across the brush border and
into the epithelial cells. (note that in contrast to protein and
carbohydrates, there are no brush border enzymes for lipids and that
there is no carrier system for lipids in the epithelial cell.). Once the
micelle empties its contents, the micelle, now containing mainly bile
salts, moves back into the intestinal lumen to pick-up more lipid
digestion products and shuttle them back to the wall of the intestine.
This process repeats several times as the meal moves down the
intestine. In the terminal ileum, bile salts are reabsorbed as described
above.

F. Absorption (Figure 5 A & B)

Absorption across the lumenal membrane of the intestinal epithelial cell is by


passive diffusion down the concentration gradient.

1. Once inside the epithelial cell, the monoglycerides and free fatty acids
are resynthesized into triglycerides. This is an active process which
requires ATP and Co-A.
2. Triglycerides (80-90%), cholesterol (3%), phospholipids (10%),and B-
lipoprotein (5%) are then combined into another particle called the
chylomicron (60-750 nm diameter).
3. The chylomicron is then expelled from the epithelial cell by
exocytosis. It diffuses through the extracellular space and is removed
from the villus via the lacteals in the center of the villus. The lipid
finally enters the circulatory system at the thoracic duct.
4. Some glycerol which is not esterified to fatty acids passes directly
through the epithelial cell and is removed from the villus by diffusion
into capillaries.

748
Digestion and Absorption 2 - Dr. Grider

Figure 5a.

Figure 5b.

II. ABSORPTION OF WATER AND ELECTROLYTES

A. Sodium and water (Figure 6-8) The movement of sodium depends on the
regions of the gut and the state (postprandial or interdigestive) of the
gut.

749
Digestion and Absorption 2 - Dr. Grider

1. Nutrient coupled, electrogenic absorption predominates in the


postprandial state.
2. Electroneutral NaCl absorption predominates in the interdigestive
state.

Figure 6.

3. The absorption of water is coupled to the absorption of sodium. If you


can move sodium, water will follow. There are generally two routes:

a. Transcellular or across the cell


b. Paracellular or between cells (Major method)

Figure 7.

4. The paracellular route depends on the permeability between


enterocytes which is governed by the tight junctions between cells. In
the duodenum, these junctions are weak (so called “leaky
epithelium”) and most water enters here (about 5000 ml in duodenum

750
Digestion and Absorption 2 - Dr. Grider

& jejunum of the total 8300ml absorbed along the entire small
intestine). The junctions between cells become tighter distally so that
there is less water absorption in the ileum (about 1300 ml).
5. The “tightness” of the junction between the enterocyte can be modified
by hormones, neurotransmitters, and nutrients. Glucose increases
water absorption not only by activating the sodium-glucose
cotransporter, but also by decreasing the tightness (i.e. increasing the
“leakiness”) of the junction between enterocytes.
6. Net water and sodium absorption is energy (ATP)-dependent as a
result of the need to move sodium out of the cells at the basolateral
surface via the Na+/K+-ATPase pump.
7. Diffusion is passive at the luminal membrane and between cells.
8. The whole process depends on the rapid removal of Na+ by capillaries
which is critical to maintaining gradients of sodium and hydrostatic
pressure which drive sodium and water from mucosa to capillary.

Figure 8.

B. Other ions

1. Generally, movement of ions is a “net” difference between fluxes from


lumen to blood vs. blood to lumen, and can be variable in different
regions and under different conditions.

2. Potassium:

a. Passively absorbed in small intestine when concentrations


increase as a result of water absorption.

751
Digestion and Absorption 2 - Dr. Grider

b. In colon, normally there is a net secretion (active process) at


lumenal concentrations <25mM. Less commonly, if lumenal
concentrations exceed 25 mM, net absorption occurs in colon.

3. Cl- and Bicarbonate

a. Proximal duodenum demonstrates net secretion of bicarbonate


by a variety of sources. This is needed to neutralize acid in
chyme leaving stomach.
b. Jejunum demonstrates net absorption of both bicarbonate and
chloride ions.
c. Ileum&colon demonstrate chloride absorption and bicarbonate
secretion under normal circumstances.
[n.b.: From secretory lectures, the enterocytes in the Crypts of
Lieberkuhn secrete chloride ion via the CFTR channel as a
major way of secreting water]

4. Calcium

a. Actively absorbed throughout intestine.


b. Ca2+ binds to a protein in the brush border membrane which
may be a carrier.
c. Once inside of the cell, Ca2+ binds to a cytosolic calcium
binding protein which transports Ca2+ across the cell.
d. Ca2+ is pumped out of cell at the basolateral membrane by an
active process and into the capillary network.
e. Parathyroid hormone and Vitamin D increase the absorption
of Ca2+.

5. Iron

a. Iron absorption is promoted by acidic pH in stomach and by


Vitamin C.
b. Fe++ much more soluble then Fe+++.
c. Epithelial cell secretes a protein, transferrin which binds iron
in the intestinal lumen.
d. The iron/transferrin complex is then absorbed by the epithelial
cell by receptor-mediated endocytosis.
e. Iron is stored in epithelial cells and released to blood (bound to
transferring) as needed.
f. If not needed, iron is lost when cell is desquamated. This
storage of iron in the cell and loss by desquamation prevents
excess iron from entering the blood and causing toxic effects.
This is often referred to as a Mucosal Block.

752
Digestion and Absorption 2 - Dr. Grider

III. VITAMIN ABSORPTION

A. Water soluble vitamins

1. All are passively absorbed except vitamin C, biotin, Vitamin B1


and Vitamin B12.
2. Multiple specializations and binding proteins.
3. Absorption of Vitamin B-12 (cobalamin) (Figure 9)

Initially, Vitamin B-12 is released from chyme by the action of


acid and pepsin. The free Vitamin-12 is bound to a group of
binding proteins including haptocorrin which is secreted by
salivary glands and stomach. In the small intestine, pancreatic
proteases degrade these binding proteins and frees Vitamin -12. In
the alkaline environment of the intestine, Vitamin B-12 binds to
intrinsic factor which was secreted from Parietal cells of the
stomach. The Vitamin B-12-Intrinsic Factor complex is recognized
by and binds to a transporter on enterocytes of the distal ileum.
Once inside the enterocyte, the intrinsic factor is degraded
generating free Vitamin-12 again. This is bound to another binding
protein and transported out of the enterocyte into the portal blood.

Figure 9.

753
Digestion and Absorption 2 - Dr. Grider

B. Fat soluble vitamins (A,D,E,K)

1. Follow same route as lipids.


2. Solubilized in micelles and chylomicrons.

IV. STUDY QUESTIONS

1. Reabsorption of bile salts occurs

A. By passive diffusion
B. Mainly in the terminal ileum.
C. In the gallbladder.
D. Via chylomicron formation.
E. Only during slow flow rates.

2. Absorption of lipids is most effective in the presence of

A. Sodium ions.
B. Intrinsic factor.
C. Bile salts.
D. Enterokinase.
E. Vitamin C.

3. Absorption of water is most closely linked to

A. Sodium ions.
B. Potassium ions.
C. Chloride ions.
D. Bicarbonate ions.
E. Ammonium ions.

Answers: 1=B, 2=C, 3=A

754
K+ Transport - Dr. Costanzo

K+ Transport
Linda Costanzo, Ph.D.

OBJECTIVES:

After studying this lecture, the student should understand:

1. The major factors involved in internal potassium balance.


2. The pattern of potassium handling along the nephron.
3. The mechanism of potassium secretion in principal cells.
4. The major factors that regulate potassium secretion in principal cells.

This lecture is concerned with K+ homeostasis, which involves both internal K+ balance
(K+ shifts and external K+ balance (renal regulation).

I. INTERNAL K+ BALANCE (K+ shifts across cell membranes)

Most of the body's K+ is localized in the ICF. Shifts of K+ can occur across cell
membranes (between ECF and ICF) that produce changes in the ECF K+
concentration. (Recall the importance of the ECF K+ concentration in setting the
resting membrane potential of excitable cells such as nerve, skeletal muscle, and
heart muscle!). A shift of K+ from ICF to ECF produces hyperkalemia (increased
ECF or blood K+ concentration). A shift of K+ from ECF to ICF produces
hypokalemia (decreased ECF of blood K+ concentration). The factors that
produce such shifts are shown in the figure below.

Figure 1. Agents affecting internal K+ balance. Arrows show


directions of K+ movement into and out of cells. ECF,
Extracellular fluid; ICF, intracellular fluid.

755
K+ Transport - Dr. Costanzo

II. EXTERNAL K+ BALANCE (renal mechanisms)

The kidneys are responsible for overall K+ balance, meaning that daily urinary
excretion of K+ must equal daily K+ ingestion. This is actually a "difficult" task
because dietary K+ is so variable among individuals and in one individual from
day to day. The kidneys regulate K+ excretion by a combination of filtration,
reabsorption, and secretion. The rate of K+ secretion is the major variable
determining final urinary excretion.

Figure 2. K+ handling in the nephron. Arrows show location of K+ reabsorption or


secretion; numbers are percentages of the filtered load reabsorbed, secreted, or
excreted.

The proximal convoluted tubule reabsorbs about 67% of the filtered K+,
proportional to Na+ and water reabsorption. The thick ascending limb reabsorbs
another 20% of the filtered load, also roughly proportional to Na+ reabsorption.
These processes are not responsible for the "fine tuning" of K+ excretion. That is
the job of the distal tubule and collecting duct. The distal tubule and collecting
duct secrete K+ or, under special conditions, even may reabsorb K+. On a very
low K+ diet, K+ is reabsorbed in the distal tubule and collecting ducts (intercalated
cells). Under all other conditions, K+ is secreted (principal cells); this K+ secretion

756
K+ Transport - Dr. Costanzo

rate is variable and is regulated by factors such as diet, aldosterone, acid-base


balance, and tubular flow rate.

III. K+ REGULATION IN LATE DISTAL TUBULE AND COLLECTING


DUCT

The late distal tubule and collecting ducts are the site of renal K+ regulation. K+
reabsorption may occur in the intercalated cells on a low K+ diet. K+ secretion
occurs in the principal cells.

Figure 3. Cellular mechanisms of K+ reabsorption in α-intercalated cells (A) and


K+ secretion in principal cells (B) of the late distal tubule and collecting duct. ATP,
Adenosine triphosphate.

757
K+ Transport - Dr. Costanzo

A. K+ reabsorption (intercalated cells)

The mechanism of K+ reabsorption in α-intercalated cells is a H+-K+ ATPase


similar to that of the gastric parietal cells (see Panel A in above figure).

B. K+ secretion (principal cells)

The principal cell is most important for K+ regulation (same principal cell that
reabsorbs Na+ and water in this segment). Panel B in the above figure shows
that K+ is brought into the cell across the basolateral membrane by the Na+-K+
ATPase, which keeps the intracellular K+ high. Then K+ moves from the cell
into the lumen (i.e., is secreted) by passive diffusion down an electrochemical
gradient. The single most important principle to understand is that the
magnitude of K+ secretion is determined by the size of this electrochemical
gradient. Maneuvers that increase the electrochemical gradient increase K+
secretion and maneuvers that decrease the gradient decrease K+ secretion.

Figure 4.

Understand the following factors that alter K+ secretion:

• Diet. Increasing dietary K+ increases the amount of K+ in the principal


cells, increases intracellular K+ concentration, and increases the driving
force for K+ secretion. Decreasing dietary K+ decreases intracellular K+
and decreases K+ secretion.

• Aldosterone induces synthesis of Na+ channels in the luminal membrane


of principal cells. More Na+ channels means increased Na+ entry across
the luminal membrane, increased Na+ extrusion by the Na+-K+ ATPase,
and increased K+ entry by the Na+-K+ ATPAse. This increases

758
K+ Transport - Dr. Costanzo

intracellular K+ concentration and increases the driving force for K+


secretion. Complimenting this action, aldosterone also induces synthesis
of the K+ channels in the luminal membrane. Thus, the second action of
aldosterone is to increase K+ secretion.

• Acid-base. There is an “informal,” one-for-one H+- K+ exchange in the


basolateral membrane. Acidemia (increases in blood H+) cause H+ to enter
the principal cells across the basolateral membranes in exchange for K+
leaving, producing a decrease in intracellular K+ and decreased driving
force for K+ secretion. Alkalemia (decreases in blood H+) cause H+ to
leave the principal cells across the basolateral membranes in exchange for
K+ entering, leading to increased intracellular K+ and increased driving
force for K+ secretion.

• Flow rate. Increases in flow rate through the distal tubule and collecting
ducts, such as those caused by loop and thiazide diuretics) dilute the K+
concentration in the lumen, leading to an increase in the size of the K+
gradient across the luminal membrane, and increased K+ secretion. Thus, a
major side effect of diuretic therapy is increased K+ excretion, resulting in
hypokalemia.

• K+ -sparing diuretics antagonize all the actions of aldosterone, including


K+ secretion.

IV. PRACTICE QUESTIONS

1. What effects would the following have on serum [K+]: lack of insulin,
treatment with a β-adrenergic agonist, serum hyperosmolarity, metabolic
alkalosis?

2. Where is K+ reabsorbed in the nephron? Where is K+ secreted?

759
K+ Transport - Dr. Costanzo

3. Which of the following will cause an increase in K+ secretion? Low


dietary K+, hyperaldosteronism, metabolic alkalosis, increased delivery of
Na+ to the distal tubule (e.g. treatment with loop diuretic)?

4. A person has an aldosterone-secreting tumor (Conn's syndrome).


What effects would you predict on blood pressure and serum K+
concentration?
Will urine K+ excretion be relatively high, low, or normal?
Will urine Na+ be high, low, or normal?
How would this person be treated?

V. ANSWERS

1. Increase, decrease, increase, decrease

2. K+ reabsorption – proximal tubule and thick ascending limb; late distal


tubule and collecting ducts on low K+ diet only
K+ secretion – late distal tubule and collecting ducts

3. No, yes, yes, yes

4. Effects on blood pressure and serum K+ concentration? Elevated blood


pressure, decreased serum K+
Will urine K+ excretion be relatively high, low, or normal? High
Will urine Na+ be high, low, or normal? Initially low, then normal because
the volume expansion that results from increased Na+ reabsorption then
inhibits reabsorption in the proximal tubule, brings Na+ reabsorption and
excretion toward normal -- called "escape from aldosterone"
Treatment? Aldosterone antagonist such as spironolactone or amiloride,
then surgical removal of the tumor

760
Concentration & Dilution of Urine - Dr. Costanzo

Concentration and Dilution of Urine 1 and 2


Linda Costanzo, Ph.D.

OBJECTIVES:

After studying this lecture, the student should understand:

1. Responses to water deprivation.


2. Responses to water drinking.
3. Mechanisms involved in production of hyperosmotic (concentrated) urine.
4. How countercurrent multiplication contributes to the creation of a corticopapillary
gradient.
5. How urea recycling contributes to the creation of a corticopapillary gradient.
6. The role of countercurrent exchange in vasa recta.
7. The cellular actions of ADH.
8. Mechanisms involved in production of hyposmotic (dilute) urine.
9. How to calculate free water clearance.

I. REGULATION OF BODY FLUID OSMOLARITY

The mechanisms which regulate our body fluid osmolarity are those regulating
water reabsorption by the kidney. Most of the filtered water (about 2/3) is
reabsorbed isosmotically by the proximal tubule. During passage through the
loops of Henle, NaCl is reabsorbed without water. A variable amount of water is
reabsorbed by the distal tubules and collecting ducts. The amount of water
absorbed in these distal segments is controlled by antidiuretic hormone (ADH).
ADH secretion from the posterior pituitary is controlled by the osmolarity of
blood, via osmoreceptors in the anterior hypothalamus.

761
Concentration & Dilution of Urine - Dr. Costanzo

Figure 1. Responses to water deprivation. Circled numbers


correspond to the text. ADH, Antidiuretic hormone.

762
Concentration & Dilution of Urine - Dr. Costanzo

Figure 2. Responses to water drinking. Circled numbers correspond


to the text. ADH, Antidiuretic hormone.

763
Concentration & Dilution of Urine - Dr. Costanzo

When a subject is deprived of water (see Figure 1), his plasma osmolarity
increases. This stimulates the secretion of ADH from the posterior pituitary gland,
which in turn increases the water permeability of the distal tubules and collecting
ducts, thereby promoting the reabsorption of water.

Conversely, when an individual drinks a large volume of water (see Figure 2),
the consequent dilution of plasma leads to inhibition of ADH secretion, causing
decreased water permeability of distal tubules and collecting ducts and excretion
of the excess water.

Other agents or situations may influence ADH secretion. Alcohol inhibits ADH
secretion, causing a water diuresis. An important stimulus for ADH secretion is
decreased blood volume (via the "volume receptors" in the left atria); this
mechanism allows the body to conserve water in severe hemorrhage.

II. FORMATION OF HYPEROSMOTIC (CONCENTRATED) URINE

The ability to form hyperosmotic urine is associated with the presence of Loops
of Henle. Between various species, the longer the loop of Henle, the greater the
ability to concentrate the urine. Desert rodents have the longest loops of Henle
and the greatest concentrating ability. The role of the loops of Henle is to create a
large osmotic gradient from the outermost part of the kidney (cortex) to the
innermost part (papilla). This gradient is called the corticopapillary osmotic
gradient.

We shall consider the following components in the formation of hyperosmotic


urine: (1)Establishment of the corticopapillary gradient by the loops of Henle
(countercurrent multiplication). (2) The vasa recta which help maintain the
gradient. (3) ADH, which makes the distal tubule and collecting duct cells
permeable to water, allowing osmotic equilibration of tubular fluid with the
hyperosmotic interstitium.

764
Concentration & Dilution of Urine - Dr. Costanzo

A. Formation of hyperosmotic urine--overview

Figure 3. Mechanisms for production of hyperosmotic (concentrated) urine in the presence of antidiuretic hormone
(ADH). Arrows show location of water reabsorption; heavy outline shows water-impermeable portions of the nephron;
numbers are osmolarity of tubular fluid or interstitial fluid.

In the above diagram, the numbers are osmolarity of tubular fluid or


interstitial fluid. The heavy outline indicates water impermeable segments
of the nephron. The following events occur along the nephron.

1. In proximal tubule, 2/3 of the glomerular filtrate is reabsorbed


isosmotically. Fluid entering the loop of Henle has an osmolarity of
equal to that of glomerular filtrate, 300 mOsm/L.
2. In thick ascending limb of Henle, the cells are impermeable to water.
Here, NaCl is reabsorbed (by Na-K-2Cl cotransport) without water.
This process makes the surrounding medullary and papillary interstitial
fluid hyperosmotic to systemic plasma; the loops of Henle serve as
countercurrent multipliers to create this hyperosmolarity (see below).
The fluid which leaves the thick ascending limb is hyposmotic.
Because the thick ascending limb reabsorbs NaCl without water and
the exiting fluid is hyposmotic, it is called the "diluting segment".

765
Concentration & Dilution of Urine - Dr. Costanzo

3. The first portion of the distal tubule is also water-impermeable and is


called the "cortical diluting segment". It is not part of the loop of
Henle, so it does not participate in countercurrent multiplication. It
can, however, further dilute the tubular fluid.
4. Circulating ADH levels are high during antidiuresis, making the cells
of the late distal tubule and collecting duct permeable to water. Water
will flow from low osmolarity (in the tubule lumen) to high osmolarity
(surrounding interstitium) until osmotic equilibrium is achieved. The
maximum urine osmolarity will be equal to the interstitial osmolarity
at the tip of the papilla (about 1200 mOsm/L in humans).

B. Countercurrent multiplication (establishment of the corticopapillary


osmotic gradient

Figure 4. Mechanism of countercurrent multiplication in a loop of Henle. Circled numbers correspond to the
text; numbers are osmolarities of tubular fluid or interstitial fluid; arrows show the direction of fluid flow;
heavy outline shows water impermeability of the ascending limb.

766
Concentration & Dilution of Urine - Dr. Costanzo

The osmolarity of the fluid in Henle's loop is shown as a series of


discontinuous steps. In the initial state the loop is filled with fluid with
osmolarity of 300 mOsm/L. In Step 1, NaCl is reabsorbed without water
along the length of the thick ascending limb (called the "single effect")
rendering the interstitial fluid hyperosmotic and the thick ascending limb
fluid hyposmotic. The thin descending limb is permeable to water and so
water leaves the descending limb until its tubular fluid has the same
osmolarity as the adjacent interstitial fluid. Now there is a 200 mOsm/L
osmotic gradient between descending and ascending limbs. In Step 2,
fluid is shifted along the nephron, introducing new 300 mOsm/L fluid into
the descending limb and ejecting dilute fluid from the ascending limb. The
"single effect" creating a 200 mOsm/L gradient occurs again in Step 3.
Continuation of the process will multiply the gradient more, until the fluid
at the bend of the loop is 1200 mOsm/L.

C. Role of urea in the corticopapillary gradient

During antidiuresis, about 60% of the solute deposited in the medulla and
papilla is NaCl (via countercurrent multiplication). The remaining 40% is
urea. Deposition of urea in the medulla and papilla is aided by "urea
recycling" which diverts some urea from medullary collecting duct fluid
back into the loops of Henle. Urea reabsorption from collecting ducts is
high during antidiuresis in part because of differential effects of ADH on
urea and water permeabilities in the terminal nephron segments. They are:

1. ADH increases H2O permeability in the late distal tubule, outer and
inner medullary collecting ducts.
2. ADH increases urea permeability in the inner medullary collecting
ducts, but not in the late distal or outer medullary collecting ducts.

The consequences of this differential permeability effect are shown in the


following figure.

767
Concentration & Dilution of Urine - Dr. Costanzo

Figure 5. Mechanism of urea recycling from inner medullary collecting ducts. Circled
numbers correspond to the text. ADH, Antidiuretic hormone; [TF], tubular fluid
concentration.

ADH increases the water, but not urea, permeability of the late distal
tubule and outer medullary collecting duct; water is reabsorbed, but urea is
not. Consequently, the urea concentration of the collecting duct fluid
increases. In the inner medullary collecting ducts, the urea concentration
has become very high. Here, ADH does increase the urea permeability, so
urea diffuses out of the tubular fluid, down this steep concentration
gradient, into the surrounding interstitium. Thus, it becomes a major solute
in the inner medulla and papilla.

D. Countercurrent exchange in vasa recta

The vasa recta are the capillaries which supply the medulla and papilla of
the kidney with oxygen and nutrients necessary for active transport. They
also remove the water reabsorbed by the descending limbs of Henle and
the collecting ducts. If blood flow to these regions was very high, the
solutes accumulated by countercurrent multiplication and urea recycling
would be washed away. Dissipation of the corticopapillary gradient is
prevented because (1) blood flow through this region is low and (2) the
vasa recta act as countercurrent exchangers. The figure below
illustrates the principle of countercurrent exchange in vasa recta.

768
Concentration & Dilution of Urine - Dr. Costanzo

Figure 6. Countercurrent exchange in the vasa recta. Solid blue arrows


show the direction of solute movement; dashed blue arrows show the
direction of water movement; thick gray arrows show blood flow through
the vasa recta; numbers are osmolarity in mOsm/L.

Countercurrent exchange is entirely passive. Blood enters the vasa recta


with an osmolarity of 300 mOsm/L. In the descending limb, water diffuses
out and solute diffuses in, increasing the osmolarity of the vasa recta blood
until it equals that of the adjacent interstitium. In the ascending limb, the
reverse occurs, with solutes diffusing out and water diffusing in,
decreasing the osmolarity of the blood. Thus, the vasa recta blood is in
osmotic equilibrium with the interstitium and helps maintain the
corticopapillary osmotic gradient.

Note that the blood leaving the vasa recta has an osmolarity of
325 mOsm/L. Thus, there is some depletion of medullary solutes; these
will be replaced by the ongoing processes of countercurrent multiplication
and urea deposition.

E. Cellular actions of ADH

V2 receptors for ADH are found in basolateral membranes of principal


cells in late distal tubule and collecting duct. When ADH binds to its
receptor, adenylate cyclase (which is coupled to the receptor by a Gs
protein), is activated; adenylate cyclase catalyzes the conversion of ATP
to cyclic AMP. Cyclic AMP, in turn activates protein kinase(s) which
phosphorylates proteins in or near the luminal membrane. The ultimate

769
Concentration & Dilution of Urine - Dr. Costanzo

physiologic action of ADH is the insertion of "water channels"


(aquaporin2, or AQP2) in the luminal membrane, making the cells
permeable to water; this occurs within minutes of hormone binding to
peritubular receptors.

Figure 7. Cellular mechanism of action of antidiuretic hormone in the principal


cell of the late distal tubule and collecting duct. Circled numbers correspond to the
text. AC, Adenylyl cyclase; ADH, antidiuretic hormone; ATP, adenosine
triphosphate; cAMP, cyclic adenosine monophosphate or cyclic AMP; Gs,
stimulatory G protein; R, V2 receptor.

III. FORMATION OF HYPOSMOTIC (DILUTE) URINE

Urine can be dilute with respect to blood (<300 mOsm/L). E.g., following
ingestion of a large volume of water, ADH secretion is suppressed, water is not
reabsorbed by the collecting ducts, and the urine is dilute. In diabetes insipidus,
where there is absence of ADH (central diabetes insipidus) or resistance of the
principal cells to ADH (nephrogenic diabetes insipidus), large volumes of dilute
urine are excreted.

770
Concentration & Dilution of Urine - Dr. Costanzo

Figure 8. Mechanisms for production of hyposmotic (dilute) urine in the absence of antidiuretic hormone
(ADH). Arrow shows location of water reabsorption; heavy outline shows water-impermeable portions of
the nephron; numbers are osmolarity of tubular fluid or interstitial fluid.

Again, the numbers are osmolarity and the heavy outline shows water
impermeability. Note the following important differences from the schematic
nephron which was making hyperosmotic urine.

A. ADH is very low or absent.


B. The osmolarity of the medullary and papillary interstitium is only about
1/2 that the concentrating kidney (600 vs. 1200 mOsm/L). There are two
major reasons: ADH increases the activity of the Na-K-2Cl cotransporter
in the thick ascending limb. When ADH is low or absent, this important
step in the countercurrent multiplication is reduced. Note that the
osmolarity of the fluid leaving the thick ascending limb is slightly elevated
(120 rather than 100), because less NaCl has been reabsorbed without
water. Second, less urea is deposited in the medulla and papilla because
the lack of ADH stops urea recycling.
C. The entire distal tubule and collecting duct is impermeable to water, so
there is no osmotic equilibration between tubular fluid and adjacent
interstitial fluid. No water is reabsorbed in these segments. In fact, the
tubular fluid becomes even more dilute than that leaving the thick

771
Concentration & Dilution of Urine - Dr. Costanzo

ascending limb, since the distal tubule and collecting ducts reabsorb some
NaCl, but no water. The final urine osmolarity can be as low as
75 mOsm/L, with volumes up to 15 L/day (8% of the GFR).

IV. FREE WATER CLEARANCE

"Free water clearance" (CH2O), is the amount of water which must be


subtracted from or added to urine to make it isosmotic with plasma.

"Free water" is water that is devoid of solute, or solute-free water. It is


“produced” in the diluting segments of the nephron (thick ascending limb and
early distal tubule) where NaCl is absorbed without water; the water that is left
behind is solute-free water. If ADH is low, the "free water" produced in the
diluting segments will be excreted ("positive free water"); relative to plasma, the
urine with be hyposmotic. If ADH is high, all the free water produced in the
diluting segments (plus more) will be reabsorbed by collecting ducts and no free
water will be excreted ("negative free water"); relative to plasma, the urine will be
hyperosmotic.

CH2O = V - Cosm

where:
V = urine flow rate
Cosm = Uosm V
Posm

(Note! CH2O is not a classical renal clearance.)

Examples:

1. If urine is isosmotic to plasma, CH2O is zero:

Posm = 300 mOsm/L


**
= 300 mosm/L
Uosm
V = 2 ml/min

CH2O = 2 ml/min - 300 mosm/L x 2 ml/min


300 mosm/L
= 0 ml/min

2. If urine is hyposmotic to plasma, CH2O is positive. A positive free water


clearance would be measured during water diuresis and reflects the function
of the diluting segments of the nephron.

772
Concentration & Dilution of Urine - Dr. Costanzo

Posm = 300 mOsm/L


** Uosm = 100 mOsm/L
V = 10 ml/min

10 ml/min - 100 mosm/L x 10


CH2O =
ml/min
300 mosm/L

= +6.7 ml/min

3. If urine is hyperosmotic to plasma, CH2O is negative. A negative free


water clearance would be measured during antidiuresis and reflects the
concentrating function of the kidney (the cortico-papillary gradient and
water reabsorption from collecting ducts).

Posm = 300 mOsm/L


** Uosm = 1000 mOsm/L
V = 0.5 ml/min

CH2O = .5 ml/min - 1000 mosm/L x .5 ml/min


300 mosm/L

= -1.2 ml/min

(Negative free water clearance was thought to be awkward, so the term


"free water reabsorption" (TcH2O) was created:

TcH2O = - CH2O

V. CLINICAL EXAMPLES
A. Central Diabetes Insipidus

Description. An 18 year old college freshman suffered a basal skull


fracture during an intramural "flag" football game. Shortly thereafter, he
developed severe polyuria with a urine output of 15 L/day and severe
thirst. He reported the need to drink almost constantly. He was unable to
sleep for more than 30 minutes at a time. A blood sample revealed a serum
osmolarity of 330 mOsm/L, a serum [Na] of 154 mEq/L and a urine
osmolarity of 90 mOsm/L. During a water restriction test, his urine
osmolarity remained below 100 mOsm/L. Administration of vasopressin
subcutaneously resulted in a prompt increase in urine osmolarity to 600
mOsm/L.

773
Concentration & Dilution of Urine - Dr. Costanzo

Explanation. The diagnosis is central diabetes insipidus subsequent to the


head injury which damaged the posterior pituitary gland. In this patient,
ADH is not being secreted, even though there is a strong osmotic stimulus
(plasma osmolarity of 330 mOsm/L). His severe thirst is result of the high
plasma osmolarity. The water deprivation test suggests that ADH is absent
or is ineffective on the renal collecting tubule. Administration of
exogenous ADH caused a prompt increase in water reabsorption, raising
the urine osmolarity; this finding confirmed that the defect is in the
patient's own pituitary, rather than in the response of the collecting tubule.
Lack of pituitary ADH has caused excessive excretion of large volumes of
dilute, solute-free urine, raising the plasma osmolarity and [Na].

The treatment would be administration of a long-acting vasopressin


analogue (dDAVP) by nasal spray. This ADH analogue has antidiuretic
activity but virtually no vasopressor activity and does not induce antibody
production, an important feature of long-term therapy.

An interesting response to chronic dehydration (such as in central diabetes


insipidus) occurs in the brain. The brain starts to synthesize osmotically
active solutes such as sorbitol, which are called osmolytes. These
osmolytes stay in the brain cells. Brain ICF osmolarity is increased by
these osmolytes to match the increased ECF osmolarity. In the rest of the
body cells, water shifts from ICF to ECF (to make ICF/ECF osmolarities
equal in steady state), but in the brain, no water shift occurs because the
osmolarites are already matched and equal; this response prevents brain
cells from shrinking in chronic dehydration.

B. Nephrogenic Diabetes Insipidus

Description. A male patient receiving lithium therapy for manic-


depressive psychosis reports severe and constant thirst and polyuria.
Recently, he has not been able to complete his 45 minute drive to work
without making a "pit-stop". In his physician's office, a urine sample had
an osmolarity of 100 mOsm/l and a serum sample had an osmolarity of
308 mOsm/L and a [Na] of 152 mEq/L. There was no glucose in his urine.
His urine osmolarity remained at 100 mOsm/L during a water restriction
test and during administration of ADH.

Explanation. Diuresis due to diabetes mellitus is excluded by the lack of


glucose in the urine. Primary polydipsia is excluded because the serum
osmolarity and [Na] are too high rather than too low as would be seen if
the primary problem was drinking too much water. The severe polyria and
very low urine osmolarity in the face of an elevated serum osmolarity
points to a defect in the concentrating ability of the kidney. (Increased
serum osmolarity should, via osmoreceptors, cause increased secretion of

774
Concentration & Dilution of Urine - Dr. Costanzo

ADH, increased water reabsorption from collecting ducts, resulting in


small volumes of concentrated urine.) The fact that the urine is dilute
shows that the "diluting segment" of the loop of Henle is functioning and
therefore the cortico-papillary gradient is probably intact. Thus, the defect
must be in ADH, either decreased secretion of pituitary ADH (Central
Diabetes Insipidus) or decreased responsiveness of the collecting duct to
ADH (Nephrogenic Diabetes Insipidus). The water deprivation test
confirms a problem with ADH, but doesn't distinguish between the two
diseases. Since injection of exogenous ADH did not raise urine
osmolarity, then the conclusion must be that his collecting ducts do not
respond to ADH. Endogenous ADH levels would actually be elevated in
this patient due to the strong osmotic stimulus.

Lithium treatment has caused the nephrogenic diabetes insipidus because


it interferes with the ability of ADH to generate cAMP in the collecting
tubule cells, possibly by inhibiting the stimulatory G protein (Gs) of the
adenylate cyclase).

Treatment would include stopping the lithium therapy. Thiazide diuretics


may be useful as they inhibit NaCl reabsorption in the "cortical diluting
segment", thus inhibiting of dilution of the urine, and bringing the urine
towards isotonicity.

C. Syndrome of Inappropriate ADH (SIADH)

Description. A patient with oat-cell carcinoma of the lung is admitted to


the hospital after he has a grand mal seizure at home. Laboratory studies
yielded the following information:

Plasma [Na] 110 mEq/L


Plasma osmolarity 225 mosm/L
Urine osmolarity 650 mOsm/L

He had a normal blood pressure, both standing up and lying down.

Explanation. The plasma [Na] and osmolarity are extremely low. The
appropriate response to this low plasma osmolarity would be to turn off
ADH secretion which would, in turn cause decreased water reabsorption
from collecting ducts, increased urine volume and decreased urine
osmolarity. Yet, the patient's kidneys are producing concentrated urine
(650 mOsm/L). Thus, the response of the kidney is “inappropriate” for the
low plasma osmolarity. The patient has Syndrome of Inapproriate ADH
(SIADH) where ADH is secreted when it should be suppressed.
Hypovolemia can, via volume receptors, cause increased ADH secretion
when there is no osmotic stimulus; however, in this patient there is no
evidence of decreased circulating blood volume since the blood pressure is

775
Concentration & Dilution of Urine - Dr. Costanzo

normal. Rather, ADH is coming from an ectopic source, the oat cell
carcinoma. The seizure was due to swelling of the brain within the skull;
severe dilution of the ECF caused water to move from extracellular to
intracellular fluid, increasing the volume of the brain cells.

The real danger is swelling of the brain, so hypertonic NaCl would be


administered acutely to raise the osmolarity of ECF and bring water out of
the brain cells. Chronically, the therapy is water restriction.

VI. PRACTICE QUESTIONS

1. A patient visits his physician complaining of frequent voiding and constant


thirst. Fasting serum and urine values are given with normal values in ( ).

Serum Urine
Sodium, mEq/L 148 (140) 10 (20-80)
Glucose, mg/dl 90 (70-100) 0 (0)
Osmolarity, mOsm/L 308 (290) 65 (50-1000)

The most likely cause of the patient's complaints is:

A. diabetes mellitus
B. excessive water drinking
C. hypoaldosteronism
D. lack of ADH
E. syndrome of inappropriate ADH (SIADH)

2. A patient has a urine osmolarity of 100 mOsm/L and a serum osmolarity of


310 mOsm/L. Which test would best distinguish between central diabetes
insipidus and nephrogenic diabetes insipidus in this patient?

A. injection of ADH
B. injection of hypertonic saline
C. injection of insulin
D. water deprivation
E. water loading

3. A subject receives an injection of ADH which causes his urine osmolarity to


increase from 200 to 600 mOsm/L. Accompanying this rise in urine
osmolarity, one would also expect an increase in:

A. serum osmolarity
B. serum Na concentration
C. positive free water clearance (CH2O)
D. urine volume

776
Concentration & Dilution of Urine - Dr. Costanzo

E. water reabsorption from collecting ducts.

4. Given the following values, what is the free water clearance (CH2O)?
Posm = 300 mOsm/L; Uosm = 100 mOsm/L; V = 9 ml/min

5. Ingestion of 2 liters of distilled water will result in which of the following


changes?

A. Increased circulating ADH.


B. Decreased TF/P osmolarity of proximal tubule fluid.
C. Increased recycling of urea from medullary collecting ducts into loops
of Henle.
D. Decreased water permeability of collecting ducts.

6. Compare the responses to water deprivation (dehydration) and water drinking


with respect to ADH levels, urine osmolarity, and urine volume.

7. What two major processes are responsible for establishing the corticopapillary
gradient?

8. What is the expected effect of low ADH (e.g. due to central diabetes
insipidus) on the corticopapillary gradient?

9. How do the vasa recta participate in the concentrating mechanism?

10. What is required for production of hyperosmotic urine?

11. What is required for production of hyposmotic urine?

12. Why is serum osmolarity lower than normal in SIADH?

13. Why is serum osmolarity higher than normal in central diabetes insipidus?

ANSWERS

1. Diabetes mellitus could cause the frequent voiding and thirst, but is ruled out by
absence of glucose in urine. Excessive (primary) water drinking would cause
serum osmolarity and Na to decrease and subsequently the urine osmolarity to
decrease; although the urine is quite dilute, the plasma osmolarity is elevated.
Hypoaldosteronism would cause decreased Na reabsorption by the distal tubule
and consequently a high urine Na concentration; instead, the urine Na is actually
low in spite of an elevated serum Na. Lack of ADH would cause the distal
tubules and collecting ducts to reabsorb less water, thus producing large volumes
of a dilute urine and as a consequence, raising the serum osmolarity and the serum
Na and causing thirst (the correct answer). SIADH is ruled out because there
would be inappropriately high levels of ADH, causing the collecting ducts to

777
Concentration & Dilution of Urine - Dr. Costanzo

reabsorb too much water, an inappropriately high urine osmolarity with low urine
volume, and as a result a low serum osmolarity. The answer is D.

2. The serum and urine osmolarity are consistent with either type of diabetes
insipidus. Injection of exogenous ADH would cause the collecting tubules to
reabsorb more water in the patient with central DI, but not in the unresponsive
collecting ducts of the patient with nephrogenic DI; thus in central DI, the urine
osmolarity would increase and the plasma osmolarity would decrease; in
nephrogenic DI, these would be unaffected and thus we have different responses
and a distingishing test. Injection of hypertonic saline or water deprivation
would not distinguish because this test would not bypass the role of the patient's
own pituitary. Insulin and water loading are nonsense answers. The answer is
A.

3. The answer is E. Injection of ADH would, as a result of this action on the


collecting ducts, cause a decrease in serum osmolarity, serum Na, and urine
volume so A, B, and D are clearly wrong answers. ADH injection would also
cause a decrease in positive free water clearance or (an increase in free water
reabsorption); free water is generated in the diluting segments (thick asc. limb and
early distal tubule) and, in the presence of ADH, reabsorbed in collecting ducts
and thus not excreted.

4.

CH2O = V - Cosm = 9 ml/min - 100 mOsm/L x 9 ml/min


300 mOsm/L
= 9 ml/min - 3 ml/min
= + 6 ml/min (a positive CH2O)

5. The answer is D

A. Increased circulating ADH. (No. ADH secretion would be turned off by


dilution of serum osmolarity.)
B. Decreased TF/P osmolarity of proximal tubule fluid. (No. Proximal water
reabsorption and TF/P osmolarity are unaffected by any of the
mechanisms involved in concentrating and diluting urine. It's always 1.0.)
C. Increased recycling of urea from medullary collecting ducts into loops of
Henle. (No. Urea recyling is increased by increased ADH due to the
differential senstitivity of cortical and medullary collecting duct urea
permeability. In this case, ADH is turned off by drinking water and so
urea recylcing would be decreased.)
D. Decreased water permeability of collecting ducts. (Yes. Water drinking
will turn off ADH secretion, less ADH to the collecting ducts and
decreased water permeability.)

778
Concentration & Dilution of Urine - Dr. Costanzo

6. Dehydration: ADH– high, urine volume – low, urine osmolarity – high


Water drinking: ADH – low, urine volume – high, urine osmolarity – low

7. Countercurrent multiplication and urea recycling

8. Decreased because ADH stimulates both countercurrent multiplication and urea


recycling

9. Vasa recta help maintain the corticopapillary gradient; the low blood flow
prevents the gradient from being “washed out”

10. Requires corticopapillary gradient and ADH

11. Requires that diluting segments (TALH and early distal) be "diluting" the urine
and no or low ADH

12. Inappropriately high H2O reabsorption in the collecting ducts, too much H2O is
returned to the circulation, dilutes the body fluids. Low serum osmolarity can't
turn off ADH secretion, because ADH is secreted "autonomously".

13. Too much H2O is excreted in urine because no circulating ADH. Loss of "free
water" causes concentration of solutes in body fluids.

779
GI Clinical Correlation: PUD - Dr. Grider

GI Clinical Correlation: Peptic Ulcer Disease (PUD)


Jack Grider, Ph.D.

OBJECTIVES:

1. Identify the factors that can lead to PUD.


2. Describe the role of H. pylori in PUD.
3. Compare the effectiveness of various treatments of PUD.

I. GENERAL:

A. Benign ulcerative lesions of stomach and duodenum


B. Mucosal defect that extends into the muscularis mucosa
C. Variety of causes

II. EPIDEMIOLOGY

A. Occurs in 5-10% of US population


B. Primary ulceration is declining, but increased secondary incidence because
of use of prescription and OTC non-steroidal anti-inflammatory drugs

III. PATHOGENESIS

A. Physiology:

1. Parietal Cell secrets acid under the positive influence of Gastrin,


Histamine and Acetylcholine (Vagal)
2. Antral G Cell secrets Gastrin in response to AA, FFA, and Vagal
stimulation
3. Somatostatin acts as paracrine agent to inhibit parietal cells in
fundus/corpus, and to inhibit G cells in antrum
4. Canniculus of the parietal cell increases the amount of H+/K+
ATPase on luminal surface leasing to increased H+ secretion.

B. Several mechanisms lead to PUD.

Figure 1.

780
GI Clinical Correlation: PUD - Dr. Grider

C. Role of Acid

1. Basal gastric acid secretion is twice as high in duodenal ulcer (DU)


patients
2. Peak acid output in response to stimulation is higher in duodenal ulcer
patients
3. Serum gastrin levels are higher in duodenal ulcer patients
4. Gastric Ulcer (GU) patients have relatively normal acid levels and
responses to stimulants.

Figure 2.

D. Role of Helicobacter Pylori (H. pylori)

Figure 3.

781
GI Clinical Correlation: PUD - Dr. Grider

1. Almost all duodenal ulcer (96%) and most gastric ulcer (75%) patients
have H. pylori present. Gastric metaplasia of duodenum enhances
growth of H. pylori in duodenum.
2. H. pylori clusters in families and seems to spread to all members
3. H. Pylori causes antral gastritis that breaks down mucosal defense
mechanisms
This promotes the erosive effects of acid and pepsin and allows
ulceration deep into the mucosa.

Figure 4.

4. H. pylori also is able to inhibit somatostatin secretion, leading to


elevated gastrin and HCl levels by removing the negative feed back
mechanisms in fundus and antrum
5. After eradication of H. pylori, basal acid output and gastrin levels
return to normal

782
GI Clinical Correlation: PUD - Dr. Grider

Figure 5.

NOTE: The role of H. pylori in PUD was the subject of 2005 Nobel Prize for
Medicine and Physiology awarded to Barry Marshall and Robin Warren.

E. Zollinger-Ellison (ZE) syndrome

1. Characterized by extremely high levels of gastrin due to presence of a


gastrin secreting tumor (gastrinoma) usually in duodenum or pancreas.
2. Secretion is autonomous and unregulated
3. Tumor is often malignant and can be associated with the presence of
other tumors secreting a variety of neuropeptides (multiple endocrine
neoplasm syndrome or MEN-1; 20% of ZE is really MEN-1)
4. No H. pylori is present in these patients and this therefore does not
resolve with antibiotic treatment
5. Because gastrin is trophic to the gastric mucosa, in ZE there is a large
increase in mucosal growth, sometimes leading to obstruction of the
pylous. These obstructions are often visible radiographically or with
endoscopy.

IV. CLINICAL FEATURES AND COMPLICATIONS

A. Epigastric abdominal pain 2-3 times a day between meals and at night

1. Pain sometimes “boring into back”


2. Pain often relieved by eating

B. Early satiety
C. Nausea & vomiting; diarrhea

783
GI Clinical Correlation: PUD - Dr. Grider

D. Can lead to:

1. Bleeding- This is the most common complications (10-80% patients)


2. Perforation –less common ~10%
3. Pyloric outflow obstruction-This is usually an acute manifestation
resulting from mucosal inflammation & spasm

a. Chronically it is due to fibrosis

4. Cigarette smoking increases incidence and recurrence

Figure 6.

V. DIAGNOSIS

A. Barium upper GI series: visual evidence but must rule out superficial
mucosal lesion
B. Endoscopy: definitive evidence
C. Noninvasive tests: Urea breath test can identify the presence of H. pylori.

This latter takes advantage of C13 or C14 labeled urea and the unique presence
of urease in H. pylori

VI. TREATMENT

784
GI Clinical Correlation: PUD - Dr. Grider

A. High-dose antiacid

1. At high dose there can be up to 80% healing after about 4 weeks

B. H2 receptor blockers:

1. After 8-12 weeks 90% healed

Solid circles in figure are with H2 blockers.


Figure 7.

C. Proton pump Inhibitor (PPI)

1. omeprazole
2. highly effective treatment
3. Takes some time to block all PPIs but once this accomplished effective
for prolonged periods.
D. Antibiotic therapy

1. Antibiotic plus Bismith eradicates H. Pylori


2. Removal of H. pylori returns acid and gastrin to normal levels
3. Often improves action of H2 blockers

785
GI Clinical Correlation: PUD - Dr. Grider

Closed circle: H2 blocker


Open circle: H2 blocker plus
antibiotic therapy
Figure 8.

4. History of ulcer is one or more recurrence ~ 90% after 1-2 years.


5. If H. Pylori is eradicated, the recurrence rate is 1%

A: No treatment for H. Pylori


B: Antibiotic treatment to eliminate H. pylori
Figure 9.

786
GI Clinical Correlation: PUD - Dr. Grider

6. Also if H pylori is eliminated, the exacerbating effects of cigarettes is


abolished

E. Surgery (Once a common treatment, but now is rare and only in extreme
cases)

1. vagotomy
2. removal of stimulus to gastrin and oxyntic cells.
(highly selective vagotomy.)
3. removal of tumor in ZE
4. repair of outlet obstruction and of perforation

787
Renal Problem Solving - Dr. Costanzo

Renal Problem Solving


Linda Costanzo, Ph.D.

I. PRACTICE [TF/P]X AND [TF/P]X/[TF/P]in

As appropriate, use the nephron locations in the list below to answer the following
questions. The correct answers may be one, more than one, or none of these
locations.

Bowman’s space
Mid-point of proximal convoluted tubule
End of proximal convoluted tubule
Bend of loop of Henle (hairpin turn)
Thick ascending limb of Henle’s loop
Early distal tubule
Late distal tubule
Cortical collecting ducts
Inner medullary collecting ducts
Final urine

A. At which site/s is [TF/P]glucose/[TF/P]in less than it is in Bowman’s space?

B. At which site/s is [TF/P]in = 1.0?

C. At which site/s is [TF/P]in < 1.0?

D. If [TF/P]in = 5.5, what fraction of the filtered water has been reabsorbed?

E. If [TF/P]Na /[TF/P]in = 0.75, what fraction of the filtered Na+ has been
reabsorbed?

F. At which site/s is [TF/P]Na < 1.0 during antidiuresis (high ADH)?

788
Renal Problem Solving - Dr. Costanzo

G. At which site (only one) is [TF/P]in highest during antidiuresis (high


ADH)?

H. If [TF/P]K /[TF/P]in = 1.3, what percentage of the filtered K+ is present in


tubular fluid?

I. If [TF/P]glucose = 0.5 and [TF/P]in = 2.0, what fraction of the filtered


glucose has been reabsorbed?

J. At what site/s would [TF/P]Na = 1.0 and [TF/P]glucose = 0?

K. At what site/s is [TF/P]PAH > 1.0?

L. [TF/P]Na changes from a value of 2.0 at the bend of loop to a value of 0.5
in the thick ascending limb. What is responsible for this change?

M. [TF/P]Na changes from a value of 0.4 in early distal tubule to a value of 0.8
in the collecting duct in the presence of ADH. What is responsible for this
change?

N. [TF/P]in = 7.0 at the bend of the loop of Henle and is 7.0 in the final urine.
What is the explanation for the lack of change in this value between these
two sites?

789
Renal Problem Solving - Dr. Costanzo

II. CASE OF TYPE I DIABETES MELLITUS

A 9-year-old boy suddenly began having symptoms of frequent urination and


constant thirst. He told his parents about the symptoms, and they immediately
took him to the doctor who detected large amounts of glucose in his urine. The
following laboratory and physical findings were noted, and he was diagnosed with
Type I diabetes mellitus and treated with insulin.

Blood Pressure (lying) 90/60


Blood Pressure (standing) 70/40
Heart rate (lying) 90 beats/min
Heart rate (standing) 120 beats/min

Fasting blood glucose 370 mg/dL (normal 70-100 mg/dL)


Plasma Na+ concentration 142 mEq/L (normal)

BUN 10 mg/dL (normal)

A. Why was he excreting glucose in his urine?

B. Why did he have polyuria (increased urine production)?

C. What was the boy’s plasma osmolarity?

D. Why was he constantly thirsty?

E. What explanation can you offer for the change in blood pressure between
lying down and standing up? What about the difference in heart rate?

790
Renal Problem Solving - Dr. Costanzo

III. CASE OF PRIMARY HYPERALDOSTERONISM

A 40-year-old woman has constant, severe headaches and muscle weakness. In


the doctor’s office, her blood pressure was severely elevated at 170/130. After
measuring the following laboratory values, she was diagnosed with an
aldosterone-secreting tumor.

Plasma Na+ concentration 141 mEq/L (normal)


Plasma K+ concentration 1.8 mEq/L (normal, 4.5 mEq/L)
Plasma creatinine concentration 1.2 mg/dL (normal)

Urinary Na+ excretion 150 mEq/day (normal)


Urinary K+ excretion Increased

Urinary creatinine excretion 1500 mg/day

Plasma renin activity Decreased


Plasma aldosterone Increased

A. Why was her blood pressure elevated?

B. What is the reason for her hypokalemia?

C. What explanation can you give for her muscle weakness?

D. What was her glomerular filtration rate?

E. What was her fractional Na+ excretion?

F. Given the diagnosis of primary hyperaldosteronism, would you expect urinary


Na+ excretion to be increased, decreased, or unchanged compared to a normal
person? How do explain the finding of normal urinary Na+ excretion?

791
Renal Problem Solving - Dr. Costanzo

G. Why was her plasma renin activity decreased?

H. What drug might the woman take while awaiting surgery to remove the
tumor?

IV. CASE OF NEPHROGENIC DIABETES INSIPIDUS

A man with advanced prostate cancer is lethargic and has severe polyuria
(increased urine production) and polydipsia (increased water drinking). In the
emergency room, his serum Ca2+ is very elevated, which was believed to be
responsible for his symptoms. The staff withheld drinking water for 2 hours and
the following values were measured.

Plasma osmolarity 305 mOsm/L


Urine osmolarity 80 mOsm/L

Following administration of dDAVP by nasal spray, there was no change in his


serum or urine osmolarity.

A. Hypercalcemia (elevated serum Ca2+) is one cause of nephrogenic diabetes


insipidus. Which results support a diagnosis of nephrogenic diabetes
insipidus?

B. During water deprivation, why was his plasma osmolarity elevated?

C. Does the value for urine osmolarity during water deprivation make sense?

D. If plasma ADH level had been measured, would you expect it to be increased,
decreased, or normal?

792
GI Tract Review - Dr. Grider

GI Tract Review
Jack Grider, Ph.D.
Linda Costanzo, Ph.D.

Figure 1.

Oral Esop Stomach Small Int Colon Rectum

peristalsis,
antiperistalsis
peristalsis peristalsis
Motility swallowing peristalsis mixing defecation
(1° & 2°) segmentation
Mass
movement

Interdigestive
no no yes yes yes no
MMC

HCl
intrinsic HCO3, Cl, H2O
Saliva
factor Panc Enzymes electrolyte
Secretion HCO3 none
pepsin mucous mucous
Amylase
lipase bile
mucous
very leaky
Epithleium very tight very tight very tight (duod) very tight very tight
to tight (ileum)

793
GI Tract Review - Dr. Grider

Nutrient (duod-
jej)
H2O &
Little (R- IF-B12 (ileum)
Absorption none none electrolyte none
OH) Bile salt (ileum)
SCFA
H2O &
electrolyte

Endocrine
no no yes yes some no
Control

Neural
yes yes yes yes yes yes
Control

ENS no distal yes yes yes yes

Skeletal
Smooth
(prox)
Muscle Skeletal Smooth Smooth Smooth (skeletal
Smooth
sphincter)
(distal)

I. SUMMARY OF PHASES

A. INTERDIGESTIVE PERIOD:

1. Migrating waves of increased activity sweep distally from stomach


to colon (MMC).
2. Short periods of increased electrical/contractile activity
accompanied by increased secretion regionally.
3. Function to maintain intestine clear of remnants of chyme, digest
and remove desquamated enterocytes, remove large indigestible
particles from stomach, maintain small amount of fluid/mucous in
lumen
4. Occurs continually until a meal is ingested

B. ORAL PHASE OF MEAL

1. Sight, sound smell of food initiates excitatory vagal preganglionic


activity.
2. Increase in: gastric, intestinal, colonic motility
Salivary, gastric, intestinal, and pancreatic secretion
Vagal fibers stimulate initial release of Gastrin

794
GI Tract Review - Dr. Grider

3. Interdigestive pattern of motility and secretion immediately


stopped.
4. Food in oral cavity masticated, neutralized by saliva,
5. All events are preparatory for coming meal.

C. GASTRIC PHASE OF MEAL

1. Bolus of food/salivary secretion enters stomach.


2. Distension of stomach activates local short arc reflexes and vago-
vagal long arc reflexes.
3. Further increase in motility and secretion of all regions but
especially gastric secretion of acid, pepsin, IF.
4. Digestion of proteins begins to generate AAs and small
polypeptides that further stimulate vagal afferents and stimulate
release of Gastrin.
5. HCl levels reach maximum and pH in stomach falls.
6. Carbohydrate digestion by salivary amylase continues in alkaline
pockets.
7. Lipid component of the meal is emulsified by gastric peristalsis.
8. vitamin B12 is complexed to IF.

D. INTESTINAL PHASE OF MEAL

1. Acidic, hypertonic chyme flows into duodenum and activates local


reflexes, long arc reflexes, and hormone release.
2. Acid releases secretin which acts to decrease gastric emptying,
inhibit acid secretion, stimulate bicarbonate secretion from
intestinal gland, pancreatic ducts, and biliary system. The copious
flow of water that accompanies the HCO3 and Chloride ion
secreted from enterocytes dilutes the chyme returning it to normal
tonicity.
3. AA and FFAs stimulate the release of CCK which acts to stimulate
pancreatic enzyme secretion, gallbladder contraction and Sphincter
of Oddi relaxation by direct effect (minor) and by activating a
vago-vagal reflex arc (major)
4. Low pH in stomach activates release of somatostatin to inhibit
release of gastrin and HCl causing levels of these fall.
5. Increased pancreatic enzymes and bile in lumen of intestine lead to
digestion of carbohydrate, protein and lipid.
6. Motility patterns are largely segmentation to mix bile, enzymes
and nutrients to ensure maximal digestion and presentation of
chyme to the intestinal mucosa.
7. Occasionally peristaltic contractions and clusters of contraction
push small amounts of material distally.
8. As chime moves distally, carbohydrates and proteins are finally
digested to single units by brush border enzymes, absorbed largely

795
GI Tract Review - Dr. Grider

by energy- and sodium- dependent carriers by a transcellular route


and eventually removed from the villus by capillary network.
9. Lipids are digested into FFA and MGs, combined into soluble
micells by bile salts. The lipids are passively absorbed, repackaged
into chylomicrons and removed by way of the lymphatics.
10. In the terminal ileum, the bile salts are resorbed by sodium- and
energy-dependent carriers and returned to the liver for recirculation
(enterohepatic portal system).
11. Vitamin B12-Intrinsic Factor is similarly reabsorbed by energy-
and sodium-dependent carriers in the terminal ileum.
12. Most of the water is absorbed from the small intestine by a variety
of mechanism that are sodium dependent.
13. Chyme enters the colon which removes the final water and
electrolytes. Any indigestible starch is fermented by colonic
bacteria to form short chain fatty acids that are absorbed and act as
fuel for coloncytes.
14. Finally solid feces are stored in the sigmoid colon/rectum until
defecation.
15. As the stimuli (chemical and mechanical) are removed from the
stomach and intestine, the interdigestive pattern returns until the
next meal.

796
Lung Volumes and Capacities - Dr. Costanzo

Lung Volumes and Capacities


Linda Costanzo, Ph.D.

OBJECTIVES:

After studying this lecture, the student should understand:

1. Lung volumes and capacities and how they are measured with spirometry.
2. The difference between anatomic and physiologic dead space.
3. How to calculate minute ventilation and alveolar ventilation.
4. Forced expiratory volumes and FEV1 and the changes in these that occur in
obstructive and restrictive disease.

I. STRUCTURE OF THE RESPIRATORY SYSTEM

The lungs exchange O2 and CO2 between air and pulmonary capillary blood. The
interface between air and blood, the alveolar-pulmonary capillary barrier, is very
thin and has a large surface area for gas exchange. The large surface area is
accomplished by wrapping pulmonary capillaries around an enormous number of
alveoli (about 300 million alveoli per human lung!).

The respiratory system includes the lungs and the airways. The conducting zone
(or conducting airways) brings air into and out of the lungs; the respiratory zone
is lined with alveoli and is the site of gas exchange.

797
Lung Volumes and Capacities - Dr. Costanzo

Figure 1.

The conducting zone includes the nose, nasopharynx, larynx, trachea, bronchi,
bronchioles, and terminal bronchioles. These structures bring air into and out of
the respiratory zone, and warm, filter, and humidify the air. The trachea divides
into two bronchi, one for each lung; these bronchi divide into two smaller
bronchi, which divide again and again, with a total of 23 such divisions. The
conducting airways are lined with mucus-secreting cells and ciliated cells that
remove inhaled particles. The walls of the conducting airways contain smooth
muscle with sympathetic and parasympathetic innervation.

A. Sympathetic adrenergic neurons activate β2 receptors that dilate the


airways; importantly, these β2 receptors are also activated by circulating
catecholamines (epinephrine) and by drugs that are β2 agonists (e.g.,
isoproterenol). Thus, β2 agonists are useful in treating asthma because they
dilate the airways and decrease resistance to airflow.
B. Parasympathetic cholinergic neurons activate muscarinic receptors that
constrict the airways and increase resistance to airflow.

The respiratory zone includes structures lined by alveoli and, therefore,


participates in gas exchange. These include respiratory bronchioles, alveolar
ducts, and alveolar sacs. The respiratory bronchioles have cilia and smooth

798
Lung Volumes and Capacities - Dr. Costanzo

muscle, like the bronchioles of the conducting zone. The alveolar ducts are lined
with alveoli, but contain no cilia and little smooth muscle. Alveolar ducts
terminate in alveolar sacs that are lined with alveoli. The alveolar walls contain
elastic fibers and epithelial cells called alveolar cells (or pneumocytes). Type II
alveolar cells synthesize surfactant, which is required to reduce surface tension
of alveoli and prevent their collapse.

II. LUNG VOLUMES AND CAPACITIES - THE SPIROMETRY DIAGRAM

Lung volumes and capacities are measured by spirometry. The subject breathes
into and out of a spirometer, displacing a bell. The volume displaced is recorded
on graph paper. The subject first breathes quietly (normally), then takes a
maximal inspiration followed by a maximal expiration.

Figure 2.

A. Lung volumes (four)

1. Tidal volume, or VT, is the volume inspired and expired in normal,


quiet breathing. Tidal volume includes the volume of air that fills
alveoli plus the volume that fills the airways. Normal tidal volume is
about 500 ml.
2. Inspiratory reserve volume is the volume that can be inspired above
tidal volume during maximal inspiration. It is approximately 3000 ml.

799
Lung Volumes and Capacities - Dr. Costanzo

3. Expiratory reserve volume is the volume that be expired following


expiration of a tidal volume during maximal expiration. It is
approximately 1200 ml.
4. Residual volume is the volume remaining in the lungs following
maximal expiration. It is approximately 1200 ml. Residual volume is
not measured by spirometry, but with a helium dilution method or a
body plethysmograph.

B. Lung capacities (four). Each lung capacity consists of two or more lung
volumes.

1. Inspiratory capacity is the tidal volume + inspiratory reserve volume.


It is the total volume inspired during maximal inspiration.
2. Functional residual capacity, or FRC, is the volume remaining in the
lungs following a normal tidal expiration. FRC is the expiratory
reserve volume + residual volume. FRC is considered the equilibrium
volume of the lungs (volume in the lungs between normal breaths).
3. Vital capacity, or VC, is the inspiratory capacity + the expiratory
reserve volume. Vital capacity is the total volume that can expired
following maximal inspiration. Vital capacity increases with male
gender, body size, and physical conditioning. In adults, it decreases
with age.
4. Total lung capacity is the sum of all the lung volumes, i.e., it is vital
capacity + residual volume.

III. DEAD SPACE

Dead space is the volume of the airways and the lungs that does not participate in
gas exchange. The anatomic dead space is the volume of the conducting airways.
Since conducting airways have no alveoli, they cannot possibly participate in gas
exchange. The volume of the anatomic dead space is about 150 ml. That is, in a
normal tidal volume of 500 ml, 150 ml (1/3) fills the anatomic dead space and 350
ml (2/3) fills the alveoli.

The figure shows a tidal volume, 1/3 of which will fill the anatomic dead space.
At the end of expiration, the conducting airways are filled with air that had been
in the alveoli (i.e., air that already exchanged gases with pulmonary capillary
blood). As you will learn, we call this alveolar gas. With inspiration of the next
breath, this air is first to enter the alveoli; however, it will not undergo further gas
exchange because it has already “been there, done that.” The next air to enter the
alveoli is fresh air from the inspired tidal volume that will undergo gas exchange.
Finally, some of the inspired tidal volume does not make it to the alveoli, but
stays in the conducting airways (anatomic dead space); this air does not undergo
gas exchange (i.e., is dead) and is the first air expired.

800
Lung Volumes and Capacities - Dr. Costanzo

Figure 3.

The physiologic dead space is comprised of the anatomic dead space plus a
“functional dead space” in alveoli. Functional dead space refers to alveoli that are
ventilated but not perfused with blood; because they are not perfused, they cannot
participate in gas exchange, i.e., functionally “dead.” In normal persons, there is
little functional dead space, and the physiologic dead space is nearly equal to the
anatomic dead space. However, in lung diseases in which a so-called
ventilation/perfusion defect develops, functional dead space increases and causes
the physiologic dead space to increase.

Thus, the physiologic dead space includes all lung spaces that are ventilated but
are not participating in gas exchange. In the physiologic dead space, no O2 or CO2
is exchanged. As a result, in the dead space alveolar PO2 and PCO2 approach
their values in inspired air. The calculation of physiologic dead space by Bohr’s
equation is shown in the lectures on V/Q defects.

IV. VENTILATION RATES

Ventilation rate is the volume of air moved into and out of the lungs per unit time,
expressed in ml/minute or L/minute.

A. Minute ventilation is the total volume of air moving into and out of the
lungs per unit time and is calculated as:

Minute ventilation = VT x breaths/minute

801
Lung Volumes and Capacities - Dr. Costanzo

Where VT is tidal volume.

B. Alveolar ventilation is the minute ventilation corrected for physiologic


dead space and is calculated as:

Alveolar ventilation = (VT - VD) x breaths/minute

Where VT is tidal volume and VD is physiologic dead space.

V. FORCED EXPIRATORY VOLUMES (FEV)

Vital capacity has already been defined as the total volume that can be expired
following a maximal inspiration. Really, we should say forcibly expired, since
that is the only way to expire all the air one possibly can. Thus, vital capacity is
the same as forced vital capacity, or FVC. In spirometry, the subject inspires
maximally and then, with forced maximal effort, expires all the air he can, as fast
as possible. The total volume expired is the forced vital capacity, or vital capacity.

Vital capacity changes with age, conditioning and, importantly, disease. We are
also interested in the time-course of expiration of the forced vital capacity
because the time-course (i.e., how fast it is expired) is altered in many lung
diseases. The volume of air that can be forcibly expired in the first second of
expiration is called FEV1, the volume that can be forcibly expired in the first two
seconds of expiration is called FEV2, and the volume that can be forcibly expired
in the first three seconds of expiration is called FEV3. (Because normal persons
expire the entire vital capacity in three seconds, there is no need for an “FEV4.”)
FEV1 is very sensitive to changes in airway resistance (a point that will be
reiterated).

In several important lung diseases (asthma, chronic obstructive lung disease, and
fibrosis), there are changes in both forced vital capacity (FVC) and FEV1. To
interpret the FEV1, therefore, it is important to calculate the ratio of FEV1 to FVC
(FEV1/FVC), i.e., the fraction of the vital capacity that can be expired in the first
second of forced expiration.

802
Lung Volumes and Capacities - Dr. Costanzo

Figure 4.

A. Normal. In normal persons, 80% of the vital capacity is expired in the first
second of forced expiration. In other words, FEV1/FVC is 0.8, or 80%.

B. Obstructive disease (e.g., asthma; chronic obstructive pulmonary disease


[COPD]). In obstructive lungs diseases, there is obstruction of airways, which
increases the resistance to airflow. Expiration and FEV1 are very sensitive to
changes in airway resistance. In obstructive lung disease, both FVC and FEV1
are decreased, but FEV1 is decreased more than FVC such that the ratio of
FEV1/FVC is decreased.

C. Restrictive disease (e.g. fibrosis). In restrictive lung diseases, there is


increased stiffness and elastic recoil of lung tissues and, as a result, increased
elastic recoil force for expiration (discussed in next lectures). Don’t be fooled
by the name “restrictive”......it does not mean increased resistance; it refers to
increased stiffness (“restriction”) of the lungs. In restrictive lung disease, like
in obstructive lung disease, FVC is decreased. However, because the elastic
recoil of lung structures is increased, FEV1 is decreased less than FVC. As a
result, FEV1/FVC is increased (or normal).

Normal Obstructive Disease Restrictive Disease


FVC --- ↓ ↓↓
FEV1 --- ↓↓ ↓
FEV1/FVC 0.8 ↓ ↑(or normal)

803
Lung Volumes and Capacities - Dr. Costanzo

VI. PRACTICE QUESTIONS

1. The volume remaining in the lungs after expiring a tidal volume is:

A. Residual volume
B. Expiratory reserve volume
C. Expiratory reserve volume + residual volume
D. Vital capacity - residual volume
E. Total lung capacity - residual volume

2. Which lung volume or capacity can be inspired above FRC?

A. Inspiratory reserve volume


B. Inspiratory capacity
C. Tidal volume
D. Inspiratory residual capacity
E. None of the above

3. FEV1 is:

A. The fraction of the vital capacity that can be expired in one second.
B. The fraction of the total lung capacity that can be expired in one
second.
C. The volume that can be expired in the first second following
maximal inspiration.
D. The volume that can be expired in the first second following
inspiration of a tidal volume.

4. In the physiologic dead space:

A. Gas exchange does not occur.


B. Gas exchange occurs only in the conducting airways.
C. Gas exchange occurs, but is decreased.
D. O2 exchange occurs, but CO2 exchange does not.
E. CO2 exchange occurs, but O2 exchange does not.

EXPLANATIONS

1. Answer = C. FRC is expiratory reserve volume + residual volume

2. Answer = B

3. Answer = C

4. Answer = A

804
Mechanics of Breathing 1 and 2 - Dr. Costanzo

Mechanics of Breathing 1 and 2


Linda Costanzo, Ph.D.

OBJECTIVES:

After studying this lecture, the student should understand:

1. The definition of compliance and its relationship to elastance.


2. How lung compliance is measured and the explanation of hysteresis.
3. How compliance of the chest wall can be demonstrated by a pneumothorax.
4. How to interpret the pressure-volume curves for the lung, chest wall, and
combined lung-chest wall system.
5. How lung diseases such as emphysema and fibrosis alter lung compliance and the
resulting effects on the pressure-volume curves.
6. The concept of surface tension in alveoli and the role of surfactant.
7. The factors that determine and alter airway resistance.
8. Changes in alveolar, intrapleural, and transmural pressures during the normal
breathing cycle.
9. Alveolar, intrapleural, and transmural pressures during forced expiration, and the
effect of emphysema on these pressures.

I. MUSCLES OF RESPIRATION

A. Inspiration

The most important muscle of inspiration is the diaphragm. It is a thin,


dome-shaped muscle that inserts into the lower ribs and is supplied by the
phrenic nerve. When the diaphragm contracts (as directed by phrenic
nerve activity), it forces the abdominal contents down and forward, and
the volume of the chest cavity is increased. In normal tidal breathing, the
excursion of the diaphragm is very slight, about 1 cm. (During heavy
breathing, however, the diaphragm moves greater distances, up to 10 cm.)
During exercise, the external intercostal muscles and accessory muscles
are used for inspiration.

B. Expiration

During normal tidal breathing, expiration is passive. The lungs and chest
wall are elastic structures and naturally “want” to return to their resting
positions after being expanded during inspiration. During exercise,
expiration becomes active, utilizing the abdominal muscles and the
internal intercostal muscles.

805
Mechanics of Breathing 1 and 2 - Dr. Costanzo

II. COMPLIANCE (inverse of elastance!)

Compliance describes the distensibility of the respiratory structures. It says how


much the volume of a structure changes for a given change in pressure (ΔV/ΔP).
If compliance is high, the volume changes a lot; if compliance is low, the volume
changes little. A point that hangs students up is that compliance is not elasticity, it
is the inverse of elasticity (or elastance). The more elastic fibers in a structure
(e.g., the alveoli), the greater its tendency to “spring back” and the stiffer and less
compliant it is (like a thick rubber band). The fewer elastic fibers in a structure,
the less its tendency to spring back and the more compliant it is. You must get
this! High compliance = less elastic recoil; low compliance = more elastic recoil.

A. How lung pressures are expressed

We will now discuss compliance of lung structures, or how their volume


changes as a function of pressure. The pressure term needs clarification, as
it can be expressed in several different ways. It can mean pressure inside a
structure (e.g., the lungs), pressure outside a structure, or even the pressure
difference across a structure, which is called transmural pressure. For
example, transpulmonary pressure (the pressure difference across the
lungs), is the difference between intra-alveolar pressure and intrapleural
pressure. (The intrapleural ‘space’ lies between the alveoli and the chest
wall.) By convention, lung pressures are referred to atmospheric
pressure, where atmospheric pressure (PB) is called “zero.” For example,
if alveolar pressure is equal to atmospheric, it is said to be zero; if it is
higher than atmospheric, it is positive, and if it is lower than atmospheric,
it is negative.

B. Compliance of the lungs

The lungs are both compliant and elastic. They must be compliant to fill
with air during inspiration. They must be elastic to recoil and push air out
during expiration. The compliance of the lungs is demonstrated by an
isolated lung in a jar.

806
Mechanics of Breathing 1 and 2 - Dr. Costanzo

Figure 1.

In the figure, lung volume is expressed as a function of pressure.


Compliance is the slope of the relationship (i.e., ΔV/ΔP). The space
outside the lung is analogous to intrapleural pressure in vivo. In the
experiment, the lungs and airways are open to the atmosphere and the
pressure inside the lungs is equal to atmospheric. Pressure outside the lung
is varied with a pump to simulate changes in intrapleural pressure. The
volume of the lung is measured at different pressures. When the outside
pressure is made more negative (i.e., lower than atmospheric), the lung
inflates and its volume increases. When the outside pressure is made less
negative, the lung deflates and its volume decreases. A sequence of
inflation and deflation creates a pressure-volume loop.

The inspiration (filling) limb of the lung’s pressure-volume loop has a


different slope (compliance) than the expiration limb (called hysteresis).
For a given outside (intrapleural) pressure, the volume of the lung is
higher during expiration than during inspiration. Thus, lung compliance is
higher during expiration than during inspiration. Lung compliance is

807
Mechanics of Breathing 1 and 2 - Dr. Costanzo

typically measured on the expiration limb because the inspiration limb


flattens at highest volumes (when lung tissue is already maximally
stretched out).

Why does hysteresis occur? The slopes of the inspiration and expiration
curves are compliance, and compliance is an intrinsic property of the lung.
Since it’s the same lung, why would compliance be higher when we expire
than when we inspire? The answer is surface tension. In the air-filled
lung, there are strong intermolecular forces between liquid molecules
lining alveoli. During inspiration, one begins at low lung volume where
the liquid molecules are close together and strongly attracted to each other
– to inflate the lung, one must break these intermolecular forces; thus, it is
harder to inflate the lung than would be expected based on compliance
alone. For expiration, one begins at high lung volume where liquid
molecules are far apart and intermolecular forces needn’t be broken. Thus,
the observed compliance curve during inspiration is determined both by
intrinsic compliance and by surface tension; the observed compliance
curve during expiration is determined only by intrinsic compliance (i.e.,
the “real” compliance).

C. Compliance of the chest wall

Figure 2.

The figure shows the relationship between the lungs and the chest wall.
The conducting airways are shown as a single tube, and the gas exchange
region is shown as a single alveolus. The intrapleural space between the
lungs and chest wall is exaggerated. Like the lungs, the chest wall also is

808
Mechanics of Breathing 1 and 2 - Dr. Costanzo

compliant, and its compliance is demonstrated by introducing air into the


intrapleural space (pneumothorax).

Figure 3.

Intrapleural pressure is normally negative. This negative intrapleural


pressure is created by two opposing elastic forces pulling on the
intrapleural space: the lungs, with their elastic properties, tend to collapse;
the chest wall, with its elastic properties, tends to spring out. When these
two opposing forces pull on the intrapleural space, a negative pressure is
created. In turn, this negative intrapleural pressure opposes the natural
tendency of the lungs to collapse and the chest wall to spring out; that is,
the negative intrapleural pressure is responsible for preventing the lungs
from collapsing and the chest wall from springing out.

When a sharp object punctures the intrapleural space, air is introduced into
the space (pneumothorax) and intrapleural pressure becomes equal to
atmospheric pressure; thus, instead of its normal negative value,
intrapleural pressure becomes zero, which has two predictable
consequences. (1) There is no longer a negative intrapleural pressure to
hold the lungs open, and the lungs collapse. (2) There is no longer a
negative intrapleural pressure to keep the chest wall from expanding, and
the chest wall springs out.

809
Mechanics of Breathing 1 and 2 - Dr. Costanzo

D. Pressure-volume Curves for the Lungs, Chest wall, and Combined


Lung and Chest Wall

Figure 4.

Pressure-volume curves are shown for the lungs alone (lung in a jar), chest
wall alone, and combined lung and chest-wall system. (The curve for the
chest wall alone is obtained by subtraction of the lung curve from the
combined lung and chest-wall curve.) The curve for the combined lung
and chest-wall system is obtained by having a trained subject breathe in
and out of a spirometer. The subject inspires or expires to a given volume.
The spirometer valve is then closed and, as the subject relaxes his
respiratory muscles, his airway pressure is measured (called relaxation
pressure). In this way, values for airway pressure are obtained at a series
of static volumes of the combined lung and chest-wall system. When the
volume is functional residual capacity (FRC), airway pressure is zero and
equal to atmospheric pressure. At volumes less than FRC, airway
pressures are negative (less volume, less pressure). At volumes higher than
FRC, airway pressures are positive (more volume, more pressure).

The slope of each curve is compliance. Note that the compliance of the
chest wall alone is similar to the compliance of the lungs alone (the slopes
are the same). However, the compliance of the combined lung and chest-
wall system is less than that of either structure alone (i.e., the combined
lung and chest wall curve is “flatter”). Visualize one balloon (the lungs)
inside another balloon (the chest wall). Alone, each balloon is compliant,
but the combined system (balloon within balloon) is less compliant and

810
Mechanics of Breathing 1 and 2 - Dr. Costanzo

harder to expand.

To interpret these curves, begin at the volume called FRC, the equilibrium
volume of the combined lung and chest-wall system. FRC is the volume
present in the lungs after a person has expired a normal tidal breath. Then
compare the graphs at volumes less than FRC and volumes greater than
FRC.

1. Volume is FRC. When the volume is FRC, the combined lung and
chest-wall system is at equilibrium. Airway pressure is equal to
atmospheric pressure, which is called zero. At FRC, because they are
elastic structures, the lungs “want” to collapse and the chest wall
“wants” to expand. If these elastic forces were unopposed, the
structures would do exactly that! However, at FRC, the collapsing
force on the lungs is exactly equal to the expanding force on the chest
wall, as shown by the equidistant arrows, and the combined lung and
chest-wall system neither tends to collapse or expand.

2. Volume less than FRC. When the volume in the system is less than
FRC (i.e., the subject forcibly expires into the spirometer), there is less
volume in the lungs and the collapsing elastic force of the lungs is
smaller. The expanding force on the chest wall is greater, however,
and the combined lung and chest wall system “wants” to expand.
(Notice on the graph that, at volumes less than FRC, the collapsing
force on the lungs is smaller than the expanding force on the chest
wall, and the combined system tends to expand.)

3. Volume greater than FRC. When the volume in the system is greater
than FRC (i.e., the subject inspires from the spirometer), there is more
volume in the lungs and the collapsing (elastic) force of the lungs is
greater. The expanding force on the chest wall is smaller, however,
and the combined lung and chest-wall system “wants” to collapse.
(Notice on the graph that, at volumes greater than FRC, the collapsing
force on the lungs is greater than the expanding force on the chest
wall, and the overall system tends to collapse. At highest lung
volumes, both the lungs and the chest wall “want” to collapse [the
chest wall curve has crossed the vertical axis] and, there is a very large
collapsing force on the combined system.)

811
Mechanics of Breathing 1 and 2 - Dr. Costanzo

E. Changes in Lung Compliance in Disease

Figure 5.

Lung compliance changes in disease, changing the slopes of the


relationships. For convenience, each component of the system is shown on
a separate graph (i.e., chest wall alone, lung alone, and combined lung and
chest wall). The chest wall alone is included only for completeness, since
its compliance is not altered by disease. The solid lines in each graph gives
the normal relationships. The dashed and dotted lines show the effects of
disease.

812
Mechanics of Breathing 1 and 2 - Dr. Costanzo

1. Emphysema (increased lung compliance). Emphysema is associated


with loss of elastic fibers in the lungs. As a result, the compliance of
the lungs increases. (Recall again, the inverse relationship between
elastance and compliance.) An increase in compliance is associated
with an increased (steeper) slope of the volume versus pressure curve
for the lung. At a given volume, the collapsing (elastic recoil) force on
the lungs is decreased. At the original value for FRC, the tendency for
the lungs to collapse is now less than the tendency of the chest wall to
expand, and these opposing forces will no longer be balanced. In order
for the opposing forces to be balanced, volume must be added to the
lungs to increase their collapsing force. Thus, the combined lung and
chest-wall system seeks a new higher FRC, where the two opposing
forces can be balanced; the new intersection point, where airway
pressure is zero, is increased. The patient with emphysema breathes at
higher lung volumes (in recognition of the higher FRC) and has a
barrel-shaped chest.

2. Fibrosis (decreased lung compliance). Fibrosis is a restrictive


disease associated with stiffer lung tissues and decreased compliance.
There is decreased slope of the volume versus pressure curve for the
lung. At the original FRC, the tendency of the lungs to collapse is
greater than the tendency of the chest wall to expand, and the opposing
forces will no longer be balanced. To reestablish the balance, the lung
and chest-wall system will seek a new lower FRC; the new
intersection point, where airway pressure is zero, is decreased.

III. SURFACE TENSION

A. Law of LaPlace

Surface tension of a sphere (such as an alveolus) is given by the Law of


LaPlace:

P = 2T
r

where P is collapsing pressure on the alveolus (or pressure required to


keep the alveolus open), T is surface tension, and r is radius of the
alveolus.

The alveoli are lined with a film of liquid and intermolecular attractive
forces create a surface tension, which creates a pressure that tends to

813
Mechanics of Breathing 1 and 2 - Dr. Costanzo

collapse the alveoli. Because alveoli are small, there is a potential problem
in keeping them open. (According to LaPlace, the smaller the radius, the
higher the collapsing pressure.) Alveoli could solve this problem by
having large radii; however, large radii means reduced surface area, which
is bad for gas exchange. Surfactant to the rescue!

B. Surfactant is synthesized by type II alveolar cells. Surfactant reduces


surface tension of alveoli, thus reducing the collapsing pressure of small
alveoli. The most important constituent of surfactant is dipalmitoyl
phosphatidylcholine (DPPC). Molecules of DPPC are amphoteric and
align themselves on the alveolar surface with their hydrophobic portions
attracted to each other and their hydrophilic portions repelled from each
other. In this way, DPPC breaks up the liquid molecules that were
responsible for high alveolar surface tension. When surfactant is present,
surface tension and collapsing pressure are reduced and small alveoli can
be kept open.

In neonatal respiratory distress syndrome, there is deficiency of


surfactant (Surfactant begins appearing at gestational week 24 and is
almost always present by gestational week 35. Thus, infants born between
weeks 24 and 35 have uncertain surfactant status.) In the absence of
surfactant, small alveoli have increased surface tension and collapsing
pressure and will collapse (called atelectasis). Absence of surfactant also
decreases lung compliance. Collapsed alveoli are not ventilated (they are
too hard to expand) and, therefore, do not participate in gas exchange. You
will learn that this is called a shunt, which causes hypoxemia (a decrease
in arterial PO2).

IV. RESISTANCE OF AIRWAYS

A. Ohm’s Law

There is a relationship between airflow, pressure, and resistance that is


analogous to the relationship between blood flow, pressure, and resistance
in the cardiovascular system, i.e., Ohm’s law:

Q = ΔP
R

where Q is airflow, ΔP is pressure difference between mouth (or nose) and


alveoli, and R is resistance of airways. The driving force for airflow is the
pressure difference.

814
Mechanics of Breathing 1 and 2 - Dr. Costanzo

Resistance of the airways is given by Poiseuille’s law:

R=8ηl
π r4

where R is airway resistance, η is viscosity of air, l is length of airway, and r


is radius of the airway. The most important aspect of Poiseuille is the inverse
dependence of resistance on r4. The smaller the airway, the higher the
resistance; the larger the airway, the lower the resistance. Incidentally,
medium-sized bronchi are the site of highest airway resistance. (Although it
would seem that smaller airways would have higher resistance, they do not
because they are arranged in parallel.)

B. What changes airway resistance?

1. Autonomic nervous system. As you have heard, bronchial smooth


muscle has autonomic innervation.

a. Parasympathetic stimulation, via muscarinic receptors,


causes constriction of bronchial smooth muscle, decreased
radius, and increased resistance to airflow.

b. Sympathetic stimulation, via β2 receptors, causes dilation


of bronchial smooth muscle, increased radius, and
decreased resistance to airflow. The β2 receptors also are
activated by circulating catecholamines (epinephrine) and
β2-adrenergic agonists (isoproterenol), which are useful in
dilating the airways in asthma.

2. Lung disease. Increased airway resistance is a feature of


obstructive lung diseases such as asthma and chronic obstructive
pulmonary disease (COPD, a combination of emphysema and
chronic bronchitis). Increased airway resistance causes decreased
airflow; this is especially a problem during expiration, which is
normally passive. (Later, we will discuss why emphysema, a
disease of increased compliance, causes increased airway
resistance and obstruction, a point that is not intuitively evident.)

3. Lung volume. You may be surprised to hear that lung volume


affects resistance of airways. Lung tissue exerts radial traction, or
pull, on airways. High lung volume exerts more traction and

815
Mechanics of Breathing 1 and 2 - Dr. Costanzo

decreases airway resistance. Low lung volume exerts less traction


and increases airway resistance. That persons with asthma breathe
at high lung volumes (higher FRC) is helpful, as it partially offsets
the high airway resistance associated with their disease.

V. THE BREATHING CYCLE: describes the changes in lung volumes and


pressures that occur during the cycle of inspiration, expiration, inspiration....etc.

A. Normal breathing. In normal breathing, the volume inspired and expired


is tidal volume. For the figures below, recall that pressures are referred to
atmospheric, which is called “zero.” Transmural pressures across the
lungs are shown by the open arrows and, by convention, are calculated as
alveolar (or airway) pressure minus intrapleural pressure; the magnitude of
the transmural pressure is shown above the open arrow. For example, if
alveolar pressure is zero and intrapleural pressure is -5 cm H2O, then
transmural pressure is +5 cm H2O (0 - [-5] = +5). If transmural pressure is
positive, the structure is open; if transmural pressure is negative, the
structure collapses. For all phases of the normal breathing cycle, despite
changes in alveolar, airway, and intrapleural pressure, transmural
pressure across lungs and airways is always positive; thus, these
structures remain open.

816
Mechanics of Breathing 1 and 2 - Dr. Costanzo

Figure 6.Figure 5-13 Volumes and pressures during the normal breathing cycle.
Intrapleural pressure and alveolar pressure are referred to atmospheric pressure.
Letters A to D correspond to phases of the breathing cycle in Figure 5-14.

817
Mechanics of Breathing 1 and 2 - Dr. Costanzo

Figure 7.Figure 5-14 Pressures during normal breathing cycle. The numbers give pressures in
cm H2O relative to atmospheric pressure (Patm). The numbers over the yellow arrows give
the magnitude of transmural pressures. The wide blue arrows show airflow into and out of
the lungs. A Rest; B half-way through inspiration; C end of inspiration; D halfway through
expiration.

1. Rest is the period between breaths. The diaphragm is at an equilibrium


position, alveolar pressure equals atmospheric pressure, lung volume is
FRC, and no air is moving into or out of the lungs (because there is no

818
Mechanics of Breathing 1 and 2 - Dr. Costanzo

pressure difference between the alveoli and the atmosphere.


Intrapleural pressure is negative (-5 cm H2O); as explained, the
lungs and the chest wall pull against the intrapleural space to create
this negative intrapleural pressure. Transmural pressures across the
lungs and airways are positive and, therefore, these structures are
open.

2. During inspiration, the diaphragm contracts (as directed by the


phrenic nerve) and the volume of the thorax increases. Lung volume
increases, lung pressure decreases (Boyle’s law), and both alveolar and
airway pressures become negative (less than atmospheric). A pressure
gradient is now created between the atmosphere and the lungs, and air
flows into the lungs down this gradient. The volume inspired is tidal
volume, so at the peak of inspiration lung volume is FRC + tidal
volume. At the peak of inspiration, intrapleural pressure is more
negative than at rest (-8 cm H2O) because thoracic volume has
increased and pulls more on the intrapleural space. The extent that
intrapleural pressure changes during inspiration is used to measure a
parameter called dynamic compliance of the lungs (ΔV/ΔP, where
ΔP is the change in intrapleural pressure). For example, if dynamic
compliance of the lungs is 0.3 L/cm H2O and a person inspires a tidal
volume of 600 ml, intrapleural pressure will decrease by 2 cm H2O
during the inspiration (e.g,. from -5 to -7 cm H2O).

3. Expiration is normally passive. Elastic recoil forces in the lungs


compress the air in the lungs and raise the pressure. Alveolar pressure
becomes higher than atmospheric pressure and this pressure gradient
drives air out of the lung. The volume expired is tidal volume.
Following expiration lung volume returns to FRC and intrapleural
pressure returns to -5 cm H2O.

B. Forced expiration. In forced expiration, expiratory muscles are used to


forcibly breath out as much as possible. Because of the effort involved,
lung and airway pressures are made more positive than in normal
expiration, and intrapleural pressure (normally negative) is also made
positive.

819
Mechanics of Breathing 1 and 2 - Dr. Costanzo

Figure 8.

1. Normal person. The figure shows the positive pressures created


during forced expiration. In the example, the exertion makes the
intrapleural pressure +20 cm H2O, the alveolar pressure +35 cm H2O,
and the airway pressure +25 cm H2O. The expiration is more forceful
than normal because the driving force for airflow is greater (i.e., +35
in the alveoli and 0 in the atmosphere). The pressures are more
positive inside the lungs and airways than outside, so the structures
remain open (i.e., transmural pressures are positive).
2. COPD. In a person with COPD (emphysema + bronchitis), there is
loss of elasticity, i.e., increased compliance. With forced expiration,
intrapleural pressure is again made +20 cm H2O, but pressures in the
lungs and airways are not as positive as in the normal person. Because
of loss of elastic tissue, there is decreased elastic recoil and less
positive pressure in the lungs and airways. Although the lungs are less
positive inside than normal, they are still more positive than
intrapleural; thus, transmural pressure is positive and the lungs remain
open. However, there is decreasing pressure along the airways (a
pressure gradient). The point where airway pressure equals intrapleural
pressure is called the equal pressure point. Further up the airway,
airways pressure becomes less than intrapleural and, at this point, the
airway collapses and creates an obstruction to airflow.

So....the answer to the question why is emphysema an “obstructive


disease” is that loss of elastic recoil leads to collapse of airways,
which causes obstruction. (Remember: in COPD, both FEV1 and
FEV1/FVC are reduced.) There are two reasons that expiration is
impaired in COPD: (a) loss of elastic recoil that normally drives air out
of the lungs and (b) collapse and obstruction of airways and increased
resistance to airflow.

Persons with COPD expire slowly through pursed lips to increase


airway pressure and thus prevent airway collapse. Note that increasing

820
Mechanics of Breathing 1 and 2 - Dr. Costanzo

the effort for expiration doesn’t help, since effort makes both
intrapleural and alveolar/airway pressure more positive.

IV. SUMMARY OF LUNG MECHANICS IN DISEASES

FRC (and Peak


Disease FEV1 FVC FEV1/FVC residual Expiratory
volume) Flow Rate
Asthma (obstructive) ↓↓ ↓ ↓ ↑ ↓
COPD (obstructive) ↓↓ ↓ ↓ ↑ ↓
Fibrosis (restrictive) ↓ ↓↓ ↑ ↓ ↑ (or --)

821
Mechanics of Breathing 1 and 2 - Dr. Costanzo

VI. PRACTICE QUESTIONS

1. Compared to restrictive lung disease, obstructive lung disease has a lower:

A. Vital capacity
B. FVC
C. FEV1
D. FEV1/FVC
E. Tidal volume

2. In persons with fibrosis, at usual (normal) values for FRC,

A. The collapsing force on the lungs is greater than the expanding


force on the chest wall.
B. The expanding force on the lungs is greater than the collapsing
force on the chest wall.
C. There is a collapsing force on both the lungs and the chest wall.
D. The collapsing force on the lungs is smaller than the expanding
force on the chest wall.
E. The collapsing force on the lungs is equal to the expanding force
on the chest wall.

3. In persons with emphysema, in order to balance the collapsing forces on


the lungs and chest wall:

A. FRC increases
B. FRC decreases
C. FRC can remain at its usual value, but residual volume decreases
D. Tidal volume increases
E. FVC increases

4. Which of the following pairs of pressures would cause collapse of the


structure?

A. Intra-alveolar pressure = +5 cm H2O; intrapleural pressure = -5 cm


H2O
B. Intra-airway pressure = +20 cm H2O; intrapleural pressure = +15
cm H2O
C. Intra-alveolar pressure = 0; intrapleural pressure = -5 cm H2O
D. Intra-airway pressure = +15 cm H2O; intrapleural pressure = +20
cm H2O

EXPLANATIONS

822
Mechanics of Breathing 1 and 2 - Dr. Costanzo

1. Answer = D. Vital capacity and FEV1 are decreased in both. It’s the ratio
of FEV1/FVC that is clearly lower in obstructive than restrictive.

2. Answer = A.

3. Answer = A

4. Answer = D. Easy, but I forced you to practice and visualize.

823
Physical Chemistry of Gases - Dr. Costanzo

Physical Chemistry of Gases: Gas Exchange


Linda Costanzo, Ph.D.

OBJECTIVES:

After studying this lecture, the student should understand:

1. Application of the gas laws to pulmonary physiology.


2. How to calculate the concentration of a gas in blood using Henry’s law.
3. Diffusion of gases according to Fick’s law and the concept of lung diffusion
capacity.
4. The overview of gas exchange across the alveolar-pulmonary capillary barrier and
the resulting values of PO2 and PCO2 in inspired air, alveolar air, mixed venous
blood, and systemic arterial blood.
5. The mechanism of diffusion-limited gas exchange.
6. The mechanism of perfusion-limited gas exchange.

I. GAS LAWS

A. General Gas Law

This one should be familiar!

PV = nRT

P is pressure (mm Hg), V is volume (L), n is the number of moles of gas,


R is the gas constant, and T is temperature in Kelvin.

B. Boyle’s Law

Boyle’s Law says that a given temperature, pressure times volume for a
gas is constant.

P1 V1 = P2 V2

For example, if the volume of the lungs increases (e.g., during inspiration),
the pressure must decrease to keep pressure times volume constant.

C. Dalton’s Law of Partial Pressures

It is critically important that you understand Dalton’s Law since, in


respiratory physiology, we are always dealing with gases in mixtures.
Dalton’s law of partial pressures states that the partial pressure of a gas
in a mixture of gases is the pressure that gas would exert if it occupied

824
Physical Chemistry of Gases - Dr. Costanzo

the whole volume of the mixture.

Px = PB x F

Px is the partial pressure of the gas, PB is barometric pressure, and F is the


fractional concentration of the gas. In humidified air, we correct for
water vapor pressure, PH2O, which is 47 mm Hg at 37° C.

Px = ( PB - PH2O) x F
B

In dry atmospheric air, there is 21% O2 and 79% N2 (no CO2, always
remember that!) and the respective partial pressures (mm Hg) at sea level
are shown in the table below. When this air is humidified air it is corrected
for the obligatory water vapor pressure and the respective partial pressures
are, accordingly, decreased.

Gas (F) Dry Air Humidified Tracheal Air


O2 (0.21) 160 150
CO2 (0) 0 0
N2 (0.79) 600 563
H2O 0 47
Total (barometric) 760 mm Hg 760 mm Hg

D. Henry’s Law for Dissolved Gases

Henry’s law applies to concentrations of gases dissolved in solution. This


is relevant since both O2 and CO2 are dissolved in blood. Important point:
if equilibration has occurred, partial pressure of a gas in the liquid
phase is equal to the partial pressure in the gas phase. Do not read
further until you have understood this point! Henry’s law is then used to
convert the partial pressure of gas in the liquid to concentration of gas in
the liquid. Henry’s law calculates the concentration of dissolved gas that is
free in solution and does not include any gas that is present in bound form
(e.g., bound to hemoglobin).

Cx = Px x solubility

Cx is concentration of dissolved (free) gas in units of ml gas/100 ml blood


(also called volume %), Px is the partial pressure of the gas in mm Hg, and
solubility is the solubility of the gas in units of ml gas/100 ml blood/mm
Hg.

825
Physical Chemistry of Gases - Dr. Costanzo

Solubilities of O2 and CO2 in blood are as follows:

Solubility in blood (ml gas/100 ml blood/mm


Hg)
O2 0.003 ml O2/100 ml blood/mm Hg
CO2 0.07 ml CO2/100 ml blood/mm Hg

II. FORMS OF GASES IN SOLUTION

In air there is only one form of gas, the gaseous form (!), which is expressed as a
partial pressure in units of mm Hg. In blood, gases can be carried in dissolved
form (Henry’s law), bound to proteins such as hemoglobin, or chemically
modified.

Dissolved gas. All the relevant gases (O2, CO2, and N2) are carried to some extent
in dissolved form. Henry’s law relates concentration of the gas to its partial
pressure. One corollary of Henry’s law is that only dissolved gas creates a partial
pressure; bound and chemically modified forms do not contribute to the partial
pressure of the gas. N2 is only found in the dissolved form, it is never bound or
chemically modified.

Bound gas. O2, CO2, and CO are bind to hemoglobin, which contribute
significantly to their carriage in blood. CO2 also binds to plasma albumin.

Chemically modified. The most important example of a chemically modified gas


is the conversion of CO2 to HCO3- in red blood cells.

III. GAS EXCHANGE

A. Diffusion of gases – Fick’s Law

Vx = D A ΔP
Δx

Vx is volume of gas transferred per unit time, D is the diffusion coefficient


for the gas, A is surface area, ΔP is partial pressure difference for the gas,
and Δx is the thickness of the membrane (e.g., the alveolar-capillary
barrier).

The driving force for gas diffusion is the partial pressure difference of
the gas (ΔP) across the membrane or capillary wall. For example, if the
PO2 of alveolar gas is 100 mm Hg and the PO2 of mixed venous blood

826
Physical Chemistry of Gases - Dr. Costanzo

entering the pulmonary capillaries is 40 mm Hg, then the driving force for
diffusion of O2 is the difference in partial pressures across the alveolar-
pulmonary capillary barrier, or 60 mm Hg. O2 will diffuse until the PO2 of
pulmonary capillary blood is 100 mm Hg, at which point the partial
pressure gradient is dissipated and there is no more driving force for O2
diffusion.

The diffusion coefficient of the gas, D, is inversely correlated with the


molecular weight of the gas and directly correlated with the solubility of
the gas. For example, DCO2 is >> DO2 (20 times greater). In respiratory
physiology, we combine diffusion coefficient, surface area, and membrane
thickness in the Fick equation into a single term called lung diffusing
capacity (DL).

B. Lung diffusing capacity – DL

As noted above, several factors (D, A, and Δx) from Fick’s diffusion
equation are combined into the lung diffusing capacity, DL. DL also takes
into account the time required for gas (e.g., O2) to combine with proteins
such as hemoglobin. DL is measured with CO (i.e., DL CO) because CO
transfer across the alveolar-capillary barrier is limited exclusively by
diffusion. (In the measurement, called the single breath method, a single
inspiration of a dilute mixture of CO is made, and the rate of
disappearance of CO from alveolar gas is measured.)

Vx = DL x ΔP

Increases in DL. DL is increased in exercise, where there are more open


capillaries and more surface area for gas exchange.

Decreases in DL. DL is decreased when there is an increased diffusion


distance (e.g., fibrosis and pulmonary edema) or decreased surface area
for diffusion (e.g., emphysema). DL also is decreased in anemia where the
decreased hemoglobin concentration in blood decreases the hemoglobin-
binding component of the DL measurement.

827
Physical Chemistry of Gases - Dr. Costanzo

IV. OVERVIEW OF GAS EXCHANGE IN THE LUNGS

Figure 1.

828
Physical Chemistry of Gases - Dr. Costanzo

Figure 2.

The first figure shows an alveolus and a pulmonary capillary. The pulmonary
capillary is perfused with mixed venous blood from the right heart. Gas exchange
occurs across the alveolar-pulmonary capillary barrier -- O2 diffuses from alveolar
gas into pulmonary capillary blood and CO2 (produced in the tissues) diffuses
from pulmonary capillary blood into alveolar gas. Pulmonary capillary blood exits
the lungs by the pulmonary vein, goes to the left heart and becomes systemic
arterial blood.

The second figure shows the average values for PO2 and PCO2 in various locations.
Dry inspired air has a PO2 of 160 mm Hg, but no CO2. When this air enters the
trachea, it is humidified and the PO2 is lowered to 150 mm Hg because of the
obligatory PH2O of 47 mm Hg ([760 mm Hg - 47 mm Hg] x 0.21 = 150 mm Hg).
In alveolar gas, the values for PO2 and PCO2 change significantly. (The notation
small capital “A” indicates alveolar gas.). PAO2 is 100 mm Hg because O2 has
diffused from alveolar gas into pulmonary capillary blood until equilibration
occurs. PACO2 is 40 mm Hg because CO2 has diffused from capillary blood into
alveolar gas until equilibration occurs. In the steady state, the amounts of O2 and
CO2 transferred correspond to the amounts of O2 consumed and CO2 produced by

829
Physical Chemistry of Gases - Dr. Costanzo

the body. Thus, pulmonary capillary blood, which becomes systemic arterial
blood, normally equilibrates with alveolar gas and has a PaO2 of 100 mm Hg and
a PCO2 of 40 mm Hg. This blood circulates to the tissues, where O2 is consumed
and CO2 is produced, and mixed venous blood has a PvO2 of 40 mm Hg and a
PvCO2 of 46 mm Hg.

V. DIFFUSION- AND PERFUSION-LIMITED GAS EXCHANGE

A. Diffusion-limited gas exchange

In diffusion-limited gas exchange, the total amount of gas transferred


across the alveolar-capillary barrier is limited by the diffusion process
(driven by the partial pressure gradient for the gas). The partial pressure
gradient for the gas is maintained along the length of the capillary.

Diffusion-limited gas exchange is illustrated by the transport of O2 during


strenuous exercise, in lung diseases such as emphysema and fibrosis, and
by the transport of CO (shown in the figure below, Panel A). In the figure,
the shaded area shows the partial pressure gradient for CO between
alveolar gas (PA) and pulmonary capillary blood (Pa) along the length of
the capillary. Blood entering the capillary has no CO and there is a huge
partial pressure gradient for CO diffusion from alveolar gas into capillary
blood. As CO diffuses into the blood, it binds to hemoglobin with a high
affinity and very little CO is left free in solution. Thus, the partial pressure
of CO rises very little along the length of the capillary, the partial pressure
gradient is maintained for the entire capillary, and diffusion continues.
There is no equilibration of CO!

B. Perfusion-limited gas exchange

In perfusion (or blood flow)-limited gas exchange, the amount of gas


transferred is limited by blood flow. The partial pressure gradient for
the gas is not maintained, i.e., there is equilibration of the gas at some
point along the pulmonary capillary. In these cases, the only way to
transfer more gas is by increasing blood flow.

Perfusion-limited gas exchange is illustrated by the transfer of O2 (resting


conditions) and CO2, and by the transfer of N2O (shown in the figure
below, panel B). N2O is not bound in the blood at all, it is only present in
the free, dissolved form that creates a partial pressure. Blood entering the
pulmonary capillary initially has no N2O. N2O diffuses down its partial
pressure gradient from alveolar gas into pulmonary capillary blood. The
partial pressure of N2O in blood rises rapidly (because none is bound);
when it equals the partial pressure in alveolar gas, there is no more transfer
of N2O. The only way to transfer more N2O is to increase blood flow.

830
Physical Chemistry of Gases - Dr. Costanzo

Figure 3.

C. O2 — sometimes perfusion-limited, sometimes diffusion-limited

O2 transport is normally perfusion-limited. That is, O2 equilibrates


between alveolar gas and pulmonary capillary blood and the partial
pressure gradient (driving force for diffusion) dissipates. The only way to
increase the amount of O2 transferred is to increase blood flow. However,
this is not the whole story. Under certain conditions (usually pathologic,
but also including strenuous exercise), O2 does not equilibrate; the O2
partial pressure gradient is maintained along the length of pulmonary
capillary, and O2 transfer converts to a diffusion-limited process.

831
Physical Chemistry of Gases - Dr. Costanzo

Figure 4.

In fibrosis, O2 transfer converts to a diffusion-limited process (Panel A).


Let’s assume that mixed venous PO2 is the usual value of 40 mm Hg. This
blood enters the pulmonary capillaries. The diffusion process is seriously
impaired, however, because of thickening of the alveolar membranes,
which decreases DL. O2 does not equilibrate between alveolar gas and
pulmonary capillary blood and the blood leaving the pulmonary capillaries
and becoming systemic arterial blood has a very reduced PO2.

At high altitude, the person with fibrosis is in even worse shape. Now the
alveolar PO2 is reduced (because of the decrease in barometric pressure).
For illustration, alveolar PO2 in this example is shown as 50 mm Hg.
People with normal lungs will equilibrate O2 (albeit more slowly because
of the decreased partial pressure gradient), and their arterial PO2 will be 50
mm Hg. People with fibrosis, however, will not equilibrate O2 and their
arterial PO2 will be less than 50 mm Hg (in this example, 30 mm Hg).

832
Physical Chemistry of Gases - Dr. Costanzo

VI. PRACTICE QUESTIONS

1. If barometric pressure is 740 mm Hg, and the fractional concentration of


O2 is 21%, of N2 is 79%, and CO2 is 0, what are the partial pressures in a
humidified mixture of these three gases?

2. If alveolar gas has a partial pressure of O2 of 150 mm Hg, what is the


concentration of dissolved O2 in blood that is equilibrated with that
alveolar gas?

3. A person at sea level breathes a mixture containing 0.1% carbon monoxide


(CO). The uptake of CO was measured in the single breath method to be
28 ml/minute. What is the lung diffusing capacity for CO (DLCO)? ( Hint:
VCO = DL x ΔP)

4. In perfusion-limited O2 exchange, the PO2 at the end of the pulmonary


capillary is

A. Less than the PO2 in alveolar air.


B. Equal to the PO2 in mixed venous blood.
C. Equal to the PO2 in alveolar air.
D. Greater than the PO2 of alveolar air.
E. Greater than the PO2 of systemic arterial blood.

EXPLANATIONS

1. PO2 = (740-47) x 0.21 = 145.5 mmHg


PN2 = (740-47) x 0.79 = 547.5 mmHg
PCO2 = (740-47) x 0 = 0

2. PaO2 = 150 mmHg


Dissolved O2 = 150 mmHg x 0.003 ml O2/100 ml blood/mmHg
= 0.45 ml O2/100 ml blood, or 0.45 vol%

3. Answer: 39.3 ml/min/mm Hg

4. Answer = C. Begin by reminding yourself of the definition/description of


perfusion-limited gas exchange as applied to O2 (i.e., O2 moves from
alveolar gas into pulmonary capillary blood). Perfusion-limited means the
gas equilibrates across the alveolar-pulmonary capillary barrier. Thus,
the PO2 of pulmonary capillary blood equilibrates with, and becomes
equal to, the PO2 of alveolar air. Additional comment on this question:
Choice D (greater than PO2 of alveolar air) is a particularly bad answer
because it implies that pulmonary capillary blood can achieve a higher
PO2 than alveolar gas – it can’t! It can go as high as, but never higher!

833
O2 Transport - Dr. Costanzo

O2 Transport
Linda Costanzo, Ph.D.

OBJECTIVES:

After studying this lecture, the student should understand:

1. How oxygen is carried in blood.


2. Features of adult hemoglobin and its variants.
3. Features of the oxygen-hemoglobin dissociation curve, including the sigmoidal
shape and P50.
4. The meaning of right- and left-shifts of the oxygen-hemoglobin dissociation curve
and what causes each.
5. The effect of carbon monoxide on the oxygen-hemoglobin dissociation curve.
6. How to calculate the oxygen content of blood.

I. FORMS OF O2 IN BLOOD

In the blood, O2 is carried in two forms: dissolved O2 and O2 bound to


hemoglobin (called O2- hemoglobin).

Dissolved O2. Dissolved O2 constitutes 2% of the total O2 content of the blood.


The quantity of O2 dissolved in blood is described by Henry’s law, which states
that the concentration of dissolved gas is proportional to its partial pressure; the
proportionality constant is the solubility. The solubility of O2 is 0.003 ml O2/100
ml blood/mm Hg. Thus, for an arterial PO2 of 100 mm Hg, the dissolved O2
content is 0.3 ml O2/100 ml blood. Alone, dissolved O2 is woefully inadequate for
O2 delivery to the tissues; we must have O2-hemoglobin!

O2-hemoglobin. The remaining 98% of O2 in blood is O2-hemoglobin, the major


topic of this lecture.

II. HEMOGLOBIN

A. Features of hemoglobin molecule

Hemoglobin, a globular protein, has four subunits. Each subunit has a


heme moiety (an iron-binding porphyrin) and a polypeptide chain. Adult
hemoglobin (called HbA) has two α chains and two β chains, thus it is
designated α2 β2. Each subunit can bind one molecule of O2; thus a
hemoglobin molecule can bind a total of four O2, which is referred to as
100% saturation. In the oxygenated form, hemoglobin is called

834
O2 Transport - Dr. Costanzo

oxyhemoglobin; in the unoxygenated form, it is deoxyhemoglobin. To


bind O2, iron must be in the ferrous state, Fe2+.

B. Variants of hemoglobin

1. Adult hemoglobin (HbA). See above.


2. Fetal hemoglobin (HbF). In fetal hemoglobin, HbF, the two β
chains are replaced by γ chains, thus it is described as α2 γ2. This
modification to the hemoglobin molecule results in a higher
affinity for O2 (than HbA), which lowers the PO2 (free, dissolved
O2) and facilitates movement of O2 from the maternal to the fetal
circulation. Within the first year of life HbF is replaced by HbA.
3. Methemoglobin. In methemoglobin, iron is in the ferric (Fe+3)
state instead of the normal Fe2+ state. Methemoglobin does not
bind O2. Methemoglobinemia is caused by drugs that oxidize Fe2+
to Fe+3, (e.g., sulfonamides) or congenital enzyme deficiency of
methemoglobin reductase (the red blood cell enzyme that normally
keeps iron in its Fe2+ state).
4. Hemoglobin S (HbS). HbS is the hemoglobin variant that causes
sickle cell disease. In HbS, the α subunits are normal and the β
subunits are abnormal, so it is designated αA2 βS2. The
deoxygenated form of αA2 βS2 forms sickle-shaped rods that distort
the shape of the red blood cells (i.e., “sickling”) and causes them to
occlude small blood vessels. αA2 βS2 has a lower affinity for O2
than does normal HbA.

III. O2-HEMOGLOBIN DISSOCIATION CURVE

Figure 1.

835
O2 Transport - Dr. Costanzo

O2 binds reversibly to the heme groups on hemoglobin. Therefore, each


hemoglobin molecule has the capacity to bind four molecules of O2. The percent
saturation (“O2 sats”) tells what percent of heme groups are bound to O2. When
all four hemes are bound to O2, there is 100% saturation, when three hemes are
bound to O2, there is 75% saturation, etc. The O2-hemoglobin dissociation curve
shows the relationship between % saturation and PO2 of the blood. This is one of
the most famous and most important curves in all of physiology! Live it, love it!

For convenience, the table below gives various values of PO2 and the
corresponding % saturation for the normal O2-hemoglobin dissociation curve.

PO2 % Saturation
10 25%
20 35%
25 50% (P50)
30 60%
40 75% (mixed venous blood)
50 85%
60 90%
80 96%
100 98% (≈ 100% arterial blood)

A. Sigmoidal shape

The O2-hemoglobin dissociation curve has a sigmoidal shape. Thus, rather


than a linear relationship between % saturation and PO2, the % saturation
increases steeply between a PO2 of zero and 40 mm Hg, then increases less
steeply between 40 and 60 mm Hg, and then is nearly flat between 60 and 100
mm Hg. (Note that one of the implications of the nearly “flat” portion of the
curve between 60 and 100 mm Hg is that changes in arterial PO2 in this range
have little effect on the % saturation of hemoglobin and therefore the amount
of O2 carried in the blood.)

The sigmoidal shape results from positive cooperativity of O2 binding to


hemoglobin. As each successive O2 binds, it increases the affinity for the next
O2. Binding of the first O2 increases the affinity for the second O2, etc.
Affinity for the fourth (last) O2 is the highest, which corresponds to the
portion of the curve where % saturation is near or at 100%.

B. P50

By definition, P50 is the PO2 that corresponds to 50% saturation. Changes in


P50 reflect changes in the affinity of hemoglobin for O2. Increases in P50
reflect decreased affinity. Decreases in P50 reflect increased affinity.

836
O2 Transport - Dr. Costanzo

C. Right- and left-shifts of the O2-hemoglobin dissociation curve

Figure 2.

Changes in affinity of hemoglobin for O2 produce changes in the P50 and shift
the O2-hemoglobin dissociation curve to the right or left.

Right-shifts of the O2-hemoglobin dissociation curve occur when there is a


decreased affinity of hemoglobin for O2, which produces an increase in P50.
When affinity is decreased, unloading of O2 is facilitated. The factors that
cause a right-shift make sense in terms of this decreased affinity and include:

1. Increases in PCO2 and decreases in pH, such as those occurring when


there is increased metabolic activity in a tissue (e.g., during exercise.
Called the Bohr effect.
2. Increases in temperature, such as during exercise.
3. Increases in 2,3-diphosphoglycerate (2,3-DPG). 2,3 DPG binds to the
β chains of deoxyhemoglobin and reduces their affinity for O2. During
hypoxia (such as at high altitude), 2,3 DPG production in red cells
increases, causing a helpful decrease in the affinity of hemoglobin for O2
(facilitates O2 unloading in tissues).

837
O2 Transport - Dr. Costanzo

Left-shifts of the O2-hemoglobin dissociation curve occur when there is


increased affinity of hemoglobin for O2, which produces a decrease in P50.
When affinity is increased, unloading of O2 is more difficult. The factors
causing a left-shift are:

1. Decreases in PCO2 and increases in pH, such as when there is decreased


metabolic activity in a tissue.
2. Decreases in temperature.
3. Decreases in 2,3-DPG.
4. Hemoglobin F (HbF). The mechanism of the left-shift with HbF relates to
2,3 DPG, which binds less avidly to the γ chains of HbF than to the
β chains of HbA. With less 2,3 DPG bound to HbF, the O2 affinity
increases, lowering the PO2 of the fetus and facilitating O2 diffusion from
mother to fetus.
5. Carbon monoxide (CO) not only causes a left-shift, but also a decrease in
O2-binding capacity (see below).

D. CO poisoning

Figure 3.

838
O2 Transport - Dr. Costanzo

Carbon monoxide (CO) binds to hemoglobin (to form carboxyhemoglobin)


with an incredibly high affinity, 250 times that of O2! Any heme groups bound
to CO cannot bind to O2, so CO poisoning decreases the O2-binding capacity
of hemoglobin. In the figure, for illustration, the O2-binding capacity of
hemoglobin was reduced to 50% of normal, meaning ½ of the heme sites were
occupied by CO. CO also increases the affinity of hemoglobin for whatever
O2 is bound (a left-shift of the O2-hemoglobin dissociation curve). Thus, the
effects of CO poisoning are devastating for O2 delivery to tissues: less O2 is
bound and the O2 that is bound is less readily released.

E. O2 Content of Blood

Blood flow and O2 content of blood are the major factors determining O2
delivery to tissues. O2 content of blood is comprised of dissolved O2 and
O2-hemoglobin.

1. Dissolved O2 is easy — it is described by Henry’s law as the PO2 times


the solubility of O2 in blood.

Dissolved O2= PO2 x solubility


P x 0.003 ml O2/100 ml
= O2
blood/mm Hg

Thus, at the normal arterial PO2 of 100 mm Hg:

100 mm Hg x 0.003 ml O2/100 ml


Dissolved O2 =
blood/mm Hg
= 0.3 ml O2/100 ml blood, or 0.3 vol %

2. O2 bound to hemoglobin depends upon three things:

a. Hemoglobin concentration. The normal value is 15 g/100 ml


of blood.
b. O2-binding capacity. The maximal amount of O2 that can be
bound to hemoglobin when there is 100% saturation of heme
sites. The O2-binding capacity is normally 1.34 ml O2/g
hemoglobin.
c. % Saturation. % of heme groups bound to O2. Varies from
zero to 100%

Thus, if the hemoglobin concentration of blood is 15 g/100 ml, the O2-


binding capacity of that hemoglobin is 1.34 ml O2/g, and the

839
O2 Transport - Dr. Costanzo

% saturation of arterial blood is 100% (at an arterial PO2 of 100 mm


Hg):

O2-hemoglobin = 15 g/100 ml x 1.34 ml O2/g x 100%


20.1 ml O2/100 ml blood, or 20.1 vol
=
%

If the % saturation of hemoglobin is less than 100% then, accordingly,


there will be less O2-hemoglobin.

3. Now, to calculate the total O2 content of blood at a PO2 of 100 mm Hg.

O2 content of blood = O2-hemoglobin + dissolved O2


20.1 ml O2/100 ml + 0.3 ml O2/100
=
ml
= 20.4 ml O2/100 ml, or 20.4 vol%

**Special note: ml gas/100 ml is also called “volume %.” Thus, an O2


content of 20.4 ml O2/100 ml blood is called “20.4 volume %” for short.

840
O2 Transport - Dr. Costanzo

4. Summary of O2 transport

Figure 4.

Humidified tracheal air has a PO2 of 150 mm Hg. Alveolar air has a lower
PO2 of 100 mm Hg because O2 has diffused from alveolar gas into
pulmonary capillary blood. Pulmonary capillary blood, which becomes
systemic arterial blood, equilibrates with alveolar gas, so it too has a PO2
of 100 mm Hg. The PaO2 of 100 mm Hg corresponds to 100% saturation
of hemoglobin on the O2-hemoglobin dissociation curve. The O2 content
of systemic arterial blood is the sum of dissolved O2 and O2-hemoglobin
per our discussion above. Dissolved O2 was 0.3 vol% and O2-hemoglobin
was 20.1 vol% for a grand total of 20.4 vol% in systemic arterial blood. In
the tissues, O2 diffuses from the capillaries to the tissues for aerobic
metabolism. Thus mixed venous blood has as lower PO2 of 40 mm Hg, a
correspondingly lower % saturation of 75% (read it off the O2-hemoglobin
curve!), and a correspondingly lower O2 content of 15 vol %. Thus 5 vol
% of O2 must have been transferred to the tissues. Mixed venous blood
will be re-loaded with O2 in the next pass through the lungs.

841
O2 Transport - Dr. Costanzo

IV. PRACTICE QUESTIONS

1. A person who is hypoxemic has an arterial PO2 of 75 mm Hg and a venous


PO2 of 30 mm Hg. This person has a normal hemoglobin concentration and
a normal O2-binding capacity. Hemoglobin is 85% saturated at 75 mm Hg
and 55% saturated at 30 mm Hg. For arterial and venous blood, calculate
dissolved O2, O2-hemoglobin, and total O2 content. What was O2
consumption by the tissues (in vol %)?

2. What is the effect of increased PCO2, increased pH, and CO on O2-binding


capacity, P50, and affinity of hemoglobin for O2?

3. A right-shift of the O2-hemoglobin curve is:

A. Associated with a decrease in P50.


B. Associated with an increase in O2 content of blood at a PO2 of 50
mm Hg.
C. Caused by an increase in blood pH.
D. Caused by an increase in 2,3 DPG concentration.

4. Which of the following causes a decrease in O2-binding capacity of


hemoglobin?

A. Decreased hemoglobin concentration


B. CO poisoning
C. Decrease in arterial PO2 to 60 mm Hg
D. Increase in arterial PO2 to 120 mm Hg
E. Left-shift of the O2-hemoglobin curve

EXPLANATIONS

1.

842
O2 Transport - Dr. Costanzo

2.

3. Answer = D. Start by drawing a normal O2-Hb curve and a right-shifted curve.


Write down what you know: associated with increased P50, decreased affinity, and
decreased O2 content at a given PO2; caused by increased PCO2, decreased pH,
and increased 2,3 DPG. I hope you see why choice B is incorrect – at a PO2 of 50
mm Hg, the right-shifted curve has a lower % saturation, thus a lower O2 content.
BTW: “associated with” means any association, cause or effect.

4. Answer = B. Get oriented! What is O2-binding capacity of hemoglobin? It is how


many ml of O2 each gram of hemoglobin can hold at 100% saturation. Decreased
hemoglobin concentration does not affect the amount of O2 each gram of
hemoglobin can hold. Decreasing or increasing PO2 changes the % saturation of
hemoglobin, but does not affect the amount of O2 that hemoglobin can hold at
100% saturation. Same with a left-shift of the curve; it increases the % saturation
at a given PO2 and increases affinity, but does not alter the maximum amount of
O2 that hemoglobin can hold. CO attaches to binding sites on hemoglobin and
prevents binding of O2, which reduces the O2-binding capacity of hemoglobin. I
suspect a few people learned something from this question!

843
CO2 Transport - Dr. Costanzo

CO2 Transport
Linda Costanzo, Ph.D.

OBJECTIVES:

After studying this lecture, the student should understand:

1. How carbon dioxide is carried in blood, especially as bicarbonate.


2. The effect of oxygen on the carbon dioxide content of blood.

CO2 is carried in the blood in three forms: dissolved CO2, CO2 bound to proteins such as
hemoglobin (called carbaminohemoglobin) and, most importantly, as HCO3-.

I. DISSOLVED CO2 is described by Henry’s law as the partial pressure times the
solubility and accounts for 5% of the total CO2 content of blood. The solubility of
CO2 in blood is 0.07 ml CO2/100 ml blood/mm Hg (more than twenty times the
solubility of O2). Thus, in arterial blood with a PCO2 of 40 mm Hg, dissolved CO2
is:

Dissolved CO2 = PCO2 x solubility

= 40 mm Hg x 0.07 ml CO2/100 ml blood/mm Hg

= 2.8 ml CO2/100 ml blood, or 2.8 vol%

II. CARBAMINOHEMOGLOBIN

CO2 binds to terminal amino groups on hemoglobin and plasma proteins such as
albumin, so-called carbamino compounds. Carbamino compounds account for
3% of the total CO2 in blood, 2/3 of which is carbaminohemoglobin.

III. HCO3-

92%, of the CO2 is carried in blood as HCO3-. The reactions that produce HCO3-
are as follows:

CO2 + H2O W H2CO3 W H+ + HCO3-


Carbonic anhydrase

In the tissues, CO2 generated from aerobic metabolism is added to venous blood.
In the red cells of venous blood, the above reactions occur, generating H+ and
HCO3-. The H+ remains inside the red cells, buffered by deoxyhemoglobin. The
HCO3- exchanges with Cl- across the red cell membrane (Cl– HCO3- exchange)
and travels to the lungs in the plasma. In the lungs the reactions occur in reverse,

844
CO2 Transport - Dr. Costanzo

HCO3- re-enters the red cells, CO2 is regenerated and then is expired.

Figure 1.

The graph below shows the relationship between the CO2 content of blood and
PCO2. In contrast to O2, which has a sigmoidal relationship with PO2, CO2 content
is linear as PCO2 changes over the physiologic range. Even though the carbamino
portion of the CO2 content is saturable as PO2 increases, the dissolved and HCO3-
forms are linear. Note that the lower the PO2, the higher the CO2 content, which is
attributed to greater binding of CO2 to hemoglobin in its deoxygenated form,
called the Haldane effect. (Makes sense, since CO2 must be carried to the lungs
in venous blood where hemoglobin is relatively deoxygenated.)

Figure 2. (Fig.35-9) Blood CO2 equilibrium curves (atrial and venous).


Venous blood can transport more CO2 than arterial blood at any given PCO2.
Compared with the hemoglobin-oxygen equilibrium curve, the CO2 curves
are essentially straight lines between PCO2 of 20 and 80 mm Hg.

845
CO2 Transport - Dr. Costanzo

IV. PRACTICE QUESTIONS

1. Which of the following statements is correct about CO2 transport in


blood?

A. Systemic venous pH is higher than systemic arterial pH.


B. Deoxyhemoglobin carries more CO2 than oxyhemoglobin.
C. Most HCO3- is carried inside the red blood cells.
D. Binding of H+ to hemoglobin increases the affinity of hemoglobin
for O2.

EXPLANATIONS

2. Answer = B. Deoxyhemoglobin carries more CO2 than oxyhemoglobin


(Haldane effect). Binding of H+ to hemoglobin decreases the affinity for
O2 (Bohr effect). Most HCO3- is carried in the plasma, not in the red cells.
Systemic venous pH is lower than systemic arterial pH because CO2, a
weak acid, has been added to it.

846
Lab Group Assignments

Lab Group 1
Monday, February 16, 2009
9 a.m. – 12 noon

Thursday, March 19, 2009


8 – 10 a.m.

Room 204, Egyptian Faculty: Dr. Gea-Ny Tseng

Abedi, Scott Sina


Aboulhosn, Lara
Adam, Jason David
Al-Hashimi, Saba Kamel
Alexander, Nebu
Ali Imtiaz, Mubdiul
Ally, Ryan Inshan
Anderson, Bradley Barrett
Anthony, Craig Lincoln
Armstrong, Amy Elizabeth

Room 206, Egyptian Faculty:Dr. Shyama Masilamani

Artrip, Sarah Beth


Avula, Danielle
Baig, Kimberlyn Maravet
Balasubramaniam, Saranya Chellam
Bauer, Clayton Thomas
Beckta, Jason Mark
Bennion, Nathan Ronald
Berhanemeskel, Mahlet Girma
Bharucha, Grishma

Room 207, Egyptian Faculty:Dr. Jack Grider

Bhowmik, Nirjhor Mondril


Binyamin, Tamar Rebecca Mya
Black, Scott Bayles
Broughton, Robert Oliver
Brown, Ashley Landon
Brueckner, Jessica Colleen
Bui, Linh Ngoc
Buquo, Lauren Elizabeth
Burk, Spencer Mark
Burrer, Sandra K

847
Lab Group Assignments

Room 208, Egyptian Faculty: Dr. Clive Baumgarten

Campbell, Heather Sterling


Carter-Loebig, Nicole Sherron
Causey, Stephen Merrill
Chande, Neha Dilip
Chang, Andrew Seung
Chang, Sherry
Chen, Hsuan-Chih
Chen, Renee Tran
Collins, Lindsay Ann

Room 209, Egyptian Faculty:Dr. Joseph Feher


Colton, Adrianne Michelle
Dante, Siddhartha Allen
Davidson, Donald Dale JR
Davis, Timothy Hamilton
Delacruz, Panfilo Castro
Dholaria, Kevin
DiPasquale, Kathleen Diana
Disharoon, Steven James
Dockery, Lauren Elizabeth
Drake, Nicholas Matthew

Room 210, Egyptian Faculty: Drs. Ivo and Luciana Torres

Egbert, Bryce Larre


Emejuru, Jason Uchechi
Emmanuel, Joann Rohini
Eskildsen, Scott Martin
Faramand, Rawan Ghassan
Filler, Robert
Forbes, Michael
Fox, Benjamin D
Garnett, Katherine Simms

Room 211, Egyptian Faculty: Dr. Scott Walsh

Gibson, Peter Desmond


Gillen, William Spencer
Givens, Nathan Campbell
Godshall, Elizabeth
Goeden, Marcie Jean
Gokli, Ami
Goldman, Ashton Howard
Goldstein, Shira Kate
Goodell, Cara Amy Raphael
Grant, Mai Sorrel

848
Lab Group Assignments

Lab Group 2

Thursday, February 12, 2009


9 a.m. – 12 noon

Friday, March 20, 2009


8 – 10 a.m.

Room 204, Egyptian Dr. Raphael Witorsch

Grewal, Sharleen Kaur


Haines, Amanda Kathryn
Hamadani, Lara
Hamdani, Tarik James
Haney, Joshua William
Hanger, Michael William
Hardenbrook, Spencer James
Harrison, Mary Virginia
Heitz, Christopher Todd

Room 206, Egyptian Dr. Roland Pittman

Hendi, Aditi S.
Herczyk, Matthew David
Hillenbrand, Karl David
Hoang, Valerie Magdelena
Hou, Angela Yingchun
Hoyt, Jennifer Renee
Hsu, David William
Humsi, Michael Kamil
Imbery, Terence Edward
Jahanshahi, Pooya

Room 207, Egyptian Dr. Martin Mangino

Janovski, Alexander John


Joyce, John Michael
Junkin, Andrew Ryan
Kanel, Jason Alan
Kelberg, Matthew Edward
Kim, Michael Youn-Il
King, Jamie Leigh
Kolber, Marcin Konrad
Koors, Paul Daniel

849
Lab Group Assignments

Room 208, Egyptian Dr. Wayne Barbee

Kye, Cecilia
Le, Anh Kim
Le, John Chuong Quang
Lee, Ran
Leonard, Rachel Ann
Lo, Patricia Wai Yin
Loken, Erik Kristen
Lung, Tina Kathy
Maldonado, Michael Damian
Mann, Nathaniel

Room 209, Egyptian Dr. Ramzi Ockaili

Manson, Mellisa Serwah


Martinovic, Maryann Elizabeth
Mateer, Erin Abbott Street
Mays, Mary Katherine
Mba, Mba Uzoma Udo
McCaughan, Michael Scott
Meliagros, Pete Dennis
Miller, Devin Tatcher
Modi, Parth Chandrakant

Room 210, Egyptian Dr. Richard Costanzo

Mohan, Shiva C.
Morehouse, Bethany Caroline
Mulye, Anita Diwakar
Muqri, Aceela
Nardone, Vincent John
Nelson, Mary Ann
Newton, Daniel Henry
Nguyen, Eric N
Nguyen, Monika Dao
Nottingham, Charles Upshur

Room 211, Egyptian Dr. Vijay Lyall

O'Connor, James Patrick


Oh, Kimberly Sun
Ohene-Baah, Nana Yaw
Painter, Matthew David
Parikh, Nishant Bharat
Patel, Jay Mukesh
Patel, Rushita Mrugendra
Patel, Vaibhav
Peterson, Sarah Rose

850
Lab Group Assignments

Lab Group 3

Friday, February 13, 2009


9 a.m. – 12 noon

Friday, March 20, 2009


1 – 3 p.m.

Room 204, Egyptian Dr. Linda Costanzo

Peterson, Timothy E
Poliquin, Rachel C
Poll, Milt Grover
Potts, Andrew J
Powell, Tanisha Michole
Powelson, Palen Ann
Raghavan, Rahul Veera
Rajendran, Bipin
Rajkumar, Jennifer Jo-Ann
Ramireddy, Archana

Room 206, Egyptian Dr. Fadi Salloum

Rankin, Trevan Monteil


Repic, Adrian
Rich, Melissa Anne
Richey, Justin Dan
Riggs, Emily Kim
Robertson, Ryan Nexsen
Rose, Hannah Iris
Rowlett, Robert Mitchell
Ruhi, Emre
Saffouri, George Bassam

Room 207, Egyptian Dr. Mohammed Kalimi

Sasinowski, Maciek
Scott, Chantal Devaru
Sethi, Ashish
Shafer, Sarah San Young
Sherwood, Alex Berry
Shoemaker, Rebecca Ryan
Shou, James Young
Siegel, Julia Anne
Sienkiewicz, Lisa

851
Lab Group Assignments

Room 208, Egyptian Dr. Margaret Biber

Singh, Malkit Kaur


Sirkin, Maxwell Richard
Smith, Cameron Reid
Smith, Christina Marie
Spetzler, Karli Ann
Spivey, Steven James
Sponholz, Luke Richard
Spradlin, Elizabeth Ann
Stempel, Hilary Elyse
Stranix, John Timothy

Room 209, Egyptian Dr. Alexander Golub

Stuart, Adam Michael


Swift, Jesse Scott
Talreja, Neha
Taormina, Yula Alexandria
Tarasidis, George Stratos
Tiangco, Noreen Enriquez
Titchner, Timothy Joseph
Titerence, Rachel Helen
Toensing, Christopher Cody

Room 210, Egyptian Dr. Helena Carvalho

Tozer, Jordan Travis


Trang, Vinh Quang
Trujillo, John Francisco
Turner, Patrick Lawton
Washington, Monica Michaelle
Watson, Sara Elizabeth
Watterson, Christopher Travis
Way, Serena
Wilkinson, Kathryn Elise

Room 211, Egyptian Dr. Murthy Karnam

Wilson, Robert Travis


Wilson, Sean Patrick
Wise, Elizabeth Anne
Wu, Chun-Yu
Yang, Daniel Tien
Yester, Jessie Wettig
Zedler, Kathryn Cecile
Zimmerman, William Fairchild

852
Pulmonary Circulation - Dr. Costanzo

Pulmonary Circulation
Linda Costanzo Ph.D.

OBJECTIVES:

After studying this lecture, the student should understand:

1. The differences between pressures in the pulmonary and systemic circulations.


2. How to calculate pulmonary vascular resistance.
3. How (and why) blood flow is unevenly distributed in the upright lung.
4. Regulation of pulmonary blood flow, especially the concept of hypoxic
vasoconstriction.
5. The relationship between alveolar ventilation and alveolar PCO2 and how it is
described by the alveolar ventilation equation.
6. The relationship between alveolar ventilation and alveolar PO2 and the alveolar
ventilation equation.

I. BIG PICTURE OF PULMONARY CIRCULATION AND PRESSURES

Figure 1.

Blood is supplied to the lungs by the pulmonary artery, which receives mixed
venous blood from the right heart. The pulmonary arteries branch (like the

866
Pulmonary Circulation - Dr. Costanzo

airways) as far as the terminal bronchioles, then break up to form dense capillary
networks in the alveolar walls. The density of the capillary network makes for
extremely efficient gas exchange.

Pulmonary vascular pressures are much lower than their counterparts in the
systemic circulation. For example, mean pulmonary artery pressure is 15 mm Hg
(25/8) compared with mean aortic pressure of 100 mm Hg (120/80). Consistent
with these low pressures, the walls of the pulmonary arteries are thin and contain
relatively little smooth muscle. One important “issue” for the pulmonary
capillaries is the fact that they are surrounded by alveoli, which are gas-filled.
Capillary pressures are actually close to alveolar pressures. Thus, it is possible
under some circumstances for capillaries to be compressed and even collapse
under the alveolar pressure (more later).

Figure 2.

II. PULMONARY VASCULAR RESISTANCE

In cardiovascular physiology, we talk about total peripheral resistance, or


systemic vascular resistance (SVR). We apply the pressure, flow, resistance
relationship that is analogous to Ohm’s law for electrical circuits. In similar
fashion, we describe pulmonary vascular resistance (PVR):

PVR = Pulmonary artery pressure - pulmonary venous pressure


Blood flow

Mean pulmonary artery pressure is 15 mm Hg, pulmonary venous pressure is


approximately 8 mm Hg, and blood flow is cardiac output of the right heart.
Pulmonary pressures are much lower than systemic pressures, yet blood flow

867
Pulmonary Circulation - Dr. Costanzo

(cardiac output) is the same on both sides of the circulation. Logically, pulmonary
vascular resistance must also be much lower than systemic vascular resistance.

III. DISTRIBUTION OF PULMONARY BLOOD FLOW (ZONES OF LUNG)

Because of gravitational forces, pulmonary blood flow is not evenly distributed in


the lungs of an upright person. Blood flow is highest at the base of the lung
(Zone 3), lowest at the apex of the lung (Zone 1), and intermediate in the middle
(Zone 2). This is because gravitational forces increase pulmonary arterial pressure
more at the base than at the apex.

Figure 3.

A. Zone 1 (apex), lowest blood flow. Because of gravitational effects, arterial


pressure is lowest in this zone . Because alveolar pressure (PA) is
approximately equal to atmospheric pressure, it turns out that arterial pressure
(Pa) is very close to PA. If Pa is lower than PA, the pulmonary capillaries will
be compressed by the higher alveolar pressure, reducing or even eliminating
blood flow in that region. Normally arterial pressure is just high enough to
prevent this capillary closure.

In hemorrhage, where arterial pressure is reduced, Pa in Zone 1 is well below


PA, and the capillaries are “smushed.” In this condition, Zone 1 will be

868
Pulmonary Circulation - Dr. Costanzo

ventilated, but not perfused (“dead space”) and no gas exchange can occur.

B. Zone 2, medium blood flow. Pa is higher in Zone 2 than in Zone 1, and is


higher than PA. However, PA is still higher than venous pressure in this zone
and so blood flow is driven by the difference between arterial and alveolar
pressures, rather than the more familiar difference between arterial and venous
pressures. Compression of capillaries is not normally a problem in Zone 2
because of the higher Pa.

C. Zone 3 (base), highest blood flow. In this zone, both arterial and venous
pressures are higher than alveolar pressure and blood flow is driven in the
traditional way by the difference between arterial and venous pressure. Zone 3
has the greatest number of open capillaries, and the highest blood flow.

IV. REGULATION OF PULMONARY BLOOD FLOW

A. Passive factors. Several passive forces can change pulmonary blood flow.
We have already discussed gravitational effects on Pa in the upright lung,
which increases blood flow to the base and reduces blood flow to the apex.
Lung volume also affects pulmonary vascular resistance (PVR), whereby
high lung volumes “pull” the blood vessels open, decreasing their
resistance and increasing blood flow; low lung volumes are associated
with higher vascular resistance and lower blood flow.

B. “Active” factors

1. Hypoxic vasoconstriction is one concept in pulmonary


physiology that you will (hopefully) remember forever! It is
memorable, in part, because it is so different from what occurs in
other tissues (e.g., coronary circulation) where hypoxia causes
vasodilation to provide more blood flow and more O2. Hypoxic
vasoconstriction describes the effect of alveolar hypoxia
(decreased PAO2) to increase the resistance of nearby arterioles,
thus reducing blood flow in that region. Hypoxic vasoconstriction
is a protective mechanism in the lungs. It diverts blood flow away
from unventilated (hypoxic) regions where blood flow would be
wasted because gas exchange cannot occur, and directs it toward
regions that are ventilated and where gas exchange can occur.
When we discuss V/Q ratios, you will further appreciate that
hypoxic vasoconstriction attempts to maintain V/Q matching.

The mechanism of hypoxic vasoconstriction is a direct effect of


low alveolar PO2 on pulmonary arterioles. (The anatomical
relationships permit this.) The likely mediator is inhibition of

869
Pulmonary Circulation - Dr. Costanzo

nitric oxide (NO) synthesis in the endothelial cells. NO dilates


arterioles (via production of cyclic GMP); inhibition of NO
synthesis leads to vasoconstriction. Inhaled NO reverses hypoxic
vasoconstriction.

a. High altitude. Hypoxic vasoconstriction occurs at high


altitude where the barometric pressure is decreased, and the
PO2 of inspired air and alveolar air are, accordingly,
decreased. The low PAO2 causes pulmonary
vasoconstriction, increased pulmonary vascular resistance,
increased pulmonary artery pressure, and can even lead to a
compensatory enlargement of the right ventricle (which has
to pump blood against an increased pulmonary arterial
pressure).
b. Fetal lungs. Of course, the “ultimate” alveolar hypoxia is
the fetal lung, which is not ventilated at all. No ventilation,
no O2 in the alveoli. Hypoxic vasoconstriction increases
fetal pulmonary vascular resistance decreases pulmonary
blood flow. With the first breath, the neonate brings O2 into
the alveoli and interrupts hypoxic vasoconstriction,
decreasing pulmonary vascular resistance and increasing
pulmonary blood flow.

2. Other. In addition to O2 (NO), many other substances can alter


pulmonary vascular resistance and, accordingly, pulmonary blood
flow. Thromboxane A2 is produced in response to lung injuries and
is a potent pulmonary vasoconstrictor. Prostaglandin I2 is a
vasodilator. Endothelins, released by pulmonary endothelial cells,
are potent vasoconstrictors.

V. ALVEOLAR VENTILATION EQUATION

Switching gears! This topic and the next topic do not really belong in a lecture on
pulmonary circulation. Mea culpa!

The alveolar ventilation equation describes the inverse relationship between


alveolar ventilation and alveolar PCO2 (or arterial PCO2).

or, rearranging to solve for PACO2:

870
Pulmonary Circulation - Dr. Costanzo

where

The constant, K, needs explanation. The value for K is 863 mm Hg under


conditions of BTPS and when VA and VCO2 are expressed in the same units (e.g.,
ml/min). BTPS means body temperature (310 K), ambient pressure (760 mm Hg),
and gas saturated with water vapor.

The bold version of the equation is used to predict the alveolar PCO2 (or arterial
PCO2) if the rate of CO2 production and alveolar ventilation are known. The most
important point is that if CO2 production is constant, alveolar and arterial PCO2
are determined by alveolar ventilation. The figure below shows the equation
graphically. The higher the alveolar ventilation, the lower the PCO2; the lower the
alveolar ventilation, the higher the PCO2. The figure also shows what happens if
CO2 production increases from 200 ml/min to 400 ml/min. Alveolar ventilation
would have to double from 5 to 10 L/min in order to keep arterial PCO2 at its
normal value of 40 mm Hg. (Alternatively, if alveolar ventilation did not double,
then arterial PCO2 would increase significantly.)

871
Pulmonary Circulation - Dr. Costanzo

Figure 4.

VI. ALVEOLAR GAS EQUATION

We use the alveolar ventilation equation to predict alveolar PCO2. We use the
alveolar gas equation to predict the alveolar PO2. But why would we want to
know the alveolar PO2? Why not just measure arterial PO2 and say that alveolar
PO2 is the same? Because, they are not always the same! Sure, in normal lungs, O2
equilibrates between alveolar gas and pulmonary capillary blood, and arterial PO2
is almost exactly equal to alveolar PO2. But, in many lung diseases, the process of
O2 diffusion is abnormal, O2 does not equilibrate, and arterial PO2 is less than
alveolar PO2. So....that’s why we want to know the value of alveolar PO2....any
difference between alveolar and arterial PO2 indicates a gas exchange problem in
the lungs. (See discussion of A-a gradient in next lectures.)

where

872
Pulmonary Circulation - Dr. Costanzo

If alveolar ventilation is halved, PAO2 decreases (less O2 is brought into the


alveoli). The alveolar gas equation predicts the change in PAO2 that occurs for a
given change in PACO2. Since the respiratory quotient is normally 0.8 (equal to
respiratory exchange ratio in the steady state), when alveolar ventilation is halved,
the PAO2 will fall slightly more than the PACO2 will rise.

The figure below shows the relationship between PO2 and PCO2 calculated by the
alveolar gas equation (the so-called O2 - CO2 diagram). One “anchor” point on
the diagram is inspired air which has a high PO2 of 150 mm Hg but no CO2.
Normal alveolar gas (or equilibrated arterial blood) has a PO2 of 100 mm Hg and a
PCO2 of 40 mm Hg. These changes reflect the respiratory exchange ratio of 0.8.
(50 mm Hg of O2 were replaced by 40 mm Hg of CO2.) Mixed venous blood has
a PO2 of 40 mm Hg (O2 was lost to the tissues) and a PCO2 of 46 mm Hg. Notice
that the variations in PO2 are much greater than the variations in PCO2.

Figure 5.

873
Pulmonary Circulation - Dr. Costanzo

VII. PRACTICE QUESTION

1. If alveolar ventilation increases two-fold and CO2 production remains


constant, arterial PCO2 changes to how many times its original value?

A. 1/4
B. ½
C. No change
D. 2
E. 4

EXPLANATIONS

1. Answer = B. Use the alveolar ventilation equation, which


describes the inverse relationship between alveolar PCO2 and
alveolar ventilation . You will be given this equation on the exam.
Use the equation to calculate the change in PACO2; assume the
same change in PaCO2 because CO2 always equilibrates in the
lungs. PACO2 = VCO2 x K/ VA. Alveolar ventilation is in the
denominator; if it doubles, then PACO2 (and PaCO2) will be reduced
to one-half.

874
Acid-Base 1, 2, and 3 - Dr. Costanzo

Acid-Base 1, 2, and 3
Linda Costanzo, Ph.D.

OBJECTIVES:

After studying this lecture, the student should understand:

1. The relationship between hydrogen ion concentration and pH.


2. Production of acid in the body: volatile and fixed.
3. The derivation of the Henderson-Hasselbalch equation.
4. Functions of extracellular and intracellular buffers.
5. Graphical presentations of the Henderson-Hasselbalch equation.
6. How the bicarbonate buffer works when fixed acid is added to the body.
7. The relationship between hydrogen and calcium ions in binding to plasma
albumin, and the effect of acid-base changes on ionized calcium concentration.
8. How hemoglobin operates as a buffer for hydrogen ions.
9. How hydrogen ions enter and leave cells.
10. The principles of the four simple acid-base disorders with respect to: effects on
blood pH, PCO2, and bicarbonate concentration, buffering, and expected
compensatory responses.
11. How compensations for acute and chronic respiratory disorders differ.

Optional Reading:
Physiology, Saunders, Costanzo; W.B. Saunders, 2006; Chapter 7.
Physiology, Berne and Levy; Mosby, 2004; Chapter 38

I. INTRODUCTION

In normal human beings, the [H+] of the body fluids is about 40 neq/liter (40x 10-9
Eq/liter), corresponding to a pH of 7.4. Recall that:

pH = -log10 [H+]

Use of pH, instead of concentration has the advantage of allowing us to deal with
manageable numbers from 0-14, representing a range of [H+] from 1 - 10-14
Eq/liter. The figure shows the relationship between pH and [H+] over the
physiological range.

875
Acid-Base 1, 2, and 3 - Dr. Costanzo

Figure 1.

Because the pH scale is logarithimic, equal changes in pH reflect unequal changes in


[H+]. For example, a change in pH from 7.0 to 7.1 represents a change in [H+] from 100
to 79 nEq/liter; however, a pH change from 7.3 to 7.4 represents a smaller change in
[H+].

The pH of the blood is normally maintained within a very narrow range, 7.37 - 7.42.
This tight control of pH is essential for virtually all normal cellular function. The extreme
range of blood pH which is compatible with life is 6.8 - 8.0.

II. ACID PRODUCTION IN THE BODY

The body must maintain a normally alkaline pH in spite of the daily production of
large amounts of two kinds of acid.

A. Volatile acid. Volatile acid is CO2. Between 13,000 and 20,000 mmoles
of CO2 is produced per day from oxidative metabolism. This CO2 yields
H+ ions as a result of one or both of the following reactions:

CO2 + H2O W H2CO3 W H+ + HCO-3

C.A.

or

CO2 + OH- W HCO-3


C.A.
8
HOH 6 H+

C.A. = Carbonic anhydrase

876
Acid-Base 1, 2, and 3 - Dr. Costanzo

The final products of both reactions are H+ and HCO-3. There is still debate
over which form is correct. For the sake of simplicity, we shall use only the
first form as this is given in most textbooks. We will consider, also for
simplicity, that carbonic anhydrase catalyzes the reaction of CO2 and H2O to
form H2CO3 and the reverse reaction. Valtin's book uses the convention
shown in Equation lb.

The CO2 produced from oxidative metabolism is of course carried to the


lungs via the blood. About 90% of this metabolic CO2 transiently generates
H+ (Equation la), threatening H+ balance. How this metabolically produced
H+ is buffered in transit to the lungs is important and will be covered in
these lectures.

B. Non-volatile or fixed acid usually comes from amino acid and


phospholipid metabolism. When meat constitutes a large part of the diet,
there is daily production of 40-60 mmoles of inorganic and organic acid.

1. Sulfuric acid is produced from the catabolism of proteins having the


sulfur-containing amino acids cysteine, cystine and methionine. The
example of methionine is shown in then next equation.

2 C5H11NO2S + 15 O2 W 4H+ + 2 SO4-2


(methionine)
+ CO2(NH2)2 + 7 H2O + 9 CO2
(urea)

2. Phosphoric acid is formed from the catabolism of phospholipids.


3. During normal exercise or in shock, the production of fixed (non-
volatile) acids may increase due to excess lactic acid production.
4. Certain pathologic conditions are associated with increased fixed
acid production. For example, aceto-acetic acid and ß -
hydroxybutyric acid are produced in excess in uncontrolled
diabetes mellitus; lack of insulin increases lypolysis in fat cells. The
free fatty acids enter the liver where they are converted to ketoacids,
weak acids which exist mainly as H+ and the ketoanion at pH 7.4.

III. EQUILIBRIUM CONCEPT AND LAW OF MASS ACTION

The behavior of acids and bases in biological fluids conforms to the kinetics of
reversible reactions, i.e.

K1
HA W H+ + A-

K2

877
Acid-Base 1, 2, and 3 - Dr. Costanzo

The rate of dissociation of HA into H+ and A- is characterized by a rate constant,


K1; the rate of the reverse reaction, association of H+ and A- to form HA, is
characterized by a second rate constant, K2. When the rates of these two opposing
reactions are exactly equal, then a state of chemical equilibrium exists and there is
no further net change in the number of either species, HA or A-. The Law of
Mass Action states that at equilibrium:

K1[HA] = K2 [H+] [A-]

Rearranging:

K1 = [H+] [A-]

K2 [HA]

The ratio of constants can be combined into one constant K', so now:

K' = [H+] [A-]

[HA]

K' is the equilibrium constant of this reaction. The "prime" sign signifies that
concentrations rather than activities are being used.

This equation can be rearranged to solve for [H+]:

[H+] = K' [HA]


[A-]

To express [H+] as pH, we take the negative log10 of both sides:

-log[H+] = -log K' - log [HA]


[A-]

Remember:

-log [H+] = pH

-log K' = pK'

-log [HA] = log [A-]


[A-] [HA]

878
Acid-Base 1, 2, and 3 - Dr. Costanzo

Making these substitutions:

pH = pK' + log [A-]


[HA]

This is the familiar Henderson - Hasselbalch equation. We will use the Henderson-
Hasselbalch equation repeatedly in acid-base physiology to calculate the pH of solutions
of weak acids and their conjugate bases.

Note that strong acids have very high equilibrium constants, low values for pK' and
dissociate almost completely; most of the acid is in the form of A- + H+. Conversely
weak acids have low equilibrium constants, high values for pK', and most of the
acid is not dissociated, but in the form of HA.

IV. BUFFERING

A. Definition

When weak acids and bases are dissolved in aqueous solutions, only
partial dissociation occurs. The resulting solution will contain both the
acid and base forms of the parent molecule. Such solutions can resist a
change in pH following addition of acid or base. This property is called
buffering. The acid-base pairs are buffers. When a strong acid is added to
a buffered solution, some of the added H+ combine with the base (A-) form
of the buffer to yield more of the acid form (HA), rather than remaining
free in solution. Thus, the increase in free [H+] is much less than following
the addition of an equivalent amount of acid to an unbuffered solution.

B. Buffering in vivo.

As already noted, normal acid-base balance in human beings requires


defense of an alkaline blood pH against the daily production of volatile
acid from CO2 and fixed acid from protein and phospholipid catabolism.
Buffering occurs in two locations:

1. Extracellular. Bicarbonate is the most important extracellular


buffer. Inorganic phosphate and plasma proteins also contribute a
small buffering capacity.
2. Intracellular. Buffering occurs by proteins (such as hemoglobin in
the erythrocyte), organic and inorganic phosphates. In bone, CO3-2
represents a large buffer store, whereby H+ are exchanged for Na+
and Ca+2 in bone matrix. Finally, most cells can exchange K+ for
H+ to help buffer extracellular H+. Each of these forms of buffers
will be discussed in detail.

879
Acid-Base 1, 2, and 3 - Dr. Costanzo

The ability of a particular buffer to protect against pH changes


depends on its concentration and its pK' in relation to the body
fluids pH. The greater the buffer concentration and the closer the
pK' to the pH of the body fluids, the more effective the buffering.

Figure 2.

The figure illustrates the buffering property of plasma. In the experiment, 156 ml of 1N
HCl was infused intravenously into a dog. The dog's plasma pH fell from 7.44 to 7.14, a
severe, but not fatal, acidosis. When the same amount of HCl was slowly added to 11.4
liters of distilled water (same volume as dog's total body water), the pH of the water fell
precipitously after just a few mEq of H+ were added. The final pH was 1.84, one that
would have been clearly fatal in the dog. Obviously, the dog's plasma had buffering
capabilities that water did not.

This example illustrates one type of buffering that occurs in humans, that of added fixed
acid. The fast, immediate buffering of added fixed acid was via the H2CO3/ HCO-3 buffer
pair in extracellular fluid (see next lecture).

V. EXTRACELLULAR BUFFERS

A. Buffering by H2CO3/HCO-3 (extracellular fluid)

The first line of defense in buffering fixed acid is fast, physicochemical


buffering in extracellular fluid. The H2CO3 - HCO-3 is by far the most
important extracellular buffer because (a) its concentration is high, (b) its
pK' is fairly close to the pH of extracellular fluid and (c) the acid form
(H2CO3) is in equilibrium with CO2 which can be rapidly excreted by the
lungs.

880
Acid-Base 1, 2, and 3 - Dr. Costanzo

1. Applying the Henderson-Hasselbalch Equation to the HCO-3/CO2


system

If HCl is buffered by the bicarbonate buffer system then:

In this reaction, the amount of a strong acid (HCl) is reduced, the


amount of a weak acid (H2CO3) is increased; the buffer salt (HCO-
3) is depleted. The decrease in pH is not totally prevented, but
minimized.

pH = pK' + log [HCO-3]


[H2CO3]

H2CO3 is in equilibrium with CO2. Remember:

CO2 + H2O W H2CO3 W H+ + HCO-3


C.A.

In the presence of carbonic anhydrase, most H2CO3 is in the form


of dissolved CO2 (400 dissolved CO2 for every 1 H2CO3). Thus a
more realistic form of equation 2 is:

pH = pK' + log [HCO-3]


[dissolved CO2 + H2CO3]

The concentration of dissolved CO2 in plasma is proportional to the


partial pressure of CO2 (PCO2). The proportionality constant which
converts PCO2 in mm Hg to CO2 concentration in mM is 0.03
(0.03 x mmHg = mM). H2CO3 concentration is negligible and can
be ignored. The final equation most useful to us is:

pH = 6.1 + log [HCO-3]

0.03 x PCO2

881
Acid-Base 1, 2, and 3 - Dr. Costanzo

Normal values for arterial plasma:

[HCO-3] = 24 mM

PCO2 = 40 mm Hg

pK' = 6.1 at 37 C

Substituting:

pH = 6.1 + log 24mM


0.03 x 40 mM Hg

pH = 6.1 + log 24mM


1.2mM

pH = 6.1 + log 20

pH = 7.40

As you have undoubtedly noted, the Henderson-Hasselbalch equation correctly predicts


the normal pH of arterial blood when the normal values for the HCO-3 buffer pair are
substituted. Likewise any of the three variables can be calculated when values for the
other two are known.

2. Graphs of Henderson-Hasselbalch Equation.

Graphical presentations of the relationship between PCO2,


bicarbonate concentration and pH (or [H+]) are used in some texts.
One such graphical representation is shown in the figure below.

882
Acid-Base 1, 2, and 3 - Dr. Costanzo

Figure 3.

The lines radiating from the origin show the relationship between arterial PCO2 and
[HCO-3] at various values of pH. These are isohydric lines. The normal range of values
for arterial blood are given by the ellipse in the center of the figure. The terms acidemia
and alkalemia denote decreases and increases in plasma [H+]. Notice that normal values
of pH may obtain from certain abnormal combinations of PCO2 and [HCO-3].

883
Acid-Base 1, 2, and 3 - Dr. Costanzo

3. Adding fixed acid; buffering by HCO-3.

If 12 mmoles of HCl were added to each liter of extracellular fluid,


then buffering by HCO-3 would occur:

Let us assume for a moment that this newly generated CO2 cannot
be expired by the lungs. The pH of arterial blood would drop to a
fatal level, 6.06:

pH = 6.1 + log 12 mM
1.2mM + 12mM

pH = 6.1 + log 12mM


13.2 mM

pH = 6.06

However, the ventilatory response to the excess CO2 produced


from buffering occurs within seconds or minutes after the
administration of HCl. If all the excess CO2 is excreted by the
lungs, then the pH will fall, but to a level compatible with life:

pH = 6.1 + log 12 mM
1.2 mM

pH = 6.1 + log 10

pH = 7.1

884
Acid-Base 1, 2, and 3 - Dr. Costanzo

Respiratory compensation usually goes even further as alveolar


ventilation increases in response to the CO2 stimulus
(compensatory hyperventilation). As a result, arterial PCO2 might
fall to 23mm Hg (instead of just back to 40mm Hg):

pH = 6.1 + log 12 mM
0.03 x 23mm Hg

pH = 6.1 + log 12 mM
0.69 mM

pH = 7.34

Consequently, the pH has been restored almost to normal as a


result of buffering by extracellular HCO-3, excretion of the
resulting CO2, and additional compensatory hyperventilation by
the lungs. The pH is not perfect, however, because the [HCO-3] in
arterial plasma is only ½ of its normal value. The kidneys are
needed to replenish the depleted HCO-3 stores by excretion of H+
and reabsorption of newly synthesized HCO-3. This is much slower
and requires several days.

4. Real life buffering of fixed acid by extracellular HCO-3

The above example is an extreme one, occurring only under


experimental conditions or in disease. It is valuable, though,
because it illustrates the buffering and compensation that occurs
normally in response to the daily loads of fixed acid added from
metabolism (e.g. H2SO4 from sulfur-containing amino acids).
Realistically, each meal might result in the release of 16 mmoles
of H2SO4. The extracellular fluid volume of a 70 kg man is 14
liters. Thus the [H+] of extracellular fluid would increase by 32
mmoles/ 14 liters or about 2 mmole/liter. Sulfuric acid is the major
fixed acid so quantitatively the reaction would be:

2H+ + SO4 + 24Na+ + 24HCO-3 W 2Na+ + SO4 + 22Na+ + 22HCO-3 + 2H2CO3


^
2CO2 + 2H20

After buffering, but before hyperventilation the pH would be:

pH = 6.1 + log 22 mM
1.2 mM + 2 mM

pH = 6.9

885
Acid-Base 1, 2, and 3 - Dr. Costanzo

After the excess CO2 is eliminated by the lungs:

pH = 6.1 + log 22 mM
1.2 mM
pH = 7.36

This might cause a small, imperceptible increase in alveolar ventilation. However, in the
steady state the kidneys are continuously excreting H+ and reabsorbing filtered and newly
synthesized HCO3- so that the plasma HCO3- stays at 24 mM. Hence, the arterial pH
remains at 7.40.

B. Buffering by HPO4-2/H2PO4- (extracellular fluid and urine)

The phosphate buffer pair plays a small role in extracellular buffering


capacity. Theoretically, phosphate should be an excellent buffer in the
physiological pH range, as its pK' is 6.8. Compare buffering by CO2/HCO-
-2 4-
3 to buffering by HPO4 /H2PO shown in the figure below.

Figure 4.

The change in pH per mEq of H+ added or subtracted is smallest in the


linear portion of each titration curve, i.e. this is the region of most
effective buffering. The linear portion of the CO2/HCO-3 curve extends
from pH 5.1 to 7.1. The linear portion of the phosphate curve extends from
pH 5.8 to 7.8, coinciding with the range compatible with life. In spite of
this, phosphate only plays a small buffering role in extracellular fluid,
because its concentration is low compared to HCO-3. Also bicarbonate has
the special property of being in equilibrium with a volatile weak acid,

886
Acid-Base 1, 2, and 3 - Dr. Costanzo

CO2, which can be rapidly excreted or retained by the lungs. Phosphate is


a more important buffer in tubular fluid as its concentration can get quite
high in the distal tubule.

C. BUFFERING BY NH3/NH4+ (urine)

NH4+ is the very weak acid of the base NH3:

NH4+ W NH3 + H+

The pK' of this reaction is about 9.2 so the NH4+/NH3 pair does not
function as a buffer for extracellular fluid in the conventional sense; even
at very alkaline pH, most will be in the NH4+ form. However, the
NH4+/NH3 buffer pair plays a special role in the renal excretion of H+ as
you will learn shortly.

VI. INTRACELLULAR BUFFERS

A. Buffering by organic phosphate

These are a heterogeneous group, comprised of 2,3-DPG, glucose-1-


phosphate, AMP, ADP, and ATP. The phosphate moiety functions much
as the HPO4-2/H2PO4-2 pair. Their pK's are in the very effective range of
6.0 to 7.5 The organophosphates contribute to the buffer capacity of the
intracellular compartment as their concentration is high.

B. Buffering by proteins
Proteins can serve as buffers because they contain a large number of acidic
or basic groups, such as -COOH, -NH2 and -NH3. Table 1 gives the pK'
values for various dissociable groups found in proteins.

Table 1. pK' values for dissociable groups on proteins


Group (amino acid) pK'

"-carboxyl 3.7

$-carboxyl (aspartic acid) 4.0

'-carboxyl (glutamic acid) 4.0

*imidazole (histidine) 6.4-7.0

"-amino 7.4-7.9

sulfhydryl (cysteine) 9.0

'-amino (lysine) 9.8-10.6

887
Acid-Base 1, 2, and 3 - Dr. Costanzo

Of course, for a given protein, buffer capacity reflects the composite effect
of all its dissociable groups. In mammals, most of the buffer capacity of
proteins can be attributed to the imidazole group of histidine. The pK'
of the imidazole group is 6.0, but is listed as 6.4 - 7.0 in various proteins
because of varying electrostatic forces.

Figure 5.

N-terminal a-amino groups feature a pK' in the optimal range, also. Other
dissociable groups in proteins play a smaller role because their pK's are
outside the useful physiological range. Largely because of the multiplicity
of pK' values on a given protein, the titration curve may approach a
straight line, rather than the typical sigmoid shape of the bicarbonate or
phosphate curves shown in the previous lecture.

1. Plasma proteins (not intracellular, of course). Albumin accounts for


much of the buffer capacity exhibited by plasma proteins because it
contains 16 histidine residues/molecule. Globulins have considerably
less buffer capacity.

888
Acid-Base 1, 2, and 3 - Dr. Costanzo

Figure 6.

H+ and Ca2+ compete for negatively charged binding sites on plasma


proteins. In acidosis, there is increased H+ in blood, H+ binding to
negative sites on albumin increases and displaces Ca2+; thus, the
ionized or free Ca2+ increases. Conversely, in alkalosis, there is
decreased H+ in blood, H+ binding to negative sites on albumin
decreases, and the ionized or free Ca2+ decreases, often causing
symptoms of hypocalcemia.

2. Hemoglobin. Hemoglobin is by far the most important non-


bicarbonate buffer of whole blood because of its high concentration
inside the erythrocyte and its high buffer capacity. Most of the
buffering capacity of hemoglobin in the physiological pH range
results from its 36 histidine residues (9 on each of its 4 polypeptide
chains). There are also α-amino groups which are sufficiently
exposed to buffer H+. Hemoglobin also has some special attributes
as a buffer. The pK' for deoxygenated hemoglobin is about 7.9.
The pK' of the oxygenated form is lower than of the
deoxygenated form.

Hbn- HbO2n-
1. pK' 7.9 6.7
2. K' lower higher
3. acidity weaker stronger
4. electrostatic bonds more less

Thus, HbO2n- is a stronger acid and gives up H+ more readily than


does Hbn-. (This property also forms the basis for the Bohr effect.)

889
Acid-Base 1, 2, and 3 - Dr. Costanzo

The mechanism is as follows. Upon oxygenation, the Hb molecule


undergoes a marked conformational change. The groups
responsible for the change in acidity (or pK') are C-terminal
histidines in the chain. These histidine residues participate in
electrostatic bonds when the subunits are deoxygenated. Binding
of O2 causes a conformational change in Hb, such that the
distances are now too great for electrostatic bonding to occur. The
electrostatic binding doesn't occur and these H+ are more readily
dissociated; thus the stronger acid and lower pK'.

The implications of this change in pK' on acid-base balance are


illustrated in the figure below.

Figure 7.

First notice that oxygenated Hb is an excellent buffer for added H+,


as expected (lower line). Deoxygenated Hb is an even better buffer
because for every H+ added a smaller reduction in pH occurs
(upper line). Thus as Hb becomes deoxygenated at the
arteriolar end of the capillary it becomes a more effective
buffer. Conveniently, this is the precise moment when CO2 (and
thus H+) are being added from the tissue cells to the capillary
blood. The reduction of HbO2n- to Hbn- would have caused a
tremendous increase in pH were it not for the H+ being added to
the blood at this time. The increase in pK' of Hb is such that 1.3
mmoles of CO2 can be added to each liter of capillary blood

890
Acid-Base 1, 2, and 3 - Dr. Costanzo

without a change in pH. 95% of the CO2 added to each liter of


blood, about 1.6 mmoles, is rapidly converted to H+. Since 1.3
mmoles can be buffered by Hbn- without a change in pH, the drop
in venous blood pH due to addition of CO2 is minimized. H+
buffering by hemoglobin will be discussed again in the context of
CO2 transport in venous blood from the tissue to the lungs.

C. Other intracellular proteins. Various cell proteins contribute to


intracellular H+ buffering, largely due to the presence of the imidazole
group of histidine.

D. How H+ enters cells to be buffered

How do H+ enter and leave the ICF in order to take advantage of the vast
supply of intracellular buffers? In respiratory disturbances, where there is
an excess or deficit of CO2, CO2 readily crosses cell membranes; thus, in
respiratory acidosis CO2 enters the cells and in respiratory alkalosis CO2
leaves the cells. In metabolic disturbances, where there is an excess or
deficit of fixed H+, H+ can cross cell membranes in exchange for another
cation, K+, or it can cross with an organic anion such as lactate, β-OH-
butyrate etc. Either way, electroneutrality must be preserved. If H+
exchanges for K+ (H+-K+ shift), then a disturbance of K+ metabolism
occurs — acidemia producing hyperkalemia and alkalemia producing
hypokalemia. More commonly, there is an accompanying organic anion to
move with H+ (e.g., lactic acidosis) and an H+-K+ shift is not required.

Figure 8.

891
Acid-Base 1, 2, and 3 - Dr. Costanzo

VII. CO2 TRANSPORT IN BLOOD

As stated before, some 13,000 to 20,000 mmoles of CO2 are produced daily in an
adult as a result of metabolism. This CO2 can generate H+ and potentially disturb
H+ balance. Elimination of CO2 by the lungs prevents acidosis, but the transport
of CO2 in the venous blood to the lungs presents a major buffering problem. We
know that buffering of CO2-generated H+ in blood is very effective because the
venous blood pH is seldom less than 7.37, only 0.03 pH units more acid than
arterial blood. The buffering events in its transport to the lungs are as follows:

Figure 9. Transport of carbon dioxide (CO2) in the blood. CO2 and H2O are
converted to H- and HCO3 inside red blood cells. H+ is buffered by hemoglobin (Hb
- H) inside the red blood cells. HCO3 exchanges for Cl and is transported in plasma.
The circled numbers correspond to the lettered steps discussed in the text below.

A. CO2 diffuses from tissues cells into the capillary blood because pCO2 is
much higher in the metabolizing cells than in the blood.

B. The red blood cell membranes are highly permeable to CO2 and hence
CO2 will also enter the erythrocytes. Both in the plasma and in the red
cells, CO2 reacts with H2O to form H2CO3 and thereafter H+ and HCO-3.
In plasma the reaction is slow due to the lack of carbonic anhydrase; in red
cells it is fast because the enzyme is present. What little H+ is produced in
plasma is buffered by plasma proteins and phosphate. In the red cell, the
H+ produced is buffered by hemoglobin which has a large buffering
capacity. You will recall that hemoglobin is an even more effective buffer
in its deoxygenated form.

892
Acid-Base 1, 2, and 3 - Dr. Costanzo

C. The HCO-3 formed from the reaction of CO2 and H2O in the red cells
diffuses back into the plasma down a concentration gradient and Cl-
moves into the red cells to maintain electrical neutrality. This is Cl-HCO-3
exchange. Most of the CO2 generated in tissues is carried to the lungs
as HCO-3 in plasma.

D. Some of the CO2 which enters the red cells is not converted to H+ and
HCO-3. Instead, it combines with hemoglobin to form
carbaminohemoglobin and is carried to the lungs in that form. A small
amount of CO2 is carried to the lungs dissolved in plasma or in the red
blood cell water.

E. In the lungs, the entire process operates in reverse as PCO2 is lower in the
alveoli than in the capillary blood perfusing the lung tissue. Thus all
reaction are driven toward the reformation of CO2 so that it can be
eliminated.

VIII. SIMPLE ACID-BASE DISORDERS

There are four "simple" acid-base disturbances (where "simple" means one acid-
base disorder at a time): metabolic acidosis, metabolic alkalosis, respiratory
acidosis, and respiratory alkalosis. The metabolic disturbances involve gain or
loss of fixed H+. The respiratory disturbances involve gain or loss of CO2. In this
section, we will compare the relative contributions of ECF and ICF buffers,
respiratory compensation (if it occurs), and renal compensation for each of the
four simple disorders.

A. Metabolic acidosis

Metabolic acidosis is caused by excess fixed acid, either due to ingestion


of acid, overproduction of acid (e.g., diabetic ketoacidosis), or loss of
HCO3- (diarrhea). The excess fixed H+ is buffered by ECF HCO3- and, as a
result, the blood HCO3- concentration decreases. This causes the arterial
pH to decrease (H-H equation) and the acidemia stimulates central
chemoreceptors. There is ensuing hyperventilation, which is the
respiratory compensation for metabolic acidosis. This hyperventilation
results in decreased PCO2, which tends to normalize the pH (i.e., is a
"compensation"). Buffering also takes place in ICF, utilizing protein and
organic phosphate buffers and there is a decrease in intracellular pH.
Ultimately, the kidneys excrete the excess fixed H+ and synthesize new
HCO3- to replace the HCO3- that was consumed in buffering the fixed H+.

893
Acid-Base 1, 2, and 3 - Dr. Costanzo

B. Metabolic alkalosis

Metabolic alkalosis is caused by loss of fixed H+ (e.g., vomiting). Loss of


H+ causes the blood HCO3- concentration to increase and the arterial pH
to increase (H-H equation). The alkalemia inhibits central chemoreceptors
and there is hypoventilation, which is the respiratory compensation for
metabolic alkalosis. The hypoventilation causes increased PCO2, which
tends to normalize the pH. Buffering also occurs in ICF and intracellular
pH increases. Ultimately, the kidneys excrete the excess HCO3- and there
is restoration of acid-base balance.

C. Respiratory acidosis

Respiratory acidosis is caused by hypoventilation leading to retention of


CO2, increased PCO2, and decreased pH (H-H equation). There is no
respiratory compensation for respiratory acidosis (that's impossible); thus,
in the acute phase, pH may be markedly decreased. Buffering takes place
almost exclusively in ICF. Within 2-3 days, the kidneys increase HCO3-
reabsorption, which increases the blood HCO3- concentration. This
increase in blood HCO3- is the renal compensation for respiratory acidosis
and tends to normalize the pH. For the same value of PCO2, chronic
respiratory acidosis will have a higher pH (more normal) than acute
respiratory acidosis.

D. Respiratory alkalosis

Respiratory alkalosis is caused by hyperventilation leading to loss of


excess CO2, decreased PCO2, and increased pH (H-H equation). There is
no respiratory compensation for respiratory alkalosis, and in the acute
phase pH may be markedly increased. Buffering is exclusively in the ICF.
Within 2-3 days, the kidneys decrease HCO3- reabsorption (the renal
compensation), which decreases the HCO3- concentration and tends to
normalize pH. For the same value of PCO2, chronic respiratory alkalosis
will have a lower pH (more normal) than acute respiratory alkalosis.

894
Acid-Base 1, 2, and 3 - Dr. Costanzo

E. Summary of contributions of extracellular and intracellular buffering


in the acid-base disorders.

Extracellular Intracellular

Metabolic acidosis 45% 55%

Metabolic alkalosis 70% 30%

Respiratory acidosis 3% 97%

Respiratory alkalosis 1% 99%

895
Ventilation/Perfusion (V/Q) 1 & 2 - Dr. Costanzo

Ventilation/Perfusion (V/Q) 1 and 2


Linda Costanzo Ph.D.

OBJECTIVES:

After studying this lecture, the student should understand:

1. Regional variations in ventilation, perfusion, and ventilation/perfusion in the


upright lung.
2. The effect of regional variations in ventilation/perfusion on PO2 and PCO2.
3. The patterns of ventilation/perfusion defects, including dead space and shunt, and
the effects on alveolar and arterial PO2 and PCO2.
4. How to measure physiological dead space.
5. The physiological shunt, right-to-left shunts, and left-to-right shunts.
6. How to calculate the A-a gradient and how to use the calculated A-a gradient in
comparing causes of hypoxemia.
7. How to measure the size of a right-to-left shunt with the shunt equation.
8. Causes and mechanisms of hypoxemia.
9. Causes and mechanisms of hypoxia.

I. OVERVIEW OF V/Q MATCHING

Ventilation/perfusion (V/Q) matching is essential for normal gas exchange in the


lungs. For normal gas exchange, alveoli must be in close proximity to pulmonary
capillaries -- ventilation must be close to blood flow. The V/Q ratio expresses the
matching of ventilation (V in L/min) to perfusion or blood flow (Q in L/min). It is
useless if ventilated alveoli are not near perfused capillaries, or if perfused
capillaries are not near ventilated alveoli.

For the entire lung, the average normal value of V/Q is 0.8. This means for the
whole lung, ventilation (L/min) is 80% of perfusion (L/min). However, V/Q is not
uniformly 0.8 throughout the entire normal lung; some regions have higher V/Q
and some regions have lower V/Q. An average V/Q of 0.8 results in an arterial
PO2 of 100 mm Hg and arterial PCO2 of 40 mm Hg, the normal values.

II. REGIONAL VARIATIONS IN V/Q

In the upright lung, there are regional variations in both ventilation and blood
flow. We have already discussed the gravitational effects that cause blood flow to
be highest at the base and lowest at the apex. There are also regional variations in

896
Ventilation/Perfusion (V/Q) 1 & 2 - Dr. Costanzo

ventilation that occur in the same direction as those for blood flow; thus,
ventilation is highest at the base and lowest at the apex. However, and
importantly, the variations in blood flow are greater than the variations for
ventilation, such that the apex has a higher V/Q and base has a lower V/Q.

Zone 1 has the lowest blood flow, the lowest ventilation, and highest V/Q. Zone
3 has the highest blood flow, the highest ventilation, and the lowest V/Q. Zone 2
is in between.

Figure 1.

897
Ventilation/Perfusion (V/Q) 1 & 2 - Dr. Costanzo

Figure 2.

These regional variations in V/Q ratio have implications for gas exchange that
produce regional variations in PO2 and PCO2.

1. The higher the V/Q, the higher the ventilation relative to perfusion, the
higher the PaO2 and the lower the PaCO2 (more gas exchange).
2. The lower the V/Q, the lower the ventilation relative to perfusion, the
lower the PaO2 and the higher the PaCO2 (less gas exchange).

These differences in V/Q are superimposed on the O2 - CO2 diagram below. Now,
instead of a single value for alveolar (arterial) PO2 and PCO2, the range is shown
for different regions of the lung. Zone 1, with the lowest blood flow, lowest
ventilation, and highest V/Q has the highest PO2 and lowest PCO2. Zone 3, with the
highest blood flow, highest ventilation, and lowest V/Q has the lowest PO2 and
highest PCO2.

898
Ventilation/Perfusion (V/Q) 1 & 2 - Dr. Costanzo

Figure 3.

899
Ventilation/Perfusion (V/Q) 1 & 2 - Dr. Costanzo

III. V/Q DEFECTS

V/Q matching means that ventilation and perfusion are “matched up”, that
ventilated alveoli are close to perfused capillaries, which provides for ideal gas
exchange. A mismatch of ventilation and perfusion (called V/Q mismatch or V/Q
defect) causes a defect in gas exchange. The defect can range from ventilated
alveoli that are not perfused (called “dead space”) to perfused capillaries that are
not ventilated (called “shunt”), and every possibility in between (high V/low Q =
high V/Q; low V/high Q = low V/Q). Any V/Q mismatch implies that inadequate
gas exchange will occur.

Figure 4.

900
Ventilation/Perfusion (V/Q) 1 & 2 - Dr. Costanzo

Figure 5.

A. Dead Space

Dead space is the volume of the airways and the lungs that does not
participate in gas exchange. The anatomic dead space is the volume of the
conducting airways; they cannot possibly participate in gas exchange because
they have no alveoli. The physiologic dead space, includes the anatomic dead
space plus functional dead space in alveoli (alveoli that are ventilated but not
perfused). In normal persons, the physiologic dead space is nearly equal to the
anatomic dead space. However, in lung diseases in which a V/Q defect
develops, the physiologic dead space increases.

So...one extreme of V/Q mismatch is called dead space. It refers to alveoli that
are ventilated, but not perfused. No O2 or CO2 can be exchanged with air
entering these alveoli because there is no blood flow to pick up O2 or to
release CO2. In regions of the lung where there is dead space, alveolar PO2
and PCO2 approach their values in inspired air.

901
Ventilation/Perfusion (V/Q) 1 & 2 - Dr. Costanzo

Physiologic dead space is calculated by Bohr’s equation, which assumes


that (1) all of the CO2 in expired air comes from functional alveoli (alveoli
that are perfused); (2) that inspired air has no CO2, and (3) that alveolar and
arterial PCO2 are equal.

VD is physiologic dead space (ml), VT is tidal volume, PaCO2 is the PCO2 of


arterial blood, and PECO2 is the PCO2 of expired air.

1. If there is no dead space, then PECO2 equals PACO2 (same as PaCO2), and VD
comes out to be zero in the calculation (see that?).
2. If dead space is the whole tidal volume then PECO2 is zero and VD equals VT in
the calculation. (That would be really bad, the person would be dead.)

B. Shunts

Shunts occur when a portion of the pulmonary blood flow bypasses the
alveoli; gas exchange cannot occur in shunted blood, i.e., the PO2 and PCO2 of
shunted blood equals their values in mixed venous blood.

1. Physiologic shunt. Normally, a small portion (2%) of the pulmonary


blood flow bypasses the alveoli (a portion of bronchial blood flow
drains into the pulmonary veins and a portion of coronary blood flow
drains directly into the left ventricle via the Thebesian veins). Thus, a
small physiologic shunt is always present and causes PaO2 to be
slightly less than PAO2, a difference we usually ignore.
2. Right-to-left cardiac shunts. Defects in the intraventricular septum
can result in as much as 50% of the cardiac output being routed from
the right ventricle to the left ventricle without going to the lungs for
gas exchange. In cardiac right-to-left shunts, there is always
hypoxemia (decreased arterial PO2) -- shunted blood is not oxygenated
in the lungs and dilutes the non-shunted (normal) blood that is
oxygenated.

More common are left-to-right cardiac shunts, which do not cause


hypoxemia. When blood is shunted from the left heart to the right
heart, there is a decrease in cardiac output of the left heart and an
increase in cardiac output of the right heart, but no “problem” with
oxygenation. A portion of the oxygenated blood from the left heart is
recycled to the lungs, raising PO2 on the right side of the heart.

902
Ventilation/Perfusion (V/Q) 1 & 2 - Dr. Costanzo

3. Intrapulmonary shunts. Blood can also be shunted within the lungs,


such that a portion of the pulmonary blood flow perfuses lung regions
that are not ventilated (regions where V/Q = 0); there can be no gas
exchange in that blood and there is always hypoxemia. For example,
if a large bronchiole is occluded, all of the blood perfusing that region
becomes a shunt. For another example, in adult respiratory distress
syndrome (ARDS), certain cytokines released by the lung cause local
vasoconstriction and re-route blood to regions that are not ventilated.
Don’t forget — a shunt is an extreme of V/Q defect in which V is zero,
Q is some value, and V/Q is zero.

Figure 6.

4. “A - a gradient”

The presence of a shunt can be detected by calculating the so-called


A- a gradient. “A” stands for alveolar PO2 (PAO2) and “a” stands for
arterial PO2 (PaO2). You must know this lingo!! Alveolar PO2, or “A”,
is calculated with the alveolar gas equation. Arterial PO2, or “a”, is
measured in the arterial blood gases.

Normally, the A - a gradient is small (nearly zero). The small normal


A -a gradient reflects the small physiologic shunt that bypasses the
lungs and is not oxygenated.

903
Ventilation/Perfusion (V/Q) 1 & 2 - Dr. Costanzo

The A- a gradient is increased when there is a shunt (either right-to-


left cardiac shunt or intrapulmonary shunt). A larger-than-normal
portion of the pulmonary blood flow is not oxygenated; the PO2 of this
shunted blood remains at the value for mixed venous blood, i.e., 40
mm Hg, and dilutes the overall PO2 of the blood leaving the lungs.

Treatment with 100% O2 tests for a shunt. When a person with a


shunt (and therefore an increased A - a gradient) breathes 100% O2,
their A - a gradient will remain increased. Although the high O2
treatment will raise the PO2 of the non-shunted blood, the PO2 of the
shunted blood remains at the value for mixed venous blood; thus,
overall PaO2 remains lower than PAO2, i.e., increased A-a gradient. The
“quick and dirty” wisdom you will hear is that a shunt is not
“treatable” with 100% O2. This wisdom is superficial. Correctly
speaking, the A - a gradient and O2 delivery are not correctable. The
overall PO2 of arterial blood will be somewhat increased by giving
100% O2, but the extent of increase arterial PO2 depends on the size of
the shunt.

5. The shunt equation

The shunt equation calculates the fraction of total pulmonary blood


flow (QT) that is shunted (QS). It is based upon the principle of
conservation of mass for O2. (Suggestion: use this equation in
combination with the previous picture of a shunt.)

a. Qs is blood flow through the shunt


b. QT is total pulmonary blood flow, or cardiac output
c. O2 content of non-shunted blood is calculated based on
equilibration of that blood with alveolar gas, i.e., PO2 = 100 mm Hg
d. Arterial O2 content is calculated based on the measured arterial PO2
e. Venous O2 content is calculated based on the measured venous PO2

904
Ventilation/Perfusion (V/Q) 1 & 2 - Dr. Costanzo

IV. HYPOXEMIA

Hypoxemia is a decrease in arterial PO2. As we have repeatedly emphasized,


normally O2 equilibrates across the alveolar-pulmonary capillary barrier and
arterial PO2 equals alveolar PO2, which is 100 mm Hg at sea level. Logically, then,
a decrease in arterial PO2 is seen if (1) there is a defect in O2 exchange in the lungs
or (2) the alveolar PO2 is decreased (e.g., decreased barometric pressure). Each of
the following scenarios causes a decrease in arterial PO2.

Causes of Hypoxemia PaO2 A - a gradient


High Altitude Decreased Normal
Hypoventilation Decreased Normal
Diffusion Defect (e.g., fibrosis, pulmonary
Decreased Increased
edema)
V/Q Defect Decreased Increased
Shunt (V/Q = 0) Decreased Increased

High altitude causes hypoxemia because barometric pressure and PAO2 are
decreased; assuming perfect O2 equilibration, PaO2 will have the same (lower)
value as PAO2. A- a gradient is normal (near zero) because PaO2 is equal to PAO2,
both are lower than normal.

V/Q defect always causes hypoxemia. You might wonder why this is “always”
true. Why can’t regions of high V/Q compensate for the regions of low V/Q so
that the final PO2 of blood leaving the lungs is relatively normal. Good idea ☺,
but things don’t work that way. Although high V/Q regions will raise their blood
to a super high value of PO2, the blood flow to those regions is relatively small.
Thus, that blood has a small quantitative effect on the PO2 of the total blood
leaving the lungs. (The low V/Q regions where PO2 is low will have the greatest
effect because they have highest blood flow.)

V. HYPOXIA

O2 delivery to the tissues is determined by blood flow and the O2 content of that
blood. In terms of the whole organism, blood flow can be considered to be cardiac
output. O2 content of the blood is the sum of dissolved O2 and O2-hemoglobin.

905
Ventilation/Perfusion (V/Q) 1 & 2 - Dr. Costanzo

Dissolved O2 is the PaO2 times the solubility of O2. O2-hemoglobin is determined


by the concentration of hemoglobin, the O2-binding capacity of that hemoglobin,
and % saturation (which is determined by PaO2).

Hypoxia is decreased O2 delivery to the tissues, which can be caused by


decreased cardiac output (blood flow) and/or decreased O2 content of blood.

A. Causes of hypoxia

1. Anemia (decreased hemoglobin concentration)


2. Decreased cardiac output
3. Hypoxemia (decreased PaO2, decreased % saturation of Hb)...any of
the causes of hypoxemia produces hypoxia
4. CO poisoning (decreased O2-binding capacity of hemoglobin)
5. CN poisoning (uncoupler of oxidative phosphorylation)

VI. PRACTICE QUESTIONS

1. Use the following information to answer the questions.

What is the volume of the physiologic dead space?

906
Ventilation/Perfusion (V/Q) 1 & 2 - Dr. Costanzo

What is the value for minute ventilation?

What is the value for alveolar ventilation?

What is alveolar PCO2 ( PACO2)?

What is the value for alveolar PO2 ( PAO2)?

2. Ventilation to the apex of the lung is 0.8 L/minute and ventilation to the
base of the lung is 2.2 L/minute. Blood flow to the apex is 0.4 L/minute
and blood flow to the base is 3.2 L/minute. What is the V/Q ratio at the
apex and the base of the lung, and what effect would you expect any
differences to have on PO2 and PCO2 in those regions?

3. A person with asthma has the following arterial blood gases while
breathing room air. What is his A -a gradient, and what does the value
represent? Why is the PaCO2 decreased from normal?

4. The following information was obtained in a person with a pulmonary


disease.

What is the magnitude of the intrapulmonary shunt in L/min?

907
Ventilation/Perfusion (V/Q) 1 & 2 - Dr. Costanzo

5. If the V/Q ratio of a lung region decreases, the alveoli in that region will
have a:

A. Higher PO2 and higher PCO2.


B. Lower PO2 and lower PCO2.
C. Higher PO2 and lower PCO2.
D. Lower PO2 and higher PCO2.
E. Lower PO2 and unchanged PCO2.

The following information applies to Questions 6 and 7:

6. The patient’s A - a gradient is closest to:

A. Zero
B. 20 mm Hg
C. 60 mm Hg
D. 270 mm Hg
E. 280 mm Hg

7. If all values remain identical except that FIO2 is lowered to 0.21, the A - a
gradient will be:

A. Increased
B. Decreased
C. Unchanged

8. Pulmonary capillary blood from which lung unit has the lowest PO2?

A. V = 2 L/min; Q = 0.2 L/min


B. V = 2 L/min; Q = 2 L/min
C. V = 0.2 L/min; Q = 2 L/min
D. V = 0; Q = 2 L/min

908
Ventilation/Perfusion (V/Q) 1 & 2 - Dr. Costanzo

9. A patient with a right-to-left cardiac shunt who is breathing room air at sea
level has the following values:

What percentage of the cardiac output is the shunt?

A. Zero
B. 38%
C. 50%
D. 62%
E. 100%

10. Which person is expected to have an increased A -a gradient?

A. Left-to-right cardiac shunt


B. Hypoventilation
C. High altitude
D. Pulmonary fibrosis

11. Which cause of hypoxia is corrected best with supplemental O2?

A. High altitude
B. Right-to-left intrapulmonary shunt
C. Right-to-left cardiac shunt
D. Anemia
E. Decreased cardiac output

12. Compared to the apex of the lung, at the base of the lung:

A. Blood flow is lowest


B. Ventilation is lowest
C. V/Q is highest
D. Alveolar PCO2 is highest
E. Alveolar PO2 is highest

909
Ventilation/Perfusion (V/Q) 1 & 2 - Dr. Costanzo

13. Given the following values, calculate alveolar ventilation:

A. 6.3 L/min
B. 4.8 L/min
C. 3.5 L/min
D. 2.5 L/min
E. 2.0 L/min

14. Using the values given for Question 13, what fraction of each tidal volume
is physiologic dead space, and how does this value compare to normal?

A. 0.06; decreased
B. 0.3; decreased
C. 0.3; normal
D. 0.44; decreased
E. 0.44; increased

15. Using the information given for Question 13, what is the average value for
V/Q in this person?

A. 1.3
B. 1.3 L
C. 0.7
D. 0.7 L
E. 0.8 L

910
Ventilation/Perfusion (V/Q) 1 & 2 - Dr. Costanzo

EXPLANATIONS

1. What is the volume of the physiologic dead space?

What is the value for minute ventilation?

What is the value for alveolar ventilation?

What is alveolar PCO2 ( PACO2)?

911
Ventilation/Perfusion (V/Q) 1 & 2 - Dr. Costanzo

What is the value for alveolar PO2 ( PAO2)?

2.

3. The A-a gradient represents the difference between alveolar PO2 (PAO2 or “A”)
and arterial PO2 (PaO2 or “a”). The A-a gradient tells us whether O2 is equilibrating
normally between alveolar gas and pulmonary capillary blood. For example, the
normal A-a gradient is close to zero because O2 equilibrates almost perfectly —
PAO2 and PaO2 are equal, or nearly equal. However, if there is a mismatch of
ventilation and perfusion (i.e., a V/Q defect), then PaO2 will be less than PAO2, and
A-a will be greater than zero. The greater the disturbance in O2 exchange, the
larger the A-a gradient.

The A-a gradient is determined by measuring “a” (the PO2 of arterial blood,
PaO2) and calculating “A” (the PO2 of alveolar gas, PAO2) with the alveolar gas
equation. Therefore, at 4 P.M.,

912
Ventilation/Perfusion (V/Q) 1 & 2 - Dr. Costanzo

Compared to a normal person, the A-a gradient is greatly increased; O2 could not
fully equilibrate between alveolar gas and pulmonary capillary blood because of
a V/Q defect (specifically, a decreased V/Q ratio).

The PaCO2 is decreased below normal because the person is hyperventilating


secondary to hypoxemia. (PaO2 < 60 mm Hg stimulates the peripheral
chemoreceptors.) Hyperventilation drives off “extra” CO2 and decreases the
PaCO2.

4. Steps:

O2-binding capacity of blood = hemoglobin concentration x O2-binding capacity


= 15 g/100 ml blood x 1.34 ml O2/g hemoglobin
= 20.1 ml O2/100 ml blood

Determine % saturation that corresponds to the various values of PO2

100 mm Hg corresponds to 100% saturation


70 mm Hg corresponds to 90% saturation
30 mm Hg corresponds to 60% saturation

Calculate the O2 content of arterial blood, venous blood, and non-shunted blood.
(Remember, non-shunted blood should have the same PO2 as alveolar gas.)

O2 content of arterial blood

913
Ventilation/Perfusion (V/Q) 1 & 2 - Dr. Costanzo

= Dissolved O2 + O2 hemoglobin

= 70 mm Hg x 0.003 ml O2/100 ml/mm Hg + 20.1 ml O2/100 ml x 0.9


= 0.21 vol % + 18.1 vol %

= 18.3 vol %

O2 content of venous blood

= Dissolved O2 + O2-hemoglobin
= 30 mm Hg x 0.003 ml O2/100 ml/mm Hg + 20.1 ml O2/100 ml x 0.6
= 0.09 vol % + 12.1 vol%
= 12.2 vol %

O2 content of non-shunted blood

= Dissolved O2 + O2-hemoglobin
= 100 mm Hg x 0.003 ml O2/100 ml/mm Hg + 20.1 ml O2/100 ml x 1.0
= 0.3 vol % + 20.1 vol %
= 20.4 vol %

Next step: calculate the shunt fraction as:

QS = O2 content of non-shunted blood - arterial O2 content


QT O2 content of non-shunted blood - venous O2 content

= 20.4 vol % - 18.3 vol %


20.4 vol % - 12.2 vol %

= 2.1 vol %
8.2 vol %

= 0.26, or 26%

Last step! Calculate the volume, in L/min, of the shunt (QS)

QS = 0.26 x cardiac output


= 0.26 x 5.2 L/min
= 1.35 L/min

914
Ventilation/Perfusion (V/Q) 1 & 2 - Dr. Costanzo

5. Answer = D. Begin by drawing a lung and labeling the apex and base with
respect to blood flow, ventilation, PO2, and PCO2. You can probably use it for
other questions, so do it right the first time. Base has highest blood flow and
highest ventilation. Blood flow is relatively higher than ventilation at the base, so
V/Q is lowest at the base and highest at the apex. V/Q determines PO2 and PCO2.
The higher the ventilation relative to perfusion, the higher the PO2 and the lower
the PCO2 (that’s what ventilation does...it adds O2 and takes away CO2, right?) If
V/Q is decreased, less O2 is brought in, less CO2 is taken away. Ok, so you didn’t
really need to draw the lung for this question.

6. Answer = D, 270 mm Hg. This question requires that you first calculate PAO2 with
the alveolar gas equation. Then, the A - a gradient is calculated as the difference
between this calculated “A” and the measured “a.” You’ll need to know the
alveolar gas equation for the exam. “A” = PAO2 = PIO2 - PACO2/R. PIO2 is
calculated from barometric pressure corrected for water vapor times the FIO2 .
PACO2 is assumed to be the same as PaCO2 (given). R is respiratory exchange ratio
or respiratory quotient, which is 0.8. “A” = PAO2 = (760 - 47) x 0.5 - 30/0.8 =
319 mm Hg. Finally, A - a = 319 mm Hg - 50 mm Hg = 269 mm Hg. The
solubilities are extraneous information.

7. Answer = B. You can calculate a new value of “A” at an FIO2 of 0.21. However,
you needn’t go to all that trouble. If everything else is the same, PIO2 at an FIO2 of
0.21 is lower than it is at an FIO2 of 0.5. Thus, the calculated value of “A” will
also be lower. Thus, the A- a gradient will be decreased.

8. Answer = D. PO2 of pulmonary capillary blood is lowest in the region where


ventilation is lowest relative to perfusion. Among the choices, where V = 0, which
is called a shunt.

9. Answer = B. This is a straight shunt calculation. On the exam, you will be given
the equation, but you must understand how to apply it ♣. Non-shunted blood is
assumed to equilibrate normally with alveolar gas; thus, non-shunted (normal)
blood has the same PO2 as alveolar gas, 100 mm Hg. Shunted blood has the same
PO2 as mixed venous blood, 30 mm Hg. Arterial blood has a measured PO2 of 50
mm Hg. First calculate the O2 content of each kind of blood: non-shunted
(normal), mixed venous, and systemic arterial. On the exam, you will be given the
% saturation that corresponds to each PO2. You are given the O2 binding capacity
of blood (always measured at 100% saturation!!) as 20.1 ml O2/100 ml blood. Set
up a table with all the calculated values, thus:

915
Ventilation/Perfusion (V/Q) 1 & 2 - Dr. Costanzo

** Units for PO2 are mm Hg, for % saturation are %, and for O2 content
are vol %.

Qs = 20.4 - 17.3
Qt 20.4 - 12.2

= 3.1
8.2

= 0.38, or 38%

Final comment, if the shunt was zero percent of the cardiac output, the
arterial PO2 would have been equal to alveolar PO2. If the shunt was 100%
of the cardiac output, arterial PO2 would have been equal to mixed venous
PO2. Final, final comment. The way the question was asked, you didn’t
need the value for cardiac output.

10. Answer = D. An A- a gradient is present when systemic arterial blood is not


equilibrated with alveolar gas (with respect to PO2). The more impaired the O2
equilibration, the larger the A- a gradient. Left-to-right cardiac shunts involve
routing already-oxygenated, equilibrated (arterialized) blood from the left heart
back to the right heart and the lungs. Persons with hypoventilation have
decreased alveolar PO2; arterial blood equilibrates with that lower alveolar PO2
and A -a gradient is small or zero. Persons at high altitude have decreased
alveolar PO2 because inspired air has a lower PO2 at lower barometric pressure;
arterial blood equilibrates and A -a gradient is small or zero. In fibrosis, the
diffusion process for O2 is impaired; arterial blood cannot equilibrate with
alveolar gas and A - a is increased.

11. Answer = A. Hypoxia at high altitude is correctable by breathing supplemental


O2; the whole, entire cause of high altitude hypoxia is breathing air with a low
PO2. The hypoxia in right-to-left intrapulmonary or cardiac shunt is not
correctable b/c shunted blood always has the same PO2 and O2 content as mixed
venous blood. The non-shunted (normal) blood can have its PO2 raised by
supplemental O2, but once that blood’s hemoglobin is 100% saturated, the
additional O2 only adds to the dissolved O2, which does little to raise O2 content
of the blood. Hypoxia persists. Hypoxia due to anemia is due to decreased
hemoglobin concentration; once that hemoglobin is fully saturated, giving
supplemental O2 only increases the dissolved O2, which helps very little. Hypoxia
due to decreased cardiac output is a blood flow delivery problem; supplemental
O2 only increases the dissolved O2.

12. Answer = D. Find that lung picture you drew for Question 5. Base has highest
blood flow, highest ventilation, lowest V/Q, thus highest PCO2 (expires less CO2).

916
Ventilation/Perfusion (V/Q) 1 & 2 - Dr. Costanzo

13. Answer = C. First calculate dead space, then calculate alveolar ventilation.
Several values were listed that you don’t need for the calculations, so be careful!

VD = VT x PaCO2 - PECO2
PaCO2

= 450 ml x 45 mm Hg - 25 mm Hg
45 mm Hg

= 200 ml

Alveolar ventilation = (VT - VD) x breaths/minute

= (450 ml - 200 ml) x 14/minute

= 3500 ml/minute, or 3.5 L/minute

14. Answer = E

15. Answer = A. The V in V/Q is total ventilation. V/Q has no units..

917
Renal Acid-Base 1 and 2 - Dr. Costanzo

Renal Acid-Base 1 and 2


Linda Costanzo, Ph.D.

OBJECTIVES:

After studying this lecture, the student should understand:

1. The mechanism for reabsorption of filtered bicarbonate.


2. The factors that change reabsorption of filtered bicarbonate.
3. The mechanism for excreting fixed acid as titratable acid, including the role of
amount and pK of urinary buffer.
4. The mechanism for excreting fixed acid as ammonium, including the role of urine
pH, serum potassium, and adaptive responses to acidosis.
5. How the excretion of fixed acid is altered in disease states such as diabetic
ketoacidosis and chronic renal failure.

OPTIONAL READING:

Physiology, 3rd Edition, Costanzo, L.S., W.B. Saunders, 2006. Chapter 7, pps. 306-312
Physiology, Berne and Levy; Mosby, 2004. Chapter 38

I. INTRODUCTION

As you have seen in the previous lectures, the respiratory system regulates the PCO2.
The kidneys regulate the HCO-3 concentration of arterial blood. HCO-3 is the major
buffer base in extracellular fluid. The role of the kidneys in normal acid-base
balance is three-fold:

A. Reabsorption of filtered HCO-3. Reabsorption of filtered HCO-3 is


essential in order to preserve the extracellular stores of HCO-3. This
process occurs without the net secretion of H+.

B. Excretion of H+ as titratable acid. This process involves net secretion of


H+ and the synthesis of new HCO-3 within the renal cells. Reabsorption
of this newly synthesized HCO-3 helps replenish HCO-3 stores in
extracellular fluid. Recall that extracellular HCO-3 is used to buffer the
fixed acids resulting from metabolism of proteins and phospholipids.
Excretion of H+ as titratable acid is limited by the amount of buffer,
usually HPO4-2, present in tubular fluid.

C. Excretion of H+ as NH4+. This process, like formation of titratable acid,


involves net secretion of H+ and synthesis of new HCO-3 within renal
cells. Reabsorption of this newly synthesized HCO-3 also helps replenish

918
Renal Acid-Base 1 and 2 - Dr. Costanzo

HCO-3 stores. Excretion of H+ as NH4+ requires synthesis of NH3 within


renal cells; the potential supply of NH3 is enormous and therefore so is the
potential for H+ excretion as NH4+.

II. REABSORPTION OF FILTERED HCO-3

A. How much filtered HCO-3 is reabsorbed?

In a 70 kg male with a glomerular filtration rate of 180 liters/day and a


plasma [HCO-3] of 24 mEq/L, the filtered load of HCO-3 is 4320 mEq/day
(GFR x plasma concentration). About 2 mEq/day of HCO-3 are excreted in
urine. Thus, 4318 mEq/day are reabsorbed into blood, well over 99% of
the filtered load.

Recall that daily ingestion and metabolism of protein and phospholipid


produces fixed acid in the body, either H2SO4 or H3PO4. These fixed acids
are buffered by extracellular HCO-3; the resulting CO2 is excreted via the
lungs. This type of buffering consumes some of the HCO-3 in extracellular
fluid. Thus, it is important that virtually all of the filtered HCO-3 be
conserved, i.e. reabsorbed. (How the HCO-3 is replenished by excretion of
H+ as titratable acid or NH4+ will be covered later in the lecture.

919
Renal Acid-Base 1 and 2 - Dr. Costanzo

B. Mechanism

Figure 1.

Recall that in normal man, virtually all (99.9%) of filtered HCO-3 is


reabsorbed back into blood by the renal tubular cells. The diagram above
shows that, primarily in early proximal tubule, H+ are secreted from the
cell across the luminal membrane into tubular fluid. This H+ combines
with filtered HCO-3 to form (briefly) H2CO3. H2CO3 quickly decomposes
to H2O and CO2 in the presence of brush border carbonic anhydrase.
The CO2 formed is highly lipid soluble and so readily crosses the luminal
membrane, recombines with H2O inside the cell to form H2CO3 catalyzed
by intracellular carbonic anhydrase. H2CO3 dissociates into H+ and HCO-3.
The HCO-3 is reabsorbed back into peritubular blood and the H+ is
secreted across the luminal membrane to combine with another filtered
HCO-3, starting the cycle over again. The net result is reabsorption of
filtered Na+ and HCO-3 without the net secretion of H+.

920
Renal Acid-Base 1 and 2 - Dr. Costanzo

The key points are:

1. For every HCO-3 reabsorbed, a Na+ ion is reabsorbed along with it.
The net result is the reabsorption of NaHCO3. Part of the Na
reabsorption, primarily in the early part of the proximal tubule, is
linked to HCO-3 reabsorption.

2. No net secretion of H+ occurs via this mechanism. For every H+


secreted into the lumen, a CO2 is formed in the lumen which diffuses
back into the cell to form more H2CO3 and more H+. Net reabsorption
of HCO-3 does occur by this mechanism. Furthermore, the more H+
secreted into the lumen, the more filtered HCO-3 which is reabsorbed.
When all of the filtered HCO-3 has been reabsorbed, then net secretion
of H+ can occur (see below).

3. There is little change in the pH of proximal tubule fluid accompanying


the reabsorption of filtered HCO-3. This observation is consistent with
lack of net H+ secretion and the presence of luminal carbonic
anhydrase.

C. Influences on reabsorption of filtered HCO-3.

1. Filtered HCO-3 load. As the plasma concentration of HCO-3 changes,


the filtered load (GFR x plasma HCO-3) changes proportionately. Up
to plasma concentrations of 40 mEq/L, the amount reabsorbed equals
the amount filtered. Above 40 mEq/L, apparent saturation of
reabsorption occurs, as if the process were Tm-limited.
In metabolic alkalosis when the plasma HCO-3 concentration and the
filtered load of bicarbonate are high, excess HCO-3 is excreted in the
urine; this constitutes an important renal mechanism. In metabolic
alkalosis, H+ concentration is low, both extracellular and intracellular,
thus diminishing the H+ available for secretion into luminal fluid;
without secreted H+, the filtered HCO-3 will not be reabsorbed, but
rather excreted in urine.

2. Plasma Cl- concentration. There is usually a reciprocal relationship


between plasma Cl- concentration and plasma HCO-3 concentration to
maintain charge balance. Thus if plasma Cl- goes up, plasma HCO-3
goes down, the filtered load of HCO-3 goes down, and the amount of
HCO-3 reabsorbed goes down.

3. Extracellular fluid volume. The effect of extracellular volume on


NaHCO3 reabsorption is similar to its effect on NaCl reabsorption.
Increased extracellular fluid volume decreases NaHCO3 reabsorption
in proximal tubule because it increases peritubular capillary

921
Renal Acid-Base 1 and 2 - Dr. Costanzo

hydrostatic pressure (Pc) and decreases peritubular capillary colloid


osmotic (oncotic) pressure (πc). These changes in Starling forces
decrease reabsorption of fluid and electrolytes (NaCl and NaHCO3)
from the lateral intracellular space into peritubular capillary blood as
shown in the figure below. Conversely, extracellular volume
contraction increases proximal HCO-3 reabsorption because of
decreased peritubular capillary hydrostatic pressure and increased
peritubular πc.

Figure 2.

4. Angiotensin II. Increased levels of angiotensin II, as would be seen in


extracellular fluid volume contraction, stimulates Na+-H+ exchange
and therefore increases reabsorption of filtered HCO-3. This
mechanism adds to the direct effects of volume contraction, via
Starling forces, described above. Together, the effects of Starling
forces and increased angiotensin II in volume contraction leads to a
condition called contraction alkalosis, i.e., metabolic alkalosis
secondary to ECF volume contraction.

922
Renal Acid-Base 1 and 2 - Dr. Costanzo

5. Arterial pCO2. Increased arterial pCO2 (as in respiratory acidosis)


increases filtered HCO-3 reabsorption by increasing the supply of CO2
to the cells. Remember that CO2 supplies the H+ for secretion.
Conversely, decreased arterial pCO2 (as in respiratory alkalosis)
decreases filtered HCO-3 reabsorption because the CO2 supply to the
cells is diminished and less H+ is available for secretion. These
mechanisms constitute the renal compensations for the respiratory
acid-base disturbances and help to restore the arterial pH toward
normal.

Respiratory acidosis:

↑ PCO2 ---> ↑ HCO-3 reabsorption ---> ↑ plasma HCO-3 ---> ↑ arterial pH toward normal
Respiratory alkalosis:
↓ PCO2 ---> ↓ HCO-3 reabsorption ---> ↓ plasma HCO-3 --->↓ arterial pH toward normal

III. EXCRETION OF H+ AS TITRATABLE ACID.

A. Mechanism of formation and excretion of titratable acid. In this


process, HPO4-2 present in the glomerular filtrate is converted to H2PO4-
by the secretion of H+ into the tubular fluid. The figure below illustrates
the process.

Figure 3.

923
Renal Acid-Base 1 and 2 - Dr. Costanzo

Formation of titratable acid occurs in all nephron segments. It depends


upon intracellular (but not luminal) carbonic anhydrase. CO2 is provided
to the renal cells either from metabolism or from peritubular blood. CO2
reacts with H2O within the cell to form H2CO3 and then generates H+ and
HCO-3. The HCO-3 is reabsorbed into peritubular blood. The H+ is
secreted into the lumen and combines with filtered HPO4-2 to form H2PO4-
which is then excreted in the urine. In contrast to the mechanism for
reabsorption of filtered HCO-3, net secretion of H+ occurs; furthermore the
HCO-3 reabsorbed has been synthesized within the renal cells to replenish
extracellular HCO-3 stores (depleted from buffering fixed acids).

B. Influences on excretion of titratable acid.

1. Availability of urinary buffers (i.e. HPO4-2). The more HPO4-2 in


tubular fluid, the greater the excretion of H+ as titratable acid.
Phosphate is filtered mainly in the HPO4-2 form because the pH of
extracellular fluid is 7.4. i.e.:

As H+ are secreted, they titrate HPO4-2 to form H2PO4-. The minimum


urinary pH is 4.4 because distal tubule and collecting duct cells
cannot secrete H+ against a gradient of more than 1:1000 (3 pH units).
When the minimum urine pH is reached, phosphate will exist
primarily in the H2PO4- form, i.e.:

Once all phosphate is in the H2PO4- form and there is no more HPO4-2 to
be titrated, any further H+ secretion would cause a drastic fall in urine pH
below 4.4 and this is prohibited. Thus, the only way to excrete more H+ as
titratable acid is to provide more HPO4-2 in the glomerular filtrate.

924
Renal Acid-Base 1 and 2 - Dr. Costanzo

Figure 4.

2. pK'of the urinary buffer. A urinary buffer is most effective when the
pH of the tubular fluid is within 1.0 unit of the buffer's pk'. The figure
above shows that the phosphate buffer pair with a pK' of 6.8 is a more
effective urinary buffer than creatinine with a pK' of 4.97. These
experiments were done by Dr. Robert Pitts, a world-renowned renal
physiologist, on himself. The shaded area under each titration curve
shows the total amount of H+ excreted. Phosphate, with its pK of 6.8,
is nearly ideal as a urinary buffer: the linear range of the titration curve
overlaps almost perfectly with the range of urinary pH from 7.4
(glomerular filtrate) to 4.4 (minimum pH of final urine). Contrast
creatinine, with its pK of 5.0. The linear, or buffering, range of the
creatinine curve lies between pH 6 and pH 4. Very little H+ can be
added to urine before urine pH falls to the minimum value of 4.4.

IV. EXCRETION OF H+ AS NH4+

The second route for excretion of H+ is NH4+ (60% of total H+ excretion). This
second route is vitally important, since excretion of H+ as titratable acid is limited
by the amount of urinary buffer; as soon as all urinary buffers (usually phosphate)
are titrated by H+ and converted to their HA forms, then urine pH would fall to
4.4 and H+ secretion would cease. The additional mechanism is the titration of
NH3 to NH4+ by secreted H+.

925
Renal Acid-Base 1 and 2 - Dr. Costanzo

A. Mechanism

Figure 5.

Excretion of H+ as NH4+ involves the proximal tubule, the thick ascending limb of Henle,
and the intercalated cells of collecting duct and occurs in the following steps:

1. In the proximal tubule, NH3 is synthesized from glutamine and is


converted to NH4+ inside the cells. NH4+ is secreted into the lumen by
substituting for H+ on the Na+-H+ exchanger. For every NH4+ secreted,
one "new" HCO3- is reabsorbed. The NH4+ travels down the nephron.

926
Renal Acid-Base 1 and 2 - Dr. Costanzo

2. Some of the NH4+ secreted by the proximal tubule is excreted in urine.


The rest of the NH4+ substitutes for K+ on the Na+-K+-2Cl-
cotransporter of thick ascending limb and is added to the solutes of the
medullary interstitium (not shown in figure above; see countercurrent
multiplication). In interstitial fluid, it is present in both NH3 and NH4+
forms, but primarily as NH4+ (pK is 9.2, so at pH 7.4, most would be
in HA or NH4+ form).

3. The intercalated cells of collecting duct have H+ ATPase and H+-K+


ATPase in the luminal membrane; both transporters secrete H+ into the
lumen of the collecting duct. Then, NH3 (which is highly lipid-soluble)
diffuses from the medullary interstitium into the lumen, combines with
the secreted H+ to form NH4+, and is excreted. This process is called
diffusion trapping because the lipid-soluble form (NH3) diffuses, is
converted to a water-soluble form (NH4+), which is trapped and then
excreted. Note that the lower the NH3 concentration in the lumen, the
more favorable the gradient for NH3 diffusion from the medullary
interstitium and the more H+ excreted as NH4+. This point is illustrated
in the following calculation based on a urine pH of 7.0:

In words, at a urine pH of 7.0, there are 167 NH4+ for every NH3; thus,
the luminal concentration of NH3 remains low, maintaining a gradient
for NH3 diffusion.

B. Effect of urine pH on NH4+ excretion

The lower the urine pH (of course, with a minimum of 4.4), the more H+
excreted as NH4+. Why is this? The answer lies in the effect of urine pH
on the relative concentrations of NH3 and NH4+ that we illustrated in the
calculation above. If urine pH is 5 instead of 7 (i.e., more acidic), even
more is in the NH4+ form and even less as NH3, making an even more
favorable gradient for NH3 diffusion.

927
Renal Acid-Base 1 and 2 - Dr. Costanzo

C. Adaptive increase in NH3 synthesis in acidosis

Metabolic and respiratory acidosis both produce an adaptive increase in


ammoniagenesis after a few days (chronic acidosis). Presumably, there is
an associated decrease in intracellular pH that induces key enzymes in the
glutamine to NH3 pathway. As a result, more H+ can be excreted as NH4+
in chronic acidosis than in acute acidosis.

D. Effect of blood [K+] on NH3 synthesis

Plasma K+ concentration also alters the rate of ammoniagenesis and,


therefore, the excretion of H+ as NH4+. Hyperkalemia inhibits NH3
synthesis and decreases H+ excretion. Hypokalemia increases NH3
synthesis and increases H+ secretion. The effects of plasma K+ may be
mediated by H+ and K+ exchange across cell membranes and, therefore,
actually may be mediated by intracellular pH. (For example, in
hyperkalemia, K+ enters the cells and H+ leaves, increasing intracellular
pH and inhibiting NH3 synthesis.)

V. RELATIVE RATES OF EXCRETION OF H+ AS TITRATABLE ACID


AND NH4+ AND ADAPTATION TO METABOLIC ACIDOSIS

928
Renal Acid-Base 1 and 2 - Dr. Costanzo

1. Normally, more H+ are excreted as NH4+ than as titratable acid, over 2/3 of the
total amount.

2. In uncontrolled diabetes mellitus, there is overproduction of the fixed acid, ß-


OH butyric acid, causing metabolic acidosis. This accounts for the large
adaptive increase in H+ excretion as NH4+ (10X the normal rate). There is
also elevated excretion of H+ as titratable acid, resulting from the increased
plasma level and filtered load of ß-OH butyrate. ß-OH butyrate actually
becomes the primary urinary buffer in diabetic ketoacidosis because of its
high urinary concentration, even though its pK' (4.8) is not ideal. Thus the
high titratable acid excretion is primarily as β-OH butyric acid. Another
example of the trade-off of buffer amount vs pK.

3. In chronic renal disease, the decrease in functioning renal cells causes


decreased H+ excretion as titratable acid and NH4+. As a result, the excretion
of fixed H+ is severely compromised and causes a type of metabolic acidosis
(broadly termed renal tubular acidosis).

929
Respiratory Rhythm - Dr. Costanzo

Respiratory Rhythm: Control of Breathing


Linda Costanzo, Ph.D.

OBJECTIVES:

After studying this lecture, the student should understand:

1. The organization of medullary and pontine centers that control breathing.


2. Effects of lung stretch receptors and peripheral and central chemoreceptors on the
dorsal respiratory group of neurons.
3. The function of the central chemoreceptors and their effect on ventilation.
4. The function of the peripheral chemoreceptors and their effect on ventilation.

To maintain constant arterial PO2 and PCO2, the frequency and depth (volume) of
breathing is tightly controlled. Breathing is normally an involuntary process controlled
by the brain stem (we don’t have to think about it.....how nice!) It can be overridden by
intentional hyperventilation or hypoventilation (breath-holding).

I. BRAIN STEM CENTERS

Centers that control breathing are located in the medulla and pons of the brain
stem. Information from chemoreceptors and lung receptors in the periphery feed
afferent information to these brain stem centers. Efferent information from the
brain stem controls the function of muscles for breathing (e.g., diaphragm).

Figure 1.

930
Respiratory Rhythm - Dr. Costanzo

A. Dorsal respiratory group (inspiratory center)–medulla

The dorsal respiratory group of neurons, in the reticular formation of the


medulla, is the inspiratory center. It controls the frequency of breaths and the
basic rhythm of breathing. These dorsal neurons cause repetitive bursts of
action potentials in the phrenic nerve that innervates the diaphragm
(causing the diaphragm to contract). The breathing cycle begins with
quiescence, followed by action potentials that ramp up in frequency, followed
by quiescence. Contraction of the diaphragm (for inspiration) follows the
same pattern: quiescence, contraction, quiescence.

The dorsal group is inhibited by the pontine pneumotaxic center and lung
stretch (via the vagus). It is also regulated by peripheral chemoreceptors (via
the vagus and glossopharyngeal nerves) and by central chemoreceptors.

B. Pneumotaxic center–upper pons

The pneumotaxic center is located in the upper pons. It inhibits the dorsal
respiratory group and turns off inspiration. In effect, the pneumotaxic center
determines tidal volume because it ends the period of inspiration. Loss of
this center prolongs inspiration.

The vagus, like the pneumotaxic center, inhibits the dorsal respiratory group.
Specifically, vagal afferents from lung stretch receptors are stimulated
during inspiration. Firing of the vagus then inhibits, or limits, inspiration.
(Information from peripheral chemoreceptors is also transmitted to the
dorsal respiratory group, both via the vagus and glossopharyngeal nerves.)

C. Apneustic center–lower pons

The apneustic center in the lower pons activates the dorsal respiratory group
and prolongs the ramped period of action potentials. When stimulated, there
are prolonged inspiratory gasps, called apneusis.

D. Ventral respiratory group (expiratory center)–medulla

The ventral respiratory group is the expiratory center of the medulla. In


normal quiet breathing, expiration is passive (due to the elastic recoil of the
lung) and no ‘center’ is involved. During exercise, the ventral group is
activated and drives activity in the abdominal and internal intercostal muscles.

931
Respiratory Rhythm - Dr. Costanzo

II. RECEPTORS THAT REGULATE BREATHING

Figure 2.

A. Lung stretch receptors are located in the smooth muscle of the airways and
are activated in response to distension (inspiration). The information is carried
via the vagus to the dorsal respiratory group, where it inhibits activity and
ends inspiration (in collaboration with the pontine pneumotaxic center).

B. Central chemoreceptors are located on the ventral surface of the medulla,


near the dorsal respiratory group (inspiratory center). They are sensitive to
changes in arterial PCO2 and are the most important factor in the minute-to-
minute control of breathing.

932
Respiratory Rhythm - Dr. Costanzo

Figure 3.

The central chemoreceptors detect changes in arterial PCO2 and alter the
breathing rate as follows. Increases in PCO2 are detected by the central
chemoreceptors, which direct an increase in breathing rate; excess CO2 is
expired and PCO2 is restored to normal. Decreases in PCO2 direct a decrease in
breathing rate; CO2 is retained and PCO2 is restored to normal. The steps in
detecting an increase in PCO2 are as follows. Note that the sensor on the
central chemoreceptors is for H+, not for CO2.

1. CO2 in blood crosses the blood-brain barrier and enters the CSF, where
it combines with H2O to form H2CO3, which dissociates into H+ and
HCO3-.

2. There is an increase in H+ concentration and decrease in pH of CSF.

3. The central chemoreceptors, located in the ventral medulla, detect


increased H+ in the CSF and direct the dorsal respiratory group to
increase the breathing rate (hyperventilation) to expire extra CO2.

C. Peripheral chemoreceptors for O2, CO2, and H+ are located in the carotid
bodies near the bifurcation of the common carotid arteries and in the aortic
arch. Information from the peripheral chemoreceptors is carried via the vagus
and glossopharyngeal nerves to the dorsal respiratory group (inspiratory
center), which directs the appropriate change in breathing rate.

1. The most important role of the peripheral chemoreceptors is to detect


decreases in arterial PO2. However, peripheral chemoreceptors are
relatively insensitive to PO2 and are activated when PO2 is less than 60
mm Hg. When activated, they increase the breathing rate and attempt
to restore PO2 to normal.

933
Respiratory Rhythm - Dr. Costanzo

2. Peripheral chemoreceptors also detect increases in arterial PCO2 and


direct an increase in breathing rate. This response is less important
than their response to a decrease in PO2 and less important than the
response of central chemoreceptors to changes in PCO2. The effects of
increased PCO2 and decreased PO2 are additive in peripheral
chemoreceptors.

3. Peripheral chemoreceptors also detect decreases in arterial pH,


independent of changes in PCO2, and direct an increase in breathing
rate. The effect of decreased pH is additive with the effect of
decreased PO2.

D. Other receptors

1. Mechanoreceptors are located in the muscles and joints that detect


movement of the limbs. For example, there is an anticipatory response to
exercise that involves an early increase in breathing rate.

2. Irritant receptors to noxious chemicals and particles, located in the


linings of airways, cause constriction of bronchial smooth muscle and an
increase in breathing rate.

3. J receptors, or juxtacapillary receptors, are located in alveolar walls.


Engorgement of pulmonary capillaries with blood, such as in left heart
failure (blood backs up into pulmonary circulation) causes rapid shallow
breathing.

934
Ventilation in Exercise - Dr. Costanzo

Ventilation in Exercise and at High Altitude


Linda Costanzo, Ph.D.

OBJECTIVES:

After studying this lecture, the student should understand:

1. Understand the respiratory responses to exercise, including effects on ventilation,


arterial blood gases, arterial pH, mixed venous blood gases, and the oxygen-
hemoglobin dissociation curve.
2. Understand the respiratory responses to ascent to high altitude, including effects
on ventilation, arterial PO2, acid-base balance.
3. Understand the adaptive responses that occur following ascent to high altitude.

I. Responses to Exercise

In exercise, the body’s demand for O2 and its production of CO2 increase. Extra
O2 is supplied and extra CO2 expired by increasing the ventilation rate. Matching
of ventilation to O2 consumption/CO2 production is remarkably effective —
trained athletes can increase their O2 consumption, CO2 production, and
ventilation rate by 15-fold!

We don’t precisely know how the increase in ventilation is accomplished – how


the lungs are told that more O2 is needed, that more CO2 needs to be expired.
Here’s a summary of what is known.

Figure 1.

935
Ventilation in Exercise - Dr. Costanzo

A. Cardiac output is increased in exercise, an important part of the O2 delivery


equation (O2 delivery = cardiac output x O2 content of blood). Thus,
pulmonary blood flow, which is the cardiac output of the right heart,
increases and more pulmonary capillary beds are open (are perfused). The
V/Q ratio becomes more ‘even’ across the lungs and there is decreased
physiologic dead space (less wasted ventilation) .

B. Arterial PO2 and arterial PCO2 do not change during exercise. This is
surprising, since the obvious hypothesis to explain how ventilation increases
in exercise is that a decrease in PO2 or an increase in PCO2 is sensed by
chemoreceptors that tell the dorsal respiratory group that more ventilation is
needed. Although mean values of arterial PO2 and PCO2 do not change during
exercise, there may be oscillations in their values (in particular for CO2)
during each breathing cycle that are immediately sensed by chemoreceptors;
the adjustment in ventilation quickly restores their values to normal during
that breathing cycle.

C. Arterial pH does not change during moderate exercise. However, in severe


exercise, if lactic acidosis develops, then arterial pH decreases.

D. Mixed venous PCO2 increases during exercise because the tissues are
producing more CO2, which is carried in mixed venous blood to the lungs to
be expired. Because matching of ventilation to CO2 production is precise, this
extra CO2 never arrives in systemic arterial blood (i.e., it is completely
expired).

E. Muscle and joint mechanoreceptors are activated early in exercise, even in


anticipation of exercise. They command the dorsal respiratory group to
increase ventilation.

F. The O2-hemoglobin dissociation curve shifts to the right because of


increased tissue PCO2, decreased tissue pH, and increased body temperature.
The right-shift is helpful because it causes increased P50 and decreased
affinity for O2, making hemoglobin release its O2 more readily in exercising
muscle.

II. Responses to High Altitude

Ascent to high altitude causes hypoxemia. Above sea level, barometric pressure
decreases. (For example, at 18,000 ft above sea level, barometric pressure is 380
mm Hg compared to 760 mm Hg at sea level). Accordingly, partial pressures of
all atmospheric gases are decreased. For example, in humidified atmospheric air
at 380 mm Hg, PO2 is:

936
Ventilation in Exercise - Dr. Costanzo

PIO2 = (PB - PH2O) x FIO2


= (380 mm Hg - 47 mm Hg) x 0.21
= 70 mm Hg

Because the air breathed at 18,000 feet has a much lower PO2 (70 mm Hg) than air
at sea level (150 mm Hg), alveolar gas will also have a much lower PO2.
Pulmonary capillary blood, which becomes systemic arterial blood, equilibrates
with alveolar gas will have that same lower PO2, i.e., hypoxemia. It is possible to
live at high altitude because of the following compensatory responses.

Figure 2.

A. Hyperventilation occurs in response to hypoxemia. Remember, arterial PO2 <


60 mm Hg stimulates peripheral chemoreceptors, which direct the dorsal
respiratory group to increase breathing rate. A consequence of
hyperventilation is that “extra” CO2 is expired, reducing arterial PCO2 and
causing respiratory alkalosis. Early in the response, the decrease in PCO2
inhibits central chemoreceptors, offsetting the increase in ventilation.
However, within days, renal compensation for respiratory alkalosis occurs
(increased HCO3- excretion), blood and CSF HCO3- concentrations decrease,
and the pH of CSF decreases toward normal; at this point, there is no more
offsetting, and hyperventilation resumes. Carbonic anhydrase inhibitors
(acetazolamide) increase renal HCO3- excretion and are used to compensate
the respiratory alkalosis.

937
Ventilation in Exercise - Dr. Costanzo

B. Hypoxemia causes increased synthesis of 2,3 DPG, which causes a right-


shift of the O2-hemoglobin curve. The right-shift increases P50 and decreases
affinity for O2, making it easier to unload O2 in the tissues (although harder to
load O2 in the lungs).

C. Hypoxemia causes increased synthesis of erythropoietin, stimulation of RBC


production, and polycythemia, i.e., increased O2-carrying capacity of blood.

D. The decreased alveolar PO2 causes hypoxic vasoconstriction, which increases


pulmonary vascular resistance. To maintain constant pulmonary blood flow,
pulmonary arterial pressure increases (pulmonary hypertension). There is
right ventricular hypertrophy in response to the increased pulmonary
arterial pressure.

938
Acidosis Alkalosis - Dr. Costanzo

Acid-Base Disturbances: Metabolic and Respiratory


Linda Costanzo, Ph.D.

OBJECTIVES:

After studying this lecture, the student should understand:

1. How to superimpose the simple acid-base disturbances on the acid-base map


(graphical presentation of Henderson-Hasselbalch equation).
2. The meaning of the plasma (or serum) anion gap and how to calculate it.
3. Causes and pathophysiology of metabolic acidosis.
4. Causes and pathophysiology of metabolic alkalosis.
5. Causes and pathophysioogy of respiratory acidosis.
6. Causes and pathophysiology of respiratory alkalosis.

Suggested Reading: Physiology, Saunders, Costanzo; W.B. Saunders, 2006. Chapter 7.

I. AIDS TO UNDERSTANDING ACID-BASE DISORDERS

The analysis of clinical problems of H+ imbalance relies on the Henderson-


Hasselbach equation. Various aids are available for distinguishing between the
types of disturbances.

A. Graphical depictions of the Henderson-Hasselbalch Equation. Such an


acid-base template is shown in the figure below. The acid-base map,
shown earlier, is now superimposed with ranges of values of pH, PCO2
and HCO3- typical for the four simple acid-base disorders.

939
Acidosis Alkalosis - Dr. Costanzo

Figure 1.

Shaded areas have been superimposed on the acid-base nomograph. These


shaded areas exemplify the ranges of arterial PCO2 and [HCO-3] for
graded degrees of simple acid-base disorders. Values within the shaded
areas are consistent with, but not always diagnostic of, the simple
disorders. Values outside the shaded areas denote mixed disorders.

There are two shaded areas for each of the respiratory disturbances,
distinguishing the acute from the chronic disorder. For illustration,
consider respiratory acidosis. The acute line depicts the immediate values
within minutes of hypoventilation and retention of CO2. The denominator
of the Henderson-Hasselbalch equation (CO2) increases immediately and
causes a large fall in pH. The chronic line shows the values after the renal
compensatory mechanisms are established (increased HCO-3
reabsorption). This compensatory response raises the HCO3-

940
Acidosis Alkalosis - Dr. Costanzo

concentration, which tends to normalize the ratio of HCO3-/CO2 and to


bring pH closer to normal. PCO2 will remain very high until the cause of
the respiratory acidosis (hypoventilation) is corrected. The metabolic
disturbances require only one line each because respiratory compensation
brings the pH into the near-normal range almost immediately.

(YOU CAN SEE FROM THIS EXAMPLE THAT GREAT


FACILITY WITH THE HENDERSON-HASSELBAlCH
EQUATION AND THE LAW OF MASS ACTION IS REQUIRED
TO UNDERSTAND THESE GRAPHS.)

B. Anion Gap (another aid)

Figure 2.

The figure above shows that in a given body fluid compartment, e.g.,
plasma, the concentration of cations and anions must be equal for
electroneutrality. In plasma, the major cation is Na+ and the major anions
are Cl- and HCO3-. When a comparison is made between Na+
concentration and the sum of Cl- and HCO3-, there is a "gap". This so-
called anion gap is comprised of unmeasured anions such as phosphate,
sulfate, organic acids and proteins; these anions are present (they must be
for electroneutrality), but they are not measured. The anion gap is defined
as:

941
Acidosis Alkalosis - Dr. Costanzo

Anion gap = [Na+] – ([Cl-] + [HCO-3])


= 140 mEq/L – (105 + 24 mEq/L)
= 11 mEq/L

The normal value for anion gap is 10-16 mEq/l. Again, the anion gap
represents anions in plasma that are present, but are unmeasured.

An increase in the anion gap is seen in some forms of metabolic acidosis,


due to the accumulation of organic acids. (a) A good example is
uncontrolled diabetes mellitus, where plasma concentrations of ß -
hydroxybutyric acid and acetoacetic acid are greatly increased. The excess
fixed H+ is buffered by HCO-3, reducing the HCO3- concentration. The
decrease in HCO3-, a measured anion, must be offset by an increase in
another anion to preserve electroneutrality. In this case, HCO3- is replaced
by the ketoanions, which are unmeasured thus increasing the anion gap.
(b) Renal failure is another situation where the anion gap is increased due
to decreased renal excretion of various anions (sulfate, phosphate, urate).
Again, there is accompanying metabolic acidosis, decreased HCO-3
concentration, and increased unmeasured anions such as sulfate and
phosphate.

The major clinical use of the anion gap is in the differential diagnosis of
metabolic acidosis. Beware! A large anion gap may exist without
metabolic acidosis as the primary disorder. An example is severe
vomiting where the overriding acid-base disorder is metabolic
alkalosis, but the anion gap increases because of the switch to fat
metabolism and resulting production of keto acids.

942
Acidosis Alkalosis - Dr. Costanzo

II. METABOLIC ACIDOSIS

A. Causes of metabolic acidosis

Metabolic acidosis is characterized by an acid blood pH, decreased


[HCO-3] (from buffering) and a decreased PCO2 (from respiratory
compensation). The following table lists a few causes of metabolic
acidosis.

Table 1. Some causes of metabolic acidosis

Inability to 1. Renal failure *


excrete H+ 2. Type 4 renal tubular acidosis (e.g., hypoaldosteronism)
1. Lactic acidosis (shock) *
2. Ketoacidosis (diabetes mellitus) *
Fixed acid load
3. Ingestion of salicylate, NH4Cl (HCl), methanol (converted to formic
acid); ethylene glycol (converted to oxalic acid)
1. Gastrointestinal HCO-3 loss (diarrhea) *
HCO3- loss
2. Carbonic anhydrase inhibitors (Type 2 renal tubular acidosis)
* Most common

B. Case study of metabolic acidosis. A 32 year old man has been followed
for 7 years after a β-hemolytic streptococcus A infection. The patient had
been working full time and eating a normal diet but recently complained
of tiring very easily.

His physician obtained the following laboratory values:

Plasma [Na+] 142 mEq/l


Plasma [K+] 4.5 mEq/l
Plasma [Cl-] 102 mEq/l
Arterial pH 7.33
PaCO2 32

His BUN and plasma creatinine were elevated.

Comments: The history and BUN suggest chronic renal failure due to
post-streptococcal glomerulonephritis. The HCO-3 was not determined in
the lab, so we must use the Henderson-Hasselbalch equation to calculate
the arterial [HCO-3] as 16.3 mEq/l. These arterial values fall within the
limits of uncomplicated primary metabolic acidosis. The extent to which
the PCO2 is lower than normal reflects compensatory hyperventilation.
The anion gap is 142 - 102 - 16 mEq/l or 24 mEq/l, clearly elevated and in
this case confirmatory of simple metabolic acidosis. This patient with
chronic renal failure is in Na+, Cl-, and K+ balance reflected by their
normal values. However, because the patient is on a normal protein diet,

943
Acidosis Alkalosis - Dr. Costanzo

he is making the normal amount of fixed acid. He cannot excrete it


because of renal failure. In particular, the renal failure results in decreased
NH3 production and thus inability to excrete H+ as NH4+.

III. METABOLIC ALKALOSIS

A. Causes of metabolic alkalosis

Table 2. Some causes of metabolic alkalosis

1. Vomiting or nasogastric suction *


H+ Loss 2. Hyperaldosteronism *
3. Diuretics (contraction alkalosis) *
HCO3- Excess 1. Administration of HCO3- (iatrogenic)

Aldosterone excess increases H+ secretion by the distal tubule, causing


excess renal H+ loss and metabolic alkalosis.

Contraction alkalosis or volume contraction alkalosis is also called Cl-


depletion alkalosis. It results from administration of loop or distal
diuretics, but not from carbonic anhydrase inhibitors. Typically, the loop
or distal diuretic produces an isotonic NaCl diuresis without the loss of
NaHCO3. Loss of extracellular NaCl causes volume contraction which in
turn causes increased reabsorption of salt (including NaHCO3) and water
in the proximal tubule (Starling Forces); the increased circulating levels of
Angiotensin II also stimulate HCO-3 reabsorption directly. Increased
HCO-3 reabsorption causes increased blood [HCO-3] and metabolic
alkalosis.

B. Case study of metabolic alkalosis. A 44-year old man had a duodenal


ulcer for eight years. For two weeks prior to admission, he was unable to
keep down food and lost more than 3 kg. On admission he was vomiting
and very weak. On examination, his skin turgor was decreased and his
jugular neck veins flat. His EKG showed flattening of T waves and
prolonged Q-T interval, interpreted to be consistent with K+ deficiency.
Results of his laboratory results were:

944
Acidosis Alkalosis - Dr. Costanzo

On Admission After Treatment


Arterial pH 7.53 7.41
Arterial PCO2 56 mmHg 51 mmHg
Arterial [HCO-3] 45 mEq/l 31 mEq/l
Plasma [Na+] 132 mEq/l 141 mEq/l
Plasma [Cl-] 65 mEq/l 97 mEq/l
Plasma [K+] 2.2 mEq/l 3.4 mEq/l
Anion Gap 22 mEq/l 13 mEq/l

Comments. On admission, the laboratory values for pH (elevated), PCO2


(elevated) and HCO-3 (elevated) are consistent with metabolic alkalosis
compensated by hypoventilation. The primary event during vomiting is the
loss of gastric HCl. The anion gap is large, because of fat metabolism
during the starvation period. Extracellular fluid volume contraction is
suggested by the decreased skin turgor and flat jugular veins.

Figure 3.

945
Acidosis Alkalosis - Dr. Costanzo

The initiating event with vomiting is the loss of HCl from the stomach.
Recall that secretion of H+ into stomach results in the addition of HCO-3 to
blood. Normally, the HCl secreted into stomach enters the duodenum
where it stimulates secretion of an equivalent amount of NaHCO3. When
HCl is eliminated by vomiting, this NaHCO3 is not secreted, but is
retained in blood instead, increasing the blood HCO concentration and
causing metabolic alkalosis.

The generation of metabolic alkalosis in vomiting is more complicated


than the simple loss of gastric H+. There are concomitant losses of Cl-,
Na+ and K+ from gastric juices. The loss of gastric Na+ and H2O, leads to
extracellular volume contraction. The resulting volume contraction
complicates the original metabolic alkalosis caused by vomiting because,
independently, it causes metabolic alkalosis by activating the renin-
angiotensin II-aldosterone system. Increased angiotensin II stimulates
HCO reabsorption in proximal tubule; increased aldosterone stimulates H+
secretion and new HCO reabsorption. Together, the effects of angiotensin
II and aldosterone increase the blood HCO concentration and cause
metabolic alkalosis.

Although the primary cause was H+ loss from the stomach, the
metabolic alkalosis is maintained by secondary volume contraction.

Hypokalemia is prominent in metabolic alkalosis due to vomiting because


of (1) loss of K+ in gastric juice; (2) increased aldosterone levels
(secondary to volume contraction) stimulate distal K+ secretion; and (3)
H+ leaves cells in exchange for K+.

Assuming the vomiting stops the patient's treatment depends upon (1)
repleting the extracellular fluid volume by giving saline and (2) correcting
the negative K+ balance by administering KCl.

946
Acidosis Alkalosis - Dr. Costanzo

Figure 4.

IV. RESPIRATORY ACIDOSIS

A. Causes of respiratory acidosis

Table 3. Some causes of respiratory acidosis

1. Opiates, anesthetics
Inhibition of the medullary
2. Cardiac arrest
respiratory center
3. Central sleep apnea
1. Myasthenia gravis
2. Guillain-Barre syndrome
Disorders of the respiratory muscles 3. Poliomyelitis
4. Amyotrophic lateral sclerosis
5. Multiple sclerosis

Upper airway obstruction

Disorders of gas exchange across 1. Chronic obstructive pulmonary disease (bronchitis,


pulmonary capillary emphysema)

947
Acidosis Alkalosis - Dr. Costanzo

B. Case study of respiratory acidosis. A 44 year old woman had asthma


since childhood and had been a heavy smoker since her teens. Over the
past 4-5 years, she experienced progressive shortness of breath,
somnolence and developed some ankle edema. For the latter, she received
mild diuretic therapy. At a routine check-up, she was observed to be short
of breath, with a barrel shaped chest, with poor respiratory excursions. Her
laboratory values are shown in the table. During the next two months, the
patient became increasingly debilitated and somnolent. At the time of her
terminal admission, she was comatose. Results of laboratory tests on that
admission are also shown. She died in her sleep 4 hours later.

Routine Check-up Terminal Admission


Arterial pH 7.34 7.09
Arterial PCO2 58 mmHg 52 mmHg
Arterial [HCO-3] 30 mEq/l 15 mEq/l
Arterial PO2 45 mmHg 30 mmHg
Anion Gap 19 mEq/l 32 mEq/l

Comment: This patient had severe pulmonary emphysema from asthma,


smoking and chronic infection. The resulting poor alveolar ventilation led
to chronic retention of CO2, reflected in her high PCO2. The high [HCO-3]
can be accounted for by two mechanisms: (1) the Law of Mass Action,
driving our favorite reaction to the right and (2) compensatory increase in
tubular reabsorption of HCO-3 which accompanied the elevation in arterial
PCO2. The mild, then pronounced elevation in the anion gap probably
reflects a superimposed lactic acidosis due to tissue anoxia.

The long-standing lung disease may lead to pulmonary hypertension, heart


failure, and edema. The somnolence was due to CO2 retention and coma
was due to elevated PCO2 plus severe acidosis.

At the final admission, the patient was probably more acidotic than would
be expected from uncomplicated chronic respiratory acidosis. The lowered
HCO-3 suggests a superimposed metabolic acidosis. At this time she had a
mixed respiratory and metabolic acidosis, the latter caused by lactic
acidosis. (Note the extremely high anion gap.)

V. RESPIRATORY ALKALOSIS

A. Causes of respiratory alkalosis

948
Acidosis Alkalosis - Dr. Costanzo

Table 4. Some causes of respiratory alkalosis

1. Some pulmonary diseases (pneumonia,


interstitial fibrosis)
Hypoxemia 2. Some congestive heart failure
3. High-altitude residence

1. Psychogenic or voluntary hyperventilation


Direct stimulation of the medullary
2. Salicylate intoxication
respiratory center
3. Neurologic (cerebrovascular accident, tumors)
4. Gram-negative septicemia
Mechanical ventilation

B. Case study of respiratory alkalosis. A 24 year old woman was brought


to the emergency room by a fellow worker. She had a fight with her
boyfriend, had been up all night and gone to work without breakfast. At
the office she became distraught, light-headed and began to hyperventilate
uncontrollably. Soon after, she experienced tingling and numbness of the
hands and feet. She recalls thinking that she must have had a stroke and
was going to die. Upon examination in the emergency room, she was
conscious, breathing rapidly and deeply. Laboratory values on her arterial
blood are shown in the table. It was concluded that the patient was
undergoing psychogenic hyperventilation. She was reassured and asked to
breath in and out of a paper bag. In less than one minute she calmed down
and all symptoms disappeared. Analysis of arterial blood 7 minutes after
arrival at the emergency room showed a return to normal H+ balance. The
patient returned to work.

On Arrival In ER Seven Minutes After Arrival


Arterial pH 7.56 7.41
Arterial PCO2 23 mmHg 41 mmHg
Arterial [HCO-3] 20 mEq/l 25 mEq/l

Comments: Two factors may have contributed to the light-headedness.


First, hypoglycemia from not eating. Second, hyperventilation induced
decreased PCO2 and the resultant alkalosis would lead to decreased
cerebral blood flow and cerebral hypoxia.

The arterial blood values on admission show an uncomplicated, acute


respiratory alkalosis. The small decrease in [HCO-3] results because
Equation 1a is driven to the left. Tingling and numbness were likely
caused by a fall in plasma [Ca+2] secondary to alkalosis. Ca+2 will

949
Acidosis Alkalosis - Dr. Costanzo

exchange with H+ on - charged binding sites on plasma albumin. In


alkalosis, more Ca+2 (less H+) will be bound, causing a decrease in ionized
[Ca+2] in blood and the symptoms of hypocalcemia.

VI. SUMMARY

Daily production of acid challenges the normally alkaline body fluid pH. (1)
Metabolic CO2 is transiently converted to H+ in its transit from the tissue cells to
the lungs; this H+ is effectively buffered primarily by deoxygenated hemoglobin
in erythrocytes of venous blood. (2) Fixed acids result from protein and
phospholipid metabolism. These are buffered by the abundant HCO-3 in
extracellular fluid, producing the volatile weak acid, CO2, which is expired by the
lungs. Intracellular buffering also defends the body fluids' alkalinity. Buffering of
fixed H+ compromises HCO-3 stores and ongoing renal transport processes are
needed to replenish HCO-3 and excrete H+.

Simple disturbances of H+ balance may be either metabolic or respiratory in


origin. Respiratory compensation for metabolic disturbances occurs almost
instantaneously, returning pH to near-normal. On the other hand, primary
respiratory disturbances are usually characterized by longer-lasting deviations in
pH because no respiratory compensation is possible. Both types of disturbances
require renal mechanisms for full restoration of acid-base balance.

VII. APPENDIX I. NORMAL VALUES RELEVANT TO ACID-BASE


PHYSIOLOGY

PNa 140 mEq/l


PK 4 mEq/l
PHCO3 24 mEq/l
PPO4 1 mm/l
Arterial pH 7.37 - 7.42 (7.40)
Arterial [H+] 40 x 10-9 Eq/l
Arterial PCO2 40 mmHg
Arterial [CO2] 1.2 mEq/l
Arterial [HCO-3] 24 mEq/l
pK': HCO-3/CO2 6.1
pK': HPO4-2/H2PO4- 6.8
pK': NH3/NH4+ 9.2

VIII. SAMPLE QUESTIONS FOR ACID-BASE PHYSIOLOGY, SET 1

950
Acidosis Alkalosis - Dr. Costanzo

1. The data below were obtained on each of four patients. Complete the
analysis of the acid-base status of each patient by filling in the blank
spaces.

Normal Values pH = 7.37 - 7.42


HCO3 = 24 mM
PCO2 = 40 mm Hg

Arterial Plasma
Cause of Disturbance Type of Disturbance
pH PCO2 HCO3
Prolonged Vomiting 7.55 44
Ingestion of HCl 28 10
Hysterical
7.57 21
Hyperventilation
Emphysema 7.33 68

2. The following laboratory tests are obtained from two patients. Would you
administer NaHCO3 to either one?

a.

Plasma Na = 140 mM Cl = 114 mM


K = 4.2 mM HCO3 = 16 mM

b.

Plasma Na = 140 mM Arterial pH = 7.32


K = 4.7 mM PCO2 = 14 mm Hg
Cl = 122 mM
HC03 = 7 mM

951
Acidosis Alkalosis - Dr. Costanzo

3. A 65 year old man has history of smoking and hypertension, which is


treated with a loop diuretic. The following arterial blood values were
obtained. What is the most likely acid-base disturbance?

pH = 7.48
PCO2 = 51 mm Hg
HCO3 = 36 mM

4. A 56 year old male was found lying in an alley, unresponsive and


breathing heavily. An empty can of antifreeze (ethylene glycol) was found
nearby. He was a known alcoholic. He was brought to the emergency
room comatose. The following blood values were obtained.

pH 6.90
PCO2 11 mmHg
HCO-3 2 mM
Na+ 142 mM
K+ 5 mM
Cl- 103 mM

a. What is the acid-base disorder?

b. What is the value for the anion gap? What interpretation could you
offer for the anion gap in this patient?

c. What is the take-home message?

952
Acidosis Alkalosis - Dr. Costanzo

5. A 78 year old woman was admitted to the hospital in a coma after


collapsing at home. She had a history of hypertension and had several
small strokes previously. On arrival at the hospital, she was unresponsive;
respirations were deep and regular. A CT scan showed an infarct in the
region of the pons. The blood values are given below.

On Admission One Week Later


pH 7.53 7.51
PCO2 25 mm Hg 17 mm Hg
HCO-3 20 mM 13 mM
PO2 95 mm Hg 105 mm Hg

a. Diagnose the acid-base disorder.

b. What physiological process is represented in the changes occurring in


the blood values between admission and one week later?

IX. ANSWERS TO ACID-BASE PROBLEM SET 1

1. All missing values should be calculated using the Henderson Hasselbalch


equation. Make sure that you are familiar with logs and anti-logs on your
calculator.

a. Prolonged vomiting

HC03- = 37 mM

pH indicates alkalosis. If respiratory alkalosis, then the CO2 would


be too low, rather than too high. Bicarbonate is elevated consistent
with metabolic alkalosis. CO2 is elevated because of respiratory
compensation.

b. Ingestion of HCl

pH = 7.18

pH indicates acidosis. If respiratory acidosis, then CO2 should be


high, not low. HC03 is very low due to buffering of added H+. CO2
is low because of respiratory compensation for the metabolic
acidosis.

953
Acidosis Alkalosis - Dr. Costanzo

c. Hysterical hyperventilation

PC02 = 24 mm Hg

pH is consistent with alkalosis. H+ and HC03- are both low because


of Law of Mass Action (driven by the low CO2 due to respiratory
alkalosis.) If it were metabolic alkalosis, then HC03 would be too
high due to decreased buffering of H+ and CO2 would be too high
because of respiratory compensation.

d. Emphysema

HC03 = 34 mM

pH is consistent with acidosis. Very high PCO2 is consistent with


respiratory acidosis.
H+ and HCO-3 are both high due to Law of Mass Action (driven by
the high CO2). If it were metabolic acidosis, then HCO-3 would be
low due to buffering and CO2 would be low due to respiratory
compensation.

2.
a. You should not administer HCO3- in this case. The arterial pH has
not been measured, so it cannot be certain that the patient has
metabolic acidosis. The low plasma HCO3- could be due to chronic
respiratory alkalosis. Wait until you have arterial pH and blood
gases.

b. You should administer HCO3- in this case. Even though pH is only


slightly reduced, this is being maintained by the profound
hyperventilation (see that pCO2 is 14 mm Hg). Administration of
HCO3- will correct the metabolic acidosis, thus decreasing the
stimulus for hyperventilation.

3. The diagnosis is metabolic alkalosis, probably secondary to the diuretic


administration (contraction alkalosis).The appropriate respiratory
compensation has taken place (hypoventilation) which raises the PCO2
and brings the pH in the near-normal range.

4.
a. The combination of a low HCO-3 and pH indicates metabolic
acidosis. The compensatory respiratory response has lowered the
PCO2 to an appropriately low level for severe metabolic acidosis.

954
Acidosis Alkalosis - Dr. Costanzo

b. The anion gap is 37 mEq/L. Ethylene glycol itself is not toxic but
metabolized to the weak acids glycolic acid, glyoxylic acid, oxalic
acid and formic acid; the salts of these acids contribute to the pool
of "unmeasured anions" and raise the anion gap. To confirm the
role of ethylene glycol and rule out other causes of metabolic
acidosis with elevated anion gap, ethylene glycol in the blood
should be measured; lactic acid (lactic acidosis) and ketones
(diabetic ketoacidosis) should also be measured. The severity of
the metabolic acidosis warrants immediate infusion of HCO-3 to
save the patient's life.

c. Don't drink antifreeze!!

5.
a. This patient represents an example of the transition from acute to
chronic respiratory respiratory alkalosis.

b. On admission, the HCO-3 was low due to the Law of Mass Action.
One week later, the HCO-3 was much lower due to renal
compensation, whereby the low PCO2 in renal cells provides less
H+ for secretion, thus less filtered HCO-3 can be reabsorbed,
lowering the blood HCO-3. This process brings the blood pH down
towards normal (compensation).

955
Acidosis Alkalosis - Dr. Costanzo

X. SAMPLE QUESTIONS FOR ACID-BASE PHYSIOLOGY, SET 2


(With annotated answers)

1. A buffer pair (HA/A-) has a pK of 5.4. At the blood pH of 7.4, the


concentration of HA is:

A. 1/100 that of A-
B. 1/10 that of A-
C. equal to that of A-
D. 2 times that of A-
E. 100 times that of A-

pH = pK + log A-/ HA
7.4 = 5.4 + log A-/HA
2.0 = log A-/HA
100 = A-/HA,
thus HA is 1/100 that of A-

Thus, Answer is A.

2. To maintain H+ balance, total daily excretion of fixed H+ should equal the


daily:

A. production of fixed acid


B. excretion of HCO3-
C. filtered load of HCO3-
D. excretion of titratable acid
E. filtered load of H+

A is the answer; by definition, we stay in H+ balance by excreting all the


fixed acid that we produce. B and C are nonsense answers, since H+
excretion is not related to HCO3- excretion or filtered load. D is incorrect
because H+ excretion is not just titratable acid but the sum of titratable
acid plus NH4+. E is incorrect because excreted H+ comes from the renal
cells, not the filtrate.

3. Which of the following is a feature of the reabsorption of filtered HCO3-?

A. Less than 80% of the filtered load is reabsorbed when plasma


concentration of HCO3- is 24 mEq/L.
B. It acidifies tubular fluid to pH 4.4.

956
Acidosis Alkalosis - Dr. Costanzo

C. It is directly linked to excretion of H+ as NH4+.


D. It is inhibited by decreases in arterial PCO2.
E. It can proceed normally in the presence of a renal carbonic
anhydrase inhibitor.

D is the correct answer since reabsorption of filtered HCO3- is dependent


on H+ secreted from the cells -- these H+ are generated in the cells from
CO2 and H2O; thus, a decrease in supply of CO2 to the cells will inhibit
HCO3- reabsorption.

A is wrong because 24 mEq/L is the normal concentration, thus normal


filtered load, where virtually 100% is reabsorbed. B is wrong because
reabsorption of filtered HCO3- does not acidify tubular fluid much since it
is not associated with net H+ secretion. C is just plain wrong. E is wrong
because both luminal and cellular carbonic anhydrase are necessary for
normal HCO3- reabsorption.

4. Excretion of H+ as NH4+ is:

A. higher in acute than chronic metabolic acidosis.


B. increased when the tubular fluid pH is increased.
C. increased in chronic renal failure.
D. normally of smaller magnitude than the excretion of H+ as
titratable acid.
E. depends on synthesis of NH3 in renal cells.

E is the simple correct answer since NH4+ excreted comes from NH3
generated in the renal cells, not the filtrate.

A is wrong because chronic acidosis shows an adaptive increase in


ammoniagenesis and so the higher rates of NH4+ excretion. B is wrong
because NH4+ excretion is increased when tubular fluid pH is decreased;
recall the shift towards NH4+ and away from NH3 (Henderson-
Hasselbalch), thus making a better gradient for NH3 diffusion. C is wrong
because the cause of metabolic acidosis in chronic renal failure is the
decreased ammoniagenesis in the renal tissue and inability to excrete H+.
D is wrong because it's the other way around.

5. Compared to acute respiratory acidosis, chronic respiratory acidosis is


characterized by (for the same values of arterial PCO2):

A. more complete respiratory compensation


B. lower values of arterial pH
C. lower values of arterial HCO3-

957
Acidosis Alkalosis - Dr. Costanzo

D. higher rates of HCO3- reabsorption

Answer is D.

A is wrong because respiratory disorders, whether acute or chronic, do not


have respiratory compensation. B is wrong because chronic respiratory
acidosis would have renal compensation which involves increased
reabsorption of filtered HCO3- and excretion of H+, thus raising the
arterial pH. C is wrong for the same reason that B is wrong -- renal
compensation will increase the arterial HCO3- by reabsorbing more. D is
the only correct statement, showing the correct renal compensation for
respiratory acidosis.

6. A patient arrives in the E.R. following 4 days of persistent vomiting after


eating contaminated shellfish. The patient has low blood pressure and
reduced tissue turgor. Arterial blood values were:

pH = 7.69
HCO3- = 55 mEq/L
PCO2 = 47 mm Hg

Which of the following would also be expected in this patient?

A. hypoventilation
B. decreased K+ secretion by distal tubule
C. exchange of intracellular H+ for extracellular K+
D. increased ratio of H2PO4-/HPO4-2 in urine

The first step is to analyze the acid base disorder. Alkaline pH says
alkalosis; high HCO3- results from mass action and high PCO2 is due to
respiratory compensation for metabolic alkalosis.

Other clues are the low blood pressure and turgor indicating ecf volume
contraction -- think of increased renin, increased Angio. II and increased
aldosterone.

A and C are the correct answers. A is correct because hypoventilation is


the respiratory compensation for metabolic alkalosis. C is correct because
alkalosis in blood will cause H+ to leave cells in exchange for K+ entering
cells; this is how cells help buffer the alkalosis.

B is wrong because in alkalosis H+ leaves cells (including renal distal


cells) and K+ enters cells; this creates a higher intracellular K+ and bigger
concentration gradient for K+ diffusion into the distal tubule lumen, thus
higher K+ secretion. Also the volume contraction leading to increased

958
Acidosis Alkalosis - Dr. Costanzo

aldosterone would also increase distal K+ secretion. D is wrong because in


alkalosis, there is decreased H+ secretion, thus a decrease in this ratio.

7. A patient has the following arterial blood values:

HCO3- = 15 mEq/L
PCO2 = 20 mm Hg

Which of the following would also be expected in this patient?

A. Arterial pH greater than 7.4


B. Hyperventilation
C. Decreased ionized Ca+2 in blood
D. Increased titratable acid excretion

The correct answers are A, B and C.

First, determine the acid base disorder. Without a pH value, this could be
metabolic acidosis with respiratory compensation or respiratory alkalosis.
Calculate pH with Henderson-Hasselbalch:

pH = 6.1 + log15
0.03 x 20
= 7.50

thus it is respiratory alkalosis

A is correct by the calculation. B is correct because hyperventilation is the


cause of respiratory alkalosis. C is correct because of exchange of H+ and
Ca+2 on plasma proteins; alkalosis means fewer H+ bound to protein, thus
more Ca+2 bound, and decreased free ionized Ca+2. D is wrong because
alkalosis does not increase H+ excretion ( either T.A. or NH+4).

8. A patient arrives in the emergency room with low blood pressure, reduced
tissue turgor and the following blood values:

HCO-3 = 57 mEq/L
PCO2 = 48 mm Hg

Assume: pK = 6.1
[CO2] = 0.03 x PCO2

Which of the following would also be expected in this patient?

A. Exchange of intracellular H+ for extracellular K+


B. Hyperventilation

959
Acidosis Alkalosis - Dr. Costanzo

C. Arterial pH less than 7.4


D. Increased ratio of H2PO-4/HPO-24 in urine
E. Decreased K+ secretion by distal tubules

First analyze the acid-base disorder. One must calculate the pH, since high
bicarbonate and high PCO2 could mean either respiratory acidosis or
metabolic alkalosis with respiratory compensation. Using Henderson-
Hasselbalch, the pH is calculated as 7.70, so the diagnosis is metabolic
alkalosis.

The correct answer is A because in alkalosis, H+ will leave the cells in


exchange for K+ entering so that the cells can participate in buffering the
deficit in fixed acid. B is wrong because the respiratory compensation for
metabolic alkalosis is hypoventilation. C is wrong by the H-H calculation.
D is wrong because increased fixed acid excretion is the response to
acidosis, not alkalosis. E is wrong because in alkalosis H+ leaves renal
distal cells and K+ enters, thus increasing intracellular K+ and increasing
the driving force for K+ secretion.

9. Reabsorption of filtered HCO-3:

A. is less than 80% of the filtered load at normal plasma concentrations.


B. lowers tubular fluid pH to 4.4.
C. involves conversion of CO2 and water to H2CO3 in the tubule lumen.
D. when inhibited, causes metabolic acidosis.
E. is inhibited by extracellular fluid volume contraction.

The correct answer is D because inhibition of reabsorption of filtered


bicarbonate results in its loss in urine, thus leaving unbuffered H+ in blood
(fixed acid). A is wrong because normally virtually 100% is reabsorbed. B
is wrong because only the processes which acidify the urine (TA and
NH+4) can lower urine pH to these values. C is wrong because the
reactions in the lumen go the other way. E is wrong because ECF volume
contraction causes an increase in reabsorption of filtered bicarbonate.

10. A patient had the following arterial blood values for three weeks:

pH = 7.32
HCO-3 = 30 mEq/L
PCO2 = 60 mm Hg

Which of the following would also be expected in this patient?

A. Buffering of H+ by extracellular HCO-3.


B. Alkalinization of intracellular fluid.

960
Acidosis Alkalosis - Dr. Costanzo

C. Compensatory hyperventilation.
D. Decreased HCO-3 reabsorption in the kidney.
E. Increased renal ammoniagenesis.

As always, make the diagnosis first -- according to the pH it's acidosis.


Then use mass action to conclude respiratory acidosis caused by
hypoventilation which increases PCO2, drives the reactions to the right,
increasing H+ and HCO-3.

The correct answer is E since in acidosis (chronic, implied by three


weeks) the kidney adapts to excrete more H+ by increasing ammonia
production. A is wrong because bicarbonate does not buffer H+ in
respiratory disturbances. B is wrong because in any kind of acidosis the
cells are acidified. C is wrong because there is no respiratory
compensation for respiratory disturbances. D is wrong because the
increased PCO2 in respiratory acidosis provides more H+ in the renal cells
to be secreted and so more bicarbonate can be reabsorbed.

11. Which of the following is/are associated with an increased value of arterial
[HCO-3]?

A. Vomiting
B. Morphine overdose causing respiratory depression
C. Chronic obstructive pulmonary disease
D. Lactic acidosis

Think of each etiology and what acid/base disorder it would cause.


Vomiting = metabolic alkalosis. Morphine = respiratory acidosis.
COPD = respiratory acidosis. Lactic acidosis = metabolic acidosis.
Increased blood bicarbonate is seen in respiratory acidosis because of
mass action and metabolic alkalosis due to loss of fixed H+.

Thus, the correct answers are A, B and C.

12. A woman with severe diarrhea lasting 6 days was evaluated in the
emergency room. On physical exam, she had dry mucous membranes,
decreased tissue turgor and postural hypotension. The following blood
values were obtained:

pH = 7.22
[CO2] = 1.0 mM
[Na+] = 132 mEq/L
[K+] = 2.2 mEq/L
[Cl-] = 111 mEq/L

961
Acidosis Alkalosis - Dr. Costanzo

pK = 6.1

Which of the following is most likely correct about her condition in the
emergency room?

A. She is hypoventilating.
B. Her heart rate is decreased.
C. Her arterial [HCO3-] is increased.
D. Her serum anion gap is elevated.
E. Her circulating levels of Angiotensin II are elevated.

First, calculate the [HCO-3] since it will be needed for choices C and D; by
Henderson-Hasselbalch it is 13.2 mEq/L (practice!). Then make the
acid/base diagnosis -- acidosis because of low pH; metabolic acidosis
because HCO-3 is low from buffering fixed H+ and CO2 is low because of
the respiratory compensation (hyperventilation).

The correct answer is E. The history of severe diarrhea with evidence of


volume contraction should lead you to conclude that the renin-angiotensin
II-aldosterone axis is activated (decreased ECF volume, decreased renal
perfusion pressure, increased renin, increased Angiotensin II, increased
aldosterone). A is wrong because the respiratory compensation for
metabolic acidosis is hyperventilation. B is wrong because the low blood
pressure would decrease barorecptor firing rate in the carotid sinus which
would increase sympathetic outflow and decrease parasympathetic
outflow; together, these would increase heart rate rather than decrease it. C
is wrong by the calculation of serum HCO-3. D is wrong by calculation of
anion gap; anion gap = Na+ - HCO-3 - Cl- = 132-111-13.2 = 7.8
(low). In fact, this is metabolic acidosis with normal anion gap or
hyperchloremic metabolic acidosis, where the decreased HCO-3 is
replaced with Cl-; since both are "measured" anions, the anion gap is
normal.

13. (Use same case as #12.)

Which of the following contributes to her hypokalemia?

A. Metabolic alkalosis
B. Exchange of extracellular H+ for intracellular K+
C. Increased K+ secretion by the distal tubule
D. Decreased circulating levels of aldosterone
E. Decreased circulating levels of ADH

The correct answer is C -- because of volume contraction, aldosterone is


stimulated which in turn causes increased secretion of K+ by distal tubule,

962
Acidosis Alkalosis - Dr. Costanzo

leading to hypokalemia. A is wrong because while metabolic alkalosis


might cause hypokalema (by H-K exchange), it does not contribute in this
person since she does not have metabolic alkalosis! B is wrong because
exchange of extracellular H+ for intracellular K+ could occur in metabolic
acidosis, but would cause hyperkalemia, not hypokalemia. D is wrong
because aldosterone is increased with volume contraction. E is wrong
because if anything, ADH would be increased with volume contraction;
moreover, it has nothing to do with blood K+ concentrations.

963
Respiratory Pathophysi Cases - Dr. Costanzo

Respiratory Pathophysiology Cases


Linda Costanzo Ph.D.

I. Case of Pulmonary Fibrosis

Susan was diagnosed 3 years ago with diffuse interstitial pulmonary fibrosis. She
tries to continue normal activities, although it has become increasingly difficult.
She tires easily and can no longer climb a flight of stairs without becoming short
of breath. She is being closely followed by her pulmonologist. At a recent
physical examination, her arterial blood gases were:

Arterial Blood Gases


PaO2 76 mm Hg (Normal, 100mm Hg)
PaCO2 37 mm Hg ( Normal, 40mm Hg)
% hemoglobin saturation 95% (Normal, 95-100%)

1. Susan had a decreased DLCO. What does this mean?

2. Diffuse interstitial fibrosis is characterized by thickening of alveolar


membranes. Use this information to explain the decreased arterial PO2.

3. What was the total O2 content her blood? Assume that the O2-binding capacity
of her blood was 1.34 ml O2/g hemoglobin, that her hemoglobin concentration
was 15 g/100 ml, and that the solubility of O2 in blood is 0.003 ml O2/100 ml
blood/mm Hg.

4. Susan was hypoxemic (i.e., she had a decreased PaO2). However, she was not
hypercapnic (i.e., she did not have CO2 retention or an increased PaCO2); in
fact, her PaCO2 was slightly below normal at 37 mm Hg. How can hypoxemia
occur in the absence of hypercapnia?

Answers and Explanations

1. Lung diffusing capacity (DL) is measured with carbon monoxide (CO). In the
single-breath method, a subject maximally inspires air containing CO, holds her
breath for ten seconds, then expires. The amount of CO transferred from alveolar
gas into pulmonary capillary blood is measured to assess the diffusion
characteristics of the alveolar-pulmonary capillary barrier.

Susan’s DLCO was decreased because of thickening of the alveolar walls. Such
thickening increases the diffusion distance for gases such as CO (and O2),

964
Respiratory Pathophysi Cases - Dr. Costanzo

diminishing the total amount of gas that can be transferred across the alveolar
wall.

2. Susan’s PaO2 was 76 mm Hg, which is lower than the normal value of 100 mm
Hg. In normal lungs, there is equilibration of O2 across the alveolar-pulmonary
capillary barrier, such that the PO2 of the blood become equal to the PO2 of
alveolar gas, or approximately 100 mm Hg. In Susan, equilibration of O2 was
impossible: thickening of her alveolar walls impaired O2 diffusion (as detected in
a decreased DlCO) and PaO2 could not become equal to PAO2.

3. The total O2 content of blood has two components — free dissolved O2 and O2-
hemoglobin. By now you know that O2-hemoglobin is, by far, the greater
contributor to total O2 content. However, to be thorough, calculate both dissolved
O2 and bound O2.

4. Although Susan has a problem with O2 exchange and she is hypoxemic, she does
not have a problem with CO2 exchange (i.e., she is not hypercapnic). In fact, her
PaCO2 was slightly lower than the normal value of 40 mm Hg. This pattern is seen
commonly in respiratory diseases: hypoxemia occurs without hypercapnia. But,
why?

Consider the sequence of events that created this pattern of arterial blood gases.
The fibrotic disease affected some, but not all, regions of her lungs. In the
diseased regions, there was thickening of alveolar walls and thickening of the
diffusion barrier for both O2 and CO2. The diffusion problem caused hypoxemia
(decreased PaO2) and may have briefly caused hypercapnia (increased PaCO2).
However, because the central chemoreceptors are exquisitely sensitive to small
changes in PCO2, they responded to the hypercapnia by increasing the ventilation
rate. The increase in alveolar ventilation in healthy regions of the lungs ridded the
body of excess CO2 that was retained in unhealthy regions. In other words, by

965
Respiratory Pathophysi Cases - Dr. Costanzo

increasing alveolar ventilation, healthy regions of the lungs compensated for


unhealthy regions with respect to CO2. As a result, PCO2 returned to normal. (Later
in her disease, she may develop hypercapnia if there is not enough healthy lung
tissue to compensate for the unhealthy tissue, or if the work of breathing
becomes so great that she is unable to sufficiently increase her alveolar
ventilation.)

You may ask: If increased alveolar ventilation can rid the body of excess CO2
retained in unhealthy regions of the lungs, why can’t increased alveolar
ventilation also correct the hypoxemia? The answer lies in the fact that well-
oxygenated blood from healthy lung regions is always mixing with, and being
diluted by, poorly-oxygenated blood from unhealthy regions, and PaO2 will
always be lower than normal. Remember: O2 added in healthy lung regions is
largely in the form of dissolved O2 (because hemoglobin will already be 100%
saturated in those regions) and this dissolved O2 does little to improve O2 content
or delivery.

Is possible for the degree of hyperventilation to be so great that the person


actually becomes hypocapnic (has decreased PaCO2). Yes, in fact, Susan’s PaCO2 is
slightly less than normal. If the PaO2 becomes low enough to stimulate the
peripheral chemoreceptors (i.e., PaO2 < 60 mm Hg), hyperventilation will be
strongly stimulated, even greater amounts of CO2 will be expired by healthy lung
regions, and the PaCO2 will fall below the normal value of 40 mm Hg.

In summary, persons with lung disease may pass through three stages of arterial
blood gases: (1) mild hypoxemia with normocapnia; (2) more severe hypoxemia
(PaO2 < 60 mm Hg) with hypocapnia, resulting in respiratory alkalosis; and (3)
severe hypoxemia with hypercapnia, resulting in respiratory acidosis. Susan is
somewhere between the first and the second stages.

II. Case of chronic obstructive pulmonary disease (COPD)

Clarence has smoked 4 packs a day for 45 years and has no intention of quitting.
He fatigues easily, is short of breath, and sleeps on two pillows. In the physician’s
office, he had a prolonged expiratory phase, expiratory wheezes, increased chest
(AP) diameter, and was cyanotic. The long history of smoking had caused chronic
obstructive pulmonary disease. The following were results of pulmonary function
tests and blood values:

Vital Capacity decreased


FRC increased
FEV1 decreased
Hemoglobin 14.5 g/dl (normal)
PaO2 48 mm Hg
PaCO2 69 mm Hg
HCO3- 34 mEq/L

966
Respiratory Pathophysi Cases - Dr. Costanzo

1. Why was Clarence’s FEV1 decreased? Why was his FRC increased? Why was his
vital capacity decreased?

2. Why was his AP diameter increased?

3. Why did he have decreased PaO2? Why was he cyanotic?

4. Why did he have increased PaCO2?

5. What was his arterial pH, and what acid-base disorder does he have?

6. Why was his HCO3- concentration increased?

Answers and Explanations

1. COPD, an obstructive disease, results in decreases in airflow especially during


expiration. Because of the loss of elastic tissue, there is increased compliance of
lung tissue. Normally, expiration is passive, driven by elastic recoil forces. With
loss of elasticity, expiration is impaired. Also, airways are normally kept open by
radial traction, which also depends on elasticity. With loss of elasticity, the
airways may collapse, producing extra resistance to airflow, especially during
expiration. For these two reasons, FEV1 is decreased, and there was a prolonged
expiratory phase and expiratory wheezes. Air that should have been expired is
trapped in the lungs and increases residual volume and FRC. Vital capacity is
decreased because the residual volume is increased (i.e., the volume that can be
inspired above residual volume is compromised).

2. AP diameter was increased (barrel-shaped chest) because of air trapping and


increased residual volume and FRC.

3. He was hypoxemic (decreased PaO2) because of a V/Q defect caused by impaired


ventilation. Regions of the lungs were perfused but not ventilated, resulting in
decreased V/Q and even shunt. Blood perfusing those regions was not oxygenated
in the lungs and mixed with oxygenated blood from well-ventilated regions. The
overall PO2 of blood leaving the lungs was thereby decreased. He was cyanotic
because there was an increased concentration of deoxyhemoglobin in blood;
deoxyhemoglobin is blue (whereas oxyhemoglobin is red).

967
Respiratory Pathophysi Cases - Dr. Costanzo

4. PaCO2 was increased because of decreased alveolar ventilation. All of the CO2
produced by his tissues could not be expired, leading to CO2 retention.

5. Clarence had respiratory acidosis secondary to CO2 retention.

6. The HCO3- concentration was increased as the renal compensation for chronic
respiratory acidosis (aided by the increased PCO2). There is increased reabsorption
of HCO3- by the kidneys, which increases the HCO3- concentration in blood
tending to normalize the ratio of HCO3-/ CO2 and to normalize the pH. Thus, his
pH was only slightly acidic, even though is PCO2 was markedly elevated.

968
Respiratory Pathophysi Cases - Dr. Costanzo

III. Useful Abbreviations and Values for Respiratory Gases

P pressure, or partial pressure (mm Hg)


PB barometric pressure
PH2O water vapor pressure
Q blood flow (L/min)
V gas flow (L/min)
F fractional concentration (no units)
A alveolar gas
A arterial blood
V venous blood
E expired gas
I inspired gas
DL lung diffusing capacity
% sat % saturation of hemoglobin
respiratory exchange quotient (CO2 production/O2
R
consumption

Normal Value
PaO2 100 mm Hg
PaCO2 40 mm Hg
PvO2 40 mm Hg
PvCO2 46 mm Hg
PIO2 160 mm Hg (dry)
PIO2 150 mm Hg (humidified)
PICO2 0
PIN2 600 mm Hg (dry)
PIN2 563 mm Hg (humidified)
PAO2 100 mm Hg
PACO2 40 mm Hg
PB 760 mm Hg (sea level)
PH2O 47 mm Hg
VO2 250 ml/min
VCO2 200 ml/min
R or R.Q. 0.8
Solubility of O2 in blood 0.003 ml O2/100 ml blood/mm Hg
Solubility of CO2 in blood 0.07 ml CO2/100 ml blood/mm Hg
Hemoglobin concentration 15 g/100 ml
O2-binding capacity 1.34 ml O2/g hemoglobin

969
Acid Base Problem Solving - Dr. Costanzo

ACID-BASE PHYSIOLOGY PROBLEM SOLVING


Dr. Linda Costanzo

I. A 40-y.o. woman developed severe "traveler's diarrhea" while on vacation in Mexico.


Despite attempts to control the diarrhea with over-the-counter medications, she continued
to have watery stools at a frequency of 8 -10 per day. She became progressively weaker
and on the third day was taken to the emergency room.

On physical examination, her eyes were sunken, mucous membranes dry and she had
reduced tissue turgor. Her blood pressure was 90/60 mm Hg while supine and was 60/40
when standing. Pulse rate was 120/minute and respirations were deep and rapid
(24/minute).

The results of laboratory tests were as follows:

Arterial blood

pH 7.25
pCO2 24 mm Hg

Venous serum

[Na+] 132 mEq/L


[K+] 2.3 mEq/L
[Cl-] 111 mEq/L

A. What is the woman's acid-base disorder and what is the likely cause?

B. What explanation can be offered for the increased depth and frequency of respiration?

C. Calculate the value for the anion gap and explain its significance in this particular
acid-base disorder?

D. What explanations can be offered for the postural hypotension and increased pulse
rate? What changes would be anticipated in circulating levels of renin, angiotensin II
and aldosterone?

E. Why is the serum [K+] severely depressed?

970
Acid Base Problem Solving - Dr. Costanzo

F. What treatment would you suggest to correct the acid-base and fluid/electrolyte
disorders?

II.A 34-year old woman known to have Type 1 diabetes mellitus for 18 years was examined by
her physician every other month. Her disease wasin good control with insulin injections and
appropriate dietary measures. Two days before admission to the hospital, the woman developed a
viral illness; she experienced chills and fever, muscle aches, frequent urination, and loss of
appetite. On the day of admission to the hospital she started vomiting and was short of breath.

Physical examination on admission revealed an acutely ill woman whose mucous membranes
were dry. She was breathing deeply (Kussmaul respiration). Her temperature was 38.6o C, her
pulse was 100/minute, and her respiratory rate was 24/minute. A urine sample contained large
amounts of sugar and ketones. The results of blood tests are shown below:

Arterial blood

pH 7.15
PCO2 13 mm Hg
HCO-3 4 mEq/L

Venous blood

Na+ 130 mEq/L


Cl- 94 mEq/L
K+ 5.5 mEq/L
Glucose 730 mg/dL

A. What is the woman's acid-base disorder and what is the likely cause? What is
responsible for the increased breathing rate?

B. Calculate the anion gap and explain its significance in this disorder.

C. What explanation can be offered for the dry mucous membranes?

971
Acid Base Problem Solving - Dr. Costanzo

D. What explanation/s can be offered for the elevated serum K+ concentration?

E. What explanation can be offered for the somewhat depressed serum Na+
concentration?

F. What treatment would you suggest for the acid/base and fluid/electrolyte disorder?

972
Endocrinology - Dr. Kalimi

Endocrinology - Reproduction – Introduction


Mohammed Y. Kalimi, Ph.D.

Objectives:
After studying this material, the student will:

1. Identify the chemical nature of thyroid hormones, TRH, TSH, GH, somatoatatin,
prolactin, insulin, glucagon and growth factors.
2. Recognize the concept of hormonal biosynthesis, storage, release, plasma half
life, degradation and control mechanisms.
3. Understand the physiological effects and mechanisms of hormonal actions.
4. Understand the concept of endocrine diseases in terms of hyper (over secretion)
and hypo (deficiency) functioning of the endocrine glands.

I. Basic Concepts

Endocrine Gland:

• A ductless gland whose secretions (hormones) are released into the vascular
system for an action on distant cells (target cells).

Hormones:

• Chemical agents synthesized by ductless glands and released into the blood
stream where they evoke a physiological response by acting on specific
tissues/organs by way of specific receptors.

Paracrine secretion:

• Secretory product is released into intercellular space to influence immediately


neighboring cells.

Neurocrine:

• A hormone secreted by a nerve cell, released from nerve endings into the blood
stream, and carried to another area where it exerts specific effects on target cells.
• The chemical substance released from nerve endings which exert effects on the
adjacent nerve cells across a short synapse is called a neurotransmitter.

Autocrine:

• Secretory product (hormone) acts back on the cell of origin or adjacent identical
cells.

973
Endocrinology - Dr. Kalimi

Target organ:

• The organ or cells on which a particular hormone elicits a response. May be


another endocrine organ.

Feedback:

• The response of a particular target organ may feedback to an endocrine gland and
may modify the output of that gland. Feedback may be either stimulatory
(positive) or inhibitory (negative).

II. Hormones

A. Classification of Hormones:

The chemical nature of hormones varies widely but three major categories are
recognized.

1. Peptide hormone: Ranging from simple dipeptides (two amino acids) to large
proteins containing over 200 amino acids.

• Examples: Hypothalamic, anterior pituitary, posterior pituitary and


pancreatic hormones.

2. Biogenic amines - Amino acid derivatives.

• Examples: Thyroid hormones and catecholamines (both derived from


modification of the amino acid tyrosine).

3. Steroid hormones: A large family of molecules based on the 17 carbon


aromatic cyclopentanoperhydrophenanthrene nucleus, among which relatively
minor chemical differences are associated with marked differences in
biological activity.

• Examples: Adrenal cortex, reproductive gland hormones and the active


metabolites of vitamin D.

B. Nomenclature and acronyms:

Hormones are named for either their gland of origin (thyroxine, testosterone),
function (calcitonin, progesterone, prolactin), or chemical structure
(triiodothyronine, aldosterone).

Abbreviations are often used in preference to the complete name for a hormone
(follicle-stimulating hormone = FSH; luteinizing hormone releasing hormone =
LHRH ; parathyroid hormone = PTH; estradiol = E2).

974
Endocrinology - Dr. Kalimi

C. Biosynthesis of hormones:

1. Biosynthesis of peptide hormones:

The peptide hormones are synthesized by rough endoplasmic reticulum as a


pre-pro-hormone.

2. Biosynthesis of steroid and amine hormones:

The steroid and amine hormones are synthesized from cholesterol and tyrosine
respectively through a series of enzymatic reactions by smooth endoplasmic
reticulum, and mitochondria.

D. Storage of hormones:

In comparison with exocrine glands, storage is minimal with the exception of the
thyroid gland.

E. Release of hormones:

Excitation-secretion coupling and release

↑ Intracellular calcium
↑ cAMP
↑ Activation of microtubular and microfilament system
↑ Fusion of membrane of the secretory granule with that of the cell
↑ Secretion of hormone by exocytosis

1. Hormone concentration in the various effluents from a particular endocrine


gland should exceed that of arterial blood supplying the gland.
2. Secretion or production rate of a hormone, nmoles/minute, ng/minute.
3. Plasma concentration of a hormone, nmoles/mL, ng/mL.

F. Transport of hormones in blood:

1. Bound to carrier proteins: Steroid and thyroid hormones circulate bound to


specific globulins. In general there is a good correlation between the amount
of bound hormone in circulation and the plasma half-life of a hormone; the
more bound the longer the half-life.
2. Unbound or free: With few exceptions (IGF-I), peptides and protein hormones
circulate unbound (also catecholamines).

G. Clearance of hormones from the circulation:

1. Biological half life (t ½)

975
Endocrinology - Dr. Kalimi

2. Metabolic clearance rate (MCR)

H. Inactivation of hormones:

1. By specific target tissues (internalization and lysosomal degradation).


2. By liver and kidneys.
3. By both 1 and 2.

I. Measurement of hormones:

1. Radioimmunoassay
2. Localization of hormones in tissues of origin and action:
Immunocytochemistry.

J. Control of hormonal secretion:

1. Feedback (positive or negative)


2. Integration between endocrine and nervous system
3. Neural control (dopaminergic, adrenergic, cholinergic etc.).
4. Others such as sleep-wake cycle, menstrual cycle, diurnal rhythm.

K. Mechanism of action (covered separately):

L. Major function of hormones:

1. Endocrine system helps initiate, mediate and regulate the processes of growth,
differentiation, development, maturation and senescence.
2. Maintenance of homeostasis, fluid and electrolyte balance (Na+, K+, Ca++,
glucose, water )
3. Regulation of cellular metabolism (fats, carbohydrates, proteins)
4. Sexual development and function, lactation and behavior.

M. Malfunctioning of the endocrine system:

Primarily caused by:

1. Overproduction of a hormone (hyperfunction)


2. Underproduction of a hormone (hypofunction)
3. Unresponsiveness of target organ (lack of receptor, etc.) = down-regulation
4. Production of abnormal hormone

N. Neuroendocrinology:

Control systems of the body

976
Endocrinology - Dr. Kalimi

III. The Nervous System and the Endocrine System

A. Similarities between the two:

1. Each synthesizes and releases specific chemical agents which are capable of
influencing other cells by interacting with specific receptors.
2. Both neurons and endocrine cells generate electrical potentials and can be
depolarized .

B. Differences between the two:

1. Nervous system:

a. Specific chemical agents released are disseminated only a very short


distance.
b. System is fast acting.
c. Actions are relatively short-lived.
d. Operates with point to point precision.
e. Affects only glandular secretions and muscular contractions.

2. Endocrine system:

a. Specific chemical agents are released and carried via the blood stream
throughout the whole body.
b. System is slow acting.
c. Actions are relatively long-lived.
d. Theoretically, has the potential of affecting every cell in the body.
e. Affects a whole variety of cell types.

C. More generalized definition is descriptive of cells that release a hormone in


response to a neural stimulation.

1. Adrenal medulla under the influence of the sympathetic nervous system.


2. Some cells of the islets of Langerhans are affected in part by the autonomic
nervous system.
3. Some cells in the CNS synthesize "hormones" that act as neurotransmitters.

977
Endocrinology - Dr. Kalimi

Figure 1

IV. Hormones Origin

A. Hypothalamus:

Thyroid-stimulating-hormone-releasing-hormone (TRH)
Corticotrophin-releasing-hormone (CRH)
Luteinizing-hormone-releasing-hormone (LHRH)
Growth-hormone-releasing-hormone (GHRH)
Somatostatin
Dopamine

978
Endocrinology - Dr. Kalimi

B. Anterior Pituitary:

Growth Hormone (GH)


Prolactin
Thyroid-Stimulating-Hormone (TSH)
Adrenocorticotrophic-Hormone (ACTH)
Luteinizing Hormone (LH)
Follicle-Stimulating-Hormone (FSH)
Melanocyte- Stimulating-Hormone (MSH)

C. Posterior Pituitary:

Vasopressin or Antidiuretic Hormone (ADH)


Oxytocin

D. Thyroid Gland:

Thyroid Hormones (T3 and T4) : Follicular cells


Calcitonin: Parafollicular cells

E. Parathyroid gland:

Parathyroid Hormone

F. Adrenal Cortex:

Glucocorticoids
Aldosterone

G. Adrenal Medulla:

Epinephrine and Norepinephrine

H. Pancreas:

Insulin
Glucagon
Somatostatin

I. Gonads (testes-male; ovary-female)

Androgens

Estrogens

Progestins

979
Endocrinology - Dr. Kalimi

J. Skin, Liver, and Kidney:

Vitamin D

K. Placenta:

Human Chorionic Gonadotropin (hCG)


Human Placental Lactogen (hPL)

V. Mechanism of Hormone Action

A. Mechanism of Steroid Hormone Action (Fig. 2):

Used by steroid hormones such as estrogens, androgens, progesterone,


aldosterone, glucocorticoids and vitamin D. In addition, thyroid hormones and
vitamin A have a similar mechanism. The estrogen, progesterone, androgen,
vitamin D and vitamin A receptors are primarily localized in the cell nucleus.

Steps:

1. Endocrine gland + stimulus → ↑ steroid release.


2. Steroid diffuses through the cell membrane into the cytoplasm of target cells.
3. Steroid binds to cytoplasmic and /or nuclear receptors forming a steroid-
receptor complex.

Steroid binding results in conformational changes called activation. The


subsequent dimerization of the liganded receptor enables the steroid-receptor
complex dimer to bind tightly to specific DNA sequences, called Steroid
Responsive Elements(SRE), and interact with coregulator proteins thus activating
or suppressing transcription (mRNA) of genes under the control of steroid
hormones.

980
Endocrinology - Dr. Kalimi

Figure 2.

981
Endocrinology - Dr. Kalimi

Figure 3.

In general, agonist ligands of receptors promote binding of coactivator proteins


that promote transcription initiation while binding of antagonists promotes
interaction with corepressor proteins that facilitate transcription repression.The
modular nature of receptors allows ligand, tissue and promoter specific interaction
with select subsets of coregulators capable of elaborating distinct transcriptional
and hence physiological responses to steroid signal.

4. mRNA is translated into specific proteins (such as metabolic enzymes).


5. ↑ specific proteins, ↑ physiological responses.

B. Cyclic AMP (cAMP) Mechanism (Fig. 3):

The cAMP mechanism appears to be used by most (not all) peptide hormones
(such as LH, FSH, TSH, ACTH, ADH via V2 receptor, hCG, MSH, GHRH, CRH,
catecholamines [ß1, ß2 receptors], calcitonin, glucagon and PTH).

982
Endocrinology - Dr. Kalimi

Steps:

1. Endocrine gland + stimulus ↑Release of Hormone (1st messenger).

2. Hormone + target cell membrane receptor → ↑ Hormone - Receptor


Complex.
Peptide hormone receptors are membrane bound, mobile, glycosylated, large
peptides.

3. ↑ Hormone-Receptor complex, ↑ activation of G-protein, ↑ adenylyl cyclase


activity.

Hormone binding to receptor releases guanine diphosphate (GDP) from a binding


site on the heterotrimeric G-protein permitting guanine triphosphate (GTP) to
bind to the G- protein (GDP-GTP exchange). Heterotrimeric G-proteins are made
up of three subunits α, β and γ, in order of decreasing mass. The binding of GTP
to α subunit results in dissociation of Gα from βγ.

The GTP bound - Gα-protein interacts with the adenylyl cyclase catalytic unit.
This results in the activation of enzyme adenylyl cyclase, and production of
cAMP from intracellular ATP.

ATP + adenylyl cyclase → ↑ cAMP + Pi

The Gα-subunit serves as a coupling protein between receptor and adenyl cyclase,
facilitating transmission of the hormonal signal.

There are two types of G-proteins (i) stimulatory (Gs) and (ii) inhibitory (Gi).

cAMP inhibitory hormones (somatostatin, dopamine) bind to Gi-protein and


suppress adenylyl cyclase activity while cAMP dependent hormones (LH, FSH,
TSH, ACTH, PTH, ADH via V2 receptor, GHRH, CRH, MSH, calcitonin,
glucagon, epinephrine, hCG ) bind to Gs-protein and activate adenylyl cyclase.

4. ATP + adenylyl cyclase → ↑ cAMP + Pi

5. Specific protein kinase(s) are activated by cAMP


Protein Kinase (inactive)+ cAMP → Protein Kinase (active)

6. The activated protein kinases catalyze phosphorylation of enzymes by ATP


resulting in physiological responses.

Protein +ATP + Activated protein kinase = Phosphoprotein + ADP

983
Endocrinology - Dr. Kalimi

Inactivation of cAMP and phosphoproteins

1. cAMP is inactivated by the enzyme phosphodiesterase


cAMP + phosphodiestrase → 5' AMP (inactive).

2. Phosphoproteins are inactivated by the enzyme phosphatase


Phosphoprotein + phosphatase = Protein + Pi

C. The Calcium-phospholipid mechanism (Fig.4):

Figure 4: Calcium-Phospholipid Pathway


The calcium-phospholipid mediated mechanism is used by GnRH (LHRH), TRH,
Angiotensin II, and ADH via the V1 receptor.

1. Hormone + membrane receptor, ↑ Hormone - receptor complex.


Formation of high affinity hormone - receptor complex results in transmission
of the signal through a G-protein (Gq) to the enzyme phospholipase c'.
2. Activation of the cell membrane enzyme phospholipase 'c.'
3. Conversion of plasma membrane phospholipid, phosphatidyl inositol 4,5,
biphosphate(PIP2) by activated enzyme phospholipase 'c' to inositol 1,4 5
triphosphate (IP3) and 1,2 diacylglycerol .

Inositol - 1,4,5 triphosphate (IP3) enhances intracellular calcium and calcium-


calmodulin processes.

1,2 diacylglycerol activates the enzyme protein kinase 'c' (PKC) which catalyzes
the phosphorylation of proteins by ATP.

984
Endocrinology - Dr. Kalimi

Figure 5: Internalization of Hormones

The overall effects of protein kinase 'c' activation and elevated calcium ion
concentrations include the opening and closing of ion channels, and increased
gene transcription by phosphorylating gene regulatory proteins directly or
indirectly by cascade.

It is quite common for hormone actions (TSH, GHRH, etc.) to depend upon the
cAMP mechanism together with the phospholipid - calcium mechanism (cross-
talk).

D. Internalization of Peptide Hormones (Fig.5).


Many peptide hormones are known to be internalized.

Steps of internalization mechanism:

1. Endocrine gland + stimulus ↑ hormone


2. Hormone binds to cell membrane receptor
3. Hormone-receptor complexes are clustered on the membrane
4. Membrane containing aggregated hormone-receptor complexes begin to fold
inward forming a coated pit.
5. Coated pits are internalized into the cell forming endocytic vesicles called
endosomes.

985
Endocrinology - Dr. Kalimi

6. Endosomes (by an ATP-dependent process) may facilitate the release of


ligand from receptor. Ligand and receptor are sorted, and the ligand is
degraded by lysosomal enzymes (hydrolases) . The internalized receptor may
be recycled to the cell surface or degraded by lysosomal enzymes.

VI. Hormone Action: Summary

A. Steroid hormone (estrogen, androgen, progesterone, aldosterone,


glucocorticoids, thyroid hormones, vitamin D, vitamin A)

Mechanism:

Binding of steroid to the cytoplasmic and/or nuclear receptors, dimerization of


the hormone-receptor complexes, transcription (mRNA synthesis),translation
(enzyme or protein synthesis), and physiological responses.

B. cAMP mechanism:

1. ↑ activation of adenylyl cyclase , ↑ cAMP, ↑ activation of phosphokinaseA

LH, FSH, TSH, ACTH, hCG, PTH, Calcitonin, Glucagon, CRH, GHRH,
Epinephrine, ADH via V2 receptor.

2. ↓adenylyl cyclase, ↓ cAMP

Dopamine, Somatostatin

C. Calcium-phospholipid mechanism:

↑ activation of phospholipase C, ↑ conversion of PIP2 to IP3 and diacylglycerol.

1. ↑ IP3, ↑ intracellular calcium,


2. ↑ diacylglycerol, ↑ PKC.

LHRH, TRH, Angiotensin II, ADH via V1 receptor.

D. Associated tyrosine kinase-linked hormone action: Prolactin, GH

E. Intrinsic tyrosine kinase-linked hormone action: Insulin, IGF-I

986
Thyroid Hormones 1, 2, & 3 - Dr. Kalimi

Thyroid Hormones 1, 2, & 3


Mohammed Y. Kalimi, Ph.D.

Thyroid hormones are iodinated derivatives of tyrosine (Fig.1).

Figure 1

I. Iodide (or iodine) turnover (fig 2):

Because thyroid hormones are iodinated molecules, their production is closely


related to iodide turnover. The following diagram represents the daily exchange of
iodide and peptide-bound iodine in a euthyroid individual on a 500 :g iodide
uptake. In this country daily iodide uptake (mainly as sodium and potassium
iodide) may easily range from 200-500 :g/day. Minimum daily requirement in
the adult is about 150 :g. Over this range, a healthy individual can compensate by
modulating renal iodide excretion so that other intercompartmental fluxes, and
thus thyroid hormone output remain constant. Beyond this range, however,
disease can result from too little, or too much, dietary iodide.

987
Thyroid Hormones 1, 2, & 3 - Dr. Kalimi

Figure 2: Iodine Metabolism

Daily iodide balance under equilibrium conditions:

500 :g dietary intake (input) daily = 480 :g excretion daily in urine and 20 :g
excretion daily in stool (output).

A. Biosynthesis of thyroid hormones:

There are four main step in the biosynthesis of thyroid hormones:

1. Iodine accumulation (Iodide “Pump” or “trap”):

The anion iodide (I-) is actively transported into the gland against
chemical and electrical gradients by a Na+-I- cotransport (symport) system
located in the basal membrane of the thyroid epithelial cells.

The Na+-I- symporter is stimulated by Na+/K+-ATPase.

988
Thyroid Hormones 1, 2, & 3 - Dr. Kalimi

CNS- (thiocyanate) and HClO4 - (perchlorate) ions inhibit I- transport.

a. Daily iodide requirement: 150 :g I-.


b. T/S (thyroid: serum) ratio:
Euthyroid (normal thyroid) T/S 20:1
TSH stimulation T/S 100:1
Hypophysectomized (no TSH) T/S 1:1

II. Conversion of I- (Iodide) to I2 (Iodine):

2I- Thyroid peroxidase enzyme I2


Hydrogen peroxide

Drugs propylthiouracil (PTU) and methimazole are inhibitors of this reaction.

III. Organification:

Iodination of tyrosyl residues of thyroglobulin form two types of inactive


iodotyrosines, monoiodotyrosine ( MIT) and diiodotyrosine (DIT).

IV. Coupling reactions:

Formation of active iodothyronines, T3 & T4:

T3 is formed by the coupling of one MIT and one DIT.

T4 is formed by the coupling of two DIT’s.

The unique tertiary structure of thyroglobulin exposes tyrosine residues for


iodination and favors coupling reactions.

Both organification and coupling reactions are catalyzed by thyroid peroxidase


enzyme and require hydrogen peroxide.

A. Organic binding and coupling reactions occur while tyrosine is an integral


part of the thyroglobulin molecule.

1. Thyroglobulin (TG):

a. Glycoprotein of follicular colloids.


b. Dimer of MW 650,000 daltons and hence too large to be transported
across acinar cell wall.

2. All of these steps (I-IV), and TG synthesis are stimulated by TSH.

B. Hydrolysis of thyroglobulin (Fig 3):

989
Thyroid Hormones 1, 2, & 3 - Dr. Kalimi

TG containing the iodotyrosines (MIT and DIT) and iodotyronines (T3 and
T4) is stored in the lumen of the follicle as colloids.

T3 and T4 release from the thyroid gland by the degradation of TG within the
follicular cells. Droplets of colloid are taken back into the follicular cells by
pinocytosis and coalesce with lysosomes. Lysosomal enzymes release the
iodinated constituent from peptide bonds by proteolysis.

Figure 3.

1. ↑ Proteolysis (stimulated by TSH).


2. ↑ Liberation of active hormones, T3 and T4
3. ↑ Liberation of inactive I- compounds MIT and DIT
4. MIT and DIT → deiodinase → I - → reuptake by the thyroid gland
(congenital defects in deiodinase result in
goitrous hypothyroidism).

In response to large quantifies of iodide (2 mg or more) there is a sharp


decrease in T3 and T4 biosynthesis due to ↓ expression of
Na+ - I- symporter and enzyme peroxidase genes.

C. Secretion of thyroid hormones (T3 and T4):

Plasma level of total T4 : 5-10 :g/100 mL.

T4 bound to the plasma proteins = 99.97%


T4 free = 0.03%

Plasma level of total T3 : 100-200 ng/100 mL.

990
Thyroid Hormones 1, 2, & 3 - Dr. Kalimi

T3 bound to plasma proteins = 99.7 %


T3 free = 0.3%

1. Ten times (of the total) T3 in the blood is free compared to T4.
2. Plasma half-lives of T4 and T3 are about 7 days and 1 day respectively.
3. T3 is two to three times more biologically potent than T4.

In the blood T3 and T4 are largely bound to proteins. The known binding
proteins are:

1. Thyroxine binding globulin (TBG, MW 50,000-80,000 daltons).


TBG binds about 70% of total T4 and about 50% of total T3.
2. Thyroxine binding prealbumin (TBPA or transthyretin):
TBPA binds 20% of total T4. TBPA does not bind T3.
3. Albumin: Binds 10% of T4 and 50% T3.

D. Degradation of thyroid hormones:

Most important catabolic reaction is deiodination (catalyzed by deiodinase, a


microsomal enzyme). Important catabolic organs are the liver and kidney.

Much of the iodide removed by the liver or kidney is recycled through the
thyroid.

1. About 35% of the circulating T4 is deiodinated peripherally to T3.


(T4 → 5' Deiodinase → T3).
2. About 45% is deiodinated to reverse T3 (Fig 4.)
(T4 → 5-Deiodinase → reverse T3).
3. The remainder is metabolized either by conjugation or oxidative
deamination.

The physiological importance of rT3 is not known at the present time.

Figure 4.

Acute and chronic illness, caloric deprivation and certain drugs such as
corticosteroids and propranolol (a beta blocker) increase peripheral
conversion of T4 to rT3.

991
Thyroid Hormones 1, 2, & 3 - Dr. Kalimi

1. There is a decrease in T3 production with fasting with reciprocal increase


in rT3.
2. The decrease of T3 production with fasting may be an important adaptive
response contributed to the observed lack of substrate and decreased
metabolic rate.
3. During fasting, total T4, and free T4 concentrations remain normal.

E. THYROID PHYSIOLOGY:

General Aspects:

1. Acts on most tissues.


2. Action-slow in onset and long in duration:

a. Large dose of T4: ↑ BMR seen after 2-3 days and reaches a maximum
in 10-12 days.
b. Large dose of T3: ↑ BMR seen after 6-12 hours, maximum cellular
activity in approximately 2-3 days.
c. TH (T3 and T4) is not necessary for life but improves the quality of
life.

Specific Actions:

1. Normal Growth and Development:

TH Promotes normal bone and skeletal development.


TH deficiency - bone remains infantile, premature epiphyseal closure,
stunting of growth.

GH secretion is deficient in the absence of TH.

Mental development: congenital TH deficiency responsible for cretinism


(mental retardation, dwarf stature).
Deficiency during the first year of neonatal life produces irreversible brain
damage

2. Metabolic (regulation of the overall rate of body metabolism):

TH influences BMR and intermediary metabolism through


mitochondrial ATP synthesis and the expression of genes encoding
various metabolic enzymes.

a. Calorigenic action: ↑ BMR, ↑ 02 consumption. Activation of the


membrane sodium-potassium pump (↑ Na+- K+-ATPase activity).
b. Thermogenic action: Rapid generation of heat.

992
Thyroid Hormones 1, 2, & 3 - Dr. Kalimi

Acute response to cold by ↑ BMR, ↑ heat production,↑ catecholamines,


↑ cardiac adjustments, ↑ lipolysis, ↑ muscular activity. TH exerts a
permissive effect on catecholamine action.

Chronic cold: cold adaptation causes an increased conversion of T4 to


T3.

c. Carbohydrate, protein and lipid metabolism: Increased carbohydrate


metabolism, ↑ gluconeogenesis, ↑ glycolysis, ↑ absorption of
carbohydrates from the GI tract.

Muscle: Increased rate of protein synthesis, increased rate of protein


degradation (overall negative nitrogen balance).

Adipose tissue: Increased lipolysis, increased fatty acid synthesis,


increased free fatty acid oxidation, decreased total and LDL
cholesterol. Overall increased demand for vitamins.

3. Neural: Promotes normal neuronal development.


4. Cardiovascular effects: ↑ TH, ↑ in the rate and strength of the heartbeat,
ventilation and cardiac output.

Thyroid hormone induces the synthesis of cardiac beta adrenergic


receptors.

F. Mechanism of Thyroid Hormone Action (Fig. 5):

T4 converts to T3 by the enzyme 5’ deiodinase in peripheral tissues (T4


5’deiodinase T3). T3 enters the nucleus and binds with high affinity to the
nuclear receptor. The thyroid hormone receptor dimer (homodimer) or thyroid
hormone receptor + 9 –cis retinoic acid (RXR) receptor complex
(heterodimer) along with coactivator bind to the thyroid response element
(TRE) and stimulates transcription of specific m-RNAs with induction of a
multitude of specific proteins such as growth hormone, malic enzyme, β
adrenergic receptor, TBG, Na+,K+-ATPase, cytochrome oxidase etc. resulting
in various physiological responses as described above.

993
Thyroid Hormones 1, 2, & 3 - Dr. Kalimi

Figure 5.

G. Control of Thyroid Hormone (TH) Secretion (Fig. 6):

The secretion of TSH is regulated by TH, in such a way that high TH


suppresses TSH, but low TH allows TSH to be secreted. Since TSH causes
TH to be secreted, but TH inhibits TSH secretion, we have a Negative
Feedback regulation.

Figure 6.

994
Thyroid Hormones 1, 2, & 3 - Dr. Kalimi

1. Hypothalamus:

Thyrotrophic Releasing Hormone (TRH):

Tripeptide -Pyroglutamine - Histidine - Proline amide.

TRH is the first hypothalamic releasing factor isolated, structurally


defined and synthesized.

TRH is synthesized in the hypothalamus and secreted into hypophyseal


portal circulation and carried to the anterior pituitary.

TRH stimulates TSH secretion by thyrotrophs of the anterior pituitary.

Thyroid hormones may reduce the effectiveness of TRH by inducing a


loss of receptors for TRH on the membranes of the thyrotrophs ( ↑ T3, T4,
↓TRH receptors, ↓ TSH).

It is also surprising that TRH is found outside the hypothalamus, and


sometimes outside the central nervous system. This suggests that TRH
may act also as a neurotransmitter.

2. Anterior Pituitary:

Thyroid stimulating hormone (TSH):

TSH is a glycoprotein of MW 28,000 daltons composed of two


noncovalently linked subunits α and ß.

α - subunit - identical with LH, FSH, hCG


ß - subunit - specific (immunologically and biologically) to TSH.
Both α and β subunits are required for receptor binding and hormone
action.
Plasma T ½ life of TSH = 1 hour.
Plasma level of TSH = 0.5-5.0 :U/ml (normal values).

TSH stimulates growth and metabolism of the thyroid gland including


biosynthesis of T3, T4 and TG.

TSH secretion declines during sleep and fasting and increases upon
exposure to cold.

TRH (↑ TRH, ↑ TSH) and somatostatin (↑ somatostatin, ↓ TSH) are


important factors in the control of TSH secretion.

995
Thyroid Hormones 1, 2, & 3 - Dr. Kalimi

↑ leptin, ↑TSH, ↑ T3,T4.

The mechanism of action of TSH is through the activation of adenylyl


cyclase. TSH-membrane receptor complex, ↑ adenylyl cyclase, ↑ cAMP, ↑
phosphokinases and the biological responses.

Prolonged (and excessive) TSH stimulation of thyroid gland results in a


goiter (enlargement of the thyroid gland).

Prolonged normal TSH stimulation by TSH occurs constantly and does not
cause a goiter.

Consequences of disrupting normal thyroid hormone regulation:

Thyroid hormones are regulated by a closed-loop negative feedback


system ( ↑ T3, T4, ↓ TSH and ↓ T3, T4 ↑ TSH) which serves to keep TH
output at, or close to, a steady level determined by the sensitivity of the
pituitary to the negative feedback of TH.

The system is analogous to a thermostat, and has in fact been called a


thyrostat, which maintains a constant, or near-constant, temperature.

This closed-loop can be opened in many ways; for example

Removal of thyroid: ↓ T3 and T4, ↑ TSH

Hypothyroid Goiter:

1. Administration of sodium thiocyanate (CNS-) or potassium perchlorate


(HClO4-).

T/S ratio 1:1 (preventing iodide access to thyroid gland by blocking the
iodide pump).

No TH is formed (↓ TH, ↑ TSH) and a goiter results.

2. Administration of propylthiouracil (inhibits conversion of Iodide to


Iodine).

No TH is formed (↓ TH, ↑ TSH), and goiter results with the notable


exception that T/S ratio becomes very high (100:1 or 200:1).

3. I- deficiency - Little I- is trapped (due to iodide deficiency) so little TH is


made and a goiter results (↓ TH, ↑ TSH).

996
Thyroid Hormones 1, 2, & 3 - Dr. Kalimi

Euthyroid goiter:

This situation, which can develop in people who live in the so-called "goiter
belts", regions remote from the ocean, where the soil and vegetation are poor
in iodine, is transient. In the case of a low-iodide diet, the deficit is never as
absolute as that produced by an enzymatic block. The gland gets some iodide
and makes some TH (overall, ↓ TH, ↑ TSH).

Hyperthyroid goiter:

A hyperthyroid goiter is produced by autoimmune stimulation (production of


Thyroid-Stimulating Immunoglobulins). Thyroid-Stimulating
Immunoglobulins (TSI) is a antibody against the TSH receptor which binds to
TSH receptor and mimic TSH actions on thyroid gland TH synthesis.

(↑ TSI, ↑ T3 and T4, ↓ TSH)

SUMMARY OF CONTROL ELEMENTS:

1. Euthyroid Goiter - Iodine deficiency, ↑ TSH to compensate for ↓ TH.


2. Hyperthyroid Goiter - Over secretion of TSI which mimics TSH, ↑ TSI, ↑
TH,
3. Hypothyroid Goiter - Administration of drugs such as propylthiouracil,
SCN -, perchlorate, ↓ TH and ↑ TSH.

H. Laboratory evaluation of thyroid functions:

1. Measurement of circulating thyroid hormones (T3 and T4) by


radioimmunoassay (RIA).
2. RIA for human TSH (h TSH)

Normal values = 0.5-5.0 micro units/ mL of blood

Elevation in serum TSH is the most sensitive measure for primary


hypothyroidism.

TSH above 5.0 (hypothyroidism), administration of Thyroxine (T4).

Below 0.5 (hyperthyroidism), radiation treatment plus T4.

3. Other measurements influenced by thyroid function:

a. BMR: Increased in hyperthyroidism and decreased in hypothyroidism.


b. Cholesterol and LDL: Decreased in hyper and increased in
hypothyroidism.

997
Thyroid Hormones 1, 2, & 3 - Dr. Kalimi

I. Thyroid abnormalities:

1. In deficiency states: Hypothyroidism.


2. In clinical states of over activity:
Hyperthyroidism or Thyrotoxicosis.

a. Disease characterized by hypothyroidism:

i. Thyroid failure (surgical, immune or radio iodide destruction of the


thyroid gland): Primary disease.
ii. Pituitary or hypothalamus failure (TRH or TSH deficiency):
Secondary disease
iii. Absence or defective thyroid hormone receptors, mutations in
thyroid receptor gene mostly single amino acid substitution in the
ligand binding domain of the receptor. This mutation reduces the
affinity of TH for receptors, reducing their function.
iv. Myxedema: Mucoproteinous (mucopolysaccharide and hyaluronic
acid) skin deposit. Bagginess under the eyes, swelling of the face,
iodine deficiency.
v. Cretinism: Extreme hypothyroidism during infancy or childhood.
Failure of growth, especially bone and brain, idiotic look,
protruding tongue, pot belly and mental retardation

b. Symptoms of Hypothyroidism:

i. Decreased BMR, decreased pulse rate, decreased vitality


ii. Increased LDL and cholesterol
iii. Growth failure, weight gain, constipation, cold intolerance, dry
scaly thickened skin
iv. Mucopolysaccharide deposition, hoarse voice, lethargic,
sluggishness, dull, puffy eyelids, enlargement of tongue
v. Depression and insomnia

c. Disease characterized by hyperthyroidism:


Graves' Disease (thyrotoxicosis):

Emotional trigger such as loss or potential loss of person results in


thyrotoxicosis in some individual.

↑ T3,T4, ↑ TSI, ↓ TSH


(TSI is an immunoglobulin of MW 150,000 daltons).

d. Symptoms of Hyperthyroidism:

a. Increased BMR, ↑ catecholamine activity, rapid heart rate, ↑


cardiac output, ↑ cardiac beta receptors. congestive heart failure

998
Thyroid Hormones 1, 2, & 3 - Dr. Kalimi

b. Increased sweating, vasodilation, ↑ body heat (feeling too hot),


moist skin.
c. Increased gastrointestinal malfunction, diarrhea, weight loss,
(despite an increase in appetite).
d. Bone: negative calcium balance, osteoporosis, fractures.
e. Eye discomfort, marked protrusion of eyes (exophthalmos).
f. Neuromuscular fatigue, sweating, hyperactive reflexes, negative
nitrogen balance
g. Psychotic behavior, hyperkinetic behavior (shaking of the hands),
muscular weakness (myopathy), irritability and anxiety

Pregnancy: ↑ estrogen, ↑ TBG, ↑ total plasma T3 and T4.

999
The Posterior Pituitary Gland - Dr. Witorsch

The Posterior Pituitary Gland and Related Issues


(Vasopressin and Oxytocin)
R.J. Witorsch, Ph.D.

OBJECTIVES:

At the end of this lecture, the student should be able to:

1. Construct the relationships between the hypothalamus and anterior and posterior
lobes of the pituitary gland and explain how neurosecretion participates in these
relationships.
2. Characterize the chemical nature of the 2 hormones of the posterior pituitary,
vasopressin and oxytocin, as well as their relationship with their precursor
molecules.
3. Describe the physiological function of vasopressin, as well as its actions at the
cellular and subcellular levels.
4. Describe how vasopressin secretion is regulated by plasma volume and
osmolarity.
5. Explain the etiologies of diabetes insipidus and SIADH.

Suggested Reading: Costanzo 3rd Edition, pp. 389-390, 395-401

I. THE CONCEPT OF NEUROSECRETION

A. Neurosecretion is the capacity of a nerve cell to secrete a hormone (a


substance that produces an effect at a remote site).

B. A neurosecretory cell possesses all of the features of the nerve cell (cell
body, axon, Nissl substance, conveys electrical impulses, etc.) but has the
capacity to secrete hormones.

II. ELEMENTS OF THE HYPOTHALAMO-NEUROHYPOPHYSEAL


SYSTEM

1000
The Posterior Pituitary Gland - Dr. Witorsch

Figure 1. Hypothalamus - Pituitary Relationships

A. Hypothalamic nuclei

1. Also referred to as magnocellular nuclei, hypothalamic nuclei are


comprised of cell bodies of neurosecretory cells.
2. They are sites of hormone precursor transcription, translation and
vesicle formation, processes that involve cell nuclei, ribosomes, and
Golgi apparatus.
3. Two nuclei of the hypothalamus regulate the hypothalamo-
neurohypophyseal sytstem:

a. Supraoptic nucleus, located above optic chiasm;


b. Paraventricular nucleus, located on either side of third
ventricle.

B. Hypothalamo-neurohypophyseal tract (pituitary stalk)

1. Is comprised of axons of neurosecretory cells.


2. Is the site of transport of the prohormone by axoplasmic flow, and post
ribosomal processing (proteolytic cleavage) to actual hormone and its
associated products.

1001
The Posterior Pituitary Gland - Dr. Witorsch

3. Hypothalamo-neurohypophyseal tract contains glycoprotein enriched


material that is transported by axoplasmic flow. This material is also
termed “neurosecretory material” (NSM) and “neurophysins.“

C. Posterior pituitary gland (also referred to as pars nervosa or infindibular


process)

1. This structure is comprised of terminals of the neurosecretory cell and


is the site of storage and release of hormones and associated products.
2. Posterior pituitary gland also contains pituicytes (sustenticular cells).

III. ELEMENTS OF HYPOTHALAMO-ADENOHYPOPYSEAL SYSTEM (Fig.


1)

A. This system controls anterior and intermediate lobe function (also referred
to as adenohypophysis because of its glandular nature).

B. The hypothalamus is connected to the adenohypophysis by a neuro-


vascular link, the hypothalamo-hypophyseal portal system.

1. A portal system is a system of veins that starts as capillaries and


ends as capillaries.

C. Neurosecretory neurons in the hypothalamus make releasing and


inhibiting hormones that are secreted into the median eminence in the base
of hypothalamus and transported to the adenohypophysis by the portal
system.

1. The median eminence, a midline protuberance in the base of the


brain, is the origin of the a portal venous system.

IV. HORMONES OF THE POSTERIOR PITUITARY

A. Vasopressin (antidiuretic hormone, ADH) and oxytocin.

B. Chemistry of posterior pituitary hormones (Fig. 2)

1. Both are nonapeptides containing a 6 amino acid ring (S-S bond


between cysteine residues 1 and 6) and 3 amino acid side chain.
2. Vasopressin (ADH) and oxytocin differ in 2 amino acid residues
(3, 8).
3. Both hormones weigh about 1100 dalton.

1002
The Posterior Pituitary Gland - Dr. Witorsch

Figure 2. Posterior Pituitary Hormones

V. SYNTHESIS, PROCESSING, AXOPLASMIC FLOW, AND RELEASE OF


VASOPRESSIN AND OXYTOCIN (Fig. 3)
A. Synthesis of vasopressin and oxytocin occur in hypothalamic nuclei first
as pre-prohormones.

1. In humans both hormones are made in both the supraoptic and


paraventricular nuclei.

a. The content of both hormones in supraoptic nucleus


exceeds that of paraventricular nucleus.
b. Synthesis of pre-prohormones involve transcription and
translation.
c. Pre-propressophysin, the precursor for vasopressin, is
comprised of a signal peptide (19 amino acids), vasopressin
(9 amino acids), neurophysin II (95 amino acids), and
glycoprotein (39 amino acids).
d. Pre-prooxyphysin, the precursor for oxytocin is comprised
of a signal peptide (19 amino acids), oxytocin (9 amino
acids), and neurophysin I (93 amino acids).
e. Initial processing of pre-prohormones occurs by removal
of signal peptide from N-terminal ends of both precursor
molecules producing a prohormone (propressophysin and
prooxyphysin).

2. Prohormones are housed as membrane encapsulated granules.

a. Packaging of these granules involves the Golgi apparatus.

1003
The Posterior Pituitary Gland - Dr. Witorsch

B. Postribosomal proteolytic processing and axoplasmic flow of


vasopressin and oxytocin occur in the hypothalamo-neurohypophyseal
tract.

1. Propressophysin is proteolytically processed to vasopressin (1100


mw), neurophysin II (about 10,000 mw) and glycoproteins.
2. Prooxyphysin is proteolytically processed to oxytocin (1100 mw)
and neurophysin I (about 10,000 mw).

C. Storage and release of hormones and associated products occur in the


pars nervosa.

1. Release involves a process called "excitation-secretion coupling"


which is analagous to excitation-contraction coupling.

a. This process involves membrane depolarization, calcium


influx and exocytosis performed by actin-myosin like
proteins which translocate secretory granules.
b. Hormones and by products are released from the pars
nervosa, but only the hormones are associated with
physiologic functions (to the best of our knowledge).

1004
The Posterior Pituitary Gland - Dr. Witorsch

Figure 3. Synthesis, Processing, Axoplasmic Flow and Release of ADH


(Vasopressin) and Oxytocin

VI. TRANSPORT AND CLEARANCE OF VASOPRESSIN AND OXYTOCIN

A. Hormones circulate unbound (as do most protein/peptide hormones) and


are cleared by liver and kidney (half-life estimates range from about 2-3 to
18 min).

VII. PHYSIOLOGIC ACTIONS OF VASOPRESSIN

A. Vasopressor action - generally considered a pharmacologic effect


(requires high concentrations), may have some physiologic relevance
since high levels of vasopressin are released in response to a 10% decrease
in blood volume.

1005
The Posterior Pituitary Gland - Dr. Witorsch

1. This action involves V1 receptor (serpentine, G-protein coupled)


and a Ca++ dependent form of signal transduction.

a. V1 receptor mediated signaling involves phospholipase C


activation and generation of IP3 and diacylglycerol.

Figure 4. Action of ADH in Kidney


Adapted from Rhoads and Pflanzer, 3rd Edition

B. Antidiuretic action - reabsorption of water in distal convoluted tubule and


collecting ducts is the major physiological action of this hormone (Fig. 4).

1. This leads to increased water permeablility - water will move in


response to osmotic gradient thus concentrating urine.
2. Antidiuretic action of vasopressin is adenylate cyclase mediated and
involves V2 receptors which are distinct from V1 receptors.

a. V2 receptors are found on blood (serosal) surface of nephron


target cells and generate cyclic AMP (a G-protein mediated
effect).
b. Signal transduction leads to increased protein phosphorylation
and water permeability at the luminal (mucosal) surface of
target cell.
c. Permeability change is due to altered activity and/or number of
water channels (aquaporin 2).

1006
The Posterior Pituitary Gland - Dr. Witorsch

VIII. REGULATION OF VASOPRESSIN SECRETION AND ITS ROLE IN


FLUID BALANCE - OVERALL ROLE CONSERVATION OF FLUID
A. Signals that regulate vasopressin release are inversely related to
extracellular fluid (ECF) volume and directly related to ECF osmolarity.
1. Hypovolemia

a. The more potent, but less sensitive stimulus for vasopressin


secretion than hyperosmolarity.

i. Threshhold for ADH release requires about a 10-


20% decline in fluid volume.
ii. Response (release of ADH) is very robust.

b. Hypovolemic response is mediated by left atrial stretch


receptors.
c. Stretch receptors lead to increased firing of vagal inhibitory
neurons leading to decreased vasopressin release.
d. Decrease of stretch (e.g., loss of volume) would have the
reverse effect (disinhibition) - increased vasopressin
release.

2. Hyperosmolarity

a. The more sensitive stimulus for vasopressin secretion than


hypovolemia.

i. occurring in response to changes in plasma


osmolarity of 1-2%.

b. This response is mediated by hypothalamic osmoreceptors


(These receptors exist near, but are distinct from,
magnocellular nuclei).
c. Osmoreceptors regulating vasopressin release are related to
drinking centers in the hypothalamus (where osmolarity has
been shown to influence drinking behavior).

3. Interaction can occur between osmotic and volume controls of


vasopressin secretion.

a. Angiotensin II can sensitize the osmotic receptors


indicating that volume depletion can enhance sensitivity of
the osmotic stimuli.

4. Other stimulators of vasopressin release

1007
The Posterior Pituitary Gland - Dr. Witorsch

a. Stress, anxiety, pain, nausea, nicotine, opiates, barbiturates,


ß-adrenergic agonists, estrogens, increased pCO2,
decreased pO2.

5. Inhibitors of vaspressin release

a. Volume expansion, hypoosmotic stimuli, alcohol,


glucocorticoids, and α-adrenergic agonists.

IX. CLINICAL DISORDERS OF VASOPRESSIN SECRETION

A. Diabetes insipidus (diabetes = "pass through" [Gr.]; insipidus - "tasteless"


[L.]) - deficiency in vasopressin secretion.

1. Symptoms

a. Polyuria - up to 10-fold increase in daily water excretion


(15 L).
b. Polydipsia - excess drinking secondary to polyuria.
c. Symptoms only manifest when anterior pituitary hormones
are present (related to maintenance of GFR).
d. In normal individuals, water deprivation would concentrate
urine, with diabetes insipidus this would not occur.

2. Etiologies

a. Neurogenic (also referred to as central diabetes insipidus)-


absence of vasopressin.

i. Usually associated with a lesion in the tract or


magnocellular nuclei rather than loss of posterior
lobe.
ii. An hereditary strain of a rat (Brattleboro strain)
exists that cannot synthesize vasopressin.
iii. A neurogenic form of diabetes insipidus requires
about an 80% deficiency in vasopressin.

b. Nephrogenic - vasopressin is unresponsiveness in target


cells of nephron, due to the following mechanisms:

i. receptor dysfunction - an X chromosome linked V2


receptor defect;
ii. defect in gene that expresses water channel;
iii. enhanced vasopressin metabolism and its clearance
from the body (during pregnancy).

1008
The Posterior Pituitary Gland - Dr. Witorsch

B. Excess vasopressin - Syndrome of inappropriate ADH secretion (SIADH)

1. Symptoms

a. ECF expansion;
b. Serum hypotonicity and hyponatremia.

2. Etiology - caused by enhanced sensitivity of osmoreceptors thus


secreting ADH release too readily.

X. OXYTOCIN

A. Functions in the female - involve contraction of myoepithelial cells

1. Parturition (Ferguson reflex) - to be discussed in a later lecture


(Pregnancy and lactation)
2. Milk ejection from lactating breast - to be discussed in a later
lecture (Pregnancy and lactation)
3. Sperm transport (possibly)

a. Oxytocin release occurs during orgasm (in nonpregnant as


well as pregnant individual).

B. Function in male - oxytocin release occurs during ejaculation.

1. Sperm transport may occur as a result of vas deferens smooth


muscle contraction.

XI. STUDY QUESTIONS - All Type A

1. The generation of vasopressin from its precursor molecule:

A. occurs in the cell bodies of the paraventricular and supraoptic


nucleus
B. involves cleavage of pre-propressophysin
C. occurs in the hypothalamo-neurohypophyseal tract
D. involves removal of the signal peptide off the C-terminal end of
the propressophysin molecule
E. occurs in the pars nervosa

2. Which structure is differentiated to produce pre-prooxyphysin?

. supraoptic nucleus
A. hypothalamo-neurohypophyseal tract
B. hypothalamo-hypophysela portal system
C. pars nervosa

1009
The Posterior Pituitary Gland - Dr. Witorsch

D. pars intermedia

3. V2 receptors for ADH (ie., those mediating the principal action of the
hormone)

. are found in vascular smooth muscle


A. are located on the luminal surface of collecting duct epithelial cells
B. are associated with a G-protein adenylate cyclase signaling system
C. are associated with a G-protein phospholipase C signaling system
D. mediate sodium transport in the kidney

4. Which of the following substances contains a glycoprotein at its C-


terminus?

. oxytocin
A. neurophysin I
B. vasotocin
C. prepropressophysin
D. vasopressin

5. Which of the following structures is associated with "excitation-secretion


coupling"?

. suprooptic nucleus
A. paraventricular nucleus
B. hypothalamo-neurohypophyseal tract
C. hypothalamo-hypophyseal portal system
D. pars nervosa

6. The conversion of pre-prooxyphysin to prooxyphysin:

. occurs in the cell bodies of the paraventricular and supraoptic


nucei
A. involves cleavage between oxytocin and neurophysin I
B. occurs in the hypothalamo-neurophyspophyseal tract
C. involves removal of the signal peptide off the C-terminal end of
the pre-prooxyphysin molecule
D. occurs in the pars nervosa.

Key: 1. C; 2. A; 3. C; 4. D; 5. E; 6. A.

1010
Adrenal Cortex 1, 2, & 3 - Dr. Witorsch

Adrenal Cortex 1, 2, & 3


R.J. Witorsch, Ph.D.

OBJECTIVES:

At the end of this block of lectures, the student should be able to:

1. Describe the anatomical relationship between the adrenal cortex and adrenal
medulla, as well as the functional zonation within the adrenal cortex.
2. Identify the systems controlling adrenocortical function as well as the major
biological actions of the principal hormones secreted by the adrenal cortex.
3. Describe the functional elements, as well as the control mechanisms, involved in
the hypothalamo-pituitary-adrenocortical system.
4. Describe the types, patterns, and mechanisms involved in ACTH and cortisol
release.
5. Describe the elements and mechanisms involved in the control of aldosterone
secretion from the zona glomerulosa of the adrenal cortex.
6. Describe the pathways involved in the synthesis of the major steroid hormones
from the adrenal cortex, namely cortisol, aldosterone, and DHEA.
7. Explain the role of proteolytic cleavage in the production of ACTH in the anterior
pituitary gland, as well as the relationship between ACTH and its precursor
molecule and related peptides.
8. Describe the principal biological actions of corticosteroids and other secretions of
the adrenal cortex.
9. Describe the mechanisms of corticosteroid action.
10. Explain the etiologies and symptomologies of the following disorders of
adrenocortical function

a. primary and secondary hypercortisolism


b. primary and secondary hypocortisolism
c. the various forms of congenital adrenal hyperplasia.
d. primary and secondary hyperaldosteronism.

Suggested Reading: Costanzo 3rd Edition, pp. 408-420

I. OVERVIEW OF ADRENAL GLAND (Figure 1.)

1011
Adrenal Cortex 1, 2, & 3 - Dr. Witorsch

Figure 1. The Adrenal Gland

• The adrenal gland is roughly triangulated in shape and sits on top of each
kidney (hence, the name "adrenal").

A. Embryogenesis and morphology of adrenal gland

1. The adrenal gland is comprised of 2 separate endocrine organs coming


from different embryonic origins and secreting chemically different
hormones.
2. The adrenal cortex or outer portion is derived from mesoderm and is
comprised of three zones of lipid laden epitheloid cells interspersed
with sinusoids.

a. Zona glomerulosa is the outer zone where epitheloid cells are


arranged as "glomeruli" or "whorls" comprising 15% of the
adrenal cortex.
b. Zona fasciculata is the middle zone where epitheloid cells are
arranged as straight columns of single cells or "fascicles"
comprising 80% of adrenal cortex.
c. Zona reticularis is the innermost zone exhibiting a haphazard
or "reticular" arrangement of epitheloid cells comprising 5% of
cortex.

1012
Adrenal Cortex 1, 2, & 3 - Dr. Witorsch

3. The adrenal medulla is the inner portion derived from ectoderm in


which the hormone secreting cells are modified postganglionic
sympathetic neurons (or chromaffin cells, based on their staining
characteristics)

B. Hormones and control systems of adrenal gland

1. The adrenal cortex produces steroid hormones, derived from


cholesterol. Steroid hormones are classified into several types based
upon their function. These types are:

a. Corticosteroids which are further classified as:

i. Glucocorticoids, which elevate blood glucose among


other effects, the most well-known example is cortisol
(also referred to as hydrocortisone).
ii. Mineralocorticoids which function to conserve sodium,
the most well-known example being aldosterone.

b. Sex hormones, which are further classified as:

i. Progestins, estrogens, and androgens.

2. The adrenal medulla produces catecholamines, hormones derived from


the amino acid, tyrosine:

a. The major catecholamine secreted by the adrenal medulla is


epinephrine and, to a lesser extent, norepinephrine.

3. Control systems of the adrenal gland

a. The adrenal cortex contains 2 distinct control systems that are


regionally localized:

i. Pituitary-adrenocortical system where the anterior


pituitary (via ACTH) controls cortisol secretion. The
system is called into play when the organism is
confronted with stress (z. fasciculata and reticularis).
ii. Renin-angiotensin-adrenocortical system controls
aldosterone secretion and is involved in electrolyte and
fluid balance (z. glomerulosa).

b. The adrenal medulla is a component of the sympathetic


nervous system also involved in the stress response.

1013
Adrenal Cortex 1, 2, & 3 - Dr. Witorsch

II. PITUITARY - Adrenocortical System: Control of cortisol secretion (Figure


2.)

Figure 2. Pituitary-adrenocortical system

A. Elements of the system and their functions

1. Central nervous system (brain) involves:

a. Extrahypothalamic structures, such as the reticular formation,


thalamo-neocortical structures, and limbic system.
b. The hypothalamus, which is a nodal way-station receiving
input from the above extrahypothalamic structures.

i. Neurosecretory neurons in the hypothalamus produces


corticotrophin releasing hormone (CRH).
ii. CRH:

• is a polypeptide of 41 amino acids.


• is transported from the median eminence of the
hypothalamus to anterior pituitary via the
hypothalamo-hypophyseal portal system.
• stimulates release of ACTH from the anterior
pituitary.

2. Anterior pituitary which contains basophilic cells (react to basic stain)


that produce the peptide hormone, ACTH (adrenocorticotrophic
hormone)

1014
Adrenal Cortex 1, 2, & 3 - Dr. Witorsch

3. Adrenal cortex ( specifically the zonae fasciculata and reticularis)


where ACTH exerts the following effects:

a. promotes adrenocortical growth;


b. stimulates steroid production and secretion (primarily cortisol,
as well as adrenal androgens);
c. maintenance of adrenocortical sensitivity to ACTH stimulation.

4. Corticosteroid binding proteins in plasma:

a. Under basal (nonstress) conditions 90-95% of circulating


cortisol is bound to plasma proteins in a noncovalent fashion.
b. The 2 prominent plasma binding proteins are:

i. Albumin

• exhibits low affinity (weak), high capacity


binding for cortisol;
• binds about 15% circulating cortisol;
• and is relatively insignificant, physiologically.

ii. CBG (corticosteroid binding globulin, also referred to


as transcortin)

• exhibits high affinity (strong), limited capacity


(or saturable) binding of cortisol;
• binds 75-80% circulating cortisol;
• physiologically significant form of cortisol
binding.

c. Functions of cortiocosteroid plasma proteins

i. Increases the solubility of steroid in plasma.


ii. Controls the distribution of (or parcels) steroids to
tissues.

5. Liver and other target tissues

a. Biological responses to corticosteroids


b. Metabolic inactivation and clearance of corticosteroids

i. Increases polarity of corticosteroids due to chemical


modification and conjugation with glucuronide and
sulfate and clearance of these conjugates by the kidney.

1015
Adrenal Cortex 1, 2, & 3 - Dr. Witorsch

ii. Determines biological half-life of corticosteroids


(between 60-90 minutes).

B. Regulation of cortisol secretion (servomechanisms) (Fig. 2)

1. Negative feedback effect of cortisol on ACTH secretion

a. An inverse relationship exists between plasma cortisol and


ACTH.

i. ACTH hypersecretion occurs after direct lowering of


cortisol (e.g., bilateral adrenalectomy, inhibition of
steroidogenesis, primary hypocortisolism).
ii. ACTH hyposecretion (and, often times, adrenal atrophy)
occurs after exogenous glucocorticoid treatment or
primary hypercortisolism (autonomous adrenal tumor).

b. Glucocorticoid feedback occurs both in brain (hypothalamic


and extrahypothalamic structures) and anterior pituitary (which
is the primary site of glucocorticoid feedback).
c. Alterations of CBG levels could influence level of function of
the pituitary-adrenocortical system via glucocorticoid
feedback.

i. Reason: unbound cortisol is the feedback signal, based


on the following evidence:

• Elevated CBG stimulates pituitary-adrenal


function;
• Decreased CBG is inhibitory.

ii. CBG production by the liver is stimulated by estrogen,


thyroxine and progesterone and inhibited by
testosterone and liver dysfunction.

d. Altered hepatic clearance of cortisol could also alter pituitary-


adrenal function by the negative feedback mechanism.

i. Reason: plasma cortisol level is the result of a balance


between cortisol secretion and clearance.

• Increased clearance (i.e., shortened half-life)


would stimulate the pituitary-adrenocortical
system.

1016
Adrenal Cortex 1, 2, & 3 - Dr. Witorsch

• Decreased clearance (prolonged half-life) would


inhibit the pituitary-adrenocortical system.

ii. Clearance is stimulated by thyroxine, food intake, and


hypertension and is inhibited by androgen, restricted
food intake, liver disease.

e. Glucocorticoid feedback is a "crude" controller of pituitary


adrenocortical function:

i. It determines the upper and lower levels of the system.


ii. It does not appear to determine the patterns of pituitary-
adrenal secretion which occur under basal and stress
conditions.

2. Types and patterns of ACTH and cortisol release.

Figure 3. Circadian pattern of cortisol in plasma

a. Under basal (non-stress) conditions one sees a circadian or


diurnal pattern (Figure 3).

i. A recurring pattern in plasma ACTH and cortisol


occurring approximately every 24 hours (hence, “circa-
dian") .
ii. Changes in cortisol are a consequences of ACTH and,
therefore, the latter precedes the former by a short
interval (15-30 minutes) (not shown in Figure 3).
iii. In an individual with a normal activity pattern (active
during day and asleep at night):

1017
Adrenal Cortex 1, 2, & 3 - Dr. Witorsch

• Average hormone (ACTH and cortisol) levels


are highest at about 8:00 a.m. (upon waking
from sleep).
• Average hormone levels decline during the day
(period of greatest activity) and are lowest
levels at about 1:00 a.m.
• Average hormone levels increase during sleep
reaching peak levels at about 8:00 a.m.

iv. In reality, hormone secretion occurs as episodic bursts


of hormone release.

• During the rising phase of the circadian pattern,


the frequency of "bursting" activity is greater
than during the declining phase.
• Circadian patterns can be absent in
adrenocortical hypersecretion disorders (e.g.,
Cushing's syndrome).
• In evaluating patients, the circadian and
episodic nature of basal secretion of the
pituitary-adrenocortical axis should be
considered, thus requiring multiple
measurements of serum hormone levels, and/or
collection of urinary steroids at different blocks
of time during the day.

v. Plasma cortisol concentrations during non-stress


conditions stays within 5 µg/100 ml to 20 µg/100 ml
(.25 to 1.00 µM) and most hormone is bound by plasma
protein.
vi. Circadian pattern is determined by the activity pattern
of the individual.

• Individuals with night-time occupations have


circadian rhythms that are the "mirror-images"
of individuals with day-time occupations.

vii. Circadian patterns are not regulated by negative


feedback.

b. Stress-induced ACTH and subsequent cortisol release result


from the release of CRH from the hypothalamus (Figure 4).

i. Acute (short-lived) stress

1018
Adrenal Cortex 1, 2, & 3 - Dr. Witorsch

• ACTH release appears as a "spike" reaching


peak levels at 2.5 to 5 minutes.

o Cortisol levels peak between 15 to 30


minutes after the stress and return to
basal levels by 60 minutes.
o Stress-induced plasma cortisol levels
may reach 70 µg/100 ml - 100 µg/100
ml (3 to 5 µM).
o Most of this cortisol is not bound to
plasma protein and hence accessible to
the cells.

ii. Prolonged stress (several hours)

• ACTH and cortisol are maintained, but not


indefinitely.

Figure 4. Serum ACTH and cortisol after acute or prolonged stress

1019
Adrenal Cortex 1, 2, & 3 - Dr. Witorsch

III. RENIN - ANGIOTENSIN SYSTEM: Regulation of Aldosterone Secretion


(Figure 5):

Figure 5. Renin-angiotensin-aldosterone system

• The general function of aldosterone is to conserve Na+ and, hence, maintain


fluid volume.
• Stimuli for aldosterone secretion are decreased blood volume (major
stimulus), electrolyte alterations (increased plasma K+); very high doses of
ACTH , and stimulation of sympathetic innervation of kidney.

A. Renin-Angiotensin System

1. Dynamics of renin-angiotensin system.

a. Decreased blood volume produces decreased renal arterial


pressure.
b. Decreased renal arterial pressure is sensed by stretch receptors
of juxtaglomerular (JG) cells.

i. This results in increased renin secretion from JG cells.

1020
Adrenal Cortex 1, 2, & 3 - Dr. Witorsch

c. The macula densa monitors Na+ and Cl- filtration in the distal
tubules.

i. Decreased delivery of these electrolytes to distal


tubules also evokes increased renin secretion from JG
cells.

d. Renin then converts angiotensinogen (a protein produced by


the liver) to angiotensin I, a decapeptide.
e. Angiotensin I then loses 2 amino acids at its C-terminal end
under the influence of angiotensin converting enzyme (ACE) to
form angiotensin II (octapeptide).
f. Angiotensin II stimulates aldosterone release from the zona
glomerulosa.
g. Stimulation of Na+ retention by aldosterone retains body fluid
and restores renal arterial pressure, thus shutting off of the
original stimulus for the system, hypovolemia.
h. Angiotensin II is inactivated in plasma by proteolysis
(angiotensinases).
i. The following alternate renin-angiotensin pathway also exists:

i. Angiotensin I is converted to a nonapeptide, (Des Asp1)


Angiotensin I, by an aminopeptidase in plasma and
tissues.
ii. (Des Asp1) Angiotensin I is converted to the
heptapeptide, (Des Asp1) Angiotensin II (or
Angiotensin III), by ACE.
iii. Angiotensin III appears to be as potent as Angiotensin
II in stimulating aldosterone release but possess only
about 50% of its pressor activity.

j. Renin and aldosterone exhibit circadian patterns like cortisol


but not as pronounced.

i. Circadian patterns occur independent of posture.


ii. Aldosterone circulates bound primarily to albumin and
is metabolized in a fashion similar to cortisol.
iii. The half life of aldosterone is approximately 30
minutes.
iv. Standing tends to elevate serum renin and aldosterone
levels. Why?

IV. BIOSYNTHESIS (Figure 6) and degradation of steroid hormones.

1021
Adrenal Cortex 1, 2, & 3 - Dr. Witorsch

A. General overview and pertinent concepts

1. Steroid hormones are derived from cholesterol and contain the


following characteristics (Fig 6):

a. a steroid nucleus, which is comprised of 17 carbons;


b. the steroid nucleus is roughly planar containing 3 hexane
rings (A,B,C) and one pentane ring (D);
c. rings may contain double bonds (represented as Δ),
hydroxyl and keto groups;
d. certain constituents (side chains and hydroxyl) may be
above or below the plane of the nucleus:

i. below the plane is designated as α (dashed line)


ii. above the plane is designated as β (solid line);

2. Adrenal cortex produces representatives of all of the different


families of naturally occurring steroids, such as:

a. 21 carbon steroids - steroid nucleus (17 carbons), methyls


18 and 19, and a 2 carbon sidechain (carbons 20 and 21)

i. progestins (e.g., progesterone)


ii. corticosteroids (e.g., cortisol, aldosterone):

b. 19 carbon steroids - steroid nucleus, methyls 18 and 19

i. androgens (DHEA, testosterone)

c. 18 carbons steroids - steroid nucleus, methyl 18

i. estrogens (estradiol-17β)

1022
Adrenal Cortex 1, 2, & 3 - Dr. Witorsch

Figure 6. Adrenal Cortex Steroidogenesis

1023
Adrenal Cortex 1, 2, & 3 - Dr. Witorsch

B. Steroidogenic pathways

1. Cholesterol side chain cleavage (also referred to as the desmolase


reaction):

a. Cholesterol (27 carbons) ------> Δ5 pregnenolone ( 21 carbons)


and isocaproic acid (6 carbons);
b. Cholesterol side chain cleavage is the site of ACTH action.

2. Steroidogenesis in the zona glomerulosa - aldosterone production

a. Daily production of aldosterone occurs independent of ACTH


stimulation.
b. Regulated by angiotensin II and III (downstream).
c. Pathway from Δ5 pregnenolone to aldosterone.

i. 3 ß-ol dehydrogenase:isomerase (3ß-hydroxysteroid


dehydrogenase:isomerase):

• Δ5 pregnenolone –> progesterone.


• progesterone is a Δ4, 3 ketosteroid.
• Δ4, 3 ketosteroid is a common feature of many
steroid hormones).

ii. 21-hydroxylase (unique to adrenal cortex):

• progesterone –> 11-deoxycorticosterone (DOC).


• DOC is a potent mineralocorticoid.

iii. 11ß-hydroxylase (unique to adrenal cortex):

• DOC –> corticosterone (Cpd. B) (a major


secretory product of the adrenal cortex).
• corticosterone exhibits both glucocorticoid and
mineralocorticoid activities.
• corticosterone production is about 3 mg/day.

iv. 18-hydroxylase (unique to adrenal cortex):

• corticosterone --> 18-hydroxycorticosterone.


• 18-hydroxylase present in z. glomerulosa not z.
fasciculata and reticularis.
• site of action of Angiotensin II and III.

1024
Adrenal Cortex 1, 2, & 3 - Dr. Witorsch

v. 18-hydroxysteroid dehydrogenase (unique to adrenal


cortex):

• 18-hydroxycorticosterone --> aldosterone.


• aldosterone is the most potent naturally
occurring mineralocorticoid.
• aldosterone circulates primarily as a hemiacetal
formed between the 18-aldehyde and 11-
hydroxy groups (Fig. 7).
• aldosterone production is about 0.15 mg/day.

Figure 7. Aldosterone and


aldosterone hemlacetal

3. Steroidogenesis in zonae fasciculata and reticularis - cortisol


production

a. Cortisol production regulated by ACTH (cholesterol side chain


cleavage).
b. Referred to as the “17α-hydroxylase pathway”.
c. Production of cortisol (in z. fasciculata and reticularis) follows
essentially the same sequence of enzyme reactions as does the
production of corticosterone (in z. glomerulosa), except for the
fact that cortisol is 17α-hydroxylated and corticosterone is not.
d. Pathway from Δ5 pregnenolone –> cortisol.

i. 17α-hydroxylase (unique to z.. fasciculata/reticularis


not z. glomerulosa):

1025
Adrenal Cortex 1, 2, & 3 - Dr. Witorsch

• Δ5 pregnenolone –> 17α hydroxypregnenolone.


• progesterone –> 17α-hydroxyprogesterone.

ii. 3-ß-ol dehydrogenase: isomerase (3ß-hydroxysteroid


dehydrogenase: isomerase):

• Δ5 pregnenolone –> progesterone.


• 17α-hydroxypregnenolone –> 17α-
hydroxyprogesterone.

iii. 21-hydroxylase (unique to adrenal cortex):

• 17α-hydroxyprogesterone –> 11-deoxycortisol.

iv. 11ß-hydroxylase (unique to adrenal cortex):

• 11-deoxycortisol –> cortisol (Cpd. F,


hydrocortisone).
• cortisol production is about 20-30 mg/day.
• cortisol exhibits both glucocorticoid and
mineralocorticoid activities.
• cortisol is the principal glucocorticoid secreted
by the adrenal cortex.

4. Steroidogenesis in zonae fasciculata and reticularis: adrenal


androgen and estrogen production (Figure 6).

a. This pathway is also a component of 17α-hydroxylase


pathway.
b. Pathway from Δ5 pregnenolone and progesterone (through
androgens) to estradiol.

i. 17α-hydroxylase:

• as above, generating 17α-hydroxyprenenolone,


and 17α-hydroxyprogesterone.

ii. C-17, 20 lyase:

• 17α-hydroxypregnenolone --> DHEA


(dehydroepiandrosterone) and 17α-
hydroxyprogesterone ---> androstenedione.
• reaction involves elimination of 20,21 side
chain to form a C19 steroid.
• DHEA is a weak androgen.

1026
Adrenal Cortex 1, 2, & 3 - Dr. Witorsch

• DHEA is a major source of androgen in the


female.
• DHEA is secreted as a sulfate (20-30 mg/day).

iii. 3ß-ol dehydrogenase:isomerase:

• DHEA --> androstenedione.


• androstenedione is a weak androgen (immediate
precursor of testosterone).
• androstenedione is a Δ4,3-ketosteroid as well as
a 17-ketosteroid.

iv. 17ß-hydroxysteroid dehydrogenase:

• androstenedione --> testosterone.


• testosterone is the major male sex hormone.

v. Aromatase (formation of aromatic A-ring):

• testosterone -----> 19 nortestosterone.


• 19-nortestosterone -----> estradiol 17β.

Note: 17ß-hydroxysteroid dehydrogenase and


aromatase are also major enzymes in the gonads
(testes and ovary).

C. Functional Zonation of the Adrenal Cortex - things to know.

1. Zona glomerulosa:

a. is the only source of aldosterone, does not produce cortisol or


sex steroids.
b. contains 18-hydroxylase.
c. does not contain of 17α-hydroxylase.
d. is the target for angiotensin II and III (18-hydroxylation)
e. is relatively insensitive to ACTH.

2. Zonae fasciculata and reticularis

a. is the source of cortisol and sex steroids, and does not produce
aldosterone.
b. contains 17α-hydroxylase and C17,20 lyase (both required for
adrenal androgen production).
c. does not contain 18-hydroxylase.
d. is dependent upon ACTH.

1027
Adrenal Cortex 1, 2, & 3 - Dr. Witorsch

D. Degradation of Corticosteroid Hormones

1. Involves modification of corticosteroids by liver.


2. Major reaction is A-ring reduction: conversion of Δ4,3-ketone to a
tetrahydroderivative of the compound.
3. Other modifications of corticosteroids that occur in liver are:

a. 11-hydroxy configuration is oxidized to 11-keto (a reversible


reaction) by the enzyme 11β hydroxysteroid dehydrogenase
(cortisol converted to cortisone).
b. Reduction of C-20 ketone configuration to C-20 hydroxyl
configuration.
c. Cleavage of C17-C20 bond (leading to formation of 17-
ketosteroid).
d. Conjugation of steroid at position 3 to form glucuronides and
sulfates which increases aqueous solubility of the compound.

4. Excretion in urine of above metabolic products which tend be more


water soluble.

V. ACTH and related peptide hormones

A. ACTH and related peptide hormones (α and ß-Melanophore stimulating


hormones [MSH], ß-Lipoptrophic hormone [LPH], and ß- Endorphin) are
derived by proteolytic cleavage (post-translational modification) from the
peptide precursor, pro-opiomelanocortin (POMC).

1. POMC is detected in anterior lobe, intermediate lobe and brain.


2. Secreted hormones of this family are produced by proteolytic
cleavage at basic amino acid residues (such as, Arg-Lys, Arg-Arg,
Lys-Arg, Lys-Lys).
3. Processing of POMC varies, depending on cellular site.

a. The anterior lobe generates primarily ACTH and ß-


endorphin.
b. The intermediate lobe generates primarily α-MSH.
c. The brain generates primarily ACTH, ß-endorphin, and α-
MSH.

4. Functions or effects of products of POMC (other than ACTH)

a. MSH's: darken skin by dispersion of melanosomes


(pigmenting granules).

i. MSH shares homology with ACTH (which has


inherent MSH activity). This homology may

1028
Adrenal Cortex 1, 2, & 3 - Dr. Witorsch

explain hyperpigmentation in humans under certain


conditions.
ii. Glucocorticoids also inhibit MSH release.

b. β-LPH (lipotropic hormone or lipotropin)

i. β-LPH is lipolytic in action.

c. β-Endorphin

i. Exhibit endogenous opiate activity and may play a


role in mood states and analgesia.
ii. ß-Endorphin release is stimulated by CRH and is
inhibited by glucocorticoids, as is ACTH.

VI. EFFECTS OF ADRENOCORTICAL SECRETIONS

A. Glucocorticoids: cortisol, corticosterone

1. Intermediary metabolism.

a. Protein breakdown via a catabolic and/or antianabolic


action in non hepatic tissues leading to the release of amino
acids.
b. Carbohydrate metabolism

i. Stimulation of gluconeogenesis indirectly (via


amino acids and lipids) and directly (via
gluconeogenic enzymes) leading to elevation in
blood glucose.

c. Lipid metabolism:

i. Glucocorticoids exhibit a permissive effect on


growth hormone induced lipolysis which promotes
a centripetal fat redistribution (accumulation of fat
in the trunk and head and loss of fat in the
extremities).

2. Feedback inhibition on ACTH secretion - as discussed previously.


3. Cardiovascular effects:

a. Glucocorticoids enhance vascular reactivity to


catecholamines (a permissive effect).

1029
Adrenal Cortex 1, 2, & 3 - Dr. Witorsch

i. This effect is produced by retardation of


catecholamine inactivation by the enzyme COMT
(catechol-o-methyl transferase).

b. Glucocorticoids also evoke a positive inotropic effect on


the myocardium.
c. Glucocorticoids also have inherent mineralcorticoid (Na
retaining) activity which tends to elevate blood pressure.

4. Nervous system effects:

a. When corticosteroids are absent (as in adrenocortical


insufficiency) the following may occur:

i. slower EEG alpha-waves.


ii. increased sensitivity (decreased threshold) to
olfactory and gustatory stimuli.

b. Cortisol also can alter behavior as follows:

i. producing euphoria, psychotic episodes and


depression.

5. Gastrointestinal effects:

a. Glucocorticoids increase gastric acid and pepsin secretion.

i. Thus, glucocorticoid therapy is contraindicated in


individuals prone to ulcers.

b. Glucocorticoids promote intestinal lipid absorption.

6. Skeletal muscle effects:

a. Skeletal muscles readily fatigue in adrenocortical


insufficiency.

i. This effect may be related to intermediary


metabolism.

b. Glucocorticoids promote muscle wasting via the protein


antianabolic effect.

7. Effects on water metabolism (involve 2 opposing actions):

1030
Adrenal Cortex 1, 2, & 3 - Dr. Witorsch

a. Increased glomerular filtration rate (GFR) which promotes


excretion of water load is a result of their vasoactive and
metabolic effects (which tend to increase solute
concentration).
b. Mineralcorticoid effect which promotes fluid retention.

8. Hematologic effects:

a. Decreased eosinophils, basophils and lymphocytes.

i. Glucocorticoids have been used in the treatment of


lymphoid-derived cancers.

b. Decreases size of lymph nodes and thymus .

i. Proliferation of some, but not all lymphocytes


inhibited.
ii. Programmed cell death ("apoptosis") of some, but
not all lymphocytes.

c. Increased neutrophils, platelets and RBC's.

9. Resistance to stress.

a. In adrenocortical insufficiency, individuals are unable to


survive even the mildest stresses.

i. Individuals collapse and may die, a situation called


"adrenal crisis"
ii. Adrenal crisis appears to result from vascular
collapse due to absence of vasoactive
corticosteroids.
iii. A rapid outpouring of stress-induced cortisol is
required for survival.

10. Effects on bone metabolism:

a. Pharmacologic or pathologically high levels of cortisol


promote bone dissolution (osteoporosis).
b. Bone dissolution is due to breakdown of the protein matrix
and glucocorticoid actions on calcium metabolism
(antagonism of Vitamin D effect on intestine and increased
renal Ca++ excretion).

11. Anti-inflammatory effect:

1031
Adrenal Cortex 1, 2, & 3 - Dr. Witorsch

a. The anti-inflammatory effect of glucocorticoids is a


pharmacologic action and of significant therapeutic value.
b. The characteristics of this effect are:

i. decreased vascular permeability


ii. decreased migration of cells
iii. decreased swelling of tissues
iv. increasing capillary resistance to infiltration.

c. The mechanism of glucocorticoid anti-inflammatory action


involve:

i. stabilization of lysosomal membranes which


inhibits autolysis of cells in response to injury, and
ii. inhibition kinin production.

12. Immunosuppressive (anti-allergic) actions of glucocorticoids:

a. This effect is related to lympholytic action of


gluococorticoids and involves inhibition of antibody
production and inhibition of histamine release after
antibody-antigen interactions.
b. Glucocorticoids are administered to patients receiving
organ transplants to reduce host rejection.
c. Glucocorticoids impair the bacteriocidal and phagocytic
potency of white blood cells.

i. Hence, they decrease the body's ability to ward off


infection and make the individual more vulnerable
to infection, even though discomfort may be
reduced.

B. Mineralocorticoids: aldosterone, DOC, corticosterone

1. Mineralocorticoids promote Na+ reabsorption in exchange for K+


and H+ in the distal tubules and collecting ducts of the kidney (as
well as the sweat glands, salivary glands and intestinal mucosa).
2. Aldosterone excess produces hypokalemic alkalosis.
3. Concomitant with Na+ retention are exchanges in intracellular and
extracellular fluid volumes.
4. Spironolactone is an aldosterone antagonist useful in the treatment
of hyperaldosteronism (and also useful as a diuretic).

C. Adrenal androgens: DHEA and derivatives.

1. Two thirds of the urinary 17-ketosteroids are adrenal in origin

1032
Adrenal Cortex 1, 2, & 3 - Dr. Witorsch

2. Adrenal androgens provide a major anabolic component of the


female (can have virilizing effects and promote protein synthesis).
3. Etiocholanolone, a 17-ketosteroid, is an A-ring reduced metabolic
product of adrenal androgen which is pyrogenic (fever producing),
presumably because it antagonizes the stabilizing action of cortisol
on lysosomal membranes.

D. Adrenal estrogens.

1. Excess adrenal estrogens can have feminizing effects.

VII. MOLECULAR MECHANISMS OF CORTICOSTEROID ACTION

A. Corticosteroids enter cells passively and bind to cytosolic or nuclear


intracellular receptors (i.e., specific, saturable, and high affinity)

1. Corticosteroid binding activates the receptor by causing its


dissociation from heat shock protein

B. Activated corticosteroid-receptor complex interacts with the genome at the


appropriate glucocorticoid response element (GRE).

C. Transcription rate of specific genes are altered affecting mRNA


production, and subsequently translation (protein synthesis).

D. Aldosterone action in kidney epithelial cell involves:

1. Increased passive diffusion of Na+ across luminal membrane into


cell.
2. Activation of the Na+-K+ ATPase on serosal membrane which
pumps Na+ into extracellular fluid.
3. Increased luminal negativity relative to cell resulting from Na
internalization promotes tubular secretion of K+ and H+.

E. Glucocorticoid vs. mineralocorticoid receptors

1. Mineralocorticoids exert their effect through Type I receptors

a. Type I receptors have an affinity for both cortisol and


aldosterone. The preference (or specificity) for
mineralocorticoids is enzymatically mediated.

i. Mineralocorticoid targets have 11ß-hdroxysteroid


dehydrogenase, an enzyme that converts cortisol to

1033
Adrenal Cortex 1, 2, & 3 - Dr. Witorsch

cortisone (cortisone has a low affinity for Type I


receptors)
ii. Because of it’s structure, aldosterone is not
transformed.by 11ß-hdroxysteroid dehydrogenase

b. A component of licorice (glycyrrhizic acid, glycyrrhetinic


acid, glycyrrhizinic acid - all the same) inhibits 11ß-
hdroxysteroid dehydrogenase.

i. Therefore, high dietary licorice (e.g., chewing


tobacco) can produce apparent mineralocorticoid
excess because cortisol is not converted to cortisone
under these circumstances.

2. Type II receptors mediate glucocorticoid effects

VIII. PHARMACOLOGY OF ADRENOCORTICAL STEROIDS (This is


traditionally MII material and is for reference only - You will not be held
responsible for this material on the exam)

A. Synthetic steroids

1. Chemical modification of the cortisol molecule tends to amplify


glucocorticoid activity and diminish mineralocorticoid activity,
among these are:

a. introduction of a double bond (Δ) in A-ring;


b. introduction of α-fluorine at position 9;
c. introduction of α-hydroxy or α-methyl at position 16;

2. These modifications produce synthetic corticosteroids such as


prednisolone, prednisone, triamcinolone, and dexamethasone
3. Altered activity is partially due to differences in receptor affinity,
prolonged biological half-lives, reduced plasma binding

B. Relative glucocorticoid and mineralocorticoid potencies of naturally


occurring and synthetic corticosteroids are listed in the following table:

1034
Adrenal Cortex 1, 2, & 3 - Dr. Witorsch

RELATIVE ACTIVITIES (in vivo)

STEROID GLUCOCORTICOID MINERALOCORTICOID

cortisol (Cpd. F) 1.0 1.0

cortisone (Cpd.E)* 0.8 0.8

corticosterone (Cpd.B) 0.3 2.0

deoxycorticosterone(DOC) 0.0 15.0

aldosterone 0.3 300-500

Δ1cortisol
5.0 0.3
(prednisolone)

Δ1-cortisone
4.0 0.2
(prednisone)

16α-hydroxy, 9α-
fluoroprednisolone 6.0 0.0
(triamcinolone)

16α-methyl, 9α-
fluoroprednisolone 30 0.0
(dexamethasone)

*Cortisone does not readily bind to the glucocorticoid receptors. However it is transformed
to cortisol by the hepatic enzyme, 11ß-hdroxysteroid dehydrogenase.

1035
Adrenal Cortex 1, 2, & 3 - Dr. Witorsch

IX. DISORDERS OF ADRENOCORTICAL FUNCTION

A. Hypercortisolism (Cushing's syndrome) (Fig. 8)

1. Cushing’s syndrome is characterized by elevated levels of cortisol


and the absence of diurnal variation in plasma cortisol.
2. Cushing syndrome results from several etiologies, as follows:

a. Primary hypercortisolism

i. Here cortisol is secreted autonomously from an


adrenal tumor (accounts for 10-15% of the cases of
Cushing's syndrome).
ii. ACTH levels are low due to negative feedback
effect of excess cortisol.

b. Secondary hypercortisolism

i. Here hypercortisolism is due ACTH hypersecretion


from the anterior pituitary (also referred to
Cushing's "Disease" and comprising about 75% of
all cases of Cushing’s Syndrome)
ii. ACTH hypersecretion can be due to

• corticotrophs in anterior pituitary (most


common cause); or
• a lesion in hypothalamus causing elevation
of CRH which would then evoke ACTH
hypersecretion.

iii. Cushing's disease also implies dysfunction of


glucocorticoid feedback mechanism. Why?
iv. ACTH hypersecretion can also result from ectopic
ACTH secretion from a nonpituitary source.
v. Bilteral adrenal hyperplasia results from excess
ACTH stimulation.

c. Iatrogenic Cushing's syndrome (physician-induced)

• Results from exogenous ACTH or corticosteroids.

d. Why would a "dexamethasone suppression test" aid in the


differential diagnosis of various etiologies of
hypercortisolism?

1036
Adrenal Cortex 1, 2, & 3 - Dr. Witorsch

Figure 8. Hypercortisolism (Cushing’s Syndrome)

3. Clinical characteristics of Cushing's syndrome

a. Symptoms resulting from protein catabolic effects of cortisol:

i. Muscle wasting.
ii. Poor wound healing.
iii. Thin skin (easy bruisability).
iv. Decreased head hair growth.
v. Centripetal fat redistribution (only if patient is well-
nourished).

• formation of stretch lines in abdomen (with


centripetal fat redistribution).
• "moon face".
• “buffalo hump" on the back.

vi. Hyperglycemia (abnormal glucose tolerance or mild


diabetes) due to cortisol effects on intermediary
metabolism.
vii. Hypertension (B.P., 180/110) due to vasoactive
glucocorticoid effects and mineralocorticoid effects of
cortisol.
viii. Facial hirsutism and acne due to the hypersecretion of
adrenal androgens and/or the conversion of cortisol to
17-ketosteroids peripherally.
ix. Virilization in females.

1037
Adrenal Cortex 1, 2, & 3 - Dr. Witorsch

x. Osteoporosis and skeletal deformities due to the


promotion of bone dissolution by cortisol.
xi. Psychological changes such as psychotic episodes and
depression which may result from high cortisol levels.
xii. Gastrointestinal difficulties such as peptic ulcers.
xiii. Poor resistance to infection due to the anti-
inflammatory and anti-allergic actions of excess
cortisol.

B. Hypocortisolism (also referred to as adrenal insufficiency) (Fig. 9)

1. Characterized by low levels of cortisol, that results from several


causes:

a. Primary hypocortisolism

i. May result from massive lesions in both adrenals


(Addison's disease), or from
ii. Deficiencies in steroidogenic enzymes.

• such enzyme deficiencies called collectively


"Congenital Adrenal Hyperplasia",
• depending on the site of the enzyme block these
disorders produce a particular array of
symptoms to be discussed below.

b. Secondary hypocortisolism

i. may result from the following:

• panhypopituitarism, or
• an isolated ACTH deficiency or,
• physician-induced (iatrogenic) after termination
of long-term corticosteroid therapy.

ii. secondary hypocortisolism involves atrophy of the


zonae fasciculata and reticularis.

1038
Adrenal Cortex 1, 2, & 3 - Dr. Witorsch

Figure 9. Hypocortisolism

2. Clinical characteristics of hypocortisolism:

a. Hypoglycemic episodes.
b. Hyperpigmentation.

i. in primary hypocortisolism (only where the pituitary is


functional).
ii. most likely due to elevated levels of ACTH (ACTH
contains homologies to MSH).

c. Hypotension (and associated dizziness, syncope, small heart


size)

i. occurs in Addison's disease where the zona glomerulosa


is non-functional (not in secondary hypocortisolism).

d. Muscle weakness and fatigue.


e. Psychological changes such as irritability and lack of
concentration.
f. Gastrointestinal difficulties - fatty stools (steatorrhea),

i. due to impaired lipid absorption

g. Lymphoid hypertrophy.
h. Adrenal crisis.

1039
Adrenal Cortex 1, 2, & 3 - Dr. Witorsch

C. Congenital Adrenal Hyperplasia (CAH) - another form of hypocortisolism


(Fig. 10)

1. Results from a relative or absolute deficiency in a steroidogenic


enzyme within the adrenal cortex.

a. Produces an impairment or elimination of cortisol secretion.


b. Glucocorticoid feedback relationship is thus disrupted causing
ACTH hypersecretion which produces:

i. bilateral adrenal hyperplasia.


ii. hypersecretion of steroids other than cortisol which
produce unique symptoms in addition to those listed
above associated with simpler forms of
hypocortisolism.

2. CAH may be manifest in utero or becomes apparent in adolescence but


rarely emerges in adult life.

Figure 10. Congenital Adrenal Hyperplasia (General Characteristics).

3. Specific enzyme deficiencies and associated specific symptoms (Fig.


11)

a. C-20 hydroxylase (cholesterol desmolase) deficiency -


congenital adrenal "lipoid" hyperplasia (Fig 11. A)

i. impaired conversion of cholesterol to pregnenolone

1040
Adrenal Cortex 1, 2, & 3 - Dr. Witorsch

ii. leads to general steroid deficiency


iii. very rare condition - about 30-40 cases
iv. occurs in utero
v. not compatible with survival

b. 3 ß-hydroxysteroid dehydrogenase (3ß-ol dehydrogenase)


deficiency (Fig. 11. B)

i. leads to impaired formation of progesterone and 17 α-


hydroxy progesterone
ii. excess ACTH and steroidogenic block results in
increased adrenocortical DHEA production
iii. excess androgen has virilizing effects - referred to as
the "adrenogenital syndrome."
iv. inability to form mineralocorticoids leads to sodium
loss and associated side effects (e.g., hypotension)

c. 21-hydroxylase deficiency (Fig. 11. C)

i. most common form of adrenogenital syndrome (


incidence 1 in 14,000)
ii. conversion of progestin to DOC and deoxycortisol is
impaired
iii. excess ACTH and block results in increased
adrenocortical androgen secretion (virilization)
iv. mineralocorticoid deficiency results in sodium loss and
associated symptoms

d. 11ß-hydroxylase deficiency (Fig 11. D)

i. a rarer form of adrenogenital syndrome


ii. conversion of DOC to corticosterone and deoxycortisol
to cortisol are impaired
iii. excess ACTH and block results in increased
adrenocortical androgen secretion ( virilization)
iv. accumulation of DOC, a potent mineralocorticoid
(hypertension along with hypokalemic alkalosis)
v. aldosterone output would be low, because of DOC-
induced expansion of fluid volume inhibiting renin-
angiotensin system.

e. 17α-hydroxylase (and 17,20 lyase) deficiency (Fig 11. E)

i. congenital adrenal hyperplasia without adrenogenital


effects

1041
Adrenal Cortex 1, 2, & 3 - Dr. Witorsch

ii. cortisol production is reduced because of the absence of


17α-hydroxylation.

• androgen production also impaired as a result of


this deficiency.

iii. pathway leading to DOC and corticosterone production


is favored, both of which possess significant
mineralocorticoid activity (hence, hypertension and
hypokalemic alkalosis)
iv. low aldosterone levels also occur due to expansion of
plasma volume and suppression of renin-angiotensin
system
v. this defect also occurs in the gonad leading to a female
phenotype at birth and male pseudohermaphroditism
vi. rare - about 50 cases reported

1042
Adrenal Cortex 1, 2, & 3 - Dr. Witorsch

Figure 11. A

1043
Adrenal Cortex 1, 2, & 3 - Dr. Witorsch

Figure 11. B

1044
Adrenal Cortex 1, 2, & 3 - Dr. Witorsch

Figure 11. C

1045
Adrenal Cortex 1, 2, & 3 - Dr. Witorsch

Figure 11. D

1046
Adrenal Cortex 1, 2, & 3 - Dr. Witorsch

Figure 11. Specific steroidogenic enzyme blocks leading to


congenital adrenal hyperplasia.

D. Hyperaldosteronism

1. Elevated plasma levels of aldosterone

a. Primary hyperaldosteronism (Conn's syndrome)

1047
Adrenal Cortex 1, 2, & 3 - Dr. Witorsch

i. due to an adrenocortical adenoma


ii. exhibits decreased levels of renin (why?)

b. Secondary hyperaldosteronism - stimulation of aldosterone


release from a source outside of the adrenal cortex

i. produced by activation of the renin-angiotensin system


due to a variety of conditions, such as:

• decreased plasma volume


• imparied renal blood flow and/or perfusion
pressure (e.g., as a result of renal
vasoconstriction).

ii. produced by primary rise in renin secretion resuting


from juxtaglomerular apparatus hyprplasia (Bartter’s
syndrome).

2. Symptoms of hyperaldosteronism, predictable on the basis of actions


of aldosterone:

a. Hypertension;
b. Hypokalemic alkalosis-due to the loss of K+ and H+ in
exchange for Na+ and accompanying weakness;
c. Polyuria resulting from nephropathy and secondary polydipsia.

X. STUDY QUESTIONS - All type A

1. Which pair of enzymes best represents the pathway for adrenal DHEA
production from delta 5-pregnenolone?

A. cholesterol desmolase, C-17,20-lyase


B. 3-beta-ol dehydrogenase:isomerase, 17-alpha hydroxylase
C. 17-alpha hydroxylase, C-17,20-lyase
D. 17-alpha hydroxylase, 3-beta-ol dehydrogenase:isomerase
E. 21-hydroxylase, 11-beta-hydroxylase

2. Which of the following hormones or treatments could produce adrenal


enlargement, hyperglycemia, and thymic involution in hypophysectomized
subjects?

A. CRH
B. ACTH
C. chronic stress
D. cortisol

1048
Adrenal Cortex 1, 2, & 3 - Dr. Witorsch

E. interleukin-1

3. Which of the following treatments would be least effective in elevating


serum cortisol levels in a subject with a deficiency in CRH?

A. mild stress
B. CRH
C. ACTH
D. cortisol
E. cortisone

4. Which of the following would be a consequence of a deficiency in


angiotensin converting enzyme (ACE)?

A. involution of the zona fasciculata


B. hypertension
C. renin deficiency
D. angiotensin II deficiency
E. aldosterone excess

5. All of the following would be a consequence of ACTH deficiency except:

A. adrenal involution
B. lymphoid hypertrophy
C. muscle wasting
D. tendency toward hypoglycemia
E. adrenal crisis

6. A deficiency in which of the following adrenocortical steroidogenic


enzymes leads to virilization and hypertension?

A. cholesterol desmolase
B. 17-alpha hydroxylase
C. 3 beta hydroxysteroid dehydrogenase
D. 21-hydroxylase
E. 11-beta hydroxylase

7. A deficiency in which of the following adrenocortical steroidogenic


enzymes leads to hypertension without virilization?

A. cholesterol desmolase
B. 17-alpha hydroxylase
C. 3 beta hydroxysteroid dehydrogenase
D. 21-hydroxylase

1049
Adrenal Cortex 1, 2, & 3 - Dr. Witorsch

E. 11-beta hydroxylase

8. A drug that selectively blocks adrenal 11 beta-hydroxylase would most


likely produce all of the following except:

A. elevated ACTH
B. enlarged adrenals
C. virilization
D. elevated aldosterone
E. hypokalemic alkalosis

9. Chronic treatment with which of the following drugs would most likely
produce adrenocortical atrophy, low cortisol levels, and little or no change
in blood pressure?

A. cortisol
B. ACTH
C. dexamethasone
D. a 17-alpha hydroxylase inhibitor
E. deoxycorticosterone

Answer Key: 1. C; 2. B; 3. A; 4. D; 5. C; 6. E; 7. B; 8. D; 9. C.

1050
Growth Hormone 1 & 2 - Dr. Kalimi

Growth Hormone, Somatostatin, and Prolactin 1 & 2


Mohammed Y. Kalimi, Ph.D.

I. Growth Hormone (somatotropin):

Growth hormone (GH) is a 191 amino acid single chain polypeptide (MW
22,000 daltons).

Growth hormone is synthesized as a preprohormone by somatotrophs of the


anterior pituitary and released in a pulsatile manner.

The half life of growth hormone in circulation is about 20 minutes.

Growth hormone has about 83% sequence homology with the human placental
lactogen hormone (hPL) and about 16% sequence homology with prolactin.

Growth hormone is species specific

Plasma level: adult 3ng/mL, child 5ng/mL.

Daily secretion in an adult human is approximately 500 micrograms.

A. Physiological action

1. Promotes linear growth (gain in height), maintains lean body mass.


2. Increases protein, RNA and DNA synthesis in most tissues (anabolic).
3. Increases fat utilization (lipolytic) and decreases carbohydrate utilization
(hyperglycemia).
4. Increases calcium and phosphate retention.
5. Growth hormone stimulates release of somatomedin ‘C’ or insulin-like
growth factor I (IGF-I), from the liver, fibroblasts, muscle etc.

a. IGF-1 with GH stimulates skeletal (cartilage) growth by stimulating


chondrocyte mitosis, collagen synthesis and matrix synthesis.
b. IGF-I production is decreased with low insulin and low dietary intake.

B. Secretion of GH:

Release of GH: - Release of GH is generally greater in women than in men,


presumably due to estrogen.

1. During slow wave sleep (stage IV of sleep)


2. Stress
3. Exercise
4. Arginine infusion

1051
Growth Hormone 1 & 2 - Dr. Kalimi

5. Hypoglycemia (fasting)
6. High serum amino acids levels
7. Cortisol and thyroid hormones (physiological concentration)
8. Neurotransmitters GABA, serotonin, acetylcholine etc.
9. Ghrelin (28 amino acid peptide), synthesized in oxyntic gland of the
stomach
10. Growth hormone-releasing hormone (GHRH)

Failure to increase GH after such stimulation is evidence of deficiency.

Inhibition of GH release by:

1. Hyperglycemia (high serum sugar levels)


2. Increase in free fatty acids
3. Cortisol (pharmacological doses)
4. Obesity
5. Somatostatin
6. Aging

Wakeful GH release:

Is great in puberty (seems to increase before sex steroids) and declines after
puberty.

Sleep-induced GH release:

Provides a major part of the diurnal quotient of growth hormone and is


maximal at puberty, diminishing progressively in later years.

C. Control of GH Secretion (Fig.1):

Two hypothalamic hormones appear to regulate the release of growth


hormone.

1. GHRH (growth hormone- releasing hormone, a polypeptide of 44 amino


acids) stimulates growth hormone release.
2. Somatostatin (a polypeptide of 14 amino acids) inhibits growth hormone
release.

IGF-I stimulates the secretion of somatostatin from hypothalamus and also


acts directly on the anterior pituitary to inhibit GH.

1052
Growth Hormone 1 & 2 - Dr. Kalimi

Figure 1: Feedback control of GH secretion. The


dashed arrows indicate inhibitory effects and the
solid arrows stimulatory. IGF-I stimulates
somatostatin secretion from hypothalamus and
acts directly on ant. pituitary to inhibit GH
secretion.

D. Mechanism of GH Action (Fig. 2):

The binding of growth hormone to the extracellular domain of its receptor


results in the dimerization of the receptor with rapid phosphorylation and
activation of Janus kinases on the cytoplasmic domain. These result in
phosphorylation of STAT, IRS, GRB2, and PLC pathways and stimulation of
gene expression.

1053
Growth Hormone 1 & 2 - Dr. Kalimi

Figure 2: GH Action

E. Growth hormone abnormalities:

GH excess (such as pituitary tumor):

1. Childhood growth hormone excess leads to gigantism characterized by


abnormally tall stature.
2. Growth hormone excess in adults leads to acromegaly. In acromegaly
patients, the head, soft tissues, and many body organs enlarge in size.
These changes lead to enlarged sinus cavities in the skull, and widened
hands and feet. Height is not increased in adults because the epiphyses of
the long bones are closed. Treated with somatostatin analogue octreotide.

Excess growth hormone can also be diabetogenic. The high levels of


blood glucose induced by excess growth hormone stimulate excessive
insulin release and can eventually lead to pancreatic β-cell “burn out”.
Kwashiorkor is a disease associated with starvation (particularly protein

1054
Growth Hormone 1 & 2 - Dr. Kalimi

deficiency) and is characterized by high growth hormone levels (possibly


due to hypoglycemia) and protein depletion.

GH Deficiency:

Growth hormone deficiency may result from damage to or malformation of


either the hypothalamus or the pituitary gland.

1. Low GH levels in adults can result in hypoglycemia.


2. In children, growth hormone deficiency leads to dwarfism, characterized
by abnormally short statue, mild obesity and delayed puberty (sexual
dysfunction), excess fat and reduced lean body mass.

State of end-organ resistance to GH (Laron dwarfism).


Mutation or gross deletions in the gene coding:

1. Growth hormone receptor (high plasma GH levels, low IGF-I levels).


2. IGF-I receptor (high plasma GH levels, high IGF-I levels).

II. Somatostatin (14 amino acid and also 28 amino acid peptide):

A. Sources of somatostatin:

Hypothalamus, D cell of pancreatic islet, GI-tract.

Somatostatin can act as an autocrine, paracrine or endocrine regulator.

B. Physiological Action:

1. Inhibits release of growth hormone, prolactin, TSH, insulin, glucagon,


gastrointestinal hormones (gastrin, secretin, motilin, vasoactive
intestinal peptide, cholecystokinin).
2. Decreases GI blood flow and motility, and inhibits secretion of gastric
acid and pepsin.
3. Increases GI water and electrolyte absorption.

C. Mechanism of action of somatostatin:

Somatostatin binds to a plasma membrane receptor and decreases intracellular


cAMP and calcium levels.

↑ somatostatin, ↓ intracellular calcium, ↓ cAMP and blockade of


potassium channel.

Half-life of somatostatin in blood is approximately one minute.

1055
Growth Hormone 1 & 2 - Dr. Kalimi

This leads to the development of analogs with prolonged duration and potency
of action such as Octreotide.

In clinical studies, octreotide is effective in reducing the hormonal


hypersecretion of acromegaly, insulinoma, TSH and VIP secreting adenoma.

III. Prolactin:

• Polypeptide of 199 amino acids (MW 23,000 daltons).


• Prolactin is produced by mammotrophs of the anterior pituitary as a
preprohormone.
• The circulatory half life of prolactin is 30-50 min.

A. Physiological action of prolactin:

1. Initiates and maintains milk production (stimulates synthesis of milk


proteins casein and lactoalbumin), development of breast.
2. Influences immune responses and reproductive function.

• Levels are higher in women (and during pregnancy) than men: a direct
consequence of estrogen.

B. Stimuli which enhance prolactin secretion:

1. TRH
2. Nursing-suckling in the post-partum period leads to PRL secretion.
3. Sleep
4. Opioids, serotonin
5. Estrogen
6. Stress
7. Pregnancy

C. Inhibition of prolactin release:

1. Bromocriptine mesylate, specific therapy for female infertility associated


with hyperprolactinemia in the absence of a demonstrable pituitary tumor.
2. Dopamine
3. GABA
4. Somatostatin

D. Prolactin Regulation (Fig.3):

1. The predominant effect of the hypothalamus is to suppress synthesis and


release of prolactin by secreting dopamine and somatostatin.
2. Hypothalamic thyrotropin hormone (TRH) promotes prolactin release.
TRH also increases TSH release as described in thyroid hormone lecture.

1056
Growth Hormone 1 & 2 - Dr. Kalimi

Figure 3: Regulators of prolactin


secretion (----negative,—— positive).

E. Mechanism of prolactin action:

The cell membrane receptor for prolactin which is, very similar in structure
and function to growth hormone receptor has been identified in adrenal,
breast, ovary, liver, prostate, and immune cells.

Prolactin binds to plasma membrane receptors and activates cytoplasmic


Janus kinases. These phosphorylate STAT proteins that stimulate expression
of genes such as milk proteins casein, lactoalbumin etc.

F. Abnormality of prolactin secretion :

Prolactin deficiency leads to decreased milk production.

Prolactin excess (such as pituitary tumors) can result in the following:

1. May produce impotency and impair testicular function in males (↑


plasma prolactin, ↓ testosterone secretion, ↓ sperm production).
2. Galactorrhea and amenorrhea (depression of secretion of
gonadotrophin releasing hormone (GnRH), thereby impeding secretion
of luteinizing hormone (LH) and (FSH) or by directly blocking the
action of FSH and LH at the ovaries) in women.

Special note:

Estrogen can decrease PRL-binding in breasts (↑ estrogen, ↓ prolactin


receptor in breasts), but increase PRL-binding in liver.

1057
Physiology of PTH & Vitamin D - Dr. Costanzo

Physiology of PTH and Vitamin D/Ca and P --1 and 2


Linda Costanzo, Ph.D.

OBJECTIVES:

After studying this lecture, the student should understand:

1. The forms of calcium in plasma.


2. The effect of acid-base disorders on ionized calcium concentration.
3. The components of overall calcium balance.
4. The regulation of PTH secretion.
5. The actions and mechanisms of action of PTH on bone, kidney, and intestine.
6. Calcium handling along the nephron and the effects of parathyroid hormone, loop
diuretics, and thiazide diuretics.
7. The pathophysiology of primary hyperparathyroidism, secondary
hyperparathyroidism, hypoparathyroidism, and pseudohypoparathyroidism.
8. Vitamin D metabolism and the regulation of 1α-hydroxylase.
9. The actions and mechanisms of 1,25 dihydroxycholecalciferol on intestine, bone,
and kidney.
10. The pathophysiology renal osteodystrophy.

OPTIONAL READING:

Physiology 3rd edition, Costanzo; W.B. Saunders, 2006. Chapter 9, pps. 428-437; Chapter
6, pps. 282-285.

I. OVERVIEW OF Ca HOMEOSTASIS

Virtually every physiological function is affected by changes in extracellular or


intracellular calcium concentration. Consider its familiar role in excitation-
contraction coupling, neurotransmission, excitation-secretion coupling (exocrine
and endocrine) and blood clotting.

The concentration of calcium in extracellular fluid is tightly controlled by the


complicated interplay of three hormonal systems: parathyroid hormone (PTH),
calcitonin, and vitamin D.

A. Forms of Ca in plasma

The total Ca in plasma comes in three forms: (1) bound to plasma


proteins, (2) complexed to anions, and (3) ionized.

1058
Physiology of PTH & Vitamin D - Dr. Costanzo

Form % mM mEg/L mg/dL


Total Ca 100 2.5 5.0 10.0
Protein-bound 40 1.0 2.0 4.0
Complexed 10 0.25 0.5 1.0
Ionized 50 1.25 2.5 5.0

40% of total calcium is protein-bound, mainly to plasma albumin. 10% is


complexed to anions in plasma, primarily HCO-3, phosphate and citrate.
50% is free, ionized Ca+2. Free, ionized Ca+2 is the biologically active
form.

Changes in plasma protein concentration may alter total Ca


concentration without altering the ionized [Ca+2]. On the other hand,
changes in the plasma concentration of a complexing anion like
phosphate or citrate may have a profound effect on ionized [Ca+2] with
serious physiologic consequences; e.g. increasing plasma phosphate will
lower ionized [Ca+2]. Acid-base disturbances may influence the fraction
of total Ca which is protein-bound. In acidemia, H+ will displace Ca+2 on
plasma proteins, increasing the free ionized Ca+2; alkalemia will decrease
the free ionized Ca+2. Thus, measurements of total Ca may not give an
accurate reflection of ionized [Ca+2].

An elevation of plasma ionized Ca+2 is hypercalcemia. Symptoms of


hypercalcemia are neurologic (lethargy, coma, weakness, hyporeflexia),
gastrointestinal (constipation) and renal (polyuria and polydipsia due to
ADH resistance).

A decrease in plasma ionized Ca+2 is hypocalcemia. Symptoms of


hypocalcemia are dramatic and include spontaneous muscle cramps in
mild hypocalcemia to tetany and clonic seizures in severe hypocalcemia.
Classically, tapping with a finger over the supramandibular portion of the
parotid gland causes spasm in the muscles innervated by the facial nerve
(twitches in upper lip)--Chvostek's sign. More specific is the Trousseau
sign where inflation of a blood pressure cuff around the upper arm evokes
carpopedal spasm.

Intracellular ionized Ca+2 is very low, about 10-7 M, maintained by a


cell membrane Ca-pump and Ca-Na exchange, and uptake and release of
Ca from intracellular organelles.

B. Overall Ca homeostasis

To maintain perfect Ca+2 balance, the amount of Ca entering the body


must equal the amount of Ca leaving the body. Consider the next diagram
which shows typical daily Ca balance in an adult human being.

1059
Physiology of PTH & Vitamin D - Dr. Costanzo

Figure 1.

The daily American diet includes about 1000 mg of elemental Ca, the
amount in one quart of milk or in 5 regular strength Tums. At this intake
level, 35% or 350 mg is absorbed from the gastrointestinal tract. About
150 mg is secreted back into the intestinal lumen, so net Ca absorption is
200 mg. Therefore 800 mg is excreted in the feces. To maintain perfect Ca
balance, net excretion of Ca in urine must equal net absorption from the
gastrointestinal tract. The daily filtered Ca load is the product of
ultrafilterable [Ca] x GFR. Ultrafilterable [Ca] is the non-protein bound
moiety, or 6 mg/dL; GFR is 125 ml/min. Thus filtered Ca load = 6mg/dL
x 125 ml/min x 1440 min/day or 10,800 mg/day. Of this, 10,600 mg/day,
or 98% of the filtered load, are reabsorbed by the renal tubules, leaving
200 mg/day to be excreted in the urine.

The Ca in extracellular fluid is in equilibrium with a readily exchangeable


Ca pool on the surface of bone. Daily, as much as 500 mg of Ca leaves
bone and enters ECF by bone resorption and 500 mg leaves ECF and is
deposited in newly formed bone; this process is bone remodeling.

The above figures describe the adult in perfect Ca balance. Naturally, the
growing child is in a state of positive Ca balance, whereby more Ca is
absorbed from intestine than is excreted in the urine, the difference being
deposited in growing bone. Conversely, a lactating female may be in

1060
Physiology of PTH & Vitamin D - Dr. Costanzo

negative Ca balance if intestinal Ca absorption is less than the sum of Ca


lost in urine and in breast milk (350 mg Ca/Liter); in this case, net Ca is
lost from maternal bone to make up the difference.

II. PARATHYROID HORMONE

The plasma Ca+2 concentration is primarily regulated by the actions of


parathyroid hormone. PTH increases the plasma Ca concentration by
coordinated actions on bone and kidney and an indirect action on intestine.

A. Synthesis and secretion of PTH

PTH is an 84-amino acid single chain protein with molecular weight of


9000 daltons.

Figure 2.

It is synthesized as follows:

1061
Physiology of PTH & Vitamin D - Dr. Costanzo

The peptide chain for pre-pro-PTH is synthesized on the ribosome where


25 amino acids are cleaved from the N-terminus, leaving pro-PTH. Pro-
PTH is transported to the Golgi apparatus where another 6 amino acids are
cleaved to form PTH. The PTH is stored in secretory granules for eventual
release by exocytosis. Some of the PTH synthesized is degraded in the
parathyroid glands rather than being stored in exocytic vesicles.

The most important regulator of PTH secretion is the plasma Ca+2


concentration. Low plasma Ca+2 stimulates PTH secretion; high plasma
Ca+2 inhibits secretion. Maximal secretory rates are achieved when total
plasma [Ca] falls below 7 mg/dL; as plasma [Ca] rises, PTH secretion
falls. Above plasma concentrations of 11 mg/dL there is a basal rate of
PTH secretion which cannot be suppressed by further increases in plasma
Ca.

Figure 3.

Alterations in PTH secretion in response to changes in plasma Ca


occur within minutes. Parathyroid cell membranes contain Ca2+-sensing
receptors linked to phospholipase C via a Gq protein. When extracellular
(plasma) Ca2+ is increased, Ca2+ binds to the receptor, activates
phospholipase C, increases intracellular IP3/Ca2+, and inhibits PTH
secretion. Conversely, when extracellular (plasma) Ca2+ is decreased, less
Ca2+ binds to the receptor, which inhibits phospholipase C, decreases
IP3/Ca2+ and stimulates PTH secretion.

Plasma Ca+2 levels also regulate PTH synthesis and degradation.


Long-term hypocalcemia stimulates transcriptional events, increases PTH
stores and eventually causes parathyroid hyperplasia. Prolonged
hypercalcemia inhibits transcriptional events and reduces glandular PTH
stores.

1062
Physiology of PTH & Vitamin D - Dr. Costanzo

Changes in plasma Mg+2 mimic the effects of Ca. Low Mg+2 stimulates
PTH secretion; high Mg+2 inhibits. Paradoxically, severe prolonged
hypomagnesemia inhibits PTH secretion.

B. PTH Actions

The actions of PTH are detectable within minutes to hours and are
coordinated to increase plasma calcium concentration and decrease
plasma phosphate concentration. They are:

1. PTH actions on bone. PTH has both direct and indirect actions on
bone. PTH receptors are located on osteoblasts, not on osteoclasts.
(1) Initially and transiently, PTH acts directly on osteoblasts to cause
an increase in bone formation. (This short-lived action is the basis for
using intermittent PTH injections to treat osteoporosis.) (2) In a
second, indirect action, PTH causes a long-lasting increase in bone
resorption. In this second action, PTH binds to its receptors on the
osteoblasts, cytokines are released from osteoblasts, which stimulate
bone resorption in osteoclasts via a paracrine action.

The overall effect of PTH is to promote bone resorption, delivering


Ca2+ and phosphate from mineralized bone into extracellular fluid. As
bone is resorbed, hydroxyproline is released from bone matrix and
excreted in the urine.

2. PTH actions on kidney. Two segments of the nephron have PTH-


sensitive adenylate cyclase: PTH increases cAMP production in
proximal tubules and distal tubules. One of the earliest detectable
effects of PTH is increased (nephrogenous) urinary cyclic AMP.

1063
Physiology of PTH & Vitamin D - Dr. Costanzo

Figure 4.

Phosphate is freely filtered at the glomerulus. 75% of the filtered


phosphate is reabsorbed in the proximal tubule, 15% is reabsorbed in
the loop of Henle, none in the distal tubule or collecting duct. In
proximal tubule, phosphate is one of several substances co-transported
with Na across the luminal membrane. The Na gradient across the
luminal membrane is maintained by the Na-K pump at the basolateral
membrane. PTH, as shown below, via activation of adenylate cyclase
at the basolateral membrane and elevation of intracellular cAMP
inhibits the Na-phosphate cotransporter. Because little phosphate is
reabsorbed after the proximal tubule, phosphaturia results. PTH also
inhibits Na, Ca, HCO-3 and fluid absorption in proximal tubule.

The cellular actions of PTH are initiated by binding of hormone to


the basolateral cell membrane receptors which are coupled to

1064
Physiology of PTH & Vitamin D - Dr. Costanzo

adenylate cyclase by a guanine nucleotide-dependent regulatory


protein (Gs). Activation of adenylate cyclase causes elevation of
intracellular cyclic AMP, followed by activation of a protein kinase
which phosphorylates a luminal membrane protein, inhibiting the Na-
phosphate cotransporter. The cyclic AMP generated as a second
messenger is transporter across the luminal membrane and excreted in
the urine (urinary cyclic AMP).

Figure 5.

1065
Physiology of PTH & Vitamin D - Dr. Costanzo

Figure 6.

60% of total plasma Ca is ultrafilterable, the ionized and the


complexed. Of the Ca filtered, 67% is reabsorbed in proximal
tubule. Proximal Ca reabsorption is tightly coupled to Na and fluid
reabsorption and is mostly passive. Paradoxically, PTH inhibits
proximal Ca reabsorption because it inhibits Na and fluid reabsorption.
The loop of Henle reabsorbs 25% of the filtered Ca, again by a
passive mechanism linked to Na reabsorption. In contrast, the distal
tubule actively reabsorbs 8% of the filtered Ca load. Quantitatively
this amount may seem insignificant. However, the distal tubule is the
last nephron segment to reabsorb Ca so it determines exactly how
much Ca will be excreted in the urine. Distal tubule Ca reabsorption
is increased by PTH. (This regulatory action of PTH on terminal
nephron Ca reabsorption is analogous to the regulation of Na
reabsorption by aldosterone and of water reabsorption by ADH in the
terminal portions of the nephron.)

1066
Physiology of PTH & Vitamin D - Dr. Costanzo

Diuretics have important effects on nephron Ca reabsorption.


(1) Loop diuretics (furosemide) cause calciuresis. They inhibit Ca
reabsorption because Ca is reabsorbed passively along with Na in the
Loop; when Na reabsorption is inhibited, by furosemide, so is Ca
reabsorption. This important action of loop diuretics forms the basis
for their usefulness in treating severe, life-threatening hypercalcemia.
(2) Distal diuretics (either thiazides or K-sparing) are totally
different from loop diuretics. They produce hypocalciuria, or lowering
of urinary Ca excretion. They do so by increasing distal tubule Ca
reabsorption while simultaneously inhibiting distal Na reabsorption.
The hypocalciuric action of thiazides makes them useful in the medical
management of hypercalciuric stone-formers.

3. PTH action on intestine. PTH increases intestinal Ca absorption


indirectly by activating the 1 α-hydroxylase enzyme in kidney to
produce more 1,25 (OH)2 vitamin D3. (See below for details).

4. Summary of PTH actions.

Figure 7.

1067
Physiology of PTH & Vitamin D - Dr. Costanzo

To summarize PTH actions, increased bone resorption provides Ca


and phosphate to extracellular fluid. Alone, the bone effect would not
elevate plasma Ca+2, because of complexation of Ca+2 with
phosphate. Importantly, the coordinated action of PTH on kidney to
inhibit phosphate reabsorption, causing phosphaturia and elimination
of the phosphate resorbed from bone, a consequent fall in plasma
phosphate, allows plasma Ca+2 to rise. Also, PTH increases renal
tubular Ca+2 reabsorption providing more Ca to the extracellular fluid
and contributing to the rise in plasma Ca+2 concentration.

C. PTH - Pathophysiology

1. Hyperparathyroidism is characterized by high circulating levels of


PTH, hypercalcemia and hypophosphatemia. The high blood Ca+2
results from increased rates of bone resorption and increased renal
reabsorption of Ca+2. The low blood phosphate results from decreased
renal reabsorption of phosphate (phosphaturia). 1°
hyperparathyroidism may be caused by parathyroid adenomas. 2°
hyperparathyroidism results from vitamin D deficiency or chronic
renal failure.

2. Hypoparathyroidism is associated with low circulating PTH,


hypocalcemia and hyperphosphatemia. Causes are congenital or
surgical.

3. Pseudohypoparathyroidism Type Ia (Albright's hereditary


osteodystrophy) is a genetically transmitted disorder (autosomal
dominant) associated with hypocalcemia and hyperphosphatemia but,
paradoxically, high circulating levels of PTH. The kidneys and bone
are resistant to PTH. The characteristic appearance is of short stature,
round face, short neck and short metacarpals and metatarsals.
Typically, PTH infusion does not increase plasma Ca, urinary
phosphate or urinary cyclic AMP, as it would in a patient with
hypoparathyroidism. The biochemical defect in the end organs has
been identified in pseudohypoparathyroidism; patients are deficient
in the guanine nucleotide regulatory protein (Gs) which couples the
PTH receptor to adenylate cyclase; thus cyclic AMP production and
the subsequent physiologic actions of PTH on kidney and bone are
blocked. Actually, patients often have multiple hormone resistance due
to defective Gs coupling protein in several hormone systems.

Pseudohypoparathyroidism Types Ib, Ic and II. Sub-populations


have variants of the disease as follows: Type Ib (hypocalcemia,
hyperphosphatemia, no Albright's , normal Gs, PTH receptor defect);

1068
Physiology of PTH & Vitamin D - Dr. Costanzo

Type Ic (hypocalcemia, hyperphosphatemia, with Albright's, normal


Gs, defect is in catalytic unit of adenyl cyclase; Type II
(hypocalcemia, hyperphosphatemia, normal Gs, normal urinary cAMP,
no phosphaturic response to PTH, defect is distal to cAMP.

Pseudopseudohypoparathyroidism occurs in a sub-population with


Albright's osteodystrophy but normal serum Ca and P and no
identifiable biochemical defects. These findings suggest that the
manifestations of Albright's do not result from hypocalcemia,
hyperphosphatemia or Gs deficiency, but are a separate, related genetic
disorder.

III. VITAMIN D

In children, deficiency of vitamin D causes rickets where bone mineralization at


the epiphyseal plates is compromised, producing characteristic growth failure and
deformities. In adults, vitamin D deficiency causes osteomalacia, with softening
and bending of long bones. Our advances in understanding the metabolism and
actions of vitamin D have provided one of the most dramatic examples of basic
science applied to clinical treatment.

A. Vitamin D metabolism
There are two sources of vitamin D in humans: (1) vitamin D3 in the skin
from UV irradiation of
7-dehydrocholesterol and (2) vitamin D2 (differing only by the addition of
a double bond between C-21 and C-22) from ergosterol in the diet. If
human beings are exposed to the ultraviolet rays of the sun, then dietary
vitamin D is unnecessary. I shall consider only vitamin D3 or
cholecalciferol.

1069
Physiology of PTH & Vitamin D - Dr. Costanzo

Figure 8.

Vitamin D3 circulates to the liver where it is hydroxylated to 25 (OH)-


cholecalciferol. The hydroxylation occurs in the endoplasmic reticulum,
requires NADPH, O2 and Mg+2, but not cytochrome P450. 25 (OH)-
cholecalciferol is the principal circulating form of vitamin D bound to
an a-globulin carrier protein.

In nephrectomized animals, neither vitamin D3 nor 25 (OH)-


cholecalciferol is active. This observation led DeLuca and others in the
early 1970's to the significant finding that the active metabolite of
vitamin D3 is produced in the kidney.
25 (OH)-cholecalciferol circulates to the kidney and is hydroxylated in
proximal tubule cells.

1,25 (OH)2-cholecalciferol is the active form of vitamin D.


Hydroxylation at C-1 occurs in renal mitochondria, catalyzed by a 1 α-
hydroxylase enzyme requiring NADPH, O2, Mg+2 and cytochrome P450.
When inadequate amounts of Ca are being absorbed from the GI
tract, 1,25 (OH)2-cholecalciferol is the preferred route of metabolism.

24,25 (OH)2-cholecalciferol is the inactive metabolite. This is the


preferred route of metabolism when adequate amounts of Ca are
being absorbed from the GI tract, i.e. when there is vitamin D
sufficiency.

Figure 9.

Hydroxylation of 25 (OH)-cholecalciferol at C-1 or C-24 is reciprocally


related. When Ca conservation is necessary, 1-hydroxylation
predominates; when Ca is sufficient, then 24-hydroxylation predominates.

1070
Physiology of PTH & Vitamin D - Dr. Costanzo

B. Regulation of vitamin D metabolism

Control of vitamin D metabolism is exerted in the kidney at the level


of the 1 α-hydroxylase enzyme. As already shown, when plasma Ca+2 is
low and there is a Ca deficit, 1 α-hydroxylation is stimulated; conversely,
when plasma Ca+2 is normal or high, then 1 α-hydroxylation is suppressed.

Regulation occurs by three potential mechanisms. The three mechanisms


are related to one another and may represent a single mechanism:

1. ↓ Plasma [Ca+2] increases 1 α-hydroxylase, stimulating production


of 1,25 (OH)2-cholecalciferol. 1,25 (OH)2-cholecalciferol will, in turn,
increase intestinal Ca+2 absorption to help restore plasma [Ca+2] back
to normal. When plasma [Ca+2] is normal or too high, 1 α-hydroxylase
is inhibited and the inactive metabolite is produced instead.

2. ↑ PTH levels increase 1 α-hydroxylase. When plasma [Ca+2] is low


(see above) then circulating levels of PTH rise. Increased PTH levels
stimulate 1 α-hydroxylase and production of the active metabolite. It is
probable that lowering plasma [Ca+2] does not directly stimulate 1 α-
hydroxylase but that the effect is mediated via increases in circulating
PTH.

3. ↓ plasma [phosphate] increases 1 α-hydroxylase. Experimentally, in


the absence of the parathyroid glands and when plasma [Ca+2] is
normal, lowering plasma [phosphate] stimulates 1 α-hydroxylase
directly. It is possible that the effect of PTH (above) is not a direct one,
but is secondary to the ↓ in plasma [phosphate] which PTH produces
by its phosphaturic action. In addition, because PTH inhibits
phosphate reabsorption in proximal tubule cells
(the same cells that make 1,25 (OH)2-cholecalciferol) one would
expect that PTH decreases [phosphate] inside the proximal cell,
stimulating the 1 a-hydroxylase enzyme.

4. Prolactin and growth hormone may increase 1α-hydroxylase activity


to provide more Ca during growth or lactation; direct evidence in
humans in lacking.

C. Actions of 1,25 (OH)2-cholecalciferol

The actions of 1,25 (OH)2-cholecalciferol are coordinated to increase plasma


Ca and phosphate concentrations to promote bone mineralization. They
are:

1071
Physiology of PTH & Vitamin D - Dr. Costanzo

The cellular actions of 1,25 (OH)2-cholecalciferol are detectable after a


minimum time lag of 12 hours. (Contrast to minutes for PTH actions.) 1,25
(OH)2-cholecalciferol acts on its target cells through mechanisms common to
steroid hormones. 1,25 (OH)2-cholecalciferol combines with a cytosolic
receptor and the hormone-receptor complex enters the nucleus where it
stimulates transcription of mRNA which is translated into a new protein. In
intestinal mucosa and renal distal tubule this is the cytosolic 10,000 dalton
vitamin D-dependent calcium binding protein (CaBP), or calbind in D-28K.

1. 1,25 (OH)2-cholecalciferol action on intestine

Figure 10.

1,25 (OH)2-cholecalciferol stimulates Ca absorption by production of


vitamin D-dependent CaBP. Intestinal CaBP is absent in vitamin D-
deficiency. CaBP probably acts as an intracellular Ca chelator or buffer,
keeping intracellular Ca+2 concentration low and thereby maintaining the
favorable electrochemical gradient for Ca diffusion across the brush
border membrane into the cell interior. CaBP may also facilitate the
diffusion of Ca through the cell to the basolateral membrane where it is
actively transported into blood via Ca-dependent ATPase. 1,25 (OH)2-
cholecalciferol also stimulates the intestinal Ca-ATPase.

On a low Ca diet, the intestine adapts to increase the fraction of ingested


Ca absorbed. This adaptation occurs via vitamin D, whereby low Ca diet

1072
Physiology of PTH & Vitamin D - Dr. Costanzo

→ decreases plasma Ca → increases PTH → increases production of


1,25... → increases intestinal Ca absorption.

1,25 (OH)2-cholecalciferol increases active phosphate absorption, but little


is known about the mechanism.

2. 1,25 (OH)2-cholecalciferol actions on kidney. The hormone stimulates


reabsorption of filtered Ca and phosphate. Quantitatively, these actions are
much less important than the parallel actions on intestine. In kidney, as in
intestine, vitamin D-dependent CaBP is only present in vitamin D-
sufficiency. The CaBP has been localized to the distal tubule, suggesting
is the site where 1,25 (OH)2-cholecalciferol stimulates Ca reabsorption.

3. 1,25 (OH)2-cholecalciferol actions on bone. 1,25 (OH)2-cholecalciferol


stimulates bone resorption, an effect requiring new protein synthesis. This
effect of 1,25 (OH)2-cholecalciferol is synergistic (cooperative) with the
bone-resorbing action of PTH.

To summarize the actions of vitamin D, increased intestinal absorption and


increased renal reabsorption of Ca and phosphate and increased bone resorption
all work to provide more Ca and phosphate to extracellular fluid. Adequate Ca
and phosphate in extracellular fluid is necessary for normal bone mineralization to
proceed. Conversely, normal bone mineralization cannot proceed when there is a
deficit of Ca and phosphate due to 1,25 (OH)2-cholecalciferol deficiency (rickets
or osteomalacia).

IV. CALCITONIN
A. Synthesis and secretion.

Calcitonin is a 32 amino acid peptide synthesized and secreted by C


(parafollicular) cells of the thyroid. Its synthesis proceeds through a pre-
pro to a pro hormone to calcitonin.

The stimuli for secretion of calcitonin are:


1. Elevation of plasma [Ca+2]. Increases in plasma Ca cause
circulating calcitonin levels to rise 2 to 10 fold. Likewise,
decreases in plasma calcium cause a fall. Ca modulates calcitonin
secretion via a cAMP-dependent mechanism in the C cell.

2. Ingestion of food, via gastrin, causes elevation of circulating


calcitonin levels.

B. Calcitonin actions

Calcitonin's major action is to inhibit bone resorption bringing


plasma [Ca+2] down towards normal. The physiologic or

1073
Physiology of PTH & Vitamin D - Dr. Costanzo

pathophysiologic relevance of calcitonin's effect on bone or role in Ca


homeostasis is debatable because: (1) the inhibition of bone resorption is
only transitory, (2) plasma [Ca] is normal after total thyroidectomy and (3)
high circulating levels of calcitonin, such as occurs in medullary
carcinoma of the thyroid do not lower plasma Ca or affect bone density.

V. CASE STUDY OF RENAL OSTEODYSTROPHY

A 30-y.o. female with advanced renal failure was receiving peritoneal dialysis
while awaiting transplantation. She was admitted to the hospital for evaluation
because in the preceding month she had experienced severe bone pain and
pruritus (itching). Upon admission, blood values were as follows (compared to
normal):

Radiologic examination revealed increased bone resorption, osteomalacia and


soft-tissue calcification.

COMMENTS. Her chronic renal disease with decreased renal mass caused
hyperphosphatemia due to decreased GFR, decreased phosphate filtered and
phosphate retention. The decreased renal mass also cause decreased production
of 1,25 (OH)2-cholecalciferol. (The hyperphosphatemia would also contribute by
inhibiting production of 1,25 in whatever renal tissue is left.) Thus, circulating
levels of active vitamin D are decreased which decreases intestinal Ca
absorption, causing hypocalcemia. (The hyperphosphatemia also decreases Ca+2
by complexing ionized
Ca+2). Hypocalcemia causes secondary hyperparathyroidism; other factors
contributing to the secondary hyperparathryoidism are skeletal resistance to PTH
and decreased renal degradation of PTH (kidney is major site of PTH catabolism).

Thus, the chronic renal disease caused phosphate retention and decreased 1,25
production. Hyperphosphatemia and decreased 1,25 caused hypocalcemia.
Hypocalcemia, decreased renal degradation of PTH and skeletal resistance to
PTH caused secondary hyperparathyroidism.

The bone pain was caused by increased bone resorption due to excess PTH and by
osteomalacia due to decreased 1,25. Calcification and pruritus was due to
deposition of Ca-P salts in skin and soft tissues.

Appropriate treatment should include restriction of dietary phosphate and


intestinal phosphate binders (Al gels) to offset hyperphosphatemia; Ca
supplementation to correct negative Ca balance; treatment with 1,25 (OH)2-

1074
Physiology of PTH & Vitamin D - Dr. Costanzo

cholecalciferol to bypass renal defect in its synthesis. These measures should


reduce secondary hyperparathyroidism; if not, parathyroidectomy may be
necessary.

VI. SAMPLE QUESTIONS AND ANSWERS FOR PTH-VITAMIN D


1. Circulating PTH levels will be elevated:
A. in primary hyperparathyroidism.
B. if serum Ca+2 is lowered by a Ca-chelating agent.
C. in chronic renal failure.
D. in vitamin D intoxication.

Ans. = A

A. Right. The primary problem is over-secretion of PTH.


B. Right. The major stimulus for PTH secretion is decreased serum
Ca+2.
C. Right. Chronic renal failure causes hyperphosphatemia, causing
hypocalcemia, causing increased PTH. Renal failure also
decreases production of 1,25 which causes hypocalcemia and
increased PTH.
D. Wrong. Increased vitamin D leads to increased gut Ca
absorption; the increased plasma Ca shuts off PTH secretion.

2. On a low Ca diet, which of the following will be increased?


A. calcitonin levels
B. fraction of the ingested Ca absorbed
C. 24,25 (OH)2-cholecalciferol levels
D. 1,25 (OH)2-cholecalciferol levels

Ans. = C

A. Wrong. If anything, serum Ca would be lowered, shutting off


calcitonin.
B. Right. See #3 and 4. ↑ 1,25 levels will ↑ the fraction of Ca
absorbed from gut.

1075
Physiology of PTH & Vitamin D - Dr. Costanzo

C. Wrong. Serum Ca will be lowered, so turns off synthesis of


24,25; turn on synthesis of 1,25.
D. Right. See #3.

3. A patient presents with the following:

↑ bone resorption
↑ serum Ca+2
↓ serum phosphate
↑ urinary cyclic AMP

Which of the following is the likely diagnosis?


A. Primary hyperparathyroidism.
B. Congenital hypoparathyroidism
C. Vitamin D intoxication
D. Pseudohypoparathyroidism
E. Chronic renal failure

Ans. = A

A. Right. ↑ PTH levels will ↑ bone resorption, increasing serum Ca.


Serum phosphate decreases because PTH inhibits renal
phosphate reabsorption. Urinary cyclic AMP increases because
PTH stimulates the renal adenyl cyclase.
B. Wrong. Hypoparathryoidism would have the inverse of A, due to
lack of PTH.
C. Wrong. Vitamin D intoxication would cause increased gut Ca
absorption and increased bone resorption, increasing both serum
Ca and P. PTH secretion would be turned off, so urinary cyclic
AMP would decrease.
D. Wrong. Bone and kidney would be resistant to PTH. So
hypocalcemia and hyperphosphatemia. Urinary cyclic AMP low
because PTH doesn't activate renal adenyl cyclase (no Gs
protein).
E. Wrong. Does cause hyperphosphatemia, but hypocalcemia
results from high phosphate and ↓ 1,25 production. Would see ↑
bone resorption. Not sure about urinary cyclic AMP; high PTH
should increase it, but renal tissue is diseased, so might not.
Distinction here is the hypocalcemia.

1076
Male Reproduction 1, 2, & 3 - Dr. Witorsch

Male Reproduction 1, 2, & 3


R.J. Witorsch, Ph.D.

OBJECTIVES:

At the end of this block of lectures, the student should be able to:

1. Describe the basic elements and mechanisms of the hypothalamo-pituitary-


gonadal axis common to both genders.
2. Describe the functional anatomy of the testes, particularly the relationship
between the Leydig cells, Sertoli cells and the germ cells.
3. Describe the principal events involved in spermatogenesis.
4. Describe the functions of the male sex accessory organs.
5. Describe the physiological events involved in erection and ejaculation.
6. Describe the principal components of the sperm cell and their functions.
7. Describe the events involved in the secretion, circulation, and biological half-life
of testosterone in the male.
8. Describe the physiological actions and mechanisms of action of testosterone.
9. Describe the mechanisms involved in the regulation of the hypothalamo-pituitary-
testicular axis.
10. Explain the etiologies and symptomologies of the following male gonadal
disorders:

a. hypogonadism
b. precocious puberty
c. testicular feminization
d. 5-α reductase deficiency.

11. Describe and explain the various strategies involved in male contraception.

Suggested Reading: Costanzo 3rd Edition, pp. 441-450

I. INTRODUCTION TO REPRODUCTIVE ENDOCRINOLOGY

A. Elements of the hypothalamo-pituitary-gonadal axis that are common to


both sexes (Fig. 1):

1077
Male Reproduction 1, 2, & 3 - Dr. Witorsch

Figure 1. Elements of the Hypothalamo-Pituitary-Gonadal Axis Common the Both Sexes

1. Hypothalamus:

a. The source of gonadotrophin releasing hormone (GnRH or


LHRH)

i. a decapeptide that stimulates gonadotrophic hormone


release from the anterior pituitary gland.
ii. GnRH is transported to the anterior pituitary via the
hypothalamo-hypophyseal portal system.
iii. GnRH is released in pulsatile fashion (approximately
every 60-120 minutes).

• a "pulse generator" appears to reside in the


hypothalamus.

2. Anterior pituitary gland:

a. the source of the two gonadotrophic hormones, namely:

i. Follicle stimulating hormone (FSH);


ii. Luteinizing hormone (LH).

b. LH and FSH

1078
Male Reproduction 1, 2, & 3 - Dr. Witorsch

i. LH and FSH are glycoproteins having a molecular


weight of about 30,000-34,000 daltons.
ii. Both hormones are produced in basophilic
gonadotrophic cells.
iii. LH and FSH are chemically related to each other, as
well as to TSH and a placental hormone, human
chorionic gonadotrophin (hCG).

• Hormones share a common α-subunit, but each


have a distinct β-subunit.

iv. LH and FSH are released in a pulsatile fashion in


response to pulsatile release of GnRH.

3. Gonads (testis-male; ovary-female):

a. The gonads have two major functions which are


compartmentalized, namely, steroidogenesis and
gametogenesis.
b. Steroidogenesis in both gonads is stimulated by LH (Fig. 2).

i. LH acts by regulating the enzyme cholesterol


desmolase and is cAMP mediated, analogous to ACTH
in adrenal cortex
ii. Pathways of steroidogenesis are similar in both gonads,

• testosterone production predominates in the


testes (Fig. 2A)
• estradiol and progesterone production
predominate in the ovaries (Fig. 2B)

c. Gametogenesis (spermatogenesis, male; oogenesis-female) is


stimulated both by FSH and the local action of gonadal
steroids.

Figure 2. Gonadal Steroid Production by


Testes (A) and Ovary (B)

1079
Male Reproduction 1, 2, & 3 - Dr. Witorsch

4. Sex steroid plasma binding proteins (Fig. 1):

a. Testosterone and estradiol are bound primarily to sex hormone


binding globulin (SHBG).
b. Progesterone is bound primarily to corticosteroid binding
globulin (CBG).

i. The progesterone binding site on CBG is distinct from


that binding cortisol.

c. The functional significance of sex steroid binding in plasma is


analogous to that described previously for corticosteroids.

5. Targets for sex steroids:

a. Gametogenesis
b. Sex accessory organ maintenance and function
c. Secondary sex characteristics (maleness and femaleness)
d. Anterior pituitary function (negative and/or positive feedback
effects).
e. Central nervous system (behavior)
f. Selective metabolic effects - such as effects on growth, binding
proteins for thyroid and adrenal hormones.

6. Sex steroid clearance occurs by chemical modification in the liver.

B. Two stages of mammalian reproduction that are common to both sexes are
puberty and climacteric.

1. Puberty

a. Puberty is the process (or time interval) when reproductive


maturation occurs (12-15 years of age).
b. Puberty is initiated by the onset of pulsatile GnRH release
which in turn stimulates LH and FSH release and, in response
to LH, gonadal steroid release.
c. Physical maturation at puberty is a consequence of sex steroid
effects.

2. Climacteric

a. Climacteric is the age dependent decline in reproductive


capacity (called menopause in females)

i. Climacteric is less dramatic in males than females.

1080
Male Reproduction 1, 2, & 3 - Dr. Witorsch

ii. Climacteric is associated with a hyporesponsiveness of


the gonad to LH and FSH.

Figure 3. Histological View of Testes (low power


magnification

II. MALE REPRODUCTIVE SYSTEM

Figure 4. Histological View of Seminiferous Tubule (high power


magnification)

A. Testes (Figs. 3 and 4): - The testes is compartmentalized, as follows:

1. Interstitial cells of Leydig - the steroidogenic compartment of the


testes.
2. Seminiferous tubules - the gametogenic (spermatogenic) compartment
of the testes.

1081
Male Reproduction 1, 2, & 3 - Dr. Witorsch

a. Spermatogenesis involves proliferation (mitosis), reduction


division (meiosis), and transformation of diploid primordial
germ cells to haploid spermatozoan cells.
b. Spermatogenesis proceeds from the periphery to the center of
the tubule.

3. Spermatogenesis and components of seminiferous tubules

a. The seminiferous tubule is divided into a basal and adluminal


compartment by the "blood-tubule" barrier.
b. Sertoli cells are the somatic cells of seminiferous tubules.

i. Sertoli cells are tall columnar cells extending from


basement membrane to the center of the lumen.
ii. Sertoli cells are targets for FSH and testosterone.
iii. Sertoli cells secrete at least 100 proteins, prominent
among these are inhibin, androgen binding protein
(ABP), and tissue plasminogen activator (TPA).
iv. Sertoli cells convert testosterone to estradiol
(aromatization).
v. Sertoli cells form "blood-tubule" barrier.

• Tight junctions formed between adjacent Sertoli


cells.
• Sertoli cells exclude materials (> 300 d) from
entering adluminal compartment of tubule.
• CdCl2 can breach barrier and destroy testes.

vi. Sertoli cells provide nurse (supportive, nurturing)


function to spermatogenesis.

c. Germ cells (cells undergoing spermatogenesis).

i. Germ cells develop between and in intimate contact


with Sertoli cells.
ii. Germ cells develop from the basal portion to the center
of tubule.
iii. Spermatogenesis involves mitosis, meiosis, and
differentiation.
iv. Among the cell types undergoing spermatogenesis are:

• Spermatogonia (the most undifferentiated) are


located in basal compartment and are diploid.

o Some spermatogonia are committed to


the process of spermatogenic

1082
Male Reproduction 1, 2, & 3 - Dr. Witorsch

development, others are replenished for


commitment at a later date (cell
renewal).

• Primary spermatocytes develop from


spermatogonia and represent the last stage
where DNA synthesis occurs.

o Prior to cell division, primary


spermatocytes have a diploid set of
chromosomes with 4N amount of DNA
(preleptotene primary spermatocyte).
o Preleptotene primary spermatocytes
enter the adluminal compartment of the
tubule, an event asssociated with
plasminogen acitivity and then undergo
meiosis I.

• Secondary spermatocytes are products of


meiosis I.

o These cells are haploid with 2N amount


of DNA.
o Secondary spermatocytes are a very
short lived stage and rapidly undergo
meiosis II.

• Spermatid is the product of meiosis II.

o Spermatid cells are haploid with N


amount of DNA.
o Spermatids develop cytoplasmic bridges
with Sertoli cells.
o Spermatids undergoes spermiogenesis, a
differentiation process involving
flagellum formation, shedding of
cytoplasm, acrosome formation from
Golgi apparatus, and elongation.

• Spermatozoa are products spermiogenesis

o Spermatozoa dissociate from Sertoli


cells by the process called spermiation.

d. Know the following about spermatogenesis:

1083
Male Reproduction 1, 2, & 3 - Dr. Witorsch

i. Although nuclear division is complete during mitosis


and meiosis, cytoplasmic division (cytokinesis) is not.

• Cytoplasmic bridges exist between cells derived


from a single spermatogonium through to a
spermatid.

ii. These bridges sever for spermatozoa prior to


spermiation.
iii. Spermatogenesis is a synchronized (species-specific)
process of constant duration.
iv. Stem cells enter into spermatogenesis at constant
intervals.
v. FSH and testostosterone control spermatogenesis via
Sertoli cells. Other hormones may also participate in
spermatogenesis.

B. Sex Accessory Organs (Fig. 5)

1. Sex accessory organs are tubulo-alveolar structures that are continuous


with the seminiferous tubules.
2. They perform several functions, namely final maturation of
spermatozoa, storage of spermatozoa, production of secretions
(seminal plasma) that nurture spermatozoa, and participation in
ejaculation.
3. Epididymides (singular, epididymis) are the major sites of
spermatozoan maturation and storage.

a. While within the epididymis, spermatozoa develop motility,


fertilizing capacity, and concentrate DNA.
b. Sperm cells live 25-50 days in the epididymis.
c. Testicular fluid is reabsorbed within the epididymis resulting in
spermatozoan concentration.

4. Vas deferens is a minor site of spermatozoan storage, and an organ


involved in the ejaculatory response.
5. Prostate secretes components of seminal plasma and is the source of
numerous seminal constituents such as: acid phosphatase, zinc, citrate,
fibrinolysin, polyamines and immunoglobulins.
6. Seminal vesicle is the source of fructose and prostaglandin.

a. Fructolysis a major source of energy for spermatozoa and


seminal fructose concentrations are 500-600 mg%.
b. Bulbourethral (Cowpers) glands produce a sialomucoprotein
called mucin.

1084
Male Reproduction 1, 2, & 3 - Dr. Witorsch

Figure 5. Male Reproductive Organs

C. Erection and Ejaculation

1. Erection is a parasympathetically-mediated dilation of arterioles in the


penis. Blood engorgement of penile erectile tissue blocks outflow of
blood leading to penile rigidity.

a. The smooth muscle relaxation resulting in penile erection is


mediated by the nitric oxide-cyclic GMP pathway.
Phosphodiesterase 5 (PDE 5) in the corpus cavernosum
degrades cyclic GMP. PDE5 inhibitors such as sildenafil,
tadalafil, and cardenafil which prolong cyclic GMP are the
basis of modern erectile dysfunction therapy.

2. Ejaculation is an sympathetically-mediated contraction of sex


accessory organs and perineal musculature, propelling semen out of
the urethra.
3. Semen is a white opalescent substance that is slightly heavier than
water, composed of spermatozoa and seminal plasma. The volume of
the ejaculate is about 3 ml. The ejaculate coagulates upon ejaculation
(due to the action of mucin) and liquifies within 10 min due to the
action of fibrinolysin. On average, semen contains 50-100 x 106
spermatozoa/ml in normal men. Sperm counts below 20 x 106/ml
constitute sterility.
4. Ejaculated spermatozoa are composed of a head and midpiece tail
complex (Fig. 6). The head contains the nucleus (condensed haploid
DNA with little cytoplasm) and acrosome (bag-like structure rich in
hydrolytic enzymes for penetration of ovum during fertilization). The

1085
Male Reproduction 1, 2, & 3 - Dr. Witorsch

midpiece-tail complex is involved in sperm motility. The midpiece is


rich in mitochondria and contains contractile proteins. The tail is
composed of contractile proteins.

Figure 6. Human Spermatozoan

5. During ejaculation prostatic contractions precede those of the seminal


vesicle. Thus the early ejaculate is rich in prostatic constituents. Late
portions of the ejaculate are rich in seminal vesicle constituents.

a. The split ejaculate technique collects early and late fractions


separately and is thus useful for diagnostic purposes.

D. Secretion, Circulation, and Degradation of Testosterone

1. Testosterone is secreted by Leydig cells under influence of LH. The


production rate of testosterone is 4-9 mg/day in normal men. Plasma
levels of testosterone range from 3-15 ng/ml and fluctuate in a
pulsatile fashion in response to LH.
2. Testosterone exhibits a circadian pattern similar to that of cortisol.
Aging is associated with a loss of this circadian pattern.
3. Most (97%) of the circulating testosterone is bound to plasma proteins
where 57% is bound to sex hormone binding globulin (SHBG) and
40% is bound to albumin.
4. The liver inactivates testosterone by A ring reduction and oxidation of
the 17-hydroxy group in the D-ring. The major testosterone
metabolites are A-ring reduced 17-ketosteroids.

1086
Male Reproduction 1, 2, & 3 - Dr. Witorsch

a. 17-ketosteroids are conjugated with glucuronide and sulfate in


the 3-hydroxyl position prior to urinary excretion.

E. Physiological Actions of Testosterone

1. Testosterone maintains spermatogenesis in the seminiferous tubules


and maturation of spermatozoan in epididymis.
2. Testosterone maintains the size and secretory activity of sex accessory
organs.
3. Testosterone is responsible for the development of secondary sex
characteristics at puberty and maintenance of these characteristics
postpubertally, such as size of external genitalia (penis and scrotum),
laryngeal enlargement leading to voice deepening, sebaceous gland
secretion leading to acne, masculine accumulation of muscle mass,
anabolic effects, and male hair distribution.
4. Testosterone also controls growth effects such as acceleration and
termination of long bone growth. This action requires the presence of
growth hormone.

a. Androgens increase secretion of growth hormone and IGF-1


production and stimulates maturation of chondrocytes and
osteoblasts at the epiphyseal plate.
b. Androgen action at epiphyseal plate leads to closure (or fusion)
of the epiphysis within the shaft of bone and the termination of
bone growth.

5. Testosterone evokes psychological effects such as increased libido and


aggressiveness.
6. Testosterone participates in negative feedback (suppression) of LH
secretion.
7. Testosterone participates prenatal differentiation of Wolffian ducts,
external genitalia and brain of male fetus. This event involves fetal
testes.

F. Mechanism of Action of Testosterone

1087
Male Reproduction 1, 2, & 3 - Dr. Witorsch

Figure 7. Processing of Testosterone by Target Cells

1. Upon entering the cell, testosterone can follow 3 metabolic pathways,


as follows (Fig. 7):

a. No metabolism at all (which occurs in such tissues as kidney


and muscle).
b. Formation of dihydrotestosterone as a result of the action of the
enzyme, 5α-reductase (which occurs in sex accessory organs).

i. Inhibition of 5α-reductase, by the drug Finasteride, is


the basis for treatment of benign prostatic hypertrophy
(BPH) and possibly prostatic cancer.

c. Aromatization to estradiol (which occurs in the pituitary, brain,


Sertoli cell)

2. Testosterone or its metabolites then interact with the androgen receptor


(for testosterone and dihydrotestosterone), and the estrogen receptor
for (estradiol) and translocate to the genomic site in the nucleus (i.e.,
“responsive element”.) to stimulate an effect.

a. Dihydrotestosterone (DHT) has a higher affinity for androgen


receptor than testosterone producing a 10-fold amplification of
response.

G. Regulation-Pituitary Testicular Axis (Fig. 8)

1088
Male Reproduction 1, 2, & 3 - Dr. Witorsch

Figure 8. Major Elements and Control of Male Reproductive


System

1. Important facts and speculations

a. Pulsatile GnRH release stimulates pulsatile FSH and LH


release from anterior pituitary. Nonpulsatile GnRH stimulates
gonadotropin release only transiently and ultimately inhibits
FSH, LH and testosterone secretion.
b. LH, via cAMP, stimulates testosterone secretion from Leydig
cells.
c. Testosterone exerts negative feedback control of LH release
from the pituitary. Testosterone may act at the pituitary level
suppressing GnRH action, or may act on the hypothalamus
suppressing GnRH release.
d. Testosterone conversion to estradiol in the brain and or
pituitary may play a role in feedback inhibition.
e. FSH and testosterone (T) are primary regulators of
spermatogenesis. Intratesticular T concentration may be 100
times that of plasma.
f. The Sertoli cell, rich in receptors for FSH and T, is considered
the target for these hormones. T stimulates the secretion of
Sertoli cell proteins (ABP, etc).

i. FSH sensitizes the Sertoli cell to T.

g. T appears to be required for spermiogenesis.

1089
Male Reproduction 1, 2, & 3 - Dr. Witorsch

h. Other hormones detected in testes may participate in


spermatogenesis, such as oxytocin, ADH, GnRH, TRH, CRH,
calcitonin, angiotensin, ß-endorphin, and tumor necrosis factor
(TNF)-α.

i. Sertoli cells appears pivotal in the control of spermatogenesis.


They may produce factors that stimulate and inhibit meiosis.
j. ABP may deliver testosterone secreted directly into the
seminiferous tubule from the Leydig cell to the maturing
spermatozoan in the testes and epididymis. However, to date,
testosterone receptors have not been detected on spermatozoa.
k. Inhibin, activin, and follistatin are made in the Sertoli cell but
are also found in the anterior pituitary gland Inhibin and activin
are suspected of serving some local function in seminiferous
tubule.

i. Inhibin and activin are members of the TGF


(transforming growth factor) peptide family.
ii. Inhibin, is the systemic inhibitor of FSH release. FSH
release is also under the negative feedback control of
testosterone. Activin may be the systemic activator of
FSH release.
iii. Follistatin is a glycoprotein that binds to and
antagonizes the action of activin.

H. Prolactin and Male Reproduction

1. Prolactin is generally considered a maternal hormone involved in milk


production post-partum. Prolactin is also detectable in male serum and
may play a role in male reproduction at physiological concentrations,
such as spermatogenesis, testosterone production, prostatic function,
sexual performance.

I. Male Gonadal Disorders

1. Hypogonadism refers to testosterone deficiency. It can be congenital or


acquired. Hypogonadism prior to adolescence is referred to as delayed
puberty. The major classifications of hypogonadism are as follows:

a. Hypogonadotropic hypogonadism (or secondary


hypogonadism) where testosterone deficiency is secondary to a
deficiency in LH.

i. This form of hypogonadism can arise from a defect in


the hypothalamus (GnRH deficiency) and/or the
pituitary which might result from CNS disorders,

1090
Male Reproduction 1, 2, & 3 - Dr. Witorsch

panhypopituitarism, LH-FSH deficiencies, isolated LH


deficiencies, etc. Pulsatile delivery of GnRH (every 2
hours) may advance puberty in some of these patients.

b. Hypergonadotropic hypogonadism (primary hypogonadism)


where testosterone deficiency is due to a primary defect in the
Leydig cell. Serum LH and FSH levels are elevated while
serum testosterone levels are low (why?)

i. This form of hypogonadism might result from genetic


abnormalities, toxic agents, or steroidogenic enzyme
deficiencies.

2. Precocious puberty is a result of premature elevation of testosterone


levels leading to precocious virilization. There are two general
classifications, as follows:

a. True precocious puberty is characterized by elevated


testosterone secondary to elevated gonadotropin.

i. This type of precocious puberty may result from early


pulsatile release of GnRH, CNS tumors, or other CNS
disorders. Nonpulsatile GnRH analogues may be of
benefit in the treatment of this disorder.

b. Incomplete precocious puberty (or precious pseudopuberty)


is characterized by testosterone secretion independent of
pituitary gonadotropins. Serum gonadotropin levels are pre-
pubertal (low) while testosterone levels are elevated.

i. This type of precocious puberty can result from


autonomous Leydig cell function or congenital adrenal
hyperplasia leading to the adrenogenital syndrome.

3. Other gonadal disorders

a. Primary impairment of sperm production with little or no effect


on testosterone production can be the result of toxic effects to
Sertoli or germ cells while the Leydig cells are spared.
b. Androgen resistance disorders have been identified, such as:

i. Testicular feminization (also referred to as androgen


insensitivity syndrome or AIS), an inherited disorder, is
an androgen receptor defect where testosterone levels
may be normal. These individuals are genotypic males

1091
Male Reproduction 1, 2, & 3 - Dr. Witorsch

(XY karyotype) showing secondary sex characteristics


that are phenotypically female.
ii. 5α-reductase deficiency is an autosomal recessive
inherited defect in which the testes appear normal and
plasma testosterone levels are normal.

• Subjects exhibit incomplete development of the


urogenital sinus and external genitalia (which
require DHT for development).
• Subjects have a small phallus, urethral opening
at the base with a blind vaginal pouch derived
from prostate and external genitalia.
• Wolffian duct derived structures (e.g.,
epididymis, vas deferens, seminal vesicle) are
well developed because they respond to T and
don’t require DHT.
• At birth and during childhood these subjects
exhibit ambiguous genitalia. Adolescent
development is controlled by testosterone so at
puberty voice deepens, masculine physique
develops, however the prostate remains
involuted.

J. Male contraception strategies

1. Androgenic, estrogenic or progestational steroids administered alone


or in combination to suppress gonadotrophin release.
2. Androgen antagonists (cyproterone acetate) administered to prevent
natural androgen binding to its receptor.
3. GnRH agonists and antagonists evoke potent antifertility effects due to
nonpulsatile GnRH suppression of LH secretion. They are also used as
a form of "medical castration" for prostate cancer and retardation of
true precocious puberty.
4. Gossypol is a nonhormone phenolic compound extracted from
gossypium (a cotton plant) that acts as a reversible antispermatic
agent.

1092
Male Reproduction 1, 2, & 3 - Dr. Witorsch

III. STUDY QUESTIONS - All Type A

1. Pulsatile GnRH may be an effective therapy for delayed puberty and/or


infertility in males with:

A. hypothalamic dysfunction
B. anterior pituitary gonadotrophic cell dysfunction
C. Leydig cell dysfunction
D. Sertoli cell dysfuncton
E. androgen receptor dysfunction

2. Which of the following functions of the Sertoli cell is involved in the


regulation of FSH secretion?

A. inhibin production
B. androgen binding protein production
C. maintenance of the "blood-testis barrier"
D. testosterone production
E. aromatization of androgen

3. Which of the following male sex accessory organs is the principal source
of fructose in seminal fluid?

A. testes
B. epididymis
C. vas deferens
D. seminal vesicle
E. prostate

4. The organ that produces most of the seminal plasma is the:

A. testes
B. epididymis
C. vas deferens
D. prostate
E. seminal vesicle

5. Non-pulsatile GnRH is potentially useful for the treatment of:

A. adrenogenital syndrome
B. hypogonadism due to gonadotropin deficiency (hypogonadotropic
hypogonadism)
C. hypogonadism due to Leydig cell defect (hypergonadotropic
hypogonadism)
D. precocious puberty due to elevated gonadotropins (true precocious
puberty)

1093
Male Reproduction 1, 2, & 3 - Dr. Witorsch

E. precocious puberty due to autonomous Leydig cell hyperfunction

6. Testosterone treatment can produce virilizing effects in all of the


following conditions except:

A. hypogonadotropic hypogonadism
B. hypergonadotropic hypogonadism
C. testicular feminization
D. hypophysectomized males
E. hypophysectomized females

7. Which of the following steroidogenic enzymes is absent from gonads?

A. cholesterol desmolase
B. 3 beta-ol dehydrogenase:isomerase
C. 17 alpha-hydroxylase
D. 21-hydroxylase
E. C-17,20-lyase

8. LH accelerates testosterone production in the testes at which enzymatic


conversion?

A. Cholesterol to delta-5 pregnenolone


B. delta-5 pregnenolone to progesterone
C. progesterone to 17 alpha-hydroxyprogesterone
D. DHEA to androstenedione
E. androstenedione to testosterone

9. Which of the following germ cell types exhibits the last DNA synthesis?

A. spermatogonia
B. primary spermatocyte
C. secondary spermatocyte
D. spermatid
E. Sertoli cell

Answer Key: 1. A; 2. A; 3. D; 4. E; 5. D; 6. C; 7. D; 8. A; 9. B.

1094
Insulin - Dr. Kalimi

Physiology and Biochemistry of


Insulin and Glucagon 1, 2, and 3
Mohammed Y. Kalimi, Ph.D.

I. Metabolic Overview: Glucose Homeostasis

Blood glucose levels:

Normal range - 70 mg% to 120 mg% (the plasma glucose level is maintained in a
narrow range around 5 mM, which is considered the physiological set point).

• Prevention of hyperglycemia:
Fed state: ↑ insulin.
• Prevention of hypoglycemia:
Fasted state: ↑ insulin antagonist hormones ( ↑ glucagon, ↑ catecholamines, ↑
glucocorticoids, ↑ growth hormone, and ↑ thyroid hormones).

Hormonal regulation of glucose:

A. Fed state: ↑ insulin: prevention of hyperglycemia.

Insulin Actions:

General functions of insulin are anabolic.

1. ↑ Protein, fat and glycogen synthesis.


2. ↑ Oxidation of carbohydrate in insulin sensitive tissues (thus lowering
blood glucose).
3. ↓ Fat breakdown.

Effects in major tissues sensitive to insulin:

1. Muscle (cardiac and skeletal):

a. ↑ Glucose transport, facilitated diffusion of glucose through increasing


concentration of specific glucose transporter protein isoforms (Gluts),
at the cell surface.
b. ↑ Glycolysis
c. ↑ Glycogen synthesis
d. ↑ Amino acid transport, independent of glucose transport.
e. ↑ Protein synthesis, gene expression, independent of amino acid
transport.
f. ↓ Protein catabolism
g. ↑ Intracellular K+ content.

2. Adipose:

1095
Insulin - Dr. Kalimi

a. ↑ Glucose transport (facilitated diffusion of glucose).


b. ↑ Fatty acid synthesis
c. ↑ Triglycerides synthesis
d. Insulin stimulates the production of α-glycerophosphate which is the
acceptor of fatty acids for the synthesis of triglycerides.
e. ↑ Storage and ↓ breakdown of fats.

3. Liver:

Unlike the other two tissues (muscle and adipose), the liver is permeable
to glucose in absence of insulin.

a. ↑ Insulin, ↑ the enzyme glucokinase in the liver cells.


Glucose + ATP + glucokinase → glucose-6-phosphate.
The phosphorylation of glucose "traps" glucose inside
the liver cells.
b. ↑ Glycogen synthesis
c. ↑ Free fatty acid synthesis
d. ↓ Hepatic glucose output due to ↓gluconeogenesis
e. ↓ Ketogenesis

B. Fasted state: Insulin low: prevention of hypoglycemia.


Increase in glucagon, glucocorticoids, growth hormone, thyroid hormones and
catecholamines.

1. ↑ Hepatic secretion of glucose by ↑ glycogenolysis (glucagon,


epinephrine)

a. Glycogen + phosphorylase →glucose-6-phosphate


b. Gucose-6-phosphate + phosphatase → glucose.

2. ↑ Protein catabolism (↑ proteolysis by glucocorticoids).


3. ↑ Lipolysis (catecholamines, growth hormone, glucagon, thyroxine and
glucocorticoids).
4. Stimulating gluconeogenesis directly or indirectly (glucocorticoids,
glucagon).

a. by glucogenic amino acids from muscle proteolysis.


b. by glycerol from lipolysis.
c. by lactate and pyruvate.

Pattern of fuel utilization during food deprivation:

1. Blood glucose:

1096
Insulin - Dr. Kalimi

a. Decreased utilization by muscle and adipose tissue.


b. Terminal oxidation (CO2 and H2O) by brain.
c. Glycolysis to lactate and pyruvate (partial oxidation of glucose) by
RBC's, WBC's, bone marrow, renal medulla, peripheral nerves, eye,
intestinal mucosa, and kidney tubules

2. Fatty acids (released by lipolysis):

a. Low levels of insulin during fasting impairs glucose utilization in


muscle, adipose tissue, etc. and shifts to lipid metabolism.
b. Terminal oxidation by muscle to CO2 and H2O.
c. Partial oxidation by liver, producing ketone bodies such as acetone,
acetoacetic acid and ß-hydroxybutyric acid.

II. Insulin (51 amino acids peptide):

A. Processing of insulin:

Pre - Pro - Insulin (about 105 amino acids):

Pre is a signal sequence (about 20 amino acids) that helps in the transfer of
newly synthesized secretory proteins into the lumen of the endoplasmic
reticulum (membrane recognition).

Cleavage of N-terminal sequence of pre -pro- insulin results in the release of


proinsulin.

Proinsulin (86 amino acids):

1. Proinsulin is made up of A, B and C chains. (Figure 1)


2. Proinsulin is packaged into secretory vesicles in the golgi apparatus. The
vesicles contain zinc and proteases such as preconvertase-I and
carboxypeptidase –H. Proteolysis removes the 'c' chain containing 35
amino acids and insulin (containing A and B chains) is cleaved and release
into the blood.
3. Proinsulin has about 10% of the biological activity of insulin.

1097
Insulin - Dr. Kalimi

Figure 1: Structure of pro-insulin

Insulin (51 amino acids peptide):

Made up of A (21 amino acids) and B (30 amino acids) chains held
together by two disulfide bonds. (Figure 2)

Figure 2: Structure of insulin

B. Secretion of insulin:

Synthesis and secretion (about 1-2 mg/day) of insulin are closely linked.

The plasma half-life of insulin in humans is about five minutes.

C. Release of Insulin:

Excitation - secretion coupling:

1098
Insulin - Dr. Kalimi

1. ↑ Blood glucose, ↑ glucose transport into beta cells of pancreas via the
GLUT 2 transporter.
2. ↑ glucose oxidation, ↑ ATP production,
3. ↑ ATP/ADP ratio, closure of potassium channels,
4. ↑ beta cell depolarization
5. ↑ in the opening of voltage-sensitive calcium channels
6. ↑ intracellular calcium
7. ↑ the fusion of insulin secretory granules of β-cells with the plasma
membrane and by exocytosis, insulin is secreted into plasma.

D. Degradation of insulin:

1. Systemic:
Liver and kidney: the enzyme insulinase degrades insulin.
2. Intracellular:
Internalization and degradation by lysosomal enzymes.
3. Excretion: In urine by glomerulus.

E. Factors influencing insulin secretion:

1. Release:

a. ↑ Blood glucose level - major stimuli


b. ↑ Amino acids (arginine and lysine are most potent)
c. Diet
d. Vagus: acetylcholine
e. An oral hypoglycemic agent: sulfonylureas. The sulfonylurea
stimulates insulin release by closing the K+ channel.
f. Gastrointestinal peptide (GIP) and GLP-1 (glucagon-like peptide- 1).
When the same dose of glucose is given orally and intravenously, the
oral dose provokes the greatest release of insulin. This is because oral
glucose stimulates the release of GIP and GLP-1. These peptides are
potent and important insulinogogues.

2. Inhibition of release:

a. Somatostatin
b. Catecholamines
c. Fasting
d. Exercise
e. Leptin
f. Diazoxide: inhibits insulin release by opening the K+ channel.

F. Mechanism of insulin action (Fig.3):

1099
Insulin - Dr. Kalimi

Figure 3: Insulin Action

Binding of insulin to its receptor results in multiple physiological responses at


several sites including the plasma membrane, cytoplasm and nucleus.

1100
Insulin - Dr. Kalimi

Table 1: Actions of Insulin

1. Stimulus + pancreas, ↑ insulin release.


2. Insulin + cell membrane receptor (glycoprotein of MW 300,000 daltons)
→ insulin-receptor complex.

Insulin receptor is a heterotetramer made up of four subunits: two identical


α chains of MW 135,000 daltons and two ß chains of MW 90,000 daltons.

The extracellular α subunit binds insulin and the β subunit spans the
membrane and contains tyrosine phosphokinase activity.

Binding of insulin (single molecule) to its α-subunit receptor site activates


autophosphorylation of the receptor on tyrosine residues of β subunit. This
results in the phosphorylation of a number of intracellular insulin receptor
substrates (IRS-1, IRS-2, IRS-3,and IRS-4).

IRS molecules bind to and activate phosphatidylinositol-3 kinase(PI-3K)


and several other proteins including G-Ras, and MAPKs resulting in the
insulin-activated transcription of specific genes (Fig.3).

PI-3K is an important molecule in the mediation of insulin stimulated


glucose transport.

G. Syndrome associated with excess insulin production:

Tumors of β-cells primarily produce insulin. The major finding is severe


hypoglycemia in the fasting state.

Since fasting activates release epinephrine and norepinephrine, these


hypoglycemic episodes are accompanied by rapid heart rate, nervousness,
sweating and hunger.

If hypoglycemia is severe, it can be accompanied by loss of consciousness,


convulsions and if not treated rapidly, brain damage.

H. Diabetes Mellitus:

1101
Insulin - Dr. Kalimi

Starvation in the midst of plenty: Diabetes mellitus is probably a


heterogeneous group of diseases characterized by the elevation of plasma
glucose.

It is currently estimated that 18 million people are afflicted with diabetes in


the US (143 million people worldwide). More than 200,000 people in the
USA die each year from diabetes related complications.

1. Primary Diabetes:

a. Juvenile diabetes or type I diabetes (thin, young, and ketosis prone):

Usually seen in the younger patient (10% of all people with diabetes
have type I diabetes).

Characterized by increased hunger, increased thirst, increased


urination, fatigue and sudden weight loss (negative nitrogen balance).

i. Acute onset often with ketosis


ii. A requirement of insulin treatment
iii. Sensitivity to exogenous insulin
iv. Pancreas often found totally lacking in insulin (complete
destruction of β cells).

b. Adult onset diabetes or type II diabetes (adult, often associated with


obesity, hypertension, or dyslipidemia):

Type II diabetes results from the combination of defects in insulin


secretion and action (defect in insulin receptor or post receptor
defects).

Characterized by peripheral insulin resistance, increased hunger,


increased thirst, increased urination, frequent infections, blurred eye
sight, fatigue, numbness or tingling in hands and feet, slow healing of
cuts or sores and impotence (sexual problems).

Commonly associated with aging, obesity, a sedentary life style, as


well as a genomic predisposition.

Two basic tests to diagnose type-II diabetes.

i. Fasting test: Blood glucose should be below 110 mg/ 100 mL after
an overnight fast.
ii. Oral tolerance test: Blood glucose should not be higher than 140
mg/100 mL two hours after the patient swigs cup of glucose (75
grams) laden fluid.

1102
Insulin - Dr. Kalimi

Therapy for type II diabetes:

Diet (100-150 less calorie consumption/day), oral agents, insulin, exercise


(30 minutes walk or on trade mill, 5 days a week) and monitoring of
glucose levels.

The drug metformin (glucophage) improves insulin resistance in a variety


of obese and diabetic animal models and patients by ↓ plasma glucose
levels, ↓ hepatic glucose production, and ↑ insulin action.

2. Secondary Diabetes:

a. Thyrotoxicosis:
Hyper function of the thyroid gland is often accompanied by elevated
blood sugar. This is usually due to rapid GI absorption of sugar rather
than an underlying disturbance of insulin secretion or carbohydrate
metabolism.
b. Acromegaly (growth hormone excess) or adrenal hyper function
(Cushing's syndrome) can cause diabetes.
c. Pancreatic insufficiency:
Destruction of the pancreas by inflammatory disease or carcinoma can
lead to mild diabetes.
d. Metabolic consequences of type II diabetes:

i. Muscle proteolysis: ↑Amino acidemia, and negative nitrogen


balance.
ii. Adipose lipolysis: ↑ Lipemia, ↑ plasma glycerol, ↑ FFA, ↑
cholesterol, and ↑ LDL.
iii. Liver: - ↑ gluconeogenesis, ↑ glucose production.
iv. Insulin sensitive tissue (muscle and adipose): impaired glucose
utilization and a shift to free fatty acids and ketone body
oxidation.
v. Hyperglycemia → glucosuria, → polyuria, → dehydration
(hypovolemia), → polydipsia, → polyphagia,
vi. Ketosis (nausea and vomiting) → ketonemia,→ ketonuria, →
metabolic acidosis (loss of K+, Na+), ↓ Blood pH, ↓ Plasma
HCO3, and coma.

3. Contribution of hormones which elevate blood glucose to diabetic


syndrome "Diabetogenic hormones". Glucagon, catecholamines, cortisol
and growth hormones generally increase output of glucose in severe
diabetes.

a. Effects of hypophysectomy or adrenalectomy in diabetes (↓ symptoms


of diabetes).

1103
Insulin - Dr. Kalimi

b. Metabolic effects of pituitary extracts and glucocorticoids (↑


symptoms of diabetes).

4. Complications of Diabetes:

Neuropathy, retinopathy and cataracts, vascular diseases, kidney


dysfunction, foot problems (severe infections that can result in
amputation), impotence (sexual problems), and problems with pregnancy.

Methods of Inducing Diabetes:

a. Anti-insulin serum
b. Drugs such as alloxan and streptozotocin (destruction of pancreatic
beta clls)
c. Pancreatectomy
d. Viral infection (Coxsackie, mumps)

III. Glucagon:

A. Major Actions: Elevation of blood glucose (glucagon enhances mobilization


rather than storage of glucose).

↑glycogenolysis Blood
↑ gluconeogenesis ↑ glucose
↑ lipolysis ↑ ketoacids
↑ FFA
↓ amino acids

B. Mechanisms of glucagon action:

1. Binding of glucagon with target cell membrane receptor.


2. Increase in cyclic AMP (by increasing adenylyl cyclase).
3. Increase in protein kinases which regulate cell metabolism.

C. Secretion:

1. Synthesized as a Preproglucagon → proglucagon → glucagon (29 amino


acids) molecular weight 3500 daltons.
2. In humans there are 100-150 :g of glucagon secreted in 24 hours.
3. The half life of glucagon in the blood is 5 minutes.

D. Factors Influencing Glucagon Secretion:

Stimuli for glucagon's release:

1. Hypoglycemia

1104
Insulin - Dr. Kalimi

2. Increased concentration of amino acids in the blood, mainly glucogenic


amino acids such as alanine, serine, glycine, etc.
3. Prolonged exercise and stress

Glucagon release inhibited by:

1. Somatostatin
2. Insulin
3. ↑ Glucose
4. ↑ Free fatty acids

E. Glucagon and three gut hormones - secretin, vasoactive inhibitory peptide,


and gastric inhibitory peptide share certain homologous amino acid sequences
and also share the ability to potentiate the stimulatory effect of glucose on β
cell insulin secretion.

F. Deficiency states are not well authenticated but may result in hypoglycemia.

The adrenal medulla probably carries out any function attributable to the loss
of α -cells.

α cell tumors produce excess glucagon. The state is characterized clinically by


loss of weight, elevated levels of blood glucose and ketoacids as well as
depressed levels of blood amino acids.

Table 2

IV. Somatostatin:

The release of somatostatin from delta or D cells of the pancreas is stimulated by


glucose, amino acids, free fatty acids and gastrointestinal hormones.

Hyper insulinism in infants is observed in association with a complete lack of


delta cells in the pancreas.

1105
Insulin - Dr. Kalimi

Patients with rare tumors of delta cells produce excess amounts of somatostatin.
Plasma levels of glucagon and insulin are below normal in these patients.

V. Hormone Release and Inhibition: Summary

Hormone
Release Inhibition
sleep (stage iv) hyperglycemia
arginine infusion ↑ plasma FFA
GH hypoglycemia (fasting) obesity
ghrelin aging
GHRH somatostatin
TRH somatostatin
nursing dopamine
Prolactin
estrogen GABA
pregnancy bromocryptin mesylate

TRH sleep
TSH leptin fasting
cold somatostatin

hyperglycemia
diet fasting
acetylcholine leptin
Insulin
sulfonylurea somatostatin
GIP & GLP-1 exercise
amino acids infusion

somatostatin
hypoglycemia
Glucagon hyperglycemia
↑ plasma amino acids
↑ plasma FFA

1106
Female Reproduction 1 and 2 - Dr. Witorsch

Female Reproduction 1 and 2: The Menstrual Cycle


R.J. Witorsch, Ph.D.

OBJECTIVES:

At the end of this block of lectures, the student should be able to:

1. Describe the functional anatomy of ovary with particular reference to events prior
to, during and after ovulation.
2. Describe the functions of female sex accessory organs.
3. Describe and explain the events and regulatory mechanisms involved in the
human menstrual cycle.
4. Describe the events involved in the secretion, circulation, and biological half-life
of estrogen and progestin in the adult cycling female.
5. Describe the physiological actions and mechanisms of action of estrogen and
progestin.
6. Describe and explain the various srategies involved in female contraception.
7. Explain the etiologies, symptomologies, and diagnostic strategies of the following
gonadal disorders in the female:

a. hypogonadism
b. precocious puberty
c. menstrual disorders.

Suggested Reading: Costanzo 2nd Edition, pp. 417-424

I. OVARY

A. Female reproduction is characterized by monthly cycles, culminating in


the extrusion of a single egg from the ovary by the process known as
ovulation.

B. The major structures of the ovary and their functions are (Fig. 1):

1107
Female Reproduction 1 and 2 - Dr. Witorsch

Figure 1. Mammalian Ovary

1. Hilus, the area where lymphatics, nerves and blood vessels enter and
exit the ovary.
2. Germinal epithelium: a misnomer, in reality a specialization of
peritoneal mesothelium.
3. Primordial follicle: a single egg cell (oocyte) surrounded by a single
layer of cells (granulosa cells). This is the basic reproductive unit of
ovary. The primordial follicle is separated from adjacent stroma by a
basement membrane surrounding the granulosa cells. Primordial
follicles are found primarily in outer cortex just beneath fibrous
capsule of ovary. The following are noteworthy features of the
primordial follicles:

a. The oocyte is small (< 25μm) and in prophase of meiosis I.


b. Primordial follicles mature and subsequently release a single
oocyte from the ovary at ovulation. Primordial follicles provide
a resting pool from which follicles develop.

1108
Female Reproduction 1 and 2 - Dr. Witorsch

c. About 1-2 x 106 primordial follicles are present in each ovary at


birth.

i. Groups of follicles undergo partial development and


atresia (degeneration) during late fetal life and through
prepubertal years.
ii. At puberty there are approximately 4 x 105 primordial
follicles in each ovary. The rest have undergone
development and atresia in earlier years.

d. From the beginning of female fertility (menarche) to the end of


female fertility (menopause) there are approximately 400
ovulatory cycles. About 1,000 primordial follicles/ovary are
mobilized (i.e., undergo development) during each cycle.

i. Only 1 follicle fully matures while the remainder


undergo atresia.
ii. Recent observations (2004) suggest that mammalian
ovaries contain stem cells that can generate new eggs
through adulthood. This challenges the assumption that
the full complement of eggs are present at birth.

4. Follicles in various stages of development

a. The primary follicle is an early stage of development beyond


the primordial stage. It contains an enlarged oocyte (80-
100μm) which is surrounded by a zona pellucida. In the
primary follicle the granulosa cells become cuboidal (having
been previously flat).
b. With further development granulosa cells proliferate and a
vesicle (antrum) forms within granulosa cell region from a
plasma exudate. Connective tissue layers formed around the
follicle are called the theca interna and theca externa.

5. The mature (Graafian) follicle is the structure of the follicle just prior
to ovulation. Fluid infiltration and antrum formation are maximal and
the follicle has a diameter of 5 mm.
6. At ovulation the follicle ruptures. The oocyte (plus some granulosa
cells) is extruded from the ovary, leaving behind most of the follicle.

a. Just prior to or at ovulation, meiosis I is completed and the


primary oocyte becomes the secondary oocyte + first polar
body.

1109
Female Reproduction 1 and 2 - Dr. Witorsch

7. The corpus hemorrhagicum is a blood-filled remnant of the ruptured


follicle retained by the ovary immediately after ovulation.

8. The corpus luteum is the structure transformed from the corpus


hemorrhagicum. Granulosa cells are increased in size and lipid-laden
(luteinized or yellow in color). Granulosa cells are the major source of
estradiol and progesterone after ovulation.
9. Regressing corpora lutea: Regression of the corpus luteum begins
toward the end of an ovarian cycle and continues into the subsequent
cycle.
10. Corpus albicans is scar tissue resulting from corpus luteum regression.
11. Atretic (or degenerating) follicles result from developing follicles that
never fully mature and undergo regression prior to reaching the
Graafian stage.
12. Ovarian stroma is fibroelastic connective tissue and interstitial cells.
Stromal cells contain lipid derived from atretic follicles. They secrete
sex steroids in response to LH and human chorionic gonadotrophin
(hCG).

II. FEMALE SEX ACCESSORY ORGANS (Figure 2)

Figure 2. Female Reproductive Organs

A. The female sex accessory organs provide a proper environment for the post-
ovulatory egg, its fertilization and the development of the fetus. The major
female sex accessory organs and their roles are:

1. The fallopian tube (or oviduct) which receives the egg after being
extruded from the ovary at ovulation and is the organ where
fertilization occurs.
2. The uterus is composed mucosa (endometrium) and muscle wall
(myometrium). The uterus is the site where the fertilized egg

1110
Female Reproduction 1 and 2 - Dr. Witorsch

(blastocyst) implants, the placenta forms and embryo develops and is


the organ involved in parturition.
3. The cervix is the constricted structure of the lower uterus and serves as
the boundary between uterus and vagina. The cervix plays a passive
role in conception and is the major site of spermatozoan storage, post-
coitus.
4. The vagina is the organ of sexual receptivity. The "sweating" response
of the vagina is the earliest response to sexual stimulation.

III. HUMAN MENSTRUAL CYCLE (Figure 3)

A. The human menstrual cycle is a recurring pattern of physiological changes


in the female reproductive system. The primary goal of the cycle is to
prepare the uterus for implantation of the fertilized ovum. On average the
menstrual cycle is of 28 days duration, although on an individual basis
cycles usually range from 25-35 days.

B. The menstrual cycle exhibits the following phases: menses (menstrual


flow) which occurs from day 1 to 4 or 5 of the cycle; the proliferative
phase (referring to uterine events) or follicular phase (referring to ovarian
events) which occurs from days 5 to 14 of the cycle; ovulation or
extrusion of the egg from the ovary which occurs on day 14 of the cycle;
and the secretory phase (referring to uterine event) or luteal phase
(referring to an ovarian event which occurs during the last half (days 15 to
28) of the cycle.

C. Cyclic changes are observed in pituitary hormone levels.

1. FSH exhibits a slow decline in days 1-14. At midcycle (day 14) a


small surge in serum FSH occurs. From days 15-28, FSH is
maintained at levels lower than at days 1-14. An abrupt small
increase in FSH occurs on days 28-1 (at the end of the old cycle
and the start of the new cycle.
2. LH levels are stable but low during the first half of the cycle (days
1-14). On day 14, a large ovulatory surge in LH occurs which
lasts about 24 h. A slow decline in LH levels occurs from days 15
to 28 followed by an abrupt small increase between days 28 and 1.
3. PRL (prolactin) levels are higher during days 15-28 than days 1-14
(not shown), a pattern which is of questionable consistency.

1111
Female Reproduction 1 and 2 - Dr. Witorsch

Figure 3. Human Menstrual Cycle

D. Cyclic changes in ovarian histology

1. Days 1-14 (follicular phase) is characterized by development of


primordial follicles . About 1,000 primordial follicles are mobilized
however usually only one is destined to become the Graafian follicle.
2. Days 14-15 is the time when ovulation occurs. This is 36-38 hr after
the start of the LH surge.
3. Days 15-21 is the time when a portion of the follicle retained in the
ovary becomes the corpus hemorrhagicum for a 1-2 day period. This
structure then becomes the corpus luteum for the duration of the
period.

1112
Female Reproduction 1 and 2 - Dr. Witorsch

4. Days 21-25 is the time when peak corpus luteum development and
function occurs.

5. Days 26-28 is the time when regression of corpus luteum initiated and
the cycle ends.
6. After 28 days (subsequent cycles) luteal regression continues and the
corpus luteum ultimately becomes a corpus albicans.

E. Cyclic changes in plasma steroid levels

1. Estradiol

a. From days 1-14 of the cycle a gradual increase in plasma


estradiol level occurs. The estradiol is produced by the
developing follicle. The theca interna makes testosterone which
is then aromatized to estradiol by the granulosa cells.

i. Thirty to sixty hrs prior to the LH surge (during the


second week of cycle) plasma estradiol rises at a more
rapid rate.

b. Days 14-15 exhibits an elevation in estradiol corresponding to


the LH surge and post-ovulation, estradiol levels drop.
c. Days 15-25 exhibits a second rise in estradiol. This estradiol is
now being produced by the corpus luteum.
d. Days 26-28 is characterized by a decline in estradiol levels
which is concomitant with the regression of the corpus luteum.

2. Progesterone

a. On a molar basis, the output of progesterone exceeds that of


estradiol. However, there are no dramatic changes observed
during the follicular phase of the menstrual cycle. From Days
1-14 serum progesterone levels are low.
b. A periovulatory increase is seen in plasma 17α-
hydroxyprogesterone (not shown in Fig. 3).
c. On days 15-25 (post-ovulation) gradual increase in
progesterone production from corpus luteum occurs.

i. Peak levels of progesterone occur between days 21-25


concomitant with maximal corpus luteum activity.

d. On days 26-28 progesterone levels decline concomitant with


corpus luteum regression. The post-ovulatory pattern of 17α-
hdroxyprogesterone is similar to that of progesterone.

1113
Female Reproduction 1 and 2 - Dr. Witorsch

F. Cyclic changes in the uterus

1. During menses (Days 1-4) there is a sloughing off of necrotic


endometrium with blood and uterine fluid discharged as menstrual
flow. This occurs because of spasmotic closure of arteries due to local
release of prostaglandin-F2α. Menses is due to withdrawal of
hormonal support (estradiol and progesterone) as a consequence of
regression of corpus luteum on days 26-28 of the preceding cycle.
2. The proliferative phase occurs from days 5-14 of the cycle and is
characterized by proliferation of uterine epithelium, glandular tissue,
stroma and vascularity. Uterine thickness increases several fold.
Uterine proliferation due to estradiol secreted by developing follicle.
3. The secretory phase occurs from days 15-25 of the cycle. During this
period the uterus exhibits increased vascularity (coiled arteries),
stromal hypertrophy, fluid accumulation (edematous tissue), further
glandular development, increased glandular secretory activity
(proteins, amino acids, carbohydrates), and glycogen accumulation.
Secretory stage changes that occur in the uterus are a consequence of
the combined effects of estradiol and progesterone after ovulation.
Secretory changes in the uterus prepare the organ for implantation of
a fertilized egg, if fertilization occurs. If fertilization does not occur,
the corpus luteum regresses and the withdrawal of hormonal support
leads to menses on day 1 of the next cycle.

G. Cyclic changes in the cervix

1. At midcycle: changes in cervical mucus occur due to the actions of


estradiol. At this time, cervical mucus is highest in quantity, most
penetrable to sperm (less hostile and less acidic), most transparent, and
least viscous (has a high degree of elasticity), a property called
"spinnbarkeit." When smeared on a slide and allowed to dry, cervical
mucus exhibits a ferning pattern.

H. Cyclic changes in the vagina

1. At midcycle, the vagina exhibits cytological (exfoliative) changes due


to the effects of estradiol. Among these changes are increased mitotic
activity and exfoliation of squamous epithelial cells.

I. Cyclic changes in basal body temperature (BBT)

1. BBT during luteal phase (post-ovulation) is approximately 1°F greater


than that during the follicular phase. The increase in BBT is attributed
to an action of progesterone on the thermoregulatory center of the
hypothalamus.

1114
Female Reproduction 1 and 2 - Dr. Witorsch

IV. HORMONAL CONTROL OF THE MENSTRUAL CYCLE (Figure 4.)

Figure 4. Feedback Control of Ovulation

A. Throughout most of the menstrual cycle, estrogen and progestin exert a


negative feedback effect producing suppressed and/or declining levels of
FSH and LH during the follicular phase and suppression of FSH and LH
during the luteal phase. This negative feedback is responsible, in part, for
luteal regression which occurs at the end of the cycle.

1. The small but abrupt rise in FSH and LH at the beginning of a new
cycle is due to corpus luteum regression and withdrawal of estradiol
and progesterone at the end of preceding cycle.

B. A rise in plasma estradiol 30-60 hours prior to the LH surge appears to


stimulate LH and FSH surge by a positive feedback mechanism. The positive
feedback effect of estradiol on LH and FSH release appears to occur at the
pituitary level. Anterior pituitary responsiveness to GnRH increases 10-50
fold. Therefore, estradiol increases pituitary sensitivity to pulsatile GnRH
(accordingly, clomiphene, a nonsteroidal estrogen, is an inducer of ovulation).

C. The ovulatory surge in LH and FSH is controlled by a single follicle which


emerges, called the dominant follicle. The dominant follicle is selected from
a cohort of 8-12 cells that are most sensitive to FSH.

1. The dominant follicle (from left or right ovary) achieves dominance


over other follicles in both ovaries.

1115
Female Reproduction 1 and 2 - Dr. Witorsch

2. Dominance of a single follicle is evident by day 7 of the cycle.


3. The dominant follicle is the source of most of the estradiol secreted
during the follicular phase.

a. It is responsible for the pre-ovulatory rise in estradiol which


then produces the positive feedback stimulus for the ovulatory
surge in LH and FSH

D. Intra-ovarian mechanisms responsible for cyclic changes that occur in the


ovary

1. During the follicular phase, the theca interna develops LH receptors


and produces androgens from cholesterol in response to LH.
2. Granulosa cells develop FSH receptors. In response to FSH, androgen
is converted to estradiol by the aromatase system. Estradiol stimulates
the proliferation, induces FSH receptors (and increases FSH
responsiveness) of granulosa cells. Toward the end of the follicular
phase (late antral stage) FSH and estradiol stimulate development of
LH receptors granulosa cells. Antral fluid contains androgen, estradiol
and progesterone as a result of the hormonal interactions described
above. Through most of the cycle, granulosa cells are not vascularized,
just prior to ovulation granulosa cells become vascular, hyperemic,
and edematous.
3. Ovulation involves plasmin-assisted degradation of the wall of the
follicle.
4. Post-ovulation granulosa cells become luteinized, cease to proliferate,
hypertrophy and secrete estradiol and progesterone in response to LH.
5. Luteal failure (luteolysis) at day 26 occurs by a mechanism that has
not been fully characterized. However, LH withdrawal via negative
feedback is implicated. Intraovarian estrogens and prostaglandin F2α
may also participate.
6. The GnRH surge is not necessary for ovulation. The ovulatory surge in
LH and FSH requires pulsatile GnRH and positive feedback effect due
to the pre-ovulatory rise of estradiol. Pulsatile GnRH (every 90 min)
delivered via a portable infusion pump is used in the treatment of
anovulatory conditions. Nevertheless, a GnRH surge occurs at
midcycle. This surge by itself appears too small to be responsible for
the surge in LH and FSH. Pulsatile GnRH may facilitate the ovulatory
surge by providing a self-priming effect.
7. Pulsatile GnRH rate can be modified. Estradiol increases and
progesterone tends to decrease pulse frequency. As ovulation
approaches, pulse frequency increases and decreases during luteal
phase.
8. The periovulatory release of 17α-hydroxyprogesterone may enhance
anterior pituitary sensitivity to GnRH.

1116
Female Reproduction 1 and 2 - Dr. Witorsch

V. ESTROGENS

A. The principal circulating estrogen is estradiol-17ß which is in


equilibrium with its 17-ketosteroid derivative, estrone.

B. Plasma binding: Ninety-nine % of circulating estradiol-17β is bound to


plasma proteins, 48% bound to albumin and 51% to sex hormone binding
globulin (SHBG which same as testosterone binding globulin TeBG).

C. Metabolism, conjugation and excretion: The major metabolite of estradiol-


17β is a 16α -hydroxylated derivative, estriol (Fig. 5). Hydroxylation of
estradiol-17β can also occur at the C-2 position forming catechol estrogen
(Fig. 5). In humans the 16α-hydroxylation and 2-hydroxylation
pathways are inversely related to one another. Tobacco smoking is
associated with increased 2-hydroxylation, whereas alcohol consumption
associated with increased 16α-hydroxylation. Sixteen α-hydroxylated
estrogens are biologically active while 2-hydroxylated estrogen are
relatively inactive. In women as well as in certain strains of mice,
increased hepatic 16α-hydroxylation of estradiol is linked with increased
incidence of breast cancer.
Estrogens (secreted and metabolites) are excreted primarily as
glucuronide and sulfate conjugates.

Figure 5. Two Major Metabolites of Estradiol-17β

D. Among the physiological actions of estrogens are the following: regulation of


gonadotrophin secretion (positive and negative feedback), influences sexual
behavior, cyclic changes in uterus, cervix and vagina, granulosa cell
proliferation and development, maturation and maintenance of sex accessory
organs and secondary sex characteristics, mild anabolic effects, maintenance
of pregnancy, lowered threshold of uterus to contractile stimuli during
parturition, development of the tubular mammary gland, stimulation of
prolactin release, decrease in serum LDL cholesterol levels, an anti-
osteoporotic effect, and induction of progestin receptors.

1117
Female Reproduction 1 and 2 - Dr. Witorsch

1. Estrogen induction of progestin receptors indicates that estrogens are


required for progestin action. This is the basis for estrogen priming
effect for progestin action. Steroid receptor analysis of breast biopsy
specimens reveals important information about the status of a breast
cancer. A tumor that is positive for both estrogen receptor (ER) + and
progesterone receptor (PR) + suggests that the tumor is hormone
dependent. ER+, PR- suggests that tumor is hormone independent (i.e.,
nonfunctional estrogen receptors).

VI. PROGESTINS

A. The principal circulating progestin is progesterone. The corpus luteum


also produces 17α-hydroxy progesterone.

B. Plasma binding: Ninety-eight % of circulating progesterone is bound to


plasma proteins, 48% bound to albumin and 50% to corticosteroid binding
globulin (CBG).

C. Metabolism and excretion (Fig 6): The principal metabolic conversions of


progesterone are A ring and 20-ketone reduction in liver which forms
pregnanediol from progesterone, and pregnanetriol from 17α-hydroxy
progesterone. Pregnanetriol may be converted further to etiocholanolone, a
17 keto derivative. Etiocholanolone is also a product of cortisol and
testosterone metabolism. Progestin derivatives are excreted either in
unconjugated form or as glucuronide or sulfate conjugates.

Figure 6. Major Products of Progestin Metabolism

1118
Female Reproduction 1 and 2 - Dr. Witorsch

D. Among the physiological actions of progestins are regulation of gonadotophin


secretion, cyclic changes in uterus, and maintenance of pregnancy. During
pregnancy progestins increase the threshold of the uterus to contractile stimuli
and promote development of the alveolar mammary gland.

VII. FEMALE ORAL CONTRACEPTIVE


A. Female oral contraception are available in several forms.

1. Synthetic progestins (derivatives of 19-nortestosterone, such as


norethindrone) + estrogens (such as ethinyl estradiol). The
original rationale for this approach was based inhibition of the
ovulatory surge of LH by negative feedback. However, in the
presence of estrogens, lower doses of progestins are contraceptive
even when they do not block ovulation. This is because, cervical
mucus becomes hostile to sperm, as well as adverse effects of
progestin on the endometrium and oviduct. Thus contraception
may be the result of local effects on cervical mucus and other
components of the female reproductive tract.
2. RU-486 is a steroid analogue (11β-(4-dimethyl-amino phenyl)-
17β-hydroxy-17α)prop-1-ynyl)-estra-4,9-dien-3-one) that has a
high affinity for the progesterone receptor but appears unable to
initiate transcriptional events. As a result, RU 486 would prevent
trophoblastic implantation.

a. RU-486 has similar antagonistic actions on the


glucocorticoid receptor system and has utility in the
treatment of disorders associated with glucocorticoid
excess such as Cushing's syndrome.

3. Plan B, an emergency contraceptive containing the progestin,


levonorgestrel given as two tablets 12 hrs apart. The procedure is
about 90% effective in preventing pregnancy if taken within 72 hrs
of intercourse. Among the possible contraceptive mechanisms of
this method are: 1) inhibition or delay of ovulation, 2) altered tubal
transport of sperm or ova, or 3) inhibition of implantation of the
blastocyst by alteration of the endometrium.

VIII. FEMALE GONADAL DISORDERS

A. Hypogonadism and precocious puberty can occur in females by the


same mechanisms described previously for males. Pulsatile GnRH has
utility for treatment of hypothalamic (secondary) hypogonadism. Long-
acting GnRH analogues have utility for treatment of complete precocious
puberty of hypothalamic origin. Adrenogenital disorders produce virilizing
syndromes in females by mechanisms described in adrenal cortex lectures.

1119
Female Reproduction 1 and 2 - Dr. Witorsch

B. Menstrual abnormalities, are the most common disorders in women and


are due to defects in the menstrual cycle, such as failure to ovulate,
shortened follicular phase, defects in timing of luteal regression, too
abrupt fall in estrogen levels at time of ovulation, hypothalamic-pituitary
dysfunction, and ovarian or uterine defects.

C. Several diagnostic tests are useful in evaluating menstrual irregularities,


among these are: basal body temperature (should increase 1°F during
luteal phase); evaluation of cervical mucus (for viscosity, ferning),
evaluation of vaginal smear (for exfoliated squamous cells), serum
hormone levels (e.g., LH, FSH, gonadal steroids) and provocative tests
(LH, FSH; GnRH) to evaluate target organ responsiveness.

IX. STUDY QUESTIONS

1. The dominant follicle is producing most of its estradiol during which days
of the menstrual cycle:

A. Days 1-4
B. Days 5-14
C. Days 15-20
D. Days 21-25
E. Days 26-28

2. The fertilized egg goes through the morula (or pre-blastocyst) stage during
which days of the menstrual cycle?

. Days 1-4
A. Days 5-14
B. Days 15-20
C. Days 21-25
D. Days 26-28

3. The corpus luteum undergoes regression during which days of the


menstrual cycle?

. Days 1-4
A. Days 5-14
B. Days 15-20
C. Days 21-25
D. Days 26-28

1120
Female Reproduction 1 and 2 - Dr. Witorsch

4. The uterus exhibits maximum secretory activity during which days of the
menstrual cycle?

. Days 1-4
A. Days 5-14
B. Days 15-20
C. Days 21-25
D. Days 26-28

5. The corpus hemorrhagicum is evident during which days of the menstrual


cycle?

. Days 1-4
A. Days 5-14
B. Days 15-20
C. Days 21-25
D. Days 26-28

6. Granulosa cells are proliferating and the antrum is being formed (i.e.
follicular development), during which days of the menstrual cycle?

. Days 1-4
A. Days 5-14
B. Days 15-20
C. Days 21-25
D. Days 26-28

Answer Key: 1. B; 2. C; 3. E; 4. D; 5. C; 6. B

1121
Growth Factors - Dr. Kalimi

Growth Factors
Mohammed Y. Kalimi, Ph.D.

Growth, differentiation and tissue maintenance require a combination of proper hormone


(and receptor) levels, nutrient availability and many regulatory and environmental
factors. In addition to the classical endocrine hormones discussed in this text, many
polypeptide hormones, like growth factors, have been discovered.

Abnormal production, expression, and/or function of growth factors can result in


unregulated growth, the hallmark of malignant transformation and many other
pathologies.

Most of the growth factors which have been characterized appear to be small proteins or
polypeptides which work through a hormone-like membrane receptor mechanism.

General Consideration:

A. Classical hormones (e.g. GH, T4) are potent growth stimulators in vivo but have
much less growth-promoting activity in vitro.
B. Peptide growth factors isolated from serum and various non-endocrine tissues are
active in stimulating cell proliferation in vitro at very low concentrations (10-9--
10- 10 M).
C. In some cases, they mediate the actions of classical hormones (e.g. IGF-I
mediates GH action).
D. Growth factors may have endocrine, paracrine, or autocrine actions.
E. Growth factors act by binding to specific receptors on the cell surface:

1. Results in phosphorylation of specific tyrosines on cytoplasmic domain of


receptors.
2. Initiates a sequence of metabolic events resulting in DNA synthesis and cell
division.

F. Single growth factors have less of a mitogenic effect than combinations of growth
factors.
G. Growth factors may act at specific times during the cell cycle.

I. Somatomedins/insulin like growth factors (IGF-I and IGF-II):

Two somatomedins have been identified, IGF- I and IGF- II. IGF-I is also known
as somatomedin C and IGF- II is also known as somatomedin A.

IGF- I appears to be far more growth hormone-dependent than IGF- II.

1122
Growth Factors - Dr. Kalimi

IGF’s play a critical role in both cell cycle control and apoptosis, two functions
involved in regulation of tumorigenesis.

Insulin, IGF-I and IGF-II, each have their own distinct receptors.

The insulin receptors share a number of common properties with the IGF-I
receptor.

Both insulin and IGF-I receptors are made up of two distinct polypeptides (MW
135,000 and 90,000 daltons) having intrinsic tyrosine specific kinase activities.

IGF-I and insulin bind each others receptor with 10-100 times lower affinity than
the homologous ligand. However, IGF-I and insulin do not bind to the IGF-II
receptor.

The IGF-II receptor is quite different from the insulin and IGF-I receptors. This
receptor is made up of a single polypeptide with MW of 250,000 daltons,
without any intrinsic kinase activity. IGF-II receptor is identical to the
mannose-6-phosphate receptor.

A. IGF-I (MW 7500 daltons):

1. Purified from serum - not stored in any tissue


2. IGF-I circulates in plasma associated with a large carrier protein(MW
150,000).
3. Due to binding protein, the plasma half- life of IGF-I is very long
compared to most peptide hormones.
4. Six distinct IGF-I binding proteins have been described to date.
These proteins function as:

a. Carriers of IGFs in blood and extracellular fluid.


b. Facilitators or inhibitors of IGF receptor binding and biological action.

5. Stimulate growth and/or differentiation and ↓ apoptosis in target cells.


6. Produced in liver and many other tissues under the influence of growth
hormone.
7. IGF-I has GH-independent actions in embryonic growth and reproductive
system (uterine growth, ovarian follicular development) function.
8. GH has both direct and indirect (via IGF-I) effects on growth.
9. Treating patients with recombinant IGF-I produces nearly normal growth.
10. Injection of IGF-I directly into the tibial epiphyseal plate can stimulate
cartilage proliferation.
11. Certain pygmies in Africa can produce normal amounts of growth
hormone but have decreased concentrations of serum IGF-I (growth
hormone resistance syndromes).

1123
Growth Factors - Dr. Kalimi

12. Children who are deficient in GH have low IGF-I levels and grow poorly.
13. In acromegaly pituitary gigantism, and in IGF-I resistance syndrome, IGF-
I levels are very high.

B. IGF-II:

1. IGF-II is a polypeptide hormone with structural homologies to insulin and


IGF-I.
2. IGF-II seems to be involved in regulation of fetal growth.

II. Epidermal growth factor (EGF):

A. Initially purified from mouse salivary glands and identified by its effect on the
acceleration in eye opening of newborn mice (S.Cohen: Nobel Prize 1986).
B. The epidermal growth factor (EGF) family of polypeptides are regulators for
tissue development, proliferation, differentiation, survival (inhibitor of
apoptosis) and repair of various cells.
C. Presence in human amniotic fluid and breast milk suggests a role in human
development.
D. Infusion of EGF into the mammalian fetus stimulates growth and
differentiation of epithelial tissues and accelerates lung maturation.
E. EGF = urogastrone, a potent inhibitor of gastric acid secretion isolated from
human urine.

III. Platelet - derived growth factor (PDGF):

A. Purified from platelet; present in serum (after clotting and platelet lysis) but
not plasma.
B. Multifaceted inflammatory mediator secreted by fibroblasts, smooth muscle
cells, macrophages and endothelial cells.

IV. Fibroblast growth factor (FGF):

A. Purified from pituitary and brain tissue but appears to be present in multiple
cell types.
B. Potent mitogen for mesodermally derived cells: fibroblast, adrenal cells,
gonadal cells, smooth muscle cells, endothelial cells, chondrocytes and
macrophages.
C. Major biological role: stimulation of angiogenesis (capillary proliferation).

V. Nerve growth factor (NGF) and Brain derived neurotrophic factor (BDNF):

A. Purified from mouse salivary glands and snake venom.


B. Promote growth, differentiation and survival of sympathetic and sensory
neurons.

1124
Growth Factors - Dr. Kalimi

C. Treatment of Alzheimer’s disease, Parkinson’s disease and amyotrophic


lateral sclerosis.

VI. Erythropoietin:

A. Regulates red blood cell production.


B. Made in kidney but purified from blood and urine.

VII. Lymphocyte growth factors (interleukins, thymosin, thymopoietin):

A. Activate lymphocyte proliferation and maturation.


B. Appear to play a key role in regulating the immune response and
inflammation.

VIII. Growth Factors and Neoplasia:

The normal growth of the cells is controlled by the opposing effects of growth
stimulatory (positive) and growth inhibitory (negative) factors.

When the balance between the positive and negative regulation is upset, however,
the result may be the uncontrolled growth of cancer cells.

The development of a variety of cancers may be influenced by a deficiency in


growth inhibition; as well as by an excess of growth stimulation.

IX. Transforming growth factors (TGF alpha and TGF beta):

A. TGF family of cytokines have fairly wide physiological actions, mainly in


inflammation and immune responses.
B. Responsible for a diverse range of signaling events within cells leading to
necrosis or apoptosis.

1. TGF α: Stimulatory
2. TGF β: Inhibitory

X. Practice Questions

1. Which of the following peptide hormones acts through activation of


adenylyl cyclase

A. PTH
B. IGF-I
C. Somatostatin
D. Prolactin
E. GH

1125
Growth Factors - Dr. Kalimi

2. GH release is induced by:

A. Dopamine
B. GHRH
C. TRH
D. Hyperglycemia
E. Aging

3. The major effect of insulin on adipose tissue in human is to:

A. Decrease lipolysis
B. Decrease glucose uptake and glycolysis
C. Increase release of free fatty acids
D. Increase ketogenesis
E. Decrease triglyceride synthesis

4. A 7-year-old child with short stature is evaluated. Which of the following


could provide an explanation for the short stature?

A. Normal plasma GH levels


B. Normal plasma GHRH levels
C. Hyperfunctioning thyroid gland
D. Normal functioning anterior pituitary
E. Low plasma levels of IGF-I

5. Both insulin and glucagon secretion are stimulated by the infusion of:

A. Sulfonylurea
B. Amino acids
C. Catecholamines
D. Leptin
E. Ketoacids

6. During fasting, which of the following changes occurs related to thyroid


hormone?

A. Decreased TBG
B. Increased total T3
C. Increased total T4
D. Increased thyroglobulin (TG)
E. Increased reverse T3

1126
Growth Factors - Dr. Kalimi

7. TSH:

A. Is a single chain peptide


B. Has a very long (8 hours) half life
C. Works through the inhibition of cAMP
D. Is a glycoprotein related to FSH and LH
E. Effectively inhibits the thyroid pump

8. A 40-year-old woman was diagnosed with type I diabetes at the age of 10.
Which would be most characteristic of her form of the disease?

A. Insulin resistance
B. Insulin deficiency
C. Lack of pancreatic alpha cells
D. Obesity
E. Sulfonylurea treatment

9. Epidermal growth factor:

A. Receptor has three main domains: extracellular, transmembrane


and intracellular
B. Stimulates growth of many cell types
C. Closely resembles urogastrone
D. is useful in burn healing
E. All of the above

10. You examine a patient for low pituitary function (plasma levels of TSH,
LH, FSH are low). When you administer a large dose of LHRH (GnRH)
and TRH, there is no change in circulating LH, FSH.and TSH. You
conclude that the:

A. Defect lies in the hypothalamus


B. Defect lies in gonads and thyroid
C. Low FSH levels has caused all the observed abnormalities
D. Pituitary is normal
E. Defect lies in the LHRH (GnRH) and TRH receptors

XI. Matching questions: Select one letter heading for each numbered word or
phrase.

A. Involved in the biosynthesis of T4 (thyroxine)


B. Stimulates cell growth
C. IGF-I
D. Is released from the hypothalamus
E. Inhibits insulin release

1127
Growth Factors - Dr. Kalimi

11. Dopamine
12. TGF alpha
13. Peroxidase enzyme

Answer Key: 1. A, 2. B, 3. A, 4. E, 5. B, 6. E, 7. D, 8. B, 9. E, 10. E, 11. D.1 2. B.


13. A.

1128
Female Reproduction 3 - Dr. Witorsch

Female Reproduction 3: Pregnancy and Lactation


R.J. Witorsch, Ph.D.

OBJECTIVES:

At the end of this lecture, the student should be able to:

1. Describe and explain the major events involved in fertilization of the egg and
implantation of the blastocyst.
2. Describe and explain the functional anatomy of the placenta as an organ of
exchange between the mother and the fetus.
3. Describe and explain the endocrinology of pregnancy and the central role played
by the placenta in this phenomenon.
4. Describe and explain secondary or “other” hormonal changes that occur during
pregnancy.
5. Describe and explain the principal events involved in the initiation and
maintenance of parturition.
6. Describe and explain the principal events involved in milk secretion (lactation)
and milk ejection.

Suggested Reading: Costanzo 3rd Edition, pp. 456-462

I. FERTILIZATION AND IMPLANTATION

A. Ejaculation during sexual intercourse deposits 150-300 x 106 spermatozoa


in the vagina. Spermatozoa obtain fertilizing capacity after several hours
contact with the female reproductive tract (uterus and fallopian tube) by a
process called capacitation. Capacitation involves several physiologic
changes in spermatozoa, namely: increased metabolism, increased
motility, alteration of surface proteins, and an acrosomal reaction enabling
sperm to penetrate the egg. Capacitation can occur in vitro but the process
takes longer than that in vivo.

B. Sperm live in the female reproductive tract for 1-3 days and can be stored
viably in the crypts of the cervix.

C. Fertilization occurs in the upper 1/3 of the fallopian tube (oviduct), at


isthmus of ampullary region. Of the original 150-300 x 106 spermatozoa
deposited in the vagina, only about 100 enter the oviduct. Loss of
spermatozoa occurs in the cervix and to a lesser extent at the utero-tubal
junction.

D. A single spermatozoan encounters the egg. Fertilization is possible if


coitus occurs 1 day prior to and 1 day post ovulation. The sperm cell

1129
Female Reproduction 3 - Dr. Witorsch

penetrates the egg with the aid of acrosomal enzymes and incorporates
itself into the egg's cytoplasm. The egg is then activated to block entry of
additional spermatozoa.

E. Following fertilization (on day 14 or 15 of the cycle), the egg begins to


divide. Meiosis II starts at time of ovulation and ends after fertilization.
Within 3-4 days (day 17-19 of cycle) the fertilized egg reaches the morula
stage, a ball of 16-32 cells. Over next 4-5 days (days 21-24 of the cycle)
the morula becomes a blastocyst, an embryonic structure containing a
fluid-filled cavity (Fig. 1). The inner layer of cells will become the embryo
and the outer layer or trophoblast cells will become the fetal portion of
the placenta and embryonic membranes. Until the time of implantation,
progesterone from the corpus luteum delays movement of fertilized egg
until proper time of implantation.

Figure 1. Blastocyst just prior to implantation (about 7 days post-


ovulation)

F. Implantation, a 4-day process, occurs at the time of maximal uterine


secretory activity (days 21-25 of cycle). The trophoblast invades the dorsal
wall of uterus and erodes the endometrium (about day 7 postovulation) and
invades the uterine stroma. Invasive enlarged trophoblastic (giant) cells lose
cell boundaries forming a syncytium (syncytiotrophoblast). Lacunar spaces
form among synciotrophoblast which develop into chorionic villi. Chorionic
villi break down maternal blood vessels, so that they are in direct contact with
maternal blood. Hemochorial placentae (i.e., where chorionic villi are bathed
in maternal blood) is a feature of some (human, rodent), but not all species.

1130
Female Reproduction 3 - Dr. Witorsch

1. The endometrium, in response to trophoblast invasion, undergoes a


proliferative response and activates a uterine stromal reaction. This
uterine response is called the decidual reaction and involves much of
the uterine endometrium and stroma (Fig. 2A). The decidua eventually
envelopes the entire conceptus (Fig. 2).

Figure 2. (A) Decidual reponse several days after implantation of blastocyst. (B) Relationship
of the feto-placental unit and the uterus at the end of the third month of pregnancy.

II. FETAL-MATERNAL RELATIONSHIP

A. The fetus is suspended in a fluid filled sac called the amniotic cavity and is
connected to the placenta via an umbilicus (Fig. 2B). The placenta serves
as an organ of exchange between fetus and mother and forms in a
localized region where trophoblast and decidua interface. The umbilicus
contains 2 arteries and 1 vein (Fig. 3).

1. Umbilical blood vessels terminate as sinusoidal capillary beds


within villus projections (chorionic villi) in the fetal portion of the
placenta (syncytiotrophoblast). The maternal (uterine) arteries
penetrate the fetal shell of the placenta so that maternal blood
bathes these placental villi. Fetal-maternal exchange occurs at this
interface. The umbilical arteries carry blood rich in CO2, low in
nutrients, and containing excretory waste from the fetus to
placenta. The umbilical vein carries cleansed, oxygenated blood
that is high in nutrients from the placenta to the fetus.

1131
Female Reproduction 3 - Dr. Witorsch

Figure 3. Maternal-fetal relationships of the human placenta

III. ENDOCRINOLOGY OF PREGNANCY

A. The placenta is also an important endocrine organ.

B. Survival and continued development of the fetus and placenta for 9


months in utero requires an appropriate endocrine milieu of sufficient
levels of estrogen and progestin. Accordingly, progressive elevation of
estrogen and progestin output continues throughout pregnancy until
parturition. This is the most important endocrinologic adjustment during
pregnancy. Estrogen and progestin achieve very high concentrations
relative to that of the cycling female. The sustained steroidogenesis results
from an interplay between the corpus luteum, fetus and placenta.

1. Throughout pregnancy, steroid hormones provide the following


specific functions: progesterone is required for the decidual
reaction; progesterone maintains endometrium for the survival of
the fetus, progesterone suppresses spontaneous excitability of the
uterus, elevated estrogen and progestin suppress serum LH and
FSH, thereby preventing a new wave of folliculogenesis. Estrogen
and progestin stimulate prelactational breast development.

C. Early in pregnancy, estrogens and progestins are produced by the corpus


luteum. Thus, fertilization and implantation of the blastocyst rescues the
corpus luteum from regression at the end of the cycle. Rescue of the
corpus luteum is attributed to the production of a placental protein
hormone called human chorionic gonadotrophin (hCG).

D. Human Chorionic Gonadotrophin (hCG) (Fig. 4)

1132
Female Reproduction 3 - Dr. Witorsch

1. hCG is a glycoprotein hormone (MW 45,000 daltons) that has LH-


like biological activity and belongs to the LH, FSH, TSH family. It
is synthesized in the syncytiotrophoblast and enters maternal
circulation. hCG maintains the corpus luteum and stimulates luteal
steroidogenesis (estrogen and progesterone production).
2. hCG is first detectable in maternal urine 9 days post-fertilization
and predominates during the first trimester of pregnancy. Peak
levels of hCG occur at 9 weeks of gestation and are maintained at
low levels thereafter. hCG in urine, determined immunologically,
is the basis of the diagnostic pregnancy test.

Figure 4. (A) Protein hormones of the placenta. (B) Steroid


hormones of the placenta.

E. Estrogen and Progesterone (Fig. 4 and 5)

1. The production of estrogen and progesterone by an hCG-stimulated


corpus luteum prevents the onset of menstrual flow. During the last
two trimesters of pregnancy (months 3-9), when hCG levels have
decreased, the corpus luteum functions at a lower level, and the
placenta assumes steroidogenic capacity.
2. The placenta becomes major source of progesterone. Progesterone
serum levels in late pregnancy are approximately 10-20 times that
found during the mid luteal phase.

1133
Female Reproduction 3 - Dr. Witorsch

Figure 5. Estrogen production by the feto-placental unit

3. Estrogen production during last two trimesters of pregnancy involves


an interplay between fetus and placenta (the feto-placental unit) (Fig.
5). Peak estrogen levels toward the end of pregnancy are 160-fold that
seen during the follicular phase. The fetal adrenals produce large
amounts of DHEA sulfate which are then 16α-hydroxylated by the fetal
liver. DHEA sulfate and 16α-hydroxy DHEA sulfate produced in the
fetus are then processed by the placenta.

a. The placenta cannot produce androstane (19-carbon) steroids


from pregnane (21-carbon) steroids, but has sulfatases (to
remove sulfates from steroids) and can aromatize androgens.
Thus, the feto-placental unit can produce estrone, estradiol and
its 16α-hydroxylated derivative, estriol from DHEA sulfate.
(The maternal adrenal is another source of DHEA sulfate for

1134
Female Reproduction 3 - Dr. Witorsch

the formation of placental estrogens).

b. Estriol is the predominant estrogen throughout pregnancy (10


fold that of estradiol). Fetal death and impending abortion are
associated with a decline in maternal estrogen levels.
c. The enormous capacity of the placenta to aromatize androgen
also protects the female fetus from the virilizing effect of
adrenal androgens.

G. Human Placental Lactogen (hPL)

1. hPL is also referred to as hCS (human chorionic


somatomammotropin).
2. hPL is another protein hormone produced by the trophoblastic portion
of the placenta. It is about 85% homologous to human growth
hormone and has a molecular weight of about 24,000 daltons. This
hormone is first detected in maternal fluid about 3 weeks post-
fertilization and increases progressively until term. At parturition
maternal serum levels of hPL reach 6:g/ml, although little hPL is
detected in the fetus (Fig. 4).
3. hPL exhibits growth hormone and prolactin-like actions and may be
involved in producing "diabetes-like" effects in the mother during
pregnancy (e.g., hyperinsulinemia, insulin resistance, increased
lipolysis, lipemia, increased gluconeogenesis).
4. The precise role of hPL is not known. It is speculated that its actions
can mobilize nutrients (e.g., glucose) in the mother which are then
carried transplacentally to the fetus. hPL is a marker for placental size
and growth during pregnancy.

H. Relaxin

1. Relaxin is a 6 Kd polypeptide homologous to insulin that is produced


in the corpus luteum and endometrium. Relaxin is detectible in plasma
during the late luteal phase (about 9 days post-ovulation) and post-
fertilization throughout gestation (although highest levels occur during
the first trimester).
2. Among the specific actions of relaxin are increased uterine weight,
endometrial angiogenesis and connective tissue development. The
putative role of relaxin is to promote developmental changes in the
uterus in the late secretory phase and early pregnancy.

I. Other endocrine changes in the mother during pregnancy

1. LH and FSH secretions are low because of feedback suppression by


high levels of estrogen and progestin.

1135
Female Reproduction 3 - Dr. Witorsch

2. Prolactin levels increase progressively during pregnancy because of


the positive feedback effects of estrogen on the pituitary. In fact,
maternal lactogen levels are very high during pregnancy (prolactin and
hPL). Enormous concentrations of prolactin are observed in amniotic
fluid during pregnancy. The source appears to be decidual and/or
trophoblastic and the significance of this prolactin is not known.
3. Blood cortisol levels increase due to an estrogen-stimulated increase in
CBG.
4. Aldosterone secretion increases are due to estrogen-induced increases
in hepatic angiotensinogen production. This results in Na+ and water
retention.
5. Total thyroxine and triiodothyronine plasma levels increase during
pregnancy due to an estrogen-stimulated increase in thyroxine-binding
globulin (TBG).

IV. PARTURITION (Labor)

A. Parturition occurs approximately 280 days after fertilization and involves


uterine contractions leading to expulsion of the feto-placental unit.

1. Recent evidence suggests that parturition is initiated by activation


of the fetal hypothalamo-pituitary-adrenocortical axis near term
resulting in the release of cortisol from the fetal adrenal cortex.
2. Cortisol produces an increase in the ratio of serum estrogen to
progestin. This would decrease the threshold of the uterus
(myometrium) to excitatory stimuli, thus producing a more
excitable uterus.
3. Oxytocin. Oxytocin sustains labor by stimulating uterine
contraction. Distention of the cervix by the fetus stimulates the
release of oxytocin from the posterior pituitary by a
neuroendocrine mechanism (the Ferguson reflex) (Fig. 6)
perpetuating labor. Nerve impulses generated by the distended
cervix travel up the spinal cord activating the magnocellular nuclei
in the hypothalamus (SON and PVN) to release oxytocin from the
posterior pituitary.

a. Oxytocin causes uterine contraction moving the fetus along


the birth canal and leading to further stretching of the
cervix, leading to more oxytocin release and ultimately
expulsion of the fetus.
b. The oxytocin response is mediated by a serpentine G
protein-coupled receptor and Ca+2 signaling.
c. During pregnancy estrogen increases uterine oxytocin
receptors 100 fold in mymetrium and decidua, reaching a
peak during early labor.

1136
Female Reproduction 3 - Dr. Witorsch

d. Oxytocin may also be a local mediator of uterine


contraction.

i. Local oxytocin expression occurs within the uterus


during pregnancy;
ii. At the end of pregnancy, oxytocin gene expression
(mRNA and protein) increases 150-fold in
endometrium.

e. Another participant labor is prostaglandin. Prostaglandin


levels increase in amniotic fluid during labor. Certain
prostaglandins cause contractions of uterine smooth
muscle.

i. Oxytocin also increases prostaglandin formation in


decidua (thus enhancing the former’s action).

Figure 6. Ferguson Reflex - Parturition


Adapted from Rhoades and Pflanzer, 3rd Edition, Saunders.

V. MILK SECRETION (LACTATION) AND EJECTION (Fig. 7)

A. During normal development of the breast estrogens promote ductile or


tubular development, and progestins promote alveolar formation. During
pregnancy, estrogens and progestins stimulate further proliferation of
mammary parenchyma (ducts and alveoli). Prolactin (PRL) also plays a
significant role in mammary growth during pregnancy.

B. Colostrum is a mammary secretion produced in late pregnancy and during


the first week post-partum. Colostrum is not milk but contains some milk

1137
Female Reproduction 3 - Dr. Witorsch

constituents such as salts, lactose, and immunoglobulins. Relative to milk,


colostrum has more protein (2-fold), less lactose (25%), less fat (25%).
Colostrum secretion is promoted by high levels of prolactin, hPL, insulin,
thyroid hormone, growth hormone and corticosteroids occurring during
pregnancy.

C. Milk is characterized by fat, lactose, minerals (calcium, iron, magnesium,


etc.), vitamins, and certain specific proteins such as lactalbumin and
casein. Milk production (or secretion) is stimulated by prolactin. Milk is
not produced until several days post-partum. Milk production does not
occur during pregnancy even though lactogenic hormone (prolactin and
hPL) levels are high. This is because high circulating levels of estrogen
(and possibly progesterone) block the action of lactogenic hormones at the
breast. The post-partum decline in progesterone and estrogen permit
prolactin stimulation of milk production and, hence, initiation of lactation.

D. The postpartum decline in hPL and in prolactin (secondary to a decline in


estrogen) tends to oppose the maintenance of lactation (galactopoiesis).
However, suckling by the infant maintains lactation by stimulating
prolactin release via a neuroendocrine reflex.

E. Suckling, via a neuroendocrine reflex, also promotes oxytocin release


from the posterior pituitary (again via involvement of hypothalalmic
magnocellular nuclei). Oxytocin induces contraction of myoepithelial cells
surrounding alveoli (again, involving a Ca+2 signaling mechanism)
ejecting milk from the mammary gland. Milk ejection is also referred to as
"milk let down". Since oxytocin effects smooth muscles, nursing mothers
usually experience uterine contractions.

1. Since suckling also stimulates prolactin release, the alveoli


emptied by milk ejection can be refilled with newly formed milk.

1138
Female Reproduction 3 - Dr. Witorsch

Figure 7. Milk Secretion and Ejection


Adapted from Hedge, Colby and Goodman, Clinical Endocrine Physiology, Saunders.

VI. STUDY QUESTIONS - All Type A

1. The decidua refers to:

A. the portion of the blastocyst that invades the uterus


B. the embryonic portion of the blastocyst
C. the maternal (uterine) response to blastocyst implantation
D. the portion of the blastocyst destined to become the fetal portion of
the placenta
E. the source of placental villi

2. The fetal adrenal cortex plays a major role in the production of which
hormone of pregnancy?

A. human chorionic gonadotropin


B. human placental lactogen
C. progesterone
D. estriol
E. prolactin

3. The major factor associated with the initiation of lactation is:

A. hypersecretion of lactogenic hormone


B. insulin action
C. cortisol action
D. hypersecretion of estrogen and progestin
E. decline of estrogen and progestin

1139
Female Reproduction 3 - Dr. Witorsch

4. Identify the hormone responsible for maintaining the corpus luteum of


pregnancy.

A. LH
B. hCG
C. progesterone
D. estradiol
E. hPL

5. All of the following is true for steroid production during pregnancy except

A. the major source of progestin and estrogen during early pregnancy


is the corpus luteum
B. during late pregnancy most progestin is produced de novo in the
placenta
C. during late pregnancy the fetal adrenal is essential for placental
estrogen production
D. during late pregnancy the fetal adrenal is essential for placental
progestin production
E. the placenta does not have the capacity to produce androgenic
(C19) steroids from progestational (C21) steroids

6. Which of the following hormones may mobilize nutrients in the mother


that cross the placenta to the fetus?

A. LH
B. hCG
C. progesterone
D. estradiol
E. hPL

7. The reason lactation does not occur during pregnancy is because

A. the mammary gland is exposed to inadequate levels of lactogenic


hormone
B. estrogen levels during pregnancy are inadequate
C. estrogen and progesterone produced during pregnancy block the
lactogenic action of prolactin at the mammary gland
D. cortisol levels are inadequate during pregnancy
E. lactation does occur during pregnancy

Answer Key: 1. C: 2. D; 3. E; 4. B; 5. D; 6. E; 7. C.

1140

You might also like